You are on page 1of 420

www.newspaperkorner.wordpress.

com

Target

BANK PO
20 Practice Sets Preliminary
& M a i n Wo r k b o o k

www.newspaperkorner.wordpress.com
www.newspaperkorner.wordpress.com

• Head Office : B-32, Shivalik Main Road, Malviya Nagar, New Delhi-110017

• Sales Office : B-48, Shivalik Main Road, Malviya Nagar, New Delhi-110017
Tel. : 011-26691021 / 26691713

How to access the Online Test(s)?


Online Test(s): Coupon Code

Typeset by Disha DTP Team INSTRUCTIONS

1. Scratch the silver coating GENTLY with a coin to reveal the


code.
2. You can access your test on any Window based Desktop,
android tablets or ipads absolutely free.
3. Go to www.mylearninggraph.com & Click on Registration
on the Top Right bar.
4. Enter your Details & Click on “Create an account”.
5. Click on your Name on the Top Right bar to open the Menu &
Click on Apply Coupon (Skip Steps 2 & 3 if you are already
registered).
6. Enter your Coupon Code & click Apply & your Course will
be added to “My Program”.
7. Contact us at support@mylearninggraph.com for any
assistance.
Note: MLG is not accessible on Mobiles.

DISHA PUBLICATION
ALL RIGHTS RESERVED

© Copyright Publisher

No part of this publication may be reproduced in any form without prior permission of the publisher. The author and the
publisher do not take any legal responsibility for any errors or misrepresentations that might have crept in. We have
tried and made our best efforts to provide accurate up-to-date information in this book.

For further information about the books from DISHA,


Log on to www.dishapublication.com or email to info@dishapublication.com

www.newspaperkorner.wordpress.com
CONTENT
www.newspaperkorner.wordpress.com

1. Information about SBI PO EXAM (iv)


2. How to prepare for Descriptive Test? (v)
A. Essay Writing (v)
B. Letter Writing (vi)

Solved Papers
SBI Bank PO Main 2016 [31/ 07/ 2016] 2016-1-19

SBI Bank PO Prelim 2016 [10/ 07/ 2016] 2016-20-30

SBI Bank PO Main 2015 [26/ 07/ 2015] M-1-24

SBI Bank PO Prelim 2015 [20/ 06/ 2015] P-1-12


SBI Bank PO Exam [21/06/2014] 2014-1-28
SBI Bank PO Exam [28/04/2013] 2013-1-28
SBI & Associates Bank PO Exam [07/08/2011] P1-1-26
SBI Bank PO Exam [24/07/2011] P2-1-26

Main Exams Practice Sets


1. Practice Set-1 with solutions 1-16
2. Practice Set-2 with solutions 17-34
3. Practice Set-3 with solutions 35-52
4. Practice Set-4 with solutions 53-68
5. Practice Set-5 with solutions 69-88
6. Practice Set-6 with solutions 89-106
7. Practice Set-7 with solutions 107-124
8. Practice Set-8 with solutions 125-144
9. Practice Set-9 with solutions 145-164
10. Practice Set-10 with solutions 165-180

Preliminary Exam Practice Sets


1. Practice Set-1 with solutions Ps-1–Ps-12

2. Practice Set-2 with solutions Ps-13–Ps-24

3. Practice Set-3 with solutions Ps-25–Ps-36

4. Practice Set-4 with solutions Ps-37–Ps-46

5. Practice Set-5 with solutions Ps-47–Ps-58

SBI Bank PO 5 Online Main Practice Sets


www.newspaperkorner.wordpress.com
www.newspaperkorner.wordpress.com

Information about SBI written examination


for probationary officers
Selection Procedure:
Phase-I: Preliminary Examination: Preliminary Examination consisting of Objective Tests for 100 marks will be conducted
online. This test would be of 1 hour duration consisting of 3 Sections as follows:

Sr. No. Name of test No. of Questions Marks Duration

1. English Language 30 30

2. Quantitative Aptitude 35 35
Composite Time of
1 hour
3. Reasoning Ability 35 35

Total 100 100

Phase – II: Main Examination: Main Examination will consist of Objective Tests for 200 marks and Descriptive Test for
50 marks.
Both the Objective and Descriptive Tests will be online.
Candidates will have to answer Descriptive test by typing on the computer.
Immediately after completion of Objective Test, Descriptive Test will be administered.

Objective Test:
Sr. No. Name of Tests (Objective) No. of Maximum Duration
Questions Marks
1 Reasoning & Computer Aptitude 45 60

2 English Language 35 40
Composite Time of
3 Data Analysis & Interpretation 35 60 3 hrs.

4 General/Economy/Banking Awareness 40 40
Descriptive Test (Essay, Letter, Paragraph 50 30 Minutes
Writing etc.)

Total 155 250

www.newspaperkorner.wordpress.com
www.newspaperkorner.wordpress.com

HOW TO PREPARE FOR DESCRIPTIVE TEST ?

A. ESSAY WRITING

HOW TO WRITE AN ESSAY :


The word essay literally means an attempt. It is an attempt or a trial to make a composition on a given subject. The
dilemma encountered is the method that should be used to write a good essay.
COLLECTING MATTER :
• Once a topic has been given for writing an essay the student will have to collect points pertaining to that particular
subject.
• Material can be collected from the Net or from the books. Studying the subject matter from all view points is
important.
• A lot of good ideas can be collected by just conversing with friends who are also preparing for the said examinations.
• Above all keep your senses open. Observe, absorb and express. That should be the motto.
STRUCTURE OF THE ESSAY :
• All essays must have an introduction. The purpose of the introduction is to explain to the reader the crux of the
subject.
• The essay must have a body which must state points partaining to the topic in a detailed manner.
• The points should be arranged in a logical manner so that the reader can understand the subject matter clearly.
• Finally there should be a conclusion which sums up your thoughts and provides a finishing touch to the essay.
METHOD OF WRITING THE ESSAY :
• One the points are ready we should first write them down.
• Next we should arrange the points in such a way that the essay wears a structured appearance.
• Points will have to be alloted for an effective Introduction.
• Once all the points are arranged all we need to do is to explain each one in detail.
• The body of the essay should have main points followed by subpoints.
• Each main points should have one paragraph.
• A compelling conclusion will have some points which summarises your thoughts effectively.
THE HALL MARKS OF A GOOD ESSAY :
• The essay should be one composite whole. We should not stray from the subject matter.
• The essay should have structure and logic. Irrelevant matter should find no place in it.
• The language should be formal, yet simple, short sentences provide greater clarity than long winding explanations.
In this way you can also avoid making too many grammatical mistakes.
• If we understand the topic very well we will automatically be able to express our thoughts clearly.
• We express our own ideas and thoughts in an essay of course they must be backed up with adequate proof so that
our reader is convened.
• In other words an essay is a piece of original composition written in a compact from. The style should be lucid and
clear.

SAMPLE ESSAY
Corruption- A way of life
When we talk of corruption, we must understand what corruption really means. The simple and straightforward meaning
of corruption is to get work done by means of illegal gratification.
At one time, not very long back, just a few decades back, to dabble in corrupt practices was something to be ashamed
of, and a known corrupt individual was singled out. People would point the charging finger at him and talk scandals in
whispers about him. This goes to show that, though corruption was there, it was a very rare phenomenon, a hush hush
affair. Both the corrupt person and the general public being awe struck about the whole affair. The corrupt person hid
his misdeed which in turn goes to show that there was fear in the minds of the corrupt, the fear of being exposed and
being punished.

www.newspaperkorner.wordpress.com
www.newspaperkorner.wordpress.com

As time passed by, this scenario on the front of corruption undertook an absolute somersault the very attitude of the
corrupt, and the public changed altogether. The malady spread through the length and breadth of the country, and the
world. As the virus of corruption was allowed to take its toll, it spread and continued to spread through the passage of
decades.
The situation as it stands today is that, corruption is no slur on any individual, it has by now become a way of life, it has
been so closely woven into our life patterns that, no one thinks much about it. This is because the system has been
let loose and in so many decades everyone, yes, everyone is so involved in corruption that the topic is not worth any
discussions. Just as life style includes a bungalow, car, servants, club etc., it also includes corruption.
The entire system has been affected by this virus and it is now like a cancer absolutely incurable. The cancer has spread
throughout the body system and now there is no cure for it. Only in the early stages, if the cancer had been treated it
may have never taken such a dangerous posture.
Let us for a moment analyze how this virus was allowed to grow. The answer to this question is quite simple. Since this
cancer was not treated in its earlier stages, it grew to this magnum size, which today defies any cure. It is not that people
did not know about the virus entering the system but, it was ignored and so the result is here for all of us to see. If it had
been nipped in the bud just when it had raised its ugly head, it would not have dared to grow.
Now, since this did not happen, the malady spread with the passage of time and took the present shape. If it had been
tackled at its very inception, more and more people would not have joined the bandwagon and the situation would not
have been as it is now. No corrupt person was ever punished for his/her corrupt practices; it gave an impetus to others to
become so. Moreover, since there was no punishment, the element of fear also flew to the winds, and people dared it all.
More and more people crossed the fence as there were greener pastures across. This situation led to the present
day scenario when it is like trying to touch the moon to look for an honest man. All the world’s glamour and happiness
attracted more and more people and corruption became an obvious way of life.
Now when the virus has struck the system we hear from pulpits and platforms long drawn out speeches on uprooting
corruption. This is however now a Herculean task. However, even at this stage, we can make an effort. This is possible
only if we give stringent punishment to the corrupt, no matter whom. Side by side, if we can find a handful of honest
people they should also be simultaneously rewarded and honored. With this twin treatment, maybe we see a reduction
of the menace after some time. Let us remember it is more difficult to improve a decayed system rather than make a
new clean system.

B. LETTER WRITING
HOW TO WRITE A LETTER :
Letter writing is an important aspect of day to day life almost all our concerns, feelings and opinions can be
expressed through letters. Hence letter writing is something which every man of letters should know.
Letters are of different kinds. They can be broadly categorised into formal and informal. Formal letters discuss
formal matters, they express opinions and feelings on certain topic of importance. Informal letters are basically letters
dealing with personal matters. Then again formal letters are written to strangers. Informal letters are written to friends
or relatives. In competitive exams the strange part is that the student is called upon to write letters to even friends and
relative on matters related to formal letters. Therefore we cannot afford such a clear demarcation. However all letters
in general have a certain structure.

www.newspaperkorner.wordpress.com
www.newspaperkorner.wordpress.com

STRUCTURE OF THE LETTER

74/6, Bhownipore
Kolkata - 700020 ¬ The Heading
19, June, 2015

Dear Sir ¬ The greeting

I am extremely aggrieved by the poor condition of some the roads in the Trans -Yamuna area
of Delhi - - - - - - - - - - - - - - - - - - - - - - - - - - - - - - - - - - - - - - - - - - - - - -- - - - - ¬ The Body
---------------------------------------

Yours Sincerely, ¬ The leave taking

Rini Mathews .......... ¬ My name.

• The Heading : This is the part of the letter which contains the address of the letter writer. It also shows the date on
which it was written.
• The Greeting : In case of letters written to friends and relatives we may write dear brother, dear friend etc. In case
of letters to strangers we use the salutation dear Sir for all concerned.
• The Body of the Letter : It is letter to divide the letter into paragraphs which elucidating the different points. Each
paragraph for a point. A good handwriting, knowledge of punctuation and a logical presentation of thoughts are
essential perquites.
• The Leave Taking : Once we have had our say we take leave of the concerned person with courteous words such
as Yours Sincerely, Yours affectionately etc. The former for strangers and the latter for friends.
• The Name of the Writer : Below the leave-taking is written the name of the letter writer that is yourself.

SAMPLE LETTER
144/13 sector-1,
Pushp Vihar,
New Delhi -110017. Date : 23.6.15
Dear Editor,
I write to you, as your newspaper has been active in highlighting corrupt practices before and during the CWG games.
I would urge you to start a signature campaign online and otherwise, where ordinary citizens can register their disgust
at:
1. Complete lack of leadership during the build up to the games;
2. Lack of government intervention even when the whole country was screaming that corruption is endemic in the
awarding of contracts;
3. The blatant involvement of the senior most level of officials;
4. The time provided to all the corrupt to siphon off the funds and hide the money - realistically it is too late to find
the money. The money would have already made its way to the multitude of Dubai and other tax haven locations,
far away from any possible legal justice.
5. The complete lack of common sense - starting the CP makeover with less than 12 months to go for the games!
I know the issues are many and most cannot be listed here given the level of blatant gluttony, horrible deceit and
completely buffoonery at all levels.
However, like in case of Jessica Lal and many others, the common citizenry has to ensure, along with the press that
the guilty are brought to justice and the enquiry is not just an eyewash for public consumption.
Best regards,
Rini Mathews

www.newspaperkorner.wordpress.com
www.newspaperkorner.wordpress.com

www.newspaperkorner.wordpress.com
www.newspaperkorner.wordpress.com

SBI Bank PO Main Exam 2016


Held On : 31-07-2016
Max. Marks : 200 (Based on Memory) Time : 3 hrs.

5. Which of the following will be the final step of the given


REASONING & COMPUTER APTITUDE input ?
DIRECTIONS (1-5) : Study the following information carefully (a) all and 16 11 best equal 25 47 my strength 49 81
and answer the given questions. (b) my and 49 81 equal best 47 25 all strength 16 11
(c) all and 81 49 best equal 25 47 my strength 11 16
When a word and number arrangement machine is given an input (d) all best 81 49 and equal 47 25 my strength 16 11
line of words and numbers, it arranges them following a particular (e) None of these
rule. The following is an illustration of Input and rearrangement.
INPUT : expect 30 more funny 41 52 sea 61 ring 36 89 joy DIRECTIONS (Qs.6-10) : Study the following information
STEP 1 : expect funny 30 more 41 52 sea 61 ring 36 89 joy carefully to answer the given questions.
STEP 2 : expect funny 89 61 30 more 41 52 sea ring 36 joy Seven persons - A, B, C, D, E, F and G live on seven different
STEP 3 : expect funny 89 61 joy more 30 41 52 sea ring 36 floors of a building but not necessarily in same order. Lowermost
STEP 4 : expect funny 89 61 joy more 52 41 30 sea ring 36 floor of building is numbered 1, one above that is numbered 2 and
STEP 5 : expect funny 89 61 joy more 52 41 sea ring 30 36 so on till top most floor is numbered 7. Each one of them also likes
STEP 6 : expect funny 89 61 joy more 52 41 ring sea 36 30 a different food recipe, namely Egg roll, Pizza, Burger, Noodles,
And step VI is the last step of the above input. As per the rules Briyani, Fried Rice and Masala Dosa.(but not necessarily in same
followed in the above step, find out the appropriate step for the order).
given output.
· A lives an odd numbered floor but not on floor numbered
Input: best 11 all 25 my 47 81 equal 49 strength 16 and
Three. Only two persons live between D and the one who
Explanation
likes Briyani. The One who likes Burger lives immediately
STEP 1 : all and best 11 25 my 47 81 equal 49 strength 16
above C.
STEP 2 : all and 81 49 best 11 25 my 47 equal strength 16
STEP 3 : all and 81 49 best equal 11 25 my 47 strength 16 · The one who likes Pizza lives on odd numbered floors above
STEP 4 : all and 81 49 best equal 47 25 11 my strength 16 D. B lives on the floors above E. Only three persons live
STEP 5 : all and 81 49 best equal 47 25 my strength 11 16 between C and the one who likes Pizza.
STEP 6 : all and 81 49 best equal 47 25 my strength 16 11 · The one who likes Egg roll lives immediately above the one
Rule : In step 1 two words picked arranged in alphabetical order who likes Masala Dosa. G lives on an odd numbered floor. E
and placed it at the left corner and in step 2 , two numbers picked does not like Burger.
and arranged it in descending order next to words and so on. · Only one person lives between B and E. The one who likes
1. Which of the following term exactly comes between 49 and Briyani lives immediately above A. Neither C nor A likes
equal in step 3 ? Noodles.
(a) my (b) 11 6. Which of the following food recipe does A like?
(c) strength (d) best (a) Fried Rice (b) Masala Dosa
(e) None of these (c) Burger (d) Pizza
2. How many elements are there between 25 and 16 in step 5 ? (e) Egg roll
(a) One (b) Three 7. Which of the following combinations is True with respect to
(c) Two (d) Five the given arrangement?
(e) None of these (a) Fried Rice - C (b) Pizza - F
3. In step 6, equal and my, 11 and 49 are interchanged then (c) Masala Dosa - G (d) Briyani - D
which will be placed between 49 and equal ? (e) Noodles - B
(a) Two (b) Three 8. If all the persons are made to sit in alphabetical order from
(c) Four (d) Six top to bottom, positions of how many persons will remain
(e) None of these
unchanged?
4. What is the position of 25 in step 4 from the right end?
(a) Four (b) None
(a) Third (b) Fourth
(c) Two (d) One
(c) Sixth (d) Fifth
(e) Three
(e) None of these

www.newspaperkorner.wordpress.com
www.newspaperkorner.wordpress.com
2016-2 SBI BANK PO MAIN EXAM 2016
9. Which of the following statements is True with respect to · Immediate neighbours of Q faces opposite directions(i.e if
the given arrangement? one neighbour faces the centre then the other neighbour
(a) The one who likes Masala Dosa immediately below A. faces outside and Vice-versa.)
(b) F likes Pizza · P does not face outside. O faces a direction opposite to that
(c) None of the given options is true. of M.
(d) Only four persons live between D and G 16. How many people sit between L and Q when counted from
(e) G lives immediately below E. the left of Q?
10. Who among the following lives on the floor numbered 2? (a) Five (b) None
(a) B (c) Four (d) One
(b) The one who likes Egg roll (e) Two
(c) The one who likes Masala Dosa 17. Which of the following statements is true as per the given
(d) D arrangement?
(e) F (a) Q faces the centre
11. In which of these expressions 'S > V' be definitely false? (b) Only three people sit between P and L
(a) S>P³Q=G³R>V (b) P<A£S£T;V³O>T
(c) R sits to the immediate right of N
(c) V£A£L=R<S (d) S>C>=F£H; V<F
(d) None of the given options is true
(e) S>T=O³P; V<J=P
(e) N is an immediate neighbour of O
12. Which of the following symbols should be placed in the
blank spaces respectively(in the same order from left to 18. Who amongst the following sits third to the left of P?
right) in order to complete the given expression in such a (a) Q (b) N
manner that both 'D>S' as well as 'E£B' definitely holds true? (c) M (d) L
B_A_ S _ E _ D (e) R
(a) >, ³, <, = (b) >, >, ³, < 19. How many people face the centre as per the given
(c) ³, ³, ³,£ (d) ³, =, ³,< arrangement?
(e) Other than those given as options (a) Four (b) One
13. In Which of the following expressions does the expression (c) Two (d) Three
'L=T' to definitely hold true? (e) More than four
(a) K ³ L ? R = P < S £ T 20. What will come in place of question mark (?) in the given
(b) U ³ T ³ M = F £ A ³ L series based on the positions as given in the arrangement?
(c) L ³ C > Q ³ B = N £ T PS LR MP SM ?
(d) G ³ L = A < B £ S £ T (a) ON (b) SO
(e) T ³ E = G ³ W = Y ³ L (c) NQ (d) OL
14. Which of the following expressions is true if the expression (e) LS
P<T<=Q>S>M>=W is definitely true?
(a) W £ P (b) S < P DIRECTIONS (Q. 21-25): Study the sets of numbers given
(c) M > P (d) W < Q below and answer the questions which follow:
(e) T £ M 129 642 921 476 308
15. Statements: Y £ K < D = S; D < V < O; G ³ D < Q 21. Which of the following numbers will be obtained if the
Conclusions: I. G > V, II. Y < Q second digit of the lowest number is added to the third digit
(a) Only I is true of the highest number after adding four to each number?
(b) Only II is true
(a) 5 (b) 6
(c) Either I or II true
(c) 7 (d) 8
(d) Neither I nor II is true
(e) None of the Above
(e) Both I and II are true
22. If the digits in each of the five numbers are arranged in
DIRECTIONS (Qs.16-20): Study the following information descending order, the position of how many numbers will
carefully to answer the given questions. not change?
Eight people L, M, N, O, P, Q, R and S are sitting around a circular (a) None (b) One
table with equal distance (c) Two (d) Three
between each other but not necessarily in the same order. Some (e) Four
of them are facing the centre while some are facing outside.(i.e 23. Based upon the given set of numbers, four of the following
away from the centre) five are alike in a certain way and so form a group. Which is
· M sits third to the left of L. Only three people sit between M the one does not belong to that group?
and S. P sits to the immediate right of S. (a) 647 (b) 264
· Immediate neighbours of P face opposite directions(i.e. if (c) 912 (d) 192
one neighbour faces the centre then the other neighbour (e) 380
faces outside and vice-versa.) Only one person sit between 24. If in each number, the digits are arranged in ascending order,
P and O. what will be sum of middle digits of each of the numbers?
· R sits second to the right of O. Both R and N face the same (a) 14 (b) 12
direction as S.(i.e if S faces the centre then R and N also faces (c) 26 (d) 9
the centre and vice-versa.) (e) None of the Above

www.newspaperkorner.wordpress.com
www.newspaperkorner.wordpress.com
SBI BANK PO MAIN EXAM 2016 2016-3
25. If in each number, the first and the last two digits are II. K stands exactly at the centre of the line. Only two
interchanged, which will be the second lowest number? people stand between K and L. Only three people
(a) 129 (b) 642 stands between L and O. Only one persons stands
(c) 921 (d) 476 between O and P
(e) 308 (a) Only I (b) Only II
(c) Both I and II (d) Either I or II
DIRECTIONS (26-30): Study the following information
(e) Neither I or II
Carefully to answer the given questions
33. What does 'Zee' represent in a code language ?
P, Q, R, S, T, U, V, W and X are sitting in a straight line, facing I. In that code language 'ah koj zee pig' mean 'can you take
North. Three of them are not males. Two females sit adjacent to that'
each other. Q is fourth to the left of V, who is second to the right II. In that code language 'et zee lin ter' means ' you may
of R, who is not the immediate neighbour of P. come now'
· U is fourth to the right of R and is second to the left of X. S (a) Only I (b) Only II
is not an immediate neighbour of either X or Q. (c) Both I and II (d) Either I or II
· S is not male. One of the persons sitting on the extreme ends (e) Neither I or II
is a female. T is not an immediate neighbour of either V or U. 34. Among K, L, M, N, O and P each has different age, who is the
· No female is an immediate neighbour of U. W does not sit youngest among them ?
second to the left of P. The immediate neighbour of S are I. L is younger than only K, and P. N is neither the oldest
male nor the youngest
26. Which of the following is a group of females ? II. M is older than N but not the oldest
(a) QTS (b) TXP (a) Only I (b) Only II
(c) SVR (d) UWX (c) Both I and II (d) Either I or II
(e) None of these (e) Neither I or II
27. Who is sitting to the immediate left of S ? 35. Which among P, Q, R, S and T is the smallest ?
(a) V (b) Q I. P is greater than or equal to R which is equal to Q and
(c) W (d) R greater than or equal to S and T.
(e) None of these II. S is equal to R and is not greater than Q. P is greater than
28. In which of the following combinations is the third person Q.
sitting between the first and the second person ? (a) Only I (b) Only II
(a) PWU (b) QTR (c) Both I and II (d) Either I or II
(c) RST (d) WUP (e) None of these
(e) None of these
29. If Q and R, V and U interchange their position then how DIRECTIONS (Qs.36-40): In each question below is given a
many persons are sitting between R and V ? statement followed by two conclusions numbered I and II. You
(a) Four (b) Five have to assume everything in the statement to be true, then
(c) Six (d) Two consider the two conclusions together and decide which of
(e) None of these them logically follows beyond a reasonable doubt from the
30. Who among the following sits third to the left of P ? information given in the statement.
(a) W (b) V Give answer:
(c) R (d) X (a) If only conclusion I follows
(e) None of these (b) If only conclusion II follows
(c) If either I or II follows
DIRECTIONS (31-35) : Each of the questions below. Consests
(d) If neither I nor II follows and
of a question and two statement numbered I and II given below
(e) If both I and II follow.
it. Read both statement and answer the questions.
36. Statements: In a one day cricket match, the total runs made
31. How far is point A from point D ? by a team were 200. Out of these 160 runs were made by
I. Point A is 4m to the north of point B .Point C is 3m to the spinners.
east of point A. Point D is to the west of point A such that Conclusions:
points C, A and S form a straight line of 7m I. 80% of the team consists of spinners.
II. Point F is3m to south of point D. Point C is 4m to the east of II. The opening batsmen were spinners.
point A. Point A is 3m to the north of point B. Point A lies on (a) Only conclusion I follows
the line formed by joining points C and D. (b) Only conclusion II follows
(a) Only I (b) Only II (c) Either I or II follows
(c) Both I and II (d) Either I or II (d) Neither I nor II follows
(e) Neither I or II (e) Both I and II follow
32. How many people are standing in a straight line. All are facing 37. Statements: The old order changed yielding place to new.
North directions. Conclusions:
I. M stands third from the left end of the line. Only one I. Change is the law of nature.
person stands between M and O. P stands second to the II. Discard old ideas because they are old.
right of O. P stands at one of the extreme lines
www.newspaperkorner.wordpress.com
www.newspaperkorner.wordpress.com
2016-4 SBI BANK PO MAIN EXAM 2016
(a) Only conclusion I follows (d) If neither I nor II is implicit
(b) Only conclusion II follows (e) If both I and II are implicit.
(c) Either I or II follows 41. Statement: "You are hereby appointed as a programmer with
(d) Neither I nor II follows a probation period of one year and your performance will be
(e) Both I and II follow reviewed at the end of the period for confirmation." - A line in
38. Statements: Government has spoiled many top ranking an appointment letter.
financial institutions by appointing bureaucrats as Directors Assumptions:
of these institutions. I. The performance of an individual generally is not
Conclusions: known at the time of appointment offer.
I. Government should appoint Directors of the financial II. Generally an individual tries to prove his worth in the
institutes taking into consideration the expertise of the probation period.
person in the area of finance. (a) Only assumption I is implicit
II. The Director of the financial institute should have (b) Only assumption II is implicit
expertise commensurate with the financial work carried (c) Either I or II is implicit
out by the institute. (d) Neither I nor II is implicit
(a) Only conclusion I follows (e) Both I and II are implicit
(b) Only conclusion II follows 42. Statement: It is desirable to put the child in school at the age
(c) Either I or II follows of 5 or so.
(d) Neither I nor II follows Assumptions:
(e) Both I and II follow I. At that age the child reaches appropriate level of
39. Statements: Population increase coupled with depleting development and is ready to learn.
resources is going to be the scenario of many developing II. The schools do not admit children after six years of age.
countries in days to come. (a) Only assumption I is implicit
Conclusions: (b) Only assumption II is implicit
I. The population of developing countries will not (c) Either I or II is implicit
continue to increase in future. (d) Neither I nor II is implicit
II. It will be very difficult for the governments of (e) Both I and II are implicit
developing countries to provide its people decent 43. Statement: "In order to bring punctuality in our office, we
quality of life. must provide conveyance allowance to our employees." - In
(a) Only conclusion I follows charge of a company tells Personnel Manager.
(b) Only conclusion II follows Assumptions:
(c) Either I or II follows I. Conveyance allowance will not help in bringing punctuality.
(d) Neither I nor II follows II. Discipline and reward should always go hand in hand.
(e) Both I and II follow (a) Only assumption I is implicit
40. Statements: Prime age school-going children in urban India (b) Only assumption II is implicit
have now become avid as well as more regular viewers of (c) Either I or II is implicit
television, even in households without a TV. As a result (d) Neither I nor II is implicit
there has been an alarming decline in the extent of readership (e) Both I and II are implicit
of newspapers. 44. Statement: Unemployment allowance should be given to all
Conclusions: unemployed Indian youth above 18 years of age.
I. Method of increasing the readership of newspapers Assumptions:
should be devised. I. There are unemployed youth in India who needs
II. A team of experts should be sent to other countries to monetary support.
study the impact of TV. on the readership of II. The government has sufficient funds to provide
newspapers. allowance to all unemployed youth.
(a) Only conclusion I follows (a) Only assumption I is implicit
(b) Only conclusion II follows (b) Only assumption II is implicit
(c) Either I or II follows (c) Either I or II is implicit
(d) Neither I nor II follows (d) Neither I nor II is implicit
(e) Both I and II follow (e) Both I and II are implicit
45. Statement: "If you trouble me, I will slap you." - A mother
DIRECTIONS (Qs.41-45): In each question below is given a
warns her child.
statement followed by two assumptions numbered I and II. You
Assumptions:
have to consider the statement and the following assumptions
I. With the warning, the child may stop troubling her.
and decide which of the assumptions is implicit in the
II. All children are basically naughty.
statement.
(a) Only assumption I is implicit
Give answer (b) Only assumption II is implicit
(a) If only assumption I is implicit (c) Either I or II is implicit
(b) If only assumption II is implicit (d) Neither I nor II is implicit
(c) If either I or II is implicit (e) Both I and II are implicit
www.newspaperkorner.wordpress.com
www.newspaperkorner.wordpress.com
SBI BANK PO MAIN EXAM 2016 2016-5

DATA ANALYSIS & INTERPRETATION DIRECTIONS (57-61) : Each of the questions below consists of
a question and two statements numbered I and II given below it.
46. A certain sum of money amounts to rupees 2900 at 4% per You have to decide whether the data provided in the statements
annum in 4 years. In how many years will it amount to are sufficient to answer the question. Read all the three
rupees 5000 at the same rate? statements and give answer:-
(a) 20 (b) 22 (a) If the data in statement I alone is sufficient to answer the
(c) 24 (d) 25 question.
(e) None of these (b) If the data in statement II alone is sufficient to answer the
47. A trader gives an additional concession of 35% on an article question.
which is already get discounted by 20% on the marked (c) If the data either in statement I alone or statement II alone
price. If the buyer pays an amount of 1300 for the article, are sufficient to answer the question.
then the marked price is (d) If the data given in both I and II together are not sufficient
(a) 2200 (b) 2500 to answer the question.
(c) 2600 (d) 2700 (e) If the data in both the statements I and II together are
(e) None of these necessary to answer the question
48. Two cities A and B are at a distance of 60 km from each other. 57. How much time did P take to reach the destination?
Two persons P and Q start from First city at a speed of 10km/ I. Q takes 24 minutes to reach the same destination
hr and 5km/hr respectively. P reached the second city B and II. Ratio of the speed of P and Q is 3:4
returns back and meets Q at Y. Find the distance between A 58. In how many days A and B together complete a work?
and Y. I. If A works alone for 10 days and B alone for 15 days, the
(a) 30 km (b) 40 km work gets completed.
(c) 50 km (d) 55 km II. A can complete the work in 14 days.
59. What is the difference between the ages of Y and X?
(e) 53 Km
I. The ratio of the age X to the age of Y is 3 : 2
49. 18 years ago the ratio of A's age to B's age was 8:13.Present II. One sixth of X's age is equal to one fourth of Y's age.
age of their ages ratio is 5:7.What is the present age of B ? 60. What is the perimeter of the rectangular field?
(a) 90 years (b) 70 years I. Area of the field is 72m2.
(c) 60 years (d) 50 years II. Length and breadth are in the ratio of 2:1.
(e) None of these 61. What is the speed of boat in still water?
50. X alone can do a piece of work in 5 days. Y can do the same I. It takes 4 hours to cover the distance between P and Q
piece of work in 4 days. X and Y are assigned to do the work downstream
for ` 5000.They complete the work in 2 days with the help of II. It takes 6 hours to cover the distance between P and Q
Z. How much is to be paid to Z ? upstream.
(a) ` 750 (b) ` 620 DIRECTIONS (62-66) : Study the following pie-charts and
(c) ` 700 (d) ` 500 information to answer the given questions
(e) None of these
Total no of employee in Company 1 - 20400
DIRECTIONS (Qs. 51-56) : You have to solve equation I and II % of 6 levels of Employee in Company 1
,Give answer
(a) If x > y
T U
(b) If x < y P
(c) If x ³ y
Q
(d) If x £ y S
(e) If x = y or cannot be established
51. I. 4/Öx + 3/Öx = Öx

(y 35 / 2
– 7 5/ 2 ) =0 Total no of employee in Company 2 - 24800
% of 6 levels of Employee in Company 2
II.
y
52. I. 25x2 + 25x + 6 = 0 U P
II. 5y2 + 20y + 20 = 0
53. I. 2x2 + 11x + 12 = 0
T
II. 2y2 + 19y + 45 = 0 Q
54. I. 4x2 – 19x + 12 = 0
II. 3y2 + 8y + 4 = 0
55. I. 4x2 – 13x – 12 = 0 S
II. y2 – 7y – 60 = 0 R
56. I. X = (1024)1/2
II. Y2 = 1024

www.newspaperkorner.wordpress.com
www.newspaperkorner.wordpress.com
2016-6 SBI BANK PO MAIN EXAM 2016
62. The number of P level employee in company 2 is DIRECTIONS (72-77) : Study the following information to
approximately what percentage of the number of S level answer the given questions
employee in company 1 ?
(a) 55% (b) 45%
(c) 47% (d) 57%
(e) 59%
63. The number of R level employee in Company 1 is
approximately equal to the number of which level employee
in Company 2
(a) U (b) S
(c) T (d) R
(e) Q
64. What is the total number of Q level employee in Company 1
is less than the Company 2 by approximately what
percentage ? 2010 2011 2012 2013 2014 2015
(a) 41.21% (b) 41.12%
(c) 42.21% (d) 42.12%
(e) 44.5%
65. The difference between the number of employees in the 2
72. Expenditure of A in 2014 and 2015 are `12lak and `14lak.
companies of which level is maximum ?
What was the total income of A in 2014 and 2015 together ?
(a) P (b) Q
(a) ` 40.8 L (b) ` 40.4 L
(c) R (d) S
(c) ` 44 L (d) ` 46.4 L
(e) T
(e) None of these
66. What is the total number of P level employee in Company 1
and R level employee in Company 2 together ? 73. Ratio of Expenditure of Companies A and B in 1993 was 4:5.
(a) 14564 (b) 13789 What was the ratio of their incomes in the same year ?
(c) 12679 (d) 12832 (a) 58:75 (b) 75:58
(e) 12932 (c) 78:55 (d) 72:55
(e) None of these
DIRECTIONS (67-71) : Study the information carefully to 74. Total Expenditure of company B in all the years together
answer the given questions was ` 125 L. What was the total income of the company in all
In a college out of 600 students, boys and girls are in the ratio 8: the years together ?
7. Each students went to either physics lab or chemical lab or (a) ` 185 L (b) ` 520 L
both, 40% of the boys attend only physics lab, total 20% children (c) ` 250 L (d) Cannot be determined
went to both the labs out of which 25% are boys, 45% of the girls (e) None of these
went to only chemistry lab. 75. If the incomes of the B in 2012 and 2013 were in the ratio 3:4,
67. Total how many boys went to only physics lab ? Find the ratio of Expenditures of that company in these 2
(a) 175 (b) 158 years ?
(c) 156 (d) 134 (a) 48 : 55 (b) 42 : 53
(e) None of these (c) 58 : 45 (d) 45 : 58
68. How many girls went to only physics lab ? (e) None of these
(a) 62 (b) 46 76. If the expenditure of A and B in 2011 were equal and total
(c) 64 (d) 60 income of A and B was ` 116L, what was the total
(e) None of these expenditure of A and B in the same years ?
69. Total number of students went to physics lab is what (a) ` 84 L (b) ` 83 L
percentage of the total number of students in the college (c) ` 80 L (d) ` 40 L
(a) 22% (b) 32% (e) None of these
(c) 33% (d) 36% 77. From a group of 4 men ,3 women, 2 persons are selected at
(e) None of these random, Find the probability at least one man is selected ?
70. What is the total number of girls went to chemistry lab ? 6 7
(a) 216 (b) 210 (a) (b)
7 6
(c) 267 (d) 261
(e) None of these 5 11
(c) (d)
71. What is the total number of students went to only chemistry 6 21
lab ?
12
(a) 244 (b) 248 (e)
21
(c) 284 (d) 288
(e) None of these

www.newspaperkorner.wordpress.com
www.newspaperkorner.wordpress.com
SBI BANK PO MAIN EXAM 2016 2016-7
Directions (78-80) : An urn contain 6 orange, 4 green,3 pink and returns. The investment was in armies and conquest. The returns
4 black balls. If 3 balls are picked at random, came through plunder and taxes from the conquered. No
78. What is the probability that all are pink ? immorality was attached to imperial loot and plunder. The biggest
conquerors were typically revered (hence titles like Alexander
2 1
(a) (b) the Great, Akbar the Great, and Peter the Great). The bigger and
650 680 richer the empire, the more the plunderer was admired. This
4 3 mindset gradually changed with the rise of new ideas about
(c) (d) equality and governing for the public good, ideas that
630 680
culminated in the French and the American Revolutions. Robert
4 Clive was impeached for making a little money on the side, and so
(e)
680 was Warren Hastings. The white man's burden came up as a new
79. If 5 balls are picked at random, what is the probability that moral rationale for conquest. It was supposedly for the The
none are orange ? Princeton Review CAT sample paper 12 good of the conquered.
This led to much muddled hypocrisy. On the one hand, the
33 442 empire needed to be profitable. On the other hand, the white
(a) (b)
442 33 man's burden made brazen loot impossible.
An additional factor deterring loot was the 1857 Sepoy
15 23
(c) (d) Mutiny. Though crushed, it reminded the British vividly that
167 235 they were a tiny ethnic group who could not rule a gigantic
35 subcontinent without the support of important locals. After 1857,
(e) the British stopped annexing one princely state after another,
235 and instead treated the princes as allies. Land revenue was fixed
80. If 3 balls are drawn at random, what is the probability that 1 in absolute terms, partly to prevent local unrest and partly to
is green and the other 2 are black ? promote the notion of the white man's burden. The empire
proclaimed itself to be a protector of the Indian peasant against
2 4
(a) (b) exploitation by Indian elites. This was denounced as hypocrisy
85 87 by nationalists like Dadabhai Naoroji in the 19th century, who
1 3 complained that land taxes led to an enormous drain from India to
(c) (d) Britain. Objective calculations by historians like Angus
85 85 Maddison suggest a drain of perhaps 1.6 percent of Indian Gross
5 National Product in the 19th century.
(e) But land revenue was more or less fixed by the Raj in
85 absolute terms, and so its real value diminished rapidly with
inflation in the 20th century. By World War II, India had ceased to
ENGLISH LANGUAGE be a profit centre for the British Empire. Historically, conquered
nations paid taxes to finance fresh wars of the conqueror. India
itself was asked to pay a large sum at the end of World War I to
DIRECTIONS (81-85) : Read the passage given below and
help repair Britain's finances.
answer the questions that follow based on the information But, as shown by historian Indivar Kamtekar, the
given in the passage. independence movement led by Gandhiji changed the political
Right through history, imperial powers have clung to their landscape, and made mass-taxation of India increasingly
possessions to death. Why, then, did Britain in 1947 give up the difficult. By World War II, this had become politically impossible.
jewel in its crown, India? For many reasons. The independence Far from taxing India to pay for World War II, Britain actually
struggle exposed the hollowness of the white man's burden. began paying India for its contribution of men and goods.
Troops from white dominions like Australia, Canada and New
Provincial self-rule since 1935 paved the way for full self-rule.
Zealand were paid for entirely by these countries, but Indian
Churchill resisted independence, but the Labour Government of costs were shared by the British government. Britain paid in the
Atlee was anti-imperialist by ideology. Finally, the Royal Indian form of non-convertible sterling balances, which mounted
Navy Mutiny in 1946 raised fears of a second Sepoy Mutiny, and swiftly. The conqueror was paying the conquered, undercutting
convinced British waverers that it was safer to withdraw the profitability on which all empire is founded. Churchill
gracefully. But politico-military explanations are not enough. The opposed this, and wanted to tax India rather than owe it money.
basis of empire was always money. The end of empire had much But he was overruled by Indian hands, who said India
to do with the fact that British imperialism had ceased to be would resist payment, and paralyze the war effort. Leo Amery,
profitable. World War II left Britain victorious but deeply Secretary of State for India, said that when you are driving in a
indebted, needing Marshall Aid and loans from the World Bank. taxi to the station to catch a life-or-death train, you do not loudly
This constituted a strong financial case for ending the no longer- announce that you have doubts whether to pay the fare. Thus,
profitable empire. World War II converted India from a debtor to a creditor with
over one billion pound in sterling balances. Britain, meanwhile,
Empire building is expensive. The US is spending one
became the biggest debtor in the world. It's not worth ruling over
billion dollar a day in operations in Iraq that fall well short of people who are afraid to tax.
fullscale imperialism. Through the centuries, empire building was (The topic of the Passage asked in the exam was based on
costly, yet constantly undertaken because it promised high African banks)

www.newspaperkorner.wordpress.com
www.newspaperkorner.wordpress.com
2016-8 SBI BANK PO MAIN EXAM 2016
81. Which of the following was NOT a reason for the emergence DIRECTIONS (86-93) : Read the following passage carefully
of the 'white man's burden' as a new rationale for empire and answer the given questions.
building in India?
(a) The emergence of the idea of the public good as an Today emerging markets account for more than half of world GDP
element of governance. on the basis of purchasing power according to the International
(b) The decreasing returns from imperial loot and Monetary Fund (IMF). In the1990s it was about a third and in the
increasing costs of conquest. late 1990s 30% of countries in the developing world managed to
(c) The weakening of the immorality attached to an increase their output per person faster than America did, thus
emperor's looting behaviour. achieving what is called 'catch-up growth'. That catching up was
(d) A growing awareness of the idea of equality among somewhat lackadaisical. The gap closed at just 1.5% a year. Some
peoples. of this was due to slower grower in America, most was not. The
(e) None of these most impressive growth was in four of the biggest emerging
82. Which of the following best expresses the main purpose of economies Brazil, Russia, India and China (BRICS). These
the author? economies have grown in different ways and for different
(a) To present the various reasons that can lead to the reasons. The remarkable growth of emerging markets in general
collapse of an empire and the granting of independence and the BRICS in particular transformed the global economy in
to the subjects of an empire. many ways. Some wrenching commodity prices soared and the
(b) To point out the critical role played by the 'white man's cost of manufacturers and labour sank. A growing and vastly
burden' in making a colonizing power give up its claims more accessible pool of labour in emerging economies played a
to native possessions. part in both wage stagnation and rising income inequality in rich
(c) To highlight the contradictory impulse underpinning ones. Global poverty rates tumbled. Gaping economic
empire building which is a costly business but very imbalances fuelled an era of financial vulnerability and laid the
attractive at the same time. ground work for global crisis. The shift towards the emerging
(d) To illustrate how erosion of the financial basis of an economies will continue. But its most tumultuous phase seems
empire supports the granting of independence to an to have more or less reached its end. Growth rates have dropped,
empire's constituents. the nature of their growth is in the process of changing too and
(e) None of these its new mode will have lesser direct effects on the rest of the
83. What was the main lesson the British learned from the world. The likelihood of growth in other emerging economies
Sepoy Mutiny of 1857? having an effect in the near future comparable to that of the
(a) That the local princes were allies, not foes. BRICS in the recent past is low. The emerging giants will grow
(b) That the land revenue from India would decline larger and their ranks will swell but their tread will no longer
dramatically. shake the Earth as it once did.
(c) That the British were a small ethnic group. After the 1990s there followed 'convergence with a
(d) That India would be increasingly difficult to rule. The vengeance'. China's pivot towards liberalization and global
Princeton Review CAT sample paper 13 markets came at a propitious time in terms of politics, business
(e) None of these and technology. Rich economies were feeling relatively relaxed
84. Which of the sfollowing best captures the meaning of the about globalization and current account deficits. America's
'white man's burden', as it is used by the author? booming and confident was little troubled by the growth of
(a) The British claim to a civilizing mission directed at Chinese industry or by off-shoring jobs to India. And the
ensuring the good of the natives. technology etc., necessary to assemble and maintain complex
(b) The inspiration for the French and the American supply chains were coming into their own, allowing firms to
Revolutions. spread their operations between countries and across oceans.
(c) The resource drain that had to be borne by the home The tumbling costs of shipping and communication sparked
country's white population. 'globalization's second unbounding' (the fiat was the simple
(d) An imperative that made open looting of resources ability to provide consumers in one place with goods from
impossible. another). As longer supply chains infiltrated and connected
(e) None of these places with large and fast growing working-age populations,
85. Why didn't Britain tax India to finance its World War II enormous quantities of cheap new labour became accessible. In
efforts? 2007 China's economy expanded by an eye-popping 14.2%. India
(a) Australia, Canada and New Zealand had offered to pay managed 10.1% growth, Russia 8.5% and Brazil 6.1%. The IMF
for the Indian troops. now reckons there will be a slowdown in growth. China will grow
(b) India had already paid a sufficiently large sum during by just 7.6% in 2013 India by 5.6% and Russia and Brazil by 2.5%.
World War I. Other countries have impressive growth potential. 'Next 11' (N
11) which includes Bangladesh, Indonesia, Mexico, Nigeria and
(c) It was afraid that if India refused to pay, Britain's war
Turkey. But there are various reasons to think that this N11
efforts would be jeopardised.
cannot have an impact on the same scale as that of the BRICS.
(d) The British empire was built on the premise that the
The first is that these economies are smaller. The N11 has a
conqueror pays the conquered.
population of just over 1.3 billion, less than half that of the
(e) None of these BRICS. The second is that the N11 is richer now than the BRICS

www.newspaperkorner.wordpress.com
www.newspaperkorner.wordpress.com
SBI BANK PO MAIN EXAM 2016 2016-9
were back in the day. The third reason that the performance of the (d) The cost of living and level of inflation in these
BRICS cannot be repeated is the very success of that countries were maintained at low levels
performance. The world economy is much larger than it used to (e) All the given options are effects of the rise in BRICS
be twice as in real terms as it was in 1992 according to IMF economies
figures. But whether or not the world can build remarkable era of 90. What does the phrase "their ranks will swell but their tread
growth will depend in large part on whether new giants tread a will no longer shake the Earth as it once did" convey in the
path towards greater global co-operation or stumble in times of context of the passage?
tumult and in the worst case fight. (a) While many countries will try and achieve the same rate
(The topic of the Passage asked in the exam was based on Brain of growth as BRICS they will not succeed
drain in China) (b) The growth of BRICS countries has changed the
86. According to the passage which of the following is a reason world's economy in ways that any further growth will
for the author's prediction regarding N11 countries? not have such a disruptive effect on the world economy
(a) N11 countries are poorer, have less resources than (c) Developing countries have strengthened their fiscal
BRICS countries and do not have much scope to grow
systems in such a way that they will not be shaken to
(b) The size of these countries is too great to fuel a high such an extent again
rate of growth as expected by BRICS countries
(d) Poverty may increase as the gap between the rich the
(c) The world economy is so large that the magnitude of
poor increase but it will never reach the same levels as
growth from these countries will have to be huge to
equal the growth of BRICS prior to the crisis
(d) These economies are agricultural and have not opened (e) Citizens in advanced countries became much better off
up their economies yet so their scope of growth is than those in emerging economies
greater than that of BRICS 91. Which of the following best describes 'catch up growth'?
(e) Other than those given as options (a) Emerging economies tried but failed to catch up with
87. What is the author's view of globalization's second America which always grew at a higher growth rate
unbounding? (b) The size of emerging economies and their purchasing
(a) It proved beneficial since it created a large number of power has caught up with and now exceeds as rich
jobs and tremendous growth in crossborder trade countries together
(b) It disturbed the fragile balance of power among BRICS (c) The growth of the America economy determines the
nations and caused internal strife growth of emerging economies
(c) It caused untold damage to America's economy since it (d) In the latter half of the 1990s some emerging economies
restricted the spread of American firms off-shore out did America in terms of output per person
(d) It proved most beneficial for the agricultural sector (e) None of the given statements describes catch up
creating huge employment opportunities growth
(e) Citizens in advanced countries became much better off 92. Which of the following can be said about 'convergence with
than those in emerging economies a vengeance'?
88. What do the comparative statistics of 2007 and 2013 for
A. After the 1990s advanced economies like America were
BRICS countries published by the IMF as cited in the
open to the idea of free trade and globalization.
passage indicate?
(a) BRICS economies are contributing less to global B. There were huge technological advances which were
growth conducive to allowing business to spread their area of
(b) As the population of these countries grows its growth operations.
rate is falling C. Rich economies felt threatened by the competition from
(c) The financial practices followed by these countries will China.
continue to pay rich dividends (a) Only A (b) Only B
(d) These countries are creating global financial (c) Only C (d) A and B
imbalances to the detriment of smaller developing (e) B and C
economies like Africa 93. What is the author's main objective in writing this passage?
(e) IMF forecasts of growth rate for these countries have A. To urge emerging economies to deal with growth which
not been fulfilled can be disruptive maturely and without conflict.
89. What effect did rise in economies of BRICS have on the B. To point out that while the period of growth of BRICS
global economy? was disruptive this disruption has almost come to a
(a) It helped stabilize the globle economy and insulate it close.
from the fall out of the global financial crisis
C. To criticize advanced economies for their handling of
(b) Labour became more highly skilled and wages rose
growth and promoting competition and conflict in
alarmingly reducing the off-shoring of jobs to
certain regions.
developing countries
(a) A and B (b) Only A
(c) Though worldwide poverty rates tumbled, the gap
between the rich and the poor in rich economies (c) Only C (d) All A, B and C
increased (e) B and C

www.newspaperkorner.wordpress.com
www.newspaperkorner.wordpress.com
2016-10 SBI BANK PO MAIN EXAM 2016

DIRECTIONS (94-100) : In each of the following questions five how to drive more carefully and wearable devices will
options are given, of which one word is most nearly the same or nudge us toward healthier lifestyles.
opposite in meaning to the given word in the question. Find the (c) The better behaviour resulting from smart devices is
correct option having either same or opposite meaning. just one threat to the insurance industry. Conventional
risk pools (for home or car insurance, for example) are
94. Snitch
shrinking as preventable accidents decline, leaving the
(a) Bode (b) Stitch slow-footed giants of the industry at risk.
(c) Suffix (d) Sneak (d) The uncertainty that underpins the need for insurance
(e) Parity is now shrinking thanks to better insights into
95. Porch individual risks.
(a) Peek (b) Demur (e) The data has enabled insurance companies to gauge
(c) Verandah (d) Capitulate the situation and plan accordingly.
(e) Bigotry 102. By calling for exempting unionized businesses from the
96. Vituperate minimum wage, unions are creating more incentives for
(a) Examine (b) Variegate employers to favor unionized workers over the non-
(c) Belittle (d) Compliment unionized sort. Such exemptions strengthen their power.
(e) Baleful ____ ___________.Once employers are obliged to pay the
97. Conundrum same minimum wage to both unionized and non-unionized
(a) Abjure (b) Quash labor, workers often see less reason to pay the dues to join
(c) Riddle (d) Thrill a union.
(e) Vendetta (a) High rates of unionization make minimum-wage rules
98. Praise unnecessary as collaborative wage setting achieves
the flexibility goals of a low minimum wage and the
(a) Portend (b) Lash
fairness goals of a high one.
(c) Fidget (d) Creak
(b) Workers who have no real alternative to employment in
(e) Visage
the unregulated shadows of the labor market are even
99. Notional
more vulnerable to exploitation and abuse than workers
(a) Quixotic (b) Unworldly with the legal right to take low wages.
(c) Ethereal (d) Impalpable (c) The labor ethos of worker solidarity seems hollow if
(e) Cosmic non-union workers are underpriced by union workers
100. Vacillate and left unemployed or scrambling for unauthorized
(a) Dally (b) Hem work.
(c) Dither (d) Sway (d) This is useful because for all the effort unions throw at
(e) Waffle raising the minimum wage, laws for better pay have an
awkward habit of undermining union clout.
DIRECTIONS (101-105) : In each of the following questions a
short passage is given with one of the lines in the passage (e) Unions have been demanding democratic vaues in the
missing and represented by a blank. Select the best out of the work cluture but on the contrary they have been
five answer choices given, to make the passage complete and practicing dictatorial ways.
coherent (coherent means logically complete and sound). 103. The premise that the choice of major amounts to choosing a
career path rests on the faulty notion that the major is
101. ______________Business is instead moving to digital- important for its content, and that the acquisition of that
native insurers, many of which are offering low premiums to content is valuable to employers. But information is fairly
those willing to collect and share their data. Yet the biggest easy to acquire and what is acquired in 2015 will be obsolete
winners could be tech companies rather than the firms that by 2020. What employers want are basic but difficult-to-
now dominate the industry. Insurance is increasingly reliant acquire skills. _______________________. They care
on the use of technology to change behaviour; firms act as about a potential employee's abilities: writing, researching,
helicopter parents to policyholders, warning of impending quantitative, and analytical skills. A vocational approach to
harm-slow down; reduce your sugar intake; call the education eviscerates precisely the qualities that are most
plumber-the better to reduce unnecessary payouts. Yet this valuable about it: intellectual curiosity, creativity and
sort of relationship relies on trust, and the Googles and critical thinking.
Apples of the world, on which consumers rely day-by-day (a) As students flock to the two or three majors they see as
and hour-by-hour, may be best placed to win this business. good investments, professors who teach in those
(a) The growing mountain of personal data available to majors are overburdened, and the majors themselves
individuals and, crucially, to firms is giving those with become more formulaic and less individualized.
the necessary processing power the ability to (b) Often it is the art historians and anthropology majors,
distinguish between low-risk and high-risk individuals. for example, who, having marshaled the abilities of
(b) Cheap sensors and the tsunami of data they generate perspective, breadth, creativity, and analysis, have
can improve our lives; blackboxes in cars can tell us moved a company or project or vision forward.

www.newspaperkorner.wordpress.com
www.newspaperkorner.wordpress.com
SBI BANK PO MAIN EXAM 2016 2016-11
(c) Furthermore, the link between education and earnings greatly diluted by the amount of time that inevitably
is notoriously fraught, with cause and effect often passes between the time of the conduct and the
difficult to disentangle. punishment.
(d) Ideas such as education is necessary to be successful (e) While some victims and their families supported and
in corporate life are unacceptable because education some opposed the decision, any expectation that
isn't that much relevant into day's society. Tsarnaev will be put to death might be misplaced.
(e) When they ask students about their majors, it is usually 106. Five statements are given below, labelled a, b, c, d and e.
not because they want to assess the applicants' Among these, four statements are in logical order and form
mastery of the content, but rather because they want to a coherent paragraph/passage. From the given options,
know if the students can talk about what they learned. choose the option that does not fit into the theme of the
104. What happens to our brains as we age is of crucial passage.
importance not just to science but to public policy. (a) Dinets first observed the behaviour in 2007 when he
_______________________ However, this demographic spotted crocodiles lying in shallow water along the
time-bomb would be much less threatening if the elderly edge of a pond in India with small sticks or twigs
were looked upon as intelligent contributors to society positioned across their snouts.
rather than as dependants in long-term decline. It is time we (b) The behaviour potentially fooled nest-building birds
rethink what we mean by the ageing mind before our false wading in the water for sticks into thinking the sticks
assumptions result in decisions and policies that were floating on the water.
marginalize the old or waste precious public resources to re- (c) The crocodiles remained still for hours and if a bird
mediate problems that do not exist. neared the stick, they would lunge.
(a) The idea that we get dumber as we grow older is just a (d) Crocodiles are way clever than thought about
myth, according to brain research that will encourage generally.
anyone old enough to know better. (e) To see if the stick-displaying was a form of clever
(b) By 2030, for example, 72 million people in the US will be predation, Dinets and his colleagues performed
over 65, double the figure in 2000 and their average life systematic observations of the reptiles for one year at
expectancy will likely have edged above 20 years four sites in Louisiana, including two rookery and two
(c) Many of the assumptions scientists currently make non-rookery sites.
about 'cognitive decline' are seriously flawed and, for 107. Five statements are given below, labelled a, b, c, d and e.
the most part, formally invalid. Among these, four statements are in logical order and form
(d) Using computer models to simulate young and old a coherent paragraph/passage. From the given options,
brains, Ramscar and his colleagues found they could choose the option that does not fit into the theme of the
account for the decline in test scores simply by passage.
factoring in experience (a) The competitive pressures in the environment have
(e) The reason it becomes harder to recall an radically altered the context in which human Resource
acquaintance's name as you grow older is that there are services are delivered in Indian organizations.
so many more of them. (b) The HR competencies, in other words, differentiate
105. The expenditure of time, money and sparse judicial and outstanding performers from average performers in the
prosecutorial resources is often justified by claims of a HR function.
powerful deterrent message embodied in the ultimate (c) The traditional role of HR, based on the image of a
punishment- the death penalty.__________ transaction and administrative oriented HR practitioner
In 2010, the average time between sentencing and execution providing services to a set of customers, is undergoing
in the United States averaged nearly 15 years. A much more change.
effective deterrent would be a sentence of life imprisonment (d) With the focus moving towards integrating HR into
imposed close in time to the crime. strategic planning of the organization, another
(a) A single federal death penalty case in Philadelphia was dimension is added to the picture of HR service deliver.
found to cost upwards of $10 million - eight times (e) This change in focus calls for HR professionals taking
higher than the cost of trying a death eligible case up the emerging roles of advocate, business partner,
where prosecutors seek only life imprisonment. and change agent in new organizational structures that
(b) The ethics of the issue aside, it is questionable whether are radically different from the past.
seeking the death penalty is ever worth the time and 108. Five statements are given below, labelled a, b, c, d and e.
resources that it takes to sentence someone to death. Among these, four statements are in logical order and form
(c) Apart from delaying justice, the death penalty diverts a coherent paragraph/passage. From the given options,
resources that could be used to help the victims' choose the option that does not fit into the theme of the
families heal. passage..
(d) But studies repeatedly suggest that there is no (a) Every campaign leader known how to pick up and kiss a
meaningful deterrent effect associated with the death child in the crowds, how to hug an old widow, how to
penalty and further, any deterrent impact is no doubt chant with the pundits, and show abeyance to the
Mullahs.

www.newspaperkorner.wordpress.com
www.newspaperkorner.wordpress.com
2016-12 SBI BANK PO MAIN EXAM 2016
(b) Did anyone hit at "quality" for infrastructure amenities, 111. If the issuer issues masala bond in rupees, then he gets rid
education, health and finally governance. of the risk in the form of currency fluctuation which he
(c) Politics is still a game of money, mind and passes on to the investor. This bond brings a new and
manipulations. diversified set of investors for Indian companies and more
(d) False promise are not entirely a sin, but let these be liquidity in foreign exchange.
redeemed by true, professional, and quality A. Risk gets passed on the investor
governance, that shows at the end of the tenure. B. More liquidity in foreign exchange
(e) Many Asian countries have transformed their work C. Masala bonds bring a new set of investors
culture, and up-scaled their economies. Pick out the option which when used to start a sentence
109. Five statements are given below, labelled a, b, c, d and e. combines both the above sentences in one.
Among these, four statements are in logical order and form (a) Only A (b) Only B
a coherent paragraph/passage. From the given options, (c) Only C (d) Only A and B
choose the option that does not fit into the theme of the (e) None of the above
passage. 112. Captain Michael allowed his men to make important
(a) The emerging web services paradigm offers the decisions in a democratic manner. This democratic attitude
promise of new efficiencies and improve integration fostered a spirit of togetherness and commitment on the
designed to enhance collaboration between internal part of Michael's fellow explorers.
and external applications. A. As soon as
(b) For example web services can serve as a bridge B. In accordance with
between an e-procurement application and an internal C. Allowing
inventory system. Pick out the option which when used to start a sentence
(c) Although, web services are relatively nascent and combines both the above sentences in one.
adoption rates currently low, it is critical for ERP (a) Only A (b) Only B
vendors to begin taking steps to prepare for their (c) Only C (d) Only A and B
rapidly maturing initiative. (e) None of the above
(d) As items are purchased through e-procurement 113. High interest rates on the loan the business procured
application, a web service specific to inventory consumed so much of their revenue that they were forced to
reduction can be invoked to adjust inventory levels. liquidate some of their holdings. All this happened when its
(e) As soon as re-order points are hit, a Web services, the refinancing attempts failed.
cycle time between buying and replenishment can be A. The high interest rates of the loan
greatly reduced and the potential for errors virtually B. The business was forced to liquidate
eliminated. C. The high interest rate of the loan
110. Five statements are given below, labelled a, b, c, d and e. Pick out the option which when used to start a sentence
Among these, four statements are in logical order and form combines both the above sentences in one.
a coherent paragraph/passage. From the given options, (a) Only A (b) Only B
choose the option that does not fit into the theme of the (c) Only C (d) Only A and C
passage. (e) None of the above
(a) Much of the modern use of metals happens behind 114. Reporting a 90 percent drop in net income during the
closed doors of corporations, under the veil of trade second quarter, dragged down by restructuring charges
secrets. and weak sales, the earnings guidance for the year was
(b) He chooses to restrict his analysis to metals and withdrawn by Best Buy Co.
metalloids, which could face more critical constraints A. Report of a 90 percent drop in net income
because many of them are relatively rare. B. Best Buy Co. is reporting a 90 percent drop in net
(c) Even if we can find out how certain metals are used, it income
may not always be possible to determine the C. Best Buy Co. has reported a 90 percent drop in net
proportions they are used in. income
(d) The authors compromise was to account for the use of Pick out the option which when used to start a sentence
80% of the material that is made available each year combines both the above sentences in one.
through extraction and recycling. (a) Only A (b) Only B
(e) Their compromise was to account for the use of 80% of (c) Only C (d) Only A and B
the material that is made available each year through (e) None of the above
extraction and recycling. 115. Percival Lowell was interested in astronomy due to his
DIRECTION (111-115) : Select the phrase/connector from the belief in canals on Mars. However, modern astronomers
given three options which can be used to form a single sentence dismiss this belief as material for pop science fiction.
from the two sentences given below, implying the same meaning A. Percival Lowell was interested in astronomy
as expressed in the statement sentences. B. Percival Lowell's interest in astronomy was due to
C. Percival Lowell was interested in astronomy

www.newspaperkorner.wordpress.com
www.newspaperkorner.wordpress.com
SBI BANK PO MAIN EXAM 2016 2016-13
Pick out the option which when used to start a sentence (a) Visakhapatnam, Andhra Pradesh
combines both the above sentences in one. (b) Ladakh, Jammu and Kashmir
(a) Only A (b) Only B (c) Ajmer, Rajasthan
(c) Only C (d) Only A and C (d) Dehradun, Uttarakhand
(e) None of the above (e) None of these
124. Dalai Lama has been given the honorary citizenship of
GENERAL /ECONOMY/BANKING which among the following cities?
(a) Moscow, Russia
AWARENESS
(b) Rome, Italy
116. Indravati National Park is a national park located in which (c) Milan , Italy
among the following states? (d) Paris, France
(a) Odisha (b) West Bengal (e) Sydney, Australia
(c) Chhatisgarh (d) Assam 125. The Election Commission of India has held the first ever
(e) Tamil Nadu International Conference on Voter Education in which city?
117. EFTPOS is an electronic payment system involving (a) Patna (b) Chennai
electronic funds transfers based on the use of payment (c) Lucknow (d) New Delhi
cards, such as debit or credit cards, at payment terminals (e) Kolkata
located at points of sale. What is E in EFTPOS? 126. The 11-member committee, headed by former Finance
(a) Electronic (b) Essential Secretary Ratan P Watal is related with which among the
(c) Every (d) Effective following committee?
(e) None of the above (a) Incentivise transactions through cards and digital
118. The Book "One Indian Girl" is the 7th fictional novel and (b) One Rank One Pension
the 9th overall novel written by the Indian author.......? (c) Indo-Pakistani war and Simla Agreement
(a) Vikram Seth (b) Arundhati Roy (d) Financial inclusion or inclusive financing
(c) Kiran Desai (d) Aravind Adiga (e) None of these
(e) Chetan Bhagat 127. Corporation Bank is a public sector banking company
119. Bhavanisagar Dam or Lower Bhavani Dam, is located in headquartered in..........?
which state? (a) Kolkata (b) Mangalore
(a) Kerala (b) Tamil Nadu (c) Chennai (d) Bengaluru
(c) Maharashtra (d) West Bengal (e) Mumbai
(e) Madhya Pradesh 128. Cash Reserve Ratio (CRR) is a specified minimum fraction
120. RBI believes this boost to consumption will lead to gross of the total deposits of customers, which commercial banks
value added (GVA) growth of _____ for 2016-17? have to hold as reserves either in cash or as deposits with
(a) 7.1 per cent (b) 7.0 per cent the central bank. CRR is set according to the guidelines of
(c) 7.9 per cent (d) 7.6 per cent the central bank of a country. CRR controls-
(e) 7.3 per cent (a) It cheaper for banks to borrow money
121. The Sheikh Zayed Stadium is a cricket ground located in (b) Is a tool, which central bank uses for short-term
which among the following cities? purposes
(a) Abu Dhabi, UAE (c) Greater control to the central bank over money supply
(b) Dubai, UAE (d) Marking a shift from earlier method of calibrating
(c) Karachi, Pakistan various policy rates separately
(d) Lahore, Pakistan (e) None of these
(e) Hyderabad, India 129. Basic Savings Bank Deposit Account (BSBDA) is a Zero
122. The Organization of the Petroleum Exporting Countries Balance Savings Account that takes care of your simple
(OPEC) is a permanent, intergovernmental Organization, banking needs with Free ATM card, monthly statement,
created at the Baghdad Conference on September 10-14, and cheque book. How much amount has withdrawal and
1960, by Iran, Iraq, Kuwait, Saudi Arabia and Venezuela. transfer limit under BSBDA?
Where is the headquartered of OPEC? (a) ` 1,00,000 per month
(a) Geneva, Switzerland (b) ` 20,000 per month
(b) Belgium, Brussels (c) ` 50,000 per year
(c) New York, USA (d) ` 10,000 per month
(d) Paris, France (e) No Limit
(e) Vienna, Austria 130. Grama Vidiyal Microfinance, a Trichy-based microfinance
123. Indian and Chinese armies have held the second joint company has acquired by which among the following
tactical exercise "Sino India Cooperation 2016 in which Banks?
city?

www.newspaperkorner.wordpress.com
www.newspaperkorner.wordpress.com
2016-14 SBI BANK PO MAIN EXAM 2016
(a) Punjab National Bank 139. In a bid to boost credit growth in the economy, the Centre
(b) Bandhan Bank announced a sum of how much crore for recapitalisation of
(c) ICICI Bank 13 public sector banks?
(d) State Bank of India (a) ` 22,915 crore
(e) IDFC Bank (b) ` 27,110 crore
131. Nassau is the capital, largest city, and commercial centre of (c) ` 10,000 crore
the Commonwealth of the............? (d) ` 50,207 crore
(a) Kenya (b) Argentina (e) ` 38,500 crore
(c) Uganda (d) Iceland 140. Payments banks are a new model of banks conceptualized
(e) Bahamas by the Reserve Bank of India (RBI). These banks can
132. Which is a hybrid security, combining features of preferred accept a restricted deposit which is currently limited to INR
stock and corporate bonds? 1 lakh per customer account. How much amount minimum
capital has requirement for Payments banks?
(a) UPI (b) NPCI
(a) ` 1 crore (b) ` 50 crore
(c) NEFT (d) MIPS
(c) ` 1000 crore (d) ` 500 crore
(e) RTGS
133. Which among the following countries North Atlantic (e) ` 100 crore
Treaty Organization (NATO) leaders will hold their next 141. Bharat Bill Payment System is a major component of the
summit in 2017? retail payment transactions in India, and is characterized by
the presence of large number of billers, who provide a
(a) Beijing, China (b) New York, USA
variety of payment options to their customers. According
(c) Paris, France (d) Brussels, Belgium to information available on the RBI website, the top 20
(e) London, UK cities are generating INR how much amount in bill
134. Kalikho Pul who has found hanging from a ceiling fan was payments every year ?
the Chief Minister of which state? (a) INR 16,810 billion
(a) Andhra Pradesh (b) INR 4,703 billion
(b) Madhya Pradesh (c) INR 10,172 billion
(c) Arunachal Pradesh (d) INR 6,223 billion
(d) Himachal Pradesh (e) None of the above
(e) Uttar Pradesh 142. Currency risk is the potential risk of loss from fluctuating
135. The highest currency note ever printed by the Reserve foreign exchange rates when an investor has exposure to
Bank of India (RBI) was a Rs 10,000 note during the British foreign currency or in foreign-currency-traded
Raj. It was printed first in- investments. Currency risk is sometimes referred to as-
(a) 1919 (b) 1946 (a) Monetary Rate
(c) 1938 (d) 1912 (b) Export Rate
(e) 1978 (c) Trade Rate
136. The Bermuda Triangle, also known as the Devil's Triangle, (d) Exchange-rate risk
is a loosely-defined region in the western part of which (e) None of the above
Ocean? 143. The government has named _________ outside experts as
(a) Australia Ocean members of the Monetary Policy Committee (MPC) of the
(b) Atlantic Ocean Reserve Bank of India (RBI), moving to a model followed in
(c) Southern, Ocean the developed world.
(d) Indian, Ocean (a) One (b) Two
(e) None of the above (c) Three (d) Four
137. The Antwerp Diamond Bank is a small, 75-year-old bank (e) Five
that specializes exclusively in serving the diamond and the 144. MasterCard is rolling out its Identity Check service,
diamond jewelry sector. Where is the headquarter of the popularly known as ________ to customers in Europe
Antwerp Diamond Bank? through a new mobile app.
(a) Vienna, Austria (a) Selfie pay (b) Voice Record
(b) Antwerp, Belgium (c) Finger print (d) Video play
(c) Frankfurt, Germany (e) None of these
(d) Beijing, China 145. Who has launched the party name as People's Resurgence
Justice Alliance (PRJA)?
(e) Washington DC, USA
138. Typhoon Haima made a second landfall in which country? (a) Niranjan Jyoti
(a) China (b) India (b) Irom Sharmila
(c) Japan (d) Malaysia (c) Aung San Suu Kyi
(e) Singapore (d) Yogi Adityanath
(e) None of these

www.newspaperkorner.wordpress.com
www.newspaperkorner.wordpress.com
SBI BANK PO MAIN EXAM 2016 2016-15
146. The Reserve Bank of India (RBI) has released the Report of (c) Bharat Bill Payment System (BBPS)
the Internal Working Group (IWG) on Rationalisation of (d) Unified Payments Interface (UPI)
Branch Authorisation Policy. The Group is chaired (e) None of these
by............? 151. Name the department that was set up by Union Finance
(a) Bibek Debroy (b) Madhukar Gupta Ministry to streamline government borrowings and better
(c) NR Nagendra (d) Ratan P Watal cash management with the overall objective of deepening
(e) Lily Vadera bond markets.
147. Mobile Money Identification Number (MMID) is a (a) Debt Management Cell
________ digit number of which the first four digits are the (b) Department of Debt Management
unique identification number of the bank offering (c) Cell for management of debt
Immediate Payment Service (IMPS). (d) All of the above
(a) Seven (b) Thirteen (e) None of these
(c) Eleven (d) Nine 152. The lock-in period in case of a term deposit means -
(e) None of these (a) Locker service is not available
148. All bank loans, including home loans, taken are now linked (b) Premature withdrawal is not possible
to the bank's marginal cost of funds based lending rate (c) Another account cannot be opened
(MCLR). MCLR has effective from?
(d) No further investment is possible
(a) January 1, 2016
(e) None of these
(b) September 1, 2015 153. Who of the following won India's first gold medal at the Rio
(c) April 1, 2015 Paralympic Games.
(d) April 1, 2016 (a) Varun Singh Bhati
(e) None of these (b) Mariyappan Thangavelu
149. Two professors from Harvard and MIT have been awarded (c) Ankur Dhama
the Nobel Prize in economics for contributions to
(d) Virender Dhanka
(a) Financial and real estate markets
(e) None of these
(b) Private prisons 154. India in September 2016 signed Air Services Agreement
(c) Contract theory under Open Sky Policy with which country ?
(d) Essential to the functioning (a) Russia (b) Thailand
(e) Research sheds light (c) Greece (d) Switzerland
150. _________ is a system that powers multiple bank (e) None of these
accounts into a single mobile application (of any 155. Which of the following financial institutions is based in the
participating bank), merging several banking features, Basel city of Switzerland?
seamless fund routing & merchant payments into one
(a) IMF (b) World Bank
hood.
(c) ADB (d) BIS
(a) Real-time gross settlement systems (RTGS)
(e) WTO
(b) Electronic Funds Transfer at Point of Sale (EFTPOS)

www.newspaperkorner.wordpress.com
www.newspaperkorner.wordpress.com
2016-16 SBI BANK PO MAIN EXAM 2016

Answers &
Explanations
1. (d) All and 81 49 best equal 11 25 my 47 strength 16 32. (a) Only I
2. (b) All and 81 49 best equal 47 25 my strength 11 16
3. (a) All and 81 11 best my 47 25 equal strength 16 49
4. (d) All and 81 49 best equal 47 25 11 my strength 16
5. (d) All best 81 49 and equal 47 25 my strength 16 11 M O P
Sol. 6-10 33. (c) ah koj zee pig Þ can you take that
Floor No Pers on Food Recipe et zee lin ter' means Þ you may come now common
7 G No o dles Zee Þ you
6 B Briyan i 34. (c) K, P>L……..N……….
5 A Pizza K.P>L>M>N>
35. (e) From I: P³R=Q³S,T
4 E Eg g ro ll
From II: S=R£Q<P
3 D M as ala d os a 36. (d) According to the statement, 80% of the total runs
2 F Burger were made by spinners. So, I does not follow. Nothing
1 C Fried Rice about the opening batsmen is mentioned in the
statement. So, II also does not follow.
6. (d) 7. (a) 8. (c) 9. (c) 10. (e)
37. (a) Clearly, I directly follows from the given statement.
11. (b) 12. (d) 13. (e) 14. (d) 15. (b)
Also, it is mentioned that old ideas are replaced by
Solution 16-20
new ones, as thinking changes with the progressing
S time. So, II does not follow.
L P 38. (e) According to the statement, Government has spoiled
financial institutions by appointing bureaucrats as
Directors. This means that only those persons should
N Q
be appointed as Directors who are experts in finance
and are acquainted with the financial work of the
R O institute. So, both I and II follow.
M 39. (b) The fact given in I is quite contrary to the given
16. (a) 17. (a) 18. (c) 19. (d) 20. (a) statement. So, I does not follow. II mentions the direct
21. (d) Lowest Number = 129 + 4 = 133 implications of the state discussed in the statement.
Highest Number = 921 + 4 = 925 Thus, II follows.
3+5=8 40. (d) The statement concentrates on the increasing
22. (c) 642 921 viewership of TV. and does not stress either on
23. (e) Third digit becomes the first while the first shift two increasing the readership of newspapers or making
shift rightwards. studies regarding the same. So, neither I nor II
24. (e) 2 + 4 + 2 + 6 + 3 = 17 follows.
25. (b) 921 246 129 674 803 41. (e) The performance of the individual has to be tested
246 - second lowest number over a span of time as the statement mentions. So, I is
Solution 26-30 implicit. The statement mentions that the individual's
worth shall be reviewed (during probation period)
Q(-) T(-) R(+) S(-) V(+) P(+) U(+) W(+) X(+)
before confirmation. So, II is also implicit.
42. (a) Since the statement talks of putting the child in school
26. (a) 27. (d) 28. (a) 29. (b) 30. (c) at the age of 5, it means that the child is mentally
31. (d) Either I or II prepared for the same at this age. So, I is implicit. But
nothing about admission after 6 years of age is
4m A 3m
Form I D D mentioned in the statement. So, II is not implicit.
4m 43. (b) Assumption I goes against the statement. So, it is not
implicit. The allowance will serve as a reward to the
B employees and shall provoke them to come on time.
D So, II is implicit.
Form II 3m A 4m
3m C 44. (a) I directly follows from the statement and so is implicit.
3m
Also, the statement is a suggestion and does not tell
about a government policy or its position of funds.
F So, II is not implicit.
B
www.newspaperkorner.wordpress.com
www.newspaperkorner.wordpress.com
SBI BANK PO MAIN EXAM 2016 2016-17
45. (a) The mother warns her child with the expectation that 55. (e) (x - 4) (4x + 3) = 0
he would stop troubling her. So, I is implicit. The x = 4, -3/4
general nature of children cannot be derived from the (x + 5) (x-12) = 0
statement. So, II is not implicit. y = 12, -5
p´4 56. (c) x = 32
46. (d) 2900 = p + ´4 y = ±32
100
x³y
\ p = 2500
3x 24
1300 ´ 100 ´100 57. (e) =
47. (b) Marked Price = = 2500 4x t
80 ´ 65 We get t = 32 minutes
48. (b) Time taken by P to reach city B is 6hr. In 6 hr, distance
1 1
covered by Q is 30km. 58. (e) From I: ´ 10 + ´ 15 = 1
Now at some x distance they will meet. So A B

x 30 – x 1 1
= Þ x = 10 From II: =
5 10 A 14
So distance between A and Y is 30+10 =40 km From both, answer can be found.
59. (d) From I: X' age = 3x, Y's age = 2x
18
49. (b) 8x + x + 18 = 5/7 1 1 1
13 From II :
× 3x = (1/4) × 2x. this gives =
56x + 126 = 65x + 90 6 2 2
9x = 36 So no conclusion.
x= 4 60. (e) From both equations, we get l = 12 and b = 6 so
B = 13 × 4 + 18 = 70 perimeter = 36
61. (d) D = 4 × (b+s) and D = 6 × (b – s)
1 æ1 1ö 5 1 So we can't find the value of b.
– + = 10 – 4 –
2 çè 5 4 ÷ø
50. (d) Z's one day work = =
20 20
3472
1 1 1 62. (d) = 56.7 = 57%
Ratio = : : = 4 : 5 :1 6120
5 4 20
63. (a) Company 1
1
Z = 5000 × = 500 20400
10
P level = 24 ´ = 4896
7 100
51. (e) = x
x 20400
x= 7 Q level = 8 ´ = 1632
100
(y35/2 – 75/2 )/ y = 0 20400
y5/2 = 75/2 R level = 14 ´ = 2856
100
y= 7
x= y 20400
S level = 30 ´ = 6120
52. (a) (5x + 2)(5x + 3) =0 100
x = -2/5, -3/5 = -0.4, -0.6
(5y + 10) (y + 2) = 0 20400
T level = 11 ´ = 2244
x= -10/5, -2 = -2, -2 100
-0.4, -0.6, -2, -2 20400
x> y U level = 13 ´ = 2652
100
53. (a) (2x + 8) (2x + 3) = 0
x = -8/2, -3/2 = -4,-1.5 Company 2
(2y + 10)(2y + 9) = 0 24800
y = -10/2, -9/2 = -5, -4.5 P level = 14 ´ = 3472
100
-1.5,-4,-4.5,-5
x> y 24800
54. (a) (x - 4) (4 x - 3) = 0 Q level = 16 ´ = 3968
100
x = 4, 3/4
(y + 2)(3y + 2) = 0 24800
R level = 32 ´ = 7936
y = -2, - 2/3 100
3 2 24800
4, , – ,-2 S level = 8 ´ = 1984
4 3 100
x> y

www.newspaperkorner.wordpress.com
www.newspaperkorner.wordpress.com
2016-18 SBI BANK PO MAIN EXAM 2016
Total = 18+22.4 = 40.4L
24800
T level = 18 ´ = 4464 73. (a) A : B = 40:50
100
100
24800 45 = (I - 40) × … (A)
U level = 12 ´ = 2976 40
100 I = 58L
Company 1 - R = 2856
Company 2 - U = 2976 100
50 = (I - 50) ×
This only approximately equal 50
64. (b) Company 1 - Q = 1632 I = 75L … (B)
Company 2 - Q = 3968 A:B = 58:75
74. (d) Over all % is not known
1632 ´100
%= = 41.12% 75. (d) Income => 1992:1993 = 3:4 = 30:40
3968
65. (c) Company 1 - R = 2856 100
45 = (30 – E) ×
Company 2 - R = 7936 E
7936 – 2856 = 5080 45E
66. (d) Company 1 - P = 4896 = 30 - E
100
Company 2 - R = 7936
7936 + 4896 = 12832 9E
= 30 - E
67. (b) 20
9E = 600 - 20E
29E = 600
600
E= ……………1992
29
100
8:7 = 320 (boys) : 280 (girls) 50 = (40 - E) ×
E
600 50E
20% both = 20 ´ = 120 = 40 - E
100 100
320 E = 2(40 - E)
B - (phy ) = 40 ´ = 128 E = 80 - 2E
100
3E = 80
120
B - (both) = 25 ´ = 30 80
100 E=
3
280 Ratio = (600/29) / (80/3) = 600*3 / 29*80
G - (chem) = 45 ´ = 126 = 1800/2320 = 180/232 = 45/58
100
Total no of boys(phy) = 128+30 = 158 76. (c) For A
68. (c) No of girls (phy) = (280-126 - 90) = 64 100
69. (b) Total no of students went to only phy = 128+64 = 192 40 = (I - E) ×
E
100 40E = (I - E)100
% = 192 ´ = 32% 2E = 5I - 5E
600
70. (a) Total no of Girls(chem) = 126+90 = 216 5I = 7E
71. (d) Total no of students(chem) = (320-128-30)B + 126G = 7E
162 + 126 = 288 I=
5
72. (b) 1994 E = 12 L For B
% P = (I – E) × 100 / E
100
100 50 = (116-I - E) ×
50 = (I – 12) × E
12
E = 2(116 - I - E)
I = 6 + 12 = 18 L E = 232 -2I - 2E
1995 E = 14L 2I = 232 - 3E
100 232 - 3E
60 = (I – 14) × I=
14 2
I = 22.4L

www.newspaperkorner.wordpress.com
www.newspaperkorner.wordpress.com
SBI BANK PO MAIN EXAM 2016 2016-19
while, compliment means to praise someone politely.
-232 - 3E
7/5E = 97. (c) Conundrum or Riddle means a confusing and difficult
2 problem.
14E = 1160 - 15E 98. (b) Praise means to express warm approval while lash
29E = 1160 means to beat with a stick.
1160 99. (a) Notional or Quixotic means not existing in reality.
E= = 40 100. (c) Vacillate or Dither means to wave between different
29
opinions or actions.
2E = 80
101. (c) 102. (d) 103. (e) 104. (b) 105. (d)
7´6 106. (d) Option (d) does not fit into the theme of the passage
77. (a) 7C =
2
= 21
2 as the given passage delineates the systematic
4C × 3C + 4C = 4 × 3 + (4 × 3)/2 = 12+6 = 18 observations of reptiles. It does focus the nature of
1 1 2
P = 18/21 = 6/7 crocodiles particularly.
78. (b) 17C = 17 × 16 × 15/3 × 2 × 1 = 4080/6 = 680 107. (b) Option (b) is odd one out into the theme of the passage.
3
3C = 1 The passage tells about the changing role of HR in the
3
P = 1/680 context of radically altered environment not about HR
79. (a) 17C = 17 × 16 × 15 ×14 ×13/5 × 4 × 3 × 2 × 1 = 742560/ competencies.
5
120 = 6188 108. (e) The passage depicts about the strategies followed by
11C = 11×10 × 9 × 8 × 7/5 × 4 × 3 × 2 × 1 = 55440/120 = politicians during the election campaign. It does not
5
462 pay attention upon economies.
P = 462/6188 = 33/442 109. (c) Option (c) does not fit into the theme of the passage.
80. (d) 17C = 17 × 16 × 15/3 × 2 × 1 = 4080/6 = 680 110. (b) Option (b) is odd one out, all other options describes
3
4C × 4C = 4 × 4 × 3/2 × 1 = 24 the modern uses of metals while option (b) is related
1 2
P = 24/680 = 3/85 to analysis of metals.
81. (b) 82. (d) 83. (c) 84. (a) 85. (c) 111. (e) 112. (c) 113. (b) 114. (c) 115. (c)
86. (e) 87. (a) 88. (a) 89. (c) 90. (b) 116. (c) 117. (a) 118. (e) 119. (b) 120. (d)
91. (d) 92. (b) 93. (b) 121. (a) 122. (e) 123. (b) 124. (c) 125. (d)
94. (d) Snitch and Sneak both have same meaning. Snitch 126. (a) 127. (b) 128. (c) 129. (d) 130. (e)
and Sneak both mean to inform on someone. 131. (e) 132. (d) 133. (d) 134. (c) 135. (c)
95. (c) A porch or verandah means a covered shelter 136. (b) 137. (b) 138. (a) 139. (a) 140. (e)
projecting in front of the entrance of a building. Both 141. (d) 142. (d) 143. (c) 144. (a) 145. (b)
have same meaning. 146. (e) 147. (a) 148. (d) 149. (c) 150. (d)
96. (d) Vituperate and compliment both are antonyms. 151. (a) 152. (b) 153. (a) 154. (c) 155. (d)
Vituperate means to insult someone in violent language

www.newspaperkorner.wordpress.com
www.newspaperkorner.wordpress.com
2016-20 SBI BANK PO ONLINE PRELIMINARY EXAM 2016

SBI Bank PO Online Preliminary


Exam 2016 Held On : 10-07-2016
Max. Marks : 100 (Based on Memory) Time : 1 hr.

(a) P and S
REASONING ABILITY (b) The person who went to tour in May and T
(c) R and V
DIRECTION (Qs.1-5): Study the following information (d) The persons who went tour in March and June
carefully to answer the given questions. (e) None of these
Eight people - P, Q, R, S, T, U, V and W are sitting in a straight DIRECTION (Qs. 6-10): Study the following information
line facing North. Each of them went to tour in different carefully to answer the given questions.
months, viz January, February, March, April, May, June, July
Eight friends - P, Q, R, S, T, U, V and W are sitting around a
and August but not necessarily in the same order.
circular table All of them are facing the centre. Each of them
V sits third to the right of the person who went to tour in May.
likes different subjects i.e. Economics, Statistics, English,
V went to tour after P. The person who went to tour in August
History, Geography, Biology, Chemistry and Physics but not
sits second to the right of V. R went to tour before July. Neither
necessarily in the same order.
P nor T went to tour in either May or August. Neither P nor S likes Physics. R and T are immediate neighbours of each
T is an immediate neighbour of V. W sits third to the right of other. The one who likes Geography sits to the immediate left
the person who went to tour in January. Neither P nor T went of Q. The one who likes biology sits second to the right of the
to tour in January. W went to tour in one of the months one likes Statistics. The one who likes Chemistry is an
mentioned above but not in August. Only two people sit immediate neighbour of the one who likes History. W is second
between T and the person who went to tour in July. The person to the right of U. Q is sitting second to the left of U. Neither
who went to tour in February sits on the immediate left of S. R nor T likes History. The one who likes Physics is an
Only one person sits between T and Q. T went to tour after immediate neighbour of U. R is second to the right of the one
April. P and T are immediate neighbours of each other. who likes Economics. There are only three people sits between
1. In which of the following months, W went to tour? the one who likes Physics and P. Only one person sits between
(a) April (b) June the one who likes History and P. The one who likes Statistics
(c) July (d) February is to the immediate right of the one who likes History.
(e) March 6. Who among the following likes English?
2. Who among the following sits exactly between T and Q? (a) Q (b) P
(a) The person who went to tour in May (c) R (d) S
(b) The person who went to tour in January (e) None of these
(c) S 7. In the arrangement, how many persons are there between
(d) P U and T?
(e) The person whose joining date is in August (a) Five (b) Two
3. Which of the following is true regarding T? (c) Three (d) Four
(a) Only two people sit to the left of T. (e) None of these
(b) S is sitting second to the right of the person who went 8. Which of the following statement is true about V?
to tour in July (a) V is an immediate neighbour of P
(c) T and Q are immediate neighbours of S (b) The immediate neighbor of V are R and T
(d) S went to tour in May (c) V is third to the right of the person who likes Biology
(e) None of the Above (d) None of the given statements is true
4. How many people sit between R and the person who went (e) V sits opposite to R, who likes Geography
to tour in April? 9. Who among the following sits third to the right of S?
(a) None (b) One (a) The person who likes Statistics
(c) Two (d) Three (b) The person who likes Economics
(e) Four (c) The person who likes Physics
5. Who among the following are sitting at the extreme ends (d) Can't be determined
of the line? (e) None of these

www.newspaperkorner.wordpress.com
www.newspaperkorner.wordpress.com
SBI BANK PO ONLINE PRELIMINARY EXAM 2016 2016-21
10. Four of the following five are alike in a certain way and 17. Which of the following combination get south facing flats?
hence form a group. Which is the one that does not belong (a) QTS (b) UPT
to that group? (c) URP (d) Data is inadequate
(a) S, R (b) R, P (e) UQS
(c) Q, R (d) S, V
DIRECTION (Q (18-22): Read the following information
(e) V, P
carefully and answer the questions that follows :
DIRECTION (Qs.11-15) : Study the following information P@Q - P is neither greater than nor equal to Q
carefully to answer the given questions P%Q - P is neither smaller than nor equal to Q
Seven Members Sania, Kamal, Pankaj, Anand, Arjun, Shewag P#Q - P is not greater than Q
and Sreejesh represents seven different states Madhya P$Q - P is not smaller than Q
Pradesh, Uttar Pradesh, Bihar, Kerala, Haryana, Odisha and P*Q- P is neither smaller than nor greater than Q
Maharashtra in seven different games Hockey, Chess, Cricket, 18. Statements: - A@B, B%C, C*D, D$E
Badminton, Table Tennis, Golf and Billiards. The order of Conclusions: - I. B%E II. A%E
persons, states and games is not nescessarily in the same order. (a) Only conclusion I follows
· Anand represents Kerala in chess. Arjun represents golf (b) Only conclusion II follows
team but not from Maharashtra or Uttar Pradesh. (c) Either conclusion I or II follows
· Sania represents Madhya Pradesh for either badminton or (d) Neither conclusion I nor II follow
table tennis. Pankaj represents Odisha but not for cricket (e) Both conclusion one and two follows
or Table tennis. 19. Statements: - A%B, B*C, C#D, D*E
· The one who represents Bihar represent in Table tennis. Conclusions: - I. A%C II. E$B
The one who represents Hockey represents Uttar Pradesh. (a) Only conclusion I follows
· Shewag represents Cricket and from Neither Bihar nor (b) Only conclusion II follows
Uttar Pradesh.Sreejesh does not represent Bihar. (c) Either conclusion I or two follows
11. Who among the following represent Bihar ? (d) Neither conclusion one nor second follow
(a) Kamal (b) Anand (e) Both conclusion one and two follows
(c) Sreejesh (d) Sania 20. Statements: - A*B, B$C, C#D, D@E
(e) None of these Conclusions: - I. E*A II. C%A
12. Which of the following combination is correct ? (a) Only conclusion I follows
(a) Pankaj - Kerala (b) Kamal - Cricket (b) Only conclusion II follows
(c) Shewag - Maharashtra (d) Anand - Golf (c) Either conclusion I or IIfollows
(d) Neither conclusion I nor II follow
(e) None of these
(e) Both conclusion I and II follows
13. Who represents Badminton ?
21. Statements: - A$B, B$C, C*D, D@E
(a) Arjun (b) Kamal
Conclusions: - I. A$E II. E%C
(c) Sreejesh (d) Sania
(a) Only conclusion I follows
(e) None of these
(b) Only conclusion II follows
14. Four among the following form a group in a certain
(c) Either conclusion I or II follows
way.Which of the following does not belong to Group ? (d) Neither conclusion I nor II follow
(a) Sania-Table Tennis - Bihar (e) Both conclusion I and II follows
(b) Anand -Kerala - Billiards 22. Statements: - A%E, E@C, C%B, B*D
(c) Arjun -Haryana - Golf Conclusions: - I. C%D II. B@A
(d) Pankaj-Uttar Pradesh - Hockey (a) Only conclusion I follows
(e) Sreejesh - Chess - Madhya Pradesh (b) Only conclusion II follows
15. Who among the following represents Uttar Pradesh ? (c) Either conclusion I or two follows
(a) Kamal (b) Arjun (d) Neither conclusion I nor II follow
(c) Pankaj (d) Sreejesh (e) Both conclusion I and II follows
(e) None of these
DIRECTION (23-25): Read the following information
DIRECTIONS (Qs.16-17): Each of the following questions is carefully to answer the following questions
based on the following information:
'P # Q' means 'P is the daughter of Q'
1. Six flats on a floor in two rows facing North and South are 'P © Q' means 'Q is the brother of P'
allotted to P, Q, R, S, T and U. 'P = Q' means 'Q is the sister of P'
2. Q gets a North facing flat and is not next to S. 'P & Q' means 'P is the son of Q'
3. S and U get diagonally opposite flats. 'P * Q' means 'P is the father of Q'
4. R next to U, gets a south facing flat and T gets North 'P @ Q' means 'P is the mother of Q'
facing flat. 23. What does the expression 'P @ R = S © T & V'?
16. If the flats of P and T are interchanged then whose flat will (a) V is the husband of P
be next to that of U? (b) R is the son of V
(a) P (b) Q (c) R is the daughter of V
(c) R (d) T (d) V is the wife of P
(e) None of the above (e) None of these

www.newspaperkorner.wordpress.com
www.newspaperkorner.wordpress.com
2016-22 SBI BANK PO ONLINE PRELIMINARY EXAM 2016
24. Which of the following indicates that 'C is the paternal 29. Which of the following statements is True with respect to
uncle of D'? the given arrangement?
(a) C & V # N @ L © D (a) The one who owns Honda went to tour on the day
(b) C & V & L @ N © D immediately after E.
(c) D & L & N @ V © C (b) F likes Fiat
(d) D & N # V @ L © C (c) None of the given options is true.
(e) None of these (d) Only four persons live between D and G
25. Which of the following can be the correct conclusion (e) G lives immediately below E.
drawn from the expression 30. Who among the following lives on the floor numbered 2?
'L = M # N © P * Q'? (a) B
(a) Q is the grandson of M (b) The one who owns Fiat
(b) L is the uncle of N (c) The one who owns Hyundai
(c) N is the uncle of Q (d) D
(d) Q is the niece of N (e) F
(e) None of these
DIRECTION (Qs.31-35): Study the following information
DIRECTION (Qs.26-30) Study the following information carefully to answer the given questions.
carefully to answer the given questions.
Ten persons from different cities viz. Delhi, Jaipur, Patna,
Seven persons - A, B, C, D, E, F and G live on seven different Indore, Mumbai, Chennai, Hyderabad, Bengaluru, Ranchi and
floors of a building but not necessarily in same order. Surat are sitting in two parallel rows containing five people
Lowermost floor of building is numbered 1, one above that is
each, in such a way that there is an equal distance between
numbered 2 and so on till top most floor is numbered 7. Each
adjacent persons. In row 1- A, B, C, D and E are seated and
one of them also own different brands of Car, namely Fiat,
all of them are facing south. In row-2 P, Q, R, S and T are seated
Hyundai, Honda, Skoda, Toyoto, Ford and Chevrolet.(but not
and all of them are facing north. Therefore in the given seating
necessarily in same order). All of them went to tour on seven
different days starting from Monday to Sunday(of the same arrangement, each
week) member seated in a row faces another member of the other
· A lives an odd numbered floor but not on floor numbered row.(All the information given above does not necessarily
Three. Only two persons live between D and the one who represent the order of seating in the final arrangement.)
owns Toyoto. The one who owns Honda lives immediately · P faces one of the immediate neighbours of the person
above C. The one who owns Toyoto went to tour on one from Jaipur. P does not face A. The person from Delhi sits
of the days before Thursday. second to the right of the person from Bengaluru.
· The one who owns Hyundai lives on odd numbered floors · D faces one of the immediate neighbours of the person
above D. B lives on the floors above E. Only three persons from Patna. S is not from Patna. D is not from Mumbai. R
live between C and the one who owns Hyundai. The sits second to the left of the persons from Surat. A sits
person who went to tour on Friday owns Fiat Car. third to the right of person from Chennai.
· The one who owns Fiat lives immediately above the one · Only One person sits between the person from Ranchi and
who owns Chevrolet. G lives on an odd numbered floor. E Q. The person from Indore sits to the immediate right of
does not own Honda. D went to tour on Sunday Q. C sits to the immediate left of the person who faces Q.
· Only one person lives between B and E. The one who Only two people sit between B and E.
owns Toyoto lives immediately above A. Neither C nor A · The person from Mumbai sits second to the right of the
owns Skoda. The one who went to tour on Wednesday one who faces S. S does not sit at an extreme end of the
immediately after the one who owns Hyundai. line. One of the immediate neighbours of the person from
· The one who owns Honda went to tour immediately before Mumbai faces Ranchi.
the one who lives in floor no 3. There are two persons live 31. Who amongst the following faces the person from
between the floor no 4 and the person who went to tour Hyderabad?
on Wednesday. The one who went to tour on Wednesday (a) The person from Delhi
is not C.
(b) D
26. Which of the following car does A own?
(a) Skoda (b) Toyoto (c) The person from Chennai
(c) Fiat (d) Hyundai (d) The person from Ranchi
(e) Ford (e) B or E
27. Which of the following combinations is True with respect 32. T is from which of the following cities?
to the given arrangement? (a) Patna (b) Indore
(a) Ford - C (b) Hyundai - F (c) Hyderabad (d) Ranchi
(c) Chevrolet - G (d) Fiat - D (e) Mumbai
(e) Ford - B 33. Which of the following is true regarding C?
28. If all the persons are made to sit in alphabetical order from (a) C sits an extreme end of the line
top to bottom, positions of how many persons will remain (b) None of the given options is true
unchanged? (c) C is from Bengaluru
(a) Four (b) None (d) The person from Indore faces C
(c) Two (d) One
(e) The person from Hyderabad is an immediate neighbour
(e) Three
of C

www.newspaperkorner.wordpress.com
www.newspaperkorner.wordpress.com
SBI BANK PO ONLINE PRELIMINARY EXAM 2016 2016-23
34. R is related to Indore in the same way as C is related to 43. A, B and C can alone complete a work in 10, 12 and 15 days
Jaipur based on the given arrangement, To who amongst respectively. A and C started the work and after working for
the following is T related to the following same pattern? 4 days, A left and B joined. In how many days the total work
(a) Delhi (b) Surat was completed?
(c) Patna (d) Hyderabad
5 2
(e) Ranchi (a) 6 days (b) 6 days
35. Who amongst the following sit at extreme end of the row? 9 9
(a) The person from Delhi and R 4
(b) The persons from Bengaluru and A (c) 6 days (d) 5 days
9
(c) A and the person from Patna
(d) The persons from Chennai and Patna 2
(e) 7 days
(e) A, E 9
44. There are 4 filling pipes and 3 emptying pipes capable of
QUANTITIATIVE APTITUDE filling and emptying in 12 minutes and 15 minutes
respectively. If all the pipes are opened together and as a
36. A certain sum of money at certain rate of interest becomes result they fill 10 litres of water per minute. Find the capacity
` 3420 after 2 years and at same rate after two and a half
of the tank.
years becomes ` 3525. Find the rate percent per annum.
(a) 8.5% (b) 8% (a) 65 ltr (b) 70 ltr
(c) 7% (d) 10% (c) 75 ltr (d) 80 ltr
(e) 11% (e) None of these
37. A sum of money becomes ` 35,280 after 2 years and ` 37,044 45. Divide ` 2340 into three parts, such that first part be double
after 3 years when lent on compound interest. Find the that of second part and second part be 1/3 of the third
principal amount. part.Find the Third part amount?
(a) ` 32,000 (b) ` 28,000 (a) ` 780 (b) ` 1170
(c) ` 31,500 (d) ` 32,500 (c) ` 750 (d) ` 390
(e) None of these
(e) None of these
38. A person sell two horses for rupees 480 each. On the first
horse he gains 25 percent and on the second horse he losses DIRECTION (Q.s 46-50) : Each of the following number series,
25 percent. Find the percent gain or loss in the transaction. a wrong number is given. Find out that number.
(a) loss 6.75% (b) gain 6.75%
(c) loss 6.25% (d) gain 6.25% 46. 3 5 13 43 178 891 5353
(e) None of these (a) 43 (b) 178
39. Ashwin has to travel from one point to another point in a (c) 891 (d) 5353
certain time. Travelling at a speed of 6kmph he reaches 40m (e) None of these
late and travelling at a speed of 8kmph he reaches 12 m 47 90720 10080 1440 240 48 10 4
earlier.What is the distance between this two points ? (a) 240 (b) 48
(a) 27 km (b) 18 km
(c) 1440 (d) 10
(c) 5 km (d) 21 km
(e) None of these (e) None of these
40. Two trains A and B start from two places P and Q towards Q 48. 3 5 10 12 17 23 24
and P respectively. After passing each other they take 4 (a) 5 (b) 17
hours 48 minutes and 3 hours and 20 minutes to reach Q (c) 24 (d) 23
and P respectively. If the train from P is moving at 45 km/h (e) None of these
then find the speed of other train. 49. 1, 11, 38, 78, 175, 301
(a) 69 km/h (b) 74 km/h (a) 11 (b) 78
(c) 54 km/h (d) 64 km/h (c) 175 (d) 301
(e) 72 Km/h
(e) None of these
41. Ronit's age is 10 years more than Rohit's age. Also Ronit
was twice old as Rohit 15 years ago. What will be the age of 50 7, 39, 85, 179, 211, 369, 879
Ronit 6 years after? (a) 369 (b) 211
(a) 41 (b) 40 (c) 179 (d) 879
(c) 35 (d) 45 (e) None of these
(e) 38
42. The average age of the group having 3 members is 84. One DIRECTIONS (Q.s 51-55) : In the following questions two
more person joins the group and now the average becomes equations numbered I and II are given. You have to solve both
80. Now a fifth person comes whose age is 3 years more the equations and give answer
than that of fourth person replaces the first person. After 51 (x + 2)(x + 1) = (x – 2)(x – 3)
this the average age of the group becomes 79. What is the
(y + 3)(y + 2) = (y – 1)(y – 2)
weight of the first person?
(a) 75 (b) 65 (a) x > y (b) x < y
(c) 68 (d) 82 (c) x ³ y (d) x £ y
(e) 85 (e) x = y or relation cannot be established

www.newspaperkorner.wordpress.com
www.newspaperkorner.wordpress.com
2016-24 SBI BANK PO ONLINE PRELIMINARY EXAM 2016
52. 12x² + 29x + 14 = 0 The following line graph gives the percentage of literates amount
y² + 9y + 18 = 0 males, foreach of the five villages.
(a) x > y (b) x < y
(c) x ³ y (d) x £ y
(e) x = y or relation cannot be established
53. 5x² – 18x + 9 = 0
3y² + 5y – 2 = 0
(a) x > y (b) x < y
(c) x ³ y (d) x £ y
(e) x = y or relation cannot be established
54. 172 + 144 ÷ 18 = x
262 – 18 × 21 = y
(a) x> y (b) x < y
(c) x ³ y (d) x ³ y
(e) x = y or relation cannot be established
55. 30x2 + 11x + 1 = 0 61. In which village is the number of females as a percentage of
42y2 + 13y + 1 = 0 the total population, the lowest?
(a) x > y (b) x < y (a) A (b) B
(c) x ³ y (d) x £ y (c) C (d) D
(e) x = y or relation cannot be established (e) E
DIRECTION (Q.s 56-60) : What approximate value should 62. What is the ratio of the number of illiterate males in B to that
come in place of the question mark (?) in the following in C?
questions? (You are not expected to calculate the exact value.) (a) 24 : 5 (b) 7 : 16
56. 16.003×29.998 – 40.002×9.009 =? (c) 9 : 13 (d) 11 : 19
(a) 110 (b) 118 (e) None of these
(c) 120 (d) 124 63. What is the average number of literate males per village?
(e) 130
57. 420.009 ÷ 13.998 + 11.997 – 17.0023 =? (a) 32240 (b) 24420
(a) 23 (b) 24 (c) 28120 (d) 29430
(c) 25 (d) 26 (e) None of these
(e) 27 64. In how many villages is the number of illiterate males, as a
58. 41% of 801 – 150.17 = ? – 57% of 910 percentage of the total population, more than 21%?
(a) 693 (b) 694
(c) 697 (d) 707 (a) 1 (b) 2
(e) None of these (c) 3 (d) 4
59. 1235.092 + 4532.998 – ? + 1279.991 = 4000.001 (e) None of these
(a) 2048 (b) 3048 65. In which village is the number of literate males, as a
(c) 3648 (d) 4048 percentage of the total population, the lowest?
(e) None of these
60. (1/2)×(1/17.01)×289×? = 4.001×2.01 (a) A (b) D
(a) 16/17 (b) 14/17 (c) B (d) C
(c) 15/17 (d) 13/17 (e) E
(e) 17/16
DIRECTIONS (Qs. 66-70) : Study the following graph to answer
DIRECTIONS (Qs. 61-65): These questions are based on the the following questions
following bar graph and line graph. The following bar graph
gives the total population of five villages and the number of Total number of students = 90,000
males in each of the five villages. % of Students in 10th in 6 different states

www.newspaperkorner.wordpress.com
www.newspaperkorner.wordpress.com
SBI BANK PO ONLINE PRELIMINARY EXAM 2016 2016-25
Number of Boys student in each district out of 90,000 interest rate sub-sidies for water pumping in the agriculture
sector. While end-users in some rural areas now have access
to solar-powered lanterns or lights and biogas systems for their
home, and agricultural operations are taking advantage of
remarkable capital subsidies for solar energy to use for water
pumping, the use of heavy capital and interest rate subsidies,
and the focus on domestic use rather than on matching
renewable energy technology applications with income
enhancement opportunities have tied the success of these
programmes to government budgets and political cycles
limiting both the breadth and depth of development and
penetration of projects that harness renewable energy
resources.
In order of fully understand the barriers to the
development of renewable energy enterprises/projects in rural
India, we must review a few of the key' characteristics of
investments in re-
66. The no of girls students in District T is what % of the total newable energy projects.
no students in District Q ? First, investments in renewable energy projects are
(a) 47% (b) 31% relatively information and capital-intensive. The greater
(c) 37% (d) 40% information intensity arises primarily from the need for more up
(e) 43% front information regarding the energy resource compared to
conventional power projects. Renewable energy resource
67. What is the average number of boys students in all the
assessments, in most cases, need to be site-specific and
districts together ? preferably with data for a significant period of time. Renewable
(a) 7533 (b) 7355 energy technologies are also more capital-intensive than
(c) 7550 (d) 7300 conventional hydrocar-bon ones because of the large upfront
(e) 7800 investment cost in generation equipment needed to utilize a
68. What is the difference between the number of girls students 'free' or low-cost energy source. Due to the capital intensity,
in District P and the total number of students in District R ? the financial viability of such investments and projects is often
(a) 11000 (b) 9700 more dependent on longer-term financing structures available
(c) 10000 (d) 10100 at the outset of the project.
(e) 10200 Second, individual renewable energy investments are
69. What is the ratio of the no of boys in District U to the total generally smaller than those made in conventional power
generation projects due to constraints on local resource
no of girls in District S ?
availability. Local resource availability is in turn limited by the
(a) 43:76 (b) 50:75 low energy density of renewable energy resources. As
(c) 53:80 (d) 80:53 extensive time and resources are necessary to catalogue the
(e) 54:82 site specific resource availability before financing can be
70 What is the total no of students in District P,R and T ? considered, the technical and managerial know how
(a) 46000 (b) 46800 requirements and associated costs that occur prior to the
(c) 48600 (d) 45000 project often represent a much larger
(e) 45500 percentage of project costs than in the case of
conventional power projects.
Third, entrepreneurs developing small-scale renew able
ENGLISH LANGUAGE energy projects face the same business and financial risks
associated with any enterprise. The viability of any enterprise
DIRECTIONS (Qs. 71-80) : Read the following passage depends on its ability to build a business based on solid
carefully and answer the questions given below it. Certain concepts and competencies in sound contexts that mitigate
words/phrases have been printed in bold to help you locate those risks.
them while answering some of the questions. Considering the aforementioned characteristics, the key
barriers to development of small-scale renewable energy
Renewable Energy Technologies (RETs) have long been projects are fourfold. First, rural entrepreneurs do not usually
recognised for their potential as environment friendly, versatile have the knowledge and expertise necessary to write business
and sustainable energy alternatives for rural areas of India. plans for projects that link renewable energy technologies with
However, despite efforts by the Ministry of Non-conventional productive use of applications, to assess the rewards and risks
Energy Sources (MNES) and the India Renewable Energy of the project and estimate the costs to mitigate the associated
Development Agency (IREDA), RETs have not yet succeeded risks. All of these are necessary inputs before a project is
as a major alternative source of energy in rural India. The presented to a financier for investment. In addition, the nego-
programmes of MNES and IREDA designed to support small- tiations with financiers may be complex, involving
scale distributed systems have relied on heavily subsidized concepts and approaches that are not familiar to the
credit, technology training and consumer awareness activities entrepreneur. Second, the paperwork and soft costs associated
to stimulate the market for end-user finance for renewable with identifying and obtaining access tofinancing for small and
energy systems for domestic use and a tiered set of capital and medium-scale projects is high relative to the financing needs.

www.newspaperkorner.wordpress.com
www.newspaperkorner.wordpress.com
2016-26 SBI BANK PO ONLINE PRELIMINARY EXAM 2016
Without critical first-stage financing, most of the (c) Too many investors willing to invest in these projects
entrepreneurs interested in developing renewable energy thereby creating confusion.
projects will not be able to take the time away from normal (d) The absence of well-established links for networking,
business operations to implement renewable energy projects. marketing and financing.
Third, many of the renewable energy technologies are still (e) These require minimum paperwork thus there are few
relatively new to the market, so the commercial chains, or no records of these projects being generated for
networks, marketing and financial links, and other institutional future reference.
structures that service traditional energy technologies are not
in place to assist the entrepreneureven if she/he has the skills, DIRECTIONS (Qs. 76 - 78): Choose the word/group of words
know-how and capital. Fourth, given the 'newness', there is a which is most similar in meaning to the word/ group of words
limited availability of investment capital to finance the high printed in bold as used in the passage.
upfront costs associated with the initial stages of developing
a renewable energy project. 76. OUTSET
71. Which of the following is/are the reason/a that the (a) Start (b) Periphery
programmes implemented by IREDA and MNES have not (c) Boundary (d) Culmination
yet succeeded in making RETs the major alternative source (e) Stage
of energy? 77. VIABILITY
1. Focus on employment generation. (a) Technicality (b) Rigidity
2. Emphasis on industrial use of renewable energy. (c) Regularity (d) Commodity
3. Interest rate subsidies linking these to Government (e) Feasibility
budgets and political cycles. 78. RECOGNIZED
(a) Only 2 (b) Only 2 and 3 (a) Predictable (b) Standard
(c) Only 3 (d) Only 1 and 2 (c) Known (d) Resembled
(e) All 1, 2 and 3 (e) Questioned
72. Which of the following is possibly the most appropriate
title for the passage? DIRECTIONS (Q.s 79 - 80) : Choose the word/group of
(a) Programmes to Implement Renewable Energy Projects words which is most opposite in meaning to the word/ group
in Rural Areas. of words printed in bold as used in the passage.
(b) Rural Renewable Energy Projects - The Barrie's 79. MITIGATE
(c) The Benefits of Alternative Sources of Energy (a) Aggravate (b) Anger
(d) The Government and Renewable Energy Projects (c) Moderate (d) Alienate
(e) Taking Forward Renewable Energy Projects in India (e) Alleviate
73. Which of the following is/are the characteristic/a of 80. COMPLEX
investment in RETs? (a) Plain (b) Simple
1. There are a number of investors willing to invest (c) Undisturbed (d) Compound
readily invest in these projects.
2. These demand more investment of capital over the (e) Unilateral
conventional energy projects. DIRECTIONS (Qs. 81-85) : Read each sentence to find out
3. These involve lesser risk as compared to the whether there is any grammatical mistakejerror in it. The
conventional energy projects. error if any, will be in one part of the sentence. Mark the
(a) Only 1 and 3 (b) Only 1 and 2 number of the part with error as your answer. If there is no
(c) Only 2 (d) Only 2 and 3 error, mark (E).
(e) All 1, 2 and 3
74. Why are renewable energy projects termed information- 81. To run a company effectively (A)/ it is very important (B)/
intensive? in knowing the strengths and weaknesses (C)/ of the
(a) The information on resources required by these employees. (D)/ No error (E)
projects is specific in terms of the location as well as (a) A (b) B
the time period. (c) C (d) D
(b) Very little information about renewable energy is (e) E
available for the implementation of these projects. 82. The land records (A)/ of this district (B)/ will computerise
(c) Information needed by these projects is mostly (C)/ by next year. (D)/ No error (E)
confidential and requires a lot of paperwork in order (a) A (b) B
to gain access. (c) C (d) D
(d) The information required by these projects is far (e) E
greater than that required by any other project related 83. The Head Office has (A)/ issued instructions that (B)/ the
to energy conservation. performance of all Zonal Managers (C)/ have to assess by
(e) The information required by these projects is very a committee. (D)/ No error (E)
vast and sometimes unrelated to the project at hand. (a) A (b) B
75. What, according to the passage, is a drawback of (c) C (d) D
renewable energy projects 'newness'? (e) E
(a) The unfamiliarity with the concept of renew-able 84. She has promised to (A)/ donate the funds to (B)/establish
energy amongst the locals. a library in many (C)/ villages in India. (D)/ No error (E)
(b) The low risk factor involved in its implementation (a) A (b) B
attracts entrepreneurs who fail to complete these (c) C (d) D
projects in time. (e) E
www.newspaperkorner.wordpress.com
www.newspaperkorner.wordpress.com
SBI BANK PO ONLINE PRELIMINARY EXAM 2016 2016-27
85. We have already (A)/ submitted our application (B)/ and 92. The criterion for ..... a player should be his recent
expect to receive (C)/ our licence in thirty days.(D)/ No performance, but unfortunately, the journalists are ..... to
error (E) be carried away by earlier successes.
(a) A (b) B (a) condemning .... satisfying
(c) C (d) D (b) judging ..... prone
(e) E (c) revealing ..... reluctant
(d) eager .... acclaiming
DIRECTIONS (Q.s 86-90) : Rearrange the following five (e) criticising .... clean
sentences 1, 2, 3, 4, and 5 in the proper sequence to form a 93. For the last half century, he ..... himself to public
meaningful paragraph then answer the questions given below affairs ........ taking a holiday.
them. (a) by ......... committed
1. What teachers repeatedly commented on was that she was (b) after ....... offered
"very creative' and had "an unusual way of thinking", etc. (c) devoted ......... without
2. A study done by the institute focuses on Roma, an (d) sacrified ......... after
individual with creative abilities. (e) prepared ......... before
3. But nowhere were Roma's these abilities recognised on her 94. You will see signs of ..... everywhere, which speak well for
report cards, because there was no check-off box for the .... of these people.
artwork and creativity. (a) decoration .... senses
4. Some of her artwork were selected to represent her school (b) clear .... debris
at art exhibition at state the level art exhibition. (c) beauty ....... careful
5. As a child, she had been a good student above average (d) industry .......... prosperity
in most areas but not an exceptional early reader and (e) repairs ........... extravaganza
writer. 95. The police arrested Ramesh on a .... of theft but for lack of
86. Which of the following should be the THIRD sentence evidence .... him.
after rearrangement? (a) crime .... imprisoned
(a) 1 (b) 2 (b) punished .... complaint
(c) 3 (d) 4 (c) left ...... condition
(e) 5 (d) tip ......... absconding
87. Which of the following should be the FOURTH sentence (e) charge .... released
after rearrangement?
(a) 1 (b) 2 DIRECTIONS (Q.s 96 - 100) : In the following passage there
(c) 3 (d) 4 are blanks, each of which has been numbered. These numbers
(e) 5 are printed below the passage and against each five words/
phrases are suggested, one of which fits the blank
88. Which of the following should be the SECOND sentence
after rearrangement? appropriately. Find out the appropriate word/phrase in each
case.
(a) 1 (b) 2
(c) 3 (d) 4 Twenty years from now, nearly 60% of the world's population
will live in urban areas. The impact of urbanization might not
(e) 5
all be positive on India as urban expansion is happening at a
89. Which of the following should be the FIRST sentence
much faster rate than infrastructure expansion.
after rearrangement?
Sustainability issues need to be (96) so that eco nomic
(a) 1 (b) 2 development is not at the (97) of public health. Some urban
(c) 3 (d) 4 services that ought to be in (98) in a city like water, electricity,
(e) 5 transport etc. need special consideration.
90. Which of the following should be the FIFTH sentence TERI has put together a detailed report that (99) sustainability
after rearrangement? in the provision of basic urban services in Indian cities.
(a) 1 (b) 2 Inadequate public transport is a major reason for the
(c) 3 (d) 4 proliferation of private vehicles on the road. Respiratory illness
(e) 5 in children living in urban areas is on the rise with more cases
of Asthma being (100) because of pollution. The future of
DIRECTIONS (Q.s 91 - 95) : In each of the following cities of Indian dreams depends on whether we can build better
sentence there are two blank spaces. Below each sentence cities today.
there are five pair of words denoted by letters a, b, c, d, and 96. (a) Speculated (b) Believed (c) Imagined
e. Find out which pair of words can be filled up in the blanks
in the sentence in the same sequence to make the sentence (d) Considered (e) Understand
meaningfully complete. 97. (a) Payment (b) Rate (c) Costs
91. He objected to the proposal because it was founded on a (d) Charge (e) Expense
.... principle arid also was .... at time. 98. (a) Abundance (b) Large (c) Functional
(a) faulty ...... desirable (d) Vicinity (e) Location
(b) imperative .... reasonable 99. (a) Bring (b) Emphasizes (c) Speculates
(c) wrong ...... inconvenient (d) Postulates (e) Requests
(d) sound ..... acceptable
100. (a) Produced (b) Develop (c) Composed
(e) unconforming .... deplorable
(d) Resulted (e) Reported

www.newspaperkorner.wordpress.com
www.newspaperkorner.wordpress.com
2016-28 SBI BANK PO ONLINE PRELIMINARY EXAM 2016

Answers &
Explanations
Solution: 1-5:
17. (c) P R U N
P T R Q U V W S
W F
Mar Jun May Jan Jul Apr Feb Aug S T Q
S
1. (d) 2. (a) 3. (e) 4. (c) 5. (a)
Sol. 6-10 Hence, URP flat combination get south facing flats.
18. (a) A<B>C=D>=E (clearly B is greater than E and no
Q, English
relation between A and E)
S, Physics 19. (e) A>B=C<=D=E (clearly A is greater than C and E is
R, Geography
greater than or equal to B)
20. (d) A=B>=C<=D<E (clearly E is not equal to A and C is
U, Chemistry T, Biology not greater than A)
21. (b) A>=B>=C=D<E ( no relation can be made between
A and E and clearly E is greater than C)
V, History P, Economics 22. (a) A>E<C>B=D (clearly C >D and No relation between
B and A can be made)
W, Statistics 23. (a) 24. (c) 25. (e)
6. (a) Q Sol. 26-30
7. (c) Three
8. (e) V sits opposite to R, who likes Geography Floor No Person Car Days
9. (a) The person who likes Statistics
7 G Skoda Wednesday
10. (c) Q, R
Sol. 11-15 6 B Toyoto Monday
5 A Hyundai Tuesday
Members S tates Games 4 E Fiat Friday
Sania M adh ya Prad es h Badminto n 3 D Chevrolet Sunday
Kamal Bih ar Table tennis 2 F Honda Saturday
Pankaj Odis h a Billiard s 1 C Ford Thursday
A nan d Kerala Ches s
26. (d) 27. (a) 28. (c) 29. (a) 30. (e)
A rju n Hary ana Golf
Sol. 31-35
Shewag M aharas h tra Cricket
Sreejes h Uttar Prad es h Ho ckey ur bai hi ai lore
Jaip Mum Del Chenn Banga
11. (a) 12. (c) 13. (d) 14. (c) 15. (d) A B/E D C B/E
16. (c) R P Q S T
P R U N
Rachi Patna Surat Indore Hyderabad
W F
S T Q 31. (e) 32. (c) 33. (d) 34. (c) 35. (b)
S 36. (c) Amount after 2.5 yrs = 3525, after 2 yrs = 3420
Interchanging flats P and T So SI for half yr = 3525 – 3420 = 105, so for 1 yr SI = 105
× 2 = 210
T R U P + 2 × SI = 3420
So P = 3420 – 2 × 210 = 3000
So 3000 × r × 2/100 = 420
37. (a) Divide both equations, [1 + r/100] = 37044/35280 = 21/20
S P Q So P[21/20]2 = 35280
Hence flat R will be next to U.

www.newspaperkorner.wordpress.com
www.newspaperkorner.wordpress.com
SBI BANK PO ONLINE PRELIMINARY EXAM 2016 2016-29
38. (c) When same quantity is sell at same price and percent 45. (b) First: Second: Third = 2:1:3
gain and loss is same then there is always loss 2340
occurred. To calculate the loss percent = Third part = 3 ´ = 1170
6
(Common loss or gain) 2 46. (b) 3 × 1 + 2 = 5
10 5 × 2 + 3 = 13
2 13 × 3 + 4 = 43…..
æ 25 ö
i.e. ç ÷ = 6.25% loss Correct Number : 177
è 10 ø 47. (d) 4 ×3 = 12
d 40 d 12 12 × 4 = 48
39. (d) – = + 48 × 5 = 240…..
6 60 8 60
Correct Number: 12
d d 12 40 48. (e) First series: 3, 10, 17, 24 (increased by 7)
– = –
6 8 60 60 Second Series: 5, 14, 23 (increased by 9)
2d/48 = 52/60 Wrong Number: 12
Correct Number: 14
52 ´ 40 49. (b) 1 + 32 + 1 = 11
D= = 20.8 = 21km
60 ´ 2 11 + 52 + 2 = 38
40. (c) Speed of 2nd train = Speed of first train × 38 + 72 + 3 = 90
90 + 92 + 4 = 175
( Time taken by 1st train after meeting 175 + 112 + 5 = 301
50. (d) 7 × 2 + 3 = 17
/ Time taken by 2nd train after meeting ) 17 × 2 + 5 = 39
41. (a) Rohit = x, Ronit = x+10 39 × 2 + 7 = 85
(x + 10) – 15 = 2 (x – 15) 85 × 2 + 9 = 179
Solve, x = 25 179 × 2 + 11 = 369
Ronit's 6 years after = x + 10 + 6 369 × 2 + 13 = 751
42. (a) Let the ages of these are A, B, C, D, E 51. (a) (x + 2) (x + 1) = (x – 2)(x – 3)
So A + B + C = 84 × 3 = 252 1
And A + B + C + D = 80 × 4 = 320 x= = 0.5
2
So D = 320 – 252 = 68, so E = 68 + 3 = 71 (y + 3)(y + 2) = (y – 1)(y – 2)
Now B + C + D + E = 79 × 4 = 316
(A + B + C + D) – (B + C + D + E) = 320 – 316 1
y= – = –0.5
So A – E = 4, so A = 71+ 4 2
52. (a) 12x² + 29x + 14 = 0
1 1 1
43. (b) (A + C) = + = . They worked for 4 days so did x = – 1.75, – 0.6
10 15 6 y² + 9y + 18 = 0
1 2 y = –6, –3
´ 4 = = 2/3rd of work
6 3 53. (a) 5x² – 18x + 9 = 0
2 1 x = 0.6, 3
Remaining work = 1 – = 3y² + 5y – 2 = 0
3 3
y = 0.33, –2
Now A left, B and C working
54. (b) 172 + 144 ÷ 18 = x
æ 1 1ö 9 3 x = 297
(B + C) = ç + ÷ = = . They worked for x 262 – 18 × 21 = y
è 12 15 ø 60 20
y = 676 – 378 = 29
3 55. (d) 30x2 + 11x + 1 = 0
days and completed 1/3rd of work so × x = 1/3, so
20 30x2 + 6x + 5x + 1 = 0
x = 20/9 days x = – 0.16, –0.19
20 42y2 + 13y + 1 = 0
Total = 4 + 42y2 + 6y + 7y + 1 = 0
9
y = –0.14, –0.16
æ 4 3ö Put on number line
44. (c) ç 12 – 15 ÷ ´ t = 1
è ø –0.19, –0.16, –0.16, –0.14
15 56. (c) 16×30 – 40×9 = 480 – 360 = 120
t= minute – in this time the tank will be filled. So 57. (c) 420/14 + 12 – 17 = 25
2
15 41 57
the capacity = ×10 = 75 litre 58. (c) ´ 800 – 150 = x – ´ 910
2 100 100

www.newspaperkorner.wordpress.com
www.newspaperkorner.wordpress.com
2016-30 SBI BANK PO ONLINE PRELIMINARY EXAM 2016
59. (b) ? = 7048 – 4000 = 3048
æ 90000 ö
60. (a) ? = (4×2×2)/17 = 16/17 (b) No of girls in T = ç 12 ´ – 7200 = 10800 – 7200
100 ÷ø
66.
61. (b) Percentage of females as a percentage of total è
population in = 3600
Total no of students in Q = 13×90000/100 = 11700
24000
A= ´ 100 = 37.5% 100
64000 % = 3600 ´ = 30.77% = 31%
1700
24000 1
B= ´ 100 = 33 % 6500 + 8900 + 5500 + 9100 + 7200 + 8000
72000 3 67. (a)
6
24000 4
C= ´ 100 = 44 % 45200
54000 9 =
6
30000 68. (d) Total no. of students in R
D= ´ 100 = 37.5%
80000
22 ´ 90000
= = 19800
36000 100
E= ´ 100 = 60%
60000
(18 ´ 90000)
62. (a) No. of girls in P = – 6500 = 9700
100
63. (c) The required average =
Difference = 19800 – 9700 = 10100
æ 80 70 ö 69. (d) Boys in U = 8000
40000 ´ + 48000 ´ + 30000
ç 100 100 ÷ Girls in S = (16×90000/100) – 9100 = 5300
ç ÷ /5 80:53
ç ´ 90 + 50000 ´ 60 + 24000 ´ 75 ÷
è 100 100 100 ø 90000
70. (b) P = 18 × = 16200
= 140600 / 5 = 28120 100
64. (a) Percentage of illiterate males in the given villages is as 90000
follows. R = 22× = 19800
100
8000 90000
A= ´ 100 = 12.5% T = 12× = 10800
64000 100
14400 Total = 46,800
B= ´ 100 = 20%
72000 71 (c) 72 (a) 73 (c) 74 (d) 75 (d)
76. (d) Outset means the start or beginning of something.
3000 77. (c) Viability means the state or degree of being easily or
C= ´ 100 = 5.55%
54000 conveniently done.
20000 78. (c) Recognized means acknowledge the existence.
D= ´ 100 = 25% 79. (a) Mitigate means to make less painful. Aggravate means
80000
to make worse.
20000 80. (b) Complex means not easy to analyse or understand.
D= ´ 100 = 25%
80000 Simple means easy to understand, presenting no
65. (e) % of litreate males in difficulty.
81 (c) 82 (e) 83 (d) 84 (c) 85 (d)
32000
A= ´ 100 = 50% 86 (e) 87 (a) 89 (b) 90 (c) 91 (c)
64000
92 (b) 93 (c) 94 (d) 95 (e)
33600 2 96. (d) Consider means to think carefully about something.
B= ´ 100 = 46 %
72000 3 97. (e) Expense means the cost required for something.
98. (d) Vicinity means the area near a particular place.
27000
C= ´ 100 = 50% 99. (b) Emphasize means to give special importance to
54000 something.
30000 100. (e) Reported means account of something that one has
D= ´ 100 = 37.5% observed.
80000
18000
E= ´ 100 = 30%
60000
\ It is the lowest in village E.

www.newspaperkorner.wordpress.com
www.newspaperkorner.wordpress.com

SBI Bank PO Main Exam 2015


Held On : 26-07-2015

Max. Marks : 200 (Based on Memory) Time : 2 hrs.

8. Statements
REASONING ABILITY
S³ M < Y = Z > F > T
Conclusions
DIRECTIONS (Qs. 1-3): Study the following information to
I. S > F II. Y> T
answer the given questions.
In a certain code ‘colours of the sky’ is written as ‘ki la fa so’, DIRECTIONS (Qs. 9-13): Study the following information
‘rainbow colours’ is written as ‘ro ki’ and ‘sky high rocket’ is carefully and answer the given questions.
written as ‘la pe jo’ and ‘the rocket world’ is written as ‘pe so ne’. (i) A, B, C, D, E, F, G and H are sitting around a circle facing
1. Which of the following is the code for ‘colours sky high’? the centre but not necessarily in the same order.
(a) ro jo la (b) fa la jo (ii) B sits second to left of H’s husband. No female is an
(c) la ki jo (d) ki jo la
immediate neighbour of B.
(e) fa ki jo
2. Which of the following will/may represent ‘the’? (iii) D’s daughter sits second to right of F, F is the sister of G. F
(a) Only fa (b) Only la is not an immediate neightbour of H’s husband.
(c) Only so (d) Either (a) or (b) (iv) Only one person sits between A and F. A is the father of G.
(e) Either (a) or (c) H’s brother D sits to the immediate left of H’s mother. Only
3. What does ‘pe’ represent in the code? one person sits between H’s mother and E.
(a) colours (b) sky (v) Only one person sits between H and G. G is the mother of
(c) high (d) rainbow C. G is not an immediate neighbour of E.
(e) rocket 9. What is position of A with respect of his mother-in-law?
(a) Immediate left (b) Third to the right
DIRECTIONS (Qs. 4-8): In these questions, relationship
between different elements is shown in the statements. These (c) Third to the left (d) Second to the right
statements are followed by two conclusions. (e) Fourth to the left
10. What is the position of A with respect to his mother-in-
Give answer grandchild?
(1) If only conclusion I follows (a) Immediate Right (b) Third to the right
(2) If only conclusion II follows (c) Third to the left (d) Second to the right
(3) If either conclusion I nor II follows (e) Fourth to the left
(4) If neither conclusion I nor II follows 11. Four of the following five are alike in a certain way based
(5) If both conclusion I and II follows on the given information and so form a group. Which is
4. Statements the one that does not belong to that group?
W³D < M < P < A = F (a) F (b) C
Conclusions (c) E (d) H
I. F > D II. P < W (e) G
5. Statements 12. Which of the following is true with respect to the given
H³ M > F < A = B > S seating arrangement?
Conclusions (a) C is the cousin of E
I. H > B II. F < S (b) H and H’s husband are immediate neighbours of each
6. Statements other
B>T>Q>R=F (c) No female is an immediate neighbour of C
Conclusions (d) H sits third to left of her daughter
I. Q ³ F II. T > F (e) B is the mother of H
7. Statements 13. Who sits to the immediate left of C?
S = R ³ Q, P < Q (a) F’s grandmother (b) G’s son
Conclusions (c) D’s mother-in-law (d) A
I. S ³ P II. R > P (e) G

www.newspaperkorner.wordpress.com
www.newspaperkorner.wordpress.com

M-2 SBI BANK PO MAIN EXAM 2015

DIRECTIONS (Qs. 14-18): In each question given four Input : gate 20 86 just not 71 for 67 38 bake sun 55
statements are followed by three Conclusions I, II and III. You Step I : bake gate 20 just not 71 for 67 38 sun 55 86
have to take the four given statements to be true even if they Step II : for bake gate 20 just not 67 38 sun 55 86 71
seem to be tat variance from the commonly known facts. Read Step III : gate for bake 20 just not 38 sun 55 86 71 67
the conclusions and decide which logically follows from the four Step IV : just gate for bake 20 not 38 sun 55 86 71 67
given statements disregarding commonly known facts. Step V : not just gate for bake 20 sun 86 71 67 55 38
Step VI : sun not just gate for bake 86 71 67 55 38 20
Qs. No. 14-15 Steps VI is the last step of the arrangement the above input.
Statements All footballers are music lover.
As per the rule followed in the above steps, find out in each of
All footballers are dancer.
the following questions the appropriate step for the given input.
No dancer is cricketer.
No cricketer is player. Input: 31 rise gem 15 92 47 aim big 25 does 56 not 85 63 with
14. Conclusions I. Dancer can be player being a possibility moon
II. All those dancer are footballers are also 19. How many steps will be required to complete the
music lovers. rearrangement?
III. Any cricketer can never be player. (a) Eight (b) Six
(a) None follow (b) All follow (c) Seven (d) Five
(c) Only I follows (d) Both II and III follow (e) None of these
(e) None of these 20. Which words numbers would be at 7th position from the
15. Conclusions I. All cricketer can be music lover. left in step IV?
II. Some player are footballers. (a) rise (b) aim
III. At least some footballers are cricketer. (c) big (d) 15
(a) Only I follows (b) I and III follow
(e) 47
(c) II and III follow (d) None follows
21. Which step number is the following output?
(e) None of these
Qs. No. 16-18 rise not moon gem does big aim 15 with 92 85 63 56 47 31 25
Statements All boys are intelligent. (a) Step V (b) Step VII
Very few girls are intelligent. (c) Step IV (d) Step VIII
None girl is leader. (e) There is no such step
Some professor are leader as 22. Which of the following represents the position of ‘92’ in
well as boys. step VI ?
16. Conclusions I. 36% of girls are intelligent. (a) Ninth from the left (b) Fifth from the right
II. 2% girls are intelligent. (c) Sixth from the right (d) Ninth from the right
III. Some boys can never be a girl. (e) Seventh from the left
(a) Only II follows (b) Only III follows
23. Which words numbers would be at 5th position from the
(c) Both I and II follow (d) None follows
right in the last step?
(e) All follow
17. Conclusions I. All leaders are professors. (a) gem (b) 63
II. There is a possibility that every girl can (c) 56 (d) 85
be professor. (e) does
III. All professor can be leaders. DIRECTIONS (Qs. 24-28) : Each of the questions below
(a) None follows consists of a question and two statements numbered I and II
(b) Only II follows given below it. You have to decide whether the data provided in
(c) Both II and III follows
the statements are sufficient to answer the question. Read both
(d) Only III follows
the statements and and answer the questions.
(e) None of these
18. Conclusions I. Atleast some professors are girls Give answer (a) if the data in statement I alone is sufficient to
II. No professors is intelligent. answer the question, while the data in statement II alone are not
III. 5% professors are leader. sufficient to answer the question.
(a) Only I follows (b) Only II follows Give answer (b) if the data in statement II alone are sufficient to
(c) Only III follows (d) Both II and III follows answer the question, while the data in statement I alone are not
(e) None of these sufficient to answer the question.
DIRECTIONS (Qs. 19-23): Study the following information Give answer (c) if the data either in statement I alone or in
carefully and answer the given question: statement II alone are sufficient to answer the question; and
Give answer (d) if the data given in both the statements I and II
A word and number arrangement machine when given an input
together are not sufficient to answer the question; and
line of words and numbers rearrange them following a particular
Give answer (e) if the data in both the statements I and II together
rule in each step. The following is an illustration of input and
are necessary to answer the question.
rearrangement. (All the numbers are two-digit numbers).

www.newspaperkorner.wordpress.com
www.newspaperkorner.wordpress.com

SBI BANK PO MAIN EXAM 2015 M-3

24. How is J related to K? (c) Shorter people suffer from inferiority complex
I. J’s father P is the brother of N. N is K’s wife. throughout them life
II. J is the son of P. P is the brother of N. N is K’s wife. (d) Taller people are found to be more positive in
25. On which floor of the building does G stay? (the building approach to daily chores of lifes.
has five floor 1, 2, 3, 4, 5.) (e) None of the above
I. Only the even-numbered floors are occupied and G 31. Statement
does not stay on the second floor. Many patients suffering form malaria were administered
II. G does not stay on an odd-numbered floor. anti-malarial drug for a week. Some of them did not respond
to the traditional drug and their condition deteriorated after
26. How many days did Raju take to complete his assignment? four days.
I. Mohit correctly remembers that Raju took more than Which of the following would weaken the findings
3 days less than 9 days to complete his assignment. mentioned in the above statement?
II. Mina correctly remembers that Raju took more than (a) Those patients who responded to the traditional drugs
7 days less than 11 days to complete his assignment. and recovered were needed to be given additional does
27. How is the word ‘GATES’ coded in the code language? as they reported relapse of symptoms
I. ‘BRICK’ is coded as ‘LDJSC’ and ‘PIN’ is coded as (b) The mosquitoes carrying malaria are found to be
‘OJQ’ resistant to traditional malarial drugs
(c) Majority of the patients suffering from malaria
II. ‘WATER’ is coded as ‘SFUBX’ and ‘DISH’ is coded responded well to the traditional malarial drugs and
as ‘ITJE’ recovered from the illness
28. Among A, B, C and D, which school has the highest number (d) Many drug companies have stopped manufacturing
of students. traditional malarial drugs
I. School A has fewer students than school D. (e) None of the above
II. School C has fewer students than school D. 32. Statement
29. In Swartkans territory, archaeologists discovered charred The cost of manufacturing cars in state A is 30% less than
bone fragments dating back one million years. Analysis of the cost of manufacturing cars in state B. Even after
transportation fee for the differential distances of states A
the fragments, which came from a variety of animals,
and B and the interstate taxes, it is cheaper to manufacture
showed that they had been heated to temperatures no higher
cars in state B than state A for selling these cars in state C.
than those produced in experimental campfires made from
Which of the following supports the conclusion drawn in
branches of white, stinkwood, the most common tree around the above statement?
Swartkans. (a) The cost of transportation from state A to state C is
Which of the following, if true, would, together with the more than 30% of the production cost
information above, provide the best basis for the claim that (b) The production costs of cars in state B are lower in
the charred bone fragments are evidence of the use of fire comparison to state A
by early hominids? (c) Only Entry Tax at state C is more for the products
(a) The white stinkwood tree is used for building material originated in state A
by the present day inhabitants of Swartkans (d) Entry Tax at state C is more for the products originated
(b) Forest fires can heat wood to a range of temperatures in state B
(e) The total of transportation cost of cars from state B
that occur in campfires
to state C and Entry Tax of cars at state C is less than
(c) The bone fragments were fitted together by the 30% of the production cost of cars in state B
archaeologists to form the complete skeletons of 33. Statement
several animals. There was a slow decline in number of patients with flu
(d) The bone fragments were found in several distinct like symptoms visiting various health facilities in the city
layers of limestone that contained primitive cutting during the last fortnight.
tools known to have been used by early hominids Which of the following substantiates the facts mentioned
(e) None of the above in the above statement?
30. Statement (a) Majority of the people suffering from flu visit the
health facilities in the city
It has been reported in a recent research report that taller
(b) There has been a continuous increase in the sale of
people are found to be happier than those who are relatively medicines for curing flu in the city limits
shorter. (c) People have started visiting the crowed places like
Which of the following contradicts the above finding? malls and cinema balls during the last fortnight after
(a) People are generally truthful when they respond to cautioned gap of one month.
the questions of such research studies (d) There is a sudden increase in the number of deaths
(b) Those people who have a comfortable lifestyle are caused by flu like symptoms followed by respiratory
found to be happier than those who lead a strenuous complications.
lifestyle irrespective of their height (e) None of the above

www.newspaperkorner.wordpress.com
www.newspaperkorner.wordpress.com

M-4 SBI BANK PO MAIN EXAM 2015

34. Statement 40. The Chess player is a:


It is reported that during the last fortnight there has been (a) Power engineer (b) Mechanical engineer
three cases of near miss situation involving two aircrafts (c) Design engineer (d) Quality inspector
over the busy city airport. In all these cases both the aircrafts (e) None of these
came perilously close to each other and could avoid collision
as the pilots acted just in time. DIRECTIONS (Qs. 41-45): Study the following information
Courses of action carefully and answer the given questions.
I. The pilots of all the six aircrafts involved in these Eight Childhood friends A, B, C, D, E, F, G and H are sitting
incidents should immediately be derostered. around a circular table facing the centre, but not necessarily in
II. Some flights should be diverted to other airports for
the same order. Each of them have completed their respected
the next few months to decongest the sky over the
city airport. courses viz MBA, MCA, B.Tech, CA, CS, B.Com, Finance and
III. Air traffic controllers of the city airport should be M.Tech.
sent for refresher courses in batches to prepare • There are two persons between those who study MCA and
themselves to handle such pressure situation. MBA and neither of them sit opposite A.
(a) Only 1 follows • C and G are immediate neighbours and neither of them
(b) Only II follows
(c) Only III follows studies M.Tech or MBA.
(d) II and III follows • A, who studies Finance, sits second to the right of one who
(e) None of these studies M.Tech.
DIRECTIONS (Qs. 35-40) : Read the following information • B and E neither study M.Tech or nor sit adjacent to A.
and answer the questions that follow : • E studies MBA and sits second to the left of G, who studies
In a Public Sector Undertaking Township, there are five executives CA.
– Ambrish, Amit, Rohit, Manu and Tarun. Two of them play • C does not study MCA.
Cricket while the other three play different games viz. Football,
Tennis and Chess. One Cricket player and a Chess player stay in • The immediate neighbour of E faces C, who sits third to the
the third flat, whereas the other three stay in different flats, i.e. left of the one who studies Finance.
2nd, 4th and 5th. Two of these five players are mechanical engineers • F does not study M.Tech.
while the other three are quality inspector, design engineer and • Both D and H do not study MCA and B.Tech.
power engineer respectively. The chess player is the oldest in
age while one of the cricket players, who plays at the national • B studies B.Com while D studies CS.
level, is the youngest in age. The age of the other cricket player, • B is not an immediate neighbour of A.
who plays at the regional level, lies between the football player 41. MCA is related to B.Com in a certain way and MBA is
and the chess player. Manu is a regional level player and stays in
related to M.Tech in the same way. Which of the following
the 3rd flat while Tarun is a quality inspector and stays in the
is related to B.Tech following the same pattern?
5th flat. The football player is a design engineer and stays in the
2nd Flat. Amit is a power engineer and plays Chess while Ambrish (a) CA (b) Finance
is the mechanical engineer and plays Cricket at the national level. (c) CS (d) can’t be determined
35. Who stays in the 4th flat? (e) none of these
(a) Ambrish (b) Amit 42. Who among the following studies MCA?
(c) Rohit (d) Manu (a) F (b) H
(e) None of these (c) E (d) can’t be determined
36. What does Tarun play? (e) none of these
(a) Chess (b) Football 43. Who among the following sits third to the right of F?
(c) Cricket (d) Tennis
(a) E (b) G
(e) None of these
37. Who plays football? (c) C (d) B
(a) Ambrish (b) Amit (e) none of these
(c) Rohit (d) Manu 44. Which of the following statements is/are true about E?
(e) None of these (a) E is sitting opposite H.
38. Agewise, who among the following lies between Manu and (b) E studies M.Tech
Tarun? (c) E is not an immediate neighbour of B
(a) Quality inspector (b) Mechanical engineer (d) All are true
(c) Power engineer (d) Design engineer
(e) none of these
(e) None of these
45. Which of the following courses is studied by C?
39. Who stay in the same flat?
(a) Ambrish and Amit (b) Maim and Tarun (a) CA (b) B.Tech
(c) Amit and Manu (d) Rohit and Tarun (c) CS (d) MBA
(e) None of these (e) None of these

www.newspaperkorner.wordpress.com
www.newspaperkorner.wordpress.com

SBI BANK PO MAIN EXAM 2015 M-5

DIRECTIONS (Qs. 46–49): Read the following information (a) 16707 (b) 16686
carefully to answer the question: (c) 16531 (d) 16668
(e) None of these
P X Q means “P is sister of Q”
52. The total number of women liking RR forms approximately
P ÷ Q means “ P is mother of Q”
P + Q means “ P is brother of Q” what per cent of the total number of women liking all the
P – Q means “ P is father of Q” teams?
46. Which of the following represent W is grandfather of H? (a) 11 (b) 15
(a) W + T - H (b) W ÷ T – H (c) 20 (d) 20
(c) W × T + H (d) W ÷ T + H (e) None of these
(e) None of these 53. What is the total number of children liking CSK?
47. Which of the following represent “M is nephew of R”? (a) 14085 (b) 16389
(a) M ÷ T – R (b) R ÷ T – M (c) 20031 (d) 14850
(c) R × T ÷ M × J (d) R ÷ T – M + J (e) None of these
(e) None of these 54. What is the ratio of total number of men liking DD to those
48. How T is related to S “W ÷ T – H + V – S”? liking RR?
(a) sister (b) mother
(c) aunt (d) uncle (a) 69 : 49 (b) 7 : 5
(e) None of these (c) 208 : 147 (d) 70 : 52
49. The expression means “S ÷ T – H × V – N”? (e) None of these
(a) S is grandmother of N 55. The number of men liking DC forms forms what per cent
(b) S is great grandmother of N of those liking RCB? (rounded off to two digits after
(c) S is mother of V decimal).
(d) N is grand son of S (a) 21.05% (b) 475.04%
(e) None of these (c) 25.56% (d) 25%
50. Statement: Exporters in the capital are alleging that
(e) None of these
commercial banks are violating a Reserve Bank of India
directive to operate a post shipment export credit DIRECTIONS (Qs. 56-60) : Study the following table carefully
denominated in foreign currency at international rates from and answers the questions given below it.
January this year.
Courses of Action: Statistics of the Quantity produced and quantity Sold by
I. The officers concerned in the commercial banks are Aishwarya Co Limited for the Five Months. The Stock at the
to be suspended. Beginning of June is 75 units.
II. The RBI should be asked to stop giving such directives Months Quantify produced Quantity sold
to commercial banks. June 300 350
(a) Only I follows (b) Only II follows July 250 275
(c) Either I or II follows (d) Neither I nor II follows August 200 175
(e) Both I and II follow September 200 200
October 325 300
DATA ANALYSIS AND INTERPRETATION 56. During which month, the company had the least stock to
began with?
DIRECTIONS (Qs. 51-55) : Study the table carefully to answer (a) June (b) July
the questions that follow. (c) August (d) October
Number of people Liking Eight Eifferent Teams in IPL-5 and (e) None of these
the percentages of Men, Women and Children Liking these 57. What was the stock at the beginning of September?
Tearms (a) 35 (b) 15
(c) 25 (d) 20
Teams Total number Percentage of
(e) None of these
of people Men Women Children
58. What should be the minimum stock at the beginnings of
CSK 45525 20 44 36 June, if the company has to supply smoothly for all the five
DD 36800 39 33 28 months?
DC 56340 45 30 25 (a) 45 units (b) 35 units
MI 62350 38 28 34 (c) 25 units (d) 30 units
RR 48300 21 44 35 (e) None of these
RCB 35580 15 35 50 59. During which month did the company have the highest
KXI 56250 24 36 40 sales?
KKR 64000 16 54 30 (a) June (b) July
51. What is the approximate average number of women liking (c) August (d) October
all the teams? (e) None of these

www.newspaperkorner.wordpress.com
www.newspaperkorner.wordpress.com

M-6 SBI BANK PO MAIN EXAM 2015

60. What is the stock at the beginning of July? Unit Price


(a) 50 (b) 25
(c) 100 (d) 75 16
14 14 15
14
(e) None of these 12
12 11
DIRECTIONS (Qs. 61-65) : The circle graph given here shows 12
10 10
the spending for a country on various sports during a particular 10 10 10
8
year. Study the graph carefully and answer the questions given 8
6
below it. 4
2
0

2004

2005

2006

2007

2008

2009

2010

2012

2013

2014
2011
Cricket Hockey
81° 63°
Football Revenue — Profit
Basket Ball 54°
50° 1600
1400
Others 1400
Tennis 1200 1100
31° 1200
45° Golf
1000 900
36° 800
800 900 700
600 700
400
400 300 400 200
61. What per cent of total expenditure is spent on tennis? 200
200 100 150 150 100 300
0 0
1 1 0
(a) 12 % (b) 22 %
2004

2005

2006

2007

2008

2009

2010

2012

2013

2014
2011
2 2
(c) 25% (d) 45%
(e) 40% 66. In which year per unit cost in highest?
62. How much per cent more is spent on hockey than that on (a) 2006 (b) 2005
golf? (c) 2009 (d) 2011
(a) 27% (b) 35% (e) 2012
67. What is the approximate average quantity sold during the
(c) 37.5% (d) 75%
period 2004-2014?
(e) None of these (a) 64 units (b) 70 units
63. If the total amount spend on sports during the year be (c) 77 units (d) 81 units
`18000000, then the amount spent on basketball exceeds (e) 87 units
on Tennis by 68. If volatility of a variable during 2000-2014 is defined as
(a) `250000 (b) `360000
Maximum value – Minimum value
(c) `375000 (d) `410000 , then which of the
Average value
(e) `30000 following is true?
64. How many per cent less is spent on football than that on (a) Price per unit has highest volatility
Cricket? (b) Cost per unit has highest volatility
2 (c) Total profit has highest volatility
(a) 22 % (b) 27%
9 (d) Revenue has highest volatility
1 1 (e) None of the above
(c) 33 % (d) 37 % 69. If the price per unit decrease by 20% during 2004-2008
3 7
and cost per unit increase by 20% during 2009-2014, then
(e) 29% during how many number of years there is loss?
65. If the total amount spent on sports during the year was `2 (a) 3 yr (b) 4 yr
crore, the amount spent on cricket and hockey together was (c) 5 yr (d) 7 yr
(a) `800000 (b) `8000000 (e) None of these
(c) `12000000 (d) `16000000 70. If the price per unit decrease by 20% during 2000-2004
(e) None of these and cost per unit increase by 20% during 2005-2010, then
the cumulative profit for the entire period 2000-2010
DIRECTIONS (Qs. 66-70): Answer the questions based on the decrease by
following two graphs, assuming that there is not fixed cost (a) `1650 (b) `1550
component and all the units produced are sold in the same (c) `1300 (d) `1250
year. (e) Data inadequate

www.newspaperkorner.wordpress.com
www.newspaperkorner.wordpress.com

SBI BANK PO MAIN EXAM 2015 M-7

DIRECTIONS (Qs. 71-75) : Study the following graph carefully in B.Tech. (computer science). 18% of boys are in B.Tech.
to answer the question that follow. (Mechanical). Girls in B.Tech. (civil) are 30% of the girls in B.
Tech. (computer science). 15% of boys are in B. Tech.
Number of foreign tourists who visited three
different places in five different days (electronics). Boys in B. Teach (Computer Science) are 50% of
Golden Temple Wagha Border Taj Mahal the girls in the same 15% of girls are in B. Tech. (aerospace).
300 The ratio of boys to girls in B.Tech. (civil) is 3:1 respectively.
24% of the total numbers of students are in B.Tech. (electronics).
250 The ratio of boys to girls in B.Tech. (aerospace) is 12:5,
Number of tourists

200 respectively.
76. What is the total number of students enrolled in B.Tech.
150 (mechanical)?
100
(a) 1062 (b) 1530
(c) 1584 (d) 1728
50 (e) 1800
0
77. Number of girls enrolled in B.Tech. (electronics) forms
Monday Tuesday Wednesday Thursday Friday approximately. What per cent of total number of students
Days in the college?
71. What is the average number foreign tourists who visited (a) 7% (b) 13%
Wagha Border on all the days together? (c) 15% (d) 22%
(a) 184 (b) 172 (e) 24%
(c) 186 (d) 174 78. What is the total number of girls enrolled in B.Tech. (nano
(e) None of these technology)?
72. Total number of foreign tourists who visited all three places (a) 144 (b) 306
on Tuesday together is what per cent of the total number (c) 365 (d) 480
foreign tourists who visited Wagha Border on Monday and (e) 522
Wednesday together? 79. Number of boys enrolled in B. Tech. (aerospace) forms,
(a) 168% (b) 124% what per cent of the total number of girls enrolled in B.Tech.
(c) 112% (d) 224% (computer science)?
(d) 212% (a) 187.5% (b) 200%
73. On which day was the total number of foreign tourists who (c) 212.5% (d) 225%
visited all the three places together second lowest?
(e) 232.5%
(a) Monday (b) Tuesday
80. What is the total number of boys enrolled in B. Tech. (civil)?
(c) Wednesday (d) Thursday
(a) 240 (b) 432
(e) Friday
(c) 630 (d) 756
74. What is the respective ratio between the number of foreign
(e) 810
tourists who visited Taj Mahal on Thursday and the number
of foreign tourists who visited Golden Temple on Friday? DIRECTIONS (Qs. 81-85): In the following questions three
(a) 14:9 (b) 8:13 equations numbered I, II and III are given. You have to solve
(c) 13:8 (d) 13:7 all the equations either together or separately, or two together
(e) None of these and one separately, or by any other method and—
75. On which day was the different between the number of Give answer If
foreign tourists who visited Taj Mahal and Golden Temple, (a) x < y = z (b) x < y < z
second highest? (c) x < y > z (d) x = y > z
(a) Monday (b) Tuesday (e) x = y = z or if none of the above relationship is established
(c) Wednesday (d) Thursday 81. I. 7x + 6y + 4z = 122
(e) Friday II. 4x + 5y + 3z = 88
DIRECTIONS (Qs. 76-80): Study the information carefully to III. 9x + 2y + z = 78
answer the following questions. 82. I. 7x + 6y = 110
II. 4x + 3y = 59
There are 7200 students in an engineering college. The ratio of III. x + z = 15
boys of girls is 7:5, respectively. All the students are enrolled in
six different specialization viz., B.Tech. (electronics), B.Tech. 83. I. x= é(36)1/ 2 ´ (1296)1/ 4 ù
(Computer science), B.Tech. (Mechanical), B. Tech. (Aerospace), ë û
B.Tech. (nano technology), B.Tech. (civil). 22% of the total II. 2y + 3z = 33
students are in B. Tech. (nano technology). 16% of the girls are III. 6y + 5z = 71

www.newspaperkorner.wordpress.com
www.newspaperkorner.wordpress.com

M-8 SBI BANK PO MAIN EXAM 2015

84. I. 8x + 7y = 135 92. 46 9 13.5 20.25 30.375?


II. 5x + 6y = 99 (a) 40.25 (b) 45.5625
III. 9y + 8z = 121 (c) 42.7525 (d) 48.5625
85. I. (x + y)3 = 1331 (e) None of these
II. x– y+ z= 0
93. 400 240 144 86.4 51.84 31.104 ?
III. xy = 28
(a) 19.2466 (b) 17.2244
DIRECTIONS (Qs. 86-90) : Each of the questions below (c) 16.8824 (d) 18.6625
consists of a question and two statements numbered I and II (e) None of these
given below it. You have to decide whether the data provided in
94. 9 4.5 4.5 6.75 13.5 33.75?
the statements are sufficient to answer the question. Read the
(a) 101.25 (b) 103.75
question and both the statements and -
(c) 99.75 (d) 105.50
Give answer (a) if the data in statement I alone are (e) None of these
sufficient to answer the question, while the data in statement II
95. 705 728 774 843 935 1050 ?
alone are not sufficient to answer the question.
(a) 1190 (b) 1180
Give answer (b) if the data in statement II alone are
sufficient to answer the question, while the data in statement I (c) 1185 (d) 1187
alone are not sufficient to answer the question. (e) None of these
Give answer (c) if the data either in statement I alone or DIRECTIONS (Qs. 96-97) : Read the information given below
in statement II alone are sufficient to answer the question. and answer the questions that follow :
Give answer (d) if the data even in both the statements I
and II together are not sufficient to answer the question. The tournament for ABC Cup is arranged as per the following
Give answer (e) if the data in both the statements I and II rules: in the beginning 16 teams are entered and divided in 2
together are necessary to answer the question. groups of 8 teams each where the team in any group plays exactly
once with all the teams in the same group. At the end of this
86. Train 'A' running at a certain speed crosses another train 'B'
round top four teams from each group advance to the next round
running at a certain speed in the opposite direction in 12
in which two teams play each other and the losing team goes out
seconds. What is the length of train 'B'?
of the tournament. The rules of the tournament are such that
I. The length of both the trains together is 450 metres.
every match can result only in a win or a loss and not in a tie.
II. Train 'A' is slower than train 'B'.
The winner in the first round takes one point from the win and
87. Area of a rectangle is equal to the area of a right angled
the loser gets zero. In case of tie on a position the rules are very
triangle. What is the length of the rectangle ?
complex and include a series of deciding measures.
I. The base of the triangle is 40 cms.
II. The height of the triangle is 50 cms. 96. A red light flashes 3 times per minute and a green light
88. What was the total compound interest on a sum after three flashes 5 times in two minutes at regular intervals. If both
years ? lights start flashing at the time, how many times do they
I. The interest after one year was ` 100/- and the sum flash together in each hour?
was ` 1,000/-. (a) 30 (b) 24
II. The difference between simple and compound interest (c) 20 (d) 60
on a sum of ` 1,000/- at the end of two years was ` 10/-. (e) None of these
89. What is the two digit number where the digit at the unit 97. Ashish is given Rs. 158 in one rupee denominations. He
place is smaller ? has been asked to allocate them into a number of bags such
I. The difference between the two digits is 5. that any amount required between Re. 1 and Rs. 158 can
II. The sum of the two digits is 7. be given by handing out a certain number of bags without
90. What is the speed of the boat in still water ? opening them. What is the minimum number of bags
I. It takes 2 hours to cover distance between A and B required?
downstream. (a) 11 (b) 12
II. It takes 4 hours to cover distance between A and B (c) 13 (d) 15
upstreams. (e) None of these
DIRECTIONS (Qs. 91-95) : What should come in place of
DIRECTIONS (Qs. 98-100) : These questions are based on the
question mark (17) in the following number series ?
information and graph given below.
91. 13 14 30 93 376 1885 ? These questions are based on the price fluctuations of four
(a) 10818 (b) 10316 commodities - arhar, pepper, sugar, and gold during February -
(c) 11316 (d) 11318 july 1999 as described in the figures below :
(e) None of these

www.newspaperkorner.wordpress.com
www.newspaperkorner.wordpress.com

SBI BANK PO MAIN EXAM 2015 M-9

Arhar (Friday quotations) Pepper (Friday quotations)


2500 20000

Price (Rs. per quintal)

Price (Rs. per quintal)


2300 19500
2100 19000
1900 18500
1700 18000
1500 17500
1300 17000
F1 F4 M3 A2 A5 M3 JN2 JL1 F1 F4 M3 A2 A5 M3 JN2 JL1
Weeks (1999) Weeks (1999)

Sugar (Friday quotations) Gold (Friday quotations)


1520 4300
Price (Rs. per quintal)

Price (Rs. per quintal)


1500 4200
1480 4100
1460 4000
1440 3900
1420 3800
1400 3700
F1 F4 M3 A2 A5 M3 JN2 JL1 F1 F4 M3 A2 A5 M3 JN2 JL1
Weeks (1999) Weeks (1999)

98. Price change of a commodity is defined as the absolute 101. She asked me (a)/ Where I was going to (b)/ and what I
difference in ending and beginning prices expressed as a had done (c)/ the previous day. (d)/ No error (e)
percentage of the beginning. What is the commodity with 102. Yesterday in the night (a)/ he came (b)/ by bus (c)/ and was
the highest price changes disturbed. (d)/ No error (e)
(a) Arhar (b) Pepper 103. Within two hours (a)/ we will approach (b)/ near Agra (c)/
(c) Sugar (d) Gold by car. (d)/ No error (e)
(e) None of these 104. Throughout the whole year (a)/ there was (b)/ not a single
99. Price volatility (PV) of a commodity is defined as follows day (c)/ without any incidence of violence. (d)/ No error
PV = (highest price during the period- lowest price during (e)
the period)/ average price during the period and
105. The P.M. said (a)/ that it was his decision (b)/ and that
Highest price + lowest price + nobody (c)/ could get it changed. (d)/ No error (e)
ending price + beginning price DIRECTIONS (Qs. 106-110): In each of the following
Average price =
4 sentences, there are two blank spaces. Below each sentence there
What is the commodity with the lowest price volatility? are five pairs of words denoted by numbers (1), (2), (3), (4) and
(a) Arhar (b) Pepper (5). Find out which pair of words can be filled up in the blanks
(c) Sugar (d) Gold in the sentences in the same sequence to make the sentence
(e) None of these grammatically correct.
100. Mr. X, a fund manager with an investment company 106. The ___________ you work, the ___________ for your
invested 25% of his funds in each of the four commodities prosperity.
at the beginning of the period. He sold the commodities at (a) more, best (b) least, best
the end of the period. His investments in the commodities
(c) harder, better (d) decent, brightest
resulted in
(a) 17% profit (b) 5.5% loss (e) better, brighter
(c) no profit, no loss (d) 5.4% profit 107. ___________ you need a duplicate ration card, you must
(e) None of these submit the ___________ of your residence.
(a) Should, proof (b) If, numbers
(c) Had guarantee (d) Do, number
ENGLISH LANGUAGE
(e) Would, document
DIRECTIONS (Qs. 101-105): Read each sentence to find out 108. It was my ___________ that ___________ to the serious
whether there is any error in it. The error, if any, will be in one problem.
part of the sentence, the name of the part is the answer. If there (a) desire, brought (b) negligence, led
is no error, the answer is (e). (c) fault, lauded (d) mistake, subjected
(e) decision, put

www.newspaperkorner.wordpress.com
www.newspaperkorner.wordpress.com

M-10 SBI BANK PO MAIN EXAM 2015

109. Because he was ___________ he left the party earlier and are printed below the passage and against each, four words are
___________ home. suggested, one of which fits the blank appropriately. Find out
(a) tired, brought (b) precarious, approached the appropriate word in each case.
(c) preoccupied, sent (d) ill, contacted Delinking of jobs 116 degrees is one of the salient 117 of our
(e) unwell, returned education 118. There has been a steep 119 in 120 in the academic
110. An impartial person ___________ others without any
field in recent years. There is a 121 of degree holders in the
___________ .
country. As a result, university degrees have lost their value and
(a) likes, reservation (b) judges, bias
charm while the number of students in colleges and universities
(c) blames, prudence (d) praises, point
of the country has been steadily 122. Consequently, thousands of
(e) wishes, malice
graduates and postgradutes come out of these institutions and
DIRECTIONS (Qs. 111-115): Which of the phrases (a), (b), (c) stand in 123 waiting to get some 124 jobs 125 in the country.
and (d) given below each sentence should replace the phrase 116.(a) to (b) with
printed in bold in the sentence to make it grametically correct ? (c) from (d) by
If the sentence is correct as it is given and no correction is (e) None of these
required mark (e) as the answer. 117.(a) factors (b) features
111. The Governor has a good collection of paintings adoring (c) reasons (d) methods
the walls of the Raj Bhavan. (e) None of these
(a) painting adoring the wall 118.(a) process (b) system
(b) painting adoring the walls (c) procedures (d) policy
(c) painting sticking the walls (e) None of these
(d) painting adorning the walls 119.(a) fall (b) rise
(e) No correction required (c) down (d) decrease
112. The principals and teachers at the selected English (e) None of these
Medium School were contacted.
120.(a) evaluation (b) assessment
(a) of the selected English Medium
(c) result (d) competence
(b) at the select English Medium
(e) None of these
(c) upon the selected English Medium
121.(a) flood (b) growth
(d) of the section English Medium
(e) No correction required (c) increase (d) slope
113. Besides these norms, the data was also analyzed to form (e) None of these
the sub groups. 122.(a) falling (b) diminishing
(a) the data had analyzed also (c) rising (d) growing
(b) the data were also concluded (e) None of these
(c) the data were also analyzed 123.(a) lines (b) long
(d) an data were analyzed also (c) rows (d) queues
(e) No correction required (e) None of these
114. He does not know the Mount Everest is the Most highest 124.(a) managerial (b) nice
mountain peak. (c) prestigious (d) available
(a) is a more highest mountain (e) None of these
(b) is the most high mountain 125.(a) vacate (b) posted
(c) is the higher mountain (c) created (d) available
(d) is the highest mountain (e) None of these
(e) No correction required
115. Walking in the morning in open air is being observed to DIRECTIONS (Qs. 126-130): There are six sentences marked
be better for the health of the people who live in congestion. S1, S6, P, Q, R, S. The positions of S1 and S6 are fixed as the first
(a) was observer the best and last sentence of the passage. You are required to choose one
(b) hand been proved far better of the five alternatives given below every passage which would
(c) has been observed good be most logical sequence of the sentences in the passage.
(d) was proved far better 126. S1 : There are numerous kinds of superstitions in different
(e) No correction required parts of the country.
DIRECTIONS (Qs. 116-125): In the following passage there S6 : A dog’s howling predicts death– this is a typical
are blanks, each of which has been numbered. These numbers superstition.

www.newspaperkorner.wordpress.com
www.newspaperkorner.wordpress.com

SBI BANK PO MAIN EXAM 2015 M-11

P : But people go on respecting it through force of blind R : But this one wouldn’t budge.
custom. S : We had been cautioned about beggars and were told
Q : Most of them have a bearing on ‘luck’–good or bad. to ignore them.
R : Superstitions usually hae their origin in fear and The proper sequence should be
ignorance. (a) SQRP (b) SQPR
S : Nobody remembers now how a superstition first started (c) QSRP (d) QSPR
in remote ages. (e) PRQS
The proper sequence should be 130. S1 : In 1934, William Holding published a small volume
(a) QPRS (b) RSPQ of peoms.
(c) RSQP (d) QSPR S6 : But Lord of the flies which came out in 1954 was
(e) PRSQ welcomed as ‘a most absorbing and instructive tale’.
127. S1 : A spider’s web, after a shower of rain, is a very P : During the World War II (1939-45) he joined the Royal
beautiful thing. Navy and was present at the sinking of the Bismarck.
S6 : They are also feared because their bites may have Q : He returned to teaching is 1945 and gave it up in 1962,
unpleasant effects like a rash on the skin. and is now a full-time writer.
P : This party explains why spiders are thoroughly R : In 1939, he married and started teaching at Bishop
disliked. Wordsworth’s School in Salisbury.
Q : But no poet has ever sung of the beauty of the spiders, S : At first his novels were not accepted.
for most spiders are not beautiful.
The proper sequence should be
R : On the contrary, most of them are rather unattractive,
(a) RPQS (b) RPSQ
if not ugly!
(c) SRPQ (d) SQPR
S : Poets have sung about the beauty of the spider’s webs,
(e) RQPS
comparing the water drops on them to ropes of pearls.
The proper sequence should be DIRECTIONS (Qs. 131-150): Read the following information
(a) SPQR (b) QSRP and answer the questions that follows:
(c) QRSP (d) SQRP PASSAGE-1
(e) PQRS True, it is the function of the army to maintain law and order in
128. S1 : Unhappiness and discontent spring not only form
abnormal times. But in normal times there is another force that
poverty.
compels citizens to obey the laws and to act with due regard to
S6 : We suffer from sickness of spirit and hence we should
the rights of others. The force also protects the lives and the
discover our roots in the internal.
properties of law abiding men. Laws are made to secure the
P : Man is a strange creature fundamentally different from
personal safety of its subjects and to prevent murder and crimes
other animals.
of violence. The are made to secure the property of the citizens
Q : If they are undeveloped and unsatisfied, he may have
against theft and damage to protect the rights of communities
all the comforts of the wealth, but will still feel that
and castes to carry out their customs and ceremonies, so long as
life is not worthwhile.
they do not conflict with the rights of other. Now the good citizen,
R : He has far horizons invariable hopes, spiritual powers.
of his own free will obey these laws and he takes care that
S : What is missing our age is the soul, there is nothing
wrong with the body. everything he does is done with due regard to the rights and
The proper sequence should be well-being of others. But the bad citizen is only restrained from
(a) PRQS (b) SPRQ breaking these laws by fear of the consequence of his actions.
(c) SPQR (d) PRSQ And the necessary steps to compel the bad citizen to act as a
(e) QSRP good citizen are taken by this force. The supreme control of law
129. S1 : On vacation in Tangier, Morocco, my friend and I sat and order in a state is in the hands of a Minister who is responsible
down at a street cafe. to the State Assembly and acts through the Inspector General of
S6 : Finally a man walked over to me and whispered, Police.
“ Hey buddy .... This guy is your waiter and he wants 131. The expression ‘customs and ceremonies’ means
your order.” (a) fairs and festivals.
P : At one point, he bent over with a big smile, showing (b) habits and traditions.
me a single gold tooth and dingy fez. (c) usual practices and religious rites.
Q : Soon I felt the presence of someone standing alongside (d) superstitions and formalities.
me. (e) None of these

www.newspaperkorner.wordpress.com
www.newspaperkorner.wordpress.com

M-12 SBI BANK PO MAIN EXAM 2015

132. A suitable title for the passage would be : (b) Criminals, who flout the law, are seldom brought to
(a) the function of the army. book.
(b) laws and the people’s rights. (c) The police hardly succeed in converting bad citizens
(c) the fear of the law and citizen’s security. into good citizens.
(d) the functions of the police. (d) The police check the citizens, whether they are good
(e) None of these or bad, and stop them from violating the law.
133. Which of the following is not implied in the passage? (e) None of thesess
(a) Law protects those who respect it. 139. Which of the following statement expresses most accurately
(b) Law ensures people’s religious and social rights the idea contained in the first sentence?
absolutely and unconditionally. (a) It is the job of the army to ensure internal peace at all
(c) A criminal is deterred from committing cirmes only times.
for fear of the law. (b) It is the police that should always enforce law and
(d) The forces of law help to transform irresponsible order in the country.
citizens into responsibile ones. (c) Army and the police ensure people’s security through
(e) None of these combined operations.
134. According to the writer, which one of the following is not (d) It is in exceptional circumstances that the army has to
the responsibility of the police? ensure peace in the country.
(a) To protect the privileges of all citizens. (e) None of thesess
(b) To check violent activities of citizens. 140. The last sentence of the passage implies that
(c) To ensure peace among citizens by safeguarding (a) The Inspector General of Police is the sole authority
individual rights. in matters of law and order.
(d) To maintain peace during extraordinary circumstances. (b) In every State maintenance of public peace is under
(e) None of these the overall control of the responsible Minister.
135. Which of the following reflects the main thrust of the (c) A Minister and a responsible State, Assembly exercise
passage. direct authority in matters pertaining to law and order.
(a) It deals with the importance of the army in maintaining (d) The Inspector General of Police is responsible to the
law and other. State Assembly for maintaining law and order.
(b) It highlights role of the police as superior to that of (e) None of thesess
the army. PASSAGE - 2
A recent report in News Week says that in American colleges,
(c) It discusses the roles of the army and the police in
students of Asian origin outperform not only the minority group
different circumstances.
students but the majority whites as well. Many of these students
(d) It points to the responsibility of the Minister and the
must be of Indian origin, and their achievement is something we
Inspector General of Police.
can be proud of. It is unlikely that these talented youngsters will
(e) None of thesess
come back to India and that is the familiar brain drain problem.
136. “They are made to secure the property of citizens against
However recent statements by the nation’s policy-makers indicate
theft and damage”, means that the law :
that the perception of this issue is changing. ‘Brain bank’ and
(a) helps in recovering the stolen property of the citizens.
not ‘brain drain’ is the more appropriate idea, they suggest since
(b) assist the citizens whose property has been stolen or
the expertise of indians abroad is only deposited in other places
destroyed.
and not lost.
(c) initiate process against offenders of law. This may be so, but this brain bank, like most other banks,
(d) safeguard people’s possessions against being stolen is one that primarily serves customers in its neighborhood. The
or lost. skills of the Asians now excelling in America’s colleges will
(e) None of thesess mainly help the U.S.A.. No matter how significant, what non-
137. Out of the following which one has the opposite meaning resident Indians do for India and what their counterparts do for
to the word ‘restrained’ in the passage? other Asian lands is only a by-product.
(a) Promoted (b) Accelerated But it is also necessary to ask, or be remained, why Indians
(c) Intruded (d) Inhibited study fruitfully when abroad. The Asians whose accomplishments
(e) None of thesess News Week records would have probably has a very different
138. Which one of the following statement is implied in the tale if they had studied in India. In America they found elbow
passage? room, books and facilities not available and not likely to be
(a) Peaceful citizens seldom violate the law, but bad available here. The need to prove themselves in their new country
citizens have to be restrained by the police. and the competition of an international standard they faced there

www.newspaperkorner.wordpress.com
www.newspaperkorner.wordpress.com

SBI BANK PO MAIN EXAM 2015 M-13

must have cured mental and physical laziness. But other things 147. The performance of Indians when they go to study in the
helping them in America can be obtained here if we achieve a West :
change is social attitudes, specially towards youth. (a) shows the fruits of hardwork done by school teachers
We need to learn to value individuals and their unique in India.
qualities more than conformity and respectability. We need to (b) should remind us that knowledge and wisdom are not
learn the language of encouragement to add to our skill in flattery. limited by the boundaries of race and nation.
141. Among the many groups of students in American colleges, (c) is better than people in the West expect of non-whites.
Asian students. (d) is better than what it would have been if they had
(a) are often written about in magazines like News Week. studied in India.
(b) are most successful academically (e) None of thesess
(c) have proved that they are as good as the whites. 148. The high level of competition faced by Asian students in
(d) have only a minority status like the blacks. America.
(e) None of these (a) helps them overcome their lazy habits.
142. The student of Asian origin in America include : (b) makes them lazy since the facilities there are good.
(a) a fair number from India. (c) makes them worried about failing.
(b) a small group from India. (d) helps them prove that they are as good as whites.
(c) persons from India who are very proud.
(e) None of thesess
(d) Indians who are the most hard working of all.
149. The author feels that some of the conditions other than the
(e) None of thesess
level of facilities that make the West attractive.
143. In general, the talented young Indians studying in America:
(a) are available in India but young people do not
(a) have a reputation for being hard working
appreciate them.
(b) have the opportunity to contribute to India’s
(b) can never be found here because we believe in
development.
conformity.
(c) can solve the brain drain problem because of recent
(c) can be created if our attitudes and values change.
changes in policy.
(d) can also give respectability to our traditions and
(d) will not return to pursue their careers in India.
customs.
(e) None of thesess
(e) None of thesess
144. There is talk now of the ‘brain bank’ This idea :
150. One of the ways of making the situation in India better
(a) is a solution to the brain drain problem
would be
(b) is a new problem caused parly by the brain drain
(a) to eliminate flattery from public life.
(c) is a new way of looking at the role of qualified Indians
(b) to distinguish between conformity and respectability.
living abroad.
(c) to give appreciation and not be tightfisted.
(d) is based on a plan to utilize foreign exchange
(d) to encourage people and no merely flatter them.
remittances to stimulate research and development.
(e) None of thesess
(e) None of thesess
145. The brain bank has limitations like all banks in the sense
that: GENERAL AWARENESS
(a) a bank’s services go mainly to those near it. 151. PM Narendra Modi launched which bank in In dia?
(b) small neighbourhoods banks are not visible in this age (a) HDFC (b) ICICI
of multinationals. (c) SBI (d) PNB
(c) only what is deposited can be withdrawn and utilized. (e) Indian
(d) no one can be forced to put his assets in a bank. 152. LIC ( Life Corporation of India) started business in which
(e) None of thesess country?
146. The author feels that what non-resident Indians do for India: (a) Nepal (b) Myanmar
(a) will have many useful side effects. (c) Bangladesh (d) Bhutan
(b) will not be their main interest and concern. (e) Sri Lanka
(c) can benefit other Asian countries as a by-product. 153. Who is the CEAT cricketer of the year 2015?
(d) can American colleges be of service to the world (a) M S Dhoni (b) Rahul Dravid
community. (c) Virat Kholi (d) Kumar Sangkkara
(e) None of thesess (e) None of these

www.newspaperkorner.wordpress.com
www.newspaperkorner.wordpress.com

M-14 SBI BANK PO MAIN EXAM 2015

154. Where is Jim Corbett National Park is located? 165. Land Border Agreement between India and Bangladesh was
(a) Uttar Pradesh (b) Odisha the _________.
(c) Bihar (d) Madhya Pradesh (a) 120th amendment (b) 100th amendment
(e) Uttarakhand (c) 117th amendment (d) 112th amendment
155. On which river Omkareswar dam is located? (e) None of these
(a) Tapti (b) Ravi 166. The sixty-eighth session of the World Health Assembly
(c) Ganga (d) Narmada (WHA)took place in ________ 18-26 May 2015.
(e) Satluj (a) Geneva (b) Rome
156. Which country recently launched the first ATM with face (c) Paris (d) Washington DC
recognition? (e) None of these
(a) China (b) Nepal 167. What term is used for maximum capital which the company
(c) India (d) Sri Lanka can raise in its life time?
(e) Bangladesh (a) Authorized Capital (b) Registered Capital
157. What is Full Form of BIOS? (c) Nominal Capital (d) All of them
(a) Basic Investment/ Output System (e) None of these
(b) Basic In/ out System 168. Which state government approved mandatory Global
(c) Basic Input/ Output System Positioning System (GPS) in all public transport vehicles
(d) Basic Input/ Output Service from June 2015?
(e) None of these (a) Delhi (b) Madhya Pradesh
158. Agartala is the capital of which state ? (c) Odisha (d) Maharashtra
(a) Tripura (b) Assam (e) Punjab
(c) Mizoram (d) Odisha 169. China signed a currency swap deal with which country?
(e) Nagaland (a) Sri Lanka (b) Russia
159. Volkswagen Group and its subsidiaries, is a ________ (c) India (d) Chile
(e) None of these
multinational automotive manufacturing company.
170. The Concept of 'Universal Banking' was implemented in
(a) British (b) American
India on the recommendations of:
(c) Chinese (d) German
(a) Abid Hussai Committee
(e) Japanese
(b) R H Khan Committee
160. In the Balance of Payment (BOP), the transfer of payment
(c) S Padmanabhan Committee
is included in which of the following ?
(d) YH Malegam Committee
(a) Current Account
(e) None of these
(b) Capital Account
171. Who among the following is the head of the committee
(c) Savings Accounts under Govt. Schemes
formed by SEBI to frame a single set of guidelines for all
(d) Official Reserve Accounts
types of foreign investors?
(e) All of them
(a) Prabhat Kumar (b) T.R. Prasad
161. Who has been selected as head coach of India U-19?
(c) K.M. Chandrashekhar (d) Kamal Pande
(a) Rahul Dravid (b) Sachin Tendulkar
(e) None of these
(c) Saurabh Gaguly (d) Virendra Sehwag
172. Largest shareholder (in percentage shareholding) of a
(e) None of these
nationalized bank is.
162. Where is Katasraj temple located?
(a) RBI (b) NABARD
(a) India (b) Pakistan (c) LICI (d) Govt. of India
(c) Afganistan (d) Nepal (e) IBA
(e) Bangladesh 173. Which of the following organization finance the Rural
163. Which of the following is not a part of Money Market? Infrastructure Development Fund?
(a) Treasury Bill (b) Commercial Papers (a) NABARD (b) SEBI
(c) Mutual Funds (d) Certificate of Deposits (c) SIDBI (d) RBI
(e) None of these (e) Govt. of India
164. The Nobel Prize in Chemistry 2014 was awarded _______. 174. Who is the author of the book 'Odyssey of My Life'?
(a) Eric Betzig (b) Stefan W. Hell (a) Karan Singh (b) Kwasi Kwarteng
(c) William E.Moerner (d) All of the above (c) James Astill (d) Shivraj V Patil
(e) None of these (e) None of these

www.newspaperkorner.wordpress.com
www.newspaperkorner.wordpress.com

SBI BANK PO MAIN EXAM 2015 M-15

175. World Photography day is observed on : 184. In word you can force a page break _______
(a) 19th August (b) 22nd August (a) by positioning your cursor at the appropriate place
(c) 20th August (d) 17th August and pressing the F1 key
(e) 16th August (b) By positioning your cursor at the appropriate place
176. Which of the statement on Islamic banking is incorrect? and pressing Ctrl + Enter
(A) Islamic banking is only for Muslims (c) By using the Insert/section break
(B) Islamic banking activity do not allow paying and (d) By changing the font size of your documents
receiving of interest (e) None of these
(C) Islamic banking promotes the principle of profit 185. Specialized programs that assist users in locating
sharing between banking institution and depositor information on the Web are called
(a) Only A (b) Only B (a) Information engines (b) locator engines
(c) web browsers (d) resource locators
(c) Only C (d) Both B and C
(e) search engines
(e) None of these
186. Compiling creates a(n)
177. The difference between the outflow and inflow of foreign
(a) error-free program (b) program specification
currency is known as ___ (c) subroutine (d) algorithm
(A) Foreign Exchange Reserves (e) executable program
(B) Current Account Deficit 187. Wha is the name given to those applications that combine
(C) Fiscal Deficit text, sound, graphics, motion video, and/or animation?
(D) Balance of Payments (a) motionware (b) anigraphics
(a) Only A (b) Only B (c) videoscapes (d) multimedia
(c) Only C (d) Only D (e) maxomedia
(e) None of these 188. How are the World Wide Web (WWW) and the internet
178. NABARD (National Bank for Agriculture and Rural different?
Development) established JLG to extract the productive (a) They aren’t different; they are two different names for
capability from rural population. JLG stands for ? the same thing.
(a) Junior Link Group (b) Join Life Group (b) The internet is text only, the WWW incorporates
(c) Joint Line Group (d) Joint Liability Group multimedia.
(e) None of these (c) The internet is primarilyused by businesses; the WWW
179. What is MZM in money market? can be used by home users.
(a) Money with highest return (d) The WWW is faster than the more archaic internet.
(b) Money with zero maturity (e) None of these
189. Which ports connects special types of music instruments
(c) Money with sovereign guarantee
to sound cards?
(d) Monetary base or total currency
(a) BUS (b) CPU
(e) None of these
(c) USB (d) MIDI
180. A person who used his or expertise to gain access to other (e) MINI
peoples computers to get information illegally or do damage 190. The long term objective of marketing is-
is (a) Customer satisfaction
(a) Hacker (b) Analyst (b) Profit maximization
(c) Instant messenger (d) Programmer (c) Cost cutting
(e) Spammer (d) Profit maximization with customer satisfaction
181. To access properties of an object, the mouse technique to (e) None of these
use is ________ 191. Zero-based budgeting (ZBB) means-
(a) dragging (b) dropping (a) A tool of marketing cost
(c) right-clicking (d) shift-clicking (b) a tool for financial analysis
(e) None of these (c) each year, budgeting starts from a scratch.
182. Computers used the _______ number systems to store data (d) a certain percentage of sales
and perform calculations (e) Both 1 & 2.
(a) binary (b) octal 192. A marketing technique where marketer plays a specific role
(c) decimal (d) hexadecimal in a particular segment is called -
(e) None of these (a) Mass Marketing
183. The default view in Excel is ________ View. (b) Niche marketing
(a) Work (b) Auto (c) Strategic Marketing
(c) Normal (d) Roman (d) Communication Marketing
(e) None of these (e) None of these

www.newspaperkorner.wordpress.com
www.newspaperkorner.wordpress.com

M-16 SBI BANK PO MAIN EXAM 2015

193. In Marketing Mix 4 P's Imply - 197. Effective Selling Skills depends on-
(a) Product, Price, Place, Promotion (a) Number of languages known to the DSA
(b) Product, Price, Policy, Place (b) Data on marketing staff
(c) Place, People, Product, Promotion (c) Information regarding IT market
(d) All of the above (d) Knowledge of related market
(e) None of these (e) Ability to talk fast
194. Good marketing strategy envisages good and proper ____. 198. The first CNG train is launched on -
(a) Product distribution (b) Networking of branches (a) Delhi-Agra Section
(c) High Pricing (d) Placement of counter staff (b) Rohtak-Chandigarh Section
(e) Relationship management (c) Delhi-Chandigarh Section
195. Marketing in banks has been necessitated due to (d) Rewari-Rohtak Section
(a) globalisation (e) None of these
(b) excess staff 199. Name the Malaysian woman who has become the world's
(c) nationalisation of bank oldest women(and also the oldest living person, presently).
(d) complacency among the staff (a) Nurul Yeoh (b) Yati Musdi
(e) poor customer service (c) Michele Chia (d) Celine Choo
196. Innovation in marketing is same as- (e) None of these
(a) Abbreviation 200. Which among the following is the theme of the World
(b) Communication Health Day-2015 ?
(c) Creativity (a) Food Safety (b) Aging and Health
(d) Aspiration (c) Water Safety (d) Small Bite Bit Thread
(e) Research work (e) None of these

www.newspaperkorner.wordpress.com
www.newspaperkorner.wordpress.com

SBI BANK PO MAIN EXAM 2015 M-17

Answers &
Explanations
(1-3) : 9. (d) A’s mother-in-law is B and A is second to the right
Colours of the sky = ki la fa so ...(i) of B.
rainbow colours = ro ki ...(ii) 10. (a) A’s grandchild is C and A is immediate right of C.
[from (i) and (ii)] 11. (c) Except C all others are women.
\ Colours = ki..... 12. (e) B is the mother of H which is true with respect to
sky high rocket = la pe jo ...(iii) the given sitting arrangement.
from (i) and (iii), sky = la
13. (a) F’s grandmother sits to the immediate left of C.
the rocket world = pe so ne
from (i) and (iv), the = so ...(iv) (14-15) :
and from (iii) and (iv), rocket = pe According to given information.
1. (e) colours sky high = ki la jo
2. (c) ‘the’ represents only ‘so’.
3. (e) ‘pe’ represents ‘rocket’. Dancer Footballer Music lover
(4-8) :
4. (a) Statements W ³ D < M < P < A = F
Conclusions Cricketer Player
I. F > D ® It follows.
II. P < W ® It does not follows. 14. (b) Possibility case
So, only conclusion I follows.
5. (d) Statements H ³ M > F < A = B > S
Conclusions
I. H > B ® It does not follow. Player Dancer Footballer Music lover
II. F < S ® It does not follow.
Neither Conclusion I nor II follows.
6. (b) Statements B > T > Q > R = F Cricketer
Conclusions
Hence, all of the conclusion follows.
I. Q ³ F Þ It does not follow because Q > F only..
15. (a) Possibility case
II. T > F Þ It follows.
Only Conclusion II follows.
7. (b) Statements S = R ³ Q, P < Q Music lover
\ S = R³ Q > P Dancer Footballer
Conclusions Cricketer
I. S ³ P Þ It does not follow because S ³ Q and Q > P..
II. R > P Þ It follows. Player
Only conclusion II follows.
8. (b) Statements S ³ M < Y = Z > F > T Hence, only Conclusion I follows.
Conclusions (16-18) :
I. S > F Þ It does not follow. According to given information,
II. Y > T Þ It follows.
Only conclusion II follows.
(9-13) : Eight persons sitting arrangements areas as follows. Intelligent Boys
(D's Daughter) Professor

(G's Uncle or E

Girls Leader
Maternal-uncle) + – (A's Wife D's
D H Sister)
16. (c) Hence, Conclusions I and II follow.
17. (c) Possibility case
(H's Mother A's – –
Mother-in-law B F (A's and H II
F's Grandmother) Daughter
(+) (–)
G's Sister) Professor
C G Boys
(G's Son A's +
(C's Mother A Girls
GrandChild) A and H Daughter) Intelligent
Leader
(H's Husband
G's Father)

www.newspaperkorner.wordpress.com
www.newspaperkorner.wordpress.com

M-18 SBI BANK PO MAIN EXAM 2015

27. (c)
III
28. (d) Even by using both the statements together we can-
not determine whether B has highest number of stu-
Boys Professor dent or D.
Intelligent 29. (d) Which additional piece of information would
Girls Leader strengthen the argument? The information that
strengthens this argument will be about the subjects
Hence, Conclusions II and III follow.
of the argument, not about tangential issues. In this
18. (c) Hence, only Conclusions III follows.
(19-23) : After careful analysis of the given input and various case, the argument is about early hominids’ use of
steps of rearrangement it is evident that in each step one word fire. Any physical evidence, that links the early homi-
and one number are rearranged. nids to the charred bone fragments strengthens the
The word are rearranged from left in alphabetical order and the argument. If these bone fragments were found in con-
numbers are rearranged from the right in descending order but junction with some other evidence of the presence of
in the final step the word get rearranged in alphabetical order in early hominids, then the evidence from the Swartkans
reserves manner and number appear in descending order. location could be used to support the claim that early
Input: 31 rise gem 15 92 47 aim big 25 does 56 not 85 63 with hominids used fire.
moon
Hence, statement (4) properly indentifies evidence that
Step I : aim 31 rise gem 15 47 big 25 does 56 not 85 63
links early hominids to these bone fragments and so
with moon 92
Step II : big aim 31 rise gem 15 47 25 does 56 not 63 with strengthens the argument.
moon 92 85 30. (b) People having a comfortable lifestyle are found to be
Step III : does big aim 31 rise gem 15 47 25 56 not with happier than leading strenuous lifestyle by their height
moon 92 85 63 on most contradicts the findings.
Step IV : gem does big aim 31 rise 15 47 25 not with moon 31. (c) Clearly, option (c) weaken the findings because suf-
92 85 63 56 fering from malaria respond well to the traditional
Step V : moon gem does big aim 31 rise 15 25 not with 92 malaria drugs, while in statement not seems so.
85 63 56 47 32. (a) The cost of transportation from state A to state C is
Step VI : not moon gem does big aim rise 15 25 with 92 85 more than 30% of the production cost can be directly
63 56 47 31
drawn from the statement.
Step VII : rise not moon gem does big aim 15 with 92 85 63
33. (c) After a cautioned gap of one month people have
56 47 31 25
Step VIII : with rise not moon gem does big aim 92 85 63 56 started visiting crowded places most substantiates the
47 31 25 15 facts.
19. (a) 34. (c) The air traffic controllers of the city should be sent to
20. (d) refresher courses so that they can improve them self
21. (b) in handling such situation is the most suitable course
22. (c) of action.
23. (c) (35 - 40) :
24. (b) I. P « N ƒ K + We can conclude the following result:
(+ ) b (–)
J
Flat No. Agewise decreasing Qualification Name
Using I only, we cannot determine if J is the nephew
or niece of K. (3) Chess player Power Engineer Amit
II. P « N ƒ K ∗
(∗ ) b (, ) (3) Cricket (Regional) Mech. Engineer Manu
J (2) Football Design Engineer Rohit
(∗ )

J is the nephew of K. (5) Tennis Quality Inspector Tarun


25. (a) I. Using I only, we can determine that G stays
on 4th floor. (4) Cricket (National) Mech. Engineer Ambrish
26. (e)
I. Raju can take 4 to 8 days to complete the work.
II. Raju can take 8 to 10 days to complete the work. Us- 35. (a)
ing both the statements together we can determine 36. (d)
that Raju took 8 days to complete the work. 37. (c)

www.newspaperkorner.wordpress.com
www.newspaperkorner.wordpress.com

SBI BANK PO MAIN EXAM 2015 M-19

38. (d) (56-60) :


39. (c) Stock at the beginning of
40. (a) June = 75 units
(41-45) : July = 75 – (350 – 300) = 75 – 50 = 25 units
A (Finance) August = 25 – (275 – 250) = 25 – 25 = 0 unit
September = 200 – 175 = 25 units
October = 25 – (325 – 300) = 50 units
D (CS) F (MCA)
November = 25 + (325 – 300) = 50 units
From the above calculation, we get the following table
Months Quantity Quantity Stock of
Produced sold the beginning
E (MBA) H (M.TECH) June 300 350 75
July 250 275 25
August 200 175 0
September 200 200 25
B (B.com) C (B.TECH)
October 325 300 25
G (CA) 56. (c) From the above table, it is clear that in the beginning
of August, the company had the least stock.
41. (c) 57. (c) The stock at the beginning of September was 25
42. (a) units.
43. (a) 58. (e) Required minimum stock should be 75 units. If we
44. (a) had the stock at the beginnings less than 75 units,
45. (b) the company would not have been able to supply in
46. (e) August.
47. (e) 59. (a)
48. (e) T is grandfather. 60. (b) The stock at the beginnings of July was 25 units.
49. (b) S is great grandmother of N. 61. (a) Percentage of money spent on tennis
50. (d) The statement mentions that the commercial banks
violate a directive issued by the RBI. The remedy is æ 45 ö 1
=ç ´100 ÷ % = 12 %
only to make the banks implement the Act. So, none è 360 ø 2
of the courses follows. 62. (d) Degree value of expenditure on hockey = 63°
51. (e) Number of women liking Degree value of expenditure on golf = 36°
CSK = 44% of 45525 = 20031 63 - 36
DD = 33% of 36800 = 12144 \ Required percentage = ´ 100 = 75%
36
DC = 30% 0f 56340 = 16902
63. (a) Amount spent on basketball exceeds that on tennis
MI = 28% of 62350 = 17458 by
RR = 44% of 48300 = 21252
RCB = 35% of 35580 = 12453 æ (50 - 45) ö
= `ç ´ 18000000 ÷ = `250000
KXI = 36% of 56250 = 20250 è 360 ø
KKR = 54% of 64000 = 34560 64. (c) Degree value of the expenditure
Total = 155050 on football = 54°
155050 on cricket = 81°
\ Required average = =19381.25 ; 19381
8 81 - 54
\ Required percentage = ´ 100
52. (e) Required percentage 81
Number of women liking RR 27 1
= ´ 100 = ´ 100 = 31 %
Number of women liking all teams 18 3
= 21252/155050 × 100 = 13.7% 65. (b) Degree value of expenditure on cricket and hockey
53. (b) Required number = 45525×36/100 = 16389 = (81 + 63)° = 144°
54. (c) Required ratio = (36800 × 39) : (48300 × 21) = 208 144
: 147 \ Required amount spent on them =` ´ 2 crore
360
55. (c) Number of men liking DC = 45% 56340 = 25353
Number of men liking RCB = 15% of 355080 = 5337 = 0.8 crore = `8000000
66. (b) Suppose x units are produced each year.
25353 So, in the year 2006,
\ Required percentage = ´ 100 = 475.04%
5337 Total revenue = 1200

www.newspaperkorner.wordpress.com
www.newspaperkorner.wordpress.com

M-20 SBI BANK PO MAIN EXAM 2015

Þ 12 × x = 1200 Checking of option (b),


Þ x = 100 Total cost = Revenue – Profit
Prifit = 300 = (700 + 1400 + 1200+ 900 + 1100 + 400 + 200
\ Cost price = 1200 – 300 = 900 + 700 + 600 + 800+ 900) – (0 + 100 + 300 + 150
+ 0 + 150 + 100 + 200 + 0 + 400 + 300)
900
\ Cost per unit = =9 = 8900 – 1700 = 7200
100
In the year 2005, 7200
Average cost per unit =
Total revenue = 1400 11
Þ 14 × x = 1400 (1400 - 100) - (200 - 100)
x = 100 Volatility cost per unit =
7200
Profit = 100
\ Cost price = 1400 – 100 = 1300 1300 - 100
= ´ 11 = 1.833
1300 7200
\ Cost per unit = = 13 Checking option (c),
100
1700
In the year 2009, Average profit =
11
Total revenue = 400
Þ 8 × x = 400 400 - 0 44
\ Profit volatility = = = 2.588
Þ x = 50 1700 17
Profit = 150 11
\ Cost price = 400 – 150 = 250 Checking option (d),
250 8600
\ Cost per unit = =5 Average revenue =
50 11
In the year 2011, 1400 - 200
Total revenue = 700 \ Revenue volatility =
8600
Þ 14 × x = 700
11
Þ x = 50
Profit = 200 132
\ Cost price = 700 – 200 = 500 = = 1.535
86
500 \ Total profit has highest volatility.
\ Cost per unit = = 10
50 69. Year New revenue Total cost
Thus, per unit cost is highest in the year 2005.
= Old revenue – Profit
67. (b) Years Unit price Revenue Total units
2004 80% of 700 – 560 700
Revenue 2005 80% of 1400 = 1120 1300
=
Unit price 2006 80% of 1200 = 960 900
2007 80% of 900 = 720 750
2000 10 700 70
2008 80% of 1100 = 880 1100
2001 14 1400 100
2009 400 120% of 250 = 300
2002 12 1200 100
2010 200 120% of 100 = 120
2003 12 900 75
2011 700 120% of 500 = 600
2004 11 1100 100
2012 600 120% of 600 = 720
2005 8 400 50
2013 800 120% of 400 = 480
2006 10 200 20
2014 900 120% of 600 = 720
2007 14 700 50
2008 10 600 60 In a year when total cost is more than new revenue then
2009 10 800 80 in that year there is loss.
2010 15 900 60 These years are 2004, 2005, 2006, 2007 and 2008
So, there are total 5 yr.
Total = 765
70. (b) Total decrease in revenues
765
\ Average units < ; 70 units = 20% of (700 + 400 + 1200 + 900 + 1100) = 1060
11 Total increase in cost
68. (c) Checking option (a), = 20% of (250 + 100 + 500 + 600 + 400 + 600) =
15 - 8 77 490
Volatility per unit = = = 0.611 Decrease in cumulative profit
126 126
= Total decrease in revenues + Total increase in cost
11
= 1060 + 490 = `1550

www.newspaperkorner.wordpress.com
www.newspaperkorner.wordpress.com

SBI BANK PO MAIN EXAM 2015 M-21

71. (b) Total number of foreign tourists who visited Wagha


12
Border on all the days together Number of boys in B.Tech. (aerospace) = ´ 450 = 1080
= 130 + 190 + 120 + 200 + 220 = 860 5
Number of boys in B.Tech. (nano technology) = Remaining
860 Number of boys = 4200–(756+630+240+432+1080)
Therefore, required average = = 172
5 = 4200–3138=1062
72. (e) Total number of tourists whos visited all the three \ Number of girls in B.Tech. (nano technology)
places on Tuesday = 70 + 190 + 270 = 530 =1584 – 1062 = 522
Total number of foreign tourists who visited Wagha Number of girls in B.Tech. (mechanical) = Remaining
Border on Monday and Wednesday together Number of girls = 3000 – (480 + 144 + 450 + 1098 +
= 130 + 120 = 250
522) = 3000 – 2694 = 306
530 Tabular form of above information is shown below.
\ Required percentage = ´ 100% = 212%
250 Subjects Number of Number of
73. (c) Total number of tourists who visited all the three boys girls
places on Monday = 90 + 130 + 200 = 420
on Tuesday = 70 + 190 + 270 = 530 B.Tech. (computer science) 240 480
on Wednesday = 120 + 150 + 180 = 450 B.Tech. (mechanical) 756 306
on Thursday = 200 + 230 + 260 = 690 B.Tech. (civil) 432 144
on Friday = 140 + 190 + 220 = 550 B.Tech. (electronics) 630 1098
Thus, the number of tourists was second lowest on B.Tech. (aerospace) 1080 450
Wednesday. B.Tech. (nano technology) 1062 522
74. (d) Number of tourists who visited Taj Mahal on Total 4200 3000
Thursday = 260
Number of thourists who visited Golden Temple on 76. (a) From the table, it is clear that total number of
Friday = 140 students enrolled in B.Tech. (meachanical)
Therefore, required ratio = 260 : 140 = 13 : 7 77. (c) Number of girls enrolled in B.Tech. (electronics) =
75. (a) Difference between the number of foreign tourists 1098
who visited Taj Mahal and Golden Temple
on Monday = 200 – 90 = 110 1098
\ Required percentage = ´ 100%
on Tuesday = 270 – 70 = 200 7200
on Wednesday = 180 – 150 = 30 =15.25% ; 15%
on Thursday = 260 – 230 = 30 78. (e) Number of girls enrolled in B.Tech. (nano technology)
on Friday = 190 – 140 = 50
= 522
Thus, the difference is second highest on Monday.
(76-80) : 79. (d) Number of boys enrolled in B.Tech. (aerospace) =
Number of students in the college = 7200 1080
Number of girls enrolled in B.Tech. (computer
7
Number of boys = ´ 7200 = 4200 science) = 480
12
1080
5 \ Required percentage = ´ 100 % = 225%
Number of girls = ´ 7200 = 3000 480
12
Number of student in B. Tech. (nano technology) 80. (b) Total number of boys enrolled in B.Tech. (civil) =
= 22% of 7200 = 1584 432
Number of girls in B. Tech. (computer science) 81. (a) 7x + 6y + 4z = 122 ... (i)
= 16% of 3000 = 480 4x + 5y + 3z = 88 ... (ii)
Number of boys in B.Tech. (mechanical) = 18% of 4200 9x + 2y + z = 78 ... (iii)
= 756 By equation (iii) × 3 – equation (ii),
Number of girls in B.Tech. (civil) = 30% of 480 = 144
Number of boys in B.Tech. (electronics) = 15% of 4200 27x + 6y + 3z = 234
= 630 4 x + 5 y + 3z = 88
Number of boys in B.Tech. (computer science) = 50% of - - - - ...(iv)
480 = 240 23x + y = 146
Number of girls in B.Tech. (aerospace) = 15% of 3000 =
4500 By equation (iii) × 4 – equation
(i),
3
Number of boys in B.Tech. (civil) = ´ 144 = 432
1 36 x + 8 y + 4 z = 312
Number of students in B.Tech (electronics) = 24% of 7200 7 x + 6 y + 4 z = 122
= 1728 - - - - ...(v)
\ Number of girls in B.Tech. (electronics) = 1728 – 630 29 x + 2 y = 190
=1098

www.newspaperkorner.wordpress.com
www.newspaperkorner.wordpress.com

M-22 SBI BANK PO MAIN EXAM 2015

By equation (iv) × 2 – equation 84. (d) By equation 1 × 5 – II × 8


(v),
40 x + 35 y = 675
46 x + 2 y = 292 40 x + 48 y = 792
29 x + 2 y = 190 - - -
- - - ...(iv) - 13 y = -117
17 x = 102
Þ y = 9
Þ x = 6 From equation I,
From equation (iv), 8x + 7 × 9 = 135
23 × 3 + y = 146 Þ 8x = 135 – 63 = 72
Þ y = 146 – 138 = 8 Þ x = 9
From equation (iii), From equation III,
9 × 6 + 2 × 8 + z = 78 9 × 9 + 8z = 121
Þ 54 + 16 – z = 78 Þ 8z = 121 – 81 = 40
Þ z = 78 – 70 = 8 Þ z = 5
Clearly, x < y = z Clearly, x = y > z
82. (c) 85. (e) I. (x + y)3 = 1331
By equation (II) × 2 – equation (I) Þ x + y = 11
Þ y = 11 – x
Number Floor Person From equation III,
6 Fifth Floor P x (11 – x) = 28
5 Fourth Floor T Þ 11x – x2 = 28
4 Third Floor V Þ x2 – 11x + 28 = 0
3 Second Floor S Þ x2 – 7x – 4x + 28 = 0
2 First Floor R Þ x (x – 7) – 4 (x – 7) = 0
1 Ground Floor Q Þ (x – 7) (x – 4) = 0
Þ x = 7, 4
From equation (I), From equation I
7 × 8 + 6y = 110 y = 4, 7
Þ 6y = 110 – 56 = 54 From equation II
Þ y = 9 7 –4 +z =0 Þz =– 3
From equation (iii), 4 –7 +z =0 Þz = 3
8 + z = 15 Þ z = 7 86. (d) Let S1 and S2 be the speed of train A and B respectively
Clearly, x < y > z
Time taken by both the trains in crossing each other.
1 1 450
(36) ´ (1296) = = 12, S1 + S2 = 37.5
83. (b) I. x = 6 ´ 6 = ±6 S1 + S2
2 4
S1 and S2 can have so many values. Both statement I
By equation II × 3 – equation I
and II are not sufficient to find speed of train B.
6 y + 9 z = 99 87. (d) Area of rectangle = Area of triangle.
6 y + 5 z = 71
- - - Þ z = 9 From the information given in both the statements,
4z = 28 we can find area of triangle or area of rectangle. For
From equation II, finding length, breadth is required, which is not
known.
2y + 3 × 7 = 33
88. (c) From the statement I.
Þ 2y = 33 – 21 = 12
Þ y = 6 100 ´100
r= = 10%
x £ y< z 1000

www.newspaperkorner.wordpress.com
www.newspaperkorner.wordpress.com

SBI BANK PO MAIN EXAM 2015 M-23

Thus we have, Hence, number 11316 will replace the question mark.
P = ` 1000, r = 10%, t = 3 years 6 9 13.5 20.25 30.375 45.5625
4
Hence, C.I. can be determined 92. (b)
From the statement II. ×1.5 ×1.5 ×1.5 ×1.5 ×1.5 ×1.5

1000 ´ r ´ 2 400 240 144 86.4 51.84 31.104 18.6624


S.I. = = 20r
1000 93. (d)
× 0.6 × 0.6 × 0.6 × 0.6 × 0.6 × 0.6
éæ r ö
2 ù
ê 1 + - 1ú 9 4.5 4.5 6.75 13.5 33.75 101.25
C.I.= 1000 ç ÷
êëè 100 ø úû 94. (a)
× 0.5 ×1 × 1.5 ×2 × 2.5 ×3
\ C.I. – S.I.
In each term in the series added by 1, 2, 3, ... and
é 200r + r 2ù then multiplied by 23
= 1000 ê 10000 ú - 20r 95. (e) 705 + 1 × 23 = 728
ëê úû
728 + 2 × 23 = 774
Þ 2000r + r2 – 200r = 100 774 + 3 × 23 = 843
Þ r = 10 843 + 4 × 23 = 935
Hence, C.I. can be determined 935 + 5 × 23 = 1050
89. (e) Let the unit’s digit be x and ten’s digit be y and x < y. \ ? = 1050 + 6 × 23
\ Number = 10y + x = 1050 + 138 = 1188
From statement I, 96. (a) Red light flashes 3 times / min. i.e after every
y–x=5 ... (i) 20 seconds.
From statement II, Green light flashes 5 times in 2 min. i.e. after every
y+x=7 ... (ii) 24 seconds.
From (i) and (ii), x, y can be calculated and two digit So, they flash together after every 2 minutes
number can be found. = 120 seconds
90. (d) Let the distance between A and B be z km. (L.C.M. of 20 & 24 = 120)
Again, let speed of boat in still water be x kmph and Hence, in 1 hour they flash together 60/2 = 30 times
that of stream be y kmph. 97. (e) Minimum number of bags we have to allocate them in
\ Rate downstream = (x + y) kmph such a way that we get all the numbers
Rate upstream = (x – y) kmph i.e., 1 + 2 + 4 + 8 + 16 + 32 + 64 + 128
From statement I, Hence, minimum no. of bags required is 8; having

z number of coins 20 , 21 , 2 2 ,...., 27


=2 ... (i)
x+ y 98. (a) Price change of each commodity is as follows :
From statement II, 2150 - 1700
Arhar = = 26.5%
z 1700
=4 ... (ii)
x- y 19250 - 18500
Pepper = = 4%
we have two equations and three variables, therefore 18500
both equations are not sufficient.
1435 - 1440
91. (c) The given number series is based on the following Sugar = = –0.3%
pattern : 1440
13 × 1 + 1 = 14 3840 - 4240
14 × 2 + 2 = 30 Gold = = 9.43%
4240
30 × 3 + 3 = 93 99. (c) Price Volatility is defined in the question PV
93 × 4 + 4 = 376
376 × 5 + 5 = 1885 H.P. - L.P.
=
A.P.
\ ? = 1885 × 6 + 6 = 11316

www.newspaperkorner.wordpress.com
www.newspaperkorner.wordpress.com

M-24 SBI BANK PO MAIN EXAM 2015

H.P. L.P. A.P. PV 106. (c) 107. (a) 108. (b) 109. (e) 110. (b)
Arhar 2300 1500 1912.50 0.42 111. (d) 112. (b) 113. (c) 114. (d) 115. (e)
Pepper 19500 17400 18622.50 0.112 116. (c) 117. (b) 118. (b) 119. (a) 120. (d)
Sugar 1500 1410 1446.25 0.06 121. (a) 122. (c) 123. (d) 124. (c) 125. (d)
Gold 4300 3800 4045 0.124 126. (d) 127. (d) 128. (a) 129. (c) 130. (a)
So the lowest price volatility is for sugar. 131. (c) 132. (d) 133. (b) 134. (c) 135. (c)
100. (d) Price change which we have calculated previously is 136. (d) 137. (b) 138. (d) 139. (d) 140. (b)
nothing but profit percent or loss percent depending 141. (c) 142. (a) 143. (d) 144. (c) 145. (a)
upon the sign. In the first two we have profit percentage
which is 26.5 + 4% = 29.5% and the loss percentage 146. (b) 147. (d) 148. (d) 149. (c) 150. (c)
is 151. (b) 152. (c) 153. (d) 154. (e) 155. (d)
0.3 + 9.4 = 9.7% then net profit % comes out to be 156. (a) 157. (c) 158. (a) 159. (d) 160. (a)
20.8% and then the average of which gives i.e. 161. (a) 162. (b) 163. (c) 164. (d) 165. (b)
20.8 166. (a) 167. (d) 168. (a) 169. (d) 170. (b)
= 5.2% so the closest answer is (d).
4 171. (c) 172. (d) 173. (a) 174. (d) 175. (a)
101. (b) Delete ‘to’. It is superfluous. 176. (a) 177. (b) 178. (d) 179. (b) 180. (a)
102. (a) Replace ‘Yesterday in the night’ by last night. 181. (c) 182. (a) 183. (c) 184. (b) 185. (e)
103. (c) Replace ‘will approach’ with ‘will be approaching’. 186. (e) 187. (d) 188. (b) 189. (d) 190. (d)
104. (a) Use of whole is superfluous. 191. (c) 192. (b) 193. (a) 194. (e) 195. (a)
105. (c) Use of that is superfluous. 196. (c) 197. (d) 198. (d) 199. (b) 200. (a)

www.newspaperkorner.wordpress.com
www.newspaperkorner.wordpress.com

SBI Bank PO Prelim Exam 2015


Held On : 20-06-2015

Max. Marks : 100 (Based on Memory) Time : 1 hour

9. A man is facing west. He turns 45 degree in the clockwise


REASONING ABILITY direction and then another 180 degree in the same direction
1. How many such pairs of letters are there in the word and then 270 degree in the anticlockwise direction. Find
SHOULDER each of which has as many letters between them which direction he is facing now?
in the word as in the English alphabet? (a) South-West (b) West
(a) None (b) One (c) South (d) East-South
(c) Two (d) Three (e) None of these
(e) More than three
2. If it is possible to make only one meaningful English word DIRECTIONS (Qs. 10 - 15) : Each of the questions are given
with the first, the fifth, the seventh and the tenth letters of four statements followed by four conclusions numbered I, II, III
the word STREAMLINE, using each letter once in each IV. You have to take the given statements to be true even it they
world, which of the following is the third letter of that word? seem to be at variance from commonly known facts Read all the
If no such word can be made, give ‘X’ as the answer and if conclusions and then decide which of the given conclusions
more than one such word can be made, give ‘Y’ as the answer. logically follows from the given statements disregarding
(a) L (b) E commonly known facts.
(c) S (d) X
(e) Y 10. Statements : All drums are tubes.
3. Four of the following five are alike in a certain way and so form Some tubes are pipes.
a group. Which is the one that does not belong to that group? No pipe is stick.
(a) Nitrogen (b) Hydrogen Some sticks are rubbers.
(c) Methane (d) Neon Conclusions I : Some rubbers are tubes.
(e) Helium Conclusions II : Some sticks are drums.
4. In a certain code BREAKING is written BFSCFMHJ, How is Conclusions III : Some pipes are drums.
MOTHERLY written in that code? Conclusions IV : Some sticks are tubes.
(a) IUPNZMSF (b) IUPNXKQD (a) None follows (b) Only I follows
(c) IUPNFSMZ (d) GSNLZMSF (c) Only I and II follow (d) Only III follows
(e) None of these (e) None of these
5. Among P, Q, R, S and T Each having a different height, Q is 11. Statements : Some pens are rooms.
taller than S. T is shorter than P. R is taller than Q but shorter All rooms are walls
than T. Who among them in the tallest? Some walls are bricks
(a) S (b) P All bricks are slates.
(c) R (d) Data inadequate Conclusions I : Some slates are walls.
(e) None of these Conclusions II : Some walls are pens.
6. In a row of 40 children, R is 11th from the right and there are Conclusions III : Some bricks are rooms.
15 children between R and M. What is M’s position from the Conclusions IV : Some slates are rooms.
left and of the row? (a) Only I and III follow (b) Only II and III follows
(a) 14th (b) 15th (c) Only I and II follow (d) Only III and IV follow
(c) 13th (d) Can’t be determined (e) None of these
(e) None of these 12. Statements : Some chairs are pencils.
7. In a certain code language ‘how many are there’ is written as Some pencils are bottles.
‘ka na ta da’ and ‘many are welcome here’ is written as ‘na pa Some bottles are bags.
ni ka’. How is ‘how’ written in that code language? Some bags are books.
(a) ta (b) da Conclusions I : Some books are pencils.
(c) ta or da (d) Data inadequate Conclusions II : Some bottles are chairs.
(e) None of these Conclusions III : No book is pencil.
8. If ‘R’ denotes ‘¸’, ‘T’ denotes ‘–’, ‘M’ denotes ‘+’ and ‘W’ Conclusions IV : Some bags are chairs.
denotes ‘×’, then (a) Only I follows
27 T 15 R 3 W 4 M 6 = ? (b) Only either I and III follows
(a) 7 (b) 13 (c) Only III follows
(c) ‘ –’ 23 (d) 1 (d) Only IV follows
(e) None of these
(e) None of these

www.newspaperkorner.wordpress.com
www.newspaperkorner.wordpress.com

P-2 SBI BANK PO PRELIM EXAM 2015

13. Statements : Some roads are buses. Volleyball, Badminton, Basketball and Carom, not necessarily in
All buses are trains. the same order, Each of them also has a specific choice of color
Some trains are trucks. from- Blue, Red, Green, Yellow, Grey, Black and White, not
All trucks are kites.. necessarily in the same order.
Conclusions I : Some trucks are roads.
Conclusions II : Some kites are buses. R likes Green and his favorite sport is Badminton. V’s choice
Conclusions III : Some trains are roads. of color is neither Red nor Black. T’S favorite sport is neither
Conclusions IV : Some kites are trains. Table tennis nor Basketball. The one who likes Blue does not like
(a) None follows (b) Only I follows Carom. The one who likes Volleyball does not like Yellow and
(c) Only II follows (d) Only III follows Grey. Q’s favorite sport is Lawn Tennis and he likes Black. S likes
(e) None of these
14. Statements : All beads are rings. White. W likes Basketball. P likes Volleyball. T likes Blue. The
All rings are bangles. one who likes Basketball does not like Grey.
All bangles are tyres. 21. What is V’s choice color?
All tyres are pendants. (a) Black (b) Grey
Conclusions I : Some pendants are beads.
Conclusions II : Some tyres are rings. (c) Yellow (d) Data inadequate
Conclusions III : Some bangles are beads. (e) None of these
Conclusions IV : Some pendants are rings. 22. What is T’s favorite sport?
(a) Only I and II follow (b) Only I, II and III follows (a) Basketball (b) Volleyball
(c) Only II, III and IV follow (d) Only I, III and IV follow (c) Chess (d) Data inadequate
(e) All follow
15. Statements : Some desks are fruits. (e) None of these
All fruits are flowers. 23. Whose favourite sport is Carrom?
No flower is branch. (a) S (b) R
Some branches are roots. (c) W (d) Data inadequate
Conclusions I : Some roots are flowers. (e) None of these
Conclusions II : No branches are desks.
Conclusions III : Some flowers are desks. 24. Whose favourite sport is basketball?
Conclusions IV : Some desks are branches. (a) S (b) T
(a) Only either II and IV follows (c) W (d) R
(b) Only III follows (e) None of these
(c) Only either II or IV and III follow.
(d) Only I and II follow 25. What is W’s choice of colour?
(e) None of these (a) Green (b) White
(c) Black (d) Data inadequate
DIRECTIONS (Qs. 16 to 20): Read the following information (e) None of these
carefully and answer the questions given below:
A, B, C, D, E, F, G and H are sitting around a circle facing the DIRECTIONS (Qs. 26 to 30): Read the following information
centre. B is 2nd to the right of D who is 3rd to the right of F. C is 2nd and answer the questions based on it.
to the left of A who is 2nd to the left of F. G is 3rd to right of E.
P @ Q means P is either greater than or equal to Q
16. In which of the following combination is the 1st person sitting
between the 2nd and the 3rd persons? P + Q means P is either smaller than or equal to Q
(a) GCD (b) FGH P % Q means P is greater than Q
(c) EFH (d) ABE P × Q means P is smaller than Q
(e) None of these P $ Q means P is neither greater than nor smaller than Q.
17. Who is 3rd to the right of H? Now in each of the following questions assuming the given
(a) G (b) D
(c) C (d) Data inadequate statement to be true, find which of the two conditions I and II
(e) None of these given below them is/are definately true? Give answer.
18. Who is to the immediate right of A? (a) If only conclusion I is true
(a) B (b) E (b) If only conclusion II is true
(c) F (d) Data inadequate (c) If either I or II is true
(e) None of these
19. What is H’s position with respect to B? (d) If neither I or II is true
(a) 5th to the right (b) 3rd to the left (e) If both I and II is true
(c) 5 to the left
th
(d) 3rd to the right 26. Statements : M @ R, R % T, T $ K
(e) 4th to the left Conclusion : (I) K × M, (II) T × M
20. Who is to the immediate left of G? 27. Statements : H % J, B + J, B @ F
(a) H (b) F
Conclusion : (I) F $ J, (II) J % F
(c) D (d) Data inadequate
(e) None of these 28. Statements : D $ M, M % W, W @ R
Conclusion : (I) R×D, (II) W + D
DIRECTIONS (Qs. 21 to 25): Study the following information 29. Statements : A + N, N×V, V$J
carefully and answer the questions given below: Conclusion : (I) J @ N, (II) A + V
P, Q, R, S, T, V and W are seven members of a club. Each of 30. Statements : K×T, T @ B, B + M
them has a favorite sport from-Chess, Table Tennis, Lawn Tennis, Conclusion : (I) M % T, (II) K + B

www.newspaperkorner.wordpress.com
www.newspaperkorner.wordpress.com

SBI BANK PO PRELIM EXAM 2015 P-3

DIRECTIONS (Qs. 31 to 35): Read the following information 40. Ms. Pooja Pushpan invests 13% of her monthly salary, i.e.
carefully and answer the questions given below it: `, 8554 in Mediclaim Policies, Later she invests 23% of her
monthly salary on Child. Education Policies; also she
In a family, there are six members A, B, C, D, E and F. A and B are invests another 8% of her monthly salary on Mutual
married couple, A being the male member. D is the only son of C,
Funds. What is the total annual amount invested by Ms.
who is the brother of A. E is the sister of D. B is the daughter-
inlaw of F, whose husband has died. Pooja Pushpan?
(a) ` 28952 (b) ` 43428
31. How is F related to A?
(a) Mother (b) Sister-in-law (c) ` 347424 (d) ` 173712
(c) Sister (d) Mother-in-low (e) None of these
(e) None of these 41. The profit earned after selling an article for `878 is the
32. How is E related to C? same as loss incurred after selling the article for `636.
(a) Sister (b) Daughter What is the cost price of the article?
(c) Cousin (d) Aunt (a) ` 797 (b) ` 787
(e) Mother (c) ` 767 (d) ` 757
33. Who is C to B? (e) None of these
(a) Brother (b) Brother-in-law
42. In a class of 240 students, each student got sweets got
(c) Nephew (d) Son-in-law
(e) None of these sweets that are 15% of the total number of students.
34. How many male members are there in the family? How many sweets were there?
(a) One (b) Two (a) 3000 (b) 3125
(c) Three (d) Four (b) 8640 (d) Cannot be determined
(e) Five (e) None of these
35. How is F related to C? 43. Sonika spent `45, 760 on the interior decoration for her
(a) Mother-in-law (b) Sister-in-law home, `7896 on buying air conditioner and the remaining
(c) Mother (d) Aunt
28% of the total amount she had as cash with her. What
(e) Sister
was the total amount?
(a) `98540 (b) `102300
QUANTITATIVE APTITUDE (b) `134560 (d) Cannot be determined
36. The simple interest accrued on an amount of `20,000 at (e) None of these
the end of three years is `7,200. What would be the 44. The ages of Khushi and Jagriti are in the ratio of 5 : 8
compound interest accrued on the same amount at the respectively. After 8 years the ratio of their ages will be
same rate in the same period? 3 : 4 what is the difference in their ages?
(a) ` 8098.56 (b) ` 8246.16 (a) 16 years (b) 8 years
(c) ` 8112.86 (d) ` 8342.36 (c) 10 years (d) 12 years
(e) None of these (e) None of these
37. If the numerator of a fraction is increased by 300% and the
45. In how many different ways can the letters of the word
denominator is increased by 500%, the resultant fraction
‘PUNCTUAL’ be arranged?
5
is . What was the original fraction? (a) 64 (b) 40320
12
(c) 960 (d) 20160
8 5 (e) None of these
(a) (b)
5 11
12 5 DIRECTIONS (46-50) : What will come in place of the
(c) (d) question mark (?) in the following number series.
5 7
(e) None of these 46. 93 95 99 ? 110 121 134
38. A sum of money is divided among A, B, C and D in the
(a) 104 (b) 96
ratio of 3 : 5 : 9 : 13 respectively. If the share of C is `.
2412 more than the share of A, then what is the total (c) 82 (d) 103
amount of money of B and D together? (e) None of these
(a) ` 4422 (b) ` 7236 47. 8 12 18 27 40.5 60.75?
(c) ` 6030 (d) ` 4824 (a) 104.125 (b) 121.125
(e) None of these (c) 96.125 (d) 83.125
39. The average age of 80 girls was 20 years, the average age (e) None of these
of 20 of them was 22 years and that of another 20 was 24
years. Find the average age of the remaining girls. 48. 4 7 11 18 29 ? 76 123
(a) 17 years (b) 19 years (a) 59 (b) 38
(c) 21 years (d) 15 years (c) 46 (d) 53
(e) None of these (e) None of these

www.newspaperkorner.wordpress.com
www.newspaperkorner.wordpress.com

P-4 SBI BANK PO PRELIM EXAM 2015

49. 3 10 ? 172 885 5346 37471 299832 Production of two companies A & B over the years (Production
(a) 39 (b) 27 in Lakh units)
(c) 24 (d) 34
(e) None of these 14 A
50. 15 22 56 183 ? 3755 22542 13 B
(a) 709 (b) 698 12
(c) 748 (d) 800
(e) None of these 11

Production in Lakh units


10
DIRECTIONS (51 - 55) : What approximate value will come
in place of the question mark (?) in the following questions? 9
8
51. 2959.85 ÷ 16.001 – 34.99 = ?
(a) 160 (b) 150 7
(c) 140 (d) 180 6
(e) 170
5
52. (1702 ÷ 68) × 136.05 = ?
(a) 3500 (b) 3550 4
(c) 3450 (d) 3400 3
(e) 3525
2
53. 2950 ÷ 12.25 + 160 = ?
(a) 440 (b) 350 1
(c) 380 (d) 360 0
(e) 400
54. 25.05% of 2845 + 14.95 × 2400 = ?
2009

2012
2008

2010

2013
2011

2014

2015
(a) 36,700 (b) 36,500
(c) 35,800 (d) 35,600
Years
(e) 36,200
61. For Company A, what is the percent decrease in production
55. (186 × 270.99) ÷ 40 = ?
from 2008 to 2009?
(a) 1160 (b) 1200 (a) 75 (b) 50
(c) 1300 (d) 1180 (c) 35 (d) 10
(e) 1260 (e) None of these
62. In 2004, the production of Company B is approximately
DIRECTIONS (Qs. 56-60) : In each of the following questions what per cent of that of in 2013?
two equations are given. You have to solve them and (a) 60 (b) 157
(c) 192 (d) 50
Give answer (a) if p < q; (e) 92
63. For Company A, in which year is the percentage increase/
Give answer (b) if p > q; decrease in the production from the previous year the
Give answer (c) if p £ q; highest?
Give answer (d) if p ³ q; (a) 2014 (b) 2005
Give answer (e) if p = q; (c) 2012 (d) 2008
(e) None of these
56. I. p2 – 7p = – 12 64. What is the difference in the total production of the two
Companies for the given years?
II. q2 – 3q + 2 = 0
(a) 27,00,000 (b) 31,00,000
57. I. 12p2 – 7q = –1
(c) 2,70,000 (d) 3,10,000
II. 6q2 – 7q + 2 = 0 (e) None of these
58. I. p2 + 12p + 35 = 0 65. Which of the following is the closest average production
in lakh units of Company B for the given years?
II. 2q2 + 22q + 56 = 0
(a) 4.1 (b) 3.5
59. I. p2 – 8p + 15 = 0
(c) 4.3 (d) 3.75
II. q2 – 5q = – 6
(e) 3.9
60. I. 2p2 + 20p + 50 = 0
DIRECTIONS (Qs. 66-70) : Study the following information to
II. q2 = 25
answer the given questions.
DIRECTIONS (Qs. 61 - 65) : Study the following graph to
Percentage of students in various courses (A, B, C, D, E, F) and
answer the given questions. Percentage of girls out of these.

www.newspaperkorner.wordpress.com
www.newspaperkorner.wordpress.com

SBI BANK PO PRELIM EXAM 2015 P-5


Total students : 1200 (D) Some have high birth rate and still have a high death rate.
(E) Compared to this is Europe the growth rate is low.
(800 girls + 400 boys)
(F) The developing countries show the most rapid growth rate.
Percentage in various courses 71. Which will the LAST sentence in the Para?
(a) A (b) B
(c) C (d) D
F
(e) E
13% A
20%
72. Which will be the FIRST sentence in the Para?
E (a) E (b) D
12% (c) C (d) B
B (e) A
D 73. Which will come at SECOND place in the para?
35% C 15% (a) A (b) B
5% (c) C (d) D
(e) E
Total Girls : 800 74. Which sentence will come at FIFTH place in the Para?
Percentage of Girls in courses (a) F (b) D
(c) B (d) A
(e) C
F 75. Which will be the THIRD sentence in the Para?
14% A (a) B (b) A
30% (c) D (d) F
E (e) E
14%
B DIRECTIONS (Qs. 76 to 80): Fill in the blanks with appropriate
D
C
10% word:-
30%
2% Let children learn to judge their own work. A child ___(76)___ to
talk does not learn ___(77)___ being corrected all the time
66. For course D what is the respec-tive ratio of boys and girls? ___(78)___corrected too much, he will ___(79)___ talking. He
(a) 3 : 4 (b) 4 : 5 notices a thousand times a day the difference between the
___(80)___ he uses and the language those around him use.
(c) 3 : 5 (d) 5 : 6 76. (a) endeavouring (b) learning
(c) experimenting (d) experiencing
(e) None of these (e) preparing
67. For which pair of courses is the number of boys the same? 77. (a) In (b) on
(a) E & F (b) A & D (c) By (d) to
(c) C & F (d) B & D (e) From
(e) None of these 78. (a) unless (b) being
68. For course, E, the number of girls is how much per cent (c) until (d) if
more than the boys for course E? (e) s o
79. (a) stop (b) halt
(a) 250 (b) 350 (c) avoid (d) shun
(c) 150 (d) 80 (e) giveup
(e) None of these 80. (a) speech (b) language
69. For which course is the number of boys the minimum? (c) talk (d) skill
(a) E (b) F (e) virtue
(c) C (d) A
(e) None of these DIRECTIONS (Qs. 81 to 85): Five alternative a, b, c, d and e are
70. How many girls are in course C ? given under each sentence, you are required to select the most
(a) 44 (b) 16 suitable alternative to fill in the blank/blanks in the sentence to
(c) 40 (d) 160 make it meaningful.
(e) None of these 81. Intelligence is an _____ part of one’s success.
(a) inseparable (b) inimitable
(c) indivisible (d) indispensable
ENGLISH LANGUAGE (e) None of these
82. Anjana impressed the interviewer with her concise, ____
DIRECTIONS (Qs. 71 to 75): Given below are six sentences i.e. answers.
A, B, C, D, E and F, which have been presented in a wrong order. (a) allusive (b) revealing
Arrange them in order to form a meaningful paragraph and (c) pertinent (d) referential
then answer the questions given below. (e) None of these
83. The coach asked the players to _____ with his ideology or
(a) Some others, like the European nations, have a low leave the team.
birth rate and a low death rate. (a) counter (b) align
(b) Many others have a high birth rate with a low death rate. (c) favour (d) separate
(C) Different countries show different patterns of growth. (e) None of these

www.newspaperkorner.wordpress.com
www.newspaperkorner.wordpress.com

P-6 SBI BANK PO PRELIM EXAM 2015


84. The seminar helped _____ the students on the harmful effects 92. The phrase lavish with his hospitatlity signifies
of smoking and alcohol. (a) miserliness in dealing with his friends.
(a) educate (b) learn (b) considerate ness in spending on guests and strangers.
(c) teach (d) insist (c) extravagance in entertaining guests.
(e) None of these (d) indifference in treating his friends and relatives.
85. Rajeev was _____ legal aid to fight his extradition. (e) none of these
(a) offered (b) granted 93. We understand from the passage that
(c) allowed (d) awarded (a) all mean people are wealthy
(e) None of these (b) wealthy people are invariably successful.
(c) carefulness generally leads to failure.
DIRECTIONS (Qs. 86 to 90): Read each sentence to find out (d) thrift may lead to success.
whether there is any grammatical error in it. The error, if any (e) none of these
will be in one part of the sentence. The letter of that part is the 94. It seems that low paid people should
answer. If there is no error, the answer is ‘D’. (Ignore the errors (a) not pay their bills promptly.
of punctuation, if any) (b) not keep their creditors waiting.
86. “The Patient is comparatively better (a)/ today and I hope (b)/ (c) borrow money to meet their essential needs
that he will recover soon” (c)/ said the doctor (d)/ No error (e) (d) feel guilty if they overspend
87. All the members (a)/ of the committee are (b)/ kindly (e) none of these
requested to appear (c)/ in the next morning (d)/ No error (e). 95. The word paradox means:
88. This is the new (a)/ book “One Night @ the call centre” (a) statement based on facts.
authored by Chetan Bhagat which my father (b)/ bought it (b) that which brings out the inner meaning.
(c)/ for you (d)/ No error (e) (c) that which is contrary to received opinion.
89. The passing marks (a)/ in Economics is thirty three (b)/ but (d) statement based on the popular opinion.
(e) none of these
he has secured (c)/ just twenty-nine (d)/ No error
96. How does the housewife, described by the writer, feel when
90. As my neighbourers (a)/ are very co-operative (b)/ I do not
she saves money?
(c)/ feel any difficultly in living in this locality (d)/ No error (e)
(a) is content to be so thrifty.
DIRECTIONS (Qs. 91 to 100): Read the following passage (b) wishes life were less burdensome.
carefully and answer the questions given below it. (c) is still troubled by a sense of guilt.
(d) wishes she could sometimes be extravagant.
It is a strange that, according to his position in life, an
(e) none of these
extravagant man is admired or despised. A successful
97. Which of the following is opposite in meaning to the word
businessman does nothing to increase his popularity by applauded in the passage?
being careful with his money. He is expected to display his (a) Humiliated (b) Decried
success, to have smart car, an expensive life, and to be lavish (c) Cherished (d) Suppressed
with his hospitality. If he is not so, he is considered mean (e) None of these
and his reputation in business may even suffer in 98. The statement she is able to face the milkman with equanimity
consequence. The paradox remains that if he had not been implies that:
careful with his money in the first place, he would never (a) she is not upset as she has been paying the milkman
have achieved his present wealth. Among the two income his dues regularly.
groups, a different set of values exists. (b) she loses her nerve at the sight of the milkman who
The yaung clerk who makes his wife a present of a new always demands his dues.
dress when the hadn’t paid his houserent condemned as (c) she manages to keep cool as she has to pay the milkman
extravagant. Carefulness with money to the point of who always demands his dues.
meanness is applauded as a virtue. Nothing in his life is (d) she remains composed and confident as she knows
considered more worthy than paying his bills. The ideal wife that she can handle the milkman tactfully.
for such a man separaters her housekeeping money into (e) none of these
joyless little piles- so much for rent, for food, for the children’s 99. As far as money is concerned, we get the impression that
shoes; she is able to face the milkman with equanimity and the writer:
never knows the guilt of buying something she can’t able to (a) is incapable of saving anything
face the milkman with equanimity and never knows the guilt (b) is never inclined to be extravagant
of buying something she can’t really afford. As for myself, I (c) would like to be considered extravagant
fall into neither of these categories. If I have money to spare (d) doesn’t often have any money to save
I can be extravagant, but when, as is usually the case, I am (e) none of these
hard up, then I am then meanest man imaginable. 100. Which of the following would be the most suitable title for
91. In the opinion of the writer, a successful businessman: the passage?
(a) is more popular if he appears to be doing nothing. (a) Extravagance leads to poverty
(b) should not bother about his popularity. (b) Miserly habits of the poor.
(c) must be extravagant before achieving success. (c) Extravagance in the life of the rice and the poor.
(d) is expected to have expensive tastes. (d) Extravagance is always condemnable.
(e) None of these (e) None of these

www.newspaperkorner.wordpress.com
www.newspaperkorner.wordpress.com

SBI BANK PO PRELIM EXAM 2015 P-7

Answers &
Explanations
1. (b) moved 180 degree clockwise, now he will be facing OC. From
2. (e) Meaningful words: here he moved 270 degree anticlockwise, Finally he is facing
SALE, SEAL OD, which is South west.
1 2 3 4 5 6 7 8 9 10
S T R E A M L I N E
3. (c) Among these, only Methane is compound gas. 10. (a) Tubes

Drums Pipes Sticks Rubbers


4. (b) B R E A K I N G

+1 or

B F S C F M H J
Tubes

MO T H E R L Y Drums Pipes

+1 –1
Sticks Rubbers
I U P N X K Q D

5. (b) P>T>R> Q>S


\ P is the tallest Conclusion I : (False)
6. (a) Conclusion II : (False)
7. (c) how many are there ® ka na ta da (i) Conclusion III : (False)
many are welcome here ® na pi ni ka (ii) Conclusion IV : (False)
From equations (i) and (ii) many are ® na ka \ None follows
how ® ta or da
8. (b) Given arrangement = 27 T 15 R 3 W 4 M 6 11. (c) Walls Slates
According to question, letters converted into
mathematical symbols Rooms Bricks
= 27 – 15 ¸ 3 × 4 + 6 = 27 – 5 × 4 + 6
= 27 – 20 + 6 = 33 – 20 = 13 Pens

9. (a) B Conclusion I : (True)


Conclusion II : (True)
270° Conclusion III : (False)
Starting Conclusion IV : (False)
point 180°
45°
A
O Chairs Pencils Bottles Bags Books
12. (b)
End point
C
Conclusion I : (False)
D
Conclusion II : (False) Either I and III follows.
The man firstly faces the direction OA. On moving 45 degree Conclusion III : (False)
clockwise [Please check carefully always if clockwise or Conclusion IV : (False)
anticlockwise], he faces the direction OB. Now again he

www.newspaperkorner.wordpress.com
www.newspaperkorner.wordpress.com

P-8 SBI BANK PO PRELIM EXAM 2015

13. (e) (16-20 :


Trains Kites

Buses Trucks B
A C
Roads

E D
Conclusion I : (False)
Conclusion II : (False) G
F
Conclusion III : (True) H
Conclusion IV : (True)
16. (d) 17. (c) 18. (b) 19. (e) 20. (a)
14. (e)
Pendants 21-25 :
Tyres
Bangles Members Sports Colors
Rings P Volleyball Red
Beads Q Lawn-tennis Black
R Badminton Green
S Carrom/Table tennis White
T Chess Blue
V Carrom/Table tennis Grey

Conclusion I : (True) W Basketball Yellow

Conclusion II : (True) 21. (b) 22. (c) 23. (d) 24. (c) 25. (e)
Conclusion III : (True) 26. (e) M³R> T=K
Conclusion IV : (True) Conclusion I : K < M (True)
Hence, All four follows. Conclusion II : T < M (True)
Hence, both conclusions are true.
27. (c) M> J³B ³F
15. (c)
Flowers Conclusion I : F = J
Fruits Either
Branches Roots Conclusion II: J > J
Desks Hence, either I or II is true.
28. (a) D= M>W³R
or
Conclusion I : R < D (True)
Flowers Conclusion II : W £ D (False)
Fruits Hence, only conclusion I is true.
Desks
29. (d) A £ N < V = J

Roots Branches Conclusion I : J ³ N (False)


Conclusion II: A £ V (False)
Hence, both conclusions are false.
30. (d) K < T ³ B £ M
Conclusion I : (False)
Conclusion I : M > T (False)
Conclusion II : (False)
Conclusion II : K £ B (False)
Conclusion III : (True)
Hence, both conclusions are false.
Conclusion IV : (True)

www.newspaperkorner.wordpress.com
www.newspaperkorner.wordpress.com

SBI BANK PO PRELIM EXAM 2015 P-9

31-35 : Þ x = 2412 × 5 = `12060


\ Amount received by B and D together
F (–)
æ (5 + 13) ö
daughter-in-law =`ç
ç ´ 12060 ÷
÷
è 30 ø
C A B = ` 7236
(+) (+) (–) 39. (a) Total age of remaining 40 girls
= (80 × 20 – 20 × 22 – 20 × 24) years
= (1600 – 440 – 480) years
D E = 680 years
(+) (–) \ Required average age
680
31. (a) 32. (b) 33. (b) 34. (c) 35. (c) = = 17 years
40
S.I.´100 40. (c) Let Ms. Pooja Pushpan’s monthly salary = ` . x
36. (a) Rate = Pr incipal ´ Time
According to the question,
7200 ´100 13% of the x = ` 8554
= = 12% per annum
20000 ´ 3 æ 8554 ´100 ö
Þ x = `ç
ç ÷
÷
éæ T ù è 13 ø
R ö
\ C.I. = P êêçç1 + ú
÷ - 1ú
÷
100 ø = ` 65800
ëêè úû
Total monthly investment in percentage
éæ
12 ö
3 ù = 13 + 23 + 8 = 44
= 20000 êêçç1 + ú
÷÷ - 1ú \ Total monthly investment
êëè 100 ø úû
= 44% of `65800
= 20000 [(1.12)3 – 1]
= 20000 × (1.404928 – 1) æ 44 ´ 65800 ö
= ` çç ÷÷
= `8098.56 è 100 ø
x = ` 28952
37. (e) Let the original fraction be y . \ Total annual investment
= `(12 × 28952)
According to the question,
= ` 347424
x ´ 400 5 41. (d) Let the C.P. of the article be ` x.
=
y ´ 600 12 According to the question,
x 5 6 5 878 – x = x – 636
Þ = ´ = Þ 2x = 878 + 636 = 1514
y 12 4 8
38. (b) Let the original sum be `x. 1514
Þx= =` 757
Sum of the Ratios 2
= 3 + 5 + 9 + 13 = 30 42. (c) Number of sweets received by each student
9x = 15% of 240
\ C 's share =`
30 15 ´ 240
= = 36
3x 100
=`
10 \ Total number of sweets
= 240 × 36 = 8640
3x
A’s share = ` 43. (b) Let the total amount be ` x.
30
The amount spent = 100 – 28 = 72%
x \ 72% of x
=`
10 = ` (45760 + 27896)
According to the question,
72 ´ x
3x x Þ = 73656
- = 2412 100
10 10
73656 ´100
2x Þx=
Þ = 2412 72
10
= `102300

www.newspaperkorner.wordpress.com
www.newspaperkorner.wordpress.com

P-10 SBI BANK PO PRELIM EXAM 2015

44. (e) Let the present ages of Khushi and Jagriti be 5x and 183 × 4 + 4 × 4 = 748
8x years respectively.
After 8 years, 748 × 5 + 5 × 3 = 3755
3755 × 6 + 6 × 2 = 22542
5x + 8 3 Hence, 748 will replace the question mark.
=
8x + 8 4 51. (b) ? = 2959.85 ÷ 16.001 – 34.99
Þ 24x + 24 = 20x + 32 » 2960 ÷ 16 – 35
Þ 4x = 32 – 24 = 8 2960
» - 35 »185 - 35
8 16
Þ x= =2
4
»150
45. (d) The word PUNCTUAL consists of 8 letters in which 52. (d) ? = (1702 ÷ 68) × 136.05
the letter ‘U’ comes twice.
1700
\ Number of arrangements. » ´136
68
8! 8 ´ 7 ´ 6 ´ 5 ´ 4 ´ 3 ´ 2 ´1
= = = 20160 » 3400
2! 1´ 2
46. (d) The given number series is based on the following 2950
53. (e) ?= + 160
pattern: 12.25
93 + 2 = 95 2950
95 + 3 = 98 » + 160 » 405.8
12
98 + 5 = 103 Clearly 12,25 » 12 < 12.25
103 + 7 = 110 Hence, 2950 ÷ 12 will give larger quotient.
110 + 11 = 121 \ Our answer should be 405.
121 + 13 = 134 54. (a) ? = 25.05% 2845 + 14.95 × 2400
Hence, 103 will replace the question mark. 25
47. (e) The given number series is based on the following » ´ 2845 + 15 ´ 2400
100
pattern:
» 711.25 + 36000
8 × 1.5 = 12
» 36711.25 » 36700
12 × 1.5 = 18
186 ´ 271
18 × 1.5 = 27 55. (e) ? » »1260.15
27 × 1.5 = 40.5 40
40.5 × 1.5 = 60.75 »1260
\ ? = 60.75 × 1.5 = 91.125 56. (b) (I) p2 – 7p = – 12
Hence, 91.125 will replace the question mark. Þ p2 – 4p + 12 = 0
48. (e) The given number series is based on the following Þ p2 – 4p – 3p + 12 = 0
pattern:
4 + 7 = 11 Þ p (p – 4) –3 (p –4) = 0
11 + 7 = 18 Þ (p – 4) (p – 3) = 0
18 + 11 = 29
Þ p = 3 or 4
\ ? = 29 + 18 = 47
(II) q2 – 3q + 2 = 0
Hence, 47 will replace the question mark.
49. (a) The given number series is based on the following Þ q2 – 2q – 9 + 2 = 0
pattern: Þ q (q – 2) –1 (q – 2) = 0
3 × 2 + 22 = 10
Þ (q – 2) (q – 1) = 0
10 × 3 + 32 = 39
39 × 4 + 42 = 172 Þ q = 1 or 2
172 × 5 + 52 = 885 Obviously p > q
885 × 6 + 62 = 5346
57. (a) (I) 12p2 – 7p = – 1
Hence, 39 will replace the question mark.
50. (c) The given number series is based on the following Þ 12p2 – 7p + 1 = 0
pattern : Þ 12p2 – 4p – 3p + 1 = 0
15 × 1 + 1 × 7 = 22
22 × 2 + 2 × 6 = 56 Þ 4p (3p –1) –1 (3p –1) = 0
56 × 3 + 3 × 5 = 183 Þ (3p –1) (4p – 1) = 0

www.newspaperkorner.wordpress.com
www.newspaperkorner.wordpress.com

SBI BANK PO PRELIM EXAM 2015 P-11

1 1 Þ q = ±5
Þp= or
4 3 Obviously, p £ q,
2
(II) 6q – 7q + 2 = 0 61. (c) Production of Company A
2
Þ 6q – 4q – 3q + 2 = 0 in 2008 = 4 lakh units
Þ 2q (3q – 2) –1 (3q – 2) = 0 in 2009 = 3 lakh units
Þ (3q – 2) (2q – 1) = 0 Decrease = 4 – 3 = 1 lakh units
2 1 \ % decrease
Þq = or
3 2
1
= ´100 = 25%
Obviously, p £ q 4
58. (c) (I) p2 + 12p + 35 = 0 62. (b) Production of Company B
2
Þ p + 7p + 5p + 35 = 0 in 2013 = 7 lakh units
Þ p (p + 7) + 5 (p + 7) = 0 in 2014 = 11 lakh units.
Þ (p + 7) (p + 5) = 0 \ Required percentage
Þ p = – 5 or – 7
11
= ´100 = 157.14 »157%
(II) 2q2 + 22q + 56 = 0 7
Þ 2q2 + 14q + 8q + 56 = 0 63. (d) It is obvious from the graph.
Þ 2q (q + 7) + 8 (q + 7) = 0 64. (a) Total production of Company A
Þ (q + 7) (2q + 8) = 0 = (4 + 3 + 8 + 8 + 8 + 7 + 8 + 12) = 58 lakh units
Þ q = – 7 or – 4 = 5800000
Obviously, p £ q Total production of Company B
2
59. (d) (I) p – 8p + 15 = 0 = (1 + 1 + 1 + 2 + 3 + 5 + 7 + 11) = 31 lakh units
2
Þ p – 3p – 5p + 15 = 0 = 3100000
Þ p (p – 3) – 5 (p – 3) = 0 \ Required difference
Þ (p – 3) (p – 5) = 0 = (58 – 31) × 100000
Þ p = 3 or 5 = 2700000
2
(II) q – 5q = – 6 65. (e) Required average production of Company B
2
Þ q – 5q + 6 = 0 31
2
= lakh units
Þ q – 3q – 2q + 6 = 0 8

Þ q (q – 3) – 2 (q – 3) = 0 = 3.875 » 3.9 lakh units


Þ q (q – 3) (q – 2) = 0 66. (a) For course D
Þ q = 3 or 2 No. of girls = 30% of 800
Obviously, p ³ q. 30 ´ 800
= = 240
2 100
60. (c) (I) 2p + 20p + 50 = 0
Þ p 2 + 10p + 25 = 0 No. of students
2
Þ (p + 5) = 0 1
= 35 ´ ´1200 = 420
Þp+5=0 100

Þp=–5 \ No. of boys = 240 – 240 = 180


(II) q2 = 25 \ Required ratio = 180 : 240 = 3 : 4

www.newspaperkorner.wordpress.com
www.newspaperkorner.wordpress.com

P-12 SBI BANK PO PRELIM EXAM 2015

67. (c) Number of boys 76. (b) 77. (c) 78. (d) 79. (a) 80. (b)
in course E = 12% of 1200 – 14% of 800 = 144 – 112 81. (d) indispensable means essential, and fits the blank.
= 32 82. (d) pertinent means “relevant or applicable to the matter
at hand”. So it fits the blank.
in course F = 13% of 1200 – 14% of 800 = 156 – 112 83. (b) align means to “adjust or support”, and fits the blank.
= 44 84. (a) The correct answer here is educate. While the other
in course A = 20% of 1200 – 30% of 800 = 240 – 240 words have similar meaning, they do not make the
=0 sentence grammatically correct.
85. (b) Legal aid is “granted”. So, it fits the blank.
in course D = 35% of 1200 – 30% of 800 = 420 – 240
86. (a) Either you have to remove “comparatively” or convert
= 180
“better” to “good” in the sentence. The meaning of
in course C = 5% of 1200 – 2% of 800 = 60 – 16 = “comparatively” is - to evaluate anything using
44 comparison estimate by comparison, etc. hence you
Obviously pair C and F is our answer. should use either “better” or “comparatively good” in
order to make the sentence correct.
68. (a) For Course E, 87. (c) “Kindly” will not be used here; because “kindly” and
No. of girls = 14% of 800 “requested” are never used simultaneously “kindly”
and “please” are used in Active Voice to denote
= 112
“request”; while denote “kindly” and “please” in
No. of boys = 32 Passive Voice, “Requested” is generally used.
\ Required percentage 88. (c) The use of “it” is superfluous here, as the usage of the
Object of “bought”- the book: is correct in the
80 sentence.
= ´100 = 250
32 89. (a) Instead of “passing marks” you should use “pass
marks”, because “passing marks” is not correct
69. (d) 90. (a) You should use “neighbours” in stead of
70. (b) No. of girls in course C “neighbourers” because, there is no word in English
like “neighbourers”. The meaning of “neighbours” is
= 2% of 800 = 16
people who live or are situated nearby.
(71 to 75) : The Correct order is:- C D A B F E 91. (d) 92. (c) 93. (d) 94. (d) 95. (c) 96. (a)
71. (e) 72. (c) 73. (d) 74. (a) 75. (b) 97. (b) 98. (a) 99. (d) 100. (c)

www.newspaperkorner.wordpress.com
www.newspaperkorner.wordpress.com

SBI PO Exam Solved Paper


Held On : 21-06-2014
Max. Marks : 200 (Based on Memory) Time : 2 hrs.

REASONING (HIGH LEVEL) 6. How many persons are standing exactly between I and O?
(a) Three (b) Four
Directions (Qs.1-2) : Study the following information carefully (c) One (d) Two
and answer the questions given below : (e) None
There are five statues - L, M, N, O and P - each of them having 7. Who among the following is to the immediate left of H?
different height. Statue L is smaller than only statue M. Statue O (a) O (b) J
is smaller than statue N. Statue O is longer than statue P. The (c) I (d) L
height of the tallest statue is 20 feet. The height of the second (e) K
smallest statue is 11 feet. 8. Four of the following five are alike in a certain way based
1. What Will be the height of statue P? on the above arrangement and hence form a group. Which
(a) 13 feet (b) 15 feet one of the following does not belong to that group?
(c) 9 feet (d) 12 feet
(a) N (b) L
(e) 14 feet
(c) O (d) I
2. What Will be the height of the third tallest statue?
(e) K
(a) 13 feet (b) 10 feet
9. Who among the following is exactly between L and J?
(c) 19 feet (d) 9 feet
(a) N (b) O
(e) 11 feet
3. lf the expressions S = T > O ³ R and P £ O < Z are true, (c) H (d) I
then which of the following is not true? (e) None
(a) T > O (b) R < S 10. Who among the following is fourth to the right of J?
(c) Z > R (d) P > T (a) N (b) I
(e) P < Z (c) H (d) L
Directions (Qs.4-10) : Study the following information carefully (e) K
and answer the questions given below : Directions (Qs.11-15) : In these questions, relationship between
Eight persons - H, I, J, K, L, M, N and O - are standing in a different elements is shown in the statements.
straight line at equidistant. Some of them are facing north while The statements are followed by two conclusions.
others are facing south. M is standing third to the right to H. M is Give answer (a) if only Conclusion I is true.
standing at one of the extreme ends. L is standing third to the left Give answer (b) if only Conclusion II is true.
to H. The immediate neighbours of J face north. N is not an Give answer (c) if either Conclusion I or II is true.
immediate neighbour of H. The persons standing at the extreme Give answer (d) if neither Conclusion I nor II is true.
ends face the same direction (Both are facing either north or Give answer (e) if both Conclusions I and II are true.
South). The immediate neighbours of H face just opposite direction 11. Statements :
as that of M. The immediate neighbours of O face opposite B>C=D³X;E£X;Z³D
directions with respect to each other. One of the immediate Conclusions : I . B > E
neighbours of L is K who is facing north. I is standing between J II . Z ³ B
and M. Not more than four persons are facing north. 12. Statements:
4. Who among the following is third to the left of N? E>F³G<H£I<J
(a) K (b) J Conclusions : I . G £ E
(c) H (d) I II . J ³ F
(e) O 13. Statements:
5. The immediate neighbours of L are: K £ L < M > N ³ O; T > M £ P
(a) M and N (b) N and O Conclusions: I . T > K
(c) K and N (d) N and H
II . P > O
(e) J and H

www.newspaperkorner.wordpress.com
www.newspaperkorner.wordpress.com

2014- 2 SBI PO EXAM 2014 SOLVED PAPER

14. Statements : 18. In which direction point 'A' is located with respect to point
B > O = K ³ L; D > K ³ S 'B'?
Conclusions : I. O < D I. A man starts walking from point 'A' towards east and
II. S £ L after walking 3 metres reaches point 'N', he turns right
15. Statements : and walks 7 metres to reach point 'M'. Then he turns
B > O = K ³ L; D > K ³ S right and walks 6 metres to reach point 'O'. He again
Conclusions : I. L > D turns right and walks 7 metres to reach point 'P'. He,
II. B > S
then, turns left and walks 2 metres to reach point 'B'.
Directions (Qs.16-20): Each of the questions below consists of a
II. A man starts walking from point 'A' towards east and
question and two statements numbered I and II given below it.
after walking 3 metres reaches point 'N'. From point
You have to decide whether the data provided in the statements
are sufficient to answer the question. Read both the statements 'N' he walks 7 metres towards south and reaches point
and— 'M'. From point 'M' he walks 6 metres towards west
Give answer (a) if the data in Statement I alone are sufficient to and reaches Point 'O'. From point 'O' he walks 7 metres
answer the question, while the data in Statement II alone are not towards north and reaches point 'P'. From point 'P' he
sufficient to answer the question. walks towards west and reaches point 'B'. The distance
Give answer (b) if the data in Statement II alone are sufficient to between points A and B is 8 metres.
answer the question, while the data in Statement I alone are not 19. How many students scored more than B in a class of 25
sufficient to answer the question. students?
Give answer (c) if the data either in Statement I alone or in I. More than four but less than ten students scored more
Statement II alone are sufficient to answer the question. than that of B. B's rank is an odd number. Seventeen
Give answer (d) if the data even in both Statements I and II students scored less than D.
together are not sufficient to answer the question. II. The rank of C is 16th from the last. B got more marks
Give answer (e) if the data in both Statements I and II together than C. Only two students are there between B and C.
are necessary to answer the question. 20. Five persons - A, B, C, D and E - are sitting around a circular
16. What is the code of 'shine' in a certain code language ?
table. Some of them are facing towards centre while others
I. In that code language shine was peeled off is written
are facing outside.
as '& # @ 9' and 'no paint but shine' is written as '7 5 # 8'.
II. In that code language 'try the new shine' is written as Who is sitting second to the left of A?
'13 # 0' and 'we try the new' is written as '6013'. I. B is facing outside. C is to the immediate left of B. D
17. How C is related to H? is second to the right of C. A is to the immediate left of
I. N is son of H. J is mother of Z. N and Z are cousins. C E.
is husband of J. II. D is to the immediate right of A. Both D and A are
II. L is father of C. A is mother of D. L is married to A. H facing towards the centre. D and B are immediate
is wife of D. J is wife of C. neighbours of each other.
Directions (Qs.21-27) : Study the following information carefully and answer the questions given below :
A word and number arrangement machine when given an input line of words and numbers rearranges them following a particular rule
in each step. The following is an illustration of input and various steps of rearrangement. (All the numbers are two digit numbers).
Input : sweet 46 nice 36 friend 26 help 96 bright 76 kind 66
Step I : sweet 46 nice 36 friend 26 help bright 76 kind 66 96
Step II : sweet nice 46 36 friend 26 help bright kind 66 76 96
Step III: sweet nice kind 46 36 friend 26 help bright 66 76 96
Step IV: sweet nice kind help 36 friend 26 bright 46 66 76 96
Step V : sweet nice kind help friend 26 bright 36 46 66 76 96
Step VI: sweet nice kind help friend bright 26 36 46 66 76 96
And Step VI is the last step of the rearrangement as the desired arrangement is obtained.
As per rules followed in the above steps, find out in each of the questions the appropriate step for the given input.
Input : arrow 98 paint 58 lamb 38 each 78 great 18 most 48 rent 88
21. Which word/number would be fifth to the left of the sixth 23. How many elements (words/numbers) are there between
element from the right in the step V? "most" and "78" as they appear in the Step VI?
(a) great (b) arrow (a) Eight (b) Seven
(c) lamb (d) 38 (c) Nine (d) Five
(e) 48 (e) Four
22. Which of the following represents the position of "58" in 24. Which step number is the following output?
the step IV? rent paint most arrow 58 lamb 38 each great 18 48 78 88 98
(a) Eighth from left (b) Third from right (a) There is no such step (b) Step II
(c) Ninth from left (d) Eleventh from left (c) Step V (d) Step VI
(e) Fifth from right (e) Step III

www.newspaperkorner.wordpress.com
www.newspaperkorner.wordpress.com

SBI PO EXAM 2014 SOLVED PAPER 2014- 3

25. Which element (word/number) would be at the eleventh 32. Starting from R, if all the persons are made to sit in the
position from the right in the Step III? alphabetical order in clockwise direction, the positions of
(a) lamb (b) arrow how many (excluding R) will remain unchanged?
(c) 58 (d) 38 (a) Two (b) Three
(e) each (c) Four (d) One
26. Which element (word/number) would be at the sixth position (e) None
from the left in the Step VI ?
33. Who among the following is sitting fourth to the left of the
(a) 18 (b) arrow
person who studies in Standard II?
(c) great (d) each
(e) 38 (a) U (b) W
27. At which of the following positions "great" would appear (c) X (d) S
from the left in the Step V ? (e) Cannot be determined
(a) Fifth (b) Sixth 34. Who among the following is sitting exactly between the
(c) Fourth (d) Second person who studies in Standard III and W?
(e) Third (a) T (b) U
Directions (Qs. 28-34): Study the following information carefully (c) R (d) V
and answer the questions given below: (e) Y
Eight friends -R, S, T, U, V, W, X and Y - are sitting around a Directions (Qs. 35-40) : Study the following information carefully
circular table facing the centre, but not necessarily in the same and answer the questions given below :
order. Each of the them studies in the different Standards viz, A college planned to conduct an education fair. The college asked
standards I to Standard VIII, but not necessarily in the same order. foreign universities to participate in the education fair so that
T is second to the right of the person who studies in standard VII. students could know about the foreign universities. Even foreign
Only one person sits between T and the person who studies in universities can tell which course is good in which university.
standard V. X is sitting third to the left of the person who studies The courses offered by these universities are–Social Science,
in Standard VIII. The person studying in Standard VIII is not an
Psychology, Medicine, Commerce, Astronomy, Computer Science
immediate neighbour of the person studying in Standard VII. T
and Architecture. The fair will start on Monday and will end on
does not study in Standard VIII. The person studying in Standard
VI to the left of U. U does not study in Standard V or Standard Sunday of the same week. Social Science stall will be set up
VIII. The persons studying in Standard VI and VII are immediate immediately before the Medicine stall. Medicine stall will not be
neighbours of each other. One of the immediate neighbours of the set up on Tuesday or Thursday. There will be a gap of one day
person studying in Standard VIII, studies in Standard II. S and Y between the stalls of Astronomy and Medicine. Architecture stall
are immediate neighbours of each other. There is one person will be set up immediately after Astronomy stall. Computer
between S and R. V does not study in Standard I. R studies in Science stall will be set up on Monday.
Standard V. Y does not study in Standard II. W is sitting between 35. Which of the following stalls will be set up on Thursday ?
the person who study in Standard VII and Standard IV. X is second (a) Astronomy (b) Psychology
to the right of the person studying in Standard I. (c) Medicine (d) Architecture
28. Who among the following is sitting third to the right of T. (e) None of these
(a) S 36. Which of the following stalls will be set up immediately
(b) The person studying in Standard II after the Computer Science stall?
(c) The person studying in Standard V (a) Medicine (b) Commerce
(d) Y (c) Psychology (d) Social Science
(e) The person studying in Standard I (e) None of these
29. Who among the following is sitting just opposite to 'R'? 37. On which of the following days of the week the stall on
(a) The person studying in Standard VI Architecture will be set up?
(b) W
(a) Saturday (b) Tuesday
(c) The person studying in Standard V
(c) Thursday (d) Friday
(d) S
(e) None of these
(e) The person studying in Standard VII
30. Which of the following pairs represents the immediate 38. Which of the following pairs of stalls will be set up between
neighbours of 'V'? the stalls on Computer Science and Psychology?
(a) R and person studying in Standard VIII (a) Social Science and Astronomy
(b) S and the person studying in Standard III (b) Medicine and Astronomy
(c) The person studying in Standard V and Y (c) Social Science and Medicine
(d) Cannot be determined (d) Medicine and Commerce
(e) None of these (e) None of these
31. What is the position of 'W' with respect to 'S'? 39. Which of the following stalls will be set up on Friday?
(a) Fourth to the right (b) Fourth to the left (a) Commerce (b) Medicine
(c) Third to the right (d) Fifth to the right (c) Psychology (d) Architecture
(e) Sixth to the left (e) None of these

www.newspaperkorner.wordpress.com
www.newspaperkorner.wordpress.com

2014- 4 SBI PO EXAM 2014 SOLVED PAPER

40. Which of the following stalls will be set up on Wednesday? (a) Only Conclusion I follows
(a) Medicine (b) Architecture (b) Only Conclusion II follows
(c) Psychology (d) Social Science (c) Either Conclusion I or Conclusion II follows
(e) None of these (d) There is possibility that some stars are triangles.
Directions (Qs. 41-45) : Below in each question are given two (e) Neither Conclusion I nor Conclusion II follows.
statements (A) and (B). These statements may be either 47. Statements :
independent causes or may be effects of independent causes or a All drums are banjos
common cause. One of these statements may be the effect of the Some drums are guitars
other statement. Read both the statements and decide which of No banjo is a flute
the following answer choice correctly depicts the relationship Conclusions :
between these two statements. I. Some guitars are flutes
Mark answer (a) if statement (A) is the cause and statement (B) II. No guitar is flute
is its effect. (a) Only Conclusion I follows
Mark answer (b) if statement (B) is the cause and statement (A) (b) Only Conclusion II follows
is its effect. (c) There is possibility that some flutes are banjos
Mark answer (c) if both the statements (A) and (B) are (d) Either Conclusion I or Conclusion II follows
independent causes. (e) Both Conclusions I and II follows
Mark answer (d) if both the statements (A) and (B) are effects 48. Statements :
of independent causes. Some pins are needles.
Mark answer (e) if both the statements (A) and (B) are effects of All needles are swords
some common cause. Some swords are knives.
41. (A) A substantial increase in the unhealthy competition has Conclusions :
been observed among the students. I. All swords being pins is a possibility
(B) A rise of 23% is reported every year in the cases of II. No needle being knife is a possibility.
suicides after declaration of grade 10th and 12th (a) Only Conclusion I follows
examination results.
(b) Only Conclusion II follows
42. (A) The glaciers at the poles of the earth are melting at a
(c) Either Conclusion I or Conclusion II follows
fast rate.
(d) Neither Conclusion I nor Conclusion II follows
(B) In recent times there has been a substantial increase in
(e) There is possibility that some pins are knives.
the incidents of earthquakes and volcanic eruptions.
49. Statements :
43. (A) Most of the shopkeepers in the locality closed their
Some schemes are offers.
shops for the second continuous day.
Some offers are discounts
(B) Two groups of people living in the locality have been
No discount is a loan
fighting with each other with bricks and stones forcing
Conclusions :
people to stay indoors.
I. Those offers which are discounts can never be loans.
44. (A) The Government has decided to increase the prices of
II. Some loans are definitely schemes
LPG gas cylinders with immediate effect.
(a) Only Conclusion I follows
(B) The Government has decided to increase the prices of
(b) There is possibility that all schemes are loans
kerosene with immediate effect.
45. (A) The country 'X' has banned the import of fruit. (c) Only Conclusion II follows
(B) The intake of that fruit causes disease. (d) Either Conclusion I or Conclusion II follows
Directions (Qs.46-50) : In each question below are given three (e) Neither Conclusion II nor Conclusion II follows
statements followed by two conclusions numbered I and II. You 50. Statements :
have to assume everything in the statements to be true even if No car is hotel
they seem to be at variance from commonly known facts and then All lodges are hotels
decide which of the given conclusions logically follows from the No lodge is house.
information given in the statements. Conclusions :
46. Statements : I. Some houses are not lodges
No star is a cone. II. No lodge is car
Some cones are triangles. (a) There is possibility that all houses are cars
All kites and stars. (b) Only Conclusion I follows
Conclusions : (c) Only Conclusion II follows
I. All stars are kites. (d) Neither Conclusion I nor Conclusion II follows
II. At least some triangles are stars. (e) Both Conclusion I and Conclusion II follows

www.newspaperkorner.wordpress.com
www.newspaperkorner.wordpress.com

SBI PO EXAM 2014 SOLVED PAPER 2014- 5

DATA ANALYSIS AND INTERPRETATION


Sales Revenue Cost of Production
Directions (Qs. 51-57) : Study the following information carefully 1000 (In thousand `)
and answer the questions. 900
800 800
For a room, the rate of painting is ` 3200 per square metre. The 800 750 725

(In thousand `)
rate of carpeting per square metre is 120% of that of tiling. The 700
625 600
cost of decorating the room is 14 times to that of carpeting on the 600 550
500 500 525
floor. The cost of electrification is 75% of that of carpeting the 500 450
floor. The rate of tiling on the floor is 125% of that of painting. 400 400
The dimensions of the room are 6m × 6m × 5m. 300
51. What is the ratio of the cost of painting the four walls of the 200
room and that of decoration?
100
(a) 10 : 63 (b) 10 : 61
0
(c) 10 : 21 (d) 21 : 10 2004 2005 2006 2007 2008 2009
(e) None of these YEARS
52. What will be the total cost of decorating the room and tiling
58. By what per cent is the amount received from the sales in
the floor when the four walls have also been tiled to a height
the year 2005 of the company, more than the expenditure
of 0.25 metre?
on production in the year 2008?
(a) ` 5287200 (b) ` 2587200
(c) ` 2588200 (d) ` 2577200 (a) 45% (b) 40%
(e) None of these (c) 48% (d) 49%
53. What will be the total cost of painting, carpeting, decoration (e) 50%
and electrification of the room if the dimensions of the room 59. In how many years the cost of production is more than the
be 21 m × 42 m × 27m ? average cost of production of the given years ?
(a) ` 30888000 (b) ` 8388000 (a) 1 (b) 2
(c) ` 80388000 (d) ` 40888000 (c) 3 (d) 4
(e) None of these (e) None of these
54. What will be the cost of tiling the floor of the room if the 60. In how many years the sales revenue is less than the average
rate of tiling be increased by 75% and the dimensions of the sales revenue of the given years?
room be 51m × 59m × 84m ? (a) 1 (b) 3
(a) ` 21163000 (b) ` 1263000 (c) 2 (d) 4
(c) ` 2163000 (d) ` 21063000 (e) None of these
(e) None of these 61. If the cost of production in 2004 and 2007 be increased by
55. If the length of the room be increased by 20%, breadth by 25% and 30% respectively, then by what per cent will the
32% and height by 12%, then what will be the total cost of total cost in these both years be more than the sales revenue
painting of the four walls of the room and tiling the floor? of the year 2008 ?
(a) ` 76996.80 (b) ` 67996.80 (a) 62.7% (b) 65.7%
(c) ` 67796.80 (d) ` 76796.80 (c) 67.7% (d) 68.5%
(e) None of these (e) None of these
56. What will be the total cost of carpeting, decoration, 62. In which year the company had maximum profit per cent ?
electrification and tiling the floor if the rate of painting be (a) 2008 (b) 2007
doubled? (c) 2006 (d) 2005
(a) ` 5771200 (b) ` 5731200 (e) 2004
(c) ` 5371200 (d) ` 7571200 63. If the sales revenues in the years 2005, 2006 and 2007
(e) None of these increase by 20%, 25% and 30% respectively and the costs
57. What will be the cost of decoration of the room if its cost of production in the years 2007, 2008 and 2009 increase by
be increased by 25%? 20%, 25% and 35% respectively, what will be the difference
(a) ` 60480 (b) ` 604800 between average sales revenue and average cost of
(c) ` 6048000 (d) ` 6448000 production?
(e) None of these (a) ` 185.59 thousands
Directions (Qs.58-63) : In the following multiple bar diagram, (b) ` 188.59 thousands
the data regarding cost of production and sales revenue of the (c) ` 174.59 thousands
company XYZ in the given years have been given. Study the bar (d) ` 200.59 thousands
diagram carefully and answer the questions.
(e) None of these

www.newspaperkorner.wordpress.com
www.newspaperkorner.wordpress.com

2014- 6 SBI PO EXAM 2014 SOLVED PAPER

Directions (Qs.64-69) : In the following multiple graphs 69. What was the total production of wheat by these three states
production of wheat (in quintals) by three states - Bihar, Madhya in the year 2007? (in quintals)
Pradesh and Punjab have been given. Study the following graphs (a) 9900 (b) 9700
carefully to answer the questions. (c) 9980 (d) 8800
(e) None of these
Bihar Madhya Pradesh Punjab
4000 Directions (Qs. 70-75) : In the following bar diagram the number
3900 3800 3800 of engineers employed in various companies has been given. Study
3800 3700 the bar diagram carefully to answer the questions.
3700 3600
3600 3500
Production of wheat (in quintals)

3500 3400 Post Graduate Engineers


3100

3400 3300 Assistant Engineers


3300 3200
3200 Junior Engineers
2900

3100 3000
3000 1000
2900 2900

Number of Engineers
2900 2800 900
2800 2900 2900
800
2700 2800
2600 2700 700 650
2500 600
2400 500
2300 2400 2400 400
2200 400
2100 2100 300
200
2001 2002 2003 2004 2005 2006 2007
Years 100
0
64. If the production of wheat by Madhya Pradesh in the years V W X Y Z
2003, 2004, 2005 and 2007 increase by 30%, 40%, 45% Companies
and 40% respectively, what will be the overall percentage
70. The number of post graduate engineers employed in the
increase in the production of wheat in the state in the given
company W is what per cent of the total engineers employed
years?
in that company ?
(a) 22% (b) 25%
(c) 35% (d) 16% 1 1
(a) 33 % (b) 30 %
(e) 19% 3 3
65. What was the average production of wheat by all three states
in the year 2005? (in quintals) 1 1
(c) 25 % (d) 36 %
3 3
1 2
(a) 2866 (b) 2866 (e) None of these
3 3 71. What is the average number of junior engineers employed
2 1 in all the companies?
(c) 2688 (d) 2688 (a) 150 (b) 170
3 3
(c) 160 (d) 180
(e) None of these
(e) 190
66. In the given years, what is the average production of wheat
72. What is the difference between the average number of junior
in Bihar? (in quintals)
engineers and assistant engineers taking all the companies
(a) 3068 (b) 3076
together?
(c) 3086 (d) 3088
(a) 18 (b) 15
(e) None of these
(c) 10 (d) 22
67. If the productions of wheat in Bihar in the years 2001, 2002,
(e) 25
2003 and 2004 increase by 20%, 25%, 28% and 35%
73. If the number of assistant engineers employed in all the
respectively; what will be the percentage increase in the
companies be increased by 37% and the number of post
average production of the state for the given years?
graduate engineers employed in all the companies b
(a) 35.7% (b) 38.7%
decreased by 20%, by what percent will the number of
(c) 40.7% (d) 42.5%
assistant engineers be less than that of post graduate
(e) None of these
engineers?
68. By what per cent is the total production of wheat by three
(a) 5.6% (b) 7.8%
states in the year 2002, 2003 and 2004 more or less than
(c) 8% (d) 9.3%
that in the years 2005, 2006 and 2007?
(e) None of these
(a) 2.5% (b) 2.6%
74. If the numbers of all the engineers in the company V,
(c) 1.9% (d) 1.09%
company X and company Y be increased by 30%, 35% and
(e) None of these

www.newspaperkorner.wordpress.com
www.newspaperkorner.wordpress.com

SBI PO EXAM 2014 SOLVED PAPER 2014- 7

40% respectively, what will be the overall percentage 80. If an increase of 40% is made in the average number of
increase in the number of all engineers of all the companies female employees, working in countries III, IV and V, then
taken together? their resulting average number will be what per cent of the
(a) 20% (b) 22% average number of female employees?
(c) 24% (d) 25% (a) 83% (b) 85%
(e) None of these (c) 65% (d) 68%
75. What is the ratio between the number of assistant engineers (e) 69%
employed in company V and company X? 81. By what per cent is the total number of employees in
(a) 3 : 5 (b) 3 : 4 countries V, VI and VII more than the number of male
(c) 2 : 3 (d) 3 : 2 employees working in the countries II, III and IV?
(e) None of these (a) 50.2% (b) 53.6%
Directions (Qs.76-81) : In the following pie charts the percentage (c) 55% (d) 48%
of employees of a company working in 8 different countries has (e) 49%
been given. Study these pie-charts carefully to answer the Directions (Qs.82-87) : In the following table, information
questions. regarding publishing books by different publishers has been given.
Total number of employees = 86000 Study the table carefully and answer the questions.
Publishing Number of Ratio of Percentage Number of
VIII I Houses Books Academic of books distributors in
8% 18%
VII published and non- distributed publishing
7%
II academic house
VI 12% books
16%
III M 28200 7: 3 81 17
V IV 8%
17% 14%
N 32200 5: 9 74 23
O 29700 6: 5 92 18
P 31200 8: 5 86 24
Number of male employees = 56000
Q 33800 7: 6 79 25
R 35700 11 : 6 82 21
VII 11%
VIII S 37800 5 : 13 89 24
VI 9% I
7% 15% 82. What is the difference between the number of academic
II books published by publishing houses M and P ?
V
18%
10% (a) 450 (b) 640
III (c) 540 (d) 504
IV 12%
18% (e) None of these
83. How many books were given to each distributor by publisher
Q if each publisher gets equal number of books ?
Note: Roman digits show the countries. (a) 1806 (b) 1068
76. What is the ratio between male employees working in the (c) 1608 (d) 1308
country I and country II respectively? (e) None of these
(a) 3 : 4 (b) 5 : 4 84. What is the average number of non-academic books
(c) 5 : 3 (d) 3 : 2 published by publisher R and S?
(e) 2 : 3 (a) 18750 (b) 18850
77. What is the ratio between the number of male employees (c) 19950 (d) 18950
and female employees in country II? (e) 19990
(a) 70 : 53 (b) 70 : 31 85. What is the total number of books distributed by publishers
(c) 70 : 59 (d) 53 : 70 O and Q?
(e) None of these (a) 26702 (b) 27324
78. What is the approximate average number of male employees (c) 55026 (d) 54026
in countries I, II and III ? (e) None of these
(a) 9670 (b) 6970 86. If the total number of books published by publishers P, Q
(c) 6907 (d) 6977 and R is increased by 30% and the total number of books
(e) None of these published by remaining publishers be decreased by 20%,
79. What is the average number of female employees in the what will be the new average of books published by all the
countries IV and VII? publishers ?
(a) 1370 (b) 1070 (a) 33418 (b) 33318
(c) 1570 (d) 1470 (c) 32518 (d) 33618
(e) None of these (e) None of these

www.newspaperkorner.wordpress.com
www.newspaperkorner.wordpress.com

2014- 8 SBI PO EXAM 2014 SOLVED PAPER

87. The ratio of the number of books published by publishers P (a) 300 (b) 315
and Q is (c) 328 (d) 345
(a) 156 : 139 (b) 156 : 169 (e) 360
(c) 3 : 5 (d) 5 : 17 Directions (Qs. 96-100) : Study the pie-charts carefully to answer
(e) None of these the following questions.
Directions (Qs. 88-95) : In the following table, the number of Percentage of students enrolled in different activities in a
candidates appeared and qualified for different posts in an institute school
during various years has been given. Read the table carefully and N = 3000
answer the questions.
Description of appeared - qualified candidates

Years 2001 2002 2003 2004 2005 2006 Dancing Craft


Po s ts A pp . Qu al. A p p . Qu al. A p p. Qual. A p p . Qu al. A p p . Qu al. A p p. Qual. 24% 25%
A 242 157 345 190 330 240 387 309 433 390 483 417
B 387 310 427 280 410 380 417 354 456 412 339 265 Singing Drawing

Sw 16%
C 378 302 411 280 520 415 340 221 381 355 335 290
21% 14%

imm
D 420 364 577 315 487 290 490 480 512 487 410 370
E 392 370 380 316 250 180 430 416 447 423 381 351

ing
F 509 412 480 423 390 322 227 190 329 279 370 337
Note : A pp . - A pp eared Qu al. - Qu alified

88. What is the approximate average number of candidates who Percentage break-up of girls enrolled in these activities out
qualified for all the posts in the year 2002 ? of the total students.
(a) 300 (b) 280 n = 1750
(c) 290 (d) 310
(e) 314
89. What is the approximate average number of candidates who Dancing Craft
appeared for all the posts in the year 2006 ? 20 % 22%
(a) 366 (b) 389
(c) 386 (d) 375 Singing Drawing
(e) 390 28% Sw 14% 16%
90. The difference between the number of candidates who
imm
appeared and qualified for the post D in the years 2001,
ing

2002 and 2003 is


(a) 615 (b) 605
(c) 601 (d) 515 96. Number of girls enrolled in Dancing form what per cent of
(e) 505 total number of students in the school? (rounded off to two
91. What approximate percentage of candidates qualified for digits after decimal)
the post of A in the year 2003? (a) 12.35 (b) 14.12
(a) 73% (b) 70% (c) 11.67 (d) 10.08
(c) 68% (d) 69% (e) None of these
(e) 75% 97. How many boys are enrolled in Singing and Craft together?
92. The respective ratio between the number of candidates who (a) 505 (b) 610
qualified for the posts of A and D in the year 2002 is (c) 485 (d) 420
(a) 67 : 38 (b) 38 : 63 (e) None of these
(c) 38 : 67 (d) 19 : 31 98. What is the respective ratio of number of girls enrolled in
(e) None of these Swimming to the number of boys enrolled in Swimming ?
93. In which year maximum number of candidates were selected (1) 47 : 49 (b) 23 : 29
for the post of A? (c) 29 : 23 (d) 49 : 47
(a) 2003 (b) 2004 (e) None of these
(c) 2005 (d) 2006 99. What is the total number of girls enrolled in Swimming and
(e) 2002 Drawing together ?
94. In which year minimum number of candidates qualified for (a) 480 (b) 525
the post of F? (c) 505 (d) 495
(a) 2002 (b) 2003 (e) None of these
(c) 2004 (d) 2006 100. What is the approximate percentage of boys in the school?
(e) 2005 (a) 34 (b) 56
95. The average number of candidates who qualified for all the (c) 28 (d) 50
posts in the year 2004 is (e) None of these

www.newspaperkorner.wordpress.com
www.newspaperkorner.wordpress.com

SBI PO EXAM 2014 SOLVED PAPER 2014- 9

(d) Assembly, Compilation. Linking


GENERAL AWARENESS, MARKETING &
(e) Compilation, Linking, Assembly
COMPUTER 110. Which of the following is the proper removal of any
101. Banks with higher proportion of demand deposits will have malicious program from a computer System?
(a) Low cost money at its disposal (a) Reconfiguration (b) Reboot
(b) Low interest money at its disposal (c) Restart (d) Uninstallation
(c) Higher number of current and savings accounts (e) Only (b) and (d)
(d) High amount of liquid money 111. What is the secured messaging system device used by
(e) All of the above IDBRT in the field of banking'?
102. Which commercial bank of India recently became the first (a) NEFT (b) SFMS
to launch Mobile Branch with ATM. titled 'Branch on (c) RTGS (d) EFT
Wheels'? (e) All of the above
112. Which of the following public sector banks recently became
(a) State Bank of India (b) Axis Bank
the first to launch the Instant Money Transfer (IMF) scheme
(c) ICICI Bank (d) ABN Amro Bank
which is a innovative domestic money remittance facility
(e) Punjab National Bank
from ATMs without an account in the bank?
103. To which of the following crops does the Rashtriya Krishi
(a) State Bank of India (b) Axis Bank
Bima Yojana (NAIS) provide insurance?
(c) Punjab National Bank (d) Bank of India
(a) Food crops (cereals and pulses)
(e) Bank of Baroda
(b) Oilseeds
113. Which of the following does not come under the second
(c) Horticultural crops
level of management in an organization?
(d) Commercial crops
(a) Purchase Managers (b) Departmental Heads
(e) All of the above (c) Supervisors (d) Branch Managers
104. According to the Reserve Bank of India which of the (e) Finance Managers
following contribute to more than three-fourth of the total 114. What is the process of copying software programs from
Non- Performing Assets (NPAs)? secondary storage media to the hard disk called?
(a) Public Sector bank (b) Foreign Bank (a) Configuration (b) Download
(c) Private Bank (d) Regional Rural Bank (c) Storage (d) Upload
(e) Co-operatives (e) Installation
105. Who has been selected for the Gandhi Peace Prize for the 115. In the field of banking, what does CRAR stand for?
year 2013? (a) Capital Reserve Adequacy Ratio
(a) Chandi Prasad Bhatt (b) Desmond Tutu (b) Capital to Risk (Weighted) Assets Ratio
(c) Medea Benjamin (d) Julius Nyerere (c) Capital Reserve Assets Ratio
(e) Gopalkrishna Gandhi (d) Capital Risk Adequacy Ratio
l06. What is the term for the cheque issued by a person to another (e) None of these
person in which the issuing date is after a few days (a date 116. Which of the following terms /expressions is used to describe
which is yet to come)? a business unit with a competitive advantage that will capture
(a) Anti-dated cheque (b) Stale cheque some of the extra economic values it creates, no matter how
(c) Post-dated cheque (d) Crossed cheque intense competition is?
(e) Order cheque (a) Consonance (b) Advantage
107. Which of the following characterizes the top level (c) Consistency (d) Feasibility
management of corporates? (e) All of the above
(a) Spend more time in co-ordinating and communicating 117. Which of the following commercial banks of India recently
(b) Executes (implements) the policies and plans opened its branch in Shanghai?
(c) Determine the objectives, policies and plans of the (a) State Bank of India
organisation (b) Axis Bank
(d) Spend more time in directing and controlling (c) ICICI Bank
(e) Coordinate the activities of all the departments (d) Oriental Bank of Commerce
108. Through which of the following ways can devices be (e) Corporation Bank
connected to a computer'? l18. Which of the following come under Policy Framework for
(a) USB (b) Bluetooth Investment Decisions as outlined by the OECD?
(c) CD/DVD Drive (d) RAM (a) Policy coherence
(e) Only (a) and (b) (b) Importance of transparency approach to policy
109. Which of the following is the correct sequence of formulation
compilation? (c) Regular evaluation of policies on investment
(a) Compilation, Assembly, Linking environment
(b) Assembly, Linking, Compilation (d) Corporate governance
(c) Linking, Assembly, Compilation (e) All of the above

www.newspaperkorner.wordpress.com
www.newspaperkorner.wordpress.com

2014-10 SBI PO EXAM 2014 SOLVED PAPER

119. What is the corpus of the National Social Security Fund? 130. President Pranab Mukherjee on the occasion of international
(a) ` 250 crore (b) ` 500 crore Women's Day presented which award to six individuals for
(c) ` 1000 crore (d) ` 1200 crore their contribution for the social development and
(e) ` 1500 crore empowerment of women in the country?
120. We regularly come across some lines marked on the back of (a) Stree Shakti Puraskar award
products. What are they called? (b) Nirbhaya Puraskar
(a) Bar-code (b) Spooler (c) Mother Teresa Award
(c) Scanner (d) Output reader (d) Florence Nightingale Award
(e) Price tag (e) Ahilyabai Puraskar
121. In the field of computers, which of the following converts 131. What is the goal in Two Fold Marketing?
input into output? (a) To attract new customers by promising superior value
(a) Mouse (b) Trackball (b) To keep current customers by delivering satisfaction
(c) Keyboard (d) Bar Code Reader (c) To undertake aggressive marketing campaigns
(e) Light Pen (d) Only (a) and (b)
122. Which application is used for the purposes of basic (e) All of the above
accounting? 132. How can one see the content of a file?
(a) Worksheet (b) Spreadsheet (a) By left clicking with the mouse once
(c) infographic (d) Data sheet (b) By left clicking with the mouse twice
(e) Document (c) By holding the mouse down
123. Which insurance company has launched Asha Kiran, an (d) From the keyboard shortcuts
health insurance cover, for a family with girl children? (e) All of the above
(a) Oriental Insurance comp. Ltd. 133. What is it called when computer connects to other computer
(b) Reliance General insurance and sends information?
(c) United India Insurance Comp. Ltd. (a) LAN (b) Remote Desktop
(d) New India Assurance comp. Ltd. (c) Boot (d) Network
(e) National Insurance Company (e) intemet Protocol
124. Who is the author of 'India at Risk'? 134. How does every bank pay interest on savings?
(a) Stephen Cohen (b) Jaswant Singh (a) Daily (b) Weekly
(c) Tony Alcott (d) Lal Krishna Advani (c) Fortnightly (d) Monthly
(e) Manmohan Singh (e) Quarterly
125. Which of the following was/ were not the subsidiary 135. What is the alphanumeric code on cheques called ?
company of the General Insurance Corporation of India? (a) RTGS (b) MICR
(a) National Insurance Company Limited (c) IFSC (d) NEFT
(b) Life Insurance Corporation (e) CC
(c) New India Assurance Company Limited 136 How can a file be saved in a computer with other name ?
(d) Orienta1 Insurance Company Limited (a) Save command (b) Save as command
(e) United India Insurance Company Limited (c) Place command (d) Duplicate command
126. Which of the following countries will participate in the (e) Export command
NATO 2014 Summit? 137. Which is the new airline company to get licence recently in
(a) Russia (b) China India ?
(c) Germany (d) India (a) Indigo (b) Spice Jet
(e) Australia (c) Air Asia (d) Kingfisher
127. Market Segmentation can be defined as (e) Go Air
(a) Segment, Target, Price 138. How instrument is used by the Reserve Bank of India to
(b) Sales, Target, Product control market liquidity?
(c) Segment, Target, Position (a) Repo rate
(d) Sales, Time, Product (b) Statutory Liquidity Ratio
(e) Service, Sales, Segment (c) Marginal Adjustment Facility
128. Which of the following provides power back to a computer (d) Reserve Repo Rate
system even when main current is off? (e) Only (a) and (d)
(a) UPS (b) Inverter 139. Banking Ombudsman is appointed by
(c) CMOS battery (d) Soft Power (a) Union Ministry of Finance
(e) Motherhoard Power Connector (b) Competition Commission of India
129. Which team won the Asia Cup 2014? (c) Reserve Bank of India
(a) India (b) Pakistan (d) Indian Banks Association
(c) Sri Lanka (d) Bangladesh (e) Securities and Exchange Board of India
(e) Both (a) and (c)

www.newspaperkorner.wordpress.com
www.newspaperkorner.wordpress.com

SBI PO EXAM 2014 SOLVED PAPER 2014-11

140. Which state in India has the highest urban population? 150. Which of the following come under the ambit of Customer
(a) Maharashtra (b) Delhi relationship management?
(c) Tamil Nadu (d) Karnataka (a) Sales (b) Marketing
(e) Gujarat (c) Customer Service (d) Technical support
141. What was the reason behind the decision of the Reserve (e) All of the above
Bank of India to withdraw currency notes issued before
ENGLISH LANGUAGE
2005?
(a) To replace them with plastic notes Directions (Qs. 151-160) : Read the following passage carefully
(b) to check counterfeit notes and answer the questions given below it. Certain words/phrases
(c) To replace mutilated and spoilt notes have been printed in bold to help you locate them while answering
(d) To check illicit notes of Pakistan some of the questions.
(e) None of these India is rushing headlong toward economic success and
142. Which film was presented the award of Best film at the 61st modernisation, counting on high-tech industries such as
National Awards ? information technology and biotechnology to propel the nation to
(a) Ship of Theseus (b) Bhaag Milkha Bhaag prosperity. India's recent announcement that it would no longer
(c) Jolly LLB (d) December 1 produce unlicensed inexpensive generic pharmaceuticals bowed
(e) Yellow to the realities of the World Trade Organisation while at the same
143. Which of the following is the fastest mode of transaction? time challenging the domestic drug industry to compete with the
(a) RTGS (b) Cheque payment multinational firms. Unfortunately, its weak higher education
(c) ATM Cash Withdrawal (d) NEFT sector constitutes the Achilles' Heel of this strategy. Its systematic
(e) None of these disinvestment in higher education in recent years has yielded
144. A marketing technique where marketer plays a specific role neither world-class research nor very many highly trained scholars,
in a particular segment is called scientists, or managers to sustain high-tech development.
(a) Mass Marketing India's main competitors especially China but also Singapore,
(b) Niche Marketing Taiwan, and South Korea are investing in large and differentiated
(c) Strategic Marketing higher education systems. They are providing access to large
(d) Communication Marketing number of students at the bottom of the academic system while at
(e) None of these the same time building some research-based universities that are
145. In Marketing Mix 4 P's imply able to compete with the world's best institutions. The recent
(a) Product, Price, Place, Promotion London Times Higher Education Supplement ranking of the
(b) Product, Price, Policy, Place world's top 200 universities included three in China. three in Hong
(c) Place, People, Product, Promotion Kong. Three in South Korea, one in Taiwan, and one in India (an
(d) People, Price, Profit, Place Indian institute of Technology at number 41- the specific campus
(e) None of these was not specified). These countries are positioning themselves
146. Which aspect of a project/ business is/are evaluated in a for leadership in the knowledge-based economies of the coming
SWOT Analysis? era.
(a) Strengths (b) Weaknesses There was a time when countries could achieve economic success
(c) Opportunities (d) Threats with cheap labour and low-tech manufacturing. Low wages still
(e) All of these help, but contemporary large-scale development requires a
147. Which of the following best explains the Delphi Method ? sophisticated and at least partly knowledge-based economy. India
(a) It is a profit-maximization method has chosen that path, but will find a major stumbling block in its
(b) It is a forecasting method university system.
(c) It is a retrenchment method India has significant advantages in the 21st century knowledge
(d) It is a output enhancement method race. It has a large higher education sector - the third largest in the
(e) It is a risk analysis method world in student numbers, after China and the United States. It
148. In Market segmentation which among the following is not uses English as a primary language of higher education and
an economic component? research. It has a long academic tradition. Academic freedom is
(a) Age (b) Gender respected. There are a small number of high quality institutions,
(c) Income Level (d) Taxes departments, and centres that can form the basis of quality sector
(e) Only (a) and (b) in higher education. The fact that the States, rather than the Central
149. What does Innovation in Marketing refer to ? Government, exercise major responsibility for higher education
(a) Creating a new idea of thought creates a rather cumbersome structure, but the system allows for
(b) Discovering new products a variety of policies and approaches.
(c) Modifying an old idea to present needs Yet the weaknesses far out-weigh the strengths. India educates
(d) Exploring new ways of being competitive approximately 10 per cent of its young people in higher education
(e) All of the above compared with more than half in the major industrialised countries
and 15 per cent in China. Almost all of the world's academic

www.newspaperkorner.wordpress.com
www.newspaperkorner.wordpress.com

2014-12 SBI PO EXAM 2014 SOLVED PAPER

systems resemble a pyramid. With a small high quality tier at the place to begin and sustain the process. India will need to create a
top and a massive sector at the bottom. India has a tiny top tier. dozen or more universities that can compete internationally to
None of its universities occupies a solid position at the top. A few fully participate in the new world economy. Without these
of the best universities have some excellent departments and universities, India is destined to remain a scientific backwater.
centres, and there is a small number of outstanding undergraduate 151. Which of the following statement(s) is/are correct in the
colleges. The University Grants. Commission's recent major context of the given passage ?
support of five universities to build on their recognised strength I. India has the third largest higher education sector in
is a step toward recognising a differentiated academic system – the world in student numbers.
and fostering excellence. At present, the world-class institutions II. India is moving rapidly toward economic success and
are mainly limited to the Indian Institutes of Technology (IITs), modernisation through high tech industries such as
the Indian Institutes of Management (IIMs) and perhaps a few information technology and biotechnology to make the
others such as the All India Institute of Medical Sciences and the nation to prosperity
Tata Institute of Fundamental Research. These institutions, III. India's systematic disinvestment in higher education
combined, enroll well under 1 per cent of the student population. in recent years has yielded world class research and
India's colleges and universities, with just a few exceptions, have many world class trained scholars, scientists to sustain
become large, under-funded, ungovernable institutions. At many high-tech development.
of them, politics has intruded into campus life, influencing (a) Only I (b) Only II
academic appointments and decisions across levels. Under- (c) Both I and II (d) Both I and Ill
investment in libraries, information technoloy, laboratories, and (e) All I, II and III
classrooms makes it very difficult to provide top-quality instruction 152. Which of the following statements in regard to the
or engage in cutting-edge research. information given in the passage is not true ?
The rise in the number of part-time teachers and the freeze on (a) The London Times Higher Education Supplement
new full-time appointments in many places have affected morale ranking of the world's top 200 universities has included
in the academic profession. The lack of accountability means that three universities of South Korea.
teaching and research performance is seldom measured. The (b) India has recently announced not to produce unlicensed
system provides few incentives to perform. Bureaucratic inertia inexpensive generic pharmaceuticals that will be a
hampers change. Student unrest and occasional faculty agitation challenge for the domestic drug industry to compete
disrupt operations. Nevertheless, with a semblance of normality, with the multinational firms.
faculty administrators are able to provide teaching, coordinate (c) Contemporary large-scale development requires a
examinations, and award degrees. sophisticated and at least partly knowledge-based
Even the small top tier of higher education faces serious problems.
economy.
Many IIT graduates, well trained in technology, have chosen not
(d) China has the fourth largest higher education sector in
to contribute their skills to the burgeoning technology sector in
the world.
India. Perhaps half leave the country immediately upon graduation
(e) None of these
to pursue advanced study abroad – and most do not return. A
153. According to the view expressed by the writer in the passage,
stunning 86 per cent of students in science and technology fields
what is a step toward recognising a differentiated academic
from India who obtain degrees in the United States do not return
system and fostering excellence ?
home immediately following their study, another significant group,
(a) The University Grant Commission's recent major
of about 30 per cent, decides to earn MBAs in lndia because local
support to five universities to build on their strength.
salaries are higher – and are lost to science and technology. A
corps of dedicated and able teachers work at the IITs and IIMs, (b) New Education policy of the new government.
but the lure of jobs abroad and in the private sector make it (c) Scholarships granted by the Central government for
increasingly difficult to lure the best and brightest to the academic research.
profession. (d) Government policy to open new world class institutions
Few in India are thinking creatively about higher education. There (e) None of these
is no field of higher education research. Those in government as 154. In writer's opinion which of the following reason(s) is/are
well as academic leaders seem content to do the “same old thing.” responsible for poor higher education in India?
Academic institutions and systems have become large and I. India's colleges and universities, with some exceptions,
complex. They need good data, careful analysis, and creative ideas. have become large under funded, ungovernable
In China, more than two-dozen higher education research centres, institutions.
and several government agencies are involved in higher education II. Politics has intruded into many compuses that
policy. influences academic appointments and decisions across
India has survived with an increasingly mediocre higher education levels.
system for decades. Now as India strives to compete in a globalised III. Under investment in libraries, laboratories, IT and
economy in areas that require highly trained professionals, the classrooms hinder cutting edge research.
quality of higher education becomes increasingly important. (a) Only I (b) Both 1 and II
India cannot build internationally recognised research-oriented (c) Both II and III (d) All I. II and III
universities overnight, but the country has the key elements in (e) None of these

www.newspaperkorner.wordpress.com
www.newspaperkorner.wordpress.com

SBI PO EXAM 2014 SOLVED PAPER 2014-13

155. Which of the following statements is not true as per the The world's fastest growing region remains home to nearly half
given information in the passage ? the world’s extreme poor. While Asia has made tremendous
(a) About fifty per cent of IIT graduates leave India to inroads in the fight against poverty, not enough of the region's
pursue advanced study abroad. economic prosperity is reaching its poorest people.
(b) About 86 per cent of students in science and technology In urban areas of China, for example, the Gini coefficient (a
fields from India who obtain degrees in US do not return measurement used to calculate inequality) has risen more than
home following their study. 35% since 1990. Nearly half a billion Asians still lack access to
(c) In China more than two-dozen higher education safe drinking water and infant mortality in many nations is more
research centres and several government agencies are than 10 times higher than the levels seen in developed economics.
involved in higher education policy. While "Factory Asia" may be true for manufacturing and
(d) The rise in the number of part-time teachers and the information technology services, vast number of its people are
freeze on new full-time appointments in many places illiterate and unemployed. Its financial sector is underdeveloped,
have boosted morale in academic profession in India. with many people having no access to simple banking, let alone
(e) None of these other financial services. Asia's future prosperity, and the
156. What in your opinion should be an appropriate title of the eradication of extreme poverty, will require much more than simply
given passage ? high growth. Growth must be accompanied by a narrowing of
(a) Poor state of Higher Education in India inequality.
(b) Politics in India's Education system It is essential to balance the region's economic expansion with
(c) Modernisation of Indian Education System more inclusive policies. Cut off by poor r oads,
(d) Higher Education Supplement Ranking telecommunications, or government policies that don't allow them
(e) None of these to easily borrow or save, Asia's poor and vulnerable are watching
Directions (Qs.157-158) : Choose the word/group of words which the chasm between rich and poor grow ever wider. That gap in
is most similar in meaning to the word/group of words printed in prosperity can aggravate simmering social, economic and
bold as used in the passage. political tensions.
157. Achi1les' Heel Asian governments can help stem widening inequality by creating
(a) weakness (b) strength better conditions for the private sector to take the lead on economic
(c) acquiescene (d) vulnerable expansion, continuing to promote economic diversification, and
(e) strong heel by spending on social services, education and healthcare, and
158. Sustain regional road, sea and air networks that will open more
(a) suffer (b) maintain opportunities to more people.
(c) swag (d) swallow There are areas where western governments can help too. By
(e) weaken investing in infrastructure alongside public lenders, they can help
Directions (Qs.159-160) : Choose the word/ group of words which attract much larger sums from the private sector. Asia can also
is most opposite in meaning to the word / group of words printed capitalise on financial lessons from the west, particularly when it
in bold as used in the passage. comes to setting banking regulations, strengthening regional links,
159. Cumbersome and promoting bonds to better utilise Asian savings.
(a) complicated (b) complex When describing where Asia stands today, it's useful to remember
(c) simple (d) bulky that what we are witnessing is not the emergence of Asia, but
(e) heavy rather the re-emergence of Asia.
160. Differentiated In 1820, Asia accounted for about 60% of total global output,
(a) Distinguished (b) similar with China and India together accounting for nearly half of global
(c) distinct (d) undistinguished GDP. This was followed by nearly two centuries of economic
(e) distraught decline once the western industrial evolution took hold – a trend
Directions (Qs. 161-170) : Read the following passage carefully that, since the information age, has been solidly reversed. By
and answer the questions given below it. Certain words/ phrases implementing structural reforms and opening their economics.
have been printed in bold to help you locate them while answering China and others have rapidly emerged as engines of the global
some of the questions. economy.
Asia's rapid rise is the most successful story of economic The recent Asian Development Bank study suggests that we could
development in recent history. Income per capita reached nearly see Asia producing over half of global GDP by mid-century, and
$5.000 in purchasing power parity terms in 2010. Investment rates 3 billions Asians would be considered part f the rich world, with
averaged 35% of GDP over the decade. The number of people capita income levels equal to that of Europe today.
living below the $1.25-a-day poverty line fell by 430 million Carefully calibrated government support can help steer Asia's
between 2005 and 2010. economic potential, reducing political risks while opening new
With such achievements at a time when much of the rest of the markets to help move the west beyond the current crisis. In the
world struggles with austerity measures and economic recovery. long run, an Asian economy built on sustainable growth can
Asian leaders might be tempted to switch to autopilot. But closer support greater levels of trade, and generate growth in tourism.
examination of the region’s economic and social prospects soon Conversely, a weaker Asia presents a host of threats to the west's
reveals many paradoxes. future growth and prosperity.

www.newspaperkorner.wordpress.com
www.newspaperkorner.wordpress.com

2014-14 SBI PO EXAM 2014 SOLVED PAPER

Embracing globalisation and regional co-operation has helped III. "Factory Asia" can make vast number of its people
bring developing Asia success. By further strengthening this literate and employed.
process by focusing on greater access and inclusion within (a) Only II (b) Only III
economics, and pursuing sustainable development and improved (c) Both 1 and III (d) Both II and III
governance, an "Asian century" is both plausible and within reach. (e) Both I and II
But policies that worked when Asia was low-income and capital 165. What suggestions has the author of this passage made that
scarce are less likely to work today and unlikely to work in the can help Asian governments stem widening inequality in
future. Asia's leaders must devise bold and innovative national
the region ?
policies while pursuing regional and global co-operation. Long
I. By creating better conditions for the private sector to
term prosperity will depend on the intensity of resource use,
including water and food, and success in managing the region's take the lead on economic expansion.
carbon footprint. II. By continuing to promote economic diversification.
Asia's challenges remain for-midable, and its future prosperity III. By spending on social services, education and
must be earned. The right policy choices today could indeed make healthcare and regional road, sea and air networks that
this the "Asian century", but this is far from preordained. will open more opportunities to more people.
161. Which of the following statement (s) is/are correct in the (a) Only I (b) Both II and III
context of the passage? (c) Both I and II (d) Both I and Ill
I. Asia's per capita income reached approximately $5000 (e) All I, II and III
in purchasing power parity terms in 2010. 166. Which of the following statements is not correct as per the
II. The number of people living below the $1.25 a day given information in the passage ?
poverty line in Asia fell by 430 million between 2005 (a) According to the Asian Development Bank study. Asia
and 2010. can produce over half of global GDP by mid-century.
III. In 1820, Asia accounted for about 60% of total global (b) In 1820. China and India together accounted for nearly
output. half of global GDP.
(a) Only I (b) Only II (c) The author opines the necessity to balance the Asian
(c) Both I and II (d) Both II and III
region's economic expansion with more inclusive
(e) All I, II and III
policies.
162. What in your opinion should be the appropriate title of the
(d) The widening gap between rich and poor people can
given passage?
(a) Asia's Economic Rise and Paradoxes aggravate simmering social, economic and political
(b) Asia's Rise and Europe's Re-emergence tensions.
(c) Economic Backwardness of Asia (e) The world's fastest growing Asian region remains home
(d) Developed and Developing Regions of the world to more than sixty five per cent of the world's extreme
(e) None of these poor.
163. Which of the following is not true as per the given Directions (Qs.167-168) : Choose the word/group of words which
information in the passage? is most similar in meaning to the word/ group of words printed in
(a) In urban areas of China, the Gini coefficient that is a bold as used in the passage.
measurement used to calculate inequality has risen 167. Austerity
more than 45% since 1990. (a) auscultation (b) decoration
(b) Nearly half a billion Asians still lack access to safe (c) extravagance (d) spendthrift
drinking water. (e) simplicity
(c) The infant mortality rate in many Asian nations is more 168. Inroads
than 10 times higher than the levels seen in developed (a) Noticeable effect (b) making road
economics.
(c) insecurities (d) inquest
(d) Investment rates in Asia averaged 35 per cent of Gross
(e) identification
Domestic Product over the decade
Directions (Qs.169-170) : Choose the word/group of words which
(e) Embracing globalisation and regional cooperation has
helped bring developing Asia success. is most opposite in meaning to the word / group of words printed
164. In the author's opinion what are the areas where western in bold as used in the passage.
government can help Asia boost economic development ? 169. Aggravate
I. By investing in infrastructure alongside public lenders. (a) worsen (b) irritate
western governments can help attract much larger sums (c) aggregate (d) assuage
from the private sector. (e) astonish
II. Asia can capitalise on financial lessons from the west, 170. Simmering
Particularly when it comes to setting banking (a) seething (b) calm
regulations, strengthening regional links and promoting (c) boiling (d) simulating
bonds to better utilise Asian savings. (e) exhilarating

www.newspaperkorner.wordpress.com
www.newspaperkorner.wordpress.com

SBI PO EXAM 2014 SOLVED PAPER 2014-15

Directions (Qs.171-175) : In each of the following questions four (c) introspection-achieved


words are given of which two words are most nearly the same or (d) search-found
opposite in meaning. Find the number of correct letter (e) cross-inspection - made
combination. 180. Education is an essential means of–women with the
171. (A) dominate (B) radical knowledge, skills and self-confidence necessary to fully—
(C) determined (D) monopolise in the development process.
(1) A - B (2) B - C (a) empower- include
(3) A - D (4) B - D (b) empowering - participate
(5) C - D (c) empowered - participating
172. (A) critical (B) equitable (d) empowerment - participate
(C) impartial (D) unearth (e) strengthening - participate
(1) A - B (2) B - C Directions (Qs.181-185) : Rearrange the following six sentences
(3) A - D (4) B - D (A), (B), (C), (D), (E) and (F) in the proper sequence to form a
(5) C - D meaningful paragraph; then answer the questions given below
173. (A) shining (B) raise them.
(C) flourish (D) thrive (A) As a consequence, even if it is plausible that ambient
(1) A - B (2) B - C air pollution plays a role for the onset and increasing
(3) C - D (4) B - D frequency of respiratory allergy, it is not easy to prove
(5) A - C this conclusively.
174. (A) Affable (B) rude (B) Another factor clouding the issue is that laboratory
(C) pacify (D) cajole evaluations do not reflect what happens during natural
(1) A - B (2) B - C exposure when atmospheric pollution mixtures are
(3) C - D (4) A - D inhaled.
(5) B - D (C) Interpretation of studies are confounded by the effect
175. (A) energetic (B) partial of cigarette smoke. exposure to indoor pollutants and
(C) diffuse (D) dispassionate to outdoors and indoors allergens.
(1) A - B (2) B - C (D) However, despite evidence of a correlation between
(3) C - D (4) A - D the increasing frequency of respiratory allergy and the
(5) B - D increasing trend in air pollution, the link and interaction
Directions (Qs.176-180) : Pick out the most effective pair of is still speculative.
words from the given pair of words make the sentences (E) Allergic respiratory diseases such as hay fever and
meaningfully complete. bronchial asthma have indeed become more common
176. Weather officials have __ below-normal rains this year. in the last decades in all industrialized countries and
If the predictions come true, farm output could __ as most the reasons for this increase are still debated.
of India's farmlands depend on rainwater for irrigation. (F) Several studies have shown the adverse effects of
(a) forecasted-shrank (b) forecast-shrank ambient air pollution on respiratory health.
(c) forecast-shrink (d) predicted-expand 181. Which of the following should be the LAST sentence after
(e) predictions-wan rearrangement ?
177. It is the role of the state to ____ crime and protect people (a) A (b) B
and property. If the state is unable to prevent a crime it falls (c) C (d) D
upon the state to __ the victim. (e) E
(a) prevent-support (b) preventing-encourage 182. Which of the following should be the FIRST sentence after
(c) prevent-supporting (d) forbid-discourage rearrangement ?
(e) forbid-discouraging (a) A (b) B
178. A person who is clean and tidy in how he dresses up (c) C (d) D
commands better–from those around him than those "who (e) E
have a–and unkempt appearance— 183. Which of the following should be the THIRD sentence after
(a) respectful - slovenly (b) respect - slovenly rearrangement?
(c) respected - untidy (d) respect - tidy (a) A (b) B
(e) respect - careful (c) C (d) D
179. Today we have achieved a milestone by completing 60 years (e) E
of independence. It's now the time for everyone or every 184. Which of the following should be the FOURTH sentence
Indian to undergo–of the achievements we already made after rearrangement ?
and also those that are to be still— (a) A (b) B
(a) self-introspection-achiver (c) C (d) D
(b) self-examination-achieve (e) E

www.newspaperkorner.wordpress.com
www.newspaperkorner.wordpress.com

2014-16 SBI PO EXAM 2014 SOLVED PAPER

185. Which of the following should be the SECOND sentence cultural practices which routinely discriminate them from having
after rearrangement? equal opportunities in education, health and livelihood? These
(a) A (b) B women are invisible and the (196) in their way prevent them from
(c) C (d) D accessing the most basic human rights and needs. The outlook is
(e) E bleak.
Directions (Qs.186-190) : Read each sentence to find out whether Women make up 70% of the world's working hours and (197)
there is any grammatical error in it. The error if any, will be in one only 10% of the world's income and half of what men earn. This
part of the sentence, the number of that part is the answer. If there leads to greater poverty, slower economic growth and a (198)
is no error, mark (e). (Ignore errors of punctuation. if any) standard of living. In developing countries, millions of women
186. In the first two months of this fiscal, tractor sales has seen also die each year as a result of gender-based (199). This deep-
(a)/a drop of about five percent (b)/ however, the industry is rooted gender discrimination creates a bleak outlook for women
waiting for the monsoon (c)/ to really arrive at a firm in developing countries. For millions of girls living in poverty, it
conclusion about growth prospects for the current year. (d)/ is often those closest to them who work against the child's interests
No error (e) and their immediate environment is often (200) and sometimes,
187. Dolphins are truly out of the ordinary because of their down-right harmful. Parents arrange marriages when you are a
intelligence (a) / and. among the many creatures that share child. Neighbours say, if you are a girl, you must limit your
the earth form (b)/they come closest to humankind in terms activities to your home. Friends say, it is OK not to go to school.
of (c)/familial traits, emotions and learining. (d)/ No error 191. (a) made (b) make
(e) (c) look (d) has made
188. Corruption indulged in by the high and mighty adversely (e) complete
impacts (a)/ our nation, and in the coming months (b) / we 192. (a) equality (b) affection
may see revival of efforts (c)/ to tackle such large scale (c) inequality (d) support
corruption. (d)/ No error (e) (e) justice
189. It is notable and welcome that the ministry of (a)/ 193. (a) vulnerable (b) weakness
environmental and forests is to issue approvals online (b)/ (c) vulnerability (d) specific
in a time bound manner, with clear timelines (c)/in place for (e) weakest
the various sub-steps along the way. (d)/No error (e) 194. (a) tackle (b) tackling
190. To portray (a)/ what a fairness cream does without (b)/ any (c) fight (d) tackled
sort of comparison or visual (c)/ references are very difficult. (e) fought
(d)/ No error (e) 195. (a) increase (b) support
Directions (Qs. 191-200) : In the following passage there are (c) weak (d) reduce
blanks, each of which has been numbered. These numbers are (e) influencing
printed below the passage and against each, five words are 196. (a) obstacles (b) make
suggested, one of which fits the blank appropriately. Find out the (c) pleasantries (d) pebbles
appropriate word in each case. (e) encouragements
Women (191) up half the world's population and yet represent a 197. (a) earns (b) carried
staggering 70% of the world's poor. We live in a world in which (c) spend (d) earn
women living in poverty face gross (192) and injustice from birth (e) spends
to death. From poor education to poor nutrition to (193) and low 198. (a) slow (b) slower
pay employment, the sequence of discrimination that a woman (c) low (d) lowest
may suffer during her entire life is unacceptable but all too (e) lower
common. 199. (a) violence (b) violent
Millennium Development Goal 3 is to promote gender equality (c) virulent (d) corruption
and empower women. This MDG is critical for (194) poverty and (e) violation
improving prospects for women. But how can women break gender 200. (a) functional (b) natural
based stereotypes to minimise discrimination and (195) gender (c) artificial (d) dysfunctional
based violence when they are trapped in societies with socio- (e) disfunctional

www.newspaperkorner.wordpress.com
www.newspaperkorner.wordpress.com
on
SBI PO EXAM 2014 SOLVED PAPER 2014-17

ANSWER KEY
1. (c) 2. (a) 3. (d) 4. (b) 5. (c) 6. (d) 7. (a) 8. (e) 9. (b) 10. (d)
11. (a) 12. (d) 13. (e) 14. (a) 15. (b) 16. (c) 17. (c) 18. (c) 19. (b) 20. (e)
21. (c) 22. (d) 23. (a) 24. (e) 25. (b) 26. (d) 27. (a) 28. (b) 29. (e) 30. (a)
31. (c) 32. (d) 33. (b) 34. (a) 35. (b) 36. (d) 37. (a) 38. (c) 39. (e) 40. (a)
41. (a) 42. (d) 43. (b) 44. (e) 45. (b) 46. (d) 47. (d) 48. (a) 49. (d) 50. (e)
51. (a) 52. (b) 53. (c) 54. (d) 55. (a) 56. (b) 57. (c) 58. (a) 59. (b) 60. (c)
61. (a) 62. (a) 63. (c) 64. (a) 65. (b) 66. (c) 67. (b) 68. (d) 69. (a) 70. (a)
71. (b) 72. (c) 73. (d) 74. (b) 75. (b) 76. (b) 77. (c) 78. (c) 79. (a) 80. (a)
81. (b) 82. (c) 83. (b) 84. (c) 85. (d) 86. (b) 87. (b) 88. (a) 89. (c) 90. (d)
91. (a) 92. (b) 93. (d) 94. (c) 95. (c) 96. (c) 97. (a) 98. (d) 99. (b) 100. (e)
101. (e) 102. (c) 103. (e) 104. (a) 105. (a) 106. (c) 107. (c) 108. (e) 109. (a) 110. (e)
111. (b) 112. (d) 113. (c) 114. (e) 115. (b) 116. (b) 117. (b) 118. (e) 119. (c) 120. (a)
121. (d) 122. (b) 123. (d) 124. (b) 125. (b) 126. (c) 127. (c) 128. (e) 129. (c) 130. (a)
131. (d) 132. (b) 133. (b) 134. (a) 135. (c) 136. (b) 137. (c) 138. (e) 139. (c) 140. (a)
141. (b) 142. (a) 143. (a) 144. (b) 145. (a) 146. (e) 147. (b) 148. (e) 149. (a) 150. (e)
151. (c) 152. (d) 153. (a) 154. (d) 155. (d) 156. (a) 157. (a) 158. (b) 159. (c) 160. (d)
161. (e) 162. (a) 163. (a) 164. (e) 165. (e) 166. (e) 167. (e) 168. (a) 169. (d) 170. (b)
171. (c) 172. (b) 173. (c) 174. (a) 175. (e) 176. (c) 177. (a) 178. (b) 179. (c) 180. (b)
181. (a) 182. (e) 183. (d) 184. (c) 185. (e) 186. (a) 187. (b) 188. (e) 189. (b) 190. (d)
191. (b) 192. (c) 193. (a) 194. (b) 195. (d) 196. (a) 197. (d) 198. (e) 199. (a) 200 (d)

Answers &
Explanations
For Qs 1 and 2. For Qs 4 - 10:
Statue L is smaller than only statue M. Therefore, M is the tallest
statue. NORTH
N>O>P
Now,
M>L>N>O>P

K L N O H J I M
20 feet 11 feet
SOUTH
1. (c) Statue P is the smallest. The height of the second
smallest statue O is 11 feet. Therefore, the height of 4. (b) N is facing south. Therefore, J is third to the left of N.
the statue of P will be less than 11 feet. Hence, (c) is 5. (c) The immediate neighbours of L are K and N.
the correct option. 6. (d) Two persons - H and J are standing between I and O.
2. (a) The tallest statue is 20 feet high. Therefore, the second 7. (a) O is to the immediate left of H.
tallest statue may be 19 feet, 18 feet ..... high. 8. (e) Except K, all others are facing south. K faces north.
Therefore, the third tallest statue will be less than 19 9. (b) O is sitting exactly between L and J.
feet and more than 11 feet high. 10. (d) L is to the fourth of J.
3. (d) S = T > O ³ R (Given) 11. (a) B>C=D³X³E
P £ O < Z (Given) B>C=D£Z
Therefore, S = T > O ³ P Conclusions:
Z>O³R I. B > E: True
Hence, P£O³R II. Z ³ B: Not True
P is smaller than T.

www.newspaperkorner.wordpress.com
www.newspaperkorner.wordpress.com

2014-18 SBI PO EXAM 2014 SOLVED PAPER


12. (d) E > F ³ G < H £ I < J It is clear from the diagram that Point A is to the east of
Conclusions: Point B.
I. G £ E : Not True From statement II
II. J ³ F : Not True From the information given in statement II, we get the
13. (e) K £ L < M > N ³ O
same diagram given above.
K£L<M<T
P³M>N³O 19. (b) From statement I
Conclusions: Five to nine students scored more than that of B. The
I. T > K: True rank of D is 8th from the top.
II. P > O: True From statement II
(14-15): Rank of C from the beginning
B>O=K³L = 25 – 16 + 1 = 10th
D>K³S
7th 10th
B>O=K³S
D>K=O³L B E
S£O=K³L Thus, six students scored more than that of B.
14. (a) Conclusions: 20. (e) From statement I
I. O < D: True
II. S £ L : Not True B
15. (d) Conclusions:
I. L > D: Not True
II. B > S: Not True D
16. (c) From Statement I : C
shine was peeled off Þ & # @ 9
no paint but shine Þ 7 5 # 8 A/E
The code for shine is # A/E
From Statement II : From statement II
try the new shine Þ 13 # 0
B
we try the new Þ 6 0 1 3
The code for shine is # D
17. (c) From statement I :
H C J (–) C/E
(+)
N Z A
(+) C/E
Hence, C is the brother of H.
From Statement II. From both the statements
L A
(+) (–) B

J D D
C
(–) (+) (+) (–)
Hence, C is brother-in-law of H. C
18. (c) From statement I
2m 3m A
B P A N E

C is sitting second to the left of A.


7m

7m 21-27: After careful analysis of the input and various


steps of rearrangement, it is evident that in each step
one word and one number are rearranged. The word
O M which comes last in the dictionary order is moved to
6m the extreme left position while the highest number is
North moved to the extreme right position in the Step I. In the
next step, the word which comes at the second position
in dictionary order is placed at the second position from
West East the left and the second highest number is placed at the
second position from the right. Such procedure is
continued till all the words get rearranged in dictionary
South

www.newspaperkorner.wordpress.com
www.newspaperkorner.wordpress.com

SBI PO EXAM 2014 SOLVED PAPER 2014-19

order but in reverse manner and all the numbers get 33. (b) V studies in Standard II. W is sitting fourth to the right
rearranged after the words in ascending order. of V.
Input: arrow 98 paint 58 lamb 38 each 78 great 18 most 34. (a) X studies on Standard III. T is sitting between X and
48 rent 88 W.
Step I: rent arrow paint 58 lamb 38 each 78 great 18 35-40:
most 48 88 98
Step II: rent paint arrow 58 lamb 38 each 78 great 18
most 48 88 98 Day Course
Step III: rent paint most arrow 58 lamb 38 each great Monday Computer Science
18 48 78 88 98
Step IV: rent paint most lamb arrow 38 each great 18 Tuesday Social Science
48 58 78 88 98 Wednesday Medicine
Step V: rent paint most lamb great arrow 38 each 18
48 58 78 88 98 Thursday Psychology
Step VI: rent paint most lamb great each arrow 18 38 Friday Astronomy
48 58 78 88 98.
Saturday Architecture
21. (c) 5th to the left of the sixth from the right means 11th
from the right. Sunday Commerce
11th from the right in the Step V Þ lamb
22. (d) The position of ‘58’ in Step IV is eleventh from the left 35. (b) The stall of Psychology will be set up on Thursday.
and fourth from right. 36. (d) The stall of Social Science will be set up immediately
23. (a) There are eight elements (lamb, great, each, arrow, 18, after the stall of Computer Science.
38, 48, 58) between “most” and “78” in the Step VI. 37. (a) The stall of Architecture will be set up on Saturday.
24. (e) This is Step III.
38. (c) The stalls of Social Science and Medicine will be set up
25. (b) 11th from the right in Step III is “arrow”.
between the stalls of Computer Science and Psychology.
26. (d) In the Step VI, “each” is at the sixth position from the
left. 39. (e) The stall of Astronomy will be set up on Friday.
27. (a) In Step V, “great” is at the fifth position from the left. 40. (a) The stall of Medicine will be set up on Wednesday.
28-34: 41. (a) Clearly statement (A) is the cause and statement (B) is
its effect.
IV
T 42. (d) Clearly, both the statements (A) and (B) are effects of
III I independent causes.
X W 43. (b) Clearly statement (B) is the cause and statement (A) is
its effect.
VR U VII 44. (e) Both the statements (A) and (B) are effects of some
common cause.
45. (b) Clearly statement (B) is the cause and statement (A) is
V Y its effect.
II S VI
VIII
Cones
28. (b) V is sitting third to the right of T. V studies in Standard
VII. Stars
29. (e) The person studying in Standard VII is sitting just Triangles
opposite to R. U studies in Standard VII. 46. (d) Kites Cones
30. (a) R and S are immediate neighbours of V. R studies in Triangles
Standard V and S studies in Standard VIII.
31. (c) W is third to the right of S. W is fifth to the left of S.
Conclusions:
T I. False
S T II. False
X From the above diagram it can say that there is
W U possibility that some stars are triangles.

32. (d) R R U V Banjos Flute

Drums
V Y Guitars
Y W 47. (d)
S
Flute
X

www.newspaperkorner.wordpress.com
www.newspaperkorner.wordpress.com

2014-20 SBI PO EXAM 2014 SOLVED PAPER

Conclusions: Calculations (51-57):


I. False Rate of painting per sq. metre = ` 3200
II. False Either Rate of tiling per square metre
3200 ´125
Swords =
100
Pins = ` 4000
Needles
48. (d) Knives Rate of carpeting per sq. metre
4000 ´ 120
=
100
OR 51. (a) Area of the four walls of the room
= 2 × 5(6 + 6) = 120 sq. metre
Swords \ Expenditure on painting
= ` (120 × 3200)
Pins Area of the floor = 6 × 6 = 36 sq. metre
Needles
\ Expenditure on decoration
= ` (36 × 4800 × 14)
Knives \ Required ratio
= (120 × 3200) : (36 × 4800 × 14)
Conclusions: = 384000 : 2419200 = 10 : 63
I. False 52. (b) Area of tiles on the walls
II. False = 2 × 0.25 (6 + 6) = 6 sq. metre
\ Required total cost = `
[(36 + 6) × 4000 + 36 × 4800 × 14]
nt
cou Loan = 168000 + 2419200
Dis
= ` 2587200
53. (c) Area of the four walls of room
49. (e) Schemes Loan
Offers Discount = 2 × 27 (21 + 42) = 3402 sq. metre
Area of the floor
= 21 × 42 = 882 sq. metre
Conclusions: Painting Þ (3402 + 882) × 3200
I. False = ` 13708800
II. False Carpeting Þ 4800 × 882
= ` 4233600
Hotels Decoration Þ 4233600 × 14
= ` 59270400
House Lodges Car
50. (e) 4233600 ´ 75
Electrification Þ
100
= ` 3175200
OR \ Total expenditure
= ` (13708800 + 4233600 + 59270400 + 3175200)
= ` 80388000
Hotels
54. (d) Area of the floor
Lodges = 51 × 59 = 3009 sq. metre
Car
4000 ´ 175
Rate of tiling =
100
= ` 7000
House \ Total cost = ` (3009 × 7000)
= ` 21063000
55. (a) New length of the room
Conclusions:
I. True 6 ´ 120
= = 7.2 metre
II. True 100

www.newspaperkorner.wordpress.com
www.newspaperkorner.wordpress.com

SBI PO EXAM 2014 SOLVED PAPER 2014-21

61. (a) New production cost in the year 2004


6 ´ 132
Breadth = = 7.92 metre 625 ´ 125
100
=
100
5 ´ 112
Height = = 5.6 metre = ` 781.25 thousand
100 New production cost in the year 2007
Area of the four walls
= 2 × height (length + breadth) 400 ´ 130
=
= 2 × 5.6 (7.2 + 7.92) 100
= 2 × 5.6 × 15.12 = ` 520 thousand
= 169.344 sq. metre \ Total cost = 781.25 + 520
Area of the floor = 7.2 × 7.92 = ` 1301.25 thousand
= 57.024 sq. metre \ Required percentage
Painting Þ 3200 × 169.344
æ 1301.25 - 800 ö
= ` 541900.8 =ç ÷ ´100
Tiling Þ 57.024 × 4000 è 800 ø
= ` 228096 501.25 ´100
Total cost = ` (54190.8 + 228096) = = 62.7%
800
56. (b) Carpeting Þ 6 × 6 × 9600 = ` 345600
Decoration Þ 36 × 9600 × 14 = ` 4838400 62. (a) Percentage profit:

345600 ´ 75 æ 750 - 625 ö


Electrification Þ Year 2004 Þ ç ÷ ´100 = 20%
100 è 625 ø
= ` 259200 æ 750 - 500 ö
Tiling Þ 6 × 6 × 8000 Year 2005 Þ ç ÷ ´100 = 45%
è 500 ø
= ` 288000
Total cost = 345600 + 4838400 + 259200 + 288000 æ 550 - 450 ö
= ` 5731200 Year 2006 Þ ç ÷ ´100 = 22.2%
è 450 ø
57. (c) Expenditure on decoration of the room
4838400 ´ 125 æ 600 - 400 ö
= Year 2007 Þ ç ÷ ´100 = 50%
100 è 400 ø
= ` 6048000 æ 800 - 500 ö
58. (a) Sales revenue Year 2008 Þ ç ÷ ´ 100 = 60%
è 500 ø
= ` 725 thousand
63. (c) Increase in sales revenue:
Cost of production in 2008
= Rs. 500 thousand 725 ´ 120
\ Required percentage Year 2005 Þ = ` 870 thousand
100
225 550 ´ 125
= ´ 100 = 45%
500 Year 2006 Þ = ` 687.5 thousand
100
59. (b) Average production cost
600 ´130
æ 625 + 500 + 450 + 400 + 500 + 525 ö Year 2007 Þ = ` 780 thousand
= ` çè ÷ø thousand 100
6
Average revenue
3000 870 + 687.5 + 780
= = ` 500 thousand =
6 3
Required years Þ 2004 and 2009.
60. (c) Average sales revenue 2337.5
=
3
æ 750 + 725 + 550 + 600 + 800 + 800 ö = ` 779.17 thousand
= ` çè ÷ø thousand
6 Increase in production cost
4225 400 ´120
= = ` 704 thousand Year 2007 Þ
6 100
In years 2006 and 2007 sales revenue is 550 and 600 = ` 480 thousand
respectively.

www.newspaperkorner.wordpress.com
www.newspaperkorner.wordpress.com

2014-22 SBI PO EXAM 2014 SOLVED PAPER

67. (b) Increase in wheat production in Bihar:


500 ´ 125
Year 2008 Þ
100 2800 ´ 120
Year 2001 Þ = ` 3360 quintals
= ` 625 thousand 100
525 ´135 2700 ´ 125
Year 2009 Þ Year 2002 Þ = ` 3375 quintals
100 100
= ` 708.75 thousand
Average cost 2800 ´ 128
Year 2003 Þ = ` 3584 quintals
100
æ 480 + 625 + 708.75 ö
Þ ç ÷ thousand 3800 ´135
è 3 ø Year 2004 Þ = ` 5130 quintals
100
1813.75
= Total new production = 3360 + 3375 + 3584 + 5130 +
3
2100 + 3800 + 3600
= ` 604.58 thousand = 29949 quintals
Difference Total earlier production = 2800 + 2700 + 2800 + 3800
= ` (779.17 – 604.58) thousand + 2100 + 3800 + 3600 = 21600 quintals
= ` 174.59 thousand Increase = 29949 – 21600
64. (a) Increase in wheat production:
= 8349 quintals
2700 ´ 130
Year 2003 Þ = 3510 quintals 8349
100 Average increase = quintals
7
2900 ´ 140 Required percentage increase
Year 2004 Þ = 4060 quintals
100
8349
3000 ´145 7 ´100
Year 2005 Þ = 4350 quintals = 21600 = 38.7%
100
7
2900 ´ 140
Year 2007 Þ = 4060 quintals 68. (d) Wheat production by three states:
100
Year 2002 Þ 3300 + 2900 + 2700 = 8900 quintals
Total earlier production of wheat in Madhya Pradesh
Year 2003 Þ 2800 + 2700 + 3700 = 9200 quintals
= 2400 + 3300 + 2900 + 2900 + 3000 + 2400 + 2800
= 19700 quintals Year 2004 Þ 2900 + 3100 + 3800 = 9800 quintals
New wheat production Total production in these years
= 2400 + 3300 + 3510 + 4060 + 4350 + 2400 + 4060 = 8900 + 9200 + 9800 = 27900 quintals
= 24080 quintals Year 2005 Þ 2100 + 3000 + 3500 = 8600 quintals
Increase = 24080 – 19700 Year 2006 Þ 2400 + 2900 + 3800 = 9100 quintals
= 4380 quintals Year 2007 Þ 2900 + 3400 + 3600 = 9900 quintals
Percentage increase Total production = 8600 + 9100 + 9900
= 27600 quintals
4380
= ´ 100 = 22% Required percentage
19700
27900 - 27600
65. (b) Required average production = ´ 100
27600
3500 + 3000 + 2100 = 1.09%
= 69. (a) Total wheat production in the year 2007
3
= (2900 + 3400 + 3600) quintals
8600 2 = 9900 quintals
= = 2866 quintals
3 3 70. (a) Post graduate engineers employed in company W
66. (c) Average production of Bihar. = 200
2800 + 2700 + 2800 + 3800 + 2100 + 3800 + 3600 200
= \ Required percentage = ´ 100
7 600
21600 100 1
= = = 33 %
7 3 3

www.newspaperkorner.wordpress.com
www.newspaperkorner.wordpress.com

SBI PO EXAM 2014 SOLVED PAPER 2014-23

71. (b) Average number of junior engineers 75. (b) Required ratio = 150 : 200 = 3 : 4

100 + 100 + 150 + 250 + 250 850


= = = 170 Country Total No. of
5 5 employees male
72. (c) Average number of assistant engineers employees
I 15480 8400
150 + 300 + 200 + 150 + 100 II 10320 5600
=
5 III 6880 6720
IV 12040 10080
900 V 14620 10080
= = 180 VI 13760 3920
5
VII 6020 5040
Required difference = 180 – 170 = 10 VIII 6880 6160
73. (d) In all companies:
Assistant engineers = 150 + 300 + 200 + 150 + 100 76. (b) Required ratio
= 900 = 8400 : 5600 = 15 : 12 = 5 : 4
Number after 37% increase 77. (c) Employees in country II:
Males = 5600
900 ´137 Females = 10320 – 5600 = 4720
= = 1233
100 Required ratio
= 5600 : 4720 = 70 : 59
Postgraduate engineers = 100 + 250 + 400 + 550 + 78. (c) Male employees:
400 = 1700 Countries (I + II + III)
Number after 20% decrease = 8400 + 5600 + 6720 = 20720

1700 ´ 80 20720
= = 1360 Required average = = 6907
100 3
79. (d) Females in country IV
\ Required percentage = 12040 – 10080 = 1960
1360 - 1233 Females in country VII
= ´ 100 = 6020 – 5040 = 980
1360
Required average
127 1960 + 980 2940
= ´ 100 = 9.3 = = = 1470
1360 2 2
80. (a) New number of female employees:
74. (b) Increase in the number of engineers:
Country III
400 ´ 130 Total employees = 6880
Company V Þ = 520 Female employees = 6880 – 6720 = 160
100
Country IV
Total employees = 12040
700 ´135
Company X Þ = 945 Female employees = 12040 – 10080 = 1960
100
Country V
Total employees = 14620
950 ´140
Company Y Þ = 1330 Female employees
100 Þ 14620 – 10080 = 4540
Total engineers = 520 + 945 + 1330 + 650 + 750 Total female employees
= 4195 = 160 + 1960 + 4540
= 6660
Total original number of engineers = 400 + 650 + 700
+ 950 + 750 = 3450 6660
Average = = 2220
Percentage increase 3

4195 - 3450 2220 ´ 140


= ´ 100 = 22% New average = = 3108
3450 100
Earlier average
= 22%

www.newspaperkorner.wordpress.com
www.newspaperkorner.wordpress.com

2014-24 SBI PO EXAM 2014 SOLVED PAPER

86000 - 56000 30000 130


= = = 3750 = 100700 × = 130910
8 8 100
\ Required percentage Books published by remaining publishers
3108 80
= ´ 100 = 83% = (28200 + 32200 + 29700 + 37800) ×
3705 100
81. (b) Total employees in countries V, VI and VII = 14620 + 127900 ´ 80
13760 + 6020 = = 102320
100
= 34400 \ Required average
Total male employees in countries II, III and IV.
= 5600 + 6720 + 10080 130910 + 102320
=
\ Required percentage 7
34400 - 22400 233230
= ´ 100 = = 33318
22400 7
87. (b) Required ratio
12000 ´ 100
= = 53.6% = 312 : 338
22400 = 156 : 169
82. (c) Academic books published by publisher M 88. (a) Required average
7 190 + 280 + 280 + 315 + 316 + 423
= 28200 ´ = 19740 =
10 6
Academic books published by publisher P 1804
= » 300
8 6
= 31200 ´ = 19200
13 89. (c) Required average
Required difference 483 + 339 + 335 + 410 + 381 + 370
= 19740 – 19200 = 540 =
6
83. (b) Books distributed
2318
33800 ´ 79 = » 386
= = 26702 6
100
90. (d) For the post of D in the years 2001, 2002 and 2003:
Books given to each distributor Appeared candidates Þ 420 + 577 + 487 = 1484
26702 Qualified candidates Þ 364 + 315 + 290 = 969
= = 1068 Difference = 1484 – 969 = 515
25
91. (a) Required percentage
84. (c) Non-academic books:
240
6 = ´ 100 = 73
Publisher R Þ 35700 × = 12600 330
17 92. (b) Required ratio
13 = 190 : 135 = 38 : 63
Publisher S Þ 37800 × = 27300 93. (d) It is clear from the table.
18
Year 2006 Þ 417
12600 + 27300 94. (c) It is clear from the table year 2004 Þ 190
Required average = 95. (c) Required average
2
309 + 354 + 221 + 480 + 416 + 190
39900 =
= = 19950 6
2
85. (d) Required number distributed books 1970
= » 328
29700 ´ 92 33800 ´ 79 6
= +
100 100 350
96. (c) Required % = ´ 100 » 1167
3000
39900
= = 19950 97. (a) Number of boys enrolled in singing and craft
2 = (630 – 490) + (750 – 385)
86. (b) Total books published by Publishers P, Q and R = 140 + 365
130 = 505
= (31200 + 33800 + 35700) ×
100

www.newspaperkorner.wordpress.com
www.newspaperkorner.wordpress.com

SBI PO EXAM 2014 SOLVED PAPER 2014-25

98. Number of girls enrolled in swimming = 245 or Middle Level of Management and Supervisory or
Number of boys enrolled in swimming = Lower Level of Management. The Top Level
(480 – 245) = 235 Management determines the objectives, policies and
Required ratio = 245 : 235 plans of the organisation.
= 49 : 47 108. (e) A peripheral is a “device that is used to put information
99. (b) Total number of girls enrolled in swimming and into or get information into or get information out of
drawing together = (245 + 280) = 525 the computer.” USB (Universal Serial Bus) is a serial
100. (e) Number of boys bus standard to interface devices. Similarly, peripherals
= 3000 – 1750 = 1250 can be connected via Bluetooth.
\ Required percentage 109. (a) Preprocessing is the first pass of any complication.
Compilation is the second pass. It takes the output of
1250 the preprocessor, and the source code, and generates
= ´ 100
3000 assembler source code. Assembly is the third stage of
= 41.67 » 42 compilation which takes the assembly source code and
101. (e) The amount lying in demand deposits can be withdrawn produces an assembly listing with offsets. Linking is
by the depositors as and when required. This amount is the final stage of compilation. It takes one or more
as liquid as the money kept at home. Since the majority object files or libraries as input and combines them to
of such Demand Deposit accounts are checking and produce a single (usually executable) file.
savings accounts, it means that that the bank is getting 110. (e) Malicious programs are used to attackers to disrupt
money at low cost, since no interest is padi on the computer operation, gather sensitive information, or
current accounts and the interest paid on savings gain access to private computer systems. Normally such
account is usually low. a program can be removed by simply uninstalling it.
102. (c) ICICI Bank Limited, India’s largest private sector bank, However, a reboot will be required to completely
on 26th February 2014, announced the launch of remove it from the system.
‘Branch on Wheels’ at Bhubaneshwar in Odisha, as 111. (b) Structured Financial Messaging System (SFMS),
part of its financial inclusion plan that aims at providing launched in December 2001, is used practically for all
banking services in villages which are so far devoid of purposes of secure communication within the bank and
banking facilities. This unique initiative will provide between banks. The intra-bank part of SFMS can be
basic banking services to the remote unbanked villages. used by the banks for secure messaging facility it
103. (e) The Government introduced the “National Agricultural provides.
Insurance Scheme” (NAIS) or “Rashtriya Krishi Bima 112. (d) Bank of India, on Monday, in March 2014, launched
Yojana” (RKBY) in 1999-2000. NAIS envisages IMT (Instant Money Transfer), which allows cardless
coverage of all food crops (cereals and pulses), oilseeds, cash withdrawal at its IMT-enabled ATMs, a first-of-
horticultural and commercial crops. It covers all its-kind initiative by any public sector bank. IMT allows
farmers, both loanees and non-loanees, under the the customer to send money to a receiver only by using
scheme. the receiver’s mobile number through the bank’s ATM
104. (a) Public sector banks contribute 86 percent of the total and retail internet banking facility.
NPAs in the banking system. Currently, a loan is 113. (c) The Middle Level Management consists of the
classified as sub-standard – the first NPA category – if Departmental Heads (HOD), Branch Managers, and the
principal or interest payment is overdue for 9 days. The Junior Executives. The Departmental heads are Finance
share of new private sector banks and foreign banks in Managers, Purchase Managers, etc. The Branch
the total NPAs has also gone up. Managers are the head of a branch or local unit. The
105. (a) Chandi Prasad Bhatt, the noted Gandhian lower level management consists of the Foremen and
environmentalist and social activist won the Gandhi the Supervisors.
Peace Prize for the year 2013 on 28 February 2014. 114. (e) The process of copying software programs from
Bhatt was one of the pioneers of the Chipko movement secondary storage media to the system’s hard disk is
for which he was awarded with the Roman Magsaysay called installation. Computers have primary memory
Award in 1982. that is not used to store bulk data, so we use storage
106. (c) If a cheque bears a date which is yet to come (future devices like hard disk, floppy disk, compact disk (C),
date) which is yet to come (future date) then it is known digital versatile disk (DVD) etc, which are called
as post-dated cheque (PDC). It is a form of a crossed secondary memory.
or account payee bearer cheque but postdated to meet 115. (b) CRAR is the acronym for capital to risk weighted assets
the said financial obligation at a future date. Such a ratio, a standard metric to measure balance sheet
cheque cannot be honoured earlier than the date on strength of banks. BASEL 1 and BASEL II are global
the cheque. capital adequacy rules that prescribe a minimum amount
107. (c) Generally, there are Three Levels of Management, viz., of capital a bank has to hold given the size of its risk
Administrative or Top Level of Management Executive weighted assets.

www.newspaperkorner.wordpress.com
www.newspaperkorner.wordpress.com

2014-26 SBI PO EXAM 2014 SOLVED PAPER

116. (b) Advantage refers to best strategies that lead to the entitled India at Risk – Mistakes, Misconceptions of
creation and sustainability of a competitive advantage. security policy in March 2014. In this book, he has
In order to have that competitive advantage it must discussed the issues, the conflicts and the consequences
possess a favourable asymmetry (e.g. lower unit cost, of the security related policy of India.
higher buyer willingness to pay for its product) between 125. (b) General Insurance Corporation of India had four fully
itself and its competitors in the market. owned subsidiary companies: National Insurance
117. (b) Axis Bank, country’s third largest private lender, Company Limited, New India Assurance Company
opened a branch in Shanghai in January 2014 after Limited, Oriental Insurance Company Limited and
receiving permission from the China Banking United India Insurance Company Limited. It is the sole
Regulatory Commission. With this, it became the first reinsurance company in the Indian insurance market
domestic private sector lender to have a branch presence with over four decades of experience.
in China. 126. (c) The 2014 summit of North Atlantic Treaty Organization
118. (e) As per the ‘Policy Framework for Investment’ of (NATO) will be held in New Port, Wales, United
OECD, there are ten principles to redress weaknesses Kingdom, in September 2014. Russia, China, India and
in the investment environment. Three principles apply Australia are not the members of NATO which is an
throughout the Framework. Policy coherence; international alliances that consists of 28 member states
importance of a transparent approach to policy from North America and Europe.
formulation and implementation; Regular evaluation 127. (c) Market segmentation pertains to the division of a set
of the impact of existing and proposed policies on the of consumers into persons with similar needs and wants.
investment environment; competition policy, corporate Market segmentation allows for a better allocation of a
governance, etc. are some of them. firm’s finite resources. Due to limited resources, a firm
119. (c) The Finance Minister, Pranab Mukherjee, while must make choices in servicing specific groups of
presenting the Union Budget 2010-11, announced the consumers. So it can be defined in terms of the STP
National Social Security Fund for unorganized sector acronym, meaning Segment, Target and Position.
workers with an initial allocation of Rs. 1000 crore. 128. (e) In 1996 PC makers started switching to the ATX
This fund will support schemes for weavers, toddy standard which defined a new 20 pin motherboard
tappers, rickshaw pullers, bidi worker setc. power connector. It has a standby 5 volt rail which is
120. (a) A barcode is an optical machine-readable representation always on even when the power supply is turned off to
of data relating to the object to which it is attached. It provide standby power to the motherboard when the
serves as the Universal Product Code. Barcodes machine is sleeping.
originally were scanned by special optical scanners 129. (c) Sri Lanka, on 8 March 2014, beat Pakistan in the final
called barcode readers. Later, scanners and interpretive to become Asia Cup champions for the fifth time. The
software became available on devices including desktop 2014 Asia Cup was the twelfth edition of the Asia Cup
printers and smartphones. cricket tournament. The tournament was held in
121. The devices which are used to input the data and the Bangladesh.
programs in the computer are known as “Input 130. (a) President Pranab Mukherjee on the occasion of
Devices”. Keyboard, mouse, track ball and light pen International Women’s Day on 8 March 2014, awarded
are all input devices. Output Devices can produce the the Stree Shakti Puraskar is a Women’s Award in the
final product of machine processing into a form usable name of eminent women personalities in Indian history,
by humans. For example, Bar code Reader reads bar who are famous for their personal courage and integrity.
codes an converts them into electric pulses to be 131. (a) According to ‘Introduction to Marketing Management’
processed by a computer. by Osman Safdar Sarwant, the two fold goal of
122. (b) A spreadsheet is an interactive computer application marketing is to attract new customers by promising
program for organization and analysis of data in tabular superior value which also keeping current customers
form. The program operates on data represented as cells by delivering satisfaction. Sarwani outlined a five step
of an array, organized in rows and columns. Microsoft model of the marketing process.
Excel is a spreadsheet application that features 132. (b) To open a file, the easiest way is to left click with the
calculation, graphing tools, pivot tables, etc. mouse twice. Another way is to right click the mouse
123. (d) Public sector general insurer, New India Assurance, in and select‘OPEN’.
March 2014, launched an exclusive health Insurance 133. (b) A remote computer is a computer to which a user does
cover, New India Asha Kiran, for a family with girl not have physical access, but when he or she can access
children. The family floater health insurance, gives a or manipulate via some kind of network. Remote
50 percent discount for the girl child. It also provides a desktop software allows a person to control a computer
personal accident cover for parents and an occurrence from another computer; this allows the user to change
of any accident to parents. anything on the linear computer, and access all of the
124. (b) Jaswant Singh, External Affairs and Finance Minister file contents.
in the previous NDA government released a book

www.newspaperkorner.wordpress.com
www.newspaperkorner.wordpress.com

SBI PO EXAM 2014 SOLVED PAPER 2014-27

134. (a) At present, banks calculate interest paid on money kept 143. (a) RTGS stands for ‘Real Time Gross Settlement’. RTGS
in the savings bank account on a daily basis. This is a is a funds transfer system where transfer of money takes
departure from the earlier practice of calculating interest place from one bank to another on a real time basis.
on the lowest balance after the 10th of every month. In This is the fastest mode of funds transfer available in
2011, the central bank had decided to give freedome to India through banking channel.
commercial banks to fix savings bank deposit rates, 144. (b) Niche marketing is concentrating all marketing efforts
the last bastion of the regulated interest-rate regime. on a small but specific and well defined segment of the
135. (c) The Indian Financial System Code (IFSC) is an population. A niche market is the subset of the market
alphanumeric code that uniquely identifies a bank- on which a specific product is focused. The niche
branch participating in the two main electronic funds market is highly specialized, and aiming to survive
settlement systems in India: the real time gross among the competition from numerous super
settlement (RTGS) and the national electronic funds companies.
transfer (NEFT) systems. 145. (a) The Marketing Mix, also known as the 4 P’s of
136. (a) When we save a file, we can save it to a folder on our Marketing, is the combination of product, price, place
hard disk drive, a network location, disk, DVD, CD, (distribution), and promotion. These four P’s are the
the desktop, flash drive, or save as another file format. parameters that the marketing manager can control,
Save As option is the way to save the data to a different subject to the internal and external constraints of the
file format. marketing environment.
137. (c) The Directorate General of Civil Aviation (DGCA), in 146. (e) A SWOT analysis is a structure planning method used
May 2014, granted an air operator’s permit (AOP) to to evaluate the strengths, weaknesses, opportunities,
AirAsia, paving the way for the airline company to and threats involved in a project of in a business venture.
launch low-cost services in the country. AirAsia India It can be carried out for a product, place, industry or
is a joint venture among Malaysia’s AirAsia Berhad, person.
Tata Sons and Telestra Tradeplace. 147. (b) The Delphi method is a structured communication
138. (e) Repo rate is the rate at which the central bank of a technique, originally developed as a systematic,
interactive forcasting method which relies on a panel
country (RBI in case of India) lends money to
of experts. It has been widely used for business
commercial banks in the event of any shortfall of funds,
forcasting and has certain advantage over another
thereby controlling liquidity. Repo rate is used by
structured forecasting approach, prediction markets.
monetary authorities to control inflation. Repo and
148. (e) Segmentation is the process of slicing a market for a
reverse repo rates form a part of the liquidity adjustment
particular product or service into a number of different
facility.
segments. The segments are usually based on factors
139. (c) The Banking Ombudsman Scheme is introduced under
such as demographics, beliefs or the occassion of use
Section 35 A of the Banking Regulation Act, 1949 by
of the product. Age and gender are demographic
RBI with effect from 1995. The Banking Ombudsman
components of segmentation.
Scheme was introduced under Section 35A of the
149. (a) Innovation is about finding a better way of doing
Banking Regulation Act, 1949 by RBI with effect from
something. Innovation can be viewed as the application
1995. of better solutions that meet new requirements. In-
140. (a) According to the 2011 Census, Maharashtra has highest articulated needs, or existing market needs. This is
number of people living in urban areas (5.08 crores). accomplished through more effective products,
Maharashtra state has 45.23 per cent of the urban processes, services, technologies, or ideas that are
population. With these figures, Maharashtra ranks the readily available to markets.
third most urbanised states in the country, behind Tamil 150. (e) Customer relationship management (CRM) is a system
Nadu (48.45%) and Kerala (47.72%) for managing a company’s interactions with current and
141. (b) The Reserve Bank of India said its move to withdraw future customers. It involves using technology to
currency notes printed before 2005 is to check organize, automate and synchronize sales, marketing,
counterfeit notes and these notes will continue to be customer service, and technical support.
legal tender even after the July deadline. It clarified 157. (a) Achilles heel (Noun) = a weak point or fault in
that the move to withdraw has been taken because they somebody’s character which can be attacked by other
have fewer security features compared to banknotes people.
printed after 2005. 158. (b) Sustain (Verb) = to make something continue for
142. (a) Anand Gandhi’s ‘Ship of Theseus’ won the Best Film sometime without becoming less; maintain.
award at the 61st National Film Awards. It is a complex Look at the sentence:
and philosophical film about an unusual photographer, She managed to sustain everyone’s interest until the
a Jain monk and a stock broker. Rakeysh Omprakash end of her speech.
Mehra directed ‘Bhaag Milkha Bhaag’ won the award 159. (c) Cumbersome (Adjective) = large and heavy; difficult
of the most popular film. to carry; bulky; complex; complicated.

www.newspaperkorner.wordpress.com
www.newspaperkorner.wordpress.com

2014-28 SBI PO EXAM 2014 SOLVED PAPER

Look at the sentence: 172. (b) Equitable (Adjective) = fair and reasonable; treating
Government should ease the cumbersome legal everyone in an equal way.
procedures. Impartial (Adjective) = not supporting one person or
160. (d) Differentiated (Adjective) group more than other; unbiased.
= distinguished; to be the particular thing; important. Hence, synonymous relationship.
Look at the sentence: 173. (c) Flourish (Verb) = to develop quickly and be
I think grey hair makes you look very differentiated. successful; to grow well; thrive.
167. (e) Austerity (Noun) = simplicity; a situation when people Look at the sentence:
do not have much money to spend because there are Few businesses are flourishing in the present economic
bad economic conditions. climate.
Look at the sentence: 174. (a) Affable (Adjective) = pleasant, friendly and easy to
War was followed by many years of austerity. talk to; genial.
168. (a) Inroads (Noun) = something that is achieved, Rude (Adjective) = discourteous; impolite; showing
especially by reducing the success of something else. lack of respect for other people.
Look at the sentence: Hence, antonymous relationship.
Tax rises have made some inroads into the country’s 175. (e) Partial (Adjective) = showing or feeling too much
national debt. support for one person, idea etc. in a way that is unfair;
169. (a) Aggravate (Verb) = to make a bad/unpleasant situation biased.
worse; worsen Dispassionate (Adjective) = not influenced by
Assuage (Verb) = to make an unpleasant situation less emotion; impartial.
severe. Hence, antonymous relationship.
170. (b) Simmering (Adjective) = to be filled with a strong 186. (a) Here subject (tractor sales) is plural. Hence, ..... tractor
feeling especially anger which you have difficulty in sales have seen ....... should be used.
controlling; seething. 187. (b) It is preposition related error. Hence, .......... that share
Calm = peaceful. the earth with us should be used.
Look at the sentence: 189. (b) Here, Noun i.e., environment and forests is ....... should
There was simmering anger among agitators. be used.
171. (c) Dominate (Verb) = to control or have a lot of influence 190. (d) Here, Infinitive i.e., To portray is subject. Hence,
over somebody/something; to be the most important singular verb i.e. references is very difficult should be
feature of something. used.
Monopolise (Verb) = to have or take control of the
larger part of something.
Hence, synonymous relationship.

www.newspaperkorner.wordpress.com
www.newspaperkorner.wordpress.com
SBI PO Exam Solved Paper
Held On : 28-04-2013

Time : 2 hrs. Based on Memory Max. Marks : 200

REASONING (High Level) Directions (Qs.6-10): Study the following information carefully
and answer the given questions.
Directions (Qs. 1-5): Read each statement carefully and answer
the following questions. A, B, C, D, E, G, and I are seven friends who study in three different
standards namely 5th, 6th and 7th such that not less than two
1. Which of the following expressions will be true if the friends study in the same standard. Each friend also has a different
expression R > O = A > S < T as definitely true? favourite subject namely History, Civics, English, Marathi, Hindi,
Maths and Economics but not necessarily in the same order.
(a) O > T (b) S < R
A likes Maths and studies in the 5th standard with only one other
(c) T < A (d) S = O
friend who likes Marathi. I studies with two other friends. Both
(e) T < R the friends who study with I like languages. (Here languages
2. Which of the following symbols should replace the questions include only Hindi, Marathi and English). D Studies in the 6th
mark (?) in the given expression in order to make the standard with only one person and does not like civics. E studies
expression ‘P ? A’ as well as ‘T £ L’ definitely true? with only one friend. The one who likes history does not study in
P> L?A³N=T 5th or 6th standard. E does not like languages. C does not like
(a) £ (b) > English, Hindi or Civics.
(c) < (d) ³ 6. Which combination represents E’s favourite subject and the
(e) either £ or < standard in which he studies?
3. Which of the following symbols should be placed in the (a) Civics and 7th (b) Economics and 5th
blank spaces respectively (in the same order from left to (c) Civics and 6th (d) History and 7th
right) in order to complete the given such a manner that (e) Economics and 7th
makes the expressions ‘B > N’ as well as ‘D £ L’ definitely 7. Which of the following is I’s favourite subject?
true? (a) History
B ––L––O––N––D (b) Civics
(a) =. =. ³, ³ (b) >, ³, =, > (c) Marathi
(d) Either English or Marathi
(c) >, < =, £ (d) <. =, =, ³
(e) Either English or Hindi
(d) >, = ³, >
8. Which amongst the following studies in the 7th standard?
4. Which of the following should be placed in the blank spaces (a) G (b) C
respectively (in the same order from left to right) in order to (c) E (d) D
complete the given expression in such a manner that makes (e) Either D or B
the expression ‘A > P’ definitely false? 9. Which of the following is definitely correct?
__>__>__>__ (a) I and Hindi (b) G and English
(a) L, N, P, A (b) L, A, P, N (c) C and Marathi (d) B and Hindi
(c) A, L, P, N (d) N, A, P, L (e) E and Economics
(e) P, N, A, L 10. Which of the following subjects does G like?
5. Which of the following symbols should be placed in the (a) Either Maths or Marathi
blank spaces respectively (in the same order from left too (b) Either Hindi or English
right) in order to complete the given expression in such a (c) Either Hindi or Civics
manner that makes the expression ‘F > N’ and ‘U > D’ (d) Either Hindi or Marathi
definitely false? (e) Either Civics or Economics
F__O__U__N__D Directions (Qs.11-15): Study the following information carefully
and answer the given questions.
(a) <. <. <, = (b) <, =, =, >
(c) <, =, =, < (d) ³, =, =, ³ In a certain code language- ‘economics is not money’ is written
(d) >, >, =, < as, ‘ka la ho go’ ‘demand and supply economics’ is written as, ‘mo
www.newspaperkorner.wordpress.com
2013-2 SBI PO EXAM 2013 SOLVED PAPER
www.newspaperkorner.wordpress.com
ta pa ka’ money makes only party’ is written as, ‘zi la ne ki’ 18. Which of the following is step III of the given input?
demand makes supply economics’ is written as, ‘zi mo ka ta’ (a) Proud 72 girl 48 family 32 beautiful 17 55 97 rich 61 nice
11. What is the code for ‘money’ in the given code language? life
(a) ga (b) ma (b) life 55 girl 48 family 32 beautiful A proud 97 rich 61 72
(c) pa (d) ta nice.
(e) la (c) girl 48 family 32 beautiful 17 proud 55 97 rich 61 72 nice
12. What is the code for ‘supply’ in the given code language? life
(a) only ta (b) only mo (d) family 32 beautiful 17 proud girl 48 55 97 rich 61 72 nice
(c) either pa or mo (d) only pa life
(e) either mo or ta (e) girl 48 life 55 family 32 beautiful 17 proud 97 rich 61 72
13. What may be the possible code for ‘demand only more’ in nice
the given code language? 19. What is the position of ‘nice’ from the left end in the final
(a) xi ne mo (b) mo zi ne step?
(c) ki ne mo (d) mo zi ki (a) Fifth (b) Sixth
(e) xi ka ta (c) Seventh (d) Eighth
14. What may be the possible code for ‘work and money’ in the (e) Ninth
given code language? 20. Which element is third to the right of ‘family’ in Step V?
(a) pa ga la (b) pa la tu (a) beautiful (b) 17
(c) mo la pa (d) tu la ga (c) proud (b)` 97
(e) pa la ne (c) 32
15. What is the code for ‘makes’ in the given code language? Directions (Qs.21-25): Study the following information carefully
(a) mo (b) pa and answer the given questions.
(c) ne (d) zi If A + B means A is the father of B
(e) ho If A × B means A is the sister of B
Directions (Qs.16-20): Study the following information carefully If A $ B means A is the wife of B
and answer the given questions. If A % B means A is the mother of B
If A ¸ B means A is the son of B
When a word and number arrangement machine is given an input
21. What should come in place of the question mark, to establish
line of words and numbers, it arranges them following a particular
that J is the brother of T in the expression?
rule. The following is an illustration of input and rearrangement:
J ¸P%H?T %L
(All the numbers are two digit numbers)
(a) × (b) ¸
Input : 40 made butter 23 37 cookies salt extra 52 86 92 fell now 19
(c) $ (d) Either ¸ or ×
Step I : butter 19 40 made 23 37 cookies salt extra 52 86 92 fell
(e) Either + or ¸
now 22. Which among the given expressions indicate that M is the
Step II: cookies 23 butter 19 40 made 37 salt extra 52 86 92 fell daughter of D?
now (a) L % R $ D + T × M (b) L + R $ D + M × T
Step III: extra 37 cookies 23 butter 19 40 made salt 52 86 92 fell (c) L % R % D + T ¸ M (d) D + L $ R + M × T
now (e) L $ D ¸ R % M ¸ T
Step IV: fell 40 extra 37 cookies 23 butter 19 made salt 52 86 92 23. Which among the following options is true if the expresssion
now ‘I + T % J × L ¸ K’ is definitely true?
Step V: made 52 fell 40 extra 37 cookies 23 butter 19 salt 86 92 (a) L is the daughter of T (b) K is the son-in-law of I
now (c) I is the grandmother of L (d) T is the father of L
Step VI: now 86 made 52 fell 40 extra 37 cookies 23 butter 19 salt (e) J is the brother of L
92 24. Which among the following expression is true if Y is the son
Step VII: salt 92 now 86 made 52 fell 40 extra 37 cookies 23 butter of X is definitely false?
19 (a) W % L × T × Y ¸ X (b) W + L × T × Y ¸ X
Step VII is the last step of the above arrangement as the intended (c) X + L × T × Y ¸ W (d) W $ X + L + Y + T
arrangement is obtained. (e) W % X + T × Y ¸ L
As per the rules followed in the given steps, find out the 25. What should come in place of the question mark, to establish
appropriate steps for the given input. that T is the sister-in-law of Q in the expression?
Input: 32 proud girl beautiful 48 55 97 rich family 61 72 17 nice life R%T ×P?Q +V
16. How many steps will be required to complete the given input? (a) ¸ (b) %
(a) Five (b) Six (c) × (d) $
(c) Seven (d) Eight (e) Either $ or ×
(e) Nine Directions (Qs.26-30): Study the following information carefully
17. Which of the following is the third element from the left end and answer the given questions.
of step VI?
Eight people - E, F, G, H, J, K, L and M are sitting around a circular
(a) beautiful (b) life
table facing the centre. Each of them is of different profession
(c) 61 (d) nice
-Chartered Accountant, Columnist, Doctor, Engineer, Financial
(e) 17
www.newspaperkorner.wordpress.com
SBI PO EXAM 2013 SOLVED PAPER 2013-3
www.newspaperkorner.wordpress.com
Analyst, Lawyer, Professor and Scientist but not necessarily in 33. Conclusions: I. No song is a paragraph.
the same order. F is sitting second to the left of K. The Scientist is No poem is a paragraph.
an immediate neighbour of K. There are only three people between 34. Conclusions: I. All rhymes are poems.
the Scientist and E. Only one person sits between the Engineer All songs are rhymes.
and E. The Columnist is to the immediate right of the Engineer. M 35. Statements: Some dews are drops. All drops are stones.
is second to the right of K. H is the Scientist. G and J are immediate Conclusions: I. Atleast some dews are stones.
neighbours of each other. Neither G nor J is an Engineer. The
II. Atleast some stones are drops.
Financial Analyst is to the immediate left of F. The Lawyer is
second to the right of the Columnist. The Professor is an immediate Directions (Qs. 36-38): Each of the questions below consists of a
neighbour of the Engineer. G is second to the right of the Chartered question and two statements numbered I and II given below it. You
Accountant. have to decide whether the data given in the statements are
26. Who is sitting second to the right of E? sufficient to answer the questions. Read both the statements
(a) The Lawyer (b) G and-
(c) The Engineer (d) F
Give answer (a) if the data to statement I alone are sufficient
(e) K
to answer the question, while the data in statement II alone
27. Who amongst the following is the Professor?
(a) F (b) L are not sufficient to answer the question.
(c) M (d) K Give answer (b) if the data to statement II alone are sufficient
(e) J to answer the question, while the data in statement I alone
28. Four of the following five are alike in a certain way based on are not sufficient to answer the question.
the given arrangement and hence form a group. Which of Give answer (c) if the data either in statement I alone or
the following does not belong to that group? statement II alone are sufficient to answer the question.
(a) Chartered Accountant - H Give answer (d) if the data neither in statement I nor II
(b) M - Doctor together are sufficient to answer the questions.
(c) J - Engineer Give answer (e) if the data in both statement I and II together
(d) Financial Analyst - L are necessary to answer the questions.
(e) Lawyer - K 36. Seventeen people are standing in a straight line facing south.
29. What is the position of L with respect to the Scientist?
What is Bhavna’s position from the left end of the line?
(a) Third to the left (b) Second to the right
I. Sandeep is standing second to the left of Sheetal. Only
(c) Second to the left (d) Third to the right
(e) Immediate right five people stand betweeen Sheetal and the one who
30. Which of the following statements is true according to the is standing at the extreme right end of the line. Four
given arrangement? people stand between Sandeep and Bhavna.
(a) The Lawyer is second to the left of the Doctor II. Anita is standing fourth to the left of Sheetal. Less
(b) E is an immediate neighbour of the Financial Analyst than three people are standing between Bhavna and
(c) H sits exactly between F and the Financial Analyst Anita.
(d) Only four people sit between the Columnist and F 37. Five letters - A, E, G, N and R are arranged left to right
(e) All of the given statements are true according to certain conditions. Which letter is placed third?
Directions (Qs. 31-35): In each of the questions below, two/three I. G is placed second to the right of A. E is to the immediate
statements are given followed by conclusions/group of right of G. There are only two letters between R and G.
conclusions numbered I and II. You have to assume all the II. N is exactly between A and G. Neither A nor G is at the
statements to be true even if they seem to be at variance from the
extreme end of the arrangement.
commonly known facts and then decide which of the given two
conclusions logically follows from the information given in the 38. Six people - S, T, U, V, W and X are sitting around a circular
statements. table facing the centre. What is T’s position with respect to
X?
Give answer (a) if only conclusion I follows I. Only two people sit between U and W. X is second to
Give answer (b) if only conclusion II follows the left of W. V and T are immediate neighbours of
Give answer (c) if either I or II follows
each other.
Give answer (d) if neither I or II follows
Give answer (e) if both I and II follows II. T is to the immediate right of V. There are only two
31-32. Statements: Some squares are circles. people between T and S. X is an immediate neighbour
No circle is a triangle. of S but not of V.
No line is a square. Directions (Qs.39-40): Study the following information carefully
31. Conclusions: I. All squares can never be triangles. and answer the given questions.
II. Some lines are circles. The convenience of online shopping is what I like best about
32. Conclusions: I. No triangle is a square.
it. Where else can you shop even at midnight wearing your night
II. No line is a circle.
33-34: Statements: All songs are poems. suit? You do not have to wait in a line till the shop assistant is
All poems are rhymes. ready to help you with your purchases. It is much better experience
No rhymes is a paragraph. as compared to going to a retail store. – A consumer’s view.
www.newspaperkorner.wordpress.com
2013-4 SBI PO EXAM 2013 SOLVED PAPER
www.newspaperkorner.wordpress.com
39. Which of the following can be a strong argument in favour Directions (Qs.43-45): Study the following information carefully
of retail store owners? and answer the given questions.
(a) Online shopping portals offer a great deal of discounts
which retail stores offer only during the sale season. Despite repeated announcements that mobile phones were not
(b) One can compare a variety of products online which allowed in the examination hall, three students were caught with
cannot be done at retail stores. their mobile phones.
(c) Many online shopping portals offer the ‘cash on (A) Mobile phones nowadays have a lot of features and it is
delivery’ feature which is for those who are sceptical easy to cheat with their help.
about online payments. (B) The invigilator must immediately confiscate the mobile
(d) Many consumers prefer shopping at retail stores which phones and ask the students to leave the exam hall
are nearer to their houses. immediately.
(e) In online shopping the customer may be deceived as (C) Mobile phones are very expensive and leaving them in bags
he cannot touch the product he is paying for. outside the exam hall is not safe.
40. Which of the following can be inferred from the given (D) There have been incidents where students who left the exam
information? (An inference is something that is not directly
hall early stole the mobile phones kept in the bags of the
stated but can be inferred from the given information)
students who were writing the exam.
(a) One can shop online only at night.
(b) Those who are not comfortable using computers can (E) The school authorities must ask the students to leave their
never enjoy the experience of online shopping. phones in the custody of the invigilator before the exam in
(c) All retail stores provide shopping assistants to each order to avoid thefts of mobile phones.
and every customer. (F) None of the other students were carrying their phones in the
(d) The consumer whose view is presented has shopped exam hall.
at retailo stores as well as online. 43. Which of the following among (A), (B), (C) and (D) may be a
(e) The consumer whose view is presented does not have strong argument in favour of, the three students who were
any retail stores in her vicinity. caught with the mobile phone?
41. Read the following information carefully and answer the given (a) Only (A) (b) Both (A) and (B)
question: (c) Both (C) and (D) (d) Only (C)
Many manufacturing companies are now shifting base to (e) Both (B) and (D)
the rural areas of the country as there is a scarcity of space
44. Which of the following among (A), (B), (E) and (F) may be
in urban areas. Analysts say that this shift will not have a
the reason behind the school making such announcements
huge impact on the prices of the products manufacutured
before the exam?
by these companies as only about 30% consumers live in
urban areas. (a) Only (B) (b) Both (B) and (E)
Which of the following may be consequence of the given (c) Only (F) (d) Only (A)
information? (e) Both (E) and (F)
(a) The prices of such products will decrease drastically 45. Which of the following among (A), (B), (D) and (F) can be an
in the urban areas. immediate course of action for the invigilator?
(b) People living in urban areas will not be allowed to work (a) Only (B) (b) Both (A) and (D)
in such manufacturing companies. (c) Only (A) (d) Both (D) and (F)
(c) These manufacturing companies has set-ups in the (e) Only (F)
urban areas before shifting base. Directions (Qs.46-50): In each of the questions given below which
(d) Those who had already migrated to the urban areas one of the five answer figures on the right should come after the
will not shift back to rural areas. problem figures on the left, in the sequence were continued?
(e) The number of people migrated from rural to urban
areas in search of jobs may reduce.
46.
42. Read the following information carefully and answer the given
question:
‘Pets are not allowed in the park premises’ - A notice put up
at the park entrance by the authority that is responsible for
maintenance of the park.
Which of the following can be an assumption according to Answer Figure:
the given information? (An assumption is something that is
supposed or taken for granted)
(a) At least some people who visit the park have pets.
(b) This is the only park which does not allow pets
(c) People who ignored this notice were fined
(d) There are more than one entrances to the park (a) (b) (c) (d) (e)
(e) Many people have now stopped visiting the park
www.newspaperkorner.wordpress.com
SBI PO EXAM 2013 SOLVED PAPER 2013-5
www.newspaperkorner.wordpress.com
47. DATA ANALYSIS AND INTERPRETATION
C S
T T
C C
S C Directions (Qs.51-55): Study the following information carefully
S T CS
T and answer the given questions.
T
S
Cost of three different fruits (in rupees per kg. in five different
Answer Figure: cities)
Grapes
Cost of three different fruits (in rupees per Guava
C C kg. in five difference cities) Apple
T C
C ST C TS 200
S S S
T T 180

Cost of Fruits (rupees per kg.)


160
(a) (b) (c) (d) (e)
140
120
48. × C B O × O 100
O B C B 80
× × C 60
B O C B O × C 40
20
Answer Figure:
0
Jalandhar Delhi Chandigarh Hoshiarpur Ropar
× C C × C × C C ×
Cities
B O
B 51. In which city is the difference between the cost of one kg of
apple and cost of one kg of guava second lowest?
B O B O O × C O B
(a) (b) (c) (d) (e) (a) Jalandhar (b) Delhi
(c) Chandigarh (d) Hoshiarpur
(e) Ropar
49.
52. Cost of one kg of guava in Jalandhar is approximately what
percent of the cost of two kgs of grapes in Chandigarh?
(a) 66 (b) 24
(c) 28 (d) 34
Answer Figure:
(e) 58
53. What total amount will Ram pay to the shopkeeper for
purchasing 3 kgs of apples and 2 kgs of guavas in Delhi?
(a) ` 530/- (b) ` 450/-
(c) ` 570/- (b) ` 620/-
(a) (b) (c) (d) (e) (e) ` 490/-
54. Ravinder had to purchase 45 kgs of grapes from Hoshiarpur.
Shopkeeper gave him discount of 4% per kg. What amount
50. 8 z D x D x O 8
did he pay to the shopkeeper after the discount?
D 3 T x O T z 8 8 T z S D (a) ` 8,280/- (b) ` 8,104/-
z x T 8 D O O z x (c) ` 8,340/- (b) ` 8,550/-
(e) ` 8,410/-
Answer Figure:
55. What is the respective ratio between the cost of one kg of
z 8 S z D O z apples from Ropar and the cost of one kg of grapes from
Chandigarh?
D S O x 8 z O x z D x x B O (a) 3: 2 (b) 2 : 3
x 8 D 8 S 8 S D 8 (c) 22 : 3 2 (d) 42 : 92
(a) (b) (c) (d) (e) (e) 92 : 4 2

www.newspaperkorner.wordpress.com
2013-6 SBI PO EXAM 2013 SOLVED PAPER
www.newspaperkorner.wordpress.com
Directions (Qs. 56-60) : Study the following information carefully Directions (Qs. 61-65) : Study the following information carefully
and answer the given questions. and answer the given questions.

Number of students (in thousands) in two Number of players participating in three different games in five
different universities in six different years different schools.
Number of students Number of players participating in three
different games in five different schools
Univeristy - 1

2007 University - 2 90
35 80 Hockey

Number of Players
30 70 Basketball
25 60 Kho-Kho
20 50
2012 2008
15 40
10
30
5
20
10
0
0
School-1 School-2 School-3 School-4 School-5
Schools
2011 2009
61. What is the total number of players participating in hockey
from all the five schools together?
(a) 324 (b) 288 (c) 342 (d) 284 (e) 248
2010 62. What is the respective ratio between number of players
participating in baseketball from school-I and the number of
56. What was the difference between the number of students in
players participating in Kho-Kho from school-3?
university - 1 in the year 2010 and the number of students in (a) 5 : 7 (b) 7 : 9 (c) 7 : 8 (d) 9 : 7 (e) 5 : 8
university - 2 in the year 2012? 63. In which school is the number of players participating in
(a) Zero (b) 5,000 hockey and basketball together second lowest?
(c) 15,000 (d) 10,000 (a) School - 1 (b) School - 2 (c) School - 3
(e) 1,000 (d) School - 4 (e) School - 5
57. What is the sum of the number of students in university - 1 64. Number of players participating in Kho-Kho from school-4
in the year 2007 and the number of students in university - 2 is what percent of number of players participating in hockey
from school-2?
in the year 2011 together?
(a) 42 (b) 48 (c) 36 (d) 40 (e) 60
(a) 50000 (b) 55000 65. 25% of the number of the players particpating in hockey
(c) 45000 (d) 57000 from School-5 are females. What is the number of the hockey
(e) 40000 players who are males in school-5?
58. If 25% of the students in university - 2 in the year 2010 were (a) 15 (b) 18 (c) 30 (d) 21 (e) 27
females, what was the number of male students in the Directions (Qs.66-70): Study the following bar-graph carefully
university -2 in the same year? and answer the following questions.
(a) 11250 (b) 12350
Earnings (in rupees) of three different
(c) 12500 (d) 11500 persons on four different days Naveen
(e) 11750 Gita
59. What was the present increase in the number of students in Rahul
University - I in the year 2011 as compared to the previous
year? Thursday
(a) 135 (b) 15
(c) 115 (d) 25
Wednesday
(e) 35
60. In which year was the difference between the number of
Tuesday
students in university - I and the number of students in
university - 2 highest?
(a) 2008 (b) 2009 Monday
(c) 2010 (d) 2011
(e) 2012 0 100 200 300 400 500

www.newspaperkorner.wordpress.com
SBI PO EXAM 2013 SOLVED PAPER 2013-7
www.newspaperkorner.wordpress.com
66. What is Gita’s average earnings over all the days together? 71. What is the difference between the total number of employees
(a) ` 285/- (b) ` 290/- (c) ` 320/- in teaching and medical profession together and the number
(d) ` 310/- (e) ` 315/- of employees in management profession?
67. What is the total amount earned by Rahul and Naveen (a) 6770 (b) 7700
together on Tuesday and Thursday together?
(c) 6700 (d) 7770
(a) ` 1,040/- (b) ` 1,020/- (c) ` 980/-
(d) ` 940/- (e) ` 860/- (e) 7670
68. Gita donated her earnings of Wednesday to Naveen. What 72. In management profession three-fourth of the number of
was Naveen’s total earnings on Wednesday after Gita’s employees are female. What is the number of male employees
donation? in management profession?
(a) ` 520/- (b) ` 550/- (c) ` 540/- (a) 1239 (b) 1143
(d) ` 560/- (e) ` 620/- (c) 1156 (d) 1289
69. What is the difference between Rahul’s earnings on Monday
(e) 1139
and Gita’s earnings on Tuesday?
(a) ` 40/- (b) ` 20/- (c) ` 50/- 73. 25% of employees from film production profession went on
(d) ` 30/- (e) ` 10/- a strike. What is the number of employees from film
70. What is the respective ratio between Naveen’s earnings on production who have not participated in the strike?
Monday, Wednesday and Thursday? (a) 3271 (b) 3819
(a) 7 : 3 : 5 (b) 8 : 6 : 5 (c) 8 : 7 : 4 (c) 3948 (d) 1273
(d) 9 : 5 : 4 (e) 9 : 3 : 4
(e) 1246
Directions (Qs. 71-75) : Study the following pie-chart and answer
the following questions. 74. What is the total number of employees in engineering
profession and industries together?
Percentagewise distribution of employees in (a) 5698 (b) 5884
six different professions
Total number of employees = 26800 (c) 5687 (d) 5896
Engineering (e) 5487
9%
Management 75. In teaching profession if three-fifth of the teachers are not
17%
Tea ching permanent, what is the number of permanent teachers in the
15% teaching profession?
Industries
13% (a) 1608 (b) 1640
Film Production
19%
(c) 1764 (d) 1704
Medica l (e) 1686
27%

Directions (Qs.76-80): Study the table carefully to answer the questions that follow:
Monthly Bill (in rupees) of landline phone, electricity, laundry and mobile phone paid by three different people in five different
months.

Monthly Bills
Month Landline Phone Electricity Laundry Mobile Phone
Ravi Dev Manu Ravi Dev Manu Ravi Dev Manu Ravi Dev Manu
March 234 190 113 145 245 315 93 323 65 144 234 345
April 124 234 321 270 220 135 151 134 35 164 221 325
May 156 432 211 86 150 98 232 442 132 143 532 332
June 87 123 124 124 150 116 213 324 184 245 134 125
July 221 104 156 235 103 131 413 532 143 324 432 543

www.newspaperkorner.wordpress.com
2013-8 SBI PO EXAM 2013 SOLVED PAPER
www.newspaperkorner.wordpress.com
76. What is the total amount of bill paid by Dev in the month of 79. In which months respectively did Manu pay the second
June for all the four commodities? highest mobile phone bill and the lowest electricity bill?
(a) ` 608/- (b) ` 763/- (a) April and June
(c) ` 731/- (b) ` 683/- (b) April and May
(e) ` 674/- (c) March and June
77. What is the average electricity bill paid by Manu over all the (d) March and May
five months together? (e) July and May
(a) ` 183/- (b) ` 149/- 80. What is the respective ratio between the electricity bill paid
(c) ` 159/- (b) ` 178/- by Manu in the month of April and the mobile phone bill
(e) ` 164/- paid by Ravi in the month of June?
78. What is the difference between the mobile phone bill paid (a) 27 : 49
by Ravi in the month of May and the laundary bill paid by (b) 27 : 65
Dev in the month of March? (c) 34 : 49
(a) ` 180/- (b) ` 176/- (d) 135 : 184
(c) ` 190/- (b) ` 167/- (e) 13 : 24
(e) ` 196/-
Directions (Qs.81-85): Study the table carefully to answer the questions that follow:
Station Arrival time Departure Halt time (in Distance travelled No. of Passengers boarding
Name time minutes) from origin (in km) the train at each station
Dadar Starting point 12.05 am - 0 km 437
Vasai Road 12.53 am 12.56 am 3 minutes 42 km 378
Surat 4.15 am 4.20 am 5 minutes 257 km 458
Vadodara 6.05 am 6.10 am 5 minutes 386 km 239
Anand Jn. 6.43 am 6.45 am 2 minutes 422 km 290
Nadiad Jn. 7.01 am 7.03 am 2 minutes 440km 132
Ahmedabad 8.00 am 8.20 am 20 minutes 486 km 306
Bhuj 5.40 pm Ending point – 977 km None

81. What is the distance travelled by the train from Surat to 84. If halt time (stopping time) of the train at Vadodara is
Nadiad Jn.? decreased by 2 minutes and increased by 23 minutes at
(a) 176 km (b) 188 km (c) 183 km (d) 193 km (e) 159 km Ahmedabad. At what time will the train reach Bhuj?
82. How much time does the train take to reach Ahmedabad (a) 6.10 am
after departing from Anand Jn. (including the halt time)? (b) 6.01 pm
(a) 1 hr. 59 min (b) 1 hr. 17 min. (c) 6.05 am
(c) 1 hr. 47 min. (d) 1 hr. 45 min. (d) 6.50 pm
(e) 1 hr. 15 min. (e) 6.07 pm
83. What is the respective ratio between the number of 85. Distance between which two stations is second lowest?
passengers boarding from Vasai Road and from Ahmedabad (a) Nadiad Jn. to Ahmedabad
in the train? (b) Anand Jn. to Nadiad Jn.
(a) 21 : 17 (b) 13 : 9 (c) Dadar to Vasai Road
(c) 21 : 19 (d) 15 : 13 (d) Anand Jn. to Vadodara
(e) 13 : 15 (e) Vasai Road to Surat
Directions (Qs.86-90): Study the table carefully to answer the questions that follow.

Temperature
Month Bhuj Sydney Ontario Kabul Beijing
Max. Min. Max. Min. Max. Min. Max. Min. Max. Min.
st
1 September 24 14 12 2 5 1 34 23 12 9
st
1 October 35 21 5 –1 15 6 37 30 9 3
st
1 November 19 8 11 3 4 0 45 36 15 1
st
1 December 9 2 –5 –9 – 11 –7 31 23 2 –3
st
1 January –4 –7 11 – 13 – 14 – 19 20 11 5 – 13
www.newspaperkorner.wordpress.com
SBI PO EXAM 2013 SOLVED PAPER 2013-9
www.newspaperkorner.wordpress.com
86. What is the difference between the maximum temperature of Table showing ratio between diesel and petrol engine cars which
Ontario on 1st November and the minimum temperature of are distributed among four different states
Bhuj on 1st January? Diesel Engine Petrol Engine
State
(a) 3°C (b) 18°C Cars Cars
(c) 15°C (d) 9°C State - 1 3 4
(e) 11°C
State - 2 5 9
87. In which month respectively the maximum temperature of
Kabul is second highest and minimum temperature of Sydney State - 3 5 3
is highest?
State - 4 1 1
(a) 1st October and 1st January
(b) 1st October and 1st November 91. What is the difference between the number of diesel engine
(c) 1st December and 1st January cars in state-2 and the number of petrol engine cars in
(d) 1st September and 1st January state-4?
(a) 159 (b) 21
(e) 1st December and 1st September
(c) 28 (d) 34
88. In which month (on 1st day) is the difference between (e) 161
maximum temperature and minimum temperature of Bhuj 92. Number of petrol engine cars in state-3 is what percent more
second highest? than the number of diesel engine cars in state-1?
(a) 1st September (b) 1st October (a) 100 (b) 200
st
(c) 1 November (d) 1st December (c) 300 (d) 125
(e) 225
(e) 1st January
93. If 25% of diesel engine cars in state-3 are AC and remaining
89. What is the average maximum temperature of Beijing over all cars are non-AC, what is the number of diesel engine cars in
the months together? state-3 which are non-AC?
(a) 8.4°C (b) 9.6°C (a) 75 (b) 45
(c) 7.6°C (d) 9.2°C (c) 95 (d) 105
(e) 8.6°C (e) 35
94. What is the difference between the total number of cars in
90. What is the respective ratio between the minimum state-3 and the number of petrol engine cars in state-2?
temperature of Beijing on 1st September and the maximum (a) 9 6 (b) 10 6
temperature of Ontario on 1st October? (c) 112 (d) 10 2
(a) 3 : 4 (b) 3 : 5 (e) 9 8
(c) 4 : 5 (d) 1 : 5 95. What is the average number of petrol engine cars in all the
(e) 1 : 4 states together?
(a) 86.75 (b) 89.25
Directions (Qs.91-95): Study the pie-chart and table carefully to
(c) 89.75 (d) 86.25
answer the questions that follow: (e) 88.75
Pie-chart showing percentagewise 96. A bag contains 7 blue balls and 5 yellow balls. If two balls
distribution of cars in four different states are selected at random, what is the probability that none is
Total cars = 700 yellow?
Distribution of Cars
5 5
(a) (b)
35 22
7 7
State - 1 (c) (d)
14% 22 33
Sta te - 4
7
26% (e)
66
97. A die is thrown twice. What is the probability of getting a
Sta te - 2 sum 7 from both the throws?
28%
5 1
(a) (b)
State - 3 18 18
32%
1 1
(c) (d)
9 6
5
(e)
www.newspaperkorner.wordpress.com
36
2013-10 SBI PO EXAM 2013 SOLVED PAPER
www.newspaperkorner.wordpress.com
Directions (Qs.98-100): Study the information carefully to 108. Prof. Muhammad Yunus, the receipient of the Nobel Peace
answer these questions. Prize 2006 is the exponent of which of the following concepts
In a team there are 240 members (males and females). Two-third of in the field of banking?
them are males. Fifteen percent of males are graduates. Remaining (a) Core Banking
males are non-graduates. Three-fourth of the females are (b) Micro Credit
graduates. Remaining females are non-graduates. (c) Retail Banking
98. What is the difference between the number of females who (d) Real Time Gross Settlement
are non-graduates and the number of males who are (e) Internet Banking
graduates? 109. Invisible Export means export of _____________.
(a) 2 (b) 24 (c) 4 (d) 116 (e) 36 (a) Services
99. What is the sum of the number of females who are graduates (b) Prohibited goods
and the number of males who are non-graduates? (c) Restricted goods
(d) Goods as per OGL list
(a) 184 (b) 96 (c) 156 (d) 84 (e) 196
(e) Other than those given as options.
100. What is the ratio between the total number of males and the
110. The European Union has adopted which of the following as
number of females who are non-graduates?
a common currency?
(a) 6 : 1 (b) 8 : 1 (c) 8 : 3 (d) 5 : 2 (e) 7 : 2
(a) Dollar (b) Dinar
GENERAL AWARENESS, (c) Yen (d) Euro
MARKETING & COMPUTERS (e) Peso
111. Banks in India are required to maintain a portion of their
101. Who is the Captain of Australian Cricket Team, which demand and time liabilities with the Reserve Bank of India.
currently (March 2013) visited India? This portion is called _____________.
(a) Michael Clarke (b) Shane Watson (a) Statutory Liquidity Ratio
(c) Shane Warne (d) Michael Hussey (b) Cash Reserve Ratio
(e) Ricky Ponting (c) Bank Deposit
102. Government as part of the recaptalisation plan, infused capital (d) Reserve Repo
in State Bank of India recently. Indicate the approximate (e) Government Securities
capital infusion. 112. Pre-shipment finance is provided by the banks only to
(a) ` 500 cr. (b) ` 1,000 cr. _____________.
(c) ` 1,500 cr. (b) ` 2,000 cr. (a) Credit Card Holders
(e) ` 3,000 cr. (b) Students aspiring for further studies
103. The NRIs while investing in the equity of a company cannot (c) Brokers in equity market
buy more than prescribed percentage of the paid up capital (d) Exporters
of that company. What is the prescribed percentage? 113. Banking Ombudsman is appointed by _____________.
(a) 2% (b) 3% (c) 4% (d) 5% (e) 6% (a) Government of India (b) State Governments
104. Every year March 20 is celebrated as what day? (c) RBI (d) E C G C
(a) World Sparrow Day (e) Exim Bank
(b) International Women’s Day 114. The Holidays for the Banks are declared as per _________.
(c) World Cuckoo Day (a) Reserve Bank Act
(d) International Child Day (b) Banking Regulation Act
(e) International Mother’s Day (c) Negotiable Instruments Act
105. One of the famous Indian Sportsperson released his/her (d) Securities and Exchange Board of India Act
autobiography - ‘Playing to Win’ in November 2012. Name (e) Companies Act
the sports person correctly. 115. Interest on Savings deposit nowadays is _____________.
(a) Saina Nehwal (b) Mary Kom (a) Fixed by RBI
(c) Yuvraj Singh (d) Sachin Tendulkar (b) Fixed by the respective Banks
(e) Sushil Kumar (c) Fixed by the Depositors
106. Which of the following term is associated with the game of (d) Fixed as per the contract between Bank and the
Cricket? Consumer Court
(a) Bouncer (b) Love (e) Not paid by the Bank
(c) Goal (d) Mid Court 116. Interest below which a bank is not expected to lend to
(e) Collision customers is known as _____________.
107. Who is the author of the book ‘Women of Vision’? (a) Deposit Rate (b) Base Rate
(a) Ravinder Singh (b) Preeti Shenoy (c) Prime Lending Rate (d) Bank Rate
(c) Amish Tripathi (d) Durjoy Dutta
(e) Discount Rate
(e) Alam Srinivas www.newspaperkorner.wordpress.com
SBI PO EXAM 2013 SOLVED PAPER 2013-11
www.newspaperkorner.wordpress.com
117. The customers’ by opening and investing in the Tax Saver 124. Government usually classifies its expenditure in term of
Deposit Account Scheme in a Bank would get benefit under planned and non-planned expenditure. Identify, which is the
_____________. correct definiton of Planned expenditure?
(a) Sales Tax (b) Customs Duty (a) It represents the expenditure of all the State
(c) Excise Duty (d) Professional Tax Governments.
(b) It represents the total expenditure of the Central
(e) Income Tax
Government.
118. In banking business, when the borrowers avail a Term Loan,
(c) It is the expenditure which is spent through centrally
initially they are given a repayment holiday and this is referred
sponsored programmes and flagship schemes of
as _____________. Government.
(a) Subsidy (b) Interest Waiver (d) It represents the expenditure incurred on Defence.
(c) Re-phasing (d) Interest concession (e) Other than those given as options.
(e) Moratorium 125. Which of the following organization is made specifically
119. One of IT company from India has become the first Indian responsible for empowering Micro, Small and Medium
Company to trade on NYSE Euronext London and Paris enterprises in India?
markets from February 2013 onwards. Which is this (a) NABARD (b) RBI
company? (c) SIDBI (d) E C G C
(a) Wipro Infotech Ltd. (e) SEBI
(b) L & T Infotech 126. ‘C’ in CPU denotes _____________.
(a) Central (b) Common
(c) HCL Technologies Limited
(c) Convenient (d) Computer
(d) Infosys Technologies Limited
(e) Circuitry
(e) Polaris Financial Technology Ltd. 127. A joystick is primarily used to/for ___________.
120. BSE (Bombay Stock Exchange), the oldest stock exchange (a) Control sound on the screen
in Asia has joined hands with one more international index (b) Computer gaming
in February 2013. This association has resulted in change of (c) Enter text
name of BSE index. What is the change of name effected? (d) Draw pictures
(a) Dow Jones BSE Index (e) Print text
(b) NASDAQ BSE Index 128. Which is not a storage device?
(c) S & P BSE Index (a) A CD (b) A DVD
(d) Euronext BSE Index (c) A floppy disk (d) A printer
(e) Other than those given as options. (e) A Hard disk
129. Which of the following uses a handheld operating system?
121. A non-performing asset is _____________.
(a) A supercomputer (b) A personal computer
(a) Money at call and short notice.
(c) A laptop (d) A mainframe
(b) An asset that ceases to generate income (e) A PDA
(c) Cash balance in till 130. To display the contents of a folder in Windows Explorer you
(d) Cash balance with RBI should __________.
(e) Balance with other banks (a) click on it (b) collapse it
122. RBI has released its guidelines for entry of new banks in the (c) name it (d) give a password
private sector in the month of February 2013. One of the (e) rename it
norms is at least a prescribed percentage of branches of new 131. The CPU comprises of Control, Memory, and ________
bank should be set in unbanked rural centres with a units.
population of upto 9,999 people. What is the percentage of (a) Microprocessor (b) Arithmetic/Logic
such branches prescribed in the norms? (c) Output (d) ROM
(a) 10% (b) 15% (e) Input
132. __________ is the most important/powerful computer in a
(c) 18% (d) 25%
typical network.
(e) Other than those given as options
(a) Desktop (b) Network client
123. The Union Budget for 2013-14 proposed by the Finance
(c) Network server (d) Network station
Minister on 28th February 2013 announced introduction of (e) Network switch
new variety of bonds by the Government. What is the name 133. A(n) ________ appearing on a web page opens another
of these bonds? document when clicked.
(a) Deep discount bonds (b) Zero Coupon bonds (a) anchor (b) URL
(c) Bullet Bonds (d) Inflation Indexed Bonds (c) hyperlink (d) reference
(e) Inflation Variable Bonds (e) heading
www.newspaperkorner.wordpress.com
2013-12 SBI PO EXAM 2013 SOLVED PAPER
www.newspaperkorner.wordpress.com
134. Which of the following refers to the rectangular area for 145. Post-sales activities include _________.
displaying information and running programs? (a) Sales presentation
(a) Desktop (b) Dialog box (b) Customer Feedback
(c) Menu (d) Window (c) Customer identification
(e) Icon (d) Product design
135. __________ is a Window utility program that locates and 146. The ‘USP’ of a product denotes __________.
eliminates unnecessary fragments and rearranges files and (a) Usefulness of the product
unused disk space to optimize operations. (b) Drawbacks of the product
(a) Backup (b) Disk Cleanup (c) Main functions
(c) Disk Defragmenter (d) Restore (d) Number of allied products available
(e) Disk Restorer (e) High selling features of a product
136. Which of the following refers to too much electricity and 147. The competitive position of a company can be improved by
may cause a voltage surge? __________.
(a) Anomaly (b) Shock (a) increasing the selling price
(c) Spike (d) Virus (b) reducing the margin
(e) Splash (c) ignoring competitors
137. The software that is used to create text-based documents (d) increasing the cost price
(e) understanding and fulfilling customers’ needs
are referred to as ___________.
148. Value added services means _________.
(a) DBMS (b) suites
(a) Low Cost products
(c) spreadsheets (d) presentation software
(b) High cost products
(e) word processors
(c) At par services
138. ___________ devices convert human-understandable data
(d) Additional services for the same cost
and programs into a form that the computer can process.
(e) Giving discounts
(a) Printing (b) Output 149. The target market for Debit Cards is _________.
(c) Solid State (d) Monitor (a) All existing account-holders
(e) Input (b) All agriculturists
139. Effective Selling Skills depends on __________ (c) All DSAs
(a) Number of languages known to the DSA (d) All outsourced agents
(b) Data on marketing staff 150 A good Brand can be built up by way of ________.
(c) Information regarding IT market (a) Customer grievances
(d) Knowledge to related markets (b) Break-down of IT support
(e) Ability to talk fast (c) Old age
140. A Direct Selling Agent (DSA) is required to be adept in (d) Large number of products
__________. (e) Consistent offering of good services
(a) Surrogate marketing (b) Training skills
(c) Communication skills (d) Market Research GENERAL AWARENESS
(e) OTC Marketing Directions (Qs.151-160): Read the following passage carefully
141. Leads can be best sourced from __________. and answer the questions given below it. Certain words have
(a) Foreign customers (b) Yellow pages been printed in bold to help you locate them while answering
(c) Dictionary (d) List of vendors some of the questions.
(e) Local supply chains
Financial Inclusion (FI) is an emerging priority for banks that
142. A successful marketing person requires one of the following
have nowhere else to go to achieve business growth. The viability
qualities __________.
of FI business is under question, because while banks and their
(a) Empathy (b) Sympathy
deliver partners continue to make investments, they haven’t seen
(c) Insistence (d) Aggressiveness commensurate returns. In markets like India, most programs are
(e) Pride focused on customer on-boarding, an expensive process which
143. Innovation in marketing is same as _________. people often find difficult to afford, involving issuance of smart
(a) Abbreviation (b) Communication cards to the customers. However, large scale customer acquisition
(c) Creativity (d) Aspiration hasn’t translated into large scale business, with many accounts
(e) Research work lying dormant and therefore yielding no return on the bank’s
144. Market segmentation can be resorted to by dividing the target investment. For the same reason, Business Correspondent Agents
group as per _________. who constitute the primary channel for financial inclusion are
(a) Income levels of customers unable to pursue their activity as a full-time job. One major reason
(b) Age of the employees for this state of events is that the customer on-boarding process
(c) Needs of the sales persons is often delayed after the submission of documents (required to
(d) Marketing skills of the employees validate the details of the concerned applicant) by the applicant
(e) Size of the Organisation and might take as long as two weeks. By this time the initial
www.newspaperkorner.wordpress.com
SBI PO EXAM 2013 SOLVED PAPER 2013-13
www.newspaperkorner.wordpress.com
enthusiasm of applicants fades away. Moreover, the delivery 153. According to the passage, for which of the following reasons
partners don’t have the knowledge and skill to propose anything is the viability of financial inclusion under question?
other than the most basic financial products to the customer and (a) The banks always prefer the cheapest package (to cut
hence do not serve their banks’ goal of expanding the offering in cost) while making a choice of technology to be used
unbanked markets. (b) The Business Correspondent Agents are highly
Contrary to popular perception, the inclusion segment is not a demotivated to pursue their activity as a full-time job
singular impoverished, undifferentiated mass and it is important (c) The investments made by banks and its delivery
to navigate its diversity to identify the right target customers for
partners are not yielding equal amount of returns
various programs. Rural markets do have their share of rich people
(d) The banks do not have adequate number of delivery
who do not use banking services simply because they are
incovenient to access or have low perceived value. At the same partners required to tap the unbanked market
time, urban markets, despite a high branch density, have multitude (e) The banks do not have adequate manpower to explore
of low wage earners outside the financial net. Moreover, the the diversity of right target customers for various
branch timings of banks rarely coincide with the off-work hours programs
of the labour class. 154. In the passage, the author has specified which of the
Creating affordability is crucial in tapping the unbanked market. following characteristics of the customer on-boarding
No doubt pricing is a tool, but banks also need to be innovative in process?
right-sizing their proposition to convince customers that they (a) It involves collection of documents from the applicants
can derive big value even from small amounts. One way of doing in order to validate their details.
this is to show the target audience that a bank account is actually (b) It involves issuance of smart cards to the customers
a lifestyle enabler, a convenient and safe means to send money - (c) It suffers from latency as it takes a long time after
to - family or make a variety of purchases. Once banks succeed in submission of documents by the customer.
hooking customers with this value proposition they must sustain (d) It is an expensive process which people find difficult
their interest by introducing a simple and intuitive user application,
to afford
unbiquitous access over mobile and other touch points, and
(e) All of the given characteristics have been specified
adopting a banking mechanism which is not only secure but also
reassuring to the customer. Technology is the most important 155. What did the author try to highlight in the passage?
element of financial inclusion strategy and an enabler of all other. (A) The ailing condition of financial inclusion business at
The choice of technology is therefore a crucial decision, which present
could make or mar the agenda. Of the various selection criteria, (B) Strategies that may help bank to expand in the
cost is perhaps the most important. This certainly does not mean unbanked market
buying the cheapest package, but rather choosing that solution (C) Role of government in modifying the existing financial
which by scaling transactions to huge volumes reduces per unit inclusion policies
operating cost. An optimal mix of these strategies would no doubt (a) Both (A) and (B) (b) All (A), (B) and (C)
offer an innovative means of expansion in the unbanked market. (c) Only (C) (d) Only (A)
151. Which of the following facts is true as per the passage? (e) Only (B)
(a) People from rural areas have high perceived value of 156. According to the passage, which of the following ways may
banking services. help banks to sustain the interest of their customers after
(b) Cost is not a valid criterion for technological package hooking them?
selection for financial inclusion initiatives
(A) Adoption of a banking mechanism which is not only
(c) The inclusion segment is a singular impoverished,
secure but reassuring to the customers
undifferentiated mass
(d) The branch timings of banks generally do not coincide (B) Increasing the number of delivery partners in rural
with the off-work hours of the labour class in urban market
markets (C) Introduction of a simple and intuitive user application
(e) All the given statements are true (a) Only (A) (b) Only (C)
152. According to the passage, for which of the following reasons (c) Only (B) (d) All, (A), (B) and (C)
do the delivery partners fail to serve their bank’s goal to (e) Both (A) and (C)
expand in the unbanked markets? Directions (Qs.157-158): Choose the word which is most similarly
(A) They do not have adequate client base to sell their in meaning to the word printed in bold as used in the passage.
financial products
(B) They do not have adequate knowledge and skills to 157. Multitude
explain anything beyond basic financial products to (a) Impoverished (b) Handful
the customers. (c) Acknowledged (d) Plenty
(C) They do not have the skills to operate advanced (e) Solitude
technological aids that are a prerequisite to tap the 158. Ubiquitous
unbanked market. (a) Quintessential (b) Popular
(a) Only (B) (b) Only (C) (c) Omnipresent (d) Simplified
(c) All (A), (B) and (C) (d) Only (A) (e) Abnormal
www.newspaperkorner.wordpress.com
2013-14 SBI PO EXAM 2013 SOLVED PAPER
www.newspaperkorner.wordpress.com
Directions (Qs.159-160): Choose the word which is most similarly great deal of contradiction surrounding BYOD and there exists an
in meaning to the word printed in bold as used in the passage. undercurrent of selfishness where users except to use their own
devices, but mostly for personal interest. They recognize the risks
159. Dormant
to the organization but are adamant that those risks are worth
(a) Emaciated (b) Pertinent taking.
(c) Cornered (d) Rejected 161. According to the passage, for which of the following reasons
(e) Active did Fortinet conduct the survey on a group of graduate
160. Delayed employees in their early twenties?
(a) Perturbed (b) Popularised (A) As this group represents the future decision makers
(c) Expedited (d) Stabilised (B) As this group represents the first generation who
(e) Repressed entered the workforce with a better understanding of
Directions (Qs.161-170): Read the following passage carefully sophisticated gadgets
and answer the questions given below it. Certain words have (C) As this group represents the first generation to enter
been printed in bold to help you locate them while answering the workplace expecting that they can use their own
some of the questions. devices for work purpose
(a) All (A), (B) and (C) (b) Only (C)
The evolution of Bring Your Own Device (BYOD) trend has been
(c) Both (A) and (C) (d) Only (A)
as profound as it has been rapid. It represents the more visible
(e) Only (B)
sign that the boundaries between personal life and work life are 162. Which of the following is not true about BYOD?
blurring. The 9 am - 5 pm model of working solely from office has (a) BYOD enables employees to access enterprise network
become archaic and increasingly people are working extended from anywhere and anytime
hours from a range of locations. At the vary heart of this evolution (b) Due to evolution of BYOD trend the 9am - 5pm model
is the ability to access enterprise networks from anywhere and of working solely from office has become outdated
anytime. The concept of cloud computing serves effectively to (c) Recent research has confirmed that BYOD boosts
extend the office out of office. The much heralded benefit of BYOD organisational productivity
is greater productivity. However, recent research has suggested (d) The concept of cloud computing facilates the BYOD
that this is the greatest myth of BYOD and the reality is that trend
BYOD in practice poses new challenges that may outweigh the 163. According to the passage, why would the decision to
benefits. A worldwide survey commissioned by Fortinet chose to embrace BYOD prove dangerous to smaller financial
look at attitudes towards BYOD and security from the users’s businesses?
point of view instead of the IT managers. Specifically the survey (a) Their employees have poor knowledge about their
was conducted in 15 territories on a group of graduate employees devices which in turn pose a threat to the confidential
in their early twenties because they represent the first generation data of the organisation
to enter the workplace with an expectation of own devide use. (b) Their employees are more vulnerable to misplacement
Moreover, they also represent tomorrow’s influences and decision of devices
makers. The survey findings reveal that for financial organizations, (c) They may lack mature IT strategies and policies
the decision to embrace BYOD is extremely dangerous. Larger required to protect confidential data
organizations will have mature IT strategies and policies in place. (d) They cannot afford to deal with damage liability issues
of employee-owned devices
But what about smaller financial businesses? They might not
(e) Their employees have a tendency to change jobs
have such well developed strategies to protect confidential data.
frequently
Crucially, within younger employee group, 55% of the people
164. According to the passage, the expectation of younger
share an expectation that they should be allowed to use their own
employees that they should be allowed to use their own
devices in the workplace or for work purposes. With this
devices in the workplace, entails which of the following risks?
expectation comes the very real risk that employees may consider
(A) Younger employees may deliberately transfer
contravening company policy banning the use of own devices.
confidential data of their companies to rivals if they
The threats posed by this level of subversion cannot be
are not allowed to use their own devices for work
overstated. The survey casts doubt on the idea of BYOD leading
purpose
to greater productivity by revealing the real reason people want
(B) Younger employees may strongly feel like leaving the
to use their own devices. Only 26% of people in this age group
company if it presents usage of own device and join
cite efficiency as the reason they want to use their own devices,
some other company that does not have such stringent
while 63% admit that the main reason is so they have access to
policies
their favourite applications. But with personal applications so
(C) Younger employees may consider flouting company
close to hand, the risks to the business must surely include
policy prohibiting usage of their own devices in the
distraction and time wasting. To support this assumption 46% of
workplace or for work purposes
people polled acknowledged time wasting as the greatest threat
(a) Only (C) (b) Only (B)
to the organization, while 42% citing greater exposure to theft or
(c) Both (A) and (C) (d) Only (A)
loss of confidential data. Clearly, from a user perspective there is
(e) All (A), (B) and (C)
www.newspaperkorner.wordpress.com
SBI PO EXAM 2013 SOLVED PAPER 2013-15
www.newspaperkorner.wordpress.com
165. According to the findings of the survey conducted by 174. Equal opportunities for advancement/ (a) across the length
Fortinet, why do majority of employees prefer using their and breadth / (b) of an organisation will/ (c) keep many
own devices for work purpose? problems away. /(d) No error (e).
(a) As they often find that the devices provided by the 175. A customised data science degree/ (a) is yet to become/(b) a
company lack quality standard programme/ (c) to India’s premier educational
(b) As they have access to their favourite applications institutes./ (d) No error (e)
while working Directions (Qs.176-180): Each question below has two blanks,
(c) As majority of them believe that output doubles when each blank indicating that something has been omitted. Choose
they use their own devices for work purpose the set of words for each blank that best fits the meaning of the
(d) As handling data from their own devices reinforces sentence as a whole.
their sense of responsiblity
176. When you want to digitalise a city __________ with
166. What is/are the author’s main objective(s) in writing the
millions, you don’t bet __________ the odds.
passage?
(a) proceeding, into (b) teeming, against
(A) To break the myth that BYOD promotes employee
(c) undergoing, adhere (d) dangling, for
efficiency and organisational productivity
(e) falling, above
(B) To suggest ways to control subversion across levels
177. The numbers __________ by the legitimate online music
of corporate chain of command
service providers indicate that a growing number of users
(C) To throw light upon the fact that employees even after
are __________ to buy music.
knowing the risks involved, prefer to use their own
(a) morphed, ignoring (b) labelled, thriving
devices for work purpose mostly for personal benefits
(c) figured, fanatic (d) painted, interested
(a) Both (A) and (C) (b) All (A), (B) and (C)
(e) touted, willing
(c) Only (C) (d) Only (A)
178. In India is __________ on protecting its resources,
(e) Only (B)
international business appears equally __________ to
Directions (Qs.167-168): Choose the word which is most similar
safeguard its profit.
in meaning to the word printed in bold as used in the passage.
(a) dreaded, fragile (b) stubborn, weak
167. Heralded (c) bent, determined (d) approaching, settled
(a) Suspected (b) Publicised 179. Brands __________ decision-simplicity strategies make
(c) Dragged (d) Objective full use of available information to __________ where
(e) Benefit consumers are on the path of decisions making and direct
168. Outweigh them to the best market offers.
(a) Control (b) Venture (a) diluting, divulge (b) tempting, maintain
(c) Perish (d) Determine (c) imputing, overdrive (d) pursuing, assess
(e) Surpass (e) employing, trust
Directions (Qs.169-170): Choose the word which is most opposite 180. Lack of financing options, __________ with HR and
in meaning to the word printed in bold as used in the passage technological __________ , make small and medium
enterprises sector the most vulnerable component of our
169. Embrace
economy.
(a) Contradict (b) Disobey
(a) except, loophole (b) coupled, challenges
(c) Curtail (d) Reject
(c) armed, benefits (d) registered, strategies
(e) Obscure
Directions (Qs.181-185): Rearrange the following six sentences
170. Subversion
(A), (B), (C), (D) and (F) in the proper sequence to form a
(a) Compliance (b) Sanity
meaningful paragraph, then answer the questions given below
(c) Popularity (d) Destabilisation
them.
(e) Clarity
Directions (Qs.171-175): Read each sentence to find out whether (A) The group desired to enhance the learning experience in
there is any grammatical mistake/error in it. The error if any, schools with an interactive digital medium that could be used
will be in any part of the sentence. Mark the number of that part within and outside the class-room
with error as your answer. If there is ‘No error’, mark (e). (B) Then the teacher can act on the downloaded data rather
than collect it from each and every student and thereby
171. There cannot be any situation where/(a) somebody makes
save his time and effort.
money in an asset/ (b) located in India and does not pay tax/
(C) Eductor, decided the group of engineers, all alumni of the
(c) either to India or to the country of his origin./ (d) No error
Indain Institute of technology, when the founded Edutor
(e).
Technologies in August 2009.
172. India has entered a downward spiral / (a) where the organised,
(D) They can even take tests and submit them digitally using
productive/ (b) and law abide sectors are subjec to / (c)
the same tablets and the teachers in turn can download the
sevage amounts of multiple taxes./ (d) No error (e).
tests using the company’s cloud services.
173. The bank may have followed/ (a) an aggressive monetary
(E) With this desire they created a solution that digitzes school
tightening policy/ (b) but its stated aim of / (c) curbin inflation
textbooks and other learning materials so that students no
have not been achieved/ (d) No error (e).
www.newspaperkorner.wordpress.com
2013-16 SBI PO EXAM 2013 SOLVED PAPER
www.newspaperkorner.wordpress.com
longer need to carry as many books to school and back as 188. (a) cannot (b) possibly
before, but can access their study material on their touch- (c) should (d) never
screen tablets. (e) must
(F) A mechanic works on motors and an accountant has his 189. (a) necessary (b) apply
computer. Likewise, if a student has to work on a machine or (c) need (d) consider
device, what should it be called? (e) requires
181. Which of the following sentences should be the FIRST after 190. (a) quantity (b) advantages
rearrangement? (c) increase (d) chaos
(a) F (b) D (e) growth
(c) A (d) C 191. (a) challenges (b) praises
(e) E (c) favour (d) leverage
182. Which of the following sentences should be the THIRD (e) esteem
after rearrangement? 192. (a) blessed (b) enjoys
(a) A (b) B (c) endows (d) prevails
(c) D (d) E (e) occurs
(e) F 193. (a) aid (b) jeopardise
183. Which of the following sentences should be the SIXTH (c) promotes (d) endure
(LAST) after rearrangement? (e) cater
(a) A (b) F 194. (a) acute (b) utilising
(c) E (d) B (c) restricting (d) inspiring
(e) D (e) increased
184. Which of the following sentences should be the FOURTH 195. (a) both (b) besides
after rearrangement? (c) combining (d) participating
(a) A (b) F (e) also
(c) E (d) B Directions (Qs.196-200): In each of the following questions four
(e) D words are given of which two words are most nearly the same or
185. Which of the following sentences should be the FIFTHafter opposite in meaning. Find the two words which are most nearly
rearrangement? the same or opposite in meaning and indicate the number of the
(a) A (b) D correct letter combination, by darkening the appropriate oval in
(c) C (d) E your answer sheet.
(e) F
196. (A) Consent (B) Nascent
Directions (Qs.186-195): In the following passage there are
(C) Emerging (D) Insecure
blanks, each of which has been numbered. These numbers are
(a) A–C (b) B-D
printed below the passage and against each, five words/phrases
(c) B–D (d) A–D
are suggested, one of which fits the blank appropriately. Find out
(e) A–B
the appropriate word/phrase in each case.
197. (A) Elated (B) Eccentric
There is a considerable amount of research about the factors that (C) Explicit (D) Abnormal
make a company innovate. So is it possible to create an environment (a) A–B (b) B-D
(186) to innovation? This is a particularly pertinent (187) for (c) A–C (d) A–D
India today. Massive problems in health, education etc. (188) be (e) D–C
solved using a conventional approach but (189) creative and 198. (A) Abundance (B) Incomparable
innovative solutions that can ensure radical change and (190). (C) Projection (D) Plethora
There are several factors in India’s (191). Few countries have the (a) A–C (b) A-B
rich diversity that India or its large, young population (192). While (c) C–D (d) B–D
these (193) innovation policy interventions certain additional steps (e) A–D
also required. These include (194) investment in research and 199. (A) Purposefully (b) Inaccurately
development (195) the government and the private sector, easy (C) Inadvertently (d) Unchangeably
transfer of technology form the academic world etc. To fulfill its (a) A–C (b) A–B
promise of being prosperous and to be at the forefront, India (c) B–C (d) B–D
must be innovative. (e) A–D
186. (a) stimuli (b) conducive 200. (A) germane (B) generate
(c) incentive (d) facilitated (C) reliable (D) irrelevant
(e) impetus (a) B–D (b) B–C
187. (a) objective (b) controversy (c) A–B (d) C–D
(c) doubt (d) question (e) A–D
(e) inference

www.newspaperkorner.wordpress.com
SBI PO EXAM 2013 SOLVED PAPER 2013-17
www.newspaperkorner.wordpress.com

Time : 60 min. Max. Marks : 50

1. Write a letter on any one of the following topics. (10 marks)


(a) Write a letter to your younger sibling explaining the importance of saving money.
(b) Write a letter to the bank manager for extension in joining data.
(c) As a manager, write a letter to your employees to conduct a meeting to discuss the PR strategy.
2. Write an essay in about 250 words on one of the following topics. (12 marks)
(a) Security of women in the work space, an increasing cause of concern.
(b) Entrance exams - a necessary evil.
(c) Pros and cons of inflation.
3. Write a paragraph in about 130 words on one of the following. (8 marks)
(a) Importance of education in vernacular languages.
(b) Pros and cons of winning lotteries.
(c) Do brokers deserve the cut that they make on deals?
4. Write a précis in about 150 words and provide a suitable title. (10 marks)
With the new legislation, India would possibly become the first country to have Corporate Social Responsibility (CSR) spending
through a statutory provision. The bill will now go for presidential assent. The lower house of parliament Lok Sabha cleared the
bill Dec 18 last year. The Bill, aimed at enhancing corporate governance, also contains provisions to strengthen regulations for
corporates as well as auditing firms and promises to ensure an equitable and sustainable growth of the country.
The new Bill has introduced numerous changes and concepts which should simplify regulations and bring greater clarity and
transparency in managing businesses. Presenting the bill in Parliament, Corporate Affairs Minister Sachin Pilot termed the
passage of the Bill as a new era for corporate law and regulation in Indian economy and said this is a 'historic moment for the
country. "After 100 years, this is the second time that a new company's law has been legislated," he said.
Industry body FICCI welcomed the passage of the Companies Bill and said this legislation is indeed a milestone in the history
of company law and will revolutionize the administration and management of businesses in the times to come.
"Industry hopes that the Working Rules which are expected to be put out in the public domain before notification would provide
greater clarity on the operative provisions in the Bill while taking into account legitimate concerns of India Inc," said Naina Lal
Kidwai, President, FICCI. "FICCI hopes that there are no inconsistencies in various laws since consistency and certainty in laws
helps in effective functioning of business, added Kidwai.
5. Read the following passage and answer the questions that follows: (10 marks)
A wire transfer is an electronic method of transferring funds from one person or company to another. Wire transfers are typically
one of the quickest ways to transfer funds from one account to another. Sending money through a wire transfer usually requires
paying a nominal fee. One of the advantages of wire transfer is that the person receiving the money can often withdraw it on the
same business day. Before sending money through wire transfer, make sure that you have sufficient funds in your account,
including wire transfers fees, in order to complete the transaction.
There are two main ways to wire transfer money: bank-to-bank or through retail outlets (cash offices). Bank-to-bank wire
transfers happen over a secure network and both banks must have reciprocal accounts with each other. When they do not,
intermediary banks are used to process the transaction.
Cash office transfers do not require senders and recipients to have accounts. Cash office wire transfers typically take longer
than a bank-to-bank wire transfer to process. Fees may be lower than going through a bank. Because no account is required to
send or receive money, cash office wire transfers are more prone to scams than bank-to-bank transfers. For a large fee, some
cash offices will allow you to send money instantaneously with a credit or debit card, which can be picked up as cash.
(i) What is wire transfer? Explain.
(ii) What is the main advantage of sending money through wire transfer?
(iii) What precautions would you take before sending money through wire transfer?
(iv) Can you distinguish between the bank-to-bank transfer and cash offices transfer?
(v) Can you elaborate the merits and demerits of sending money through cash offices?

www.newspaperkorner.wordpress.com
2013-18 SBI PO EXAM 2013 SOLVED PAPER
www.newspaperkorner.wordpress.com
Answers & Explanations
1. (a) R > O = A > S > T 6. (c) 7. (a) 8. (a)
or, O > S > T 9. (c) 10. (b)
Therefore, O > T (11-15)
2. (b) P > L > A ³ N = T Economics is not money - ka la ho ga .......(i)
3. (d) B > T = O = N ³ D demand and supply economics - mo ta pa ka ....... (ii)
4. (e) By options: money makes only part - zi la ne ki ...... (iii)
(1) L < N < P > A i.e., A < P, definitely true demand makes supply economics - zi mo ka ta ... (iv)
(b) L < A < P > N i.e., A < P, definitely true From (i) & (iii)
(c) A < L < P > N i.e., A < P, definitely true money ® ‘la’
(d) N < A < P > L i.e., A < P, definitely true From (iii) & (iv)
(e) P < N < A > L i.e., A > P, definitely false makes ® ‘zi’
5. (c) F < O = U = N < D From (i) & (iv)
(6-10): economics – ‘ka’
Also, and - ‘pa’
Standard Subject demand - ‘mo’ or ‘ta’
supply - ‘mo’ or ‘ta’
A 5th Maths only - ‘ne’ or ‘ki’
B 7th Hindi or English part - ‘ne’ or ‘ki’
is - ‘ho’ or ‘ga’
C 5th Marathi not - ‘ho’ or ‘ga;
11. (e) 12. (e) 13. (a)
D 6th Economics 14. (b) 15. (d)
E 6th Civics
G 7th Hindi or English
I 7th History

16-20:

Input 32 proud girl beautiful 48 55 97 ric h family 61 72 17 nice life


Step I beautiful 17 32 proud girl 48 55 97 rich family 61 72 nice life
Step II family 32 beautiful 17 proud girl 48 55 97 rich 61 72 nice life
Step III girl 48 family 32 beautiful 17 proud 55 97 rich 61 72 nice life

Step IV life 55 girl 48 family 32 beautiful 17 proud 97 rich 61 72 nice


Step V nice 61 life 55 girl 48 family 32 beautiful 17 proud 97 rich 72

Step VI proud 72 nice 61 life 55 girl 48 fa mily 32 beautiful 17 97 rich


Step VII r ich 97 proud 72 nice 61 life 55 girl 48 family 32 beautiful 17

16. (c) (21-25):


17. (d) A +B Þ A is the father of B.
18. (c) A ×B Þ A is the sister of B.
19. (a) A $B Þ A is the wife of B.
20. (b) A %B Þ A is the mother of B.
A ¸B Þ A is the son of B.

www.newspaperkorner.wordpress.com
SBI PO EXAM 2013 SOLVED PAPER 2013-19
www.newspaperkorner.wordpress.com
21. (a) J ¸ P % H > T % L can be represented in diagram. As
(26-30):
follows. (Columnist)
(Chartered M
P Accountant)E L (Engineer)
- Male
- Female
J = H = T
= - Sibling Relation (Lawyer) J K (Professor)
- Spouse Relation
L

22. (b) L + R $ D + M × T
G H (Scientist)
(Financial F
L Analyst) (Doctor)

R D 26. (b)
27. (d)
28. (c)
M =T 29. (b)
30. (a)
23. (b) I + T % J × L ¸ K
31-32:
I
Squares Circles Line
T K

line
J = L Triangles

24. (d) W $ X + L + Y + T
Triangles

W X

L 31. (a) Conclusion I - True


Conclusion II - False
32. (a) Conclusion I - False
Y Conclusion II - False
33-34:
T
Rhymes
25. (d) R % T × P $ Q + V
Poems

R Paragraph
Songs

T = P Q

V
33. (e) Conclusion I - True
Conclusion II - True
34. (b) Conclusion I - True
Conclusion II - False

www.newspaperkorner.wordpress.com
2013-20 SBI PO EXAM 2013 SOLVED PAPER
www.newspaperkorner.wordpress.com 41. (e) The number of people migrating from rural to urban
areas in search of jobs may reduce.
Stones 42. (a) At least some people who visits the park have pets.
43-45.
A supports the given information as Mobile phones can
Drops aid in cheating so it has been banned in the examination
35. (e) Dews
hall.
B is an immediate course of action the invigilator should
take as the 3 students have violated the rule. It does not
Conclusion I - True matter whether they were using their mobile to cheat at that
Conclusion II - True time or not.
36. (e) Combining statements I and II together. We get C is a reason because of which students would not like to
position of leave their mobile in their bag outside classroom.
D is again a reason as C.
17 11 9 7 4 E can be a possible course of action for student's worry of
R L their mobile being stolen while they are in the examination
hall.
Sheetal Sandeep Anita Bhavna F is statement which supports the rule.
L R 43. (c) Clearly C & D as stated above.
37. (c) Statement I : 44. (d) Clearly A as mobiles used as cheating.
R A N G E 45. (a) B among the given statements is the only course of
action for the invigilator.
L R 46. (d) 47. (c) 48. (b) 49. (b) 50. (c)
Statement II:
A N G 51. (b) Difference between cost of 1 kg apple and cost of 1 kg
guava in 5 cities.
J 160 – 60 = 100
38. (b) Statement I : T / V D 130 – 90 = 40
V / T
C 180 – 120 = 60
H 90 – 30 = 60
R 40 – 20 = 20
\ Cost is second lowest in Delhi.
U W 52. (d) Cost of 1 kg guava in Jalandhar = ` 60
Cost of 2 kg grapes in chandigarh = ` 90 × 2 = ` 180
60
%= ´ 100 = 33.3 » 34%
180
X 53. (c) Cost of 3 kgs apples for Ram = 3 × 130 = ` 390
Cost of 2 kgs guavas for Ram = 2 × 90 = ` 180
From Statement I: We cannot get the exact position of Total cost that Ram pay = 390 + 180 = ` 570
T. 54. (a) Total cost of 45 kgs grapes from Hoshiarpur = 45 × 190
= ` 8550
Statement II: S
8550 ´ 4
X After discount 4% Ravinder paid = 8550 –
100
= ` 8208
55. (c) Cost of 1 kg apples from Ropar :
Cost of 1kg grapes from chandigarh
40 : 90
4 : 9 or 22 : 32
56. (a) Number of students in university I in 2010 = 20,000
V Number of students in university II in 2012 = 20,000
T Difference = 20,000 – 20,000 = 0
57. (e) Number of students in university I in 2007 = 10,000
From Statement II: We get the position of T and X. Number of students in university II in 2011 = 30,000
39. (c) In online shopping the customer may be deceived as Total students = 10,000 + 30,000 = 40,000
he cannot touch the product he is paying for. 58. (a) Total no. of students in Uni-2 in 2010 = 15,000
40. (d) The customer whose view is presented has shopped No. of females students = 25% of 15000
at retail stores as well as online.
www.newspaperkorner.wordpress.com
SBI PO EXAM 2013 SOLVED PAPER 2013-21
www.newspaperkorner.wordpress.com
25 3
= ´ 15, 000 = 3750 72. (e) In management employees are female
100 4
No. of male students = 15,000 – 3750 = 11250
1
59. (d) Number of students in university I in 2011 = 25,000 \ employees in management are male
Number of students in university I in 2010 = 20,000 4
Increase = 25000 – 20000 = 5000 26800 ´17 1
\ No. of male employees = ´ = 1139
5000 100 4
Increase % = ´ 100 = 25%
20000 73. (b) In film production employees on strike = 25%
60. (e) Difference between Number of students in university No. of employees not in the strike = 75%
I & Number of students in university II for the year \ No. of employees not in strike
2007 – 10,000
26800 ´ 19 75
2008 – 10,000 = ´ = 3819
100 10
2009 – 10,000
2010 – 5,000 74. (d) Total No. of employees in engineering industries
2011 – 5,000 = 9 + 13 = 22%
2012 – 15,000 26800 ´ 22
\ Difference was highest in year 2012. \ = 5896
100
61. (b) No. of hockey players in all schools
= 68 + 80 + 54 + 50 + 36 = 288 3
62. (c) No. of basket-ball players in school-1 = 42 75. (a) th of the teachers are not permanent
5
No. of Kho-Kho players in school-3 = 48
Ratio = 42 : 48 = 7 : 8 2
63. (b) \ th of the teachers are permanent
5
64. (d) No. of Kho-Kho players in school-4 = 32
No. of hockey players in school-2 = 80 26800 ´15 2
\ No. of permanent teachers = ´ = 1608
32 100 5
%= ´100 = 40%
80 76. (c) Total amount paid by Dev in June for all commodities
= 123 + 150 + 324 + 134
65. (e) No. of hockey players in school-5 = 36
No. of females = 25%, No. of males = 75% = ` 731
77. (c) Average electricity bill paid by Manu in all five months
36 ´ 75
No. of males = = 27 = 315 + 135 + 98 + 116 + 131
100
66. (b) Gita's average earning over all the day's 795
= = ` 159
5
400 + 420 + 200 + 140 1160
= = = ` 290 78. (a) Mobile phone bill paid by Ravi in May = ` 143
4 4
67. (d) Rahul earned on Tuesday and Thursday Loundry bill paid by Dev in March = ` 323
= 180 + 340 = ` 520 Difference = 323 – 143 = ` 180
Total = 520 + 420 = ` 940 79. (d)
68. (c) Gita donated Naveen = ` 420 80. (a) Electricity bill paid by Manu in April = 135
Naveen's total earning on Wednesday Mobile bill paid by Ravi in June = 245
= 420 + 120 = ` 540
Ratio = 135 : 245 = 27 : 49
69. (a) Rahul's earning on Monday = ` 240
81. (c) Distance travelled by train from Surat to Nadiad Jn.
Gita's earning on Tuesday = ` 200
DiH = 240 – 200 = ` 40 = 440 – 257 = 183 kms
70. (e) Naveen's earning on Monday, Wednesday and 82. (a) Total time taken = 1hr. 35min + 2min + 2min + 20min =
Thrusday 1hr 59 min
= 380 : 120 : 180 83. (a) Ratio between No. of passengers boarding from Vasai
=9:3:4 Road and from Ahmedabad
71. (c) No. of employees in teaching and medical = 42% = 378 : 306 = 21 : 17
No. of employees in management = 17% 84. (b) Total time increase = 23 – 2 = 21 min.
Difference = 42 – 17 = 25%
\ Train will reach Bhuj at = 5:40PM + 21 min = 6:01
26800 ´ 25 PM
= = 6700
100 85. (d) Distance between Anand Jn. to Vadodara is second
www.newspaperkorner.wordpress.com
lowest.
2013-22 SBI PO EXAM 2013 SOLVED PAPER
www.newspaperkorner.wordpress.com
86. (e) Difference between the max. temperature of Ontario 97. (d) Total possible outcomes when A die is thrown twice
on 1st Nov. and the min. temperature of Bhuj on 1st = 36
Jan. = 4 – (7) = 11°C Outcome for getting a sum 7 from both throwns
87. (b) = 6{(2, 6), (2, 5), (3, 4), (4, 3), (5, 2), (6, 1)}
88. (c) Difference between max. and min. temperature of Bhuj 6 1
\ P(E) = =
Sept. = 10°C 36 6
Oct. = 14°C 98–100.
Nov. = 11°C Total members = 240
Dec. = 7°C Males Females
Jan. = 3°C Total 160 80
89. (a) Average of max. temperature of Beijing over all the Graduates 24 60
Non-graduates 136 20
43
months = = 8.6°C 98. (c) Difference between No. of non-graduates females and
5
no. of graduates males = 24 – 20 = 4.
90. (b) Ratio between minimum temperature of Beijing on 1st 99. (e) Sum of (graduates females and non-graduates males)
sept. and maximum temperature of Ontario on 1st Oct. = 60 + 136 = 196
= 9 : 15 = 3 : 5 100. (b) Ratio between total No. of males and no. of non-grad
From (91 – 95) females
Total Cars = 700 = 160 : 20 = 8 : 1
101. (a) Michael Clarke was the captain of Australian team in
State Total Diesel Petrol the first three test matches of the series. Shane Watson
Cars Cars Cars captained the side in the fourth and final test match
1 98 42 56 which was played in New Delhi.
2 196 70 126 102. (e) State Bank of India, the country's largest lender,
3 224 140 84 received Rs. 3,004 crore from the government as part
4 182 91 91 of state-run ban ks. After the infusion , the
government's shareholding in the bank has increased
91. (b) Difference between No. of diesel cars in S-2 and No. of to 62.31 per cent.
pertrol cars in S-4 103. (d) An NRI can purchase up to a maximum of 5% of the
= 91 – 70 = 21 aggregate paid up capital of the company (equity as
92. (b) Petrol cars in S-3 = 84 well as preference capital) or the aggregate paid up
Diesel cars in S-1 = 42 value of each series of convertible debentures as the
case may be. Total investment by NRIs cannot exceed
84
%= ´100 = 200 10% of the paid-up capital in an Indian company.
42
104. (a) March 20 is observed as World House Sparrow Day
93. (d) 25% diesel cars in S-3 are AC (WHSD) to focus on raising awareness about the
\ 75% diesel cars in S-3 are non-AC decline of the common House Sparrow.
140 ´ 75 105. (a) Playing to Win: My Life... on and Off Court is the
\ No. of non-AC cars = = 105 autobiography of Saina Nehwal. She has the distinction
100
of being the first Indian to win a medal in Badminton at
94. (e) Difference between total cars in S-3 and petrol cars in the Olympics in 2012.
S-2 106. (a) In the sport of cricket, a bouncer (or bumper) is a type
= 224 – 126 = 98 of delivery, usually bowled by a fast bowler which is
95. (b) Average of petrol cars in all states together pitched short so that it bounces on the pitch well short
of the batsman and rears up to chest or head height.
56 + 126 + 84 + 91 357
= = = 89.25 107. (e) 'Women of Vision: Nine Business Leaders in
4 4
Conversation' is a book by Alam Srinivas. It is a
96. (c) Blue balls = 7 collection of freewheeling chats with women such as
None-ball out of two yellow television and film producer Ekta Kapoor: ICICI Bank
Yellow balls = 5 Ltd. managing director and CEO Chanda Kochchar,
\ Both balls are blue Biocon Ltd's Kiran Mazumdar. Shaw and Shobhana
Total balls = 12 Bhartia, chairperson of HT Media Ltd.
108. (b) Muhammand Yunus is a Bangladeshi banker and
7 6 7
\ P (both blue balls) = ´ = economist who developed the concepts of microcredit
12 11 22 and microfinance.
www.newspaperkorner.wordpress.com
SBI PO EXAM 2013 SOLVED PAPER 2013-23
www.newspaperkorner.wordpress.com
109. (a) Inversible Exports are services which are sold to other 119. (d) Indian IT Services and out sourcing giant Infosys got
nations by domestic firms, like banking, insurance, and listed on the NYSE Euronext London and Paris markets
tourism. from February 20, 2013 becoming the first Indian
company to trade on the stock exchange. The
110. (d) The euro is the currency used by the Insitututions of
Bangalore-based company is also listed on the Bombay
the European Union and is the official currency of the
Stock Exchange (BSE) and National Stock Exchange
eurozone, which consists of 17 of the 27 member states of India (NSE).
of the European Union. It is the second largest reserve 120. (c) On 19 February 2013 BSE entered into Strategic
currency as well as the second most traded currency Partnership with S&P Dow Jones indices and the
in the world after the United States dollar. SENSEX was renamed as "S&P BSE Sensex." It will
111. (b) The Cash Reserve Ratio is the amount of funds that use the S&P brand for Sensex and other indices.
the banks are bound to keep with Reserve Bank of 121. (b) A non-performing asset (NPA) is defined as a credit
India, with referece to the demand and time liabilities facility in respect of which the interest and/or
(NDLT) to ensure the liquidity and solvency of the installment of principal has remained 'past due' for a
Banks. This proportion is specified by RBI and could specified period of time. Once the borrower has failed
change from time to time. to make interest or principal payments for 90 days the
loan is considered to be a non-performing asset.
112. (e) Pre-Shipment Finance is issued by a financial
122. (d) According to the conditions stipulated by the RBI,
institution when the seller wants the payment of the
the bank shall open at least 25 per cent of its branches
goods before shipment. Pre-shipment or post-shipment
in unbanked rural centres (population up to 9,999 as
finance can be provided to exporters of IT and software per the latest census). Besides, the bank shall comply
services in case of specific orders from abroad. with the priority sector lending targets and subtargets
113. (c) According to the Banking Ombudsman Scheme, 2006, as applicable to the existing domestic banks.
the Banking Ombudsmand is a senior official appointed 123. (d) The Union Budget 2013-14 announced the proposal
by the Reserve Bank of India to redress customer to launch Inflation Indexed Bonds or Inflation Indexed
complaints against deficiency in certain banking National Security Certificates to protect savings from
services. inflation. These bonds will protect the principal and
the interest components from inflation risks.
114. (c) In terms of Section 25 of the Negotiable instruments
124. (b) Planned expenditures are estimated after discussions
Act, 1881, the expression "public holiday" include between each of the ministries concerned and the
Sunday and any other day declared by the Central Planning Commission. The expenditure incurred on the
Government by notification in the Official Gazette to items relating to five year plans is termed as plan
be a public holiday. However, this power was delegated expenditure. Such expenditure is incurred by the Central
by the Central Government to State Governments vide Government.
the Government of India, Ministry of Home Affairs in 125. (c) The Small Industries Development Bank of India
June 1957. (SIDBI) was set up in 1990 with the mission to empower
115. (b) With effect from October 25, 2011, saving bank deposit the Micro, Small and Medium Enterprises (MSME)
interest rate stands deregulated. Accordingly, banks sector with a view to contributing to the process of
economic growth, employment generation and
are free to determine their savings bank deposit interest
balanced regional financial institution responsible for
rate.
promotion, flancing and development of the section.
116. (b) The Base Rate is the minimum interest rate of a Bank 126. (a) CPU is the abbreviation for Central Processing Unit.
below which it cannot lend, except in cases allowed by 127. (b) Joysticks are primarily used for computer gaming.
the Reserve Bank of India. The Base Rate system These are input devices that are connected to a
replaced the Benchmark Prime Lending Rate (BPLR computer to manage game controls such as moving
system) with effect from July 1, 2010. front, behind and sideways, shooting or other gaming
117. (e) In the Finance Act 2006, Section 80C of Income Tax tasks. Hardcore games prefer joysticks over keyboard
controls.
Act 1961 was amended to include deposits placed with
128. (d) Storage devices are data storage devices that are used
a scheduled bank to qualify for deduction under 80C.
in the computers to store data. CD, DVD, floppy disk
The amount of investment up to Rs. 1,00,000 under the
and Hard disk are all types of storage devices. Printers
notified deposit scheme will be eligible for deduction are output devices used to prepare permanent output
under Section 80C of Income Tax Act for individuals/ on paper.
Hindu Undivided Families. 129. (e) A handheld operating system (also known as mobile
118. (e) The Repayment holiday is called moratorium. It is operating system) is an operating system that operates
actually a delay granted in the repayment of a debt or on smart phones, tablet, PDA (personal digital
in the fulfuillment of another legal obligation. assistant) or other digital mobile devices.
www.newspaperkorner.wordpress.com
2013-24 SBI PO EXAM 2013 SOLVED PAPER
www.newspaperkorner.wordpress.com
130. (a) By double-clicking a folder in the main window or in 142. (a) Five essential qualities of a successful marketing person
the list along the left side to open the folder (also called are empathy, focus, responsibility, optimism and ego-
left pane in windows explorer), the contents of the drive (competitiveness). Empathy is the ability to
folder gets displayed. identify with the customers, to feel what they are feeling
131. (b) The CPU comprises of control unit (CU), memory and and make customers feel respected. A sales person
arithmetic/logic unit (ALU). ALU performs arithmetic showing empathy can gain trust and establish rapport
and logic operations and the CU extracts instructions with customers.
from memory (called main memory/primary memory/ 143. (c) Innovation marketing involves applying the principle
RAM) and decodes and executes them, calling on the of creativity to find new ways of marketing effectively.
ALU when necessary. 144. (a) Market segmentation helps to determine targets groups
132. (c) A network server is a computer that provides various or indentifying prospects. It can be resorted to by way
shared resources to workstations and other servers of- segmenting by customer's age, segmenting by
on a computer network. They are built with more customer's income, segmenting geographically,
powerful components than individual workstations. segmenting by customer's tastes & preferences, etc.
133. (c) A hyperlink (or link) is a word, group of words or image 145. (b) The purpose of post sales activities is to support the
that appears on a web page which can be clicked on to customer in use of the product or service and thereby
jump to another document. A link is a part of anchor continuing customer satisfaction. It includes delivering
tag. support to agreed expectations, handling client
feedback, strengthening client relationships, offering
134. (d) Each program displays information in one or more
additional benefits, etc.
windows-rectangular areas on the screen that display
146. (e) USP (unique selling proposition) denotes high/unique
information from a running program. Each program's
selling features of a product i.e., USP helps in creating
window(s) can be minimized, maximised or restored.
a unique place in the mind's of customers relative of
135. (c) Disk fragmentation is a utility program that locates
competition.
and eliminates unnecessary fragments. As operations
147. (e) The competitive position of company can be improved
& processes are loaded & deleted from computer
by being customer-centric not product-centric i.e., not
memory, free memory space is broken in little pieces.
viewing product in isolation, but as a part of a full-
136. (c) A power surge is a period of abnormally high voltage
long lasting relationship with a customer. This means
in an electrical system, which pushes too much current
offering those products and services that fulfill their
(or electricity) through electrical devices. A brief power
needs.
surge that lasts less than one power cycle is called a
148. (d) A value added service is something extra that is
voltage spike.
provided to a customer at no additional charge. For
137. (e) A word processor is a software application that is used example, a sample product is given for free when
to produce, edit and format mainly text documents such another related product is bought whether at regular
as letters, memos, reports, labels, etc. or discounted price.
138. (e) It is the function of the input devices to take the input 149. (a) For a bank, the foremost target market for selling their
from the user (which is in human readable form) and debit cards is all their existing account holders.
convert it into machine readable form before it is 150. (e) Brands loyalty built through goodwill means providing
presented to the computer. consistent good serivces to their customers. For
139. (d) Effective selling skill depends on market information example, providing effective post sales maintenance
i.e., knowledge of related markets. It involves details & support services: listening & responding properly
about customer profile and product mix. when customer provides feedback; etc.
140. (c) A DSA markets and sells goods to customers without 151. (d) In paragraph 2 of the passage, it is mentioned that
the benefit of a retail, online or catalog store. No bank branch timings rarely coincide with off work hours
educational requirements is necessary for DSA of labour class.
position. But DSA should have good oral and written 152. (a) in paragraph 1, the last 5 lines explain this point. There
communication skills. is no mention of the other points in the passage.
141. (b) Lead generation can be achieved by browsing Hence, 1 is the correct answer.
telephone directory, yellow pages, internet sites, list 153. (c) paragraph 1 , line 3rd to 6th explain this aspect.
of existing customers, etc. 154. (e) all the points are covered in paragraph 1.
www.newspaperkorner.wordpress.com
SBI PO EXAM 2013 SOLVED PAPER 2013-25
www.newspaperkorner.wordpress.com
155. (a) the role of government is nowhere mentioned, the other 175. (d) in India’s ..... in place of to India’s ...
2 points are covered, hence (a) is the correct answer 176. (b)
156. (e) the passage explains only points A and C, hence (e) is 177. (e) others do not fit
the correct answer. 178. (c)
157. ‘(d) multitude means ‘a large number of people or things, 179. (d)
hence plenty (d) is correct 180. (b)
158. (c) ubiquitous means-‘present, appearing, or found 181. (a) sentence F is first ( correct sequence- F,C,A,E,D,B)
everywhere’, hence (c) omnipresent is the correct 182. (a) A is the third sentence
answer 183. (d) sentence B will be the correct answer.
159. (e) dormant means inactive, hence opposite would be
184. (c) E
active (e)
185. (b) D
160. (c) delayed means late, so opposite would be to rush or
186. (b) conducive ; the other words do not fit in correctly
hurry, hence expedited (c) is correct
161. (c) as per paragraph 1 A and C are correct, hence correct 187. (d) question
answer is (c) 188. (a) cannot fits correctly
162. (c) paragraph1 says (in line 13-14) that it is a myth... hence 189. (c) need
answer is (c) 190. (e) growth
163. (c) paragraph 1 – last few lines 191. (c) favour
164. (a) paragraph 2 , the 4th and 5th line explains this point. 192. (b) enjoys
165. (b) 193. (a) aid is used correctly meaning facilitate
166. (a) the passage does not talk of point B, hence A and C is 194. (e) increased
correct (a) 195. (a) both
167. (a) means indicate, hence (a) suspected is correct
196. (c) emerging/ nascent mean almost the same (just coming
168. (e) outweigh means to be more significant- hence it means up)
surpass
197. (b) eccentric / abnormal mean almost the same (strange
169. (d) embrace means to accept, hence opposite would be
behaviour)
reject (d)
198. (e) abundance and plethora mean the same ( large quantity
170. (a) compliance
of something)
171. (d) it should be ‘either in India or in the country of his
199. (a) purposefully / inadvertently are opposites , former
origin
means deliberately and latter by mistake
172. (c) and law abiding ....
173. (d) .... has not been achieved instead of have 200. (e) germane / irrelevant are opposites – germane means
174. (e) the sentence is correct ( no error) relevant

www.newspaperkorner.wordpress.com
2013-26 SBI PO EXAM 2013 SOLVED PAPER
www.newspaperkorner.wordpress.com
Solution of Descriptive Test
LETTERWRITING For this, you will be provided with media training to ensure
you represent your business correctly and be able to communicate
1. (a) 8th April, 2014
its key messages.
32 B, Pali Hill,
I should also welcome your suggestions and opinions on
Mumbai.
how to improve these strategies as getting different perspectives
Dear Sanju,
and ideas will surely lead to more ideas for PR strategies.
Hope you are well. How are your studies? I was a bit With warm regards,
surprised to know that you have lately become a little spendthrift Manager
in your expenditures. I must tell you that life is full of surprises Ashutosh Rana
and most of them cost a lot of money. So, you should know the
importance of saving money. ESSAYWRITING
Here I am suggesting you some tips which may help you a
lot in saving money. To start with, make a list of your shopping, 2.
so you don't forget anything. Only buy what's on the list. Put (a) Security of women at the workplace –
your grocery money in an envelope. Don't take any other cash or An increasing cause of concern
Women lately have excelled in all fields including space
cards with you so you can only spend what you have. Eat a meal
exploration and rocket science. They also play a crucial role in
or snack before you go to the supermarket.
economic development of the country and their contribution is
Hope you will remember these tips and follow accordingly.
no less than their male counterparts. However, there are still lot of
Your loving brother,
issues and problems that they encounter today. More often than
Rajat not, they are treated unequally at their workplaces and are
(b) The Manager, considered as inferior to their male co-workers. The major issues
8th April, 2014 and problems that women face at their workplace includeunequal
Canara Bank, pay, security, sexual harassment, lack of proper family support,
Jaipur. deficient maternity leave, etc.
Sir, Sexual harassment is a major issue that women face at their
Sub: Extension of Joining date workplace and many women fall victim of sexual harassment. At
It is really a great pleasure for me to be part of your team. I times employers try to take sexual favours from them in return of
just read the job offer letter you`ve sent and it made me more other benefits and promotions. It can be classified into various
excited on my start date. I truly appreciate the value you have categories like physical contact and advances, a demand or
given me and I`m willing to repay it with hard work, faith and request for sexual favours, any other unwelcome physical, verbal
loyalty. or non-verbal activities such as obscene jokes, comments about
Now as you know I have just recovered from jaundice and physical appearance, threats, innuendos, gender based
have been advised by the doctor to take rest up to 1 month, I`ll be derogatory remarks etc.
joining my duties soon after a month. This will give me ample time Unequal pay comes next that women face at the hands of
to rest and recover from the illness. their employers. Even though, they prove to be more efficient
Hope you will take a sympathetic view of the situation I am than male employees, most of the time, they are not paid equally.
in and grant me leave as desired. The issues and problems that women face at their
Thanking you in anticipation. workplaces should be put to an end and then only it can be said
Yours faithfully, that men and women have equal status. Policies and legal
Rajesh Kumar mechanisms alone cannot help in curbing the problems faced by
women - the overall attitude and acceptance level of the people
(c) To, needs to be changed.
All employees, (b) Entrance Exams - A Necessary Evil
8th April, 2014 Examinations are an age-old practice of evaluating students'
Mercantile Enterprises, performance. However, many students dread the world
'examination'. This is something they have to face right from
Jaipur.
primary school to higher centers of learning. They would probably
Ladies and Gentlemen
have a great sigh of relief if they are eliminated altogether from
Sub: A staff meeting to be held on 9th April, 2014
their academic career.
to discuss the PR strategy
The whole career of a student depends on how he performs
It is a great honour for me to have this opportunity to call a during the short duration of an examination. If he/she suddenly
meeting of all staff members tomorrow at 11 am to discuss the PR falls ill or is not lucky enough to get aneasy question paper, they
strategy. As you are all well aware, I have lately been too busy to fail, their future is ruined. Examiners go just by what is written in
look into each and every affair of the company. It would, therefore, the answer sheet. Again, assessment varies from examiner to
be helpful to delegate some of company's PR activities to you to examiner. It does not, in any way, reflect the real merit of the
help ease my workload. students.
www.newspaperkorner.wordpress.com
SBI PO EXAM 2013 SOLVED PAPER 2013-27
www.newspaperkorner.wordpress.com
The phenomenon has been accepted by all. Yet no other One can find that in the depth of the problem, the public
method of testing students on a mass scale has as yet been schools are providing education through English. The ideal
devised. So, examinations have become a necessary evil. However, solution for this is that, in primary education and higher education,
if you see the brighter side, if the examinations are done away the medium should be English language. But, the local / native
with, it is unlikely that the majority of students would bother to language should be used in explanation. Students will gain skills
study at all. We would never be able to separate the brilliant in the English language and will also be able to reconcile the
students from the average ones. Then, who would we select for national and international scene.
higher professional training? (b) Pros and Cons of Winning a Lottery
Various researches show that despite the evil nature of Winning a lottery is something we all fantasize about. And
examinations, we are bound to go along with them. This is because though many of us believe scoring the mega millions would solve
we have not evolved any effective alternatives to them. However, all our problems, there's a chance we could be wrong.In fact, not
the tests and evaluation systems should be so designed that too many of us are prepared to handle what comes next. Your
lucky day could turn into your unlucky day if you're not
they make a real test of a student's mental ability, originality and
careful.Once you have won the money, greed starts setting in
faculty of critical thinking.
unless you know love is the most important thing in life, not
money. The world gives money the importance, though there are
(c) Pros and Cons of Inflation
some level-headed people who can handle winning the lottery,
Like most things in economics, there are merits and
but the majority doesn't think with common sense.
demeritsof just about every economic variable, and inflation is
There are already too many stories of couples and
no different. Inflation does certainly hurt a lot of people but it
individuals whose lives have fallen apart after hitting the jackpot.
also helps others. While the chances of winning the lottery might be slim, the lessons
On the negative side, inflation raises the cost of living for are good for all to remember, as increased money from a raise or
everyone. So that means that people need more income in order inheritance could upset still waters just like the mega millions.
to keep up with rising prices. But particularly for those with debt, (c) Do brokers deserve the cut that they make on deals?
inflation actually helps because it makes the value of the rupee When a buyer wants to make a purchase on a house, his
less. And so it really reduces both the value of the debt and the dependence on his broker is much higher. And so is the risk of his
value of those debt payments. being misled. A broker is just an intermediary. He is not answerable
On the other hand, if you are an exporter and selling your in case of overpricing or any other issue.
products to overseas buyers, inflation helps because it tends to The broker puts in the effort to find a suitable house and
lower the value of the rupee against dollar on international get the best deal, which saves you time. However, the brokerage
markets, thereby making Indian exports cheaper in foreign fee could be high and if you and the seller don't have to pay it,
countries, and so you as an exporter can sell more. you could negotiate for a lower selling price.
Inflation is bad making things more expensive except that it Brokers get a commission from sellers for each sale they
can also be good when it brings you a raise or makes it easier to make, which is usually a percentage of the deal value. It is
pay your debts.When inflation raises your income, your debt imperative that the broker state his commission and give evidence
payments get easier to handle.But, if you had a pension with no that the property has a clear title and we'll have to see if the cut
cost of living adjustment, higher inflation would eat more deeply they make on this particular deal is worth paying.
at your buying power.
It has now been proved how inflation can touch every aspect PRECIS
of our life, including that of business. It can destroy a nation's 4.
economy in a matter of days or simply be the unavoidable effect India becoming the first country with legislated corporate
of a healthy growing economy. social responsibility
With India becoming the first country to have Corporate
PARAGRAPH WRITING Social Responsibility (CSR) spending by virtue of a statutory
3. provision is just awaiting the presidential assent. Passed by the
lower house of parliament on Dec 18 last year, the Bill, aimed at
(a) Importance of Education in Vernacular Language
enhancing corporate governance, also comprises of provisions
Mother tongue is psychologically significant to students,
to strengthen regulations for corporates as well as auditing firms.
especially children. It helps develop perspective as language and
The new bill which ensures an equitable and sustainable
thought are connected and it is impossible to think without
growth of the country has introduced lot of changes and concepts
knowing language.
which should simplify regulations and bring larger clarity and
Vernacular language is also important socio-culturally. It
transparency in managing businesses. Terming the passage of
has been found through research that changing the learning the bill as a new era for corporate law and regulation in Indian
medium from mother tongue to a different language makes economy and a 'historic moment for the country, Corporate Affairs
students insecure and leads to low self esteem. Students stop Minister Sachin Pilot said that a new law has been legislated.
liking school, education and teachers.
www.newspaperkorner.wordpress.com
2013-28 SBI PO EXAM 2013 SOLVED PAPER
www.newspaperkorner.wordpress.com
Various Industry bodies including FICCI also welcoming (iii) Before sending money through wire transfer, make sure
the bill termed it a milestone in the history of company law which that you have sufficient funds in your account,
will bring a sea change in the future administration and including wire transfers fees. Secondly, both senders
management of businesses.Lauding the bill, Naina Lal Kidwai, and recipients have accounts in case of bank-to-bank
President, FICCI hoped that the Working Rules which are expected wire transfer.
to be put out in the public domain would provide greater clarity (iv) Bank-to-bank wire transfers happen over a secure
on the operative provisions in the Bill as well as take into account network and both banks must have reciprocal accounts
legitimate concerns of India Inc. Stating further that there are no with each other while cash office wire transfers are
inconsistencies in various laws he said that consistency and more prone to fraud and fees may be lower than bank-
certainty in laws will help function of business effectively. to-bank.
(v) The main advantages of cash office transfers are that
PASSAGE they do not require senders and recipients to have
accounts. Fees are lower than going through a bank
5. (i) The wire transfer is an electronic method of transferring
and for some extra money, cash offices will allow you
funds from one person or company to another. It is
to send money instantaneously with a credit or debit
one of the quickest ways to transfer funds from one
card. The downside is that cash office wire transfers
account to another.
typically take longer than a bank-to-bank transfer.
(ii) The main advantages of sending money through wire
Secondly, cash office wire transfers are more prone to
transfer include their being the one of the quickest
fraud than bank-to-bank transfers.
ways to transfer funds from one account to another. It
requires paying a nominal fee and the person receiving
the money can often withdraw it on the same business
day.

www.newspaperkorner.wordpress.com
www.newspaperkorner.wordpress.com
SBI (Associates) PO Exam
Held On : 07-08-2011

Time : 2 hrs. Based on Memory Max. Marks : 200

REASONING (High Level) Directions (Qs. 7-12): Study the following information to
answer the given questions:
Directions (Qs.1-3): Study the following information carefully
and answer the given questions. Twelve people are sitting in two parallel rows containing six
people each such that they are equidistant from each other. In
Four of the following five are alike in a certain way and
row 1: P, Q, R, S, T and V are seated and all of them are facing
hence form a group. Which is the one that does not belong to the
South. In row 2: A, B, C, D, E and F are seated and all of them are
group? facing North. Therefore, in the given seating arrangement, each
1. (a) Break (b) Change
member seated in a row faces another member of the other row.
(c) Split (d) Divide
S sits third to the right of Q. Either S or Q sits at an extreme
(e) Separate end of the line. The one who faces Q sits second to the right of E.
2. (a) Train (b) Instruct
Two people sit between Band F. Neither B nor F sits at an extreme
(c) Educate (d) Advocate
end of the line. The immediate neighbour of B faces the person
(e) Coach who sits third to the left of P. R and T are immediate neighbours.
3. (a) Extend (b) Higher
C sits second to the left of A. T does not face the immediate
(c) Upward (d) Rise
neighbour of D.
(e) Ascend 7. Who amongst the following sit at the extreme ends of the
4. Which of the following symbols should replace question
rows?
mark (?) in the given expression in order to make the
(a) S, D (b) Q, A
expressions ‘ A > D’ and ‘F ³ C’ definitely true? (c) V, C (d) P, D
A > B ³ C ? D £ E=F
(e) Q, F
(a) > (b) <
8. Who amongst the following faces S?
(c) £ (d) = (a) A (b) B
(e) Either = or ³
(c) C (d) D
5. Which of the following expressions is definitely true if the
(e) F
given expressions ‘R < P’ as well as ‘S > Q’ are definitely 9. How many persons are seated between V and R?
true?
(a) One (b) Two
(a) P > Q = R £ T < S (b) S > T ³ R > Q < P
(c) Three (d) Four
(c) Q > R £ T > P ³ S (d) S > T ³ R > Q > P (e) None of these
(e) None of these
10. P is related to A in the same way as S is related to B based
6. Read the following information carefully and answer the
on the given arrangement. Which of the following is T
question which follows: related to, following the same pattern?
‘A × B’ means ‘A is the father of B’.
(a) C (b) D
‘A + B’ means ‘A is the daughter of B’.
(c) E (d) F
‘A ÷ B’ means ‘A is the son of B’. (e) Cannot be determined
‘A – B’ means ‘A is the sister of B’.
11. Which of the following is true regarding T?
What will come in place of question mark to establish that P
(a) F faces T.
is the son-in-law of S in the following expression? (b) V is an immediate neighbour of T.
P×Q+R–T ?S
(c) F faces the one who is second to the right of T.
(a) + (b) ×
(d) T sits at one of the extreme ends of the line.
(c) – (d) ÷ (e) Q sits second to the right of T.
(e) Either + or ÷
www.newspaperkorner.wordpress.com
P1-2 SBI (Associates) PO EXAM 2011 SOLVED PAPER
www.newspaperkorner.wordpress.com
12. Four of the following five are alike in a certain way based on 17. Is F granddaughter of B?
the given arrangement and so form a group. Which is the I. B is the father of M. M is the sister of T. T is the
one that does not belong to that group? mother of F.
(a) A-T (b) B-T II. S is the son of F. V is the daughter of F. R is the brother
(c) F-P (d) C-V of T.
(e) E-Q Directions (Qs. 18-20): Study the following information to
Directions (Qs. 13-17): Each of the questions below consists of answer the given questions:
a question and two statements numbered I and II given below
A word and number arrangement machine when given an
it. You have to decide whether the data provided in the statements
input line of words and numbers rearranges them following a
are sufficient to answer the question. Read both the statements
particular rule in each step. The following is an illustration of
and give answer.
input and rearrangement. (All the numbers are two-digit numbers
(a) if the data in statement I alone are sufficient to answer and are arranged as per some logic based on the value of the
the question, while the data in statement II alone are number.)
not sufficient to answer the question. Input : win 56 32 93 bat for 46 him 2811 give chance
(b) if the data in statement II alone are sufficient to answer Step I : 93 56 32 bat for 46 him 28 11 give chance win
the question, while the data in statement I alone are Step II : 11 93 56 32 bat for 46 28 give chance win him
not sufficient to answer the question. Step III : 56 11 93 32 bat for 46 28 chance win him give
(c) if the data either in statement I alone or in statement II Step IV : 28 56 11 93 32 bat 46 chance win him give for
alone are sufficient to answer the question. Step V : 46 28 56 11 93 32 bat win him give for chance
(d) if the data even in both statements I and II together are Step VI : 32 46 28 56 11 93 win him give for chance bat
not sufficient to answer the question.
Step VI is the last step of the arrangement the above input.
(e) if the data in both statements I and II together are
As per the rules followed in the above steps, find out in
necessary to answer the question.
each of the following questions the appropriate steps for the
13. Which bag amongst P, Q, R, S and T is the heaviest?
given input.
I. Bag Q is heavier than R and S. Bag T is heavier than
Input for the questions:
only bag P.
Input: fun 89 at the 28 16 base camp 35 53 here 68 (All the
II. Only three bags are lighter than R. The weight of bag
numbers given in the arrangement are two-digit numbers.)
Q is 50 kg, which is 2 kg more than bag R.
18. Which of the following would be step II?
14. Are all the five friends - A, B, C, D and E - who are seated
around a circular table facing the centre? (a) 89 fun at 28 16 base camp 35 53 here 68 the
I. A sits to the left of B. B faces the centre. D and E are (b) 35 53 28 68 16 89 the here fun camp base at
immediate neighbours of each other. C sits second to (c) 16 89 at fun 28 camp base 35 53 68 the here
the right of E. (d) 53 28 68 16 89 35 the here fun camp base at
II. D sits second to right of C. C faces the centre. Both E (e) None of these
and A are immediate neighbours of D. B sits second to 19. Which word/number would be at seventh position from the
the right of A. left in step IV?
15. In a college, five different subjects, viz Physics, Chemistry, (a) base (b) at
Botany, Zoology and Mathematics, are taught on five (c) 35 (d) the
differeIlt days of the same week, starting from Monday and (e) 53
ending on Friday. Is Chemistry taught on Wednesday? 20. Which step number would be the following output?
I. Two subjects are taught between Zoology and 53 28 68 16 89 at 35 the here fun camp base
Mathematics. Mathematics is taught before Zoology.
(a) There will be no such step.
Chemistry is taught on the day immediately next to the
(b) III
day when Physics is taught. Botany is not taught on
Friday. (c) II
II. Three lectures are scheduled between the lectures of (d) V
Botany and Zoology. Mathematics is taught (e) IV
immediately before Physics. Directions (Qs. 21-27): Study the following information
16. Is it 9 o'clock now? carefully and answer the given questions.
I. After half an hour, the minute and the hour hands of Eight colleagues, A, B, C, D, E, F, G and H, are sitting around
the clock will make an angle of exactly 90° with each a circular table facing the centre but not necessarily in the same
other.
order. Each one of them holds a different post–Manager, Company
II. Exactly 15 minutes ago, the hour and the minute hands Secretary, Chairman, President, Vice President, Group Leader,
of the clock coincided with each other. Financial Advisor and Managing Director.
www.newspaperkorner.wordpress.com
SBI (Associates) PO EXAM 2011 SOLVED PAPER P1-3
www.newspaperkorner.wordpress.com
A sits third to the right of the Managing Director. Only two Directions (Qs. 28-31): Read the following information and
people sit between the Managing Director and H. The Vice the sentences (A), (B), (C), (D) and (E) given below it carefully
President and the Company Secretary are immediate neighbours. and answer the questions which follow:
Neither A nor H is a Vice President or a Company Secretary. The
A host of foreign companies are in talks with the Indian
Vice President is not an immediate neighbour of the Managing
government for selling B 150, a tough, short-haul plane ideal for
Director. The Manager sits second to the left of E. E is not an connectivity of smaller towns which is lacking in India at present.
immediate neighbour of H. The Manager is an immediate (A) B 150 planes have not only low operating costs than
neighbour of both the Group Leader and the Financial Advisor. competing planes like Cezana but also a much better
The Financial Advisor sits third to the -right of B. B is not the Vice track record in terms of safety and efficiency.
President. C sits on the immediate right of the Chairman. A is not (B) The profit margin of road transport operators in the
the Chairman. F is not an immediate neighbour of A. G is not an smaller towns connected by B 150 planes has been
immediate neighbour of the Manager. reduced substantially as a majority of people prefer air
21. Who amongst the following sits third to the left of E? transport over other means of transport.
(a) Manager (b) G (C) Smaller towns, at present, are better connected by roads
(c) A (d) Financial Advisor and railways as compared to flight services.
(e) B (D) B 150 planes are capable of operating in sectors where
large airlines cannot fly due to challenging conditions
22. Four of the following five are alike in a certain way based on
such as mist, short runways, etc. Such planes can also
the given arrangement and thus form a group. Which is the
double up as cargo planes and charter flights for the
one that does not belong to that group?
rich and the elite.
(a) F-Chairman (b) G-President (E) B 150 planes need to operate in the existing airports
(c) D-Manager (d) A-Financial Advisor which are situated in bigger cities only and are poorly
(e) Managing Director connected to the smaller cities.
23. Who among the following is the President of the company? 28. Which of the statements (A), (B), (C), (D) and (E) can be
inferred from the facts/information given in the statement?
(a) A (b) C
(An inference is something which is not directly stated but
(c) H (d) G can be inferred from the given facts.)
(e) D (a) Only A (b) Only B
24. Which of the following is true with respect to the given (c) Only C (d) Both B and D
seating arrangement? (e) Only E
(a) The Group Leader of the company is an immediate 29. Which of the statements (A), (B), (C), (D) and (E) mentioned
neighbour of the Vice President. above would weaken the offer made by the foreign
(b) G sits second to the right of D. companies for selling B 150 planes to Indian government?
(a) A (b) B
(c) The Group Leader and the Company Secretary are
(c) C (d) D
immediate neighbours.
(e) E
(d) The Chairman of the company sits to the immediate 30. Which of the statements (A), (B), (C), (D) and (E) mentioned
left of the Managing Director. above represents a possible consequence of the success
(e) The Group Leader sits second to the left of D. of B 150 planes in smaller cities?
25. Which of the following posts does B hold in the company? (a) A (b) B
(a) Chairman (b) Manager (c) C (d) D
(c) Company Secretary (d) Vice President (e) E
31. Which of the statements (A), (B), (C), (D) and (E) would
(e) Financial Advisor
favour the foreign companies’ bid to sell B 150 planes in
26. Who among the following sits exactly between the India?
Managing Director and H? (a) Only A (b) Only B
(a) H and the Chairman (b) B and G (c) Both B and C (d) Both A and D
(c) The Chairman and C (d) F and C (e) Both E and C
(e) E and the Group Leader Directions (Qs. 32-37): Study the following information to
27. Who among the following is the Group Leader? answer the given questions:
(a) C (b) F In a certain code, ‘always create new ideas’ is written as ‘ba
(c) G (d) H ri sha gi’, ‘ideas and new thoughts’ is written as ‘fa gi ma ri’,
(e) A ‘create thoughts and insights’ is written as ‘ma jo ba fa’, and
‘new and better solutions’ is written as ‘ki ri to fa’.
www.newspaperkorner.wordpress.com
P1-4 SBI (Associates) PO EXAM 2011 SOLVED PAPER
www.newspaperkorner.wordpress.com
32. What is the code for ‘ideas’? (a) House owners too have demanded for laws that make
(a) sha (b) ba it easier to evict tenants who default on rent payment.
(c) gi (d) ma (b) Such a tax law would be difficult to implement as it
(e) Cannot be determined would be difficult to record the number of vacant flats
33. What does 'fa' stand for? in the city.
(c) People with surplus money buy many houses and rent
(a) thoughts (b) insights
these out while many consumers cannot afford even
(c) new (d) and
their first house.
(e) solutions (d) The number of vacant flats in Mumbai is much lower
34. ‘fa lo ba’ could be a code for which of the following? than other metros such as Delhi and Chennai where
(a) thoughts and action (b) create and innovate vacancy tax already exists.
(c) ideas and thoughts (d) create new solutions (e) Such a tax would compel the house owners to sell!
(e) always better ideas rent properties which in turn would control hoarding
35. What is the code for ‘new’? and skyrocketing property prices.
(a) ki (b) ri Directions (Qs. 40-45): In each question below are given two/
(c) to (d) fa three statements followed by two conclusions numbered I and
(e) ba II. You have to take the given statements to be true even if they
36. Which of the following may represent ‘insights always seem to be at variance with commonly known facts. Read all
better’? the conclusions and then decide which of the given conclusions
logically follows from the given statements, disregarding
(a) jo ki to (b) ki to ri
commonly known facts. Give answer
(c) sha jo ri (d) to sha jo
(a) if only conclusion I follows.
(e) sha to ba
(b) if only conclusion II follows.
37. What is the code for ‘throughts’? (c) if either conclusion I or conclusion II follows.
(a) ma (b) fa (d) if neither conclusion I nor conclusion II follows.
(c) ba (d) jo (e) if both conclusion I and conclusion II follow.
(e) Either jo or fa 40. Statements : All rings are circles.
Directions (Q. 38): Read the following information carefully All squares are rings.
and answer the question which follows. No ellipse is a circle.
38. All existing and upcoming hotels within a 5 km radius of Conclusions : I. Some, rings being ellipses is a possibility.
national parks and sanctuaries in India will have to pay : II. At least some circles are squares.
30% of their annual turnover as tax to the government. 41. Statements : No house is an apartment.
Which of the following statements can be inferred from the Some bungalows are apartments.
facts/information given in the above statement? Conclusions : I. No house is a bungalow.
(a) The tax collected from the hotels will be used for the II. All bungalows are houses.
betterment of these national parks and sanctuaries. 42. Statements : Some gases are liquids.
(b) Hotels which are sponsored by the government will All liquids are water.
not have to pay any tax even if these are located within Conclusions : I. All gases being water is a possibility.
the 5 km radius of such wildlife hotspots. II. All such gases which are not water can
(c) The ecosystem of the national parks and sanctuaries never be liquids.
is adversely affected even if the hotels are located 43. Statements : All minutes are seconds.
outside the 5 km radius. All seconds are hours.
(d) Government allows the construction of hotels within No second is a day.
5km radius of national parks and sanctuaries. Conclusions : I. No day is an hour.
(e) Such a step is taken by the environment ministry to II. At least some hours are minutes.
boost eco-tourism and perk up revenue collection of (44-45): Statements : Some teachers are professors.
State governments. Some lecturers are teachers.
44. Statements : I. All teachers as well as professors being
Directions (Q. 39):Read the following paragraph and answer lecturers is a possibility.
the question which follows. II. All those teachers who are lecturers are
39. Tenants’ associations have demanded a ‘vacancy tax’ on also professors.
all vacant and unsold flats in Mumbai. 45. Statements : I. No professor is a lecturer.
Which of the following would support the demand made II. All lecturers being professors is a
by the tenants' association? possibility.
www.newspaperkorner.wordpress.com
SBI (Associates) PO EXAM 2011 SOLVED PAPER P1-5
www.newspaperkorner.wordpress.com
Directions (Qs. 46-50): In each of the questions given below which one of the five Answer Figures on the right should come after
the Problem Figures on the left, if the sequence were continued?

PROBLEM FIGURES ANSWER FIGURES


46.
S C O B E B S O L O O L B
E E B K E S K O S L S S K
C S K L C S L O C E E E L C
O B K L L O B K C E S O L B K C E L C B K C B K C B K E S O

(a) (b) (c) (d) (e)

47.

(a) (b) (c) (d) (e)

48.

(a) (b) (c) (d) (e)

o o

c
49.

c
o c c o c o o o
c

c o
o o c
c

(a) (b) (c) (d) (e)

50. E L S S S
L L
E E H H L L
=HOLE H K C C L H L L
O H L C H
K H S S H

(a) (b) (c) (d) (e)

QUANTITATIVE APTITUDE 53. 25.05 × 123.95 + 388.999 × 15.001 =?


(a) 900 (b) 8950
Directions (Qs. 51-55): What will come in place of question
mark (?) in the following questions? (c) 8935 (d) 8975
(e) 8995
51. 32.05% of 259.99 =?
54. 561 ÷ 35.05 × 19.99 = ?
(a) 92 (b) 88
(a) 320 (b) 330
(c) 78 (d) 90
(e) 83 (c) 315 (d) 325
(e) 335
1 2 3
52. of of of 1715 = ?
8 3 5 55. (15.01)2 × 730 = ?
(a) 80 (b) 85 (a) 6125 (b) 6225
(c) 90 (d) 95 (c) 6200 (d) 6075
(e) 75 (e) 6250
www.newspaperkorner.wordpress.com
P1-6 SBI (Associates) PO EXAM 2011 SOLVED PAPER
www.newspaperkorner.wordpress.com
Directions (Qs. 56-60): In each of these questions, a number 65. Inside a square plot, a circular garden is developed which
series is given. In each series, only one number is wrong. Find exactly fits in the square plot and the diameter of the garden
out the wrong number. is equal to the side of the square plot which is 28 metres.
What is the area of the space left out in the square plot after
56. 3601 3602 1803 604 154 36 12
developing the garden?
(a) 3602 (b) 1803
(a) 98 m2 (b) 146 m2
(c) 604 (d) 154
(c) 84 m2 (d) 168 m2
(e) 36
(e) None of these
57. 4 12 42 196 1005 6066 42511
(a) 12 (b) 42 66. Amit and Sujit together can complete an assignment of data
(c) 1005 (d) 196 entry in five days. Sujit’s speed is 80% of Amit’s speed and
(e) 6066 the total key depressions in the assignment are 5,76,000.
58. 2 8 12 20 30 42 56 What is Amit’s speed in key depressions per hour if they
(a) 8 (b) 42 work for 8 hours a day?
(c) 30 (d) 20 (a) 4800 (b) 6400
(e) 12 (c) 8000 (d) 7200
59. 32 16 24 65 210 945 5197.5 (e) None of these
(a) 945 (b) 16 67. Out of five girls and three boys, four children are to be
(c) 24 (d) 210 randomly selected for a quiz contest. What is the probability
(e) 65 that all the selected children are girls?
60. 7 13 25 49 97 194 385
(a) 13 (b) 49 1 1
(a) (b)
(c) 97 (d) 194 14 7
(e) 25
5 2
61. Mr X invested a certain amount in Debt and Equity Funds (c) (d)
in the ratio of 4 : 5. At the end of one year, he earned a total 17 17
dividend of 30% on his investment. After one year, he (e) None of these
reinvested the amount including the dividend in the ratio of 68. Profit earned by an organisation is distributed among
6 : 7 in Debt and Equity Funds. If the amount reinvested in
officers and clerks in the ratio of 5 : 3. If the number of
Equity Funds was `94,500, what was the original amount
invested in Equity Funds? officers is 45 and the number of clerks is 80 and the amount
(a) `75,000 (b) `81,000 received by each officer is `25,000, what was the total
(c) `60,000 (d) `65,000 amount of profit earned?
(e) None of these (a) `22 lakh (b) `18.25 lakh
62. The age of the father is 30 years more than the son’s age. (c) `18 lakh (d) `3.25 lakh
Ten years hence, the father’s age will become three times (e) None of these
the son’s age that time. What is the son’s present age in
69. A shopkeeper labelled the price of his articles so as to
years?
(a) Eight (b) Seven earn a profit of 30% on the cost price. He,then sold the
(c) Five (d) Cannot be determined articles by offering a discount of 10% on the labelled
(e) None of these price. What is the actual per cent profit earned in the
63. If the length of a rectangular field is increased by 20% and deal?
the breadth is reduced by 20%, the area of the rectangle will
(a) 18% (b) 15%
be 192 m2. What is the area of the original rectangle?
(a) 184 m2 (b) 196 m2 (c) 20% (d) Cannot be determined
(c) 204 m 2 (d) 225 m2 (e) None of these
(e) None of these 70. Mr Shamin’s salary increases every year by 10% in June. If
64. The product of one-third of a number and 150% of another there is no other increase or reduction in the salary and his
number is what per cent of the product of the original salary in June 2011 was `22,385, what was his salary in June
numbers? 2009?
(a) 80% (b) 50% (a) `18,650 (b) `18,000
(c) 75% (d) 120% (c) `19,250 (d) `18,500
(e) None of these (e) None of these
www.newspaperkorner.wordpress.com
SBI (Associates) PO EXAM 2011 SOLVED PAPER P1-7
www.newspaperkorner.wordpress.com 75. What is the cost of flooring a rectangular hall?
Directions (Qs. 71-75): In each of these questions, one question
is given followed by data in three statements I, II and III. You Statements :
have to study the question and the data in statements and decide I. The length of the rectangle is 6 metres.
the question can be answered with data in which of the II. The breadth of the rectangle is two-thirds of its length.
statements and mark your answer accordingly. III. The cost of flooring the area of 100 cm2 is `45.
71. What is the rate of interest pcpa? (a) Only I and III
Statements: (b) Only II and III
I. The difference between the compound interest and (c) All I, II and III
simple interest earned in two years on the amount (d) Question cannot be answered even with data in all
invested is `100. three statements.
II. The amount becomes `19,500 in three years on simple (e) None of these
interest. Dircctions (Qs. 76-83): Study the following graph carefully to
III. The simple interest accrued in two years on the same answer these questions.
amount at the same rate of interest is `3,000.
Per cent profit earned by two companies producing
(a) Only I and II (b) Only I and III
electronic goods over the years
(c) Only II and III (d) Only I and either II or III
(e) None of these Profit Earned
% Profit = ´ 100
72. What is the speed of the train in kmph? Total Investment
Statements : Profit Earned = Total Income – Total Investment in the year
I. The train crosses an ‘x’ metre-long platform in ‘n’
seconds.
100
II. The length of the train is ‘y’ metres. 90 90 90
Per cent Profit Earned
III. The train crosses a signal pole in 'm' seconds. 80 80 80
(a) Any two of the three (b) Only II and III 70 70
65 75
(c) Only I and III (d) All I, II and III 60 55 70
50 60
(e) Either I and II or II and III 55 55
40 50
45
73. How many students passed in first class?
30
Statements : 20
I. 85% of the students who appeared in examination have 10
passed either in first class or in second class or in pass 0
class. 2004 2005 2006 2007 2008 2009 2010
II. 750 students have passed in second class.
Year
III. The number of students who passed in pass class is
76. If the profit earned in 2006 by Company B was `8,12,500,
28% of those passed in second class.
what was the total income of the company in that year?
(a) All I, II and III
(a) `12,50,000 (b) `20,62,500
(b) Only I and III
(c) `16,50,000 (d) `18,25,000
(c) Only II and III
(e) None of these
(d) Question cannot be answered even with information
77. If the amount invested by the two companies in 2005 was
in all three statements.
equal, what was the ratio of the total income of the Company
(e) None of these A to that of B in 2005?
74. What is the amount invested in Scheme ‘B’? (a) 31 : 33 (b) 33 : 31
Statements : (c) 34 : 31 (d) 14 : 11
I. The amounts invested in Schemes ‘A’ and ‘B’ are in (e) None of these
the ratio of 2 : 3.
78. If the total amount invested by the two companies in 2009
II. The amount invested in Scheme ‘A’ is 40% of the total was `27 lakh, while the amount invested by Company B
amount invested. was 50% of the amount invested by Company A, what was
III. The amount invested in Scheme ‘A’ is `45,000. the total profit earned by the two companies together?
(a) Only I and II (b) Only I and III (a) `21.15 lakh (b) `20.70 lakh
(c) Only II and III (d) All I, II and III (c) `18.70 lakh (d) `20.15 lakh
(e) Only III and either I or II (e) None of these
www.newspaperkorner.wordpress.com
P1-8 SBI (Associates) PO EXAM 2011 SOLVED PAPER
www.newspaperkorner.wordpress.com
79. If the income of Company A in 2007 and that in 2008 were 85. What is the average number of candidates (approximately)
equal and the amount invested in 2007 was `12 lakh, what found eligible for Post III from all states?
was the amount invested in 2008? (a) 6700 (b) 6200
(a) `10,87,500 (b) `10,85,700 (c) 4200 (d) 4500
(c) `12,45,000 (d) `12,85,000 (e) 5500
(e) None of these 86. What is the overall percentage (rounded off to one digit
80. If the amount of profit earned by Company A in 2006 was after decimal) of candidates short listed over the total number
`10. 15 lakh, what was the total investment? of candidates eligible for Post I from all the States together?
(a) `13.8 lakh (b) `14.9 lakh (a) 9.5% (b) 12.5%
(c) `15.4 lakh (d) `14.2 lakh (c) 7.2% (d) 6.5%
(e) None of these (e) None of these
81. If the amount invested by Company B in 2004 is `12 lakh 87. What is the ratio of the total number of candidates shortlisted
and the income of 2004 is equal to the investment in 2005, for all the posts together from State E to that from state G?
what is the amount of profit earned in 2005 by Company B? (a) 307 : 369 (b) 73 : 79
(a) `6.6 lakh (b) `18.6 lakh (c) 6 : 5 (d) 9 : 7
(c) `10.23 lakh (d) `9.6 lakh (e) None of these
(e) None of these 88. The total number of candidates found eligible for Post I
82. If the investments of Company A in 2007 and 2008 were from all states together is approximately what per cent of
equal, what is the difference between the profits earned in the total number of candidates found eligible for Post VI
from all States together?
the two years if the income in 2008 was `24 lakh?
(a) 45% (b) 50%
(a) `2.25 lakh (b) `3.6 lakh
(c) 60% (d) 55%
(c) `1.8 lakh (d) `2.6 lakh
(e) 65%
(e) None of these
89. Which state had the lowest percentage of candidates short
83. If each of the companies A and B invested `25 lakh in 2010, listed with respect to candidate eligible for Post IV?
what was the average profit earned by the two companies? (a) G (b) F
(a) `18 lakh (b) `22.5 lakh (c) E (d) C
(c) `17.5 lakh (d) `20 lakh (e) None of these .
(e) None of these 90. What is the ratio of the total number of candidates short
Directions (Qs. 84-90): Study the following table carefully and listed for post V to that for post VI from all states together?
answer the questions which follow. (a) 6 : 7 (b) 55 : 96
(c) 165 : 278 (d) 16 : 25
Number of Candidates found Eligible and the
(e) None of these
Number of Candidates Short listed for Interview for a
recent Recruitment Process for Six Posts from Directions (Qs. 91-95) : These questions are based on the
following data. Study it carefully and answer the questions that
Different States follow.
E-Eligible S- Short listed
In a school having 400 students, boys and girls are in the
ratio of 3 : 5. The students speak Hindi, English or both the
Post 1 II III IV V VI
languages. 12% of the boys speak only Hindi. 22% of the girls
S t a te E S E S E S E S E S E S speak only English. 24% of the total students speak only Hindi
and the number of boys speaking both the languages is six times
A 2500 65 7200 240 5200 76 3600 200 4600 110 5400 380 the number of boys speaking only Hindi.
B 3200 220 8500 420 8400 190 6200 320 5800 180 6200 430 91. How many boys speak Hindi?
C 2800 280 4500 350 7600 160 8200 440 7300 310 3700 250 (a) 18 (b) 126
D 2400 85 4800 200 2600 55 7500 350 3900 160 4800 360 (c) 108 (d) 26
E 3000 120 5600 280 3800 75 6800 280 6100 260 7800 520 (e) None of these
F 4800 325 6400 320 4400 220 4700 180 4900 220 8800 640
92. How many girls speak only Hindi?
(a) 55 (b) 117
G 6500 550 7000 140 6000 325 5500 220 8100 410 2700 200
(c) 96 (d) 78
84. From State B, which post had the highest percentage of (e) None of these
candidates short listed? 93. How many students speak English?
(a) V (b) IV (a) 304 (b) 79
(c) VI (d) II (c) 225 (d) 117
(e) None of these (e) None of these
www.newspaperkorner.wordpress.com
SBI (Associates) PO EXAM 2011 SOLVED PAPER P1-9
www.newspaperkorner.wordpress.com
94. The number of girls speaking only Hindi is what per cent of Directions (Q. 99) : Study the following pie-chart to answer the
the total number of students speaking only Hindi? question that follow.
(a) 38.2% (b) 71.8% Break-up of students having preference for each subject
(c) 31.2% (d) 78%
(e) None of these
95. What is the ratio of the number of boys to the number of
girls speaking both the languages?
Geography
(a) 23 : 25 (b) 12 : 25
History 28%
(c) 12 : 13 (d) 25 : 13 44%
(e) None of these
Economics
Directions (Qs. 96-100): Study the information given in eaeh of 16%
these questions and then answer the questions. Maths
12%
96. The area of the circle is 616 cm2. What is the area of the
rectangle? (• or ‘dot’ or indicates centre of the circle the.)
99. Out of a total 550 students, how many students did not
prefer Maths or Economics?
(a) 462 (b) 154
(c) 196 (d) 396
(e) None of these

100. Pers on Salary (in ` Iakh )


(a) 784 cm2 (b) 196 cm2 A 8.5
(c) 392 cm2 (d) Cannot be determined B 7.6
(e) None of these C 12.8
D 5.4
97. Population in Million E 10.5
City Total Population Male Population What is the difference (in ` lakh) between the average salary
A 12 6.5 and the lowest salary?
B 15 7.2 (a) 4.2 (b) 2.65
C 17 9.0 (c) 3.65 (d) 4.06
D 19 9.9 (e) None of these
E 22 10.8 GENERAL AWARENESS WITH SPECIAL
REFERENCE TO BANKING INDUSTRY
What is the average female population in million?
101. A major Public Sector Bank raised interest rates on loans by
(a) 8.32 (b) 8.86
25 basis points – was news in some major financial
(c) 8.68 (d) 9.12 newspapers recently. This means the bank has raised interest
(e) None of these by 25 basis points of
98. What is the per cent rise in production in 2007 from 2006? (a) Savings Bank Interest Rate
(Round off to two digits after decimal.) (b) Base Lending Rate
(c) Repo Rates
1400 (d) Present Rates on Deposits
1300 (e) Discounted Rates of Interest
1200 102. The Finance Minister of India in one of his press
1100 conferences said that inflationary pressure is likely to
1000 continue following recent increase in rates of some
900 commodities in international markets. Which of the following
800 commodities was he referring to as it gets frequent increase
2005 2006 2007 2008 2009 2010 at international levels and disturbs our Home Economy
substantially?
(a) 28.18% (b) 18.18% (a) Gold and Silver (b) Petroleum products
(c) 16.28% (d) 26.18% (c) Tea and Coffee (d) Sugar
(e) None of these (e) Jute and Jute products
www.newspaperkorner.wordpress.com
P1-10 SBI (Associates) PO EXAM 2011 SOLVED PAPER
www.newspaperkorner.wordpress.com
103. Immediately prior to change in the measure of Food Inflation, 111. Which of the following international agencies/ organizations
which of the following indexes was being used for measuring had initiated an 'International Convention for the Protection
it? of All Persons from Enforced Disappearance', which came
(a) Wholesale Price Index into effect recently?
(b) Consumer Price Index
(a) World Health Organisation (WHO)
(c) Interest Rates offered by banks on deposits
(d) Sensex and Nifty (b) International Court of Justice
(e) None of these (c) International Maritime Organisation
104. What is the full form of ‘FINO’, a term we see frequently in (d) International Atomic Energy Agency (IAEA)
financial newspapers? (e) United Nations Organisation (UNO)
(a) Financial Investment Network and Operations 112. ‘BRIC’ which was the organisation of 4 nations, namely
(b) Farmers’ Investment in National Organisation
Brazil, Russia, India and China, has now become 'BRICS',
(c) Farmers Inclusion News and Operations
Which is the fifth nation included in it?
(d) Financial Inclusion Network and Operations
(e) None of these (a) South Korea (b) Sri Lanka
105. Which of the following is/are commodity/ies on which (c) Singapore (d) Spain
subsidy is given to farmers/poor in India? (e) South Africa
(1) Fertilizer 113. The process of acquisition of agricultural land and its
(2) Kerosene compensation to farmers recently came up as a major issue
(3) LPG in which of the following states?(It also created law and
(a) Only (1) (b) Only (2)
order problem in the State.)
(c) Only(3) (d) Only (1) and (3)
(e) All (1), (2) and (3) (a) Haryana (b) Gujarat
106. ‘Bhagyam Oilfields’, which were recently in news, are (c) Uttar Pradesh (d) Rajasthan
located in which of the following states in India? (e) Punjab
(a) Rajasthan (b) Gujarat 114. What does the letter 'L' denote in the term ‘LAF’ as referred
(c) Assam (d) Maharashtra every now and then in relation to monetary policy of the
(e) Karnataka RBI?
107. The Finance Minister of India recently decided to review
(a) Liquidity (b) Liability
the position of ‘Bad Debts’ in priority sector. Which of the
following is/are the reason(s) owing to which FM took this (c) Leveraged (d) Longitudinal
decision? (e) Linear
(1) Bad Debt in agriculture is still rising substantially every 115. BP Plc, which was in news recently, is a major international
year despite special treatment to it. company in the field of
(2) Bad Debt in other areas of priority sector is almost nil (a) Heavy Machinery
now.
(b) Oil exploration and processing
(3) Govt is planning to disburse another instalment of
`1,60,000 crore, which will bring 'Bad Debts' almost at (c) Atomic Energy
'Nil level' and no special treatment will be needed in (d) Information Technology
next fiscal year. (e) None of these
(a) Only (1) (b) Only (2) 116. Who amongst the following has been appointed the new
(c) Only (3) (d) Both (1) and (2) coach of the Indian Cricket team for a period of two years?
(e) All (1), (2) and (3) (a) Stuart Law (b) Allan Border
108. Which amongst the following countries is NOT a member
(c) Gary Kirsten (d) Duncan Fletcher
of the TAPI pipeline project, which was in news recently?
(a) India (b) Afghanistan (e) Allan Donald
(c) Portugal (d) Pakistan 117. Why was Kanishtha Dhankhar's name in news recently?
(e) Turkmenistan (a) She was crowned Femina Miss India 2011.
109. An agreement on Arms Reduction popularly known as (b) She was adjudged Best Actress 2010 by Filmfare
'START' is one of the major issues in the way of smooth (c) She is the new captain ofIndia's woman cricket team
relations between which of the following two countries?
(d) She has become the new Dy Minister of Textile in place
(a) India - Pakistan (b) India - China
of Dayanidhi Maran
(c) China - Japan . (d) USA - Russia
(e) Russia - Iran (e) None of these
110. The International WWW Conference-2011 was organized 118. K Balachander, who was awarded prestigious Dada Saheb
in which of the following places in March 2011? Phalke Award 2010, is basically a/ an
(a) London (b) Paris (a) Actor (b) Director
(c) Vienna (d) Manila (c) Photographer (d) Music Director
(e) Hyderabad (e) Choreographer
www.newspaperkorner.wordpress.com
SBI (Associates) PO EXAM 2011 SOLVED PAPER P1-11
www.newspaperkorner.wordpress.com
119. Banks and other financial institutions in India are required 127. An online discussion group that allows direct "live" ,
to maintain a certain amount of liquid assets like cash, communication is known as
precious metals and other short-term securities as a reserve (a) Webcrawler
all the time. In Banking World this is known as (b) chat group
(a) CRR (b) Fixed Asset (c) regional service provider
(c) SLR (d) PLR (d) hyperlink
(e) None of these (e) e-mail
120. Which of the following space agencies recently launched 128. Which of the following is a program that uses a variety of
three satellites and placed them into Polar Sun Synchronous different approaches to identify and eliminate spam?
Orbit successfully? (a) Directory search
(a) NASA (b) ROSCOSMOS (b) Anti-spam program
(c) Web server
(c) CNSA (d) ISRO
(d) Web storefront creation package
(e) JAXA
(e) Virus
121. As per the decision taken by the Govt of India recently,
129. Connection or link to other documents or Web Pages that
“Bamboo is a liberated item now”. What does it really mean?
contain related information is called
(1) Bamboo, henceforth, will be treated as a minor forest
(a) dial-up (b) electronic commerce
produce only.
(c) hyperlink (d) e-cash
(2) Now villagers are free to sell bamboo in their community (e) domain name
forest areas.
130. Which of the following is a programming language for
(3) Gram Sabha has been given right to issue transport creating special programs like applets?
passes to legally transport the produce. (a) Java (b) cable
(a) Only (1) (b) Only (2) (c) domain name (d) Net
(c) Only (3) (d) All (1), (2) and (3) (e) COBOL
(e) None of these 131. The system unit
122. Union Cabinet recently passed a `7000-crore project to clean (a) coordinates input and output devices
up which of the following rivers in India? (b) is the container that houses electronic components
(a) Narmada (b) Ravi (c) is a combination of hardware and software
(c) Ganga (d) Chambal (d) controls and manipulates data
(e) Krishna (e) does the arithmetic operations
123. As per estimates of the Planning Commission of India, the 132. System software
target growth rate of 12th Five Year Plan is set at (a) allows the user to diagnose and troubleshoot the device
(a) 7 to 8% (b) 8.0% to 8.5% (b) is a programming language
(c) 9% to 9.5% (d) 10% to 10.5% (c) is part of a productivity suite
(e) 11% (d) is an optional form of software
124. Goodluck Jonathan was recently re-elected the President (e) helps the computer manage internal resources
of 133. Computer and communication technology, such as
communication links to the Internet, that provide help and
(a) South Africa (b) Nigeria
understanding to the end user is known as
(c) Kosovo (d) Haiti
(a) presentation file (b) information technology
(e) None of these (c) program (d) worksheet file
125. Which of the following schemes has NOT been launched (e) FTP
by the Ministry of Rural Development, Govt of india? 134. Which of the following is contained on chips connected to
(a) National Old Age Pension Scheme the system board and is a holding area for data instructions
(b) National Maternity Benefit Scheme and information? (processed data waiting to be output to
(c) National Family Benefit Scheme secondary storage)
(d) Mid-Day Meal Scheme (a) program (b) mouse
(e) All are launched by Ministry of Rural Development (c) Internet (d) memory
(e) modem
126. Where are you likely to find an embedded operating system?
135. Microsoft's Messenger allows users to
(a) on a desktop operating system (a) bypass a browser to surf the Web
(b) on a networked PC (b) create a blog
(c) on a network server (c) communicate via direct live communication
(d) on a PDA (d) identify and eliminate spam
(e) on a mainframe (e) make graphic presentations
www.newspaperkorner.wordpress.com
P1-12 SBI (Associates) PO EXAM 2011 SOLVED PAPER
www.newspaperkorner.wordpress.com
136. Portable computer, also known as laptop computer, 146. Social Marketing means
weighing between 4 and 10 pounds is called (a) Share market prices
(a) general-purpose application (b) Marketing by the entire society
(b) Internet (c) Internet Marketing
(c) scanner (d) Marketing for a social cause
(d) printer (e) Society bye-laws
(e) notebook computer 147. Service Marketing is the same as
137. The main circuit-board of the system unit is the (a) Internet Marketing (b) Telemarketing
(a) computer program (b) control unit (c) Internal Marketing (d) Relationship Marketing
(c) motherboard (d) RAM (e) Transaction Marketing
(e) None of these 148. Market-driven strategies include
138. Which of the following is billionth of a second? (a) Identifying problems
(a) Gigabyte (b) Terabyte (b) Planning marketing tactics of peers
(c) Nanosecond (d) Microsecond (c) Positioning the Organisation and its brands in the
(e) Terasecond marketplace
139. Online Marketing is the function of which of the following? (d) Internal marketing
(a) Purchase Section (b) Production Department (e) Selling old products
(c) IT Department (d) Designs Section 149. Innovation in marketing is same as
(e) A collective function of all staff (a) Motivation (b) Perspiration
140. Customisation is useful for
(c) Aspiration (d) Creativity
(a) Designing customer specific products
(e) Team work
(b) Call centres
150. Personal Loans can be canvassed among
(c) Publicity
(a) Salaried persons (b) Pensioners
(d) Motivating the staff
(c) Foreign Nationals (d) NRI customers
(e) Cold calls
(e) Non-customers
141. The key challenge to market-driven strategy is
(a) Selling maximum products GENERAL ENGLISH
(b) Employing maximum DSAs
(c) Delivering superior value to customers Directions (Qs. 151-160): Read the following passage carefully
(d) Being rigid to changes and answer the questions given below it. Certain words/phrases
have been printed in bold to help you locate them while
(e) Adopting short-term vision
answering some of the questions.
142. Effective selling skills depends on
(a) Size of the sales team Jagir Singh has sold red onions at a market in south Delhi
(b) Age of the sales team every day for the past half-century. Perched on an upturned crate,
(c) Peer strength wrapped tight against the chill air, he offers pyaz, a staple for
much Indian cooking, for 60 rupees a kilo, the most he can
(d) Knowledge level of the sales team
remember. Business is brisk but most customers pick up only a
(e) Educational level of the sales team
small handful of onions. That is just as well. Wholesale supplies
143. Generation of sales leads can be improved by
are tight, he says, and the quality is poor.
(a) Being very talkative
As India's economy grows by some 9% a year, food prices
(b) Increasing personal and professional contacts are soaring. In late December, the commerce ministry judged that
(c) Being passive food inflation had reached 18.3%, with pricey vegetables mostly
(d) Engaging Recovery Agents to blame. Officials have made some attcmpts to temper the rise in
(e) Product designs the past month-scrapping import taxes for onions, banning their
144. A Market Plan is export and ordering low-priced sales at government-run shops.
(a) Performance Appraisal of marketing staff But there is no quick fix.
(b) Company Prospectus Heavy rain in the west of lndia brought a rotten harvest.
(c) Documented marketing strategies Vegetables from farther a field-including a politically sensitive
(d) Business targets delivery from a neighbouring country – are costly to move on
(e) Call centre India’s crowded, pot-holed roads. Few refrigerated lorries and
145. Marketing channels mean poor logistics mean that much of each harvest is wasted.
(a) Delivery objects (b) Sales targets Newspapers allege hoarders are cashing in.
(c) Deliavery outlets (d) Delivery boys The biggest problems are structural. Food producers,
(e) Sales teams hampered by land restrictions, archaic retail networks and bad
www.newspaperkorner.wordpress.com
SBI (Associates) PO EXAM 2011 SOLVED PAPER P1-13
www.newspaperkorner.wordpress.com
infrastructure fail to meet extra demand from consumers. It was 156. What can be said about the sale of onions at present as
estimated in October that a 39% rise in income per person in the given in the passage?
previous five years might have created an extra 220 million regular (a) Vegetable vendors are unwilling to sell onions.
consumers of milk, eggs. meat and fish. Supplies have not kept (b) People are not buying as much as they used to.
up with this potential demand. (c) The sale of onions has picked up and is
The broader inflation rate may be a less eye-watering unprecedented.
problem than the onions suggest. The central bank has lifted (d) People are buying more onions than they used to.
interest rates steadily in the past year and is expected to do so (e) None of these
again later this month. Headline inflation fell to 7.5% in November, 157. The usage of the phrase ‘cashing in’ in the passage can
down by just over a percentage point from October. though it is possibly mean
still above the central bank's forecast of 5.5% for March. (a) Profiting (b) Running away
151. What is responsible for the increased demand of certain (c) Paying money (d) Bailing out
food items amongst consumers? (e) Buffering
(a) There has been an increase in the wholesale supplies 158. Which of the following is most similar in meaning to the
of this food stuff. word ‘tight’ as used in the passage?
(b) The vegetables in the market are very highly priced. (a) Firm (b) Loose
(c) There has been an increase in the incomes of people. (c) Limited (d) Taut
(d) There is a lack of availability of vegetables in the market. (e) Tense
(e) There has been a surge in population. 159. Which of the following is most similar in meaning to the
152. Which of the following is not true in the context of the word ‘temper’ as used in the passage?
passage? (a) Displeasure (b) Anger
(a) The overall inflation is not as bad as the food inflation (c) Rage (d) Harness
in India. (e) Control
(b) Help from other countries to counter food inflation 160. Which of the following is most opposite in meaning of the
word ‘archaic’ as. used in the passage?
has proved to be quite encouraging.
(a) Simple (b) Straightforward
(c) Government is banning the export of certain types of
(c) Modern (d) Lively
vegetables in order to check food inflation.
(e) Ancient
(d) Highly priced vegetables are mostly responsible for
the increased food inflation. Directions (Qs. 161-165): Which of the phrases (a), (b), (c) and
(e) All the above statements are true (d) given below each sentence should replace the phrase printed
153. Which of the following is/are the reason/s for increase in in bold in the sentence to make it grammatically correct? If the
food/vegetable prices? sentence is correct as it is given and no correction is required,
mark (e) as the answer.
(1) Bad weather
(2) Land restrictions 161. Seeing that there was an ongoing sale in one of her favourite
stores, Seeta made a bee line for it immediately after entering
(3) Poor infrastructure for storage and transportation
the mall.
(a) Only (2) (b) Only (1) and (3)
(a) made a bee’s line for
(c) Only (1) and (2) (d) Only(2) and (3)
(b) make bees lined to
(e) All (1), (2) and (3)
(c) made a bee line to
154. Which of the following is/are TRUE in the context of the
(d) make bee line to
passage?
(e) No correction required
(1) Interest rates are being raised in India.
162. Sharon made it to work in the nicks of times, or else she
(2) India is witnessing a steady economic growth. would have missed the meeting.
(3) It has been proven that the food inflation is mainly (a) nick of time (b) nicked time
because of hoarding. (c) nick of timeliness (d) nick and time
(a) All (1), (2) and (3) (b) Only (1) and (2) (e) No correction required
(c) Only (3) (d) Only (2) and (3) 163. Varun was on cloud nine after having stood first in his class.
(e) None is true (a) in ninth cloud (b) on nine clouds
155. Which of the following is possibly the most appropriate (c) a cloudy nine (d) cloud on nine
title for the passage? (e) No correction required
(a) Food Inflation In India 164. Vithal had a habit of pass the buck when it came to important
(b) Onions And Vendors issues at work.
(c) Food Deficit Worldwide (a) pass to bucking (b) passing buck
(d) Food Imports In India (c) passing the buck (d) pass buck
(e) Benefits To Indian Consumers (e) No correction required
www.newspaperkorner.wordpress.com
P1-14 SBI (Associates) PO EXAM 2011 SOLVED PAPER
www.newspaperkorner.wordpress.com
165. Puneet raked his brains and tried to find an answer to a India's main competitors-especially China, but also
tricky question given in the paper but couldn't find one. Singapore, Taiwan, and South Korea-are investing in large and
(a) rake his brain (b) racked his brains differentiated higher education systems. They are providing
(c) racked brains (d) raked brain access to a large number of students at the bottom of the academic
(e) No correction required system while at the same time building some research-based
Directions (Qs. 166-170): Each question below has two blanks, universities that are able to compete with the world's best
each blank indicating that something has been omitted. Find institutions. The recent London Times Higher Education
out which option can be used to fill up the blank in the sentence Supplement ranking of the world's top 200 universities included
in the same sequence to make it meaningfully complete. three in China, three in Hong Kong. three in South Korea, one in
166. The water transport project on the west coast is ____ to Taiwan, and one in India. These countries are positioning
get a shot in the arm with a new plan in which the Road themselves for leadership in the knowledge-based economies of
Development Corporation will build the infrastructure and the coming era.
___ a private party to operate the service. There was a time when countries could achieve economic
(a) scheduled, let (b) verge, permit success with cheap labour and low-tech manufacturing. Low
(c) set, sanctions (d) slated, allow wages still help, but contemporary large-scale development
(e) bound, task requires a sophisticated and at least partly knowledge-based
167. As the weekend finally rolled around. the city folk were economy. India has chosen that path, but will find a major
only ___ happy to settle down and laugh their cares _____. stumbling block in its university system.
(a) just, afar (b) too, away
India has significant advantages in the 21st century
(c) extremely, off (d) very, up
knowledge race. It has a large higher education sector – the third
(e) so, on
largest in the world in terms of number of students, after China
168. The flood of brilliant ideas has not only ___ us, but has
and the United States. It uses English as a primary language of
also encouraged us to ___ the last date for submission of
entries. higher education and research. It has a long academic tradition.
(a) overwhelmed, extend (b) enjoyed, stretch Academic freedom is respected. There are a small number of high-
(c) dismayed, decide (d) scared, scrap quality institutions, departments, and centres that can form the
(e) happy, boundary basis of quality sector in higher education. The fact that the
169. ___ about prolonged power cuts in urban areas, the States, rather than the Central Government, exercise major
authorities have decided to ___ over to more reliable and responsibility for higher education creates a rather cumbersome
eco-friendly systems to run its pumps. structure, but the system allows for a variety of policies and
(a) Worried, shift (b) Frantic, move approaches.
(c) Troubled, jump (d) Concerned, switch Yet the weaknesses far outweigh the strengths. India
(e) Endangered, click educates approximately 10 per cent of its young people in higher
170. The high cutoff marks this year have ___ college admission- education compared to more than half in the major industrialised
seekers to either ___ for lesser known colleges or change countries and 15 per cent in China. Almost all of the world's
their subject preferences. academic systems resemble a pyramid, with a small, high-quality
(a) cajoled, ask (b) pressured, sit tier at the top and a massive sector at the bottom. India has a tiny
(c) forced, settle (d) strained, compromise top tier. None of its universities occupies a solid position at the
(e) entrusted, wait top. A few of the best universities have some excellent departments
Directions (Qs. 171-185): Read the following passage carefully and centres, and there are a small number of outstanding
and answer the questions given below it. Certain words/phrases undergraduate colleges. The University Grants Commission's
have been printed in bold to help you locate them while recent major support to five universities to build on their
answering some of the questions. recognised strength is a step towards recognising a differentiated
India is rushing headlong towards economic success and academic system and fostering excellence. These universities,
modernisation, counting on high-tech industries such as combined, enrol well under one per cent of the student population.
information technology and biotechnology to propel the nation 171. Which of the following is TRUE in the context of the
to prosperity. India's recent announcement that it would no longer passage?
produce unlicensed inexpensive generic pharmaceuticals bowed (a) The top five universities in India educate more than 10
percent of the Indian student population.
to the realities of the World Trade Organisation while at the same
(b) India’s higher education sector is the largest in the
time challenging the domestic drug industry to compete with the
world.
multinational firms. Unfortunately, its weak higher education
(c) In the past, countries could progress economically
sector constitutes the Achilles’ heel of this strategy. Its systematic
through low manufacturing cost as well as low wages
disinvestment in higher education in recent years has yielded·
of labourers.
neither world-class research nor very many highly trained
(d) India has recently invested heavy sums in the higher
scholars, scientists or managers to sustain high-tech
education sector leading to world-class research.
development.
(e) All are true
www.newspaperkorner.wordpress.com
SBI (Associates) PO EXAM 2011 SOLVED PAPER P1-15
www.newspaperkorner.wordpress.com
172. What does the phrase ‘Achilles’ heel’ mean as used in the (a) Only (1) and (2) (b) Only (2)
passage? (c) Only (3) (d) Only (1) and (3)
(a) Weakness (b) Quickness (e) All (1), (2) and (3)
(c) Low quality (d) Nimbleness 178. Which of the following, according to the passage, is/are
(e) Advantage needed for the economic success of a country?
173. Which of the following is/are India's strength/s in terms of (1) Cheap labour
higher education? (2) Educated employees
(1) Its system of higher education allows variations. (3) Research institutions to cater to the needs of
(2) Medium of instruction for most higher learning is development
English. (a) Only (1) and (2) (b) Only (2)
(3) It has the paraphernalia, albeit small in number, to build (c) Only (3) (d) Only (2) and (3)
a high-quality higher educational sector.
(e) All (1), (2) and (3)
(a) Only (2) (b) Only (1) and (2)
(c) Only (3) (d) Only (2) and (3) Directions (Qs. 179-182): Choose the word of words which is
(e) AlI (1), (2) and (3) most similar in meaning to the word of words printed in bold as
174. What are the Asian countries, other than India, doing to used in the passage.
head towards a knowledge-based economy? 179. FOSTERlNG
(1) Building competitive research-based universities (a) Safeguarding (b) Neglecting
(2) Investing in diverse higher education systems (c) Sidelining (d) Nurturing
(3) Providing access to higher education to select few
(e) Ignoring
students
180. PROPEL
(a) Only (1) (b) Only (1) and (2)
(c) Only (2) and (3) (d) Only (2) (a) Drive (b) Jettison
(e) All (1), (2) and (3) (c) Burst (d) Acclimatise
175. Which of the following is possibly the most appropriate (e) Modify
title for the passage? 181. MASSIVE
(a) The Future of Indian Universities (a) Lump sum (b) Strong
(b) Methods of overcoming the Educational Deficit in India (c) Little (d) Gaping
(c) India and the Hunt for a Knowledge-Based Economy
(e) Huge
(d) Indian Economy Versus Chinese Economy
182. STUMBLING BLOCK
(e) Indian Economy and Its Features
176. What did India agree to do at the behest of the World Trade (a) Argument (b) Frustration
Organisation? (c) Advantage (d) Hurdle
(a) It would stop manufacturing all types of (e) Fallout
pharmaceuticals. Directions (Qs. 183-185): Choose the word/group of words
(b) It would ask its domestic pharmaceutical companies which is most opposite in meaning to the word/group of words
to compete with the international ones.
printed in bold as used in the passage.
(c) It would buy licensed drugs only from the USA.
(d) It would not manufacture cheap common medicines 183. CUMBERSOME
without a licence. (a) Handy (b) Manageable
(e) None of these (c) Breathtaking (d) Awkward
177. Which of the following is/are India's weakness/es when it (e) Difficult
comes to higher education?
184. RESEMBLE
(1) Indian universities do not have the requisite teaching
(a) Against (b) Similar to
faculty to cater to the needs of the higher education
sector. (c) Mirror (d) Differ from
(2) Only five Indian universities occupy the top position (e) Unfavourable to
very strongly in the academic pyramid when it comes 185. DIFFERENTIATED
to higher education. (a) Similar (b) Varied
(3) India has the least percentage of young population
(c) Harmonized (d) Synchronized
taking to higher education as compared to the rest of
(e) Discriminated
the comparable countries.
www.newspaperkorner.wordpress.com
P1-16 SBI (Associates) PO EXAM 2011 SOLVED PAPER
www.newspaperkorner.wordpress.com Around the world, forests are being (191) at a rate of about
Directions (Qs. 186-190): Rearrange the following six sentences
(1), (2), (3), (4), (5) and (6) in the proper sequence to form a thirteen million hectares a year and deforestation accounts for an
meaningful paragraph; then answer the questions given below estimated 17%-20% of all global emissions. In addition, forests
them. and other terrestrial carbon sinks play a (192) role in preventing
runaway climate change, soaking up a full 2.6 Gt of atmospheric
(1) Its prevalence reflects very badly on a society that is
carbon every year. The destruction of forests, therefore, not only
not able to stop this evil.
emits carbon–a staggering 1.6 Gt a year, which severely (193)
(2) Though elimination of child labour is an impossible forests' capacity to absorb emissions from other sources-but also
task considering the current socio-economic scenario drastically (194) the amount of forested land available to act as a
of these poor families, the Indian government is carbon sink in the future.
committed to the task of ensuring that no child remains However, the effects of deforestation extend beyond carbon.
illiterate, hungry and without medical care.
Rainforests (195) a wide variety of ecosystems services, from
(3) Therefore, unless the socio-economic status of the regulating rainfal1 to purifying groundwater and keeping fertile
poor families is improved. India has to live with child soil from (196); deforestation in one area can seriously damage
labour. food production and (197) to clean water in an entire region. The
(4) The members of these households have to send their value of global ecosystem services has been estimated at 33
children to work, even if the future of these innocent trillion USD each year (almost half of global GDP), but these
children is ruined, as that is the only choice open for services have been taken for granted without a mechanism to
them to survive in this world. make the market reflect their value. Rainforests are also a home
(5) Child labour is, no doubt, an evil that should be done and (198) of income for a huge number of people in Africa, Asia,
away with at the earliest. and South America. (199) this, economic pressures frequently
(6) But in a society where many households may have to drive both locla communities and national government in the
suffer the pangs of hunger if the children are withdrawn developing world to (200) these forests in ways that are
from work, beggars can’t be choosers. unsustainable, clear-cutting vast areas for fuel, timber, mining, or
186. Which of the following should be the FIRST sentence after agricultural land
rearrangement? 191. (a) ended (b) destroyed
(a) 1 (b) 5 (c) extinct (d) killed
(c) 3 (d) 6 (e) wasted
(e) 4 192. (a) tough (b) important
187. Which of the following should be the THIRD sentence (c) vital (d) biggest
after rearrangement? (e) effective
(a) 2 (b) 1 193. (a) affects (b) diminishes
(c) 3 (d) 6 (c) increases (d) alternates
(e) 5 (e) impairs
188. Which of the following should be the SECOND sentence 194. (a) plagues (b) develops
after rearrangement? (c) reduces (d) shortens
(a) 1 (b) 6 (e) influences
(c) 4 (d) 2 195. (a) sell (b) offer
(e) 3 (c) give (d) provide
189. Which of the following should be the FIFTH sentence after (e) earns
rearrangement? 196. (a) transforming (b) decoding
(a) 5 (b) 1 (c) erupting (d) draining
(c) 6 (d) 3 (e) eroding
(e) 4 197. (a) handiness (b) excess
190. Which of the following should be the SIXTH (LAST) (c) availability (d) access
sentence after rearrangement? (e) supply
(a) 1 (b) 3 198. (a) beginning (b) source
(c) 5 (d) 4 (c) ways (d) reference
(e) 2 (e) measure
199. (a) Despite (b) Also
Directions (Qs. 191-200): In the following passage, there are
(c) Inspite (d) Apart
blanks, each of which has been numbered. These numbers are (e) Beside
printed below the passage and against each, five words are 200. (a) exploit (b) encompass
suggested, one of which fits the blank appropriately. Find out
(c) nurture (d) work
the appropriate word in each case.
(e) improve
www.newspaperkorner.wordpress.com
SBI (Associates) PO EXAM 2011 SOLVED PAPER P1-17
www.newspaperkorner.wordpress.com

Time : 60 min. Max. Marks : 50

1. Write a letter on any one of the following topics. (10 marks)


(a) Write a letter to the manager of a small firm to increase the credit amount of loan taken.
(b) Write a letter to the manager of the bank to apologize for the delayed payment of EMI.
(c) Write a letter to your brother indicating him as to why I want to choose banking as a career.
2. Write a paragraph in about 130 words on one of the following. (8 marks)
(a) Cashless cards for students.
(b) Advantages of Internet banking.
(c) Can India achieve the status of most developed economy?
3. Write an essay in about 250 words on one of the following topics. (12 marks)
(a) Does providing education loan helps in empowering employment to the youth.
(b) The AADHAR UID card. How it is useful will it be?
(c) Recent unearthing of Black money and its impact on India Economy.
4. Write a précis in about 150 words and provide a suitable title. (10 marks)
The Reserve Bank of India asked private and foreign lenders to put in place a system of internal vigilance machinery and appoint
a Chief of Internal Vigilance (CIV) by August to check frauds and malpractices. The directive aims at bringing in private and
foreign banks in line with the practices followed by state-owned banks to check frauds.
“In an Endeavour to align the vigilance function in private sector and foreign banks to that of the public sector banks the
existing vigilance functions of a few private sector and foreign banks were mapped with the existing guidelines in the matter and
it was observed that the practices vary widely among the banks,” RBI said in a notification.
“It has therefore been decided to lay down detailed guidelines for private sector and foreign banks on similar lines so that all
issues arising out of lapses in the functioning of the private sector and foreign banks especially relating to corruption,
malpractices, frauds etc can be addressed uniformly by the banks for timely and appropriate action,” it said.
In a separate notification, RBI asked all the banks and financial institutions to streamline procedure for detection of fraud and
recovery of misappropriated fund. The matter assumes significance in the light of Rs 460 crore frauds in Citibank's Gurgaon
branch discovered last year. The Central Vigilance Commission has issued guidelines to public sector banks on the appointment
of Chief Vigilance Officer (CVO).
The purpose of having such system in place is to ensure that all the internal vigilance functions in the public sector banks are
addressed through a set of pre-determined and structured procedures to ensure comprehensive treatment and transparency.
As per the guideline, the designated officer similar to CVO in case of public sector banks acts as a special assistant or advisor
to the CEO of the concerned bank in the discharge of these functions. An officer of suitable seniority is required to be
designated as Chief of Internal Vigilance (CIV) who will head the Internal Vigilance Division of the bank, it said.
RBI on staff rotation policy:
In a circular to banks, the RBI said the banks should immediately put in place “staff rotation” policy and policy for “mandatory
leave” for staff. The internal auditors as also the concurrent auditors must be specifically required to examine the implementa-
tion of these policies and point out instances of breaches irrespective of apparent justifications for non-compliance, if any.
5. Read the following passage and answers the questions. (10 marks)
ATMs (Automated Teller Machines) are an integral part of the banking system. They are an important aspect of the banking
delivery channel. Therefore these days banks are very concerned about this in the proper organizing of ATMs. ATMs not only
provide usual paying facilities but also help in paying tax payments, mobile recharge, credit card bill payment. The Breston
Boston marketing survey stated that about 49% Indians will switch to another competitive bank if they were provide with the
facilities of personalizing accounts more effectively and some other facilities. ATMs technology is highly superior and conve-
nient to the customers. You can also pay you credit card bill through ATMs but you need to have a debit card also for that. Just
go to you near ATMs and proceed with payment and after checking the fund payment is done and you can receive the
confirmation receipt after that. Now with ATMs income tax payment is pretty easy. Just go to bank website and register yourself
for the online tax payment and go to the nearest ATM and select under the income tax tab and then the payment options and
after the payment go to the bank and take print out of the challan with a SIN which was generated earlier and can be shown while
filing the return later on.
(i) Why banks so much concerned about the proper organizing of the ATMs?
(ii) What was stated in the Breston Boston survey?
(iii) What kind of facilities are provided by ATMs?
(iv) How ATMs help in credit card payments
(v) How ATMs help in income tax payments?
www.newspaperkorner.wordpress.com
P1-18 SBI (Associates) PO EXAM 2011 SOLVED PAPER
www.newspaperkorner.wordpress.com
Answers & Explanations
1. (b) All others are synonyms. 15. (c) Using statement I :
2. (d) All others are synonyms.
3. (a) All others imply ‘UP’. Day Subject
4. (d) In the expression A > B ³ C D £ E = F to make Monday Botany
A > D true and F ³ C true. T uesday Mathematics
5. (a) Trying option (a) Wednesday Physics
P>Q=R£T<S Thursday Chemistry
Friday Zoology
P>R
R < S As Q = R so Q < S Chemistry is not taught on Wednesday.
Both the expressions are true in option (a) Using statement II :
6. (e) P × Q means P is a father of Q.
Day Subject Subjec t
Q + P means Q is daughter of R
Monday Botany/Zoology Botany/Zoology
R – T means R is sister of T.
It is clear that P is husband of R. If he establish that T Tuesday Mathematics Chemistry
is either son or daughter of S, then P would be son-in- Wednesday Physics Mathematics
law of S. Thursday Chemistry Physics
T + S means T is daughter of S. Friday Zoology/Botany Zoology/Botany
T ÷ S means T is son of S. 16. (c) From statement I :
(7-12) : At 9 : 30 the minute and the hour hands will make an
Row 1. ¯ P V S T R Q angle alightly more than 90°. So now, the time is not
Row 2. ­ C F A E B D 9 O’ clock.
7. (d) 8. (a) 9. (b) From Statement II : At 8 : 45, the hour and the minute
hands would coincide with each other. So, time now is
10. (b) 11. (c) not 9 O’ clock.
12. (e) There is a difference of one in all the rest. 17. (d) From statement I :
13. (c) Using statement I : B is the father of M and T.
Q > R, S > T > P B is the grandfather of F.
Q is the heaviest. The gender of F is not known.
Using statement II : From statement II :
There is no mention of B.
Q > R > S, T, P From both the statements.
Q is the heaviest. The gender of F is not known.
14. (b) Using statement I : (18-20) : The word and number arrangement machine rearranges
A A the input with the logic that in step I, it shifts the largest
C E C number to the left-most place and the last word coming
D
in English alphabetical series to the rightmost place.
E B D B In step II, it shifts the smallest number to the leftmost
We cannot determine if all the friends are facing the place and the next word (in reverse alphabetical order)
centre. to the rightmost. In step III 2nd largest number is shifted
Using statement II. to the leftmost place and so on.
Input : fun 89 at the 28 16 base camp 35 53 here 68
A Step I : 89 fun at 28 16 base camp 35 53 here 68 the
D C Step II : 16 89 fun at 28 base camp 35 53 68 the here
Step III : 68 16 89 at 28 base camp 35 53 the here fun
E B Step IV : 28 68 16 89 at base 35 53 the here fun camp
A is facing outside and C is facing the centre of circle. Step V : 53 28 68 16 89 at 35 the here fun camp base
So, all friends are not facing the centre. Step VI : 35 53 28 68 16 89 the here fun camp base at
18. (e) 19. (c) 20. (d)

www.newspaperkorner.wordpress.com
SBI (Associates) PO EXAM 2011 SOLVED PAPER P1-19
www.newspaperkorner.wordpress.com
(21-27) :
(Chairman) Appar- Bun-
(Managing E 41. (d) House
tment glows
Director) C F (Group
Leader)
OR
(Company
Secretary) B H (Manager)
Bun- Appar-
House glows tment
(Vice
President) G D (Financial
A Conclusion I : False
Advisor)
(President) Conclusion II : False

21. (d) 22. (e) 23. (a)


24. (d) 25. (c) 26. (e) 42. (a) Gases Liquids
27. (b)
Water
28. (c) Even (C) can be inferred only probably.
29. (e) If the infrastructure does not exist, the deal will be a OR
wastage of money.
30. (b) Getting connected by air is bound to have an impact
on road traffic.
31. (d) Takes care of the compatibility factor and (A) states
the advantage over other competing planes. Gases Liquids
(32-37) : ‘always create new ideas’ ® ‘ba ri sha gi’ ... (1)
‘ideas and new thoughts’ ® ‘fa gi ma ri’ ...(2) Water
‘create thoughts and insights’ ® ‘ma jo ba fa’ ...(3)
‘new and better solutions’ ® ‘ki ri to fa’ ...(4) Conclusion I : True
Using (1) and (4), Conclusion II : False
new ® ri
Using (1), (2) and (4), Hours
ideas ® gi Seconds
and ® fa 43. (b) Day
thoughts ® ma
Using (1) and (3),
create ® ba
always ® sha
insights ® jo OR
better solutions ® ki to
32. (c) 33. (d) 34. (b) Hours
35. (b) 36. (d) 37. (a) Seconds
38. (a) This seems to be the only logic behind the imposition Day
of tax in such specific areas.
39. (e) This is the only choice which goes in favour of tenants.

40. (b)
Circles
Rings ellipse Conclusion I : False
Conclusion II : True
Squares (44 -45)

s
or
ss
Teachers

e
of
Pr
rs
ture
Conclusion I : False Lec
Conclusion II : True
www.newspaperkorner.wordpress.com
P1-20 SBI (Associates) PO EXAM 2011 SOLVED PAPER
www.newspaperkorner.wordpress.com
OR 58. (a)
10
Professors 2 8 12 20 30 42 56
Lecturers
Teachers +4 +6 +8 + 10 + 12 + 14
8 is written in place of 6.
OR 59. (e)
s 60
or
ess Teachers, 32 16 24 65 210 945 5197.5
of
Pr
Lecturers × 0.5 × 1.5 × 2.5 × 3.5 × 4.5 × 5.5
65 is written in place of 60.
44. (a) Conclusion I : True 60. (d)
Conclusion II : False 193
45. (b) Conclusion I : False
7 13 25 49 97 194 385
Conclusion II : True.
46. (d) The shifting of the elements takes place in such a way +6 + 12 + 24 + 48 + 96 + 192
that the change is completed in four steps. So the
194 is written in place of 193
change from figure five to six will be similar to the
change from figure one to figure two. 61. (a) Amount reinvested in Equity Funds = 94500
47. (a) In each step, two pairs of elements get changed, Amount reinvested in debt + Equity Funds
beginning from the upper left and lower right. The
13
change takes place in three successive steps. = 94500 ´ = 175500
48. (c) The main design rotates respectively through 45° 7
clockwise, 180° and 90° anticlockwise after every two Amount invested earlier in Debt + Equity Funds
figures. The shaded leaflet rotates through 45° 175500
clockwise, 90° clockwise, 135° clockwise, 45° = = 135000
1.3
anticlockwise and 90° anticlockwise.
49. (d) Watch the rotation of each element separetely. The Original amount invested in equity funds
triangel rotates by 90° ACW and 180°. The circle rotates 5
45°, 135°, 90° CW 45° CW ... ‘C’ rotates 45°, 90°, 135°, = ´135000 = 75000
9
180° CW.
62. (c) Let the son’s present age be x years. Then the father’s
50. (b) In each step elements interchange in pairs while one
element beginning from one end is replaced by a new present age is (x + 30) years.
one. The line of orientation rotates by 45° ACW. Father’s age after 10 years = (x + 40) years
32 Son’s age after 10 years = (x + 10) years
51. (e) ´ 260 = 83.2 » 83
100 (x + 40) = 3(x + 10)
1 2 3 x + 40 = 3x + 30
52. (b) ´ ´ ´1715 = 85.75 » 85 2x = 10
8 3 5
53. (c) 25 × 124 + 389 × 15 = 3100 + 5835 = 8935 \ x=5
561 63. (e) Let the length and breadth of the original rectangle be
54. (a) ´ 20 = 320.5 » 320 ‘L’ m and ‘B’ m respectively.
35
After increasing the length by 20% and decreasing
55. (d) (15)2 ´ 730 = 225 ´ 27 = 6075
the breadth by 20% area is 192.
56. (d) (1.2 L) × (0.8 B) = 192
155 or 0.96 LB = 192
3601 3602 1803 604 154 36 12
LB = 200
÷1+1 ÷2+2 ÷3+3 ÷4+4 ÷5+5 ÷6+6 64. (b) Let the original numbers be x and y and their product
154 is written in place of 155.
be xy.
57. (a)
1 x 3 xy
45 Product of rd of x and 150% of y = ´ y =
4 12 42 196 1005 6066 42511 3 3 2 2
xy
Required answer = ´ 100 = 50%
2 ´ xy
42 is written in place of 45.
www.newspaperkorner.wordpress.com
SBI (Associates) PO EXAM 2011 SOLVED PAPER P1-21
www.newspaperkorner.wordpress.com From Statement III,
65. (d)
PR ´ 2
= 3000 ...(iii)
28 100
Dividing equation (i) by (iii),

We have to calculate the area of the shaded region PR 2 1000000 20


= =
which is equal to area of square – Area of the circle PR 150000 3
22 20
Required answer = (28)2 - ´14 ´ 14 ÞR= p.c.p.a
7 3
= 784 – 616 = 168 m2 72. (e) From Statement I and II
66. (c) Ratio of the work done by Sujit and Amit = 4 : 5
x+y æ x + y 18 ö
Total key depressions done by Amit Speed of train = m/s = ç ´ ÷ km/h
n è n 5ø
5 From statement II and III
= ´ 576000 = 3, 20,000
9
y æ y 18 ö
Amit’s speed in key depressions per hour Speed of train = m/s = ç ´ ÷ km/h
m èm 5ø
320000
= = 8000 73. (d) Using statement II and III, we can find the number of
8´ 5
students in second class and pass class only. As there
67. (a) Total number of ways of selecting 4 children out of 8
is no link given between the first class and the other
8´7´ 6´5
= C4 = = 70
8 classes, we cannot find the number of students in first
1´ 2 ´ 3 ´ 4 class.
Number of ways of selecting 4 girls out of 5 = 5C4 = 5
74. (e) Using Statement I :
5 1
Required probability = = A 2
70 14 =
B 3
68. (d) Amount received by all the officers Using Statement II :
= 45 × 25000 = 11,25,000 A is 40% of total. So B is 60% of total amount invested.
3
Amount received by each clerk = ´ 25000 = 15000 A 40 2
= =
5 B 60 3
Amount received by all the clerks Using statement III:
= 80 × 15000 = 12,00,000 A = 45000
Total amount of profit earned = 11,25,000 + 12,00,000 Putting the value of statement III in any of the
= `23.25 lakh. statements I or II, we can find the amount invested in
69. (e) Let the cost price of the articles be `100 scheme B.
Marked Price = `130 75. (c) Using statement I and II we can find the area of the
After giving a discount of 10% the selling price of the rectangle and using statement III we can find the cost.
articles = 0.9 × 130 = 117 76. (b) Profit earned by Company B in 2006 is 65% of
investment or 812500.
(117 - 100)
So, actual profit per cent = ´100 = 17% 812500
100 \ Income = ´ 165 = 2062500
70. (d) Salary in June 2011 = 22385 65
77. (c) Let the amount invested by Company A and B in the
22385
Salary in June 2009 = = 18500 year 2005 be `x each.
1.1 ´1.1 Income of A in 2005 = 1.70x
71. (b) From statement I, Income of B in 2005 = 1.55 x
If the investment amount be ` P and rate of interest be
A 1.70x 34
R p.c.p.a. then Ratio = = =
B 1.55x 31
PR 2 78. (b) Amount invested by Company B in 2009
Differenece = = 100 ...(i)
10000 1
From statement II, = ´ 27 ´105 = 9 lakh
3
PR1 ´ 3 Amount invested by Company A in 2009
= 19500 - P ...(ii)
100 2
= ´ 27 ´105 = 18 lakh
www.newspaperkorner.wordpress.com
3
P1-22 SBI (Associates) PO EXAM 2011 SOLVED PAPER
www.newspaperkorner.wordpress.com
Profit earned by Company B 87. (a) Number of candidates shortlisted from state E for all
80 the posts = 120 + 280 + 75 + 280 + 260 + 520 = 1535
´ 9 ´ 105 = 72 ´104
100 Number of candidates shortlisted from state G for all
75 posts = 550 + 140 + 325 + 220 + 410 + 200 = 1845
Profit earned by company A = ´18 = 13.5 lakh
100 1535 307
Required answer = =
Total profit = 13.5 + 7.2 = 20.7 lakh 1845 369
145 88. (e) Total number of candidates eligible form all states for
79. (a) Income of A in 2007 = ´12 ´105 = 174 ´104 post I = 25200
100
Total number of candidates eligible form all states for
174 ´104
Amount invested in 2008 = ´100 = 1087500 post VI = 39400
160
25200
80. (e) Let total investment be `x. Required answer = ´ 100 = 63.9%
39400
55% of x = 10.15 × 105
89. (c)
10.15 ´105 90. (c) Total candidates shortlisted for post V = 1650
x= ´100 = 1845454 » 18.45 lakh
55 Total candidates shortlisted for post VI = 2780
81. (c) Income of Company B in 2004 1650 165
Required ratio = =
= 1.55 × 12 × 105 = 18.6 lakh 2780 278
Investment in 2005 = 18.6 lakh. (91-95) : Eng Hindi Both Total
55 Boys 24 18 108 150
Profit earned in 2005 = ´ 18.6 ´ 105 = 10.23 lakh
100 Girls 55 78 117 250
Total 79 96 225 400
24 ´105
82. (a) Investment of Company A in 2008 = = 15 lakh 91. (b) 18 + 108 = 126
1.60
92. (d)
Profit in 2008 = 24 – 15 = 9 lakh.
93. (a) 79 + 225 = 304
45
Profit in 2007 = ´15 ´ 105 = 6.75 lakh 78
100 94. (e) ´100 = 81.25%
96
Required answer = 9 – 6.75 = 2.25 lakh
108 12
90 70 95. (c) Ratio = =
´ 25 ´ 105 + ´ 25 ´105 117 13
83. (d) Required answer = 100 100 96. (c) Area of the circle = pr2 = 616
2 Þ r2 = 196
Þ r = 14 cm
25 ´ 105 é 90 + 70 ù
= Length of the rectangle = Diameter of the circle.
100 êë 2 úû Breadth of the rectangle = Radius of the circle
= 25 × 103 × 80 = 20 lakh Area of rectangle = 28 × 14 = 392 cm2.
97. (a) The total population of all cities = 85 million
430 Total males in all cities = 43.4 million
84. (c) ´100 = 6.9%
6200 Total females = 85 – 43.4 = 41.6 million
85. (e) Required average 41.6
Average female population = = 8.32 million
5200 + 8400 + 7600 + 2600 + 3800 + 4400 + 6000 5
=
7 (1300 - 1100)
98. (b) ´100 = 18.18%
38000 1100
= = 5428.5 » 5500
7 72
99. (d) ´ 550 = 396
86. (d) Number of candidates eligible for post I 100
= 100 (25 + 32 + 28 + 24 + 30 + 48 + 65) 8.5 + 7.6 + 12.8 + 5.4 + 10.5
100. (e) Average salary =
= 25200 5
Number of candidates shortlisted for post I 44.8
= 65 + 220 + 280 + 85 + 120 + 325 + 550 = = 8.96
5
= 1645 Required answers = 8.96 –5.4 = 3.56 lakh
1645 101. (b) 102. (b) 103. (a) 104. (e) 105. (e)
Required answer = ´ 100 = 6.52% 106. (a) 107. (a) 108. (c) 109. (d) 110. (e)
25200
www.newspaperkorner.wordpress.com
SBI (Associates) PO EXAM 2011 SOLVED PAPER P1-23
www.newspaperkorner.wordpress.com
111. (e) 112. (e) 113. (c) 114. (a) 115. (b) 174. (b) Only (1) and (2)
116. (d) 117. (a) 118. (b) 119. (c) 120. (d) 175. (c) India And The Hunt for A Knowledge Based Economy
121. (d) 122. (c) 123. (c) 124. (b) 125. (d)
176. (d) It would not manufacture cheap common medicines
126. (d) 127. (b) 128. (b) 129. (c) 130. (a)
131. (c) 132. (e) 133. (e) 134. (d) 135. (c) without a license.
136. (e) 137. (c) 138. (c) 139. (c) 140. (a) 177. (c) Only (3)
141. (c) 142. (d) 143. (b) 144. (c) 145. (c) 178. (d) Only (2) and (3)
146. (d) 147. (d) 148. (c) 149. (d) 150. (a) 179. (d) The meaning of the word Foster (Verb) as used in the
151. (c) There has been an increase in the incomes of people. passage is : to encourage something to develop;
152. (b) Help from other countries to counter food inflation
promote; nurture.
has proved to be quite encouraging.
153. (e) All (1), (2) and (3) 180. (a) The meaning of the word Proper (Verb) as used in the
154. (b) Only (1) and (2) passage is : to move, drive or push something forward
155. (a) Food inflation in India or in a particular direction.
156. (b) People are not buying as much as they used to. Look at the sentence :
157. (a) The meaning of the phrase Cash in (on something) as He succeeded in propelling the ball across the line.
used in the passage is : to gain an advantage for
181. (e) The meaning of the word Massive (Adjective) as used
yourself from a situation that is thought to be immoral;
profiting. in the passage is : extremely large or serious; huge.
Look at the sentence : 182. (d) The meaning of the word Stumbling block (Noun) is :
The film studio is being accused of cashing in on the something that causes problems and prevents you
singer’s death. from achieving your aim; hurdle; obstacle.
158. (c) The meaning of the word Tight (Adjective) as used in 183. (b) The meaning of the word Cumbersome (Adjective) is:
the passage is : difficult to manage with because there
large and heavy; bulky; slow and complicated.
is not enough; limited.
159. (e) The meaning of hte word Temper (Verb) as used in the The word manageable means : possible to deal with or
passage is : mitigate; control; to make something less control.
severe by adding something that has the opposite Look at the sentence :
effect. The debt has been reduced to a more manageable level.
Look at the sentence : 184. (d) The meaning of the word Resemble (Verb) is : to look
Justice must be tempered with mercy.
like or be similar to another perosn or thing.
160. (c) The meaning of the word Archaic (Adjective) as used
in the passage is : old and no longer used; outdated. Its antonym should be differ from.
Look at the sentence : 185. (c) The meaning of the word Differentiate (Verb) is : to
The system is archaic and unfair and needs changing. show or recognise that two things are not the same;
161. (e) Idiom make a beeline for something means : to go distinguish.
straight towards something / somebody as quickly as Hence, the antonym of differentiated should be
you can.
harmonized which means : made similar.
162. (a) Idiom in the nick of time means : at the very last
moment; just in time before something bad happens. 186. (b) 5
163. (e) Idiom on cloud nine means : extremely happy. 187. (d) 6
164. (c) passing the bucks 188. (a) 1
165. (b) Idiom rack your brain (s) means : to think very hard or 189. (d) 3
for a long time about something. 190. (e) 2
166. (e) bound - task
191. (b) destroyed
167. (a) just - afar
168. (a) overwhelmed - extend 192. (c) vital
169. (d) Concerned - switch 193. (e) impairs
170. (c) forced - settle 194. (a) plagues
171. (c) In the past, countries could progress economically 195. (d) provide
through low manufacturing cost as well as low wages 196. (e) eroding
of labourers.
197. (d) access
172. (a) The word Achilles heel means : a weak point or fault in
somebody’s character which can be attacked by other 198. (b) source
people. 199. (a) Despite
173. (d) Only (2) and (3) 200. (a) exploit

www.newspaperkorner.wordpress.com
P1-24 SBI (Associates) PO EXAM 2011 SOLVED PAPER
www.newspaperkorner.wordpress.com
Solution of Descriptive Test
LETTERWRITING As you are aware there is a lot of competition faced by us
from other leading groups and we are employing MBA graduates
1. (a) Letter to increase the credit amount of loan
as Sales Marketing personnel to push up our sales. We are hopeful
Mr. Abhishek Sharma
of improved sales in the next few months. We were given a break
20, Fortview Apt.
of 1 months for payment of EMI of `2 lakhs. Since the complete
Kalkaji, New Delhi - 19
decoration work could not be completed on time and sales having
20 Jan. 2013
not picked up well as per our estimates, we are unable to pay our
EMI for October 2012. Our first EMI will have to be paid by 16th
Mr. Kaushal Singh
December 2012. We have been paying the interest on loan debited
Head, Loan and Credit Dept.
in our account regularly by debit to our cash credit account.
The Urban Cooperative Society,
In view of the reasons explained above, we are sure you will
211, Lajpat Nagar -I
consider our request for postponement for payment of EMI.
New Delhi -24
Thanking you
Sub.: To increase the credit amount of loan
Yours faithfully
Dear Sir,
General Manager (Finance)
I have been a member for your society 2 years now, and I
M. S. Enterprises
believe my credit standing with you is in fine order. My overall
1. (c) Letter to brother to choose banking as carrer.
credit rating is also excellent.
5/11, Janakpuri
I had taken a credit from your society nearly three months
New Delhi -110058
back. I had taken Home improvement loan, payable by me in the
18, Jan., 2013
time period of 1 year. I have been regularly paying the interest on
the amount taken. But now due to some miscalculations on part
Mr. Ramesh Kumar
of the renovators the amount of the work undertaken has been
14, Ashirwad Apt.
increased. I thus, want to increase the credit amount from ` 30000
Kolkata
to `50000.
Sub.: Choosing banking as a career
I have been in touch with the society and they have asked
Dear brother
me to give a written application, along with all the related
I received your letter yesterday and I am glad to know that
documents. I am hereby, enclosing the required papers of my
mother is now perfectly well. I have taken banking and
loan.
accountancy as my special subjects. Although the subjects are
I hope that my application for increasing the credit amount
quite new. I find them immensely interesting. From the very
will be entertained by your society.
beginning of my student career I had been thinking of adopting
I hope you will consider my request, looking forward to a
Banking as a career. There are several reasons which induced me
positive response.
to advice you to take Banking as a career. In the first place Bank
Thanking You
job occupies a middle position between government and private
Yours sincerely
service. Being an autonomous public company it provides the
Abhishek Sharma
advantages of both the government and the private services.
1. (b) Letter to the manager of the bank to apologize for the
There is security of government job independence of private
delayed payment of EMI
service.
M.S. Enterprises
Secondly a bank job carries handsome salary. A person
(Dealers in Electronic Gadgets, Mobiles….)
with certain qualifications is paid more by banks than by other
15, North Mukherjee Nagar, Delhi -110037
departments. Banks provide sufficiently wide opportunity for
8 Nov. 2012
further rise in life. Thirdly banks advance loans to their personnel
on very easy terms. A person serving in a Bank can very easily
The Manager
get his own house constructed with the help of loans advanced
Bank of India
by the bank. Besides if one wants to have a scooter or motor
North Mukherjee Nagar branch
cycle one can easily avail oneself of the facilities provided by the
Delhi -110037
bank and can have one's desire satisfied thereby. Fourthly banks
Sub: Request for EMI postponement (Ref. IB/Term loan/1/2012)
play a crucial role in the development of the country. By serving
Dear Sir,
in a bank I can feel a sense of participation in the development of
We would like to thank your Bank management for having
the country.
sanctioned us term loan of `20 lakhs, using which we were able
As a bank employee, I shall help many people to overhaul
to decorate our show room and provide AC and other facilities.
the deposit and withdrawal system so as to ensure greater
We availed the term loan on 16th August 2012 and started the
convenience to the customers. I shall try to train the new recruits
work in right earnest. However on account of delay in execution
in courtesy and civility. I shall help my officers to thoroughly
of work at different levels the full decoration of the shop could
examine the applications for advances and shall endeavour to
not be completed. In spite of that we commenced our sales show
reorient the bank policy in the direction in which it can render
room on 15th October 2012 and are receiving good number of
maximum of service to society.
enquiries. www.newspaperkorner.wordpress.com
SBI (Associates) PO EXAM 2011 SOLVED PAPER P1-25
www.newspaperkorner.wordpress.com
I need not say that you also approve of my idea of choosing than ever before! You can pay your credit card, electric, water,
banking as a career. I seek your blessings for my success in the phone and other bills by just logging into the bank's web portal.
competition. 2 (c)
Best regards Can India achieve the status of
Reena most developed economy?
2 (a) India is already on its way to becoming a developed country.
Cashless cards for Students However, certain things are still lacking. Although, today the
The ability to store money in an electronic purse on a card government of India may claim to be an emerging super power
is fast becoming standard practice throughout the workplace. but the reality is quite different from what the government of
The electronic purse is then used to purchase products at a India or the media of India portray to the world.
vending machine. By cashless card, one can use a tag on any Lately the government of India has come up with several
personnel belonging like a mobile phone or ID card and make developmental plans and no doubt it has helped boost the
payments at shops through it. Cashless cards for students settle economy of the country in some ways. But the long term impact
their bills, purchase books and eat in canteens without shelling of these plans do not seem to serve the purpose and prosperity
out a real note. of the common man.
This tag concept is a part pf the 'virtual wallet' project In India most of the people are living in villages. In those
launched by Itz cash world, Canara Bank and Rupay. With this villages people are unable to fulfill their basic needs of life. In
concept, a cashless world no longer seems like a far-fetched dream. those villages youth is unemployed, they follow unhygienic
The technology used for this is NFC i.e. Near Field Communication activities, which leads to serious health hazards, youth is turning
where in devices establishes radio communication by touch or towards crime because of unemployment, literacy is less, and
bringing them in close contact. This new system hopes to most of the families are below poverty line. In such circumstances
decreases cases of theft among students. Since students won't it is very difficult for India to become a developed nation.
have to bring cash with them, lost-money incidents can be But, India also has some strength to its name.
prevented. The most dominating technology in today's scenario is
Students can also enjoy eating their lunch or relaxing during “Information Technology” and because of its highly intellectual
their lunch break instead of spending precious minutes on the minds, India is dominating throughout the world in this particular
lunch line. The new cashless payment system also encourages technology. The revenue/profits of the India-based software
parents to go online and check out what their kids are eating for companies are in thousands of corers. So they are contributing
lunch. By logging in into the student's account, a parent can greatly in the development of nation.
track what his kid bought for lunch, what time he had lunch, and So to conclude, we can say that there are many hurdles for
how much the meal cost them. Parents also have the option to India to become a developed nation, but India also has its own
transfer funds to their children's account using through the strengths. India is growing all the time and it is only a matter of
internet. time before it is a “developed nation”. Its economy is one of the
2 (b) fastest growing in the world.
Advantages of Internet Banking 3. (a)
Internet banking also referred to as online banking, uses Does providing education loan helps in
the internet as the delivery channel by which one uses to conduct empowering employment to the youth
banking activities. Internet banking will allow an individual to Yes, providing education loan for education can empower
manage his accounts by doing the following activities; make employment among the youth. The greatest wealth and strength
payments, view accounts and inquire about the balances, apply of any nation is its youth. The future of a nation lies in the hands
for credit cards and loans electronically. There are many reasons of its posterity. The quality of its youth determines the kind of
why online banking has emerged as a staple of daily life. It's fast, future, the nation will have. Therefore, if we want to ensure a
safe and convenient. It also means that there is no more need to bright future for our country, we first need to strengthen and
travel to your nearest bank for basic services any longer. empower our youth.
Banking online means that the banks no longer need to The best and the first and foremost way to strengthen our youth
keep their branches open around the clock. That means one can are to provide them education. Not just any kind of education,
simply login to the website of the bank and get what there are but the right kind of education which makes them scientific,
looking for. logical, open-minded, self respecting, responsible, honest and
With online banking there is no need to visit the bank, call patriotic. Without these virtues being developed, our youth
up the branch, or wait for a financial statement to come in the cannot walk in the desired way and they will remain in a deep
mail. All you have to do is login online and download a copy of slumber of complacency.
your statement. This adds a great deal of convenience for those Education is central to the Human Resources Development and
who are hard pressed for time. empowerment in any country. National and State level policies
Through banking on the internet any customer can apply are framed to ensure that this basic need of the population is met
for a loan without even having to visit their local branch. One can through appropriate public and private sector initiatives. While
sell and buy stocks and shares. government endeavour to provide primary education to all on a
With internet banking, it doesn't matter where you're located. universal basis, higher education is progressively moving into
You could be in any part of the globe and still be able to check on the domain of private sector. With a gradual reduction in
any financial transaction online. government subsidies higher education is getting more and more
Banking through the internet has made bill payment easier costly and hence the need for institutional funding in this area.
www.newspaperkorner.wordpress.com
P1-26 SBI (Associates) PO EXAM 2011 SOLVED PAPER
www.newspaperkorner.wordpress.com
The scope of education has widened both in India and abroad Favoritism and patronage by those in authority also lead to
covering new courses in diversified areas. Development of human generation of black money. Other factors for generation of black
capital is a national priority and it should be the endeavour of all money include (i) high rates of taxation under taxation system
that no deserving student is denied opportunity to pursue higher which encourages tax evasion, (ii) controls, permits and licenses
education for want of financial support. Loans for education (iii) price control without adequate machinery for effective
should be seen as an investment for economic development and distribution (iv) donations to political parties for elections (v)
prosperity. corrupt business practices, (vi) smuggling and other illegal anti-
3. (b) social activities.
The AADHAR (UID) card, how it is useful In recent years black money generation has been growing at a
The Unique Identification Authority of India (UIDAI) is an agency very fast rate in our country. So that it is difficult to isolate a
of the Government of India responsible for implementing the single sector of economy or administration that moves without
AADHAAR scheme, a unique identification project. Aadhar is a the use of black money. Most economist are unanimous in their
12-digit unique number which the Unique Identification Authority view that black money generation has become a major factor for
of India (UIDAI) will issue for all residents in India. UID scheme galloping inflation because it not only misallocates resources,
will enable inclusive growth by providing each citizen with a but also shifts them from investment to consumption. The parallel
verifiable identity; that it will facilitate delivery of basic services; economy of black money has made nonsense of all planning and
that it will plug leakages in public expenditure and that it will anti-poverty programmers. Beside, tax evasion creates great
speed up achievement of targets in social sector schemes The inequities between the tax payers, who are honest or who can't
UID scheme could have many benefits for the privileged of the evade taxes and those who are dishonest and are in the position
population who have bank accounts, driving licenses, passports to evade taxes.
and gas cylinders by providing a clear proof of identity. Aadhar
4. Précis Writing
will also facilitate entry for poor and underprivileged residents
into the formal banking system and the opportunity to avail RBI asks private, foreign banks to set up
services provided by the government and the private sector. vigilance systems
Aadhar can be used in the delivery of the following programs: The RBI has issued guidelines to both private and foreign lenders
Food & Nutrition - Public Distribution System, Food Security, to have some action against money fraud & black money. A central
Mid Day Meals, Integrated Child Development Scheme. vigilance commission need to be formed for that and a central
Employment - Mahatma Gandhi National Rural Employment vigilance officer (CVO) should be appointed by the banks. Both
Guarantee Scheme, Swarnajayanti Gram Swarozgar Yojana, Indira the private and foreign banks have to follow these guidelines.
Awaas Yojana, Prime Minister's Employment Guarantee Program The same practice has been adopted by the public sector banks
Education - Sarva Shiksha Abhiyan, Right to Education (RTE) for a long time. This was done by RBI in lieu of the 400 crores of
Inclusion & Social Security - Janani Suraksha Yojana, scam happened in one of the nationalized banks last year. The
Development of Primitive Tribe Groups, Indira Gandhi National Banks need to be having the central vigilance commission as a
Old Age Pension Scheme integral part of the working of the banks. And the central vigilance
Healthcare - Rashtriya Swasthya Bima Yojana, Janashri Bima officer appointed is acting as an assistant and consultant to the
Yojana, Aam Aadmi Bima Yojana CEO of the bank. The officer appointed can be one of the senior
Other miscellaneous purposes including Property Transactions, officials of the bank. The normal tenure of the CVO thus appointed
Voter ID, PAN Card etc. should be of 3 years can be increased to maximum 5 years but in
3. (c) case of transfer to another bank it should be of 6 years.
Black money and its impact on Indian Economy 5. Passage
The money that is earned and hoarded by dishonest and unfair
(i) ATMs (Automated Teller Machines) are an integral part of
means and kept in secret to evade taxes is called black-money. It
the banking system. They are an important aspect of the
becomes black because no taxes are paid on it. The massive amount
banking delivery channel. Therefore banks are very
of black money in our country has created a parallel economy.
concerned about this in the proper organizing of ATMs.
Continuing generation and accumulation of black money poses a
(ii) The Breston boston marketing survey stated that about
threat to our economy and is likely to threaten the very security
and survival of the country. It is the source and at the same times 49% Indians will switch to another competitive bank if they
cause of graft, corruption and the debasing of social and moral were provide with the facilities of personalizing accounts
values. There are various factors responsible for the origin, more effectively and some other facilities.
existence and flourishing state of the parallel economy of black (iii) ATM provides usual paying facilities, tax payments, mobile
money. It originates with the combination of money and power. recharge, and credit card bill payment.
With black money any one can be bought or bribed, essential (iv) Visit near ATMs and proceed with payment and after
commodities can be corned and hoarded, healthy competition checking the fund payment is done and one can receive the
can be driven out and honest men and public servants can be confirmation receipt after that.
blackmailed. (v) Visit bank website and register yourself for the online tax
One of the sources of black money generation in developing payment and go to the nearest ATM and select under the
countries like India is through commission or kick backs obtained income tax tab and then the payment options and after the
for arranging government deals and purchase. Agents and payment go to the bank and take print out of the challan
middlemen pocket considerable sums as their share of with a SIN which was generated earlier and can be shown
commission in the payoff to influence political decisions. while filing the return later on.
www.newspaperkorner.wordpress.com
www.newspaperkorner.wordpress.com
SBI PO Exam Solved Paper
Held On : 24-07-2011

Time : 2 hrs. Based on Memory Max. Marks : 200

REASONING (High Level) 5. If ‘OD’ is related to 'GF' and 'EB is related to 'NP' in a certain
way, to which of the following is 'AL' related to, following
1. How many meaningful English words can be formed with the the same pattern ?
letters NCEO using each letter • only once in each word ? (a) KD (b) QV
(a) None (b) One (c) KL (d) KV
(c) Two (d) Three
(e) DQ
(e) More than three
6. What will come in place of question mark (?) in the following
2. The positions of first and the fourth letters of the word
series based on the above alphabetical series?
LIQUID are interchanged, similarly, the positions of second
and fifth letters and third and Sixth letters are interchanged. TE JU NK ? GH
In the new arrangement thus formed, how many letters are (a) IS (b) IR
there in the English alphabetical series between the (c) AG (d) AR
alphabets which are at the extreme ends? (e) AM
(a) None (b) Two 7. If in a certain code 'GRIM' is coded as 'RMOS' and 'DUSK' is
(c) Three (d) Four coded as 'LQPI' how will 'STOP' be coded in the same code
(e) More than four language?
Directions (Qs. 3-4): Study the following information to answer (a) MJIS (b) PJGB
the given questions : (c) JPJIS (d) MJGB
Point P is 5 m towards the South of Point M. Point Q is 3 m (e) PGJB
towards the East of Point P. Point O is 3 m towards the East of 8. Among five friends, P, Q, R, S and T, each scored different
Point M. Point N is 2 m towards the South of Point Q.
marks in the examination. P scored more than Q but less
3. A person, facing North, takes a left turn from point M, walks
than R. S scored more than' only T. Who amongst the
4m and stops. He then takes another left turn, walks 5 m and
following scored the second highest marks?
stops at point R. Which of the following points, including
R, fall in a straight line ? (a) P (b) Q
(a) M, O, R (b) N, R, P (c) R (d) S
(c) R, O, Q (d) R, Q, N (e) T
(e) Q, P, R 9. If each of the alphabets of the word MIRACLE is arranged
4. How far and towards which direction is Point O from Point in alphabetical order from left to right and then each vowel
N? in the new word thus formed is changed to the next letter in
(a) 5 m towards South (b) 7 m towards North the. English alphabetical series and each consonant is
(c) 8 m towards West (d) 7 m towards West changed to the previous letter in the English alphabetical
(e) 5 m towards North series which of the followingwill be fifth from the right?
Directions (Qs.5-7): The following questions are based upon (a) K (b) S
the alphabetical series given below: (c) F (d) E
(e) J
T J E N UQ AK I O G R M S PB HF D LVC
www.newspaperkorner.wordpress.com
P2-2 SBI PO EXAM 2011 SOLVED PAPER
www.newspaperkorner.wordpress.com
10. How many such, pairs of letters are there in the word 18. Starting from A, if all the friends are made to sit in the
INCLUDE, each of which has as many letters between them alphabetical order in clockwise direction, the positons of
in the word (in both forward and backward directions) as how many (except A) will remain unchanged ?
they have between them in the English alphabetical order?
(a) None (b) One
(a) None (b) One
(c) Two (d) Three
(c) Two (d) Three
(e) Four
(e) More than three
19. H is related to C and B is related to E in a certain way. To
Directions ( Qs. 11-15) : Study the following information to whom amongst the following is G related following the same
answer the given questions: pattern ?
In a certain code 'support the other group' is written as 'ja pe la no' (a) F (b) H
'the mission gains support' is written as' ke ja zi la', 'gains other
(c) C (d) A
than money' is written as 'fu no ho zi' and 'more support and
(e) D
money' is written as ' re qi fuja' .
11. What is the code for 'group' ? 20. What will come in place of the question mark ?
( a ) ja (b) pe DC DB DF DA
(c) la (d) no (a) DG (b) DE
(e) Cannot be determined (c) DH (d) DK
12. What does 'zi' stand for ? (e) Either DK or DE
(a) mission (b) than 21. Who sits third to the left of A?
(c) other (d) the (a) H (b) G
(e) gains (c) C (d) F
13. Which of the following may represent 'more than the group'? (e) B
(a) la qi ho pe (b) re la qi ho 22. Which of the following pairs represents the immediate
(c) re no la pe (d) pe ke qi la neighbours of C?
(e) qi ho la fu (a) FH (b) AB
14. What is the code for 'mission'? (c) BD (d) EK
(a) la (b) zi (e) DF
(c) ke (d) ja Directions (Qs.23-27): In each of the question below is given a
(e) ke or la statement followed by two assumptions numbered I and II. An
15. Which of the following may represent 'money matters more'? assumption is something supposed or taken for granted. You
(a) fue re bu (b) re bu qi have to consider the statement and the following assumptions
(c) zi qi yo (d) yo fu no and decide which of the assumptions is implicit in the statement.
(e) la fu bu Give answer (a) if only Assumption I is implicit.
Directions (Qs. 16-22): Study the following information Give answer (b) if only'Assumption II is implicit.
carefully and answer the given questions. Give answer (c) if either Assumption 1 or Assumption II is implicit.
Nine friends A, B, C, D, E, F, G, H and K are sitting around a circle Give answer (d) if neither Assumption I nor Assumption II is implicit.
facing the centre. A sits second to left of D. K sits third to right of
Give answer (e) if both Assumptions I and II are implicit.
F. Neither K nor F is an immediate neighbour of A
or D. G and H are immediate neighbours of each other. E sits third 23. Statement: The biggest private airline decided to increase
to right of H. B is not an immediate neighbor of F. the number of flights between cities A and B to 10 flights
16. What is the position of F with respect to the position of B ? everyday.
(a) Second to the right (b) Third to the left Assumptions:
(c) Second to the left (d) Third to the right I. Other private airlines may also increase the frequency
(e) Sixthtotheright of daily flights between cities A and B.
17. Who amongst the following is an immediate neighbour of H ? II. There may be adequate passenger load on all the
(a) C (b) B flights of the biggest private airline even after increased
(c) K (d) F frequency.
(e) A
www.newspaperkorner.wordpress.com
SBI PO EXAM 2011 SOLVED PAPER P2-3
www.newspaperkorner.wordpress.com
24. Statement: Majority of the employees of the organisation 29. (A) Government has increased the procurement price of
decided to join with their family the overnight picnic funded kharif crops by about ten per cent for the current year.
by the organisation. (B) Inadequate monsoon rainfall has created a drought
Assumptions: like situation in many parts of the country.
I. The management of the organization may not welcome
30. (A) The Government has recently lifted ban on import of
the employees' enthusiasm.
II. The management of the organization may provide sugar for selling in the retail market.
adequate funds for the picnic. (B) The prices of sugar in the retail market have stabilized
25. Statement : The reputed management institute in the city after a considerable gap.
increased the fees by 300 percent from the next academic 31. (A) Many vehicles met with accidents during the last
year. fortnight at the major junction on the main arterial road
Assumptions : in the city.
I. The institute may still attract good number of students
(B) The automatic signalling system at the major junction
for all its courses.
of the arterial road of the city.
II. The students may now opt for other institutes in the
city which charge less fees. 32. (A) The english medium school in the locality has decided
26. Statement : Many residents of the locality decided not to to admit only those students who reside in area. where
attend the cultural function organised by the local club to the school is located from the next academic session.
protest against the club's limited invitations. (B) Many school in the city admit students residing
Assumptions: anywhere within the city limit.
I. The local club may cancel the cultural function.
Directions (Qs. 33-37): In each of the questions below are three
II. The local club may stop all its activities.
27. Statement : Government, has announced a relief package statements followed by two conclusions numbered I and II. You
for all the drought hit farmers in the country and advised have to take the three statements to be true even if they seem to
the state Governments to put in a mechanism for be at variance from commonly known facts and then decide
disbursement. which of the given conclusions logically follows from the three
Assumptions: statements disregarding commonly known facts.
I. The state Governments may be able to put in place the Give answer (a) if only conclusion I follows.
system for disbursement of Government relief to the Give answer (b) if only conclusion II follows.
affected farmers.
Give answer (c) if either conclusion I or conclusion II follows.
II. Government may be able to identify all the farmers
affected by drought in the country. Give answer (d) if neither conclusion I nor conclusion II follows.
Directions (Qs.28-32) : Below in each questions are given two Give answer (e) if both conclusion I and conclusion II follow.
statements (A) and (B). These statements may be either 33. Statements:
independent causes or may be effects of independent causes or All plants are bottles.
a common cause. One of these statements may be the effect of
All bottles are caps.
the other statement. Read both the statements and decide which
of the following answer choice correctly depicts the relationship All caps are crowns.
between these two statements. Conclusions:
Mark answer (a) if statement (A) is the cause and statement (B) I. Atleast some crowns are bottles.
is its effect. II. All plants are caps.
Mark answer (b) if statement (B) is the cause and statement (A)
is its effect. 34. Statements:
Mark answer (c) if both the statements (A) and (B) are Some shoes are handkerchiefs.
independent causes. Some handkerchiefs are calculators.
Mark answer (d) if both the statements (A) and (B) are effects of All calculators are paper.
independent causes.
Conclusions:
Mark answer (e) if both the statements (A) and (B) are effects of
some common cause. I. No calculator is a shoe.
28. (A) The local traders' association urged all its members to II. No shoe is a paper.
shut their establishments foi two days and participate 35. Statements:
in the protest march.
All zebra are cows.
(B) Government has recently cancelled licenses of many
All camels are cows.
traders in the locality as they did not pay their taxes in
time. All tigers are zebra.
www.newspaperkorner.wordpress.com
P2-4 SBI PO EXAM 2011 SOLVED PAPER
www.newspaperkorner.wordpress.com
Conclusions: Give answer (c) If the data in either Statement I alone or statement
I. All tigers are cows. II alone are sufficient to answer the question.
II. All camels being tigers is a possibility. Give answer (d) If the data in both the Statements I and II are not
36. Statements: sufficient to answer the question.
Some pencils are mobiles. Give answer (e) If the data in both the Statements I and II are
All mobiles are grass. together necessary to answer the question.
All grass is green. 38. Who is the tallest of A, B, C, D and E ?
Conclusions: I. B is shortest amongst the five people.
I. All grass being pencils is a possibility.
II. C is taller than D.
II. No Green is mobile.
39. How is ‘stand’ coded ?
37. Statements:
I. ‘cost of pen’ is coded as 'lake if.
All watches are cards.
Some cards are clips. II. ‘pen stand’ is coded as ‘kehu’.
All clips are chairs. 40. How many marks did Sudhir score in Maths ?
Conclusions: I. Sudhir has scored 6 5% marks overall.
I. All chairs being cards is a possibility. II. The difference between Sudhir’s marks in Maths and
II. All clips being watches is a possibility. English i s 12 marks.
Directions (Qs. 38 - 42) : Each of the following questions below 41. On which day of the week did Mansi take leave f r om her
consists of a question and two statements numbered I and II office ?
given below it. You have to decide whether the data provided in I. Mansi correctly remembers that she took leave before
the statements are sufficient to answer the question. Read both
Friday but after Monday.
the statements and-
II. Mansi’s friend correctly remembers that Mansi took
Give answer (a) If the data in Statement I alone is sufficient to
answer the question, while the data in statement II alone are not leave before Saturday but after Thursday.
sufficient to answer the question. 42. How many brothers Mary have (Mary is a girl) ?
Give answer (b) If the data in Statement II alone is sufficient to I. Mary has only one younger brother.
answer the question, while the data in statement I alone are not II. Mary's father has only one son.
sufficient to answer the question.

Directions (Qs. 43-47) : In each of the question given below which one of the five answer figures on the right should come after the
problem figures on the left, if the sequence were continued?
Problem Figures Answer Figures
43.

(a) (b) (c) (d) (e)


44.

(a) (b) (c) (d) (e)


45.

(a) (b) (c) (d) (e)


46. Z U Z

Z Z Z Z Z Z
U U Z U U U U U Z U U

(a) (b) (c) (d) (e)


47.

(a) (b) (c) (d) (e)


www.newspaperkorner.wordpress.com
SBI PO EXAM 2011 SOLVED PAPER P2-5
www.newspaperkorner.wordpress.com Yrs ®
Directions (Qs. 48-50) : A word arrangement machine, when 1997 1998 1999 2000 2001 2002
given a particular input, rearranges it following a particular Com. ¯
rule. The following is the illustration of the input and the steps BS 3:5 8:7 1:2 4:5 6:5 5:2
of arrangement : TIS 1 : 2.5 9 : 8 13 : 15 7:6 6:7 14 : 9
Input : and band land hand hind lack job SA I 13 : 15 7 : 9 1:3 11 : 7 10 : 7 62 : 71
Step I : hind and band lack land hand job MPI 41 : 59 17 : 8 100 : 119 53 : 42 24 : 25 7 : 9
Step II : hind band land job and lack hand ES 2 : 7 12 : 19 6:5 3 : 11 10 : 13 19 : 21
Step III : hind and lack band hand land job LTS 13 : 11 21 : 23 5:3 4 : 7 1 : 1.29 7 : 5
Step IV : land band and job hand lack hind Table-II
Step V : hand land band lack and job hind 51. The production of steel A by company MPI in 1999 is
Step VI : hand band and hind land lack job approximately what per cent of production in 2001?
and so on. (a) 51 % (b) 53%
As per the rule followed in the above steps, find out the (c) 55% (d) 60%
appropriate step for the given input or vice versa in the (e) 64%
following questions. 52. In 2001, in how many companies production of A type steel
48. Input : do we he is it at all is more than that of B type steel?
Which of the following steps would be
(a) No company (b) Two
“all we he is do at it”?
(c) Three (d) Cannot be determined
(a) It is not possible to get the above step.
(b) Step VI (c) Step IX (e) None of these
(d) Step X (e) None of these 53. For how many companies did the production of steel A
49. If Step IV of an input is “he is to do what her observe”. increase every year and the production of steel B decrease
Which of the following would definitely be the input? every year together from that of the previous year?
(a) to is he what observe her do (a) No company (b) One
(b) he is to what observe her do (c) Two (d) Three
(c) is he to what observe her do (e) None of these
(d) Can’t say 54. Production of TIS increases by 10% in 2003 and production
(e) None of these of SAI decreases by 10% in 2003 in comparison to 2002. If
50. If step III of an input is the ratio of production remains the same as in 2002, find the
“when then men can how are you”. ratio of production of A type to B type steel for both the
What would be step VII of the input? companies together.
(a) then can are when you men how (a) 68.11% (b) 87.12%
(b) how are men can you then when (c) 97.11% (d) 79.11%
(c) you then can men are when how (e) None of these
(d) how can then men are when you
55. If the profit ratio per unit tonne of steel A and B is 3 : 4 for
(e) None of these company BS, what is the actual profit ratio of the company
BS for the year 1998 for steel A and B?
DATA ANALYSIS & INTERPRETATION
(a) 6 : 7 (b) 3 : 4
Directions (Qs. 51-55) : Study the following table to answer the (c) 8 : 7 (d) Can’t be determined
given questions : (e) None of these
Each company produces two types of steel. In table I the 56. On the ground 12 stones are placed. The distance between
total production (in lakh tonnes) of both types of steel together the first and the second is 1 metre, between second and 3rd
of six companies over the years is given. In table II the ratio of 3 m, between 3rd and 4th 5 m, and so on. How far will a boy
production of two types A and B (A : B) is given over the years. have to run to touch the last stone if he starts from the first?
(a) 144 m (b) 121 m
Yrs . ® (c) 132 m (d) 110 m
1997 1998 1999 2000 2001 2002 Total
Com. ¯ (e) None of these
BS 424 390 258 756 319 427 2574 57. A dishonest dealer prefers to sell his goods at cost price
TIS 339.5 663 812 598 663 782 3857.5
SA I 532 576 364 936 595 665 3668 1
but uses less weight for a kg weight and gains 4 % .
MPI 620 850 876 1045 1274 1296 5961 6
ES 612 806 627 406 874 760 4085 What does he use for a kg weight?
LTS 840 836 776 748 384.72 816 4400.72 (a) 950 gm (b) 980 gm
Total 3367.5 4121 3713 4489 4109.72 4746 24546.22
(c) 960 gm (d) 840 gm
Table-I (e) None of these
www.newspaperkorner.wordpress.com
P2-6 SBI PO EXAM 2011 SOLVED PAPER
www.newspaperkorner.wordpress.com
58. In a town three newspapers A, B and C are published. 42% 64. The ratio of the number of boys to that of girls was 1 : 2 but
of the people in that town read A, 68% read B, 51% read C, when 2 boys and 2 girls left, the ratio became 1 : 3. How
30% read A and B, 28% read B and C, 36% A and C and 18% many people were at the party originally?
do not read any paper. Find the % of population of town (a) 12 (b) 15
that reads all the three. (c) 10 (d) 18
(a) 15% (b) 25% (e) None of these
(c) 20% (d) 35% 65. A wheel of a motorbike has radius 35 cm. How many
(e) None of these revolutions per minute must the wheel make so that the
59. The time taken by a man to walk five times around the speed of the bike is 33 km/hr?
boundary of a square field having 16 hectares as area, at (a) 300 (b) 250
the rate of 5 km per hour is (c) 200 (d) 220
(a) 16 minutes (b) 24 minutes (e) None of these
(c) 48 minutes (d) 96 minutes Directions (Qs. 66 & 67) : Contribution of different sources of
(e) None of these water to fulfil the requirement in Delhi and consumption of
60. When the numerator and the denominator of a fraction are water for different uses by two major sources.
Total consumption of water = 720 million litres
2
increased by 1 and 2 respectively, the fraction becomes ,
3 Miscellaneous
Sutlej Jhelum
and when the numerator and the denominator of the same 7% 6% Building 5%
Projects
fraction are increased by 2 and 3 respectively, the fraction Under- 15%
ground Agric-
5 12% ulture
becomes . What is the original fraction? Yamuna 30%
7 40%
Industry
10%
Ganga
5 3 35% Domestic
(a) (b) 40%
6 4
Yamuna
3 6
(c) (d)
5 7
(e) None of these Agriculture
Miscellaneous 20%
61. Find the number of numbers between 300 and 3000 that can 11%
be formed with the digits 0, 1, 2, 3, 4 and 5, no digit being Building
repeated. projects
12%
(a) 120 (b) 160
Domestic
(c) 240 (d) 60 23%
(e) None of these Industry
62. A salesgirl’s terms were changed from a flat commission of 34%

5% on all her sales to a fixed salary of ` 1000 plus 2.5%


commission on all sales exceeding `4000. If her remuneration Ganga
as per the new scheme was `600 more than that by the
previous scheme, her total sales was 66. How many litres of Yamuna water is consumed for building
projects in Delhi?
(a) `10000 (b) `5000
(a) 39.9m litre (b) 43.2 m litre
(c) `2000 (d) `12000
(e) None of these (c) 47.3 m litre (d) 51.9 m litre
63. A can do a piece of work in 10 days and B is 25% more (e) None of these
efficient than A. In what time will the work be finished if A 67. What is the ratio of supply of Jehlum water and underground
and B work together? water together to consumption of Yamuna water for domestic
purposes and Ganga water for Agriculture purposes
4 5 together?
(a) 4 days (b) 5 days
9 7 (a) 17 : 23 (b) 5 : 8
2 (c) 17 : 22 (d) 18 : 23
(c) 5 days (d) 6 days
3 (e) None of these
(e) None of these
www.newspaperkorner.wordpress.com
SBI PO EXAM 2011 SOLVED PAPER P2-7
www.newspaperkorner.wordpress.com
Directions (Qs. 68-73) : Study the following table carefully and
Mark s ®
answer the questions given below it : 0 -2 9 30 -5 9 60 -8 9 9 0-11 9 1 20 -1 50
S ub ¯
Area and Population of different states Maths 22 47 74 25 12
S tates Area (in s q k ilometres ) Population (in lak hs ) S cience 39 38 67 22 14
A 6230 1122 19 59 47 36 19
Hindi
B 2540 838
C 8135 649 Englis h 24 41 58 34 23
D 7436 572 Geography 42 32 52 41 13
E 4893 711 Average of
F 3718 286 27 45 60 31 17
five s ubjects
G 4297 860
74. If for passing, the student has to obtain minimum 60% marks
68. Among the given states, in case of how many states the in the average of five subjects, how many students will
area of that state was more than 15 per cent of the total pass?
areas taken together? (a) 108 (b) 58
(a) One (b) Three (c) 48 (d) 72
(e) None of these
(c) two (d) Can’t say
75. How many students will pass in Geography if minimum
(e) None of these passing marks is 40%?
69. For which two states the density of population is (a) 74 (b) 106
approximately equal? (c) 96 (d) Can’t say
(a) No state (b) A and G (e) None of these
(c) D and F (d) C and F 76. How many students have obtained 60 or more marks in at
(e) None of these least one of the five subjects?
(a) 111 (b) 103
70. Approximately how much more is the density of population
(c) 108 (d) 106
of state B in comparison to that of state A?
(e) Data inadequate
(a) 15000 (b) 18000
77. If the criteria for distinction is minimum 75% marks in Maths,
(c) 13000 (d) 14000 how many students will get distinction?
(e) 17000 (a) 37 (b) 27
71. In case of how many states the density of population was (c) 12 (d) Can’t say
more than 12 thousand per square kilometre? (e) None of these
(a) Two (b) Five 78. The no. of students who obtained more than or equal to
(c) Three (d) Four 40% marks in Science is what per cent less than that of
those who scored less than 60% in Hindi?
(e) None of these
(a) 17.60% (b) 15.40%
72. What is the approximate ratio of the areas of state B to the (c) 19.80% (d) 24.30%
areas of state A and G together. (e) None of these
(a) 1 : 3.8 (b) 1 : 3.5
Directions (Qs. 79-83) : Study the following data carefully and
(c) 1 : 5.2 (d) 1 : 4.5 answer accordingly.
(e) 1 : 4.1 Following chart shows the number of students in different
73. The surface area of a spherical part of a bowl with a flat universities
circular detachable cover, excluding the cover, is 616 sq cm. Delhi Uni
Indraprastha 35%
The area of the cover is 38.5 sq cm. What is the volume of
Uni 13%
the bowl?
(a) 1339 cm3 (b) 1430 cm3
(c) 1570 cm3 (d) Cannot be determined Hamdard
14%
(e) None of these
Direction (Qs. 74-78) : Study the following table carefully and J.N.U
20%
answer accordingly : Jamia
18%
The distribution of marks (out of 150) obtained by 180
students in each of the five subjects. Total no. of students = 120,000

www.newspaperkorner.wordpress.com
P2-8 SBI PO EXAM 2011 SOLVED PAPER
www.newspaperkorner.wordpress.com
Percentage of listeners of different FM channels in 85. How much time will Dinesh take to walk a km distance?
National Capital Region I. The ratio of the speeds at which Dinesh and Ranjay
walk is 5 : 6.
FM Chann els II. The average walking speed of Dinesh and Ranjay is
Un ivers ities ¯ Rad io Radio Red FM known.
Rain bo w
M irch i City FM Go ld 86. Is a two-digit integer ‘x’ divisible by 12?
Ind rap ras tha 76% 72% 46% 54% 48% I. When ‘x’ is divided by 5, the remainder is 2.
Hamd ard 63% 64% 59% 47% 53% II. When ‘x’ is divided by 3, the remainder is 1.
JNU 52% 65% 64% 51% 54% 87. Is the average of a, b and c equal to b?
DU 82% 44% 32% 35% 45% I. b - a = c - b
II. a, b and c are positive integers.
Jamia 75% 32% 36% 52% 64%
88. What is the monthly income of Rahim?
79. How many students of JNU listen to Radio city? I. Total monthly income of Rahim and Suresh is ` 27000,
(a) 15200 (b) 15600 which is 150% of their total monthly expenditure.
(c) 14400 (d) 14600 II. The ratio of their monthly expenditures is 5 : 4
(e) None of these Directions (Qs. 89 - 92) : Answer these questions on the basis of
80. The no. of Indraprastha students listening to Rainbow is the information given below :
what per cent of the no. of Jamia students listening FM
In a survey of 1000 boys conducted in an area, it is found that
Gold?
65% play Cricket, 48% play Football and 40% play Hockey. Of the
(a) 65 (b) 56
total, 30% play both Football and Cricket, 25% play Football and
(c) 68 (d) 58
Hockey, while 24% play Cricket and Hockey. Only 5% do not play
(e) None of these
any of the three games.
81. From which of the following universities, the no. of students
liking Red FM is minimum? 89. Find the number of players who play Football but not
(a) Indraprastha (b) Jamia Hockey.
(c) JNU (d) DU (a) 180 (b) 230
(e) Hamdard (c) 350 (d) Can’t be determined
82. How many students of Indraprastha and Jamia together (e) None of these
listen to Red FM? 90. How many play all the three above-mentioned games?
(a) 12562 (b) 12872 (a) 180 (b) 240
(c) 14952 (d) 14272 (c) 230 (d) 210
(e) None of these (e) None of these
83. Which of the following channels is the most popular among 91. How many play Hockey but neither Cricket nor Football?
the students of Hamdard and JNU? (a) 140 (b) 320
(a) Radio Mirchi (b) Radio city (c) 120 (d) Can’t be determined
(c) Red FM (d) FM Gold (e) None of these
(e) Rainbow 92. Find the percentage of players who play only Football.
(a) 12 (b) 14
Directions (Qs. 84 - 88) : Each of the questions below consists
of a question and two statements numbered I and II given below (c) 32 (d) 18
it. You have to decide whether the data provided in the statements (e) None of these
are sufficient to answer the questions. Read both the statements Directions (Q s.93-97) : The following questions are
and: accompanied by three statements A, B and C. You have to
Give answer (a) if the data in statement I alone are sufficient to determine which statement(s) is/are necessary/sufficient to
answer the question, while the data in statement II alone are not answer the question.
sufficient to answer the question.
93. P, Q and R together invested an amount of `42000 in the
Give answer (b) if the data in statement II alone are sufficient to ratio of 4 : 3 : 7 for different periods of time. What was the
answer the question, while the data in statement I alone are not amount of profit earned by them individually at the end of
sufficient to answer the question. one year?
Give answer (c) if the data either in statement I alone or in A. They invested for periods in the ratios of 1 : 2 : 1.
statement II alone in sufficient to answer the question. B. R’s profit is ` 5500 less than Q’s investment.
Give answer (d) if the data even in both the statements I and II C. Total amount of profit at the end of one year is `8800.
together are not sufficient to answer the question. (a) Only A and B together
Give answer (e) if the data in both the statements I and II together (b) Only A and C together
are necessary to answer the question. (c) A and either B or C
84. What is the height of a triangle? (d) All statements are required
I. It is a right-angled triangle. (e) Question can’t be answered even after using all the
II. The area of the triangle is 5 times its base. informations
www.newspaperkorner.wordpress.com
SBI PO EXAM 2011 SOLVED PAPER P2-9
www.newspaperkorner.wordpress.com
94. What was the sum of the ages of the mother and the son Directions : (Qs. 98-100) : Study the following table carefully
ten years earlier? and answer accordingly :
A. Ratio of the mother’s present age to the son’s age after The different five-star hotel projects completed
10 years is 5 : 3. by different companies.
B. The difference between thrice the present age of the No. of Cos t Year of
mother and five times the present age of the son is 50 Project Company
rooms (` cr) completion
years.
A 600 275 1998 P
C. The ratio of the age of the mother after 10 years to the B 320 210 1999 Q
age of the son after 16 years is 5 : 3. C 250 250 1999 R
(a) Any two of them D 400 430 1998 S
(b) Any one of them E 520 310 2000 T
F 450 400 1998 U
(c) B and either A or C
G 500 250 2000 V
(d) All statements are required
98. Which project had the minimum cost per room?
(e) Question can’t be answered even after using all the
(a) A (b) B
informations
(c) G (d) E
95. What will be the cost of fencing a rectangular plot?
(e) D
A. Cost of fencing a circular plot whose area is 616m2 is 99. What is the investment in projects to be completed in 1998
` 968. as percentage of investment in projects to be completed in
B. Perimeter of the rectangular plot is 200 m. all three years?
C. Perimeter of the square whose length is equal to the (a) 41% (b) 52%
breadth of the rectangular plot is 20 m. (c) 47% (d) 56%
(a) Only C (b) A and C together (e) None of these
(c) A and B together (d) A and either B or C 100. What is the approximate average number of rooms that
would be built per crore of rupees over the three-year period?
(e) Question can’t be answered even after using all the
(a) 1.25 (b) 1.70
informations
(c) 0.90 (d) 1.40
96. What is the cost price of an article? (e) 1.55
A. After allowing a discount of 10% on marked price the
ENGLISH LANGUAGE
shopkeeper charges ` 810.
B. If the shopkeeper does not give the discount the Directions (Qs. 101-109) : Read the following passage carefully
shopkeeper gets a profit of 50%. and answer the questions given below it. Certain words are
C. If the shopkeeper gives only 5% discount on marked given in bold to help you locate them while answering some of
price, he will have 42.50% profit. the questions.
(a) Any two of them (b) Only A and B together It may be quite a while before climatologists are able to
predict rainfall in the American Midwest by measuring snow-fall
(c) Only B and C together (d) A and either B or C in the Himalayas. But there is one prediction which they can
(e) Only A confidently make now, and that is that the earth’s ice cover —
97. What is the rate of interest at which Binod has invested from the polar ice caps to the Himalayas — is thawing at an
money? alarming rate. So much so that over 50 per cent of the planet’s
mountain glacier mass could be history by the turn of the next
A. The compound interest at this rate of ` 2500 in 2 yrs is
century. According to the latest findings of the US-based
2 environmental thinktank, Worldwatch institute, the Arctic Sea
equal to the simple interest in 3 yrs of ` 1716 at the ice has shrunk by nearly 40 per cent in the last 25 years, even as
3
Antarctica’s extensive ice fields and glaciers have been badly
same rate. ‘bleeding’ at their edges. This would indicate that the earth has
B. The total simple interest on an investment of ` 12000 entered a period of climatic change that is likely to cause
for 3 yrs and `10000 for 5 yrs at this rate is `5160. widespread environmental, economic and social disruption over
C. In 3 yrs `1500 at the same rate becomes `1725 by simple the next century if emissions of heat-trapping gases are not
reduced. As a result of global warming, average planetwide
interest.
temperatures have been going up steadily.
(a) Any of them (b) Either B or C If the levels of carbon dioxide — the bad boy of global
(c) Only C (d) Only A and either B or C warming — in the atmosphere are allowed to increase at the
(e) Any two of them present rate, more heat will be trapped in the planetary cocoon,
www.newspaperkorner.wordpress.com
P2-10 SBI PO EXAM 2011 SOLVED PAPER
www.newspaperkorner.wordpress.com
raising global temperatures to scorching highs. The polar ice 106. Over 50 per cent of the planet’s mountain glacier mass could
caps will melt and the resultant rise in sea levels will be catastrophic be history by the turn of the next century means
for low-lying island-states and countries with large coastal (a) In the next century mountain glacier will be taught as
populations, such as Marshall Islands and Bangladesh. Regional a history subject.
flooding will threaten water supplies and dramatically alter the (b) As the next century starts, over half of the mountain
habitats of many flora and fauna. This is particularly bad news glaciers will have become extinct.
for such regions as northern India, home to half of the total Indian (c) After the next century less than half of the planet’s
populace who depend wholly on the glacier-fed rivers for their mountain glacier will be remaining.
drinking water and irrigation needs.
(d) Next century will damage the glaciers heavily.
With the Himalayan ice caps melting like ice-cream on a hot
(e) Glacier will be a subject of interest in the next century.
summer day, these snow-fed rivers will first swell and then run
107. What efforts need to be taken to stabilise the climate?
dry, triggering off devastating floods, followed by a desolating
drought. People used to think there was time to sort out problems (a) Use of carbon should be checked and alternatives for
related to climate change, but no longer. The chilling prospect of it should be looked for to drive our industries and
an imminent global glacial melt calls for immediate damage control transport.
exercises to stabilise the climate. A good way to begin, perhaps, (b) Mountains and glaciers should be preserved at any
will be to overhaul the energy and transportation systems which cost
drive the world’s fossil fuel economy and, instead develop low- (c) The Himalayas should be given special protection as
carbon energy systems based on electronic technologies. they shape the climate.
101. Prediction of rainfall in the American Midwest depends on (d) People should not reside near natural flora and fauna.
(a) the rainfall in the region in previous years. (e) None of these
(b) the climatic conditions in the Himalayan region. 108. If the polar ice cap melts and the sea level rises
(c) the condition of glaciers. (a) it will bring more rain.
(d) the changing patterns of the season. (b) it will increase the amount of water in the world.
(e) None of these (c) it will submerge low coastal areas, thus destroying life
102. Which of the following will be the consequence(s) if there and property.
is a rapid decline in the ice cover of earth? (d) hilly areas will come under water.
(I) It will bring ecological disaster.
(e) excess water will generate diseases and epidemic.
(II) It will have negative effect on the economy.
109. Give a suitable title to the passage.
(III) It will affect the normal life mainly in the advanced
nations. (a) Glacier as a source of water
(IV) It will snatch the dreamland of our poets. (b) Glacier causing floods
(a) Only I and II (b) Only III and IV (c) Global warming and human survival
(c) Only III (d) Only I (d) Glacier and its importance
(e) None of these (e) Save water.
103. What is the prime cause behind the shrinking of ice fields? Directions (Qs.110-112) : Choose the word which is same in
(a) human activity taking place at these places meaning as the word given in bold as used in the passage.
(b) scientific experiments being done in these area
110. THAWING
(c) rising temperature due to pollution in atmosphere
(a) diminishing (b) receding
(d) drying up of rivers which are snow-fed
(c) evaporating (d) melting
(e) None of these
104. What measure is imminent for saving our mountain glaciers (e) breaking
and ice fields? 111. SCORCHING
(a) launching a worldwide campaign to save them (a) extreme (b) mild
(b) checking the emission of carbon dioxide into the (c) uneven (d) odd
atmosphere (e) rapid
(c) Making people aware of our environment 112. IMMINENT
(d) depending less on the environment for livelihood (a) future (b) impending
(e) None of these (c) supposed (d) thought
105. Which of the following is false in the context of the passage? (e) surmise
(a) Worldwatch Institute is related to America.
(b) Ice cover of the earth is receding at a fast pace. Directions (Qs. 113-115) : Choose the word which is opposite
(c) Carbon dioxide is the major contributor to global in meaning of the word given in bold as used in the passage.
warming. 113. SHRUNK
(d) Snow-fed rivers will always have water in it. (a) developed (b) emerged
(e) None of these (c) built (d) widened
(e) multiplied
www.newspaperkorner.wordpress.com
SBI PO EXAM 2011 SOLVED PAPER P2-11
www.newspaperkorner.wordpress.com
114. CATASTROPHIC 123. (A) Unkempt (B) Unremitting
(a) fortunate (b) yielding (C) Slackening (D) Distasteful
(c) contributing (d) ushering (a) A-B (b) B-C
(e) jovial (c) C-D (d) A-D
115. STEADILY (e) B-D
(a) gradually (b) systematically 124. (A) Gregarious (B) Quixotic
(c) slowly (d) simply (C) Sociable (D) Discernible
(e) inconstantly (a) A-B (b) B-C
Directions (Qs. 116-120) : In each of the following sentences (c) C-D (d) A-C
there are two blank spaces. Below each sentence there are five (e) B-D
pairs of words denoted by (a), (b), (c), (d) and (e). Find out 125. (A) Apathetic (B) Wrath
which pair of words can be filled up in the blanks to make the (C) Whirl (D) Twirl
sentence meaningfully complete. (a) A-B (b) A-C
116. _________ of illiteracy from a nation that is set to become (c) A-D (d) B-C
the most populated in the world is by no ________easy. (e) C-D
(a) Countering, task (b) Driving, measure
Directions (Qs. 126-130) : Some sentences are given below.
(c) Curbing, way (d) Eradication, means You have to arrange them in order to frame a meaningful
(e) Removal, point paragraph and then answer the following questions.
117. It is time to ________ ongoing programmes and ______ A. Invaders came to India and looted it of its wealth.
new horizons. B. Today we may be rich in wealth, but not rich at heart.
(a) value, choose (b) speculate, experiment C. Sadly, now the situation has changed.
(c) reject, consider (d) scrutinise, impound D. India has a glorious past with rich cultural heritage.
(e) assess, seek E. In fact, materialism has taken the place of spiritualism.
118. This approach would ________ the enormous illiteracy F. But they too admired the Indians.
problem to be ________ in a holistic manner. 126. Which sentence will come at FIRST place in the paragraph?
(a) enable, tackled (b) focus, viewed (a) A (b) B
(c) envision, dealt (d) combine, judged (c) C (d) D
(e) review, countered (e) E
119. The _______ of criminalisation of politics needs to be 127. Which will be the LAST sentence of the paragraph?
_________ far more seriously. (a) A (b) B
(a) lacuna, dealt (b) issue, addressed (c) C (d) D
(c) system, broken (d) continuation, suppressed (e) E
(e) process, diverted 128. Which sentence will come at SECOND place in the
paragraph?
120. It would be proper for India to judge Pakistan by its _______
(a) E (b) D
rather than ________ .
(c) F (d) A
(a) credentials, potentials (b) culture, politics
(e) B
(c) actions, words (d) promises, assurances
129. Which will be the FIFTH sentence in the paragraph?
(e) nature, behaviour (a) A (b) B
Directions (Qs. 121-125) : In each of the following questions (c) C (d) D
four words are given, of which two are most nearly the same or (e) E
opposite in meaning. Find the two words which are most nearly 130. Which will be the THIRD sentence of the paragraph?
the same or opposite in meaning and mark the number of the (a) F (b) B
correct letter combination as your answer. (c) D (d) E
121. (A) Discomfit (B) Baffle (e) A
(C) Epicure (D) Enumerate Directions (Qs. 131-140) : Read the following passage carefully
(a) A-B (b) A-C and answer the questions given below it. Certain words are
(c) A-D (d) B-C given in bold to help you locate them while answering some of
(e) B-D the questions.
122. (A) Testimony (B) Aura Nearly 6000 years ago, man took a giant step forward in his
(C) Augment (D) Decrease evolution on this earth when, giving up a nomadic type of life, he
(a) A-B (b) B-C took to agriculture. This memorable event happened in certain
(c) C-D (d) A-D river valleys of India, China, Mesopotamia (modern Iraq) and
(e) B-D Egypt. The reason was that the conditions in these areas were
www.newspaperkorner.wordpress.com
P2-12 SBI PO EXAM 2011 SOLVED PAPER
www.newspaperkorner.wordpress.com
favourable for the cultivation of grains like wheat, rice etc because 131. Which step of man marks the end of his life as wanderers
of plenty of rich soil and water. Consequently, the farmer was on this earth?
able to produce more grains than he required for his needs. This (a) his taking up of jobs which were useful to the society
helped in the development of settled cooperative living and (b) his shift towards agricultural activities
villages appeared where some people could make a living using
(c) development of the sense of brotherhood
some specialised skills or take up other jobs useful to the society
instead of tilling the soil. (d) evolution of trade and commerce
As time passed more and more families started living (e) None of these
together for the sake of security against barbarians and wild 132. What led to the development of farming in India, China,
animals. They built big buildings and cities sprang up. Such Mesopotamia and Iraq?
developments took place in many parts of the world where (a) curiosity for producing grains among the people in
conditions were suitable for agriculture, trade and commerce. Since these areas
it was not hard to make a good living, men had spare time for the (b) early knowledge of agriculture among the people in
development of arts and crafts and engage in other activities these areas
which characterise a cultured life. (c) proper climate for agriculture
The ancient civilizations, though quite far removed from (d) vast area was available
each other, did not develop in complete isolation. There was
(e) None of these
some trade and commerce accompanied at times by sharing of
knowledge. It was not uncommon to have from time to time small 133. Development of villages became easy because
or big wars and mass exodus of people for greener pastures. By (a) people gave up agriculture and took to alternate jobs.
1000 BC, with considerable intermingling of races, several well- (b) a sense of security led people to live in units like
developed and prosperous civilisations had come into existence. villages.
Their borders of course were not well defined but kept on shifting (c) people were fed up of nomadic lifestyle.
due to wars. Here we shall be concerned essentially with some of (d) people got easy lifestyle in such a condition
the fundamental contributions of the ancient Indian and Greek
(e) None of these
civilisations to human knowledge. One may like to know why
these two could make more important and enduring contributions 134. Which of the following is true in the context of the passage?
than other civilisations. The answer briefly is that by and large (a) Agriculture, trade and commerce marked the
both in India and Greece conditions for free, sober and intelligent development of the society.
thought were an integral part of the social structure. Furthermore, (b) Living together made people brave enough to kill the
the philosophers were highly respected and they could without a wild animals.
care (c) Man’s nomadic lifestyle gave birth to agriculture
Follow knowledge like a sinking star (d) Man took up to nomadic lifestyle nearly 6000 years
Beyond the utmost bound of human thought. from now.
To begin with, the philosophical developments in the two (e) None of these
civilisations started along similar lines, with attempts to answer 135. Which of the following is false in the context of the passage.
some basic questions like : Who am I? How was the world created?
(a) Concern for security was one of the reasons which
Is there life after death? And so on. This is not surprising since
made families live together.
the forefathers of the Indo-Aryans and the Greeks lived together
in Central Asia before mass migration. But in due course, around (b) Fertile lands supported agriculture in India, China,
500 BC, the effects of different environments and some intermixing Mesopotamia and Egypt.
with other races began to have their effect and change in emphasis (c) Ancient civilisations developed without having any
in the pursuit of knowledge was noticeable. The Greeks, because support from one another.
of their rugged surrounding and contacts with other seafaring (d) Trade and commerce supported the cause of
people in trade and commerce, had to be practical. They turned development.
more and more to the study and understanding of the ‘outer (e) None of these
world’. So successful were they in their endeavour and so basic 136. Why did the established civilisations have no fixed places
were their contributions that it is now quite generally accepted or properly demarcated borders?
that the entire modern intellectual education of the West, both
(a) because several civilisations established themselves
philosophical and scientific, originates from the Greeks. The Indo-
close to each other
Aryans, on the other hand, once they had settled down
comfortably in different parts of India did not have to face many (b) because of the changing nature of the civilisations
challenges from the outside world. Consequently, they turned (c) because of the merging of smaller civilisations into
more and more towards religion and the study of man’s ‘inner bigger ones
world’. The degree of success and the mastery they achieved is (d) because of wars and migration of people to different
best summed up by Schopenhauer, a German philosopher. areas
Commenting about Upanishidic knowledge, he remarked, (e) None of these
“Thinking was finished on the banks of river Ganges.”
www.newspaperkorner.wordpress.com
SBI PO EXAM 2011 SOLVED PAPER P2-13
www.newspaperkorner.wordpress.com
137. What led the Greeks to expand their knowledge of the ‘outer 145. (a) grey (b) deteriorating
world’? (c) dry (d) rustic
(a) their conquest of various countries (e) uneven
(b) their access to the other countries of the world via sea 146. (a) continued (b) drastic
route (c) multiple (d) unjustified
(c) their social recognition in foreign countries (e) insignificant
(d) their constant fighting with other countries 147. (a) unique (b) disastrous
(e) None of these (c) umpteen (d) extensive
138. The Indo-Aryans with a religious bent of mind turned to (e) amazing
the study of man’s inner world because 148. (a) pollution (b) cultivation
(a) they were at peace in their home and having little (c) deployment (d) demand
interference from the outside world (e) erosion
(b) they never reflected aggression even if challenged by
149. (a) urgency (b) misconception
other countries
(c) activities (d) action
(c) the study of the outer world was assigned to the
(e) misdeed
Greeks.
150. (a) major (b) demaging
(d) they had no idea of the outer world.
(c) effective (d) dastinctive
(e) None of these
(e) concerned
139. Which of the following is most similar in meaning toward
‘Exodus’ as used in the passage? GENERAL AWARENESS
(a) departure (b) trial 151. Which of the following books is written by British Nobel
(c) awareness (d) sensitivity Prize-winning author Doris Lessing?
(e) fleet (a) Wolf hall (b) Sea of Poppies
140. Which of the following is most opposite in meaning to word (c) Silent Spring (d) The Grass is Singing
‘Rugged’ as used in the passage? (e) None of these
(a) smooth (b) straight 152. The ‘Type-66’ agreement between China and Pakistan is
(c) furnished (d) polished related to the development of
(e) curved (a) Nuclear reactors (b) Industrial units
Directions (Qs. 141-150) : In the following passage there are (c) Roads/Bridges (d) Oil refinary
blanks, each of which has been numbered. These numbers are (e) None of these
printed below the passage and against each five words are 153. Who is the chairman of the committee constituted by RBI
suggested, one of which fits the blank appropriately. Find out to study issues and concerns in the Micro Finance
the appropriate words. institutions (MFI) Sector.
There has been a 141 change in people’s 142 towards and (a) Y H Malegam (b) Vijay kelkar
awareness 143 the environment. People are more 144 about the (c) C. Rangrajan (d) M. Damodaran
145 condition of their environment. Land in all societies is (e) Pranab Sen
subjected to 146 usage, viz. crop production, forest cover, 154. Which of the following awards is given in the field of Science
grassland, urbanisation and industrialisation etc. During the last & Technology?
five decades 147 developmental activities have taken place. Slope (a) Kalinga Prize (b) Dhyanchand Award
failure and 148 of soil due to such 149 are 150 environmental (c) Arjun Award (d) Moortidevi Award
hazards. (e) Shanti Swarup Bhatnagar Award
141. (a) special (b) tremendous 155. Who among the following has won the title of Swiss Open
(c) extreme (d) developmental Grand Prix Gold held in March 2011?
(e) supportive (a) Saina Nehwal (b) Ji Hyun Sung
142. (a) information (b) feasibility (c) Shixian Wang (d) Eriko Hirose
(c) debate (d) participation (e) None of these
(e) attitude 156. RBI and the government has decided to increase the
143. (a) protecting (b) covering percentage of people connected to bank to 80%. At present
(c) towards (d) surrounding how much percent of peoples have access to banking
(e) of facilities?
144. (a) concerned (b) biased (a) 30% (b) 38%
(c) casual (d) desperate (c) 40% (d) 52%
(e) confused (e) 60%
www.newspaperkorner.wordpress.com
P2-14 SBI PO EXAM 2011 SOLVED PAPER
www.newspaperkorner.wordpress.com
157. The government and the RBI had set a target to cover 73,000 165. At present, who is Prime Minister of Nepal?
villages having population in excess of 2000 to provide (a) Girija Prasad Koirala (b) Ram Baram Yadav
access to banking services, by March 2012. The name given (c) Jhalanath Khanal (d) Madhav Kumar Nepal
to this scheme is (e) None of these
(a) Swabhimaan (b) Swavalamban 166. A device which make use of pen for drawing pictures or
(c) Saral (d) Connecting Banks writing information to interact with a computer is called
(e) None of these ___.
158. Who among the following has been awarded with the (a) Printer (b) Scanner
Russia’s highest honour ‘The Order of St. Andrew’? (c) Touch-Pen writer (d) Plotter
(a) Sonia Gandhi (b) Angela Markel (e) Stylus
(c) Mikhail Gorbachev (d) Nicolas Sarkozy 167. RBI appointed a committee under the chairmanship of D.
(e) None of these Mohanty to look into
159. Which of the following agencies/institutions report asked (a) 2G spectrum scam
the income tax department to ensure that a single tax payer (b) Implementation of Base Rate
is not issued multiple PAN cards because gap between PAN (c) Bank's Saving Interest rate
holders and the number of returns filed was 617.1 lakh? (d) Inflation
(a) IRDA (e) None of these
(b) SEBI 168. A mouse and printer make use of ________.
(c) RBI (a) infrared technology (b) network resources
(d) Comptroller and Auditor General (c) device drivers (d) web protocols
(e) None of these (e) None of these
160. Mitalee Jagtap Paradhar has won the best actress award at 169. A service which provides user interface to access the
58th National Film Awards ceremony for her performance in available multimedia resources over a network is called ____.
which of the following movies? (a) UseNet (b) URL
(a) Baboo Band Baaja (b) Aadukalam (c) Web (d) Internet Relay Chat (IRC)
(c) Adaminde Makan Abu (d) Do Dooni Char (e) None of these
(e) None of these 170. To communicate with each other, computers make use of
161. Who among the followings has been appointed as a director __________over a network.
on board of Bank of America – the first non-American on (a) File Server (b) URL
the board of the one of the largest financial institutions in (c) Domain (d) Website
the world? (e) Protocols
(a) Mukesh Ambani (b) Ratan Tata 171. A ________ is when you turn the computer on from an off
(c) Azim Premji (d) K V Kamath position.
(e) Chanda Kocher (a) cold boot (b) warm boot
162. According to 2011 Census figures which of the following (c) floppy boot (d) bootfailure
states is most populus? (e) None of these
(a) Maharashtra (b) Uttar Pradesh 172. A RAM in a computer provides _________ of data.
(c) Bihar (d) West Bengal (a) permanent storage (b) volatile storage
(e) Madhya Pradesh (c) virtual storage (d) distributing storage
163. Which of the following schemes is restructured as National (e) None of these
Rural Livelihood Mission (NRLM), an ambitious scheme to 173. Unsolicited or undesired electronic messages which are
eradicate poverty in rural areas? designed to trick recipients into giving up personal and
(a) SHGs financial information is called ________ .
(b) Swarnajayanti Gram Swarojgar Yojna (a) web-bug (b) spyware
(c) Mid Day Meal (c) malware (d) trojan horse
(d) Integrated Rural Development Programme (e) spam
(e) None of these 174. A common tool used to limit access to certain material on
164. The six-unit nuclear power project is being set up at Jaitapur the World Wide Web or protecting the childs from
in Maharashtra by which of the following countries? questionable sites on the Internet is called __________.
(a) France (b) USA (a) webroot (b) spybot
(c) China (d) Japan (c) web utility (d) keylogger
(e) Russia (e) filter
www.newspaperkorner.wordpress.com
SBI PO EXAM 2011 SOLVED PAPER P2-15
www.newspaperkorner.wordpress.com
175. The administrator in a organization consider a 183. A prospect is a
microcomputer as (a) business goals
(a) a simple electronic machine (b) document for marketing
(b) an important component of information system (c) potential buyer of a product or service
(c) a powerfull tool of productivity (d) day-to-day function
(d) a calculating machine (e) None of these
184. Marketing of goods is required if
(e) None of these
(a) when goods become stale
176. The disk-caching feature improves the
(b) demand>supply
(a) performance of hard disk
(c) supply> demand
(b) speed of processor (d) supply = demand
(c) performance of monitor (e) None of these
(d) performance of CD drive 185. Selling is _____.
(e) None of these (a) different from Marketing
177. What does the term POS stands for (b) a sub-function of marketing
(a) preparation for sale (c) same as Marketing
(b) position of sales (d) more than Marketing
(c) point of sale (e) All of these
(d) point of superiority 186. Good selling skills involves _____.
(a) patience (b) perseverance
(e) primary outlook of salesman
(c) empathy (d) knowledge
178. Customer Center also known as
(e) All of these
(a) collection centre
187. Consumer information sources are.
(b) relationship center (a) personal and commercial sources
(c) customer friendly center (b) public source
(d) Loyality center (c) experiential source
(e) None of these (d) All of the above
179. Cross-selling is defined as (e) only (a) and (b)
(a) sale of bank products and services to existing 188. A target market is ____ .
customers (a) entire country
(b) a time bound seasonal function (b) entire city
(c) sharpens the mind-set of sales persons (c) entire globe
(d) control the flow of information into buying centre (d) customers who need a specific product
(e) None of these (e) None of these
189. 'Boost' is a product of which company?
180. Which is not a proper type of direct marketing
(a) Cadbury (b) Vicco
(a) boosting sales
(c) Wipro (d) Dabur
(b) having a focussed approach
(c) better customer contacts (e) None of these
(d) Advertising 190. Oracle and i2 Technologies are connected with
(e) None of these (a) Computer hardware (b) Networking
181. Marketing size means (c) Education (d) Operation systems
(a) getting new clients (e) None of these
(b) interacting with strangers 191. With which one of the following activities are Golden Globe
Awards associated?
(c) Market survey
(a) Journalism (b) Social work
(d) Area allocation
(c) Peace initiatives (d) Films
(e) None of these
(e) None of these
182. Marketing of Banking is the responsibilites of
192. Which of the following schemes of the Govt. of India is
(a) sales staff only associated with the health sector?
(b) back office staff only (a) NRDP (b) IRDP
(c) front office sales staff only (c) ASHA (d) Bharat Nirman
(d) all employees (e) All of these
(e) None of these
www.newspaperkorner.wordpress.com
P2-16 SBI PO EXAM 2011 SOLVED PAPER
www.newspaperkorner.wordpress.com
193. Which of the following is a sort of Poverty Eradication 197. What does SAPTA stand for:
Programme of the Govt. of India, specially designed for (a) South Asian Preferential Trade Agreement
rural areas ? (b) South Asian Post Trade Agreement
(a) Self Help Group (c) SAARC Preferential Trade Agreement
(b) Antodaya Anna Yojana
(d) SAARC Post Trade Agreement
(c) Social Security for Agricultural and Unorganized
(e) None of these
Labour
198. Which of the following state has registered the maximum
(d) Enforcement of Minimum Wages
growth in Gross State Domestic Product in the year?
(e) None of these
194. Which company was the first to launch mobile phone (a) Chhattisgarh (b) Kerala
operations in India? (c) Gujarat (d) Goa
(a) Modi Telstra (b) Reliance India (e) None of these
(c) Bharati (d) Tata Indicom 199. Which of the following banks launched India’s first food
(e) None of these and agriculture private equity fund?
195. Which from the following wireless networks has a range of (a) Robo Bank (b) Citibank
50 kilometres?
(c) HSBC (d) SBI
(a) Macintosh (b) Wi-Max
(e) None of these
(c) Wi-Fi (d) Blue Tooth
200. Which of the following banks has recently entered into a
(e) None of these
joint venture agreement with Insurance Australia Group
196. Who is the author of the book Patrons of the Poor : Caste
(IAG) for launching general insurance business in India?
Politics and Policy Making in India?
(a) Allahabad Bank (b) PNB
(a) Narayan Lakshman (b) Greg Lindsay
(c) Khushwant Singh (d) Salman Rushdie (c) SBI (d) Indian Bank
(e) None of the above (e) None of these

www.newspaperkorner.wordpress.com
SBI PO EXAM 2011 SOLVED PAPER P2-17
www.newspaperkorner.wordpress.com

Time : 60 min. Max. Marks : 50

1. Write a letter on any one of the following topics :- (10 marks)


(a) Write a letter to the Bank Manager for closing of your savings account.
(b) Write a letter to invite a friend to attend a seminar on Net Banking.
(c) Write a letter to your bank for a loan to have higher studies in USA.
2. Write a paragraph in about 150 words on one of the following : (8 marks)
(a) Rural development in India.
(b) Banking Marketing is Highly Competitive and Enjoying autonomous state.
(c) If I were an author.
3. Write an essay in about 250 words on one of the following topics :- (12 marks)
(a) Right to education.
(b) Technological Development Affecting Banking Industry.
(c) Financial Inclusion.
4. Write précis in about 150 words and provide a suitable title. (10 marks)
Growth in bank lending to local government finance vehicles in China has been steadily slowing as Beijing take steps to curb
such reckless practices.
Chinese banks have extended a huge number of loans to local government finance vehicles since Beijing started its economic
stimulus package in late 2008, with the newspaper saying some analysts put the number at 14 trillion yuan ($2.2 trillion).
But an unidentified Chinese central bank official told the paper that such estimates were exaggerated.
The real figure was actually less than 30 percent of China's overall outstanding loans, which stood at 47 trillion yuan as of the
end of last year, the official said, without providing a specific figure.
Risk associated with this lending was “generally speaking controllable” and lending growth was steadily dropping back, the
newspaper cited the official as saying.
Under Chinese law, local authorities are barred from borrowing directly to pay for projects such as the building of roads and
highways. As such, district governments have set up thousands of special financing vehicles to beat the rules.
Beijing launched a 4 trillion yuan pump-priming package in late 2008 to cushion the impact of the global financial crisis,
triggering a surge in bank credit and investment.
The stimulus resulted in unfettered lending to local government financing vehicles, hybrid government-company bodies that
local governments used to get around official borrowing restrictions.
Beijing has clamped down on such borrowing but in a sign it wants to clean up the debt mess, sources told Reuters it is
studying ways to manage losses from 2 trillion-3 trillion yuan of local government loans which it estimates may sour.
5. Read the following passages and answers the questions :- (10 marks)
The retail loan has a taken a major step and the public sector banks are providing housing loans as regular paying amount and
with almost average default loan capacity these housing loan seekers are an attractive customer base for the public sector
banks.
Public sector banks foreign banks and non banking finance companies NBFCs are also providing housing loans.
The loan lending rate is differing to different banks as each bank has different cash deposits, customer profiles different
documentation procedures.
Each bank has overhead staff costs, cost methods etc result in different interest rates in different banks. Higher the cost,
higher is the interest rate.
PSU banks are also providing educational loans which is less provided by the private sector banks , foreign sector banks and
NBFCs and apart from that housing loans , car loans but for car loans NBFCs are preferred to be the better option as interest
rates are adjusted with the given period as far as the personal loans are concerned the psu banks provide it to the government
employees only but the private and foreign banks have a much lucrative approach and it provide personal loans to all and off
course the interest rates are pretty higher for these banks .
(i) What made the retail credit have a sudden change?
(ii) What made the PSU banks go for the retail credit?
(iii) Give your own view how PSU banks have made an impact then other banks in this arena?
(iv) What is the reason behind the increased interest rates different for different banks?
(v) What is distinct feature of the PSU banks in terms of providing loans?
www.newspaperkorner.wordpress.com
P2-18 SBI PO EXAM 2011 SOLVED PAPER
www.newspaperkorner.wordpress.com
Answers & Explanations
REASONING (HIGH LEVEL) 7. (b) G R I M R M O S
+1
1. (c) Meaningful Words Þ CON ONCE +1
+1
2. (c) U I D L I Q +1
U T S R Q
D U S K L Q P I
+1
(3 - 4) : M 3m O +1
+1
5m 5m +1
Similarly,
P 3m Q S T O P P J G B
+1
2m +1
+1
N
+1
8. (a) R > P > Q > S > T
North P scored the second highest marks.
NW NE
9. (c) M I R A C L E
West East A C E I L M R
+1 –1 –1 +1 –1 –1 –1

SW SE B B F J K L Q
South
5th from the right

3. (e) 4m 3m
10. (e)
9 14 3 12 21 4 5
1 N C L U D E
5m
(11-15 ) :
3m
P
support the other group ja pe la no
2m
N the mission gains support ke ja zi la

Points R, P and Q are in a straight line. gains other than money fu no ho zi


4. (b) Points O is 7 metres towards North of Point N.
more support and money re qi fu ja
+1 +1
5. (a) O ¾¾® G E ¾¾® N
-1 11. (b) The code for ‘group’ is ‘pe’.
D ¾¾® F B ¾¾® P
–1
12. (e) ‘zi’ stands for ‘gains’.
Similarly.
13. (a) more Þ re or ql
+1 -1
A ¾¾® K L ¾¾® D than Þ ho
+1 +2 +3 +4
the Þ la
6. (d) T ¾ ¾® J ¾ ¾® N ¾ ¾® A ¾ ¾® G group Þ pe
+2 +3 +4 +5
E ¾ ¾® U ¾ ¾® K ¾ ¾® R ¾ ¾® H 14. (c) The code for ‘mission’ is ‘ke’.

www.newspaperkorner.wordpress.com
SBI PO EXAM 2011 SOLVED PAPER P2-19
www.newspaperkorner.wordpress.com
15. (a) money Þ fu of students will still join the courses. It is mentioned in
more Þ re or ql the statement that the institute is reputed. Therefore,
The code for ‘matters’ may we can assume that students will still prefer the institute
be ‘bu’. of repute despite hike in fees.
26. (d) None of the assumptions is implicit in hte statement. It
is mentioned in the statement that many residents of
(16-22) :
the locality, not all residents, decided not to allend the
A
B function. This does not constitute the strong reason
E for cancellation of function. Assumption II is a long
drawn conclusion.
D 27. (a) Only assumption I is implicit in the statement. The
K
Government advised the state governments to ensure
disbursement of relief package assuming that state
C governments are capable for evolving a mechanism
G for disbusrsement of relief.
The use of term ‘all’ in the assumption II makes it invalid.
F H 28. (b) It is clear that statement (B) is the cause and statement
16. (d) F is third to the right of B. (A) is its effect.
17. (d) F and G are immediate neighbours of H. 29. (b) It is clear that statement (B) is the cause and statement
18. (b) (A) is its effect.
A 30. (a) It is clear that statement (A) is cause and statement (B)
K A is its effect. If there is adequate supply in the retail
B E B market, the price would stabilise.
31. (b) It is clear that accidents occurred due to faulty
H
D automatic signalling system.
K C
32. (d) It is clear that both the statement (A) and (B) are effects
of independent causes.
C 33. (e) All plants are bottles.
G GD

F H
F E All bottles are caps.
19. (a) H is second to the right of C. A + A Þ A-type of Conclusion
B is second to the right of E. “All plants are caps.”
G is second to the right of F. This is Conclusion II.
20. (c) DC, DE Þ Immediate neighbours of D. All bottles are caps.
DF, DA Þ F is second to the right of D.
A is second to the right of D.
Therefore, ? = DH All caps are crowns.
H is third to the right of D. A + A Þ A-types of Conclusion
21. (b) G sits third to the left of A. “All bottles are crowns.”
22. (e) Immediate neighbours of C are D and F. Its Converse would be :
23. (e) Clearly both the assumptions are implicit in the “Same crowns are bottles”
statement. This is Conclusion I.
If there are sufficient number of passengers, the other 34. (d) Some handkerchiefs are calculators.
private airlines may also increase the frequency of daily
flights. The airline decided to increase the frequency
of daily flights assuming that there are adequate
passenger load. All calculators are papers.
24. (b) Only assumption II is implicit in the statement. If the I + A Þ I-type of Conclusion.
management of the organisation has organised picnic “Some handkerchiefs are papers.”
for its employees, it would earmark adequate funds for 35. (e) All tigers are zebra.
picnic. Again, it is not prudent to assume that the
management of the organisation would not welcome
the employees with their family at the picnic.
25. (a) Only assumption I is implicit in the statement. The All zebra are cows.
institute increased the fees assuming that good number A + A Þ A–type of Conclusion
www.newspaperkorner.wordpress.com
P2-20 SBI PO EXAM 2011 SOLVED PAPER
www.newspaperkorner.wordpress.com
“All tigers are cows.” I + A Þ I–type of Conclusion
This is Conclusion I. “Some cards are chairs.”
“All tigers are cows” can be shown as : Its Converse would be :
“Some chairs are cards.”
If some chairs are cards, then all
or Tigers, chairs being cards is a possibility.
Tigers Cows
ws
38. (d) From both the statements
Co
B
I II
Now, “All camels are cows” can be shown as
C>D
39. (e) From both the statements
cost of pen ® la ke ij
or Camels,
Camels Cow Cows pen stand ® ke hu
Therefore, stand ®hu
40. (d) Data insufficient
III IV 41. (b) From statement I
Combine the figures II and IV Mansi took leave on Tuesday.
Wednesday or Thursday
Tigers, From statement II
Cows, Mansi took leave on Friday
Camels 42. (b) From statement I
Mary has only one younger brother. She may have
Therefore, “All camels being tigers is a possibility.” elder brother.
36. (a) Some pencils are mobiles. From statement II
Mary has only one brother.
43. (e) From Problem Figure (1) to (2) the shaded block moves
All mobiles are grass. one sector in anticlockwise direction and one more
I + A Þ I–type of Conclusion block gets shaded. From Problem Figure (2) to (3) the
shaded blocks move two sectors in anticlockwise
“Some pencils are grass”.
direction and one more block gets shaded. Similar
Its Converse would be :
changes ooccur from Problem Figure (4) onwards.
“Some grasses are pencils”.
44. (d) From Problem Figure (1) to (2) one design moves
If some grasses are pencils, then all grasses being
diagonally after being rotated through 90°
pencils is a possibility.
anticlockwise, the square moves one-half step and
Some pencils are grass.
shaded part of it also moves in anticlockwise direction.
Similar changes occur from Problem Figure (3) to (4)
and from Problem Figure (5) to Answer Figure.
All grass is green. 45. (e) In the subsequent figures respectively two, three, one,
I + A Þ I–type of Conclusion two three curves are added in clockwise direction to
“Some pencils are green.” complete the leaflets.
All mobiles are grass. 46. (d) The following changes occur in the subsequent figures :
(1) to (2) (2) to (3),
(3) to (4) (4) to (5)
(5) to (6)
All grass is green.
A + A Þ I–type of Conclusion
“All mobiles are green”.
37. (a) Some cards are clips.
47. (d) After Problem Figure (3) the pattern is repeated in the
reverse order and the outer line segment moves to the
opposite side.
All clips are chairs.
www.newspaperkorner.wordpress.com
SBI PO EXAM 2011 SOLVED PAPER P2-21
www.newspaperkorner.wordpress.com
(48-50) : 58. (a) n (A È B È C) = n(A) + n(B) + n(C) - n(A Ç B)
and-A, band - B, land - C, hand - D, hind - E, lack - F, job - G
- n(A Ç C) - n(B Ç C) + n(A Ç B Ç C)
Chart Þ 100 – 18 = 42 + 68 + 51 – 30 – 28 – 36 + x
Þ x = 15
Inpu t A B C D E F G
59. (d) Perimeter = 4 ´ 160000 = 1600 m
Step I: E A B F C D G
Distance 1600 ´ 5 ´ 60
Step II: E B C G A F D Time= = = 96 min
Speed 5000
Step III: E A F B D C G
x +1 2
Step IV: C B A G D F E 60. (b) = Þ 3x - 2y = 1
y+2 3
Step V: D C B F A G E
x+2 5
= Þ 7x - 5y = 1
Step VI: D B A E C F G y+3 7
Step VII: D C F B G A E x 3
Step VIII: A B C E G F D or, 3x – 2y = 7x – 5y Þ 3y = 4x Þ =
y 4
Step IX: G A B F C E D 61. (e) 3 5 4

Step X: G B C D A F E (3 or 4 or 5)
3 × 5 × 4 = 60
48. (d) 49. (a) 50. (b) 2 5 4 3
DATA ANALYSIS & INTERPRETATION
(1 or 2)
2 × 5 × 4 × 3 = 120
æ 100 ö Total = 120 + 60 = 180
876 ç
è 219 ÷ø 400
51. (e) ´ 100 = ´ 100 » 64% 5x 5x
æ 24 ö 624 62. (d) + 600 = 1000 + (x - 4000)
1274 ç ÷
è 49 ø 100 2
Þ x = 12000
52. (b) Using table II, comparing the ratio we find that only
BS and SAI have such thing. 100
63. (a) Time taken by B = 10 ´ = 8 days
53. (a) From the table it can be observed. 125
54. (e) In 2003, TIS total production
8 ´ 10 4
Required answer = = 4 days
æ 110 ö 18 9
= 782 ç = 860.2
è 100 ÷ø or, Reqd days
æ 90 ö = 10 ¸ (1 + 1.25) =
10
=
40
=4
4
SAI total production = 665 çè ÷ = 598.5
100 ø 2.25 9 9
64. (a)
æ 860.2 ö æ 598.5 ö
Total of A-type steel = çè ´ 14÷ + ç ´ 62÷ 44
23 ø è 133 ø 65. (b) Circumference = ´ 35 = 220 cm
7
= (523.6) + (279) = 802.6
Distance travelled in 1 minute
Total of B-type steel
33000
æ 860.2 ö æ 598.5 ö = = 550 m
=ç ´ 9÷ + ç ´ 71÷ 60
è 23 ø è 133 ø
= (336.6) + (319.5) = 656.1 550 ´ 100
Required no. of revolutions = = 250
Required ratio = 8026 : 6561 220
66. (b) Required answer
3 8 6
55. (a) Required ratio = ´ = or 6 : 7 40 15
4 7 7 = 720 ´ ´ = 43.2 million litres
1 2 3 4 100 100
67. (d) Total supply from Jhelum and underground water
56. (e)
1m 3m 5m (12 + 6) = 18
= 720 ´ = 129.6 million litres
Sum of first n odd no. = n 2 100
Ans = (11 – 1)2 = 100 Consumption of Yamuna water for domestic purposes
1000 25 40 40
57. (c) 100 ´ - 100 = Þ x = 960 gm = 720 ´ ´ = 115.2
x 6 100 100

www.newspaperkorner.wordpress.com
P2-22 SBI PO EXAM 2011 SOLVED PAPER
www.newspaperkorner.wordpress.com
Consumption of Ganga water for agriculture purposes No. of students who obtained less than 60% marks in
Hindi = 19 + 59 + 47 = 125
35 20
= 720 ´ ´ = 50.4 125 - 103
100 100 Reqd % = ´ 100 = 17.60%
\ Required ratio 125
79. (b) No. of students (of JNU) listening to Radio City
129.6 18
= = = 18 : 23
(115.2 + 50.4 =)165.6 23 = 120000 ´
20 65
´ = 15600
[3 × 3 × 2 × 2 × 2 is common.] 100 100
68-73 : 80. (e) The no. of Indraprastha students listening to FM
Following table can be made easily : Rainbow = 120000 × 13% × 48%
Total area = 37249 sq kilometres The no. of Jamia students listening to
State Density of population Approx FM Gold = 120000 × 18% × 52%
(in thousands) % area \ reqd percentage
112200 120000 ´ 13% ´ 48%
A = 18 (approx) 16.7 = ´ 100 = 66.66%
6230 120000 ´ 18% ´ 52%
81. (a) Indraprastha University
83800
B = 33 (approx) 6.8 82. (c) The no. of Indraprastha students listening to
2540
13 46
64900 Red FM = 120000 ´ ´ = 12 ´ 598
C = 8 (approx) 21.8 100 100
8135 The no. of Jamia students listening to Red FM
572 1 18 36
D = ´ 100 = 7.69 (approx) 20 = 120000 ´ ´ = 12 ´ 648
7436 13 100 100
71100 \ Total students = (12 × 598) + (12 × 648)
E = 14.5 (approx) 13 = 12 × 1246 = 14952
4893 83. (b) Radio City
286 1 84. (e) From statements I and II, since it is a right-angled
F = ´ 100 = 7.69 (approx) 10 triangle, area
3718 13
1
G
86000
= 20 (approx) 11.5 Þ ´ b ´ h = 5b \ h = 10
4297 2
[Note : Density of population is the population per square 85. (e) Combining I & II, we get speed of Dinesh = 10 km/hr
kilometre.] 60
68. (b) From thae above (column 3) it can be answered. \ to walk 1 km he takes = 6 minutes.
10
States A, C and D have more than 15 per cent of total 86. (b) From statement II, we find the negative answer since
areas. the number is not divisible by 3 then it can’t be divisible
69. (c) From column 2 of the above table, it can be observed. by 12.
70. (a) Required answer = (33 - 18 =) 15 thousand. 87. (a)
71. (d) For state A, B, E and G, ie for 4 states. 88. (d) Using both the statements, we can find separate
72. (e) Required answer = 2540 : (6230 + 4297 =) 10527 » 1 : 4.1 expenditures of Rahim and Suresh but can’t find their
73. (d) separate incomes.
(89-92) :
n(C) = 650, n(F) = 480, n(H) = 400
n(C Ç F) = 300, n(F Ç H) = 250, n(C Ç H) = 240
and n(C È F È H) = (100 - 5)% of 1000 = 950
Since, n(C È F È H) = n(C) + n(F) + n(H) – n(C Ç F)
– n(F Ç H) – n(C Ç H) + n(C Ç F Ç H)
Þ n(C Ç F Ç H) = 210
The radius of the spherical part of the bowl can’t be
determined. Hence volume cannot be calculated. 50
74. (c) 60% of 150 = 90. It means those who obtained either 90 =6 F = 480
C
or more than 90 marks in the average of five subjects 90
320 140
will be declared as passed. Therefore,
210
the required number = 31 + 17 = 48
30
75. (b) Reqd no. = 52 + 41 + 13 = 106 40
76. (e) 77. (d)
H = 400
78. (a) No. of students who obtained more than or equal to 120
40% marks in Science
= 67 + 22 + 14 = 103 89. (b) 90. (d) 91. (c) 92. (b)
www.newspaperkorner.wordpress.com
SBI PO EXAM 2011 SOLVED PAPER P2-23
www.newspaperkorner.wordpress.com
93. (c) The given question gives the amounts of investment
of P, Q and R, which is P = `12000, Q = `9000 and
ENGLISH LANGUAGE
S = `21000. Statement (A) combined with the 101. (e) 102. (a)
question’s information will give us the ratio of their 103. (c) The ice fields are melting down and shrinking due to
profits, i.e. 4 : 6 : 7. Now, combine this either with B or
global warming, which is caused by emission of carbon
C, and profit earned by them can be determined.
Therefore, A and either B or C is sufficient. dioxide in the atmosphere.
94. (e) Let the present ages of mother and son be M and S yrs 104. (b) It is necessary to check and control the cause of global
respectively. warming.
105. (d) 106. (b) 107. (a)
M 5
A. Þ = or, 3M – 5S = 50 108. (c) It will be catastrophic for low-lying island states and
S + 10 3
countries with large coastal populations.
B. Þ 3M – 5S = 50
109. (c) 110. (d) 111. (a) 112. (b)
M + 10 5 113. (d) 114. (a) 115. (e) 116. (d)
C. Þ = or, 3M – 5S = 50
S + 16 3 117. (e) 118. (a) 119. (b) 120. (c)
Here, all the three equations are the same. Therefore 121. (a) same
option (e) is the correct choice. 122. (c) opposite
95. (c) Statement (A) gives the cost of fencing one metre 123. (b) opposite
ofthe plot. Combining this with statement (B), total
124. (d) same
cost of fencing can be determined.
96. (d) A. Þ Market price of the article 125. (e) same
(126-130) : DAFCBE
100
= ´ 810 =` 900 126. (d) 127. (e) 128. (d) 129. (b)
90 130. (a)
B. Þ CP of the article (with the help of A) 131. (b) Man gave up his nomadic type of life after he took to
900 ´ 100 agriculture.
= =` 600
150 132. (e) plenty of fertile land and water
Now, combining (A) with (C), 133. (e) There was boost in agriculture production which
SP = 900 × 0.95 = ` 855 helped generate other jobs useful to the society.
100 134. (a) 135. (c) 136. (d) 137. (b)
CP = 855 ´ =`600 138. (a) 139. (a) 140. (a) 141. (b)
142.50
Hence, A and either B or C are sufficient. 142. (e) 143. (e) 144. (a) 145. (b)
97. (a) Let the rate of interest be r%. 146. (c) 147. (d) 148. (e) 149. (c)
150. (a)
éæ r ö
2 ù 5150 ´ r ´ 3
A. Þ 2500 êçè1 + ÷ø - 1ú =
ë 100 û 100 GENERAL AWARENESS
12000 ´ r ´ 3 10000r ´ 5 151. (d) 152. (a) 153. (a) 154. (e)
B. Þ + = 5160
100 100 155. (a) 156. (c) 157. (a) 158. (c)
159. (d) 160. (a) 161. (a) 162. (b)
1725 - 1500
C. Þ r = ´ 100 = 5% 163. (b) 164. (a) 165. (c) 166. (e)
3 ´ 150
167. (b) 168. (c) 169. (c) 170. (e)
Hence, any one of them is sufficient.
98. (a) Only for project A, the value of cost per room is less 171. (a) A warm boot is when you reset a computer that is
than half crore, whereas in other projects it is either already on.
more than or equal to half crore. 172. (b)
99. (b) Total investments for all the projects 173. (e) Actual name given to such message is 'phishing' but
= 275 + 210 + 250 + 430 + 310 + 400 + 250 = ` 2125 cr phishing is a part/category of spam.
Investment on those projects which are completed in 174. (e) 175. (c) 176. (a) 177. (c)
1998 = 275 + 430 + 400 = `1105 cr 178. (b) 179. (a) 180. (d)
1105 181. (c) Market penetration means the depth of sales of a
\ reqd % = ´ 100 = 52%
2125 particular product in a given market. Marketing Size
100. (d) Total no. of rooms over the three-year period involves the number of sellers and buyers in the market.
= 600 + 320 + 250 + 400 + 520 + 450 + 500 = 3040 182. (a) 183. (c) 184. (c) 185. (b)
reqd average no. of rooms per crore of rupees 186. (e) 187. (e) 188. (d) 189. (a)
3040 190. (b) 191. (d) 192. (c) 193. (b)
= » 1.4 194. (a) 195. (b) 196. (b) 197. (a)
2125
198. (a) 199. (a) 200. (c)
www.newspaperkorner.wordpress.com
P2-24 SBI PO EXAM 2011 SOLVED PAPER
www.newspaperkorner.wordpress.com
Solution of Descriptive Test
accept my invitation and will attend the seminar. I await
LETTER WRITING
your reply.
1. (a) Letter to closing of savings account Yours Sincerely,
144/13, Sector 1, Rakesh.
Pushp Vihar, 1. (c) Letter to your bank for a loan to have higher studies
New Delhi - 110017 in USA.
19 Feb. 2012 65/5, Chittaranjan Park,
The Bank Manager, New Delhi - 110017
State Bank of India, 19.02.2012
J-20, Saket. The Bank Manager,
New Delhi - 110018 ICICI Bank,
Dear Sir, K-250, Saket,
Sub: Closing of Savings Account New Delhi-110017.
This is to inform you that I would like to close my bank Sub : Request for a loan
account in your bank. The details of my bank account are Sir,
given below. I would like to request to you for a loan of ten Lakhs for
Saving Account No : 986800201999747 higher studies in the USA. I have just completed my
Unused cheque leaves start from 674118 Engineering from IIT Kanpur. I wish to pursue an MBA
There are no pending cheques to be cleared under this from Stanford University. Ineed a loan of ten lakh for the
account. Please send the balance amount lying in this same. I am a long standing customer of your bank. My
account (by cheque) to my postal address. If this is too Account number is - 854328. It is where three years old. I
will be doing part time work in the states alongside my
cumbersome then the amount may be transferred to my
studies. My father will also support me in this venture. He
new bank account at DHFC Bank. The account number is
has agreed to help me pay off the loan. He is a gazette
- 060100501368
officer in the Central Government. I hope you will sanction
In case you need to contact me for further queries, you the loan as soon as possible so that I can travel to the
can either send an email to Dr. sanjeev. Singh@yahoo.com; USA in peace. There is only a month left for me to join the
or call me at 09999936857. course.
Thanking you for your kind assistance, Yours Sincerely,
Yours Sincerely, Ritesh Hitwani.
Dr. Sanjeev Singh
2. (a)
1. (b) Letter to friend for inviting to attend seminar on Net
Rural Development in India
Banking.
It has traditionally been behaved that Agriculture is one
E-266, Saket of the main occupations in India. This is because nearly
New Delhi-110017 70% of India's population is found in the villages. So it is
19 Feb 2012 naturally felt that the government should pay attention to
My Dear Rinku, rural development for the betterment of the masses. In
I am pleased to inform you that Canara Bank from which fact as far as the villages in India are concerned there is
my father retired is going to conduct a seminar on Net much to be done. Some of the villages still report lack of
Banking on the 26th of February 2012. The seminar is proper civic amenities, educational facilities, medical
being held at the Lodhi International centre. It will be facilities etc. Many programmes and projects have been
covering a number of topics of interest to the common undertaken by the government for the welfare of the rural
population. The community Development Programme in
man. One of the most important advantages of Net Banking
1952 was setup to improve the lot of the Indian farmer.
will be the accessibility to Bank Accounts from within the
Higher quality seeds, modern implements, chemicals and
comfort of your home. The seminar will focus on how to pesticides were to be made available under this plan.
operate your account through the net and get maximum Irrigation facilities, breeding of livestock, were enhanced.
advantage from it. The good thing that there is no Small scale Industries were encouraged. The Ministry of
registration fees. Participants will be given complementary Rural Construction was set up in August 1979. The IRDP
booklets on net banking. I think it will be wonderful or the Integrated Rural Development Programme was also
opportunity for all of us to obtain much needed information started in 1997. The programme aims to generate better
on such a modern tool as net banking. I hope you will employment opportunities for the poor. A number of smaller
www.newspaperkorner.wordpress.com
SBI PO EXAM 2011 SOLVED PAPER P2-25
www.newspaperkorner.wordpress.com
programmes were set up under IRDP. These include the field and make available to them a database of all
DPA or Drought Prone areas Programme, the CAD or information they would ever need. This would be mutually
Command Area Development, the DDP or Desert Area satisfying to both of us.
Development, the SFDA or the Small Farmers Development 3. (a)
Agency. A rural health scheme has also been lauched. Right to Education
"Operation Flord" assisted by the WEP or the World Food The Right to education is a human right. It ensure
Programme was set up to augment milk production. The education for one and all. Those who have not been
list is endless but the so is the need. The task continues educated at all have to be provided with basic education
and there is no end in sight. at least as stated in the International covenant, on
2. (b) Economic, Social and cultural Rights. Primary education
Banking Marketing is highly competitive should be made free and compulsory to all. Secondary
and Enjoying autonomous state. education should be made affordable and even free at
Gone are the days when Banks dominated the Financial certain levels. In fact right to education is a law by article
services Industry. The changing face of society and the 26 of the Universal Declaration of Human Rights. The
advancement of technology have ensured the fall of this successful implementation of the Right to Education can
bastion. A whole new range of financial services are being be judged with the help of the 4A's framework designed
offered by a plethora of institutions. Bank have lost much by Katarina Tomasevki, a former UN special Rapportenr.
of the auto loan market to companies like the Ford, General The 4 A's are - Availability - the availability of proper
motors and other auto manufacturing units. The credit infrastructure and facilities in any educational institution
card market has also been largely. Taken over by is an absolute must. Hygienic surroundings, drinking
companies formed for this purpose. Apart from this there water, availability of the requisite furniture books, etc. make
are also employee credit unions to contend with. In earlier an institution worth standing in properly liained and
times they were very few private banks people deposited qualified staff can make students high achievers.
their money mostly in the Nationalised banks. The Accessibility - the institution should be accessible to all
government has nationalized 14 major banks. The State regardless of caste, creed and religion. Schools must be
Bank of India is one among them. These banks are enjoying centrally placed or else safe and convenient transportation
the autonomous state. Banking Marketing is an important should be provided especially in rural area. Textbooks,
aspect of their existence since now they have to deal with uniforms etc. should be available free of cost at least at
not only private banks but a whole lot of other the primary level.
competitions as well. These autonomous banks have to Acceptability - the education provided should be of a
now plan out their marketing strategies, to establish secular nature. Teachers should be told not give corporal
customer loyalty. They are autonomous bodies. In that punishment to students. They should have a professional
there will not be any interference from the government in approach.
spite of the nationalization. Different marking tools used Adaptability - the education system should be in keeping
by them include Advertising in various media, giving away with the times. Care should be taken to observe all religions
of complimentary gifts promotional items, improving holidays. Disabled students should be provided adequate
customer relations through telecalling, SMS. etc., giving care.
customers access to Internet Banking, ATMS and other In India the Right to Education grants free education to all
modern services. fill the age of 14. Child labour is banned. At secondary
2. (c) levels there are reservations for the underprivileged.
If I were an author. 3. (b)
If I were an author I would like to write of magic, miracles, Technological Development
happiness, joy, romance. Today when I look all around I Affecting Banking Industry.
find sorrow, tears, pain, struggles. I see youngsters like Technological development has affected the Banking
me undergoing stress and strain, struggling to meet the Industry both negatively and positively. Fortunately more
expectations of their parents and relatives; buckling under positively and less negatively. On the one hand the
peer pressure, taking to drugs. All I want to do is to alleviate traditional concept of bank where you simply deposited
this suffering; to embalm their wounded minds. As an money for safe - keeping has undergone a drastic change.
author I can take them into a world of fantasy, where there On the other the modern banking system sets out to
is no sorrow or pain. There are other concerns too. This is provide customers with a myriad range of facilities due to
the age of information technology. Citizens across the the advancement of technology. The three most important
globe irrespective of their age need to be well informed factors affecting the banking environment today are -
about their environment if they are to be successful in life. Imaging technology - this is also known as electronic
Students, business magnates, housewives, returned cheque Placement. Cheques now a days have MICR or
people all require information in some department or the Magnetic Ink Character Recognition embossed on them.
other. As an author I can specialize in any one of these The depositing bank uses the MICR to transmit payment
www.newspaperkorner.wordpress.com
P2-26 SBI PO EXAM 2011 SOLVED PAPER
www.newspaperkorner.wordpress.com
information to the paying bank in this way it is ensured 600 million new customer's accounts by the year 2020. But
that fraudulent cheques are not transmitted. Use of paper unfortunately there are several stumbling blocks. Illiteracy,
documentation is considerably reduced. All the data about law income savings and lack of branches in rural areas will
a particular customer is stored in a single database making make things difficult for the proper implementation of
it easy to store and retrieve information at a much faster Financial Inclusion. Even so the RBI or the Reserve Bank
pace. Obviously this cuts costs in many ways. Banks are of India which was formed in 2009 has taken some steps to
able to ensure customer satisfaction. expedite the process. Hence no-frills accounts and GCCS
Electronic Data Transfer - Close on the heels of ECP for small deposits and credit have been made available for
came EDT or electronic Data transfer by which payments the poor. Efforts are on, there is hope in the air.
were make electronically without the intervention of 4. Precise Writing
humans. There was reduction of work force. Revenue was Cleaning up the Debt Mess
generated by the fees levied for EDT
Chinese banks have been giving huge loans to local
-Smart cards and Virtual Banking -
government finance vehicles since 2008. This was the time
With the coming of ATMs, Internet and Smart Cards banking when Beijing first started its economic stimulus package
can be done from virtually. Anywhere in the world with due to the global financial cruses. A 4 trillion yuan pump-
the coming of smart cards the fraudulent transactions and priming package was launched newspapers stated that
insecurities associated with the credit card have been the loans ran into a staggering 14 trillion you that is 12.2
overcome. The PIN (Personal Identification Number) trillion dollars. These loans resulted in losses of 2-3 trillion
number on the smart card ensures that no one other than yuans of local government loans. This led to the Beijing
the true owner is able to use the card. Overdrawing of government taking. Some strict steps to curb these reckless
cash is also prevented. The smart card can be used for a practices. The Chinese law had banned local authorities
wide range of transactions from grocery purchases, to from borrowing directly for projects like the building of
loan payment, or insurance transaction, etc. the bank earns roads and highways. In a bid to get around this ban local
a tidy sum in terms of taxes bevied on these transactions. government financial vehicles were set up. But the
3. (c) government has taken steps to ensure that bank lending
Financial Inclusion. to these vehicles are considerably reduced. It is also
Financial Inclusion is the process of making available studying more ways to curb this det ness.
financial services, in an accessible manner to the poorer 5. (i) Retail credit has had a sudden change. Because now
sections of society. It is a widely accepted fact that banking it is possible to obtain huge loans from not only
services should be made accessible at easily affordable private and foreign bank but even from the public
rates to all sections of the society. This is the hallmark of sector banks. The trend continues unabated.
a well developed nation. Hence governments across the (ii) Retail credit is able to provide Public sector Banks
world, as a rule should try to attain these standards. The with a much larger customer base. Since more and
AFI or the Alliance for Financial Inclusion is a global more people are seeking loan especially housing
network of financial policymakers from developing loans, with ability to pay back in regular monthly
countries. The RBI or the Reserve Bank of India became a amounts Public sector Banks do not want to stay
member of this institution in 2012. In India, financial back in the race.
inclusion was first introduced in 2005 by K. C. (iii) PSU banks have made a greater impact than other
Chakraborthy, the chairman of the Indian Bank. Mangalam banks in this arena because the interest rates are
village was the first village in India to provide banking comparatively lower moreover for students like PSU's
facilities to all households Rules were relaxed for people are an attractive option as they are willing to provide
opening accounts with annual deposits of less than Rs. educational loans at reasonable interest rates.
50,000. General credit cards (GCCS) were issued to the (iv) The increased interest rates are different for different
poor to help them get access to easy credit. In January banks because each bank is differentiated by the
2006, commercial banks were allowed to use the services amount of cash deposits, the type of customers, the
of non-governmental organizations, micro-finance documentation procedures, overhead staff costs cost
institutions and other civil society organizations as methods etc. the greater the costs, the greaby the
intermediaries for providing financial and banking services interest rate.
by the Reserve Bank of India. These intermediaries would (v) The distinctive feature of PSU banks is that their
be the business facilitators. Commercial Banks in different interest rates are generally lower than private and
region were urged by the RBI to start a 100% Financial foreign banks. PSU banks offer education loans which
Inclusion campaign on an urgent basis. States like Himachal are not so easily available in other banks. Personal
Pradesh and Kerala responded with great enthusiasm. loans are given only to government employees in
Even Puducherry a union territory was not to be left behind. PSUS.
The Reserve Bank of India envisions the opening of nearly

www.newspaperkorner.wordpress.com
www.newspaperkorner.wordpress.com

MAIN EXAMS

Practice Set - 1

INSTRUCTIONS
• This practice set consists two parts. One part is Objective test and other part is Descriptive test.
• Each question has five options, of which only one is correct. The candidates are advised to read all the
options thoroughly.
• There is negative marking equivalent to 1/4th of the mark allotted to the specific question for wrong answer.

Time : 3 hrs. Max. Marks : 200

REASONING AND COMPUTER APTITUDE 6. Which of the following is true regarding F?


(a) F sits second to right of C.
1. Which of the following expressions is correct if the (b) F is not an immediate neighbour of A.
expression ‘Z > Y > W < V’ is definitely correct? (c) F sits third to left of D.
(a) V > Y (b) W < Z (d) F sits at one of the extreme ends of the line.
(c) Z > V (d) Y > V (e) F faces V.
(e) None of these 7. Who amongst the following sits exactly between P and Q?
Directions (Qs. 2-7): Study the following information to answer (a) R (b) V
the given questions: (c) S (d) T
Twelve people are sitting in two parallel rows containing six (e) Cannot be determined
people each, in such a way that there is an equal distance between
adjacent persons, In row 1, P, Q, R, S, T and V are seated and all of Directions (Qs. 8-12): Each of the questions below consists of a
them are facing south. In row 2, A, B, C, D, E and F are seated and all questions and two statements numbered I and II given below it.
of them are facing north. Therefore, in the given seating arrangement You have to decide whether the data provided in the statements
each member seated in a row faces another member of the other row. are sufficient to answer the question.
Read both the statements and —
A sits third to right of D. Neither A nor D sits at extreme Give answer (a)if the data in statement I alone are sufficient to
ends. T faces D. V does not face A and V does not sit at any of the answer the question, while the data in statement II alone are not
extreme ends. V is not an immediate neighour of T. B sits at on of the sufficient to answer the question.
extreme ends. Only two people sit between B and E. E does not face Give answer (b) if the data in statement II alone are sufficient to
V. Two persons sit between R and Q. R is not an immediate neighbour answer the question, while the data in statement I alone are not
of T. C does not face V. P is not an immediate neighbour of R. sufficient to answer the question.
2. Who amongst the following sit at extreme ends of the rows? Give answer (c) if the data either in statement I alone or in
(a) B, F (b) S, T statement II alone are sufficient to answer the question.
(c) P, R (d) B, F Give answer (d) if the data in both the statements I and II together
(e) None of these are not sufficient to answer the question.
3. Who amongst the following faces A? Give answer (e) if the data in both the statements I and II together
are necessary to answer the question.
(a) R (b) T 8. In a row of five A, B, C, D and E, who is standing in the middle?
(c) P (d) Q I. D is to the immediate right of E and B is to the immediate
(e) S left of E.
4. How many persons are seated between T and S? II. B is at the extreme left of the row.
(a) One (b) Two 9. How is Sushma related to Nandini?
(c) Three (d) Four I. Sushma’s husband is the only son of Nandini’s mother.
(e) None II. Sushma’s brother and Nandini’s husband are cousins.
10. Who has secured the maximum marks among six friends A,
5. P is related to V in the same way as C is related to F. Which B, C, D, E and F ?
of the following is E related to, following the same pattern? Statements :
(a) B (b) D I. B secured less marks than A and F but not less than C,
(c) C (d) A D and E.
(e) None of these II. F secured more marks than B but not as much as A.

www.newspaperkorner.wordpress.com
2 Practice Set - 1
www.newspaperkorner.wordpress.com
11. Out of the four teams A, B, C and D which team is not likely corners face the centre while those who sit in the middle of the
to win as per the opinion poll? sides face outside.
I. As per the opinion poll, chances of team C’s winning Bharat sits second to the right of Shikha. Bharat does not sit at
are more than that of team A but not as much as that of any of the corners. Meenal sits third to the right of Peter. Peter is
team B, whose chances of winning are more than that not an immediate neighbour of Shikha. Rumia and Ketan are
of team A. immediate neighbours of each other but Rumia does not sit at
II. As per the opinion poll team C’s chances of winning any of the corners of the table. Harleen is an immediate neighbour
are less than that of team B but not less than that of of neither Peter nor Shikha.
team D, whose chances of winning are more than that 17. Four of the following five are alike in a certain way and so
of team A. form a group. Which is the one that does not belong to that
12. How is ‘go’ written in a code language? group?
I. ‘you may go’ is written as ‘pit. ja ho’ in that code
language. (a) Peter (b) Rumia
II. ‘he may come’ is written as, ‘ja da na’ in that language. (c) Harleen (d) Shikha
Directions (Qs. 13 - 16) : Study the following information (e) Bharat
carefully and answer the given questions: 18. Who sits third to the left of Ali?
A word and number arrangement machine when given an (a) Bharat (b) Rumia
input line of words and numbers rearranges them following a (c) Shikha (d) Peter
particular rule in each step. The following is an illustration of (e) Cannot be determined
input and rearrangement.
19. What is the position of Peter with respect to Meenal?
Input : joy far 35 27 16 96 height star (a) Immediate to the left
Step I : 96 joy far 35 27 16 height star (b) Second to the left
Step II : 96 far joy 35 27 16 height star (c) Third to the left
Step III : 96 far 35 joy 27 16 height star (d) Third to the right
Step IV : 96 far 35 height joy 27 16 star (e) Second to the right
Step V : 96 far 35 height 27 joy 16 star 20. Who amongst the following sits second to the right of
And Step V is the last step of the rearrangement. Ketan?
As per the rules followed in the above steps, find out in each of (a) Shikha (b) Ali
the following questions the appropriate step for the given input. (c) Bharat (d) Harleen
13. Input: organize 19 12 stable room 35 72 house. How many (e) Meenal
steps will be required to complete the arrangement? 21. Who amongst the following represent the immediate
(a) Five (b) Six neighbours of Harleen?
(c) Seven (d) Four
(a) Meenal and Ketan (b) Bharat, Rumia
(e) None of these
14. Input: always go there 39 62 47 time 24. Which of the (c) Bharat, Meenal (d) Ali, Rumia
following steps will be the last but one? (e) Ali, Ketan
(a) VI (b) VII 22. Who amongst the following sits exactly between Peter and
(c) VIII (d) IX Ali?
(e) None of these (a) Only Bharat
15. Step III of an input is: 86 box 63 18 gear card 51 new. How (b) Ketan and Rumia
many more steps will be required to complete the (c) Only Harleen
arrangement?
(d) Harleen and Meenal
(a) Three (b) Two
(c) Four (d) Five (e) No one sits between Peter and Ali.
(e) None of these Directions (Qs. 23 - 28): In each question below are two/three
16. Step IV of an input is: 59 bend 46 card 14 27 win now. statements followed by two conclusions numbered I and II. You
Which of the following will be step VII? have to take the given statement to be true even if they seem to be
at variance with commonly known facts and then decide which of
(a) 59 bend 46 card now 27 win 14
the given conclusions logically follows from the given statements
(b) 59 bend 46 card 27 now win 14 disregarding commonly known facts. Give answer.
(c) 59 bend 46 card 27 now 14 win
(a) If only conclusion I follows.
(d) 59 bend 46 card 27 14 win now
(b) If only conclusion II follows.
(e) There will be no such step.
(c) If either conclusion I or conclusion II follows.
Directions (Qs. 17-22): Study the following information carefully
(d) If neither conclusion I nor conclusion II follows.
and answer the given questions.
(e) If both conclusion I and conclusion II follow.
Eight friends, Meenal, Rumia , Shikha, Ali, Peter, Harleen,
Ketan and Bharat, are sitting around a square table in such a way 23-24: Statements: All buildings are houses.
that four of them sit at four corners of the square while four sit in No house is an apartment.
the middle of each of the four sides. The ones who sit at the four All apartments are flats.
www.newspaperkorner.wordpress.com
Practice Set - 1 3
www.newspaperkorner.wordpress.com
23. Conclusions: I. No flat is a house. (b) Only (B) is implicit
II. No buildings is an apartment. (c) Only (A) and (B) are implicit
24. Conclusions: I. All buildings being flats is an (d) Only (C) is implicit
possibility. (e) None of these
II. All apartments being building is a
Directions (Qs. 31-35): In each of the questions, there are two
possibility.
sets of figures. The figures on upper side are problem figures
25-26: Statements: Some oceans are seas.
marked by letters (1), (2), (3), (4) and (5), and on the bottom side
All oceans are rivers.
are answer figures marked by numbers (a), (b), (c), (d) and (e). A
No river is a canal.
series is established, if one of the five answer figures is placed
25. Conclusions: I. All rivers can never be oceans.
in place of the (?) sign in the problem figures. That figure is
II. All canals being oceans is a your answer.
possibility.
31 Problem Figures
26. Conclusions: I. No ocean is a canal.
II. At least some seas are rivers.
27-28 : Statements: No day is night.
All nights are noon.
?
No noon is an evening. (1) (2) (3) (4) (5)
27. Conclusions: I. No day is noon.
II. No day is an evening. Answer Figures
28. Conclusions: I. No evenings are nights.
II. All days being noon is a possibility.
Directions (Qs. 29 & 30): In each question below is given a
statement followed by three assumptions (A), (B) and (C). An
assumption is something supposed or taken for granted. You (a) (b) (c) (d) (e)
have to consider the statement and the following assumptions 32. Problem Figures
and decide which of the assumptions is implicit in the statement.
29. Statement: The apex body controlling universities in the · S · = S S 0 C S C ?
country has decided to revise the syllabus of all the = 0 C ? · ? = 0
technical courses to make them focused towards he present
needs of the industry, thereby making the technical
? C 0 ? = ·
graduates more employable than they are at present. (1) (2) (3) (4) (5)
Which of the following assumptions is/are implicit in the Answer Figures
above statement?
(A) Technical colleges affiliated to different universities · S C 0 S ·
S ·
? S C = S C
may not welcome the apex body’s decision and may 0 = C 0 = 0 = ? ·
continue with the same syllabus as at present. ? ? = C ? 0 ·
(B) The industry may welcome the decision of the apex (a) (b) (c) (d) (e)
body and scale up their hiring from these colleges.
33. Problem Figures
(C) The Govt. may not allow the apex body to implement
its decision in all the colleges as it may lead to chaos. × ×
(a) None is implicit
(c) Only (B) is implicit
(b) Only (A) is implicit
(d) Only (C) is implicit
? ×
(e) Only (A) and (B) are implicit
(1) (2) (3) (4) (5)
30. Statement: Govt. has urged all the citizens to use electronic
media for carrying out their daily activities, whenever Answer Figures
possible, instead of using paper as the manufacture of paper
requires the cutting down of a large number of trees causing
severe damage to the ecosystem.
Which of the following assumptions is/are implicit in the
above statement? (a) (b) (c) (d) (e)
(A) Most people may be capable of using electronic media
to carry out various routines. 34. Problem Figures
(B) Most people may have access to electronic media for
carrying out their daily routine activities.
(C) People at large may reject the govt’s appeal and ?
continue using paper as before.
(a) Only (A) is implicit (1) (2) (3) (4) (5)
www.newspaperkorner.wordpress.com
4 Practice Set - 1
www.newspaperkorner.wordpress.com
Answer Figures 43. The __________ directory is mandatory for every disk.
(a) Base (b) Sub
(c) Folder (d) Root
(e) None of these
44. A compiler is a translating program which
(a) Translate instruction of a high level language into
(a) (b) (c) (d) (e) machine language
(b) Translate entire source program into machine
35. Problem Figures
language program
(c) It is not involved in program’s execution
(d) All of above
? (e) None of above
45. A computer programmer
(1) (2) (3) (4) (5) (a) Does all the thinking for a computer
(b) Can enter input data quickly
Answer Figures (c) Can operate all types of computer equipments
(d) Can draw only flowchart
(e) None of above
DATA ANALYSIS & INTERPRETATION

(a) (b) (c) (d) (e) Directions (Qs. 46 - 50) : In each of the following questions a
36. A software that allows a personal computer to pretend as a number series is given. Only one number is wrong in each
computer terminal is series. Find out that wrong number, and taking this wrong
number as the first term of the second series formed following
(a) Terminal adapter (b) Bulletin board
the same logic, find out the third term of the second series.
(c) Modem (d) Terminal emulation
46. 1 2 8 21 88 445
(e) None of these
(a) 24.5 (b) 25 (c) 25.5 (d) 25
37. ‘.mpg’ extension refers usually to what kind of file?
(e) None of these
(a) WordPerfect Document file 47. 6 7 18 63 265 1365
(b) MS Office Document (a) 530 (b) 534 (c) 526 (d) 562
(c) Animation / movie file (e) None of these
(d) Image file
48. 7 23 58 127 269 555
(e) None of these
38. Which of the following best describes a computer-based (a) 263 (b) 261 (c) 299 (d) 286
information system? (e) None of these
(a) Data input for processing 49. 5 4 9 18 66 195
(b) Output in produced in the form of Hard copy (a) 12 (b) 25 (c) 20 (d) 18
(c) Processing data (e) None of these
(d) System where computer is used to turn data into 50. 2 7 28 146 877 6140
information (a) 242 (b) 246 (c) 252 (d) 341
(e) None of these
(e) None of these
39. Which of the following enables you to make multiple
changes in a document at once? Directions (Qs. 51-55): These questions are based on the graph
(a) Find command (b) Replace command given below:
(c) Drag and Drop (d) Copy command Per cent profit earned by six companies during 2014 and 2015
(e) None of these Profit = Income – Expenditure
40. What is FTP program used for?
(a) Transfer files to and from an Internet Server Income – Expenditure
% Profit = ×100
(b) Designing a website Expenditure
(c) Connecting to the internet
(d) All of the above Year 2014 Year 2015
(e) None of above
41. By an intranet we mean 70
Per cent profit earned

(a) a LAN of an organization 60


(b) a Wide Area Network connecting all branches of an 50
organization 40
(c) a corporate computer network
(d) a network connecting all computers of an organization 30
and using the internet. 20
(e) None of these 10
42. The code for a web page is written using __________ .
(a) machine language (b) winzip 0
A B C D E F
(c) perl (d) HTML
(e) None of these Company
www.newspaperkorner.wordpress.com
Practice Set - 1 5
www.newspaperkorner.wordpress.com
51. If the income of company C in the year 2014 was ` 35 lakhs, (a) 2011 (b) 2012
what was its expenditure in that year? (c) 2013 (d) 2014
(a) ` 24 lakhs (b) ` 21 lakhs (e) 2015
(c) ` 25 lakhs (d) Can’t be determined 59. The total production of Company C in 2012 and 2013 is
(e) None of these what percentage of the total production of Company A in
2010 and 2011?
52. If, in the year 2015, total expenditure of companies B and C was
(a) 95 (b) 90
` 48 lakhs, then what was their total income in the same year?
(c) 110 (d) 115
(a) ` 32 lakhs (b) ` 28.6 lakhs
(e) None of these
(c) ` 34.2 lakhs (d) Can’t be determined
60. What is the difference between the average production per
(e) None of these year of the company with highest average production and
53. If, in the year 2014, expenditure of Company C was ` 32 that of the company with lowest average production in lakh
lakhs, what was the income of the company in the same year? tonnes?
(a) ` 44.2 lakhs (b) ` 48.4 lakhs (a) 3.17 (b) 4.33
(c) ` 46.4 lakhs (d) ` 38 lakhs (c) 4.17 (d) 3.33
(e) None of these (e) None of these
Directions (Qs. 61-65): These questions are based on the
54. If the expenditures of Company E in the years 2014 and following pie-chart which shows the viewership of different
2015 were the same, what was the ratio of the incomes of sports channels in the month of February 2015 in India. There
the company in the same years respectively? are no overlaps in viewership of channels.
(a) 19 : 21 (b) 11 : 12
(c) 29 : 31 (d) 9 : 11 Star
(e) None of these ESPN Sports
45° 30°
55. The income of Company D in the year 2014 was ` 31 lakhs.
What was the earned profit?
(a) ` 11 lakhs (b) ` 20 lakhs Others
60°
(c) ` 17 lakhs (d) ` 12 lakhs Set Max
(e) None of these 150°
Direction (Qs. 56-60) : Study the following graph carefully to
answer the question given below it. DD Sports
60°
Production of paper (in lakh tonnes) by 3 different Ten Sports
companies A, B & C over the years 15°
70 61. If 60000 people watched Star Sports on an average per day
60 60 60 60 in February 2015, then how many more people watched Set
60 55 55 55 55
50 50 50 50 50 Max than Ten Sports on an average per day for the same
50 45 45 45
40 period?
40
40 (a) 27000 (b) 270000
30 (c) 36000 (d) 360000
20 (e) None of these
62. During the given period for how many sports channels the
10 viewership is more than 20% of the total viewership?
0 (a) 1 (b) 2
2010 2011 2012 2013 2014 2015 (c) 3 (d) Cannot be determined
A B C (e) None of these
63. If the viewership of DD Sports of the first half of February
56. What is the difference between the production of company is half that of the second half of February, then what is the
C in 2010 and the production of Company A in 2015? ratio of viewership of DD sports for the second half to that
(a) 50,000 tonnes (b) 5,00,00,000 tonnes of ESPN for the whole month?
(c) 50,00,000 tonnes (d) 5,00,000 tonnes (a) 2 : 3 (b) 5 : 6
(e) None of these (c) 8 : 9 (d) 9 : 10
57. What is the percentage increase in production of CompanyA (e) None of these
from 2011 to 2012? 64. By mistake viewership of DD Sports has been under quoted
by 20%. If this mistake is corrected, then what is the correct
(a) 37.5 (b) 38.25
share of viewership of Set Max?
(c) 35 (d) 36
(e) None of these 3
(a) 41 % (b) 35%
58. For which of the following years the percentage of rise/fall 2
in production from the previous year the maximum for (c) 40% (d) Cannot be determined
Company B? (e) None of these
www.newspaperkorner.wordpress.com
6 Practice Set - 1
www.newspaperkorner.wordpress.com
65. If the viewership of ESPN on an average was 90000 per day, 71. What is the ratio of number of candidates failing the exam
than what was the viewership for all the sports channels on from City D to that of those failing the exam from City A?
an average per day? (a) 289 : 42 (b) 42 : 289
(a) 540000 (b) 600000 (c) 227 : 50 (d) 50 : 227
(c) 630000 (d) 720000 (e) None of these
(e) None of these 72. The number of candidates appearing for the exam from City
Directions (Qs. 66-70): These questions are based on the C is what per cent of the number of candidates appearing
following information. for the exam from City B? (rounded off to the nearest integer)
A total of 250 students of a class play different games viz (a) 27 (b) 34
football, hockey, chess, badminton, table-tennis and tennis. The (c) 42 (d) 21
ratio of girls to boys in the class of 250 is 13 : 12 respectively. 50% (e) 38
of the girls play table tennis and badminton only. 20% of the 73. The number of candidates passing in the exam from City F
boys play football, hockey and tennis only. 15% of the boys play is what per cent of the total number of candidates appearing
tennis and chess only. The ratio of number of girls to boys playing from all the cities together ? (rounded off to two digits after
tennis and chess only is 2 : 3 respectively. 30% of the girls plays the decimal).
hockey and chess only. 10% of the girls play chess, badmintonand (a) 12.93 (b) 14.46
table tennis only. The remaining girls play only football. Boys
(c) 10.84 (d) 11.37
playing table-tennis and badminton only is 20% of the girls playing
(e) None of these
the same. 40% of the boys play only football, The remaining
74. Which city has the highest number of students failing the
boys play only chess.
entrance exam?
66. What is the total number of students playing football?
(a) F (b) C
(a) 72 (b) 64
(c) B (d) D
(c) 68 (d) 73 (e) None of these
(e) None of these 75. What is the number of candidates passing the exam from
67. How many students play tennis? City E?
(a) 35 (b) 48 (a) 13,000 (b) 11,10,000
(c) 50 (d) 59 (c) 1,13,000 (d) 11,000
(e) None of these (e) None of these
68. How many girls do not play chess? 76. Pradip spends 40 per cent of his monthly income on food
(a) 66 (b) 62 items, and 50 per cent of the remaining on clothes and
(c) 58 (d) 55 conveyance. He saves one-third of the remaining amount
(e) None of these after spending on food, clothes and conveyance. If he saves
69. What is the ratio of number of girls playing hockey to the ` 19,200 every year, what is his monthly income?
number of boys playing hockey respectively? (a) ` 24,000 (b) ` 12,000
(a) 5 : 16 (b) 13 : 8 (c) ` 16,000 (d) ` 20,000
(c) 8 : 13 (d) 16 : 5 (e) None of these
(e) None of these 77. A man bought a horse and a carriage for ` 3000. He sold the
70. What is the total number of students playing badminton? horse at a gain of 20% and the carriage at a loss 10%, thereby
(a) 84 (b) 88 gaining 2% on the whole. Find the cost of the horse.
(c) 91 (d) 78 (a) ` 1000 (b) ` 1200
(e) None of these (c) ` 1500 (d) ` 1700
Directions (Q. 71-75): Study the tables carefully to answer the (e) None of these
questions that follow: 78. Two workers A and B working together completed a job in 5
Number of candidates (in lakhs) appearing in an entrance days. If A worked twice as efficiently as he actually did and
examination from six different cities and the ratio of candidates 1
passing and failing the same B worked as efficiently as he actually did, the work would
3
City A B C D E F have completed in 3 days. Find the time for A to complete
the job alone.
Number of
1.25 3.14 1.08 2.27 1.85 2.73 1 3
Candidates (a) 6 days (b) 5 days
4 4
Ratio of candidates passing and failing within the city. (c) 5 days (d) 3 days
(e) None of these
City Passing Failing 79. A man walks half of the journey at 4 km/h by cycle does one
A 7 3 third of journey at 12 km/h and rides the remainder journey
in a horse cart at 9 km/h, thus completing the whole journey
B 5 3
in 6 hours and 12 minutes. The length of the journey is
C 4 5
1332
D 1 3 (a) 36 km (b) km
67
E 3 2
(c) 40 km (d) 28 km
F 7 5 (e) None of these
www.newspaperkorner.wordpress.com
Practice Set - 1 7
www.newspaperkorner.wordpress.com
80. A bag contains 2 red, 3 green and 2 blue balls. 2 balls are to 89. What is sauce for the __________ is sauce for the
be drawn randomly. What is the probability that the balls __________
drawn contain no blue ball? (a) cock, hen (b) fox, vixen
5 10 (c) goose, gander (d) duck, doe
(a) (b) (e) None of these
7 21
90. Everyone was __________ by surprise when she
2 11 __________ her plan to marry that boy.
(c) (d) (a) moved, said (b) taken, announced
7 21
(e) None of these (c) shaken, declare (d) dead, asked
(e) None of these
ENGLISH LANGUAGE Directions (Qs. 91-100): In the following passage there are
blanks, each of which has been numbered. These numbers are
Directions (Qs. 81-85): Rearrange the following five sentences
printed below the passage and against each, five words are
(A), (B), (C), (D) and (E) in the proper sequence to form a suggested, one of which fits the blank appropriately. Find out the
meaningful paragraph; then answer the questions given below. appropriate word in each case.
A. Unfortunately, because of modernisation, much of nature Delinking of jobs 91 degrees is one of the salient 92 of our
is now yielding to towns, roads and industrial areas. education 93. There has been a steep 94 in 95 in the academic
B. In a few places, some natural reserves are now being carved field in recent years. There is a 96 of degree holders in the country.
out to avert the danger of destroying nature completely. As a result, university degrees have lost their value and charm
C. We should preserve nature to preserve life and beauty. while the number of students in colleges and universities of the
D. Man will perish without nature, so modern man should country has been steadily 97. Consequently, thousands of
continue this struggle to save plants from extinction. graduates and postgradutes come out of these institutions and
stand in 98 waiting to get some 99 jobs 100 in the country.
E A beautiful landscape, full of green vegetation, will not just
attract our attention but will fill us with infinite satisfaction. 91. (a) to (b) with
(c) from (d) by
81. Which of the following should be the FIRST sentence after
rearrangement? (e) None of these
92. (a) factors (b) features
(a) A (b) B (c) C (d) D (e) E
(c) reasons (d) methods
82. Which of the following should be the SECOND sentence
(e) None of these
after rearrangement?
93. (a) process (b) system
(a) A (b) B (c) C (d) D (e) E
(c) procedures (d) policy
83. Which of the following should be the THIRD sentence after
(e) None of these
rearrangement?
94. (a) fall (b) rise
(a) A (b) B (c) C (d) D (e) E
(c) down (d) decrease
84. Which of the following should be the FOURTH sentence (e) None of these
after rearrangement?
95. (a) evaluation (b) assessment
(a) A (b) B (c) C (d) D (e) E (c) result (d) competence
85. Which of the following should be the FIFTH sentence after (e) None of these
rearrangement? 96. (a) flood (b) growth
(a) A (b) B (c) C (d) D (e) E (c) increase (d) slope
Directions (Qs. 86-90): In each of the following sentences there (e) None of these
are two blank spaces. Below each sentence there are five pairs 97. (a) falling (b) diminishing
of words denoted by the numbers (a), (b),(c), (d) and (e). Find out (c) rising (d) growing
which pair of words can be filled up in the blanks to make the (e) None of these
sentence meaningfully complete.
98. (a) lines (b) long
86. To __________ time, please go __________ foot and not (c) rows (d) queues
by bus.
(e) None of these
(a) speed, with (b) kill, towards 99. (a) managerial (b) nice
(c) utilise, on (d) gain, on (c) prestigious (d) available
(e) None of these (e) None of these
87. There to __________ be an air of peace now __________ 100. (a) vacate (b) posted
in our country. (c) created (d) available
(a) seems, blowing (b) looks, flowing (e) None of these
(c) feels, advancing (d) seem, floating Directions (Qs. 101-111): Read the following passage carefully
(e) None of these and answer the questions given below it. Certain words/phrases
88. Satish reminded him __________ his promise but he was in the passage are printed in bold to help you locate them while
quite indifferent to __________ Satish said. answering some of the questions.
(a) of, what (b) for, when In a country where consumers have traditionally had a raw
(c) on, while (d) to, which deal, the Consumer Protection Act was one of the most
(e) None of these progressive acts of legislation introduced in 1986. Before this, a
www.newspaperkorner.wordpress.com
8 Practice Set - 1
www.newspaperkorner.wordpress.com
shop could get away easily with the line “goods once sold will (a) Only A and C (b) Only A and B
not be taken back or exchanged” or a car parking contractor with (c) Only B and C (d) Only B and C
“park at your own risk”. It is not that things have changed now (e) None of these
but at least a legislation is in place and a forum is available to seek 104. What does the author mean by ‘mystification of the Act’?
redressal . One of the basic limitations of this act is its mystification (a) The mysterious Act is yet to be resolved.
and general ignorance. No consumer agency or group has made (b) The consumer is wary of the Act.
its provisions general, nor has any redressal commission or (c) The Act is not easily accessible.
forum. Restricted as it is by a lack of in frastructure and personnel (d) The consumer remains unaware of his rights and
and great verdicts to encourage consumers. The legislation is privileges.
comprehensive. It gives consumers the right to redress against (e) The plight of the consumer is yet to end.
defective goods, deficient services and unfair trade practices. 105. Which of the following best describes the judge’s
Consumer courts must deliver their judgements within 40 days, replacement?
but rarely is this deadline adhered to. This reviewer had a first- (a) He was partial towards the airline as it was a foreign
hand experience of the chairman of a consumer court in Delhi one.
who adjourned a case against a foreign airline for two years on (b) He never bothered to safeguard the interests of the
the grounds that he did not have staff to type the orders. His reviewer.
replacement found the backlog so shocking that he dismissed (c) He dismissed cases without even giving a second
several cases without applying his mind, in the process working thought to what cases came to him.
against the interests of consumers. But what is more important is (d) He was apathetic and uninterested about the direction
that the law has it that a consumer can approach court on his own the case might head in.
without having to pay legal fees. In practice, this does not happen. (e) He passed irrelevant verdicts indifferently.
The chairperson of the National Commission, who is a sitting 106. What does the Act broadly cover?
judge, is so attuned to delivering judgments which can stand (a) It protects the right to redress.
scrutiny in a civil court of law that it is insisted upon that a (b) It is a forum that protects the redresser.
consumer must be represented by a lawyer. If not, cases are (c) It shields the consumer from deceptive and unfair trade
adjourned with impunity and set for another day. Girimaji’s attempt practices.
is creditable in that it is the first of its kind and has addressed (d) It enables the plaintiff to fight his case free of cost.
almost all possible angles. She has discussed redressals in (e) None of these
complaints about housing, basic telephony, rail transportation, 107. Which of the following is a limitation of the Act?
power supply, life insurance and medical negligence. There are (a) It does not cover the international law of torts.
even tips on how to file a complaint. But it is mired in the case (b) It is not comprehensive with regard to liberal economy.
files of the National/ State Commissions of the Consumer Forum. (c) No forum or commission has come forward to bring it
A useful dimension would have been a comparison with the Law to light.
of Torts practised abroad. It is necessary here also, especially in (d) Its red-tapism
an era of economic liberalisation, when the consumer is like ly to (e) None of these
be swept off his feet by free-market forces. Directions (Qs. 108-109): Choose the word which is most
101. Why is the consumer likely to be swept off his feet? SIMILAR in meaning to the word printed in bold as used in the
(a) He is easily taken in by the deceptive publicity. passage.
108. Forum
(b) He is wooed by the charm of foreign brands readily
(a) Dias (b) Podium
available in the market.
(c) Platform (d) Stage
(c) He is not aware of the Law of Torts as practised abroad. (e) None of these
(d) He is not aware of the benefits of the consumer rights. 109. Attuned
(e) The Consumer Protection Act has been implemented (a) Brought into harmony (b) Adjusted
and he can seek redressal. (c) Hazardous (d) Out of tune
102. What does ‘lack of... verdicts’ imply? (e) Malpractice
Directions (Qs. 110-111): Select the word which is most
(a) A lack of the basis of the system, trained staff and
OPPOSITE in meaning of the word printed in bold as used in the
decisions based on fact
passage.
(b) A paucity of funds, jury and judgement 110. Impunity
(c) A lack of resources, employees and final decision (a) Penalised (b) Fine
based on facts (c) Sentence
(d) Not having the required manpower, economy and (d) Freedom from punishment
decisive ruling (e) None of these
(e) None of these 111. Mired
(a) Buried (b) Muddy
103. Which of the following statements is/are true? (c) Steeped (d) Free
A. Girimaji’s attempt is comprehensive but could have (e) None of these
done with an angle or two more. Directions (Qs. 112-113) : Below is given a single word with
B. Though the Act allows the consumer to approach the options to its meaning in different contexts. You have to select
court on his own, yet a lawyer to represent him is all those options which are synonyms of the word when the
insisted upon. context is changed. Select the correct alternatives from (a), (b),
C. Despite the Act, much remains the same. (c), (d) & (e) which represent all those synonyms.
www.newspaperkorner.wordpress.com
Practice Set - 1 9
www.newspaperkorner.wordpress.com
112. HAMPER 120. Which of the following organisations / agencies frame the
(1) Prevent (2) A large basket Monetary and Credit Policy which is followed by all banks
(3) A gift (4) A laundry in India?
(a) 1, 2 (b) 2, 4 (c) 3, 1 (d) 1, 4 (e) 4, 3 (a) Indian Bank’s Association
113. DENT (b) Reserve Bank of India
(1) Tooth of Gearwheel (2) Sleaze (c) Securities and Exchange Board of India
(3) Depression (4) Penetration (d) Government of India
(a) 1, 4 (b) 2, 4 (c) 3, 1 (d) 1, 2 (e) 4, 3 (e) None of these
114. STRIKE 121. When the RBI wants to inject liquidity into economy, it may
(1) Military Attack (2) Discover adopt the following :
(3) Lockout (4) Snap (1) Buy the government securities from the banks.
(a) 1, 2, 4 (b) 2, 4 (2) Enter into reverse repo operations
(c) 3, 1 (d) 1, 4 (3) Raise cash Reserve Ratio
(e) 4, 3 (4) Reduce SLR.
115. GLANCE Select the correct answer using the codes given below:
(1) A quick look (2) Scan (a) 1 only (b) 1 and 4 only
(3) Review (4) Reflect (c) 1, 2 and 4 only (d) 1, 2, 3 and 4
(a) 1, 2, 3 (b) 1, 2 (e) None of these
(c) 2, 1, 4 (d) 2, 3, 4 122. Which of the following definitions are correct?
(e) All of (1), (2), (3), (4) (i) Basis points: increase in interest rates in percentage
terms.
GENERAL/ECONOMY/BANKING AWARENESS
(ii) Repo rate: rate at which commercial banks borrow from
116. Which of the following amendments in the Banking the RBI by selling their securities or financial assets to
Regulation Act 1949 has the Reserve Bank of India (RBI) the RBI for a long-period of time.
asked for finalising guidelines for new banking licences? (iii) Reverse repo rate: rate of interest at which the central
(a) Any acquisition of 5% or more of the share capital of bank borrows funds from other banks for a short
a bank should be subject to the Central Bank’s duration.
approval (iv) Cash reserve ratio: minimum percentage of cash
(b) RBI should have the authority to supersede the board deposits that banks must keep with itself to avoid
of directors of a banking company and appoint an liquidity issues.
administrator during the period of supersession (a) (i) & (ii) (b) (ii), (iii) & (iv)
(c) RBI should have the authority to call for information (c) (ii) & (iv) (d) (iii) & (iv)
or inspection of any associate enterprise of a banking (e) None of these
company. 123. The Bank Case Information System (BCIS) is being
(d) All the above developed by which of the following bodies, in the wake
(e) None of the above of increasing cases of bank frauds? It would contain names
117. Which of the following activities can lead to financial of accused persons besides the details of borrowers and
inclusion in India? the public servants concerned.
(1) Issuing of general purpose credit cards. (a) Reserve Bank of India
(2) Strict know your customer’ (KYC) norms (b) Indian Bank’s Association
(3) Opening of Bank branches in unbanked rural areas. (c) Ministry of Finance
(4) Opening of no-frills account. (d) Central Statistical Organisation
Select the correct answer using the codes given below : (e) Central Bureau of Investigation
(a) 1, 2 and 3 only (b) 2, 3 and 4 only 124. Which of the following terms is NOT a financial term?
(c) 1, 3 and 4 only (d) 2 and 3 only (a) Investment (b) El Nino effect
(e) None of these (c) Core banking Solution (d) RTGS
118. Consider the following statements : (e) All are financial terms
(1) Fiat money is a term used for Gold coins 125. Which of the following is NOT a public sector bank?
(2) Currency Deposit Ratio is the proportion of the total (a) Corporation Bank (b) United Bank of India
deposits commercial banks keep as reserves. (c) Vijaya Bank (d) Bank of Maharashtra
Which of the above statements is/are correct? (e) Axis Bank
(a) 1 only (b) 2 only 126. Core Banking Solution(CBS) provides:
(c) Both 1 and 2 (d) Neither 1 nor 2 1. multiple delivery channels to the customers.
(e) None of these 2. better MIS and reporting to external agencies such as
119. As we know, the Government is paying much attention to government, RBI, etc.
“Micro Finance” these days. Which of the following is 3. better asset liability management and risk management
one of the examples of Micro Finance? by banks.
(a) Insurance for life Select the answer from the codes given below:
(b) Investment in Mutual Funds Codes:
(c) Self Help Groups (a) 1 and 2 (b) 2 and 3
(d) Letter of Credit (c) 1 and 3 (d) All of the above
(e) All of these (e) None of these
www.newspaperkorner.wordpress.com
10 Practice Set - 1
www.newspaperkorner.wordpress.com 138. Jnanpith Award is given for excellence in the field of
127. A worldwide financial messaging network which exchanges
messages between banks and financial institutions is (a) Music (b) Social Service
known as __________ . (c) Literature (d) Classical Dance
(a) CHAPS (b) SWIFT (e) None of these
(c) NEFT (d) SFMS 139. Which of the following is NOT a public sector bank?
(e) CHIPS (a) Corporation Bank (b) United Bank of India
128. The term 'Smart Money" refers to __________ . (c) Vijaya Bank (d) Bank of Maharashtra
(a) Foreign Currency (b) Internet Banking (e) Axis Bank
(c) US Dollars (d) Travelers' cheques 140. Who is the author of book Durbar?
(e) Credit Cards (a) Amitav Ghosh (b) Tavleen Singh
129. Which one of the following is a retail banking product ? (c) Amit Chaudhuri (d) Arundhati Roy
(a) Home Loans (b) Working capital finance (e) None of these
(c) Corporate term loans (d) Infrastructure financing 141. RBI was established in 1935 pursuant to the recommendations
(e) Export Credit of
130. Which of the following books has been written by Bill Gates? (a) The Hilton Young Commission
(a) Microsoft Secrets (b) All India Rural Credit Survey Committee
(b) The Road Ahead (c) Gorawala Committee
(c) The Elephant Paradigm (d) Talwar Committee
(d) How Nations Compete
(e) None of these
(e) None of these
142. Which of the following does not come in the purview of
131. The expansion for the BIS, in the context of the banking
retail banking?
industry is
(a) Advances to traders
(a) Bank for International Settlements
(b) Advances of small scale units
(b) Bank for Industrial Settlements
(c) Bank for Industrial Sectors (c) Financing of weaker sections of the society
(d) Bank for International Services (d) Loan syndicate
(e) None of these (e) None of these
132. In 1955, Imperial Bank of India, a leading commercial bank 143. Insurance Business can be divided into
of that time, was nationalised and renamed as (a) Life Insurance (b) General Insurance
(a) Central Bank of India (b) Bank of India (c) Fire Insurance (d) Both (1) and (2)
(c) Indian Bank (d) State Bank of India (e) None of these
(e) None of these 144. Indian Financial System comprises of
133. What is the full form of ‘FINO’, a term we see frequently in (a) Scheduled Commercial Banks
financial newspapers? (b) Non-Banking Financial Institutions
(a) Financial Investment Network and Operations (c) Urban Co-operative Banks
(b) Farmers’ Investment in National Organisation (d) All of the above
(c) Farmers’ Inclusion News and Operations (e) None of these
(d) Financial Inclusion Network and Operations 145. None-Banking Financial Institutions comprise of
(e) None of these (a) Development Finance Institutions (DFIs)
134. Which of the following terms is NOT a financial term? (b) Insurance Companies
(a) Investment (b) El Nino effect (c) Non-Banking Financial Companies (NBFCs), Primary
(c) Core banking Solution(d) RTGS Dealers (PDs) and capital market intermediaries e.g.,
(e) All are financial terms mutual funds.
135. Which of the following rates is/are NOT decided by RBI? (d) All of the above
(a) Bank Rate (b) Repo rate (e) None of these
(c) Prime Lending Rate (d) Cash Reserve Ratio 146. Which of the following is a Term Lending institution?
(e) None of these (a) EXIM Bank (b) IDBI Bank Ltd
136. Which of the following is not a tax/duty levied by the (c) ICICI Bank Ltd (d) All of these
Government of India?
(e) None of these
(a) Service Tax (b) Education Cess
147. Which of the following is an investment institution?
(c) Customs Duty (d) Toll Tax
(a) Life Insurance Corporation of India
(e) None of these
137. Who is the first woman to win Nobel prize? (b) IDBI Bank Ltd
(a) Marie Curie (b) Pearls Buck (c) ICICI Bank Ltd
(c) Bertha Von Suffner (d) Mother Teresa (d) State Bank of India
(e) None of these (e) None of these
www.newspaperkorner.wordpress.com
Practice Set - 1 11
www.newspaperkorner.wordpress.com
148. The commerical banking system in India consists of 152. The break-even point is where-
(a) Nationalised Banks and Private Sector Banks (a) Average revenue equals average cost
(b) Scheduled and Non-scheduled Banks (b) Marginal revenue equals mariginals cost
(c) Regional Rural Banks, Co-operative Banks and Land (c) Total Revenue equals total cost
Development Banks (d) Mariginal revenue equals total cost
(d) All of the above (e) None of these
(e) None of these 153. A short-term government security paper is called?
149. Which of the following is known as the central Bank of the
(a) Treasury Bill (b) Share
country?
(c) Debenture (d) Mutual Fund
(a) Central Bank of India (b) Reserve Bank of India
(e) None of these
(c) IDBI (d) IBRD
(e) SBI 154. The Industrial Finance Corporation of India was set up in
_____?
150. Accounts are allowed to be operated by cheques in respect
of:- (a) March 1948 (b) April 1948
(a) both saving and fixed accounts (c) July 1948 (d) October 1948
(b) saving and current accounts. (e) None of the above
(c) current and fixed accounts. 155. The Industrial Finance Corporation of India provides loans
(d) all type of accounts available. to _____?
(e) None of these (a) Industries in public sector only
151. What does the letter 'M' denote in the name 'MFI' as used (b) Industries set up for export promotion
in financial sector? (c) Joint-stock companies in the public or private or joint
(a) Mega (b) Micro sector or co-operative sector
(c) Mutiple (d) Monetary (d) All of the above
(e) None of these (e) None of these

www.newspaperkorner.wordpress.com
12 Practice Set - 1
www.newspaperkorner.wordpress.com

Time : 30 min. Max. Marks : 50

1. Write a letter on any one of the following:- (20 marks)


(a) Write a letter to the manager of your bank asking for an account statement.
(b) Write a Letter to the bank informing a bank that you have lost your credit card or ATM card need a replacement.
(c) Write a letter to the Bank Requesting an Application for a new mortgage loan.
2. Write a paragraph on one of the following topic in about 150 words : (10 marks)
(a) Dominance of Information Technology
(b) Role of Newspapers
(c) Banning the plastic bags
3. Write an essay on one of the following topic in about 250 words. (20 marks)
(a) Pollution in developed country
(b) Honour killings in India
(c) Sexual Exploitation of Women

www.newspaperkorner.wordpress.com
Practice Set - 1 13
www.newspaperkorner.wordpress.com
A nsw e r Ke y
1 (b) 21 (b) 41 (d) 61 (b) 81 (c ) 101 (b) 121 (b) 141 (a )
2 (c ) 22 (e) 42 (d) 62 (a ) 82 (e) 102 (b) 122 (c ) 142 (d)
3 (e) 23 (b) 43 (d) 63 (c ) 83 (a ) 103 (d) 123 (e) 143 (d)
4 (b) 24 (d) 44 (d) 64 (c ) 84 (b) 104 (d) 124 (b) 144 (d)
5 (a ) 25 (d) 45 (a ) 65 (d ) 85 (d) 105 (c ) 125 (e) 145 (d)
6 (e) 26 (e) 46 (e) 66 (d ) 86 (e) 106 (c ) 126 (d) 146 (a )
7 (d) 27 (d) 47 (e) 67 (e) 87 (e) 107 (e) 127 (b) 147 (a )
8 (e) 28 (e) 48 (b) 68 (a ) 88 (a ) 108 (c ) 128 (e) 148 (b)
9 (c ) 29 (c ) 49 (d) 69 (b ) 89 (c ) 109 (b) 129 (b) 149 (b)
10 (e) 30 (c ) 50 (d) 70 (c ) 90 (b) 110 (a ) 130 (b) 150 (b)
11 (b) 31 (a ) 51 (c) 71 (c ) 91 (c ) 111 (d) 131 (a ) 151 (b)
12 (d) 32 (a ) 52 (d) 72 (b ) 92 (b) 112 (a ) 132 (d) 152 (c )
13 (d) 33 (c ) 53 (e) 73 (a ) 93 (b) 113 (a ) 133 (d) 153 (a )
14 (e) 34 (e) 54 (c) 74 (d ) 94 (a ) 114 (a ) 134 (c ) 154 (c )
15 (a ) 35 (a ) 55 (a ) 75 (e) 95 (d) 115 (c ) 135 (c ) 155 (c )
16 (e) 36 (d) 56 (d) 76 (c ) 96 (a ) 116 (d) 136 (d)
17 (c ) 37 (c ) 57 (a ) 77 (b ) 97 (c ) 117 (c ) 137 (a )
18 (a ) 38 (d) 58 (b) 78 (a ) 98 (d) 118 (d) 138 (c )
19 (d) 39 (b) 59 (e) 79 (a ) 99 (c ) 119 (c ) 139 (e)
20 (d) 40 (a ) 60 (c) 80 (b ) 100 (d) 120 (b) 140 (b)

Answers & Explanations


P T Q V S R occupies the second position from the left, pushing the
(2-7) : rest of the line rightward. Thus the numbers and words get
Row 1 arranged alternately till all the numbers are placed in the
descending order and the words in the alphabetical order.
13. (d) Input: organize 19 12 stable room 35 72 house
Step I: 72 organize 19 12 stable room 35 houses
C D E F A B Row 2 Step II: 72 house 35 organize 19 12 stable room 35
8. (e) From I, we get BED as a sequence. Now, II tells us that Step III: 72 house 35 organize 19 12 stable room
B is at the extreme left. Clearly then, D is third from left, Step IV: 72 house organize 19 room 12 stable
i.e. in the middle of the five. 14. (e) Input: always go there 39 62 47 time 24
9. (c) From I: Sushma’s husband is Nandini’s brother (only Step I: 62 always go there 39 47 time 24
son of mother) Sushma is Nandini’s sister-in-law. Step II: 62 always 47 go there 39 time 24
From II: Sushma’s brother = cousin of Nandini’s Step III: 62 always 47 go 39 there time 24
husband Step IV: 62 always 47 go 39 there 24 time
Þ Sushma = cousin of Nandini’s husband Hence step III will be the last but one.
= Nandini’s cousin-in-law 15. (a) Step III: 86 box 63 18 gear card 51 new
10. (e) From I: A, F > B > C, D, E Step IV: 86 box 63 card 18 gear 51 new
Either A or F has secured maximum marks. Step V: 86 box 63 card 51 18 gear new
From II: A > F > B Step VI: 86 box 63 card 51 gear 81 new
From I and II, A secured the maximum marks. Hence 6 – 3 = 3 more steps will be required.
11. (b) From (I), B > C > A 16. (e) Step IV: 59 bend 46 card 14 27 win now
D is not frosent here, so no conclusion. Step V: 59 bend 46 card 27 14 win now
isom (II), Step VI: 59 bend 46 card 27 now 14 win
B> C> D >A Since the line is already arranged, there will be no step
So, A is not likely to win. VII.
12. (d) We do not know whether the codes of the given words
are in the same order as the order of the words. (17-23) : Peter Ali
Ketan
Therefore, statement I alone is not sufficient. Again,
statement II does not consist of the word ‘go’. Hence,
statement 11 alone is not sufficient. Even statements I
Rumia Shikha
and II together give the code for ‘may’ only.
(13-16) : In the first step, the largest number goes to the leftmost
position, pushing the rest of the line rightward. In the next
step, the word that comes first in the alphabetical order Harleen
Bharat Meenal
www.newspaperkorner.wordpress.com
14 Practice Set - 1
www.newspaperkorner.wordpress.com
17. (c) All others sit in the middle of the sides. 52. (d) Here, the percentage profits of Companies B and C in
(23-24) : All buildings are houses + No house is an apartment = 2001 were not the same. Therefore, can’t be determined
A + E = E = No building is an apartment (i). Again, No house is the correct choice.
is an apartment + All apartments are flats = E + A = O* = 53. (e) Income of Company C in 2014
Some flats are not house (ii). Again, No building is an 140
apartment + All apartments are flats = E + A = O* = Some = 32 ´ = ` 44.80 lakhs
flats are not buildings (iii) 100
23. (b) Conclusion (I) above is the conclusion II. 54. (c) Reqd ratio = 145:155 = 29 : 31
24. (d) None follows 55. (a) Expenditure of Company D in 2014
(25-28) : Some oceans are seas (I) ® conversion ® Some seas are 100
oceans (I) + All oceans are rivers = I + A = I = Some seas are = 31 ´ = ` 20 lakhs
155
rivers (i). Again, All ocean are rivers + No river is a canal = A +
E = E = No oceans is a canal (ii). Again, Some seas are rivers + Profit = Income – Expenditure
No river is a canal = I + E = O* = Some canals are not seas (iii). = 31 – 20 = ` 11 lakhs
25. (d) All rivers can never be oceans ® implication ® Some 56. (d) Difference of production of C in 2010 and A in 2015
rivers are oceans. This conclusion is the converse of = 5,00,000 tonnes
the given premise “All oceans are rivers.” 57. (a) Percentage increase of A from 2011 to 2012
26. (e) Conclusion II is the above conclusion (ii). Conclusion 55 - 40
I is the above conclusion (i). ´ 100 = 37.5%
40
(27-28) : No day is night + All nights are non = E + A = O* = Some 58. (b) Percentage rise/fall in production for B
noon are not days (i). Again, All nights are noon + No
noon is an evening = A + E = E = No night is an evening (ii). 2011 2012 2013 2014 2015
27. (d) None follows. 9% –16.6% 10% – 9% 10%
28. (e) Conclusion I is converse of the above conclusion (ii).
Here, the maximum difference is from 2011 to 2012,
29. (c) Whenever such a decision is taken, the assumptions
which is 10. And the second nearest to it is fall or rise
are that it would be welcome and allowed to implement.
of 5. So, undoubtedly the answer is 2012.
30. (c) The urging of the govt makes sense only when (A)
and (B) are implicit. 120
59. (e) Percentage production = ´ 100 = 133.3%
31. (a) From problem figure (4) to (5) the lower design is 90
reversed laterally while the other design moves to the 60. (c) Average production of A = 50
opposite side. Similar changes would occur from Average production of B = 54.17
problem figure (2) to (3). Average production of C = 50
32. (a) Symbol changes positions as shown in both the Difference of production = 54.17 – 50 = 4.17
diagrams alternately and symbols in place of the sign
‘·’ remain unchanged in each of the successive figure. (150 - 15°) ´ 60000
61. (b) No. of equidistant =
30°
· · ·
135
= ´ 60000 = 270000
· · 30
Figure (1) to (2) Figure (2) 20
62. (a) Angle showing 20% viewership = ´ 360 = 72°
33. (c) Each symbol of the figure rotates in Anticlockwise and 100
a new symbol replaces the symbol at the top which is From pie-chart, it is clear that only Set Max has shown
inverted alternatively in each of the successive figures. angle more than 72°
34. (e) From problem figure (1) to (2), first and second figure 60
interchange their position and are reversed at the new 63. (c) Required ratio = ´ 2 : 45 = 40 : 45 = 8 : 9
3
positions and at the same time, third figure remains 64. (c) It is given 80% of actual value of DD sports = 60°
unchanged. From problem figure (2) to (3), second and Hence, actual value of DD sports = 75°
third figures interchange their positions and are
reversed at the new position and the figure (1) remain 150
unchanged. The same problem is repeated alternatively. Hence, % share of Set Max = 360 + ( 75 - 60 ) ´ 100
35. (a) In each step, both the line segments close to the sides
of the hexagon, move to the adjacent sides in a 150
= ´ 100 = 40%
Clockwise. Also the line segments at the corner move 375
to the adjacent corner Clockwise and their number 360
increases by one, in the first, third, fifth steps. 65. (d) Required viewership = ´ 90000 = 720000
46. (e) The series is × l + 1, × 2 + 2, × 3 + 3, ... So 8 is wrong. 45
47. (e) The series is × l + 12, × 2 + 22, × 3 + 32 .... (66 to 70): Total = 250 students
48. (b) The series is × 2 + 9, × 2 + 11, × 2 + 13 .... 250 ´ 13 250 ´ 12
49. (d) The series is × 1 – 1, × 2 + 2, × 2 – 2, × 3 + 3, .... Girls = = 130; Boys = = 120
13 + 12 13 + 12
50. (d) The series is × 3 + 1, × 4 + 1, × 5 + 1, .... 50% girls play only table-tennis and badminton =
51. (c) Expenditure of Company C in 2014 130 ´ 50
100 = 65
= 35 ´ = ` 25 lakhs 100
140 www.newspaperkorner.wordpress.com
Practice Set - 1 15
www.newspaperkorner.wordpress.com 75. (e) Number of passed students from City E
130 ´ 30
30% Girls play hockey and chess only = = 39
100 æ3 ö
= ç ´1.85 ÷ lakhs = 111000.
10% Girls play chess, badminton and table tennis only è 5 ø
130 ´ 10 76. (c) Food items = 40%
= = 13
100 1
Clothes + conveyance = of 60% = 30%
18 2
Girls play tennis and chess only = ´ 2 = 12
3 1 19, 200
of 30% = Þ 10% = 1600
Girls play football only = 130 – (65 + 39 + 13 + 12) = 1 3 12
20% of boys play football, hockey and tennis only \ 100% = ` 16,000
120 ´ 20 77. (b) Let the C.P. of horse = ` x
= = 24 Then the C.P. of carriage = ` (3000 – x)
100
20% of x – 10% of (3000 – x) = 2% of 3000
120 ´ 15
15% of boys play tennis and chess only = = 18 x (3000 – x)
100 Þ - = 60
65 ´ 20 5 10
Boys playing table tennis and badminton = = 13 Þ 2x - 3000 + x = 600
100
Þ 3x = 3600 Þ x = 1200
120 ´ 40
40% of boys play only football = = 48 1
100 78. (a) (A + B)’s one day’s work = th work
Boys play only chess = 120 – (24+ 18 + 13 + 48) = 17 5
66. (d) Number of students playing football = 48 + 24 + 1 = 73 Let A can do job in x days. Then,
67. (e) Number of students playing tennis 1
= 65 + 13 + 12 + 24 + 18 + 13 = 145 A’s one day’s work = th work
x
68. (a) Number of girls playing chess = 12 + 39 + 13 = 64
Number of girls not playing chess = 130 – 64 = 66 1 1 x -5
69. (b) Required ratio = 39 : 24 = 13 : 8 and B’s one day’s work = – = th work
5 x 5x
70. (c) Number of students playing badminton
= 65 + 13 +13 = 91 æ 1ö 1
Now , (2)A 's work + ç ÷ B's work = rd work
3 3
è 3ø 3
71 (c) Required ratio = ´ 2.27 : ´ 1.25
4 10 2 1æ x -5ö 1 25 1
Þ + ç ÷= Þx= = 6 days
= 1.7025 : 0.375 = 227 : 50 x 3 è 5x ø 3 4 4
1.08 79. (a) Let the length of the journey =x km.
72. (b) Required percentage = ´ 100 » 34
3.14
\ Journey rides by horse cart = x æç1 - - ö÷
1 1
73. (a) Total number of candidates appearing from all the cities
è 2 3ø
together
= (1.25 + 3.14 + 1.08 + 2.27 + 1.85 + 2.73) lakhs 1
= x km.
= 12.32 lakhs 6
Number of candidates passing from City F
31
7 Then, total time taken to complete journey = hr
= ´ 2.73 = 1.5925 lakh 5
12
1.5925 31
´ 100 = 12.93 Þ t1 + t 2 + t 3 =
Required percentage = 5
12.32
74. (d) Number of failures: x 1 x 1 x 31
Þ ´ + ´ + =
3 2 4 3 12 6 ´ 9 5
City A ® × 1025 lakhs = 0.375 lakh
10 31 216
3 Þx= ´ = 36.2km » 36km
City B ® × 3.14 lakhs = 1.1775 lakh 5 37
8
5
5 C2 5 ´ 4 10
City C ® × 1.08 lakhs = 0.6 lakh 80. (b) Reqd probability = = =
9
7
C2 7 ´ 6 21
3 112. (a) Meaning of Hamper -
City D ® × 2.27 lakh = 1.7025 lakh To prevent the free movement, action, or progress of
4
A large basket, usually with a cover
2 • New policy of tax shifting can hamper the bubbling
City E ® × 1.85 lakhs = 0.74 lakh
5 industrial sector.
5 • His boyfriend sent her a big gift hamper on her
City F ® × 2.73 lakh = 0.455 lakh birthday.
12
www.newspaperkorner.wordpress.com
16 Practice Set - 1
www.newspaperkorner.wordpress.com
113. (a) Meaning of Dent- 118. (d) Currency notes and coins are called fiat money. They
1. A depression in a surface made by pressure or a don’t have intrinsic value like a gold or silver coin.
blow: The currency-deposit ratio measures the relationship
• A dent in the side of a car. between the cash people have on hand and what they
2. A significant, usually diminishing effect or have in their accounts.
impression: The loss put a dent in the team's 121. (b) By Buying the government securities from the banks
confidence. and reducing SLR may inject money into the system.
• Irresponsible statement like this has dented However Raising cash Reserve Ratio may take away
his image. liquidity from the market as the banks will have to
3. Informal Meaningful progress; headway deposit more money with the RBI and similarly entering
• He has not made a dent in his carrier for last the reverse repo operations may also take away
some years. liquidity from the market.
114. (a) Meaning of Strike - 122. (c) Basis points: It is the increase in interest rates in
1. To hit sharply, as with the hand, the fist, or a percentage terms. For instance, if the interest rate
weapon. increases by 50 basis points (bsp), then it means that
To inflict (a blow). interest rate has been increase by 50%. One percentage
• US drone fighter planes strike the terrorist point is broken down into 100 basis points. Therefore,
airbase in Pakistan in stillness of night. an increase from 2% to 3% is an increase of one
2. To penetrate or pierce
percentage point or 100 basis points.
• Nail got fixed by striking with hammer.
Repo rate: Repo rate is the policy rate and is part of
3. (a) To collide with or crash into
RBI’s Liquidity Adjustment Facility (LAF). It is the
(b) To cause to come into violent or forceful
rate at which commercial banks borrow from the RBI
contact
by selling their securities or financial assets to the RBI
• Speeding bus strikes the car coming from
for a short-period of time. It comes with an agreement
the wrong side of road with such a force
that the sold securities will be repurchased by the
that a deafening sound can be heard from
commercial banks from the RBI at a future date at
a kilometer distance.
predetermined price. The repo rate is used by the central
(d) To damage or destroy, as by forceful contact
4. To make a military attack on; assault. bank to increase liquidity in the system. Reverse repo
5. To afflict suddenly, as with a disease or impairment rate: Reverse Repo Rate is also a part of LAF. It is the
6. To cause to become by or as if by a blow rate of interest at which the central bank borrows funds
7. (a) To snap at or seize (a bait). from other banks for a short duration. The banks
• Their 10 years long marriage strike with deposit their short term excess funds with the central
not much ado. bank and earn interest on it. This rate is used by the
(b) To hook (a fish that has taken the bait) by a central bank to absorb liquidity from the economy.
pull on the line Generally it is one percentage less than the Repo rate.
115. (c) Meaning of GLANCE - Bank rate: The only way the bank rate is different from
1. (a) To direct the gaze briefly the repo rate is that the bank rate is the rate at which
(b) To move rapidly from one thing to another. banks borrow money from the central bank without
Used of the eyes. any sale of securities. It is generally for a longer period
2. To shine briefly; glint. of time. Cash reserve ratio: CRR is the minimum
3. To strike a surface at such an angle as to be percentage of cash deposits that banks must keep with
deflected the central bank. The current rate is 4%, which means
4. To make a passing reference; touch briefly for a cash deposit of Rs. 100, the bank has to park 4
Usage - rupee with the central bank.
• He glanced through the report and nodded in 126. (d) The CBS also provides quick and safe transfer of
positive. funds, low transaction cost, and anytime anywhere
• It glanced in the sky for a brief period and then banking facilities. Nowadays, most banks use core
went missing in the night sky. banking applications to support their operations where
• His casual glances will not help the issue. CORE Banking stands for “centralized online real-time
117. (c) Financial inclusion or inclusive financing is the electronic banking”. This basically means that all the
delivery of financial services at affordable costs to bank’s branches access applications from centralized
sections of disadvantaged and low-income segments datacenters. This means that the deposits made are
of society. For financial inclusion ‘Know your reflected immediately on the bank’s servers and the
customer’ norms should be relaxed and no frills account customer can withdraw the deposited money from any
should be opened for low income segments which are of the bank’s branches throughout the world. These
looking for basic banking only . Along with general applications now also have the capability to address
purpose credit cards should be issued and bank the needs of corporate customers, providing a
branches should be opened in unbanked rural areas. comprehensive banking solution.
www.newspaperkorner.wordpress.com
www.newspaperkorner.wordpress.com

Practice Set - 2

INSTRUCTIONS
• This practice set consists two parts. One part is Objective test and other part is Descriptive test.

• Each question has five options, of which only one is correct. The candidates are advised to read all the
options thoroughly.

• There is negative marking equivalent to 1/4th of the mark allotted to the specific question for wrong
answer.

Time : 3 hrs. Max. Marks : 200

REASONING AND COMPUTER APTITUDE 3. Statements: All planets are stars.


All stars are asteroids.
Directions (Qs. 1-5): In each of the questions below are given All asteroids are moons.
four statements followed by four conclusions numbered I, II, III Some moons are rocks.
and IV. You have to take the given statements to be true even if Conclusions: I. All asteroids are planets.
they seem to be at variance with commonly known facts. Read all II.
All asteroids are stars.
the conclusions and then decide which of the given conclusions III.
All moons are stars.
logically follows from the given statements disregarding IV.
Some rocks are stars.
commonly known facts. (a) None follows
(b) Only I follows
1. Statements: All snakes are eagles. (c) Only II follows
Some eagles are rabbits. (d) Only either I or II follows
All rabbits are birds. (e) None of these
Some birds are animals. 4. Statements: Some bats are toys.
Conclusions: I. Some animals are snakes. Some toys are plastics.
II. Some birds are snakes. Some plastics are mirrors.
III. Some birds are eagles. No mirror is glass.
IV. All birds are rabbits. Conclusions: I. Some toys are mirrors.
(a) None follows (b) Only II follows II. Some plastics are glasses.
III. Some bats are mirrors.
(c) Only III follows (d) Both II and III follow
IV. No glass is plastic.
(e) None of these
(a) Only III follows
2. Statements: Some cameras are calculators. (b) Only either II or IV follows
Some calculators are diaries. (c) Only either I or III follows
All notebooks are diaries. (d) Only either III or IV follows
All diaries are computers. (e) None of these
Conclusions: I. Some notebooks are calculators 5. Statements: All graduates are advocates
II. Some calculators are computers. Some advocates are judges.
III. All notebooks are computers. All judges are lawyers.
IV. Some diaries are cameras. Some lawyers are doctors.
(a) None follows (b) Only II follows Conclusions: I. Some doctors are advocates.
(c) Only III follows (d) Both II and III follows II. All graduates are judges.
(e) None of these III. Some doctors are graduates.
IV. Some lawyers are advocates.
www.newspaperkorner.wordpress.com
18 Practice Set - 2
www.newspaperkorner.wordpress.com
(a) None follows 10. Statements: W Ó T, T d N, N%D
(b) Only I follows Conclusions: I. D é T
(c) Only II follows II. W Ó N
(d) Either III or IV follows III. D @ T
(e) None of these
(a) None is true
Directions: (Qs. 6-10): In the following questions, the symbols
é, d, %, @ and Ó are used with the following meaning illustrated (b) Only I is true
below: (c) Only II is true
‘P%Q’ means ‘P is not smaller than Q’. (d) Only III is true
‘PÓQ’ means ‘P is neither smaller than nor equal to Q’. (e) Only I and II are true
‘PéQ’ means ‘P is neither greater than nor equal to Q’. Directions(Qs. 11-14): Each question given below is followed by
‘PdQ’ means ‘P is not greater than Q’. five statements numbered I, II, III, IV and V. The answer choice
given below each question consists of one or more statements.
‘P @ Q’ means ‘P is neither greater than nor smaller than Q’.
You have to choose the choice which gives more relevant / useful
Now, in each of the following questions, assuming the given information in answering the question correctly. Read all the
statements to be true, find which of the three conclusions I, II statements together with the question and choose your answer.
and III given below them is/are definitely true and give your
11. For what reason Purohit did not get th e offer of
answer accordingly.
employment?
6. Statements: R d K, K é M, M@J
Statement :
Conclusions: I. J Ó K
I. Purohit passed in the interview.
II. M Ó R
II. Purohit's friend passed the medical test who passed
III. R éJ the interview along with Purohit.
(a) Only I and II are true III. Purohit's father did not want him to take the job.
(b) Only II and III are true IV. Purohit has another employment offer from another
(c) Only I and III are true company.
(d) All I, II and III are true V. Purohit did not clear the mandatory medical test.
(e) None of these (a) III and IV only (b) III, IV and V only
7. Statements: Z @ M, M Ó K, K éF (c) I, III and IV only (d) V only
Conclusions: I. F Ó Z (e) None of these
II. K é Z 12. What were the possible reasons for incurring losses by
III. F Ó M DESCO for the last two years?
(a) None is true Statement :
(b) Only I is true I. The company's shares are not registered in the stock
(c) Only II is true exchange.
(d) Only III is true II. The company does not export its products.
(e) Only II and III are true III. The company has inefficient labour force.
8. Statements: B é J, J % W, WÓM IV. The price of its product has fallen in the last two years
Conclusions: I. M é J due to competitive market.
II. W é B V. Entry of similar foreign goods at a cheaper rate.
III. B Ó M (a) Only III, IV and V (b) Only II, III and IV
(a) None is true (c) Only IV and V (d) Only I and II
(b) Only I is true (e) None of these
(c) Only II is true 13. On which day of the week did Sunil get his letter of
promotion?
(d) Only III is true
Statement :
(e) Only I and III are true
I. Sunil purchased a new shirt on Friday
9. Statements: V % H, H @ F, FdE
II. Sunil was given a party that Saturday.
Conclusions: I. F @ V
III. Sunil was given the letter of promotion on the day
II. F é V
before he purchased the shirt.
III. E % H
IV. Tuesday being his birthday, Sunil gave a party to all
(a) Only either I or II is true
his friends.
(b) Only III is true
V. Sunil's friend was promoted on Friday.
(c) Only I and II are true
(a) I and II only (b) II, III and IV only
(d) All I, II and III are true (c) I and III only (d) II, III and V only
(e) Only either I or II and III are true (e) None of these
www.newspaperkorner.wordpress.com
Practice Set - 2 19
www.newspaperkorner.wordpress.com
14. Who among A, B, C, D and E is the heaviest? 18. If the input is ‘true se veto be nuke my like’,
Statement : which of the following will be the IV step?
I. B and C are heavier than A and D. (a) like nuke true veto be se my
II. C is heavier than D. (b) be my like se true veto nuke
III. C is heavier than A and lighter than B. (c) be my se like true veto nuke
IV. E is heavier than B (d) veto true nuke like so be my
V. D is lighter than E. (e) Cannot be determined
(a) I, III and IV only (b) II, III and V only
19. Input: ‘more fight cats cough sough acts idea’.
(c) III and IV only (d) I, III and V only
(e) None of these Which of the following steps would be the last step for this
15. Many companies now have employee assistance input?
programmes that enable employees, free of charge, to (a) III (b) IV
improve their physical fitness, reduce stress, and learn ways (c) V (d) VI
to stop smoking. These programmes increase worker (e) VII
productivity, reduce absenteeism, and lessen insurance 20. If the V step of an input is ‘more pure soft cat not so sir at’,
costs for employee health care. Therefore, these
what will be the Il step?
programmes benefit the company as well as the employee.
Which of the following, if true, most significantly (a) at so more pure cat not soft sir
strengthens the conclusion above? (b) more pure soft so sir cat at not
(a) Physical fitness programmes are often the most (c) more pure soft cat so sir at not
popular services offered to employees. (d) more so sir soft pure cat at not
(b) Studies have shown that train ing in stress (e) Cannot be determined
management is not effective for many people. Directions (Q. 21-25) : Study the following information and
(c) Regular exercise reduces people’s risk of heart disease answer the questions given below it.
and provides them with increased energy.
Seven members H, I, J, K, L, M and N are working in different
(d) Physical injuries sometimes result from entering a cities Ahmedabad, Bangalore, Chennai, Hyderabad, Kolkata, Delhi
strenuous physical fitness programme too quickly.
and Mumbai, not necessarily in the same order. Each one has a
(e) None of these
different mother tongue–Tamil, Kannada, Telugu, Hindi, Marathi,
Directions (Qs. 16-20) : Study the following information to answer Punjabi and Bangla, not necessarily in the same order.
the given questions.
J works in Bangalore and his mother tongue is not Tamil or
A word arrangement machine when given an input line of words,
Marathi. K’s mother tongue is Punjabi and he works in Ahmedabad.
rearranges them following a particular rule in each step. The
L and M do not work in Chennai and none of them has Marathi
following is an illustration of the input and the steps of
rearrangement. mother tongue. I works in Hyderabad and his mother tongue is
Input : going but for crept te light sir Telugu. The one who works in Delhi had Bangla mother tongue.
Step I : crept going but for te light sir N works in Mumbai and his mother tongue is Hindi. L does not
work in Kolkata.
Step II : crept going light but for te sir
21. What is J’s mother tongue?
Step III : crept going light but for sir te
(a) Telugu (b) Hindi
(Step III is the last step for this input)
(c) Bangla (d) Kannada
As per the rules followed in the above steps, find out in the given
(e) None of these
questions the appropriate step for the given input.
22. Who works in Chennai?
16. Input: the in car as he may me
(a) H (b) L
Which of the following will be the third step for this input?
(c) M (d) L or M
(a) car the in as he may me
(e) None of these
(b) car may the as in he me
23. Which of the following combinations is correct?
(c) car as may he the in me (a) Marathi-I-Hyderabad (b) Tamil-M-Kolkata
(d) car may the in as he me (c) Marathi-I-Chennai (d) Punjabi-K-Delhi
(e) None of these (e) None of these
17. If the second step of an input is ‘clever remand window 24. Who works in Delhi?
sales batch tiger never’ which of the following will be its (a) H (b) M
sixth step?
(c) L (d) K
(a) clever remand window batch sales tiger never (e) None of these
(b) window remand clever sales batch tiger never 25. What is M’s mother tongue?
(c) batch never sales tiger clever remand window (a) Bangla (b) Marathi
(d) clever remand window tiger batch sales never (c) Telugu (d) Cannot be determined
(e) It cannot have sixth step. (e) None of these
www.newspaperkorner.wordpress.com
20 Practice Set - 2
www.newspaperkorner.wordpress.com
Directions (Qs. 26 & 27) : Each of the following questions 29. Which of the following means ‘very’ in that code language?
contains a paragraph followed by a question based on it. (a) na (b) da
Carefully read the paragraph and answer the question. (c) pa (d) Data inadequate
26. Advertisement: Today’s customers expect high quality. (e) None of these
Every advance in the quality of manufactured products 30. Which of the following statements is/are redundant to
raises customer expectations. The company that is satisfied answer the above two questions?
with the current quality of its products will soon find that (a) None (b) (i) and (iii)
its customers are not. At MegaCorp, meeting of exceeding (c) (ii) or (iv) (d) (i) or (iv)
customer expectations is our goal. (e) None of these
Which of the following must be true on the basis of the Directions (Qs. 31-35): In each of the questions given below,
statements in the advertisement above? which one of the following five Answer Figures on the bottom
should come after the Problem Figures on the top, if the sequence
(a) MegaCorp’s competitors will succeed in attracting
were continued?
customers only if those competitors adopt
31. Problem Figures
MegaCorp’s goal as their own.
(b) A company that does not correctly anticipate the
expectations of its customers is certain to fail in
advancing the quality of its products.
(c) MegaCorp’s goal is possible to meet only if continuing Answer Figures
advances in product quality are possible.
(d) If a company becomes satisfied with the quality of its
products, then the quality of its products is sure to
decline.
(e) None of these (a) (b) (c) (d) (e)
27. The local board of education found that, because the current 32. Problem Figures
physics curriculum has little direct relevance to today’s = 7 7 7 7 P P 7
world, physics classes attracted few high school students. 2 = P P 2 2
So to attract students to physics classes, the board =
é 2 2
proposed a curriculum that emphasizes principles of physics Answer Figures
involved in producing and analyzing visual images.
7 P 7 7 7 2
Which of the following, if true, provides the strongest
reason to expect that the proposed curriculum will be P 2 7 O P O P P O
successful in attracting students? O 2 2 2
(a) Several of the fundamental principles of physics are (a) (b) (c) (d) (e)
involved in producing and analyzing visual images. 33. Problem Figures
(b) Knowledge of physics is becoming increasingly é C éé T C UU ·
important in understanding the technology used in T · T C é· C é
today’s world. = · · U T
C U = = U T = =
(c) Evidence that a large producer of photographic Answer Figures
equipment has donated to the high school could be
used in the proposed curriculum.
= U U = = U = U · =
T C T C é T Té T C
(d) In today’s world the production and analysis of visual é é C C é
images is of major importance in communications, · ·· · U
business, and recreation. (a) (b) (c) (d) (e)
(e) None of these 34. Problem Figures
Directions (Qs. 28-30): In a certain code language meanings of
some words are as follows: éZ R RZ é RZ Ré Ré
(i) ‘pit na sa’ mean ‘you are welcome’.
TV é VT VT TV Z T V Z
(ii) ‘na ho pa la’ means ‘they are very good’.
(iii) ‘ka da la’ means ‘who is good’? Answer Figures
(iv) ‘od ho pit la’ means ‘they welcome good people’.
Té R é R é R éR éR
28. Which of the following means ‘people’ in that code
language? VZ ZV T Z VT Z VT Z VT
(a) ho (b) pit
(c) la (d) od (a) (b) (c) (d) (e)
(e) Data inadequate
www.newspaperkorner.wordpress.com
Practice Set - 2 21
www.newspaperkorner.wordpress.com
35. Problem Figures 45. _________ can be use to send an e-mail to a large group
T Z T Z T at one time.
é C
O C Z T Ué O = C V Z T O = éV 5 CZ (a) Group (b) Alias
= C é V (c) Mail server (d) List server
U = O é =
Answer Figures (e) Remote server

Z é Z Z Z DATA ANALYSIS & INTERPRETATION


V V C V V
éC = T TC = Z TV é TC = é T C = é Directions (Qs. 46-50): Study the following table carefully to
5 5 = 5
5 5 answer the questions that follow :
(a) (b) (c) (d) (e) Number of Clerks and Officers recruited (In thousands) by five
36. Window registry consists of ________ and _____ files. different banks during six different years
(a) user.dat, win, ini (b) user.dat, system.dat
Bank
(c) win.reg, user.dat (d) autoexec.bat, config.sys
Year A B C D E
(e) win.reg, system.dat
Clerk Officer Clerk Officer Clerk Officer Clerk Officer Clerk Officer
37. ________ technology is typically used by Extranet and
Intranet. 2010 4.2 1.6 3.6 2.2 5.3 3.8 6.9 5.6 7.7 6.9

(a) Protocol (b) SSL 2011 5.2 2.7 5.8 4.6 6.9 4.0 7.0 3.8 8.8 4.4

(c) Proprietary (d) Internet 2012 3.3 3.9 4.9 2.8 6.4 4.6 9.5 7.8 9.9 7.9
(e) SET 2013 5.7 4.1 5.8 5.6 7.9 5.7 8.6 6.3 11.5 9.8
38. A program which is in execution is called 2014 6.9 3.9 6.9 5.7 5.8 6.4 11.7 9.8 14.8 11.3
(a) data (b) job
2015 7.2 5.5 8.9 5.1 6.8 5.5 10.6 8.9 15.9 10.7
(c) process (d) information
(e) None of these
46. In which bank is the total number of Officers and Clerks
39. Which major development led to the production of
recruited together in the year 2014 the second highest?
microcomputer?
(a) Logic gates (b) Floppy disks (a) E (b) C (c) B (d) A (e) D
(c) Magnetic disks (d) Magnetic tape 47. What was the ratio of the total number of Officers recruited
(e) Integrated circuits by Bank C and D together in the year 2012 to the number of
40. The arranging of data in a logical sequence is called Clerks recruited by Bank A in the year 2015?
(a) classifying (b) summarizing (a) 31:18 (b) 31:17
(c) reproducing (d) sorting
(c) 18:35 (d) 19:11
(e) sequencing
41. In a computer system, _________ device is functionally (e) None of these
opposite of a keyboard. 48. What was the average number of Officers recruited by Bank
(a) joystick (b) mouse Cover all the years?
(c) trackball (d) printer (a) 3,000 (b) 5,000
(e) scanner
(c) 30,000 (d) 5,500
42. In modems data transfer rate is measured in
(e) None of these
(a) bandwidth (b) bits per second
(c) bits per minute (d) All the above 49. In which bank did the number of Clerks recruited
(e) None of these continuously increase during the years 2010 to 2015?
43. A ________ code is used in retail stores to identify (a) Only D and E (b) Only B
merchandise. (c) Only E (d) Only C and D
(a) mnemonics (b) gray code
(e) None of these
(c) machine code (d) universal product code
50. Total number of Officers recruited by all the banks together
(e) access-3 code
44. What is the short-cut key to highlight the entire column? in the year 2010 was approximately what percentage of the
(a) Ctrl + Page up (b) Ctrl + Page down total number of Clerks recruited by Bank B in the years 2014
(c) Ctrl + Enter (d) Ctrl + Space bar and 2015 together?
(e) Ctrl + C (a) 112 (b) 77 (c) 93 (d) 127 (e) 102
www.newspaperkorner.wordpress.com
22 Practice Set - 2
www.newspaperkorner.wordpress.com
Directions (Qs. 51-55): Study the following Pie-chart and the tennis, cricket and baseball. 15 per cent of the total players have
table given below it carefully to answer these questions. participated in badminton. Two-fifths of the total players have
Percentage-wise Distribution of lecturers in six different participated in hockey. 6 per cent of the total players have
subjects in a university participated in lawn tennis. 25 per cent of the total players have
Total Number of Lecturers: 1600 participated in cricket. Remaining players have participated in
Percentage of Lecturers baseball. One-fourth of the hockey players are females. 20 per
cent of badminton players are males. Half the players who have
Chemistry participated in Lawn Tennis are males. There are 45 female cricket
13% players. No female player has participated in baseball.
Zoology
22% Education 56. The number of female players participating in badminton is
18% approximately what percentage of the number of players
Physics
participating in baseball?
Hindi
14% 12% (a) 72 (b) 75
Mathematics (c) 80 (d) 95
21% (e) 86
Ratio of male to Female Lecturers in the University 57. What is the difference between the number of male players
participating in hockey and the number of female players
Lecturers Males Females participating in lawn tennis?
Mathematics 3 4 (a) 92 (b) 9
(c) 102 (d) 108
Education 5 3
(e) None of these
Hindi 1 3
58. If due to certain reason cricket game was dropped and all
Chemistry 1 7 the cricket players left the tournament, then what would be
Physics 9 5 the total number of male players in the tournament?
Zoology 7 9 (a) 200 (b) 210
(c) 190 (d) 220
51. Total number of lecturers (both male and female) in Hindi is (e) None of these
approximately what per cent of the total number of female
59. What is the ratio of the number of male players participating
lecturers in Mathematics and Chemistry together?
in badminton to the number of female players participating
(a) 58 (b) 43 (c) 47 (d) 51 (e) 40 in hockey?
52. What is the difference between the total number of lecturers (a) 3:11 (b) 3:10
(both male and female) in Zoology and the total number of (c) 6:11 (d) 11:6
male lecturers in Chemistry and Education together? (e) None of these
(a) 192 (b) 182 60. What is the total number of female players participating in
(c) 146 (d) 136 the tournament?
(d) None of these (a) 130 (b) 120
53. What is the difference between the number of female (c) 145 (d) 155
lecturers in Zoology and the number of male lecturers in (e) None of these
Hindi? Directions (Qs. 61-65) : Study the following table carefully and
(a) 156 (b) 160 answer the questions given below it.
(c) 150 (d) 153
(e) None of these Number of Teachers in various Universities (Males &
Females) and percentage of PhDs amongst them
54. What is the total number of male lecturers in the university?
(a) 696 (b) 702 Males Females
Univers- % of No of % of
(c) 712 (d) 668
ities No. of Teachers PhDs Teachers PhDs
(e) None of these
A 175 32 125 44
55. What is the ratio of the number of female lecturers in Physics B 250 74 105 40
to the number of male lecturers in Mathematics? C 180 45 120 55
(a) 5:9 (b) 2:9 D 320 65 80 80
(c) 3:7 (d) 5:3 E 290 30 100 35
(e) None of these 61. What is the average number of female teachers in all the
Directions (Qs. 56-60): Study the following information carefully Universities together ?
to answer the questions that follow :
(a) 100 (b) 105
In a tournament, a total number of 400 players have
(c) 108 (c) 106
participated in five different sports, viz badminton, hockey, lown
(e) None of these
www.newspaperkorner.wordpress.com
Practice Set - 2 23
www.newspaperkorner.wordpress.com
62. What is the total number of non PhD male teachers from 67. The number of people working in legal department of
University A and C together ? organization A is approximately what per cent of the total
(a) 137 (b) 208 number of people working in that organization?
(a) 5 (b) 8
(c) 145 (d) 218
(c) 3 (d) 10
(e) None of these
(e) 12
63. What is the respective ratio of the non-PhD male teachers
68. What is the total number of people working in organization
from University D to the non-Ph.D. female teachers from
B?
the same University ?
(a) 1755 (b) 1525
(a) 7:1 (b) 13 : 4
(c) 1675 (d) 1500
(c) 9:5 (d) 16:13
(e) None of these
(e) None of these
69. What is the difference between the number of people working
64. What is the difference between the number of female PhD in HR, Legal and Marketing departments of organization A
teachers from University E and male non-PhD teachers from and the number of people working in Accounts, Export and
the same University ? IT department of organization B ?
(a) 165 (b) 52 (a) 150 (b) 225
(c) 158 (d) 75 (c) 375 (d) 300
(e) None of these (e) None of these
65. The number of PhD teachers in University B (Both males 70. If 20% more than the existing number of people in the
and females) is approximately what per cent of the total Exports department of organization B, join the department,
number of teachers in the University ? what will be the total number of people working in the
(Both males and females) Exports department of both the organizations (A and B)
(a) 54 (b) 68 together ?
(c) 64 (d) 52 (a) 715 (b) 745
(e) 58 (c) 700 (d) 675
Directions (Qs. 66-70) : Study the following graph carefully and (e) None of these
answer the questions given below it. Directions (Qs. 71-75) : Study the tables of the Indian foreign
trade given below to answer the questions :
Number of People Working in Various Departments of
Two Different Organizations Commodities' Weight (%)
A B 2012-13 2013-14 2014-15
400
Plantations 0.92 0.78 0.71
Agri & allied products 8.39 7.61 7.21
350
M arine products 2.08 1.60 1.40
300 Ores & minerals 3.69 5.29 6.02
250
Leather & mfrs. 3.19 2.89 2.56
Gems & jewellery 16.56 17.29 15.13
200
Sports goods 0.15 0.12 0.13
150 Chemicals & related 15.43 16.00 15.10
100
products
Engineering goods 16.41 18.41 18.66
50 Electronic goods 2.74 2.28 2.18
0 Project goods 0.09 0.06 0.13
HR IT Textiles 18.86 15.16 14.80
Legal

Marketing

Accounts

Exports

Handicrafts 0.70 0.43 0.40


Carpets 0.90 0.75 0.81
Departments Cotton raw incl. Waste 0.28 0.10 0.61
66. What is the respective ratio of the number of people working Petroleum products 5.54 8.57 11.21
in the IT department of organization A and the number of Unclassified exports 4.07 2.66 2.94
people working in the Marketing department of organization Grand Total 100.00 100.00 100.00
B? Total Exports in Rupees 293,366.75 375,339.53 454,799.97
(a) 9 : 2 (b) 7 : 5 crore
(c) 5 : 7 (d) 3 : 7 US Dollar Exchange Rate 45.9513 44.9315 44.2735

(e) None of these


www.newspaperkorner.wordpress.com
24 Practice Set - 2
www.newspaperkorner.wordpress.com (a) 20 years (b) 16 years
Principal Weight (% )
Commodities ' Import (c) 12 years (d) 18 years
Commodities 2012-13 2013-14 2014-15 (e) 21 years
Bulk imports 37.87 39.09 42.56 77. A candidate appearing for an examination has to secure
Pearls , precio us & 9.25 8.80 6.42 35% marks to pass. But he secured only 40 marks and failed
s emi-precious s tones by 30 marks. What would be the maximum marks of test?
Machinery 10.63 10.00 10.94
(a) 280 (b) 180 (c) 200 (d) 150 (e) 210
Project goods 0.49 0.54 0.57
78. The length of a rectangular floor is twice its breadth. If Rs.
Others 41.76 41.57 39.51
Total Imports 100.00 100.00 100.00 256 is required to paint the floor at the rate of Rs. 2 per
Total Imports (in 339,107.66 501064.54 630,526.77 square metre, then what would be the length of floor?
Crore of Rupees ) (a) 16 metres (b) 8 metres
71. The three commodities which had highest export growth (c) 12 metres (d) 32 metres
rate in the year 2013-14 as compared to the previous year, (e) 20 metres
arranged in descending order of growth rates are 79. Angle ‘A’ of a quadrilateral ABCD is 26° less than angle B.
(a) petroleum products, ores and minerals, engineering Angle B is twice angle C and angle C is 10° more than angle
goods D. What would be the measure of angle A?
(b) ores and minerals, gems and jewellry, chemicals and (a) 104° (b) 126° (c) 56° (d) 132° (e) 106°
related products
80. A man walked at a speed of 4 km/hr from point A to B and
(c) gems and jewellery, chemicals and related products,
came back from point B to A at the speed of 6 km/hr. What
agriculture and allied products
would be the ratio of the time taken by the main in walking
(d) ores and minerals, chemicals and related products,
agriculture and allied products from point A to B that from point B to A?
(e) ores and minerals, engineering goods, chemicals and (a) 5 : 3 (b) 2 : 3 (c) 2 : 1 (d) 4 : 3 (e) 3 :2
related products ENGLISH LANGUAGE
72. In the year 2014-15, the commodity which witnessed
maximum growth in exports (in Indian Rupees) as compared Directions (Qs. 81-85): Read each sentence carefully and detect
to the year 2013-14 is the error which may be in an one of the parts a, b , c or d . In case
(a) petroleum products (b) project goods of ‘No error’, (e) will be the answer.
(c) ores and minerals (d) sports goods 81. To hit a man (a)/ when he is down (b) / is a contemptuous
(e) None of these (c)/ thing to do (d)/ No error(e).
73. In the two years, period from 2013-14 to 2014-15, the average 82. The climate on this (a)/ altitude is cool and dry (b)/ but
growth in import (in Indian Rupees) of which commodity to down in the valley (c)/ it is very sultry(d)/ No error(e).
India was maximum? 83. Since her mother (a)/ died when she (b)/ was young (c)/ she
(a) Bulk imports did lack maternal are (d)/ No error(e).
(b) Pearls, precious and semi-precious stones 84. In 1665 a (a)/ great pestilence caused (b)/ the death of 63000
(c) Machinery persons (c)/ in the city of London (d)/ No error(e).
(d) Project goods 85. Because of his (a)/ lack of interest in the (b)/ affairs of the
(e) Others society, the members (c)/ decided to depose the president
(d)/ No error(e).
74. Growth of trade imbalance (exports less imports) in dollar
Directions (Qs. 86-90) : Rearrange the sentences given below
terms in the year 2014-15 as compared to the previous year
in away which would make an appropriate and meaningful
was
paragraph. Mark the correct order of sentences and answer the
(a) 39.77 (b) 41.85 (c) 91.24 questions given below.
(d) 95.98 (e) None of these
A. In most of the tourist destinations ofthe valley, the sombre
75. Given that the weight (%) Petroleum crude and products in environs greet the visitor.
the total imports of India is 26.70, 27.87, and 30.87 in the B. But the turmoil has restricted most of the pilgrims only to
years 2012-13, 2013-14, and 2014-15 respectively. What is visit the Amarnath Shrine.
the ratio of yearly difference in the export of petroleum
C. Besides, most of the Amarnath Yatris too used to visit the
products and import of petroleum crude and products, in valley’s tourist spots after their pilgrimage to the holy
dollar terms, in the year 2014-15 versus 2013-14? Amarnath cave shrine.
(a) 1.36 (b) 1.38 (c) 1.46 D. The tourism industry was expecting a booming business
(d) 1.48 (e) None of these this summer.
76. The average age of a family of five members is 24. If the E But the unrest has dashed the hopes of those associated
present age of the youngest member is 8 years, what was with this industry, the mainstay of Kashmir’s economy.
the average age of the family at the time of the birth of the F. The industry was expecting at least ten lakh visitors this
youngest member? season.
www.newspaperkorner.wordpress.com
Practice Set - 2 25
www.newspaperkorner.wordpress.com
86. After rearrangement which of the following sentences will funds rate—15 times dating back to June 2004, and is widely
come ‘FIRST’? expected to raise it one or twice more over the next few months. A
(a) A (b) B (c) C (d) D (e) E brief recession and the Sept. 11 terrorist attacks in 2001 spurred a
87. After rearrangement which will be the ‘SECOND’ one? prolonged period of very low interst rates. That boosted U.S.
consumption—in particular the ratesensitive housing market—
(a) A (b) B (c) C (d) D (e) E
and kept the global economy humming. But long-term rates are
88. After rearrangement which will be the ‘FOURTH’ one?
now beginning to tick upward: last week the U.S. 30- year treasury
(a) E (b) F (c) D (d) C (e) A bond reached 5.04% its highest level since late 2004, and the
89. After rearrangement which will be the ‘FIFTH’ one? housing market is cooling off—potentially triggering an economic
(a) B (b) C (c) D (d) E (e) F slowdown as homeowners cut their spending.
90. After rearrangement which will be the ‘LAST’ one? 91. According to the passage, what makes people hopeful
(a) F (b) E (c) D (d) B (e) A about the markets ?
Directions (Qs. 91-99) : Read the following passage carefully (a) The descent in the value of stocks
and answer the questions given below it. Certain words / phrases (b) The trend of substantial increase in value of stocks
have been printed in bold to help you locate them while answering (c) Safety provided by banks to their deposits
some of the questions. (d) Optimism of the stock market players
For more than three years, Anna Feng didn’t tell her (e) The interest rates are going up
husband that she had sunk nearly half of their savings into the 92. Which of the following is TRUE about the comparison
Shanghai stock market. While he thought all their money was between market indices of Mumbai and Hong Kong on the
safely sitting in a bank, the value of the stocks plunged by almost one hand and Singapore, Jakarta and Sydney on the other?
75%. (a) The indices in the former case are increasing while
But over the past couple of months, the Shanghai market has those in the latter are decreasing
shown signs of life, and Feng, a 56-year-old retiree, has recouped (b) There is no rerqarkable trend visible between the two
half her losses. She’s quietly hopeful that may be she’ll make it all sets of indices
back. “Everyone seems to be so optimistic about the markets (c) Indices in both the groups of exercises are stable over
now,” she says.
a period of last five years
Around the world, stocks have been on a tear. In Asia, for
(d) The markets in both the groupd of countries have
example, the Tokyo TOPIX stock index hit a 14-year high last
shown upward trend
week as a bull run in once-dormant Japan gathered momentum;
Mumbai’s main equity index hit an all-time high in trading early (e) None of these
Friday amid India’s continuing economic boom; and Hong Kong 93. What is the impact of increasing long term interest rates?
shares reached a five-year high while indices in Singapore, Jakarta (A) Demand in housing market is gradually diminishing.
and Sydney set new records. And though stock in Asia, in (B) Retardation in economic growth.
particular, are on fire, they are not alone. From Germany to (C) Restrictions imposed by central bank.
Venezuela to South Africa, equity markets in both mature and (a) (A) only (B)only (b) (C) and (B) only
emerging markets have moved up sharply this year— and show
little sign of slowing. (c) (C) only (d) (A) & (C) only
The underpinning for stocks' strong performance, global (e) None of these
bulls say, is straight-forward. Economic growth continues to be 94. In what way did the terrorist attack in the US influence the
strong in places where it has been buoyant for several years (the markets ?
U.S., China and India) and is finally picking up in places where it (a) It led to a brief recession
had been notably absent—Japan and parts of “old” Europe. (b) It increased long term need for housing
Moreover, earning and corporate balance sheets around the world (c) It helped increase the interest rates on housing
are as healthy as they have been in years. In Japan, corporate
profits have climbed for four straight years and consumer spending (d) It prolonged the low interest rate regime
is rising briskly on the back of declining unemployment. (e) None of these
Economists say that Japan is now in a golden cycle. So, for now, 95. Which of the following is/are the reason(s) for the statement
is much of the world. “It comes down to very simple that ‘Japan is now in golden cycle’ ?
macroeconomics.” says Subir Gokam, an economist at CRISIL, (A) It is an economic growth without much inflationary
India's largest credit-rating firm. “The global economy is growing pressure.
without much inflationary pressure.” (B) Japan witnesssed a substantial increase in corporate
Is anything wrong with this picture? One very big thing, profits for the last four years.
warn the skeptics. Interest rates are rising nearly everywhere,
(C) There are more employment avenues open and
and if there is one simple adage that many investment advisers
consumer’s spending has increased significantly.
live by, It's this: “When rates are high, stocks will die.” Indeed,
one of the most impressive—or scariest— aspects of the current (a) All the three-(A), (B) and (C)
global bull run is that it has come in the teeth of central- bank (b) (A) and (B) only
tightening, most importantly by the U.S. Federal Reserve, which (c) Either (B) or (C) only
could slow growth in the world’s key economic locomotive. The (d) (A) and (C) only
Fed has increased key short-term interest rate—the so-called Fed (e) Either (A) or (C) only
www.newspaperkorner.wordpress.com
26 Practice Set - 2
www.newspaperkorner.wordpress.com
Directions (Qs. 96-97) : Which of the following is most (a) realistic, zeal
OPPOSITE in meaning of the word given in bold as used in the (b) lower, conviction
passage ? (c) loud, argument
96. Briskly (d) soft, appeal
(a) vigorously (b) efficiently (e) pitch, statement
(c) hurriedly (d) insignificantly 104. The tunnel was so ................. and congested, that we became
(e) slowly ................. .
97. Boosted (a) long, enthusiastic
(a) aggravated (b) elevated (b) deep, cautious
(c) deflated (d) damaged (c) dark, frightened
(d) crowded, isolated
(e) stopped
(e) sharp, worried
Directions (Qs. 98-99): Which of the following is most nearly
the SAME in meaning as the word given in bold as used in the Directions (Qs. 105-114): In the following passage, there are
blanks, each of which has been numbered. These numbers are
passage?
printed below the passage and against each, five words, are
98. Buoyant suggested one of which fits the blank appropriately. Find out the
(a) drawing (b) haphazard appropriate word in each case.
(c) upbeat (d) extravagant Most of us are (105) of open conflict and avoid it if we can.
(e) sailing And there is a (106) to expressing and working through conflict.
99. Spurred If the working through involves harsh words and name-calling
(a) shortened (b) widened people feel deeply hurt and relationships can be (107). Sometimes
permanently. Some group members may be afraid that if they
(c) thronged (d) stimulated
really (108) their anger, they may go out of control and become
(e) escalated Violent, or they may do this. These fears can be very (109) and
Directions (Qs. 100-104) : In each of the following sentences based on experience. So why take the risk ? Why not avoid conflict
there are two blanks spaces. Below each sentence there are five at all costs ? Conflict is rather like disease (110) is best, that
pairs of words denoted by numbers (a), (b), (c), (d) and (e). Find means attending to areas where (111) may occur before they
out which pair of words can be filled up in the blanks in the become an issue. If you have not (112) a conflict happening,
sentence in the same sequence to make the sentence your next choice is to treat it early, 93. or hope that it goes away.
grammatically correct and meaningfully complete. If it goes away over time fine. If it (113). then you will still have to
100. More is ................. of conditions of the tribals in Maharashtra handle (treat) it and it is likely to be more (114).
than ................. conditions of those in the other parts of the 105. (a) scared (b) careful
country. (c) reckless (d) aware
(a) certain, the (e) worried
(b) known, of 106. (a) challenge (b) measure
(c) aware, of (c) principle (d) chance
(d) favourable, those (e) risk
(e) uncertain. all 107. (a) established (b) maligned
101. Although ................. is not a very desirable feeling, we need (c) damaged (d) rebuilt
a certain amount of it to .......................... well. (e) involved
(a) anxiety, exist 108. (a) sublimate (b) express
(b) grief, enjoy (c) minimize (d) regulate
(e) control
(c) impatience, preach
109. (a) baseless (b) imaginary
(d) anger, define
(c) exaggerative (d) real
(e) jealousy, bad
(e) national
102. Although he is a ................. person, he occasionally loses
110. (a) cure (b) diagnosis
his ................. .
(c) prescription (d) prevention
(a) quiet, power
(e) medicine
(b) cheerful, grief
111. (a) harmony (b) discomfiture
(c) balanced, temper
(c) disagreement (d) consensus
(d) thoughtful, anxiety
(e) statement
(e) generous, wealth
112. (a) expressed (b) ignored
103. In a ................. tone, the leader made a powerful ................. to
(c) induced (d) seen
the mob.
(e) perverted
www.newspaperkorner.wordpress.com
Practice Set - 2 27
www.newspaperkorner.wordpress.com
113. (a) doesn’t (b) won’t 118. Indian companies are allowed to arrange funds from sources
(c) don’t (d) not abroad through which of the following means?
(e) hasn’t (A) External Commercial Borrowings
114. (a) credible (b) serious (B) Foreign Currency Convertible Bonds
(c) fraudulent (d) urgent (C) Preference Shares
(e) skilled (a) Only (A) (b) Only (B)
Directions (Qs. 115) : In each of the given sentences, select the (c) Only (C) (d) Only (A) and (B)
sentence which would either follow or precede the given sentence (e) All (A), (B) and (C)
in grammatically and conceptually appropriate manner. The 119. The purpose(s) of public distribution system (PDS) is/are
instruction is given at the end of every statement. (A) to control the open market prices of various goods.
115. Unfortunately, however, these slum dwellers are looked (B) to provide competitive prices to the produce of farmers
upon by the society as an appendix causing ills in the urban (C) to provide food grain and other essential goods to
society. people of weaker sections
(a) Only (A) (b) Only (B)
Which of the following sentences would immediately preced
(c) Only (C) (d) Only (A) and (C)
the above sentence?
(e) All (A), (B) and (C)
(a) Health officials have been warning the government
120. RBI constituted an expert committee for examining its
against the transmission of contagious diseases from
current monetary policy framework. Who is the chairman of
the slum areas to other parts of the city.
the committee?
(b) Slum dwellers not only play a significant role in urban (a) Dr. Urjit Patel (b) H R Khan
economy but also provide cheap labour for everyday (c) Dr Anand Sinha (d) K C Chakrabarty
work of the cities. (e) None of these
(c) A recent report suggested that 34 per cent of 121. Who was the first chairperson and managing director of
government land is illegally occupied by the slum public sector Bharatiya Mahila Bank (BMB)?
dwellers. (a) Shilpa Phadnis
(d) The slum dwellers live in sub-human, unhygienic (b) Usha Anantha Subramanian
conditions in their tiny shanties and are very often (c) Reeba Zachariah
hubs for criminal activities. (d) Sudha Misra
(e) None of these (e) None of these
122. Who is the author of the recently published book Durbar?
GENERAL/ECONOMY/BANKING AWARENESS (a) Amitav Ghosh (b) Tavleen Singh
116. The Reserve Bank of India takes which of the following (c) Amit Chaudhuri (d) Arundhati Roy
measures when it intends to ease the liquidity crunch in (e) None of these
the country? 123. Which of the following rates is/are NOT decided by RBI?
(A) It prints and supplies additional currency notes in the (a) Bank Rate (b) Repo rate
market. (c) Prime Lending Rate (d) Cash Reserve Ratio
(B) It takes steps to increase the flow of foreign direct (e) None of these
investment. 124. Consider the following types of loans.
(C) It reduces Statutory Liquidity Ratio (SLR) and Cash (A) Home loans
Reserve Ratio (CRR). (B) Personal loans
(a) Only (A) (b) Only (B) (C) Reverse Mortgage loans
(c) Only (C) (d) Only (A) and (B) (D) Crop loans
(e) All (A), (B) and (C) Teaser Rates are associated with which of the above
117. We often come across the term SWIFT in financial mentioned types of loans?
newspapers. What is the expanded form of this term? (a) Only (A) (b) Only (B)
(a) Society for Worldwide Interbank Financial (c) Only (C) (d) Only (A) and (B)
Telecommunication (e) Only (C) and (D)
(b) Secure Worldwide Interbank Financial 125. Foreign Exchange Reserves of India are kept in the custody
Telecommunication. of which of the following?
(c) Society for Worldwide Intr a-bank Financial (a) State Bank of India
Transaction. (b) Reserve Bank of India
(d) Security for Worldwide Interbank Finan cial (c) Government Treasury
Transaction (d) International Monetary Fund
(e) None of these (e) Indian Bank Association
www.newspaperkorner.wordpress.com
28 Practice Set - 2
www.newspaperkorner.wordpress.com
126. Insurance Regulatory and Development Authority (IRDA) 133. The paid up share capital of Regional Rural Bank is
is an autonomous apex statutory body which regulates contributed by
and develops the insurance industry in India. Which of (a) Nationalised bank
the following statements is/are correct about IRDA? (b) NABARD
(a) It was constituted by a Parliament of India act called (c) Central Government
Insurance Regulatory and Development Authority (d) Central Government, State Government and the
Act, 1999 Sponsoring Comerical Bank in the ratio of 50 : 15 : 35
(b) The agency operates from its headquarters at respectively
Hyderabad in Andhra Pradesh where it shifted from (e) Reserve Bank of India
Delhi in 2001. 134. Regional Rural Banks are managed by
(c) It works to bring about speedy and orderly growth of (a) Reserve Bank of India (b) a board of directors
the insurance industry, protect the interest of (c) the sponsor bank (d) the State Government
policyholders and put in place effective grievance
(e) None of these
redressal machinery.
135. Deposits with Regional Rural Banks are insured by
(d) Only (a) and (b)
(a) Life Insurance Corporation of India
(e) All (a), (b) and (c)
(b) General Insurance corporation
127. The abbreviation IRDA stands for
(c) Deposit Insurance and Credit Guarantee Corporation
(a) Industrial Research and Development Authority of
India (d) All of the above
(b) Insurance Research and Development Authority of India (e) None of these
(c) Insurance Regulation Development Authority of India 136. For opening a new branch, a Regional Rural Bank requires
(d) Industrial Research and Demands Agency (a) permission of NABARD
(e) None of these (b) permission of Director, Institutional Finance
128. We read a term 'ECB' in the financial newspapers. What is (c) approval of SIDBI
the full form of ECB? (d) RBI license
(a) Essential Credit and Borrowing (e) approval of DRDA
(b) Essential Commercial Borrowing 137. Regional Rural Banks are classified as
(c) External Credit and Business (a) scheduled commercial banks
(d) External Commercial Borrowing (b) subsidiaries of the sponsor banks
(e) None of these (c) subsidiaries of NABARD
129. Which is known as the ‘Land of the Midnight Sun’? (d) All of the above
(a) Japan (b) Norway (e) None of these
(c) Newzealand (d) Finland 138. Regional Rural Banks
(e) None of these (a) can issue guarantees against 100% cash margin or 50%
130. Which of the following Acts help Union government to cash margin plus collateral
control its fiscal deficit? (b) have been freed from Service Area Approach. They
(a) Banking Companies Act can operate in the entire area of their Jurisdiction.
(b) Fiscal Responsibility and Budget Management Act Commercial banks will take over the villages allotted
(c) Finance Act to RRBs under SAA
(d) Both (a) and (b) (c) are permitted to open Extension Counters at the
(e) None of these premises of the borrowers, if they are the principal
131. Article 370, which is one of the temporary, transitional and banker to the borrower
special provisions in the constitution of India is enshrined (d) All of the above
in which among the following parts of Indian Constitution? (e) None of these
(a) Part XV (b) Part XVII 139. The Regulatory Authority for Regional Rural Banks is
(c) Part XXI (d) Part XXII (a) Sponsor bank (b) Central Governement
(e) None of these (c) State Government (d) RBI and NABARD
132. Section 20 of the Banking Regulation Act, prohibits a bank (e) IDBI
from granting any advance against the security of 140. The short and medium term co-operative credit structure in
(a) its own shares our country is federal in character consists of three tiers
(b) partly paid shares viz.,
(c) personal guarantee of directors and managers (a) The State co-operative banks at the State level, the
Central co-operative banks at the district level and
(d) guarantee of other banks
primary co-operative banks at the district level and
(e) None of these
primary co-operative credit societies at the village level
www.newspaperkorner.wordpress.com
Practice Set - 2 29
www.newspaperkorner.wordpress.com
(b) the State co-operative banks in the State, NABARD 147. Which of the following statements does the term Finance
and RBI incorporates?
(c) primary credit co-operative societies at the grass root (a) The study of money and other assets
level, district co-operative bank at district level and (b) The management and control of those assets
NABARD at All-India level (c) The science of managing money
(d) All of the above (d) All of the above
(e) None of these (e) None of these
141. Central co-operative banks 148. Which term is used for "Money which is used to purchase
(a) occupy a crucial importance in the co-operative credit assets that will remain permanently in the business and
structure help it to make a profit"?
(b) form an important link between the State co-operative (a) Fixed Capital (b) Working Capital
bank at the apex and the primary agricultural credit (c) Capital Budget (d) Cash Budget
societies at the base (e) None of these
(c) are closer to the primary societies than an apex bank 149. What is the name of the proposed long distance superfast
could be train, which will be introduced for unreserved passengers?
(d) All of the above (a) Anandoya Express (b) Antyodaya Express
(e) None of these (c) Anand Express (d) All of the above
142. The primary function of a central co-operative bank is to (e) None of these
(a) mobilise the resources in the district for financing its 150. In the Rail Budget 2016-2017, Suresh Prabhu announced
members to the maximum extent possible introduction of a service named Sarthi Seva for senior
(b) to channelise the flow of funds from the State co- citizens. The service will be first introduced in which Indian
operative banks Railways Zone?
(c) Both of the above (a) Southern Railway (b) Northern Railway
(d) All of the above (c) Konkan Railway (d) North Central Railway
(e) None of these (e) None of these
143. Regional Rural Banks are managed by 151. Which of the following statements is wrong about the Rail
(a) Reserve Bank of India (b) a board of directors Budget 2016-2017?
(c) the aponsor bank (d) the State Goverment (a) Introduction of Humsafar trains for senior citizens
(e) Central Government (b) Coolies will be called Sahayaks
144. The long-term co-operative credit structure consists of two (c) All new Railway stations to be built as per Accessible
tiers only viz., India guidelines
(a) Central land mortgage banks at State level and primary (d) The cleaning of toilets will be requested through SMS
land mortgage banks at the district level (e) None of these
(b) State land mortgage banks at the State level and 152. As per the Union Budget 2016-2017, what is the amount
NABARD at All-India level allocated for the Swachch Bharat Abhiyan?
(c) Regional Rural Banks at the district level and (a) 10,000 crore rupees (b) 9,000 crore rupees
NABARD at central level (c) 11,000 crore rupees (d) 6,000 crore rupees
(d) All of the above (e) None of these
(e) None of these 153. The Union Budget 2016-2017 saw the highest ever amount
145. Co-operation as a form of economic activity was formally allocated for the rural employment scheme MNREGA. What
recognised in India in the year 1904 when is that amount?
(a) Mahatma Gandhi inaugurated the first Indian co- (a) 40,000 crore rupees (b) 50,500 crore rupees
operative bank
(c) 38,500 crore rupees (d) 39,500 crore rupees
(b) when the first Co-operative Societies Act was passed
(e) None of these
(c) when the first co-operative bank was established in
154. India's first IT co-operative UL CyberPark has been set up
Bombay
in which state?
(d) All of the above
(a) Kerala (b) Gujarat
(e) None of these
(c) Arunachal Pradesh (d) Tamil Nadu
146. The first Union budget of independent India was presented
(e) None of these
by:
155. What is the maximum speed of the India's first semi high-
(a) Jawahar Lal Nehru
speed train "Gatimaan Express"?
(b) Vallabh Bhai Patel
(a) 160 kmph (b) 170 kmph
(c) R. K. Shanmukham Chetty
(c) 180 kmph (d) 190 kmph
(d) Morarji Ranchhodji Desailn
(e) None of these (e) None of these

www.newspaperkorner.wordpress.com
30 Practice Set - 2
www.newspaperkorner.wordpress.com

Time : 30 min. Max. Marks : 50

1. Write a letter on any one of the following : (20 marks)


(a) Write a Letter to authority for requesting a change in the Coverage limits of your current policy.
(b) Write a letter informing a Financial Institution that you plan to close out your account.
(c) Write a letter to the Superintendent of nearest Railway station, complaining about the lack of proper facilities at the
station.
2. Write a paragraph on any one of the following in not more than 150 words : (10 marks)
(a) Censorship is sometimes justified.
(b) Human Trafficking in India.
(c) Cloud Computing
3. Write an essay on any one of the following in about 250 words : (20 marks)
(a) Impact of Media in day to day life
(b) Status of women in India.
(c) Importance of opposition in a Democracy.

www.newspaperkorner.wordpress.com
Practice Set - 2 31
www.newspaperkorner.wordpress.com
Answer Key
1 (c) 17 (e) 33 (a) 49 (e) 65 (c) 81 (e) 97 (d) 113 (a) 129 (c) 145 (b)
2 (d) 18 (a) 34 (c) 50 (d) 66 (b) 82 (a) 98 (c) 114 (b) 130 (b) 146 (c)
3 (a) 19 (d) 35 (a) 51 (d) 67 (a) 83 (d) 99 (d) 115 (b) 131 (c) 147 (d)
4 (b) 20 (e) 36 (b) 52 (c) 68 (e) 84 (e) 100 (b) 116 (c) 132 (a) 148 (a)
5 (e) 21 (d) 37 (d) 53 (c) 69 (c) 85 (e) 101 (b) 117 (a) 133 (d) 149 (b)
6 (d) 22 (a) 38 (c) 54 (a) 70 (b) 86 (d) 102 (c) 118 (e) 134 (c) 150 (c)
7 (c) 23 (b) 39 (e) 55 (a) 71 (a) 87 (e) 103 (d) 119 (c) 135 (c) 151 (a)
8 (b) 24 (c) 40 (d) 56 (e) 72 (b) 88 (b) 104 (c) 120 (a) 136 (d) 152 (b)
9 (e) 25 (e) 41 (d) 57 (d) 73 (d) 89 (b) 105 (a) 121 (b) 137 (a) 153 (c)
10 (a) 26 (c) 42 (b) 58 (a) 74 (b) 90 (d) 106 (e) 122 (b) 138 (d) 154 (a)
11 (d) 27 (d) 43 (d) 59 (b) 75 (a) 91 (b) 107 (c) 123 (c) 139 (d) 155 (a)
12 (a) 28 (d) 44 (d) 60 (c) 76 (a) 92 (d) 108 (b) 124 (a) 140 (a)
13 (c) 29 (c) 45 (d) 61 (d) 77 (c) 93 (e) 109 (d) 125 (b) 141 (d)
14 (a) 30 (e) 46 (e) 62 (d) 78 (a) 94 (d) 110 (d) 126 (e) 142 (d)
15 (c) 31 (b) 47 (a) 63 (a) 79 (e) 95 (c) 111 (b) 127 (c) 143 (b)
16 (b) 32 (c) 48 (b) 64 (e) 80 (e) 96 (e) 112 (b) 128 (d) 144 (a)

Answers & Explanations


1. (c) IV does not follow by converting the third statement. are mirrors + No mirror is glass = I + E = O = Some
Some eagles are rabbits + All rabbits are birds = 1 + A plastics are not glasses. Hence II and IV do not follow.
= I Some eagles are birds ® conversion ® Some birds However, the two make a complementary I-E pair. Hence
are eagles. Hence III follows. All snakes are eagles + either II or IV follows.
Some eagles are birds = A + I = No conclusion. Hence 5. (e) All graduates are advocates + Some advocates are
II and consequently I do not follow. judges = A + I = No conclusion. Hence II and
2. (d) All notebooks are diaries (A) ® conversion ® Some consequently III do not follow. Some advocates are
diaries are notebooks (I). Now, Some calculators are judges + All judges are lawyers = I + A = I = Some
diaries + Some diaries are notebooks = I + I = No advocates are lawyers ® conversion ® Some lawyers
conclusion. Hence I does not follow. Some calculators are advocates. Hence IV follows. Some advocates are
are diaries + All diaries are computers = I + A = I = Some lawyers + Some lawyers are doctors = I + I = No
calculators are computers. Hence II follows. All conclusion. Hence I does not follow.
notebooks are diaries + All diaries are computers = A + 6. (d) R £ K ... (i); K < M ... (ii); M = J ... (iii) Combining these,
A = All notebooks are computers. Hence III follows. we get R £ K < M = J
Some cameras are calculators + Some calculators are
Hence J > K and I follows.
diaries = I + I = No conclusion. Hence IV does not
follow. Also, M > R and II follows.

3. (a) All planets are stars + All stars are asteroids = A + A = Again, R < J and III follows.
A = All planets are asteroids ® conversion ® Some 7. (c) Z = M ... (i); M > K ... (ii); K < F ... (iii)
asteroids are planets (I). Hence I does not follow. Nor Combining these, we get Z = M > K < F
does II follow by converting the second statement From this F and Z can’t be compared. Neither can F
Again, All stars are asteroids + All asteroids are moons and M.
= A + A = A = All stars are moons ® conversion ®
Hence, I and III do not follow. But K < Z and II follows.
Some moons are stars (I). Hence III does not follow.
Some moons are rocks (I) ® conversion ® Some rocks 8. (b) B < J ... (i); J ³ W... (ii); W > M ... (iii)
are moons (I) + Some moons are stars = I + I = No Combining these, we get B < J ³ W > M.
conclusion. Hence IV does not follow. Hence M < J and I follows.
4. (b) I-type statements can’t be combined among But W and B can’t be compared. Neither can B and M.
themselves. Hence I and III do not follow. Some plastics
Hence II and III do not follow.
www.newspaperkorner.wordpress.com
32 Practice Set - 2
www.newspaperkorner.wordpress.com
9. (e) V ³ H ... (i); H = F ... (ii); F £ E ... (iii) (e) ‘are’ is na [ from (i) and (ii)].
Combining these, we get. (f). ‘very’ is pa [ by elimination in (ii)].
30. (e) Only iii is redundant
V ³ H = F £ E. Hence V ³ F.
Which means either I (F = V) or II (F < V) follows. 31. (b) In alternate steps the left element shift to right ® middle.
The second and fourth from left interchange places
Again, E ³ H. Hence III follows.
while all of them get inverted.
10. (a) W > T ... (i); T £ N ... (ii); N ³ D ... (iii)
32. (c) In alternate steps the upper left, the upper middle and
No comparisons can be made. left middle shift one step ACW in a cyclic order. The
16–20: The words are arranged according to the number of letters
centre shifts to lower middle and a new element appears
they have, one at a time. The word with the maximum number of
at centre.
letters is put first. If two words have the same number of letter, we
go for alphabetical arrangement. 33. (a) In the first step three outer corner elements shift one
16. (b) Input : the in car as he may me step in a cyclic order while the three corresponding
Step I : car the in as he may me inner elements also shift in the same way. The remaining
Step II : car may the in as he me one inner and outer elements interchange places. In
Step III : car may the as in he me the next step the upper four and lower four elements
17. (e) Step II : clever remand window sales batch tiger never shift one step in a cyclic order.
Step III : clever remand window batch sales tiger never 35. (a) In alternate steps the upper middle, the left middle and
Step IV : clever remand window batch never sales tiger the right middle shift one step ACW in cyclic order
Now, step IV would be the last step. while the lower three elements of the middle column
18. (a) Input : true se veto be nuke my like shift one step downward in cyclic order.
Step I : like true se veto be nuke my
47. (a) Ratio = (4.6 + 7.8) : 7.2
Step II : like nuke true se veto be my
Step III : like nuke true veto se be my = 12.4 : 7.2 = 31:18
Step IV : like nuke true veto be se my 48. (b) Total number of officers recruited by C over all the
19. (d) Input : more fight cats cough sough acts idea years (in thousand)
Step I : cough more fight cats sough acts idea = 3.8 + 4 + 4.6 + 5.7 + 6.4 + 5.5 = 30.0
Step II : cough fight more cats sough acts idea
Step III : cough fight sough more cats acts idea 30.0
Average Number = = 5000
Step IV : cough fight sought acts more cats idea 6
Step V : cough fight sough acts cats more idea
50. (d) Total number of officers recruited by all banks in 2010
Step VI : cough fight sough acts cats idea more
= 1.6 + 2.2 + 3.8 + 5.6 + 6.9 = (20.1) × 103
20. (e) We can’t move backward.
(21-25) : Number of clerks recruited by B in 2014 and 2015
Member City Mother tongue = (6.9 + 8.9)103 = (15.8) × 103
H Chennai Marathi
Required percentage
I Hyderabad Telugu
J Bangalore Kannada
K Ahmedabad Punjabi 20.1 103
= ´ = 127%
L Delhi Bangla 15.8 103
M Kolkata Tamil
N Mumbai Hindi 51. (d) Total number of lecturers (male + female) in Hindi =
25. (e) Tamil 12% of 1600 = 192
28-30 pit na sa Þ you are welcome… (i) Total number of female lecturers in Mathematics &
na ho pa la Þ they are very good… (ii) Chemistry together
ka da la Þ who is good… (iii)
4 7
od ho pit la Þ they welcome good people… (iv) = × 21% of 1600 + × 13% of 1600
7 8
Code for
(a) ‘good’ is la [ from (ii) and (iv)]. = 192 + 182 = 374
(b) ‘they’ is ho [ from (ii), (iv) and (a)].
192
(c) ‘welcome’ is pit [ from (i) and (iv)]. Required percentage = ´100 » 51%
(d) ‘people’ is od [ by elimination in (iv)]. 374
www.newspaperkorner.wordpress.com
Practice Set - 2 33
www.newspaperkorner.wordpress.com 65. (c) Reqd. %
52. (c) Total number of lecturers in Zoology
= 22% of 1600 = 352
æ 74 40 ö 100
Total number of male lecture in Chemistry and = ç 250 ´ + 105´ ÷ ´ %
è 100 100 ø 355
1 5
Education = ´13% of 1600 + × 18% of 1600 (18500 + 4200) 22700
8 8 = % = % = 63 – 94%
355 355
= 26 + 180 = 206
= 64%(App.)
Required difference = 352 – 206 = 146
66. (b) Reqd. ratio = 350 : 250 = 7 :5
9 67. (a) Total no. of workers in org. A
53. (c) Number of female lectures in Zoology = ´ 22% of
16 = (150 + 75 + 300 + 225 + 325 + 350) = 1425
1600 = 48
75 ´100
Required difference =198 – 48 = 150 Reqd.% = % = 5.26% ; 5%
1425
54. (a) Total number of male lecture in university
68. (e) Required number
3 5 1 = (200 + 225 + 250 + 300 + 350 + 250) = 1575
= ´ 21%160 + ´18% of 1600 + × 12% of 1600 +
7 8 4 69. (c) Reqd. difference

1 7 = (150 + 75 + 300) – (300 + 350 + 250)


9
× 13% 1600 + × 14% of 1600 + × 22 of 1600 = 525 – 900 = 375
8 14 16
70. (b) Reqd. number
= 144 + 180 + 48 + 26 + 144 + 154 = 696
55. (a) Respective ratio 120
= 350 ´ + 325 = 420 + 325 = 745
100
æ 14 5 ö 21 3 71. (a)
= ç 1600 ´ ´ ÷ :1600 ´ ´
è 100 14 ø 100 7
Commodities 2012-13 2013-14 Growth
=5:9
Plantations 0.92 0.78 -ve
56. (e) Approximate percentage Agri & allied prodts 8.39 7.61 -ve
Marine products 2.08 1.60 -ve
48 Ores & minerals 3.69 5.29 43.36
= ´100 = 87.71 » 86%
56 Leather & mfrs. 3.19 2.89 -ve
Gems & jewellery 16.56 17.29 4.41
57. (d) Required difference = 120 – 12 » 108 Sports goods 0.15 0.12 -ve
59. (b) Ratio = 12 : 40 = 3 : 10 Chemicals & related products 15.43 16.00 3.69
61. (d) Average number of female teachers Engineering goods 16.41 18.41 12.19
Electronic goods 2.74 2.28 -ve
125 + 105 + 120 + 80 + 100 530 Project goods 0.09 0.06 -ve
= = = 106 Textiles 18.86 15.16 -ve
5 5
Handicrafts 0.70 0.43 -ve
62. (d) Reqd. number Carpets 0.90 0.75 -ve
Cotton raw incl. Waste 0.28 0.10 -ve
68 55 Petroleum products 5.54 8.57 54.69
=175 ´ + 180 ´ = 119 + 99 = 218
100 100 Unclassified exports 4.07 2.66 -ve
It is clear that the three commodities with highest export
63. (a) Reqd. ratio
growth rate are petroleum products, ores & minerals
æ 35 ö æ 20 ö and engineering goods (in that order) as seen from the
ç 320 ´ ÷ : ç 80 ´ ÷ = 112:16 = 7:1 table.
è 100 ø è 100 ø
72. (b) Among the commodities given 7 show a positive
64. (e) Reqd. difference increase in its weight.
There is a 21% increase in total exports (in Rupees).
æ 35 ö æ 70 ö
÷ : ç 290 ´
On clearly visualising the 7 commodities, we see that
ç 100 ´ ÷ = 35 ~ 203 = 168
è 100 ø è 100 ø there is more than 100% growth in project goods (0.06
to 0.13). So, it will show maximum growth.
www.newspaperkorner.wordpress.com
34 Practice Set - 2
www.newspaperkorner.wordpress.com
73. (d)
120 - 8 ´ 5
Total bulk pearls machine project others 76. (a) = 20
4
imports goods
2003-04 Weight 359,107.66 38 9 11 0 42 77. (c) 35% of total marks = 40 + 30 marks 100% = 200 marks.
Imports 135994 33217 38173 1760 149963 78. (a) Let breadth = x
2004-05 Weight 501,064.54 39 9 10 1 42
Imports 195866 44094 50106 2706 208293 \ length = 2x
Growth 44 33 31 54 39
2005-06 Weight 630,526.77 43 6 11 1 40 256
Imports 268352 40480 68980 3594 249121 \ 2 x2 = Þ x 2 = 64;
Growth
2
37 -8 38 33 20
Average Growth 41 12 34 43 29 \ x = 8 and 2x = 16
74. (b) 79. (e) A + B + C + D = 360°

2012-13 2013-14 2014-15 B B


Rs / Dollar 45.9513 44.9315 44.2735 Þ B – 26° + B + + - 10° = 360°
2 2
Exports (Rs ) 293366.75 375339.53 454799.97
Exports (Dollars ) 6384.30 8353.59 10272.51 Þ B = 132°; A = 132° – 26° = 106°
Imports (Rs ) 359107.66 501064.54 630526.77
Imports (Dollars ) 7814.96 11151.74 14241.63 82. (a) Replace ‘on’ by ‘at’.
Trade Imbalance 1430.66 2798.15 3969.12 83. (d) Replace ‘did lack’ by ‘lacked’.
Growth 95.58 41.85
(86-90) . DEAFCB
75. (a) 121. (b) The union government appointed Usha Subramaniam
2003-04 2004-05 2005-06 as the first chairperson and managing director of
Total Imports 100.00 100.00 100.00 Bhartiya Mahila Bank.
Total Imports (in 359,107.66 501,064.54 630,526.77
Crore of Rupees)
Rs/ Dollar 45.95 44.93 44.27
Total Imports (in
Crore of Dollars) 7814.96 11151.74 14241.63
Petroleum crude & 26.70 27.87 30.87
pdts; weight(% )
Imports (1) 2086.59 3107.99 4396.39
Total Exports (in 6384.30 8353.59 10272.51
Crore of Dollars)
Petroleum pdts; 5.54 8.57 11.21
weight(%)
Exports (2) 353.69 715.90 1151.55
Difference (1) - (2) 1732.90 2392.09 3244.84
Ratio 1.36

www.newspaperkorner.wordpress.com
www.newspaperkorner.wordpress.com

Practice Set - 3

INSTRUCTIONS
• This practice set consists two parts. One part is Objective test and other part is Descriptive test.
• Each question has five options, of which only one is correct. The candidates are advised to read all the
options thoroughly.
• There is negative marking equivalent to 1/4th of the mark allotted to the specific question for wrong
answer.

Time : 3 hrs. Max. Marks : 200

REASONING AND COMPUTER APTITUDE 4. (i) 60 $ 20 % 4 = n


(ii) 8 @ n Ó 10 = ?
1. Which of the following symbols should replace the question (a) 20 (b) 40
mark in the given expression in order to make the expressions (c) 10 (d) 60
‘K £ H’ and ‘M > J’ definitely true? (e) None of these
H ³ I=J?K£L<M 5. (i) 15 · 12 l 5 = p
(a) > (b) ³ (ii) 4 $ 12 % p = ?
(c) £ (d) Either < or £ (a) 720 (b) 228
(e) = (c) 108 (d) 93
2. In which of the following expression will the expression (e) None of these
‘P > S’ be definitely false? 6. (i) 16 @ 12 Ó 8 = z
(a) P > Q ³ R = S (b) S £ R £ Q < P (ii) z # 20 · 8 = ?
(c) R = P > Q ³ S (d) S > Q ³ R < P (a) 240 (b) 80
(e) S < Q £ R < P (c) 120 (d) Can not be determined
Directions (Qs. 3-7) : In a certain instruction system the different (e) None of these
computation processor written as follows: 7. (i) 80 · 15 l 8 = t
(i) x $ y % z means z is multiplied by the sum of x and y. (ii) t $ 5 % 8 = ?
(ii) x # y * z means that when y is subtracted from x and the (a) 40 # 40 * 10 (b) 40 $ 40 % 10
resultant is divided by z. (c) 40 @ 40 Ó 10 (d) 40 · 40 l 10
(iii) x @ y Ó z means x is added to the resultant when y is
(e) None of these
divided by z.
Directions (Qs. 8-12): Study the following information carefully
(iv) x · y l z means x is subtracted from the product of y and z.
and answer the given questions:
In each of the questions below, a set of instruction sequence
is given. You are required to find out the outcome which A word and number arrangement machine when given an
should come in place of the question mark (?) in each of the input line of words and numbers rearranges them following a
given sets of sequence. particular rule in each step. The following is an illustration of
3. (i) 20 # 10 * 2 = m input and rearrangement.
(ii) m · 6 l 4 = ? Input : day 74 night 36 25 68 all for
(a) 19 (b) 29 Step I : all day 74 night 36 25 68 for
(c) 4 (d) Cannot be determined Step II : all 74 day night 36 25 68 for
(e) None of these Step III : all 74 day 68 night 36 25 for
Stepwww.newspaperkorner.wordpress.com
IV : all 74 day 68 for night 36 25
36 Practice Set - 3
www.newspaperkorner.wordpress.com
Step V : all 74 day 68 for 36 night 25 14. Which of the following among (A), (B), (C) and (D) is the
and Step V is the last step of the rearrangement of the above cause of success?
input. (a) Only (A) (b) Only (B)
As per the rules followed in the above steps, find out in (c) Only (C) (d) Only (D)
each of the following questions the appropriate step for the (e) None of the above
given input. 15. Which of the following (C), (D), (E) and (F) is a weak
8. Step III of an input : bond 86 goal 12 33 like high 46. argument in favour of successful man?
Which of the following will be step VII? (a) Only (C) (b) Only (D)
(a) bond 86 goal 46 like 12 33 high. (c) Only (E) (d) Only (F)
(b) bond 86 goal 46 high 33 12 (e) Both (E) and (F)
(c) bond 86 goal 46 high 33 like 12. 16. Whether we look at the intrinsic value of our literature, or at
(d) There will be no such step. the particular situation of this country, we shall see the
strongest reason to think that of all foreign tongues the
(e) None of these
English tongue is that which would be the most useful to
9. Input : mind new 27 35 19 59 own tower.
our native subjects.
Which of the following steps will be the last but one? It can be inferred that
(a) VI (b) IV (a) The speaker is a die- hard colonist
(c) V (d) VII (b) The speaker has the good of the nation at heart
(e) None of these (c) The speaker is addressing an issue related to a colonial
10. Step IV of an input : dear 63 few 51 16 29 yrs now. empire
How many more steps will be required to complete the (d) None of the above
arrangement? (e) All of these
(a) Four (b) Five 17. Statement: But because the idea of private property has
(c) Three (d) Two been permitted to override, with its selfishness, the common
(e) None of these good of humanity, it does not follow that there are not
11. Step II of an input is : car 73 18 18 25 wear 49 long for which limits within which that idea can function for the general
of the following is definitely the input? convenience and advantage. Which of the following is
most likely to weaken the argument?
(a) 18 25 wear 49 long for car 73
(a) All the people of the society should progress at an
(b) 73 18 car 25 wear 49 long for
equitable rate and there should be no disparities and
(c) 18 73 25 car wear 49 long for private property does bring about a tremendous
(d) Cannot be determined disparity.
(e) None of these (b) One should not strive for the common good of
12. Input: was 52 and peace 43 16 now 24. humanity at all, instead one should be concerned with
How many steps will be required to complete the maximising one’s own wealth.
rearrangement? (c) One should learn from the experiences of former
(a) Four (b) Five communist nations and should not repeat their
(c) Six (d) Seven mistakes at all.
(e) None of these (d) Even prosperous capitalist countries like the USA
have their share of social problems.
Directions (Qs. 13-15) : Study the following information carefully
and answer the given questions. (e) None of these
The successful man has the ability to judge himself correctly. Directions (Qs. 18-23): Study the following information carefully
and answer the given questions.
(A) Inability to judge correctly causes failure.
Eight friends Q, R, S, T, V, W, Y and Z are sitting around a
(B) To judge others is of no use to a successful man.
circular table, facing the centre, There are three males and five
(C) The successful man cannot make a wrong judgement. females in the group of friends. No two males are immediate
(D) Hard-working is the key of success. neighbours of each other.
(E) A successful man can not judge others. · V sits second to the right of his wife.
(F) A successful man does not look in to the future. · S sits third to the right of V.
13. Which of the following among (A), (B), (C) and (D) is implicit · W sits second to the right of her husband Z. Z is not an
in the information given above? immediate neighbour of V’s wife.
(a) Only (A) (b) Only (B) · T is a male and Y is not an immediate neighbour of V.
(c) Only (C) (d) ONly (D) · R sits second to the right of Q.
(e) (A), (B ) and (C) 18. What is the position of T with respect to Z?
(a) Second to the left
www.newspaperkorner.wordpress.com
Practice Set - 3 37
www.newspaperkorner.wordpress.com
(b) Immediately to the right Give answer (a) if only conclusion I follows.
(c) Third to the left Give answer (b) if only conclusion II follows.
(d) Second to the right Give answer (c) if either conclusion I or conclusion II follows.
(e) Third to the right Give answer (d) if neither conclusion I nor conclusion II follows.
19. Which of the following statements regarding S is definitely Give answer (e) if both conclusion I and conclusion II follow.
correct? 24-25 Statements: Some institutes are banks. All institutes are
(a) S is one of the male members of the group. academies. All academies are schools.
(b) Both the immediate neighbours of S are females. 24. Conclusions:
(c) S sits third to the left of T. I. Some institutes are not schools.
(d) W is an immediate neighbour of S. II. All academies being banks is a possibility.
(e) S sits second to the right of Q. 25. Conclusions:
20. Who amongst the following is V’s wife? I. All banks can never be schools.
(a) Q (b) Y II. Any bank which is an institute is a school.
(c) R (d) T 26-27 Statements: All energies are forces. No force is torque. All
(e) None of these torques are powers.
21. Who amongst the following has a male sitting to the 26. Conclusions:
immediate left and the right? I. All energies being power is a possibility.
(a) Y (b) R II. All powers being force is a possibility.
(c) Q (d) S 27. Conclusions:
(e) None of these I. All those powers if they are forces are also energies.
22. Which of the following is not true regarding T? II. No energy is torque.
(a) T is an immediate neighbour of Z’s wife. 28. Statements: All circles are squares. Some squares are
(b) No male is an immediate neighbour of T. rectangles.
(c) Q sits second to right of T. Conclusions:
(d) The one who sits third to the left of T is a male. I. All rectangles being squares is a possibility.
(e) All are true. II. All circles being rectangles is a possibility.
23. Which of the following pairs represents the immediate 29. Statements: No gadget is a machine. All machines are
neighbours of T? computers.
(a) RQ (b) WZ Conclusions:
(c) YV (d) WY I. No computer is a gadget.
(e) None of these II. All computers being gadgets is possibility.
Directions (Qs. 24-30): In each question below are two/three 30. Statements: Some paintings are drawings. All sketches are
statements followed by two conclusions numbered I and II. You paintings.
have to take the two/three given statements to be true even if
Conclusions:
they seem to be a variance with commonly known facts and then
decide which of the given conclusions logically follows from the I. All sketches are drawings.
given statements disregarding commonly known facts. II. Some sketches being drawings is a possibility.

Directions (Qs. 31-35): In each of the following questions, a related pair of figures is followed by five lettered pairs of figures. Select
the pair that has relationship similar to that in the question figure. The best answer is to be selected from a group of fairly close
choices.
31.

I II I II I II I II I II I II
(a) (b) (c) (d) (e)
32.

I II I II I II I II I II I II
(a) (b) (c) (d) (e)
33.

I II I II I II I II I II I II
(a) (b) (c) (d) (e)

www.newspaperkorner.wordpress.com
38 Practice Set - 3
www.newspaperkorner.wordpress.com
34.

I II I II I II I II I II I II
(a) (b) (c) (d) (e)
35.

I II I II I II I II I II I II
(a) (b) (c) (d) (e)

36. The database environment has all of the following 41. Communication between a computer and a CPU uses
components except ________ transmission mode.
(a) database (b) DBA (a) automatic (b) simplex
(c) users (d) SQL (c) half-duplex (d) full-duplex
(e) separate files (e) None of these
37. A number of related records that are treated as a unit is 42. ISDN is an acronym for
called a
(a) Integerated Standard Digital Network
(a) field (b) data
(b) Intelligent Services Digital Network
(c) file (d) batch
(c) Integerated Services Digital Network
(e) group (d) Integrated Services Data Network
38. Disk checking is associated with (e) None of these
(a) debugging a program (b) running a program 43. In MS Excel, to display current date only _____ is used.
(c) compiling a program (d) coding program (a) date () (b) now ()
(e) assembling a program (c) today () (d) time ()
39. What is the common name given to a program written in (e) current date ()
any one of the high-level languages? 44. To rename currently highlighted object ______ is used.
(a) System program (b) Compiler program (a) F1 (b) F2
(c) Object program (d) Source program (c) F3 (d) F5
(e) Data program (e) F6
40. A group of magnetic tapes, videos or terminals usually 45. Which generation is natural language related to?
under the control of one master is called a
(a) First generation (b) Second generation
(a) cluster (b) track
(c) Third generation (d) Fourth generation
(c) cylinder (d) All the above (e) Fifth generation
(e) None of these

DATA ANALYSIS & INTERPRETATION


Directions (Qs. 46-50): Study the following graph and table carefully and answer the questions given below them.

40
Percentage of Obese men, Obese
women and Obese children

35
30
25 Obese men
20 Obese women

15 Obese children

10
5
0
2010 2011 2012 2013 2014 2015
Years

Total Number of Men, Women and Children in the state over the years
www.newspaperkorner.wordpress.com
Practice Set - 3 39
www.newspaperkorner.wordpress.com 52. How many candidates have prior experience of working in
Years Men Women Children Public Sector Banks (Urban and Rural areas together)?
2010 54, 000 38, 000 15, 000 (a) 12,450 (b) 8,400
2011 75, 000 64, 000 21, 000 (c) 10,050 (d) 10,650
2012 63, 000 60, 000 12, 000 (e) None of these
2013 66, 000 54, 000 16, 000 53. What is the ratio of the candidates who have a prior
experience of working in Public Sector Banks in rural areas
2014 70, 000 68, 000 20, 000 only to the candidates who have a prior experience of
2015 78, 000 75, 000 45, 000 working in Private Sector Banks in rural areas only?
(a) 4 : 3 (b) 3 : 2
46. What was the approximate average of obese men, obese
(c) 2 : 3 (d) 3 : 4
women and obese children in 2013?
(e) None of these
(a) 12,683 (b) 12,795
54. What is the total number of candidates who have worked in
(c) 12,867 (d) 12,843 Private Sector Banks in urban areas?
(e) 12,787 (a) 1,800 (b) 2,250
47. The number of obese men in the year 2015 was what per
(c) 4,050 (d) 36,600
cent of the men not suffering from obesity in the same year?
(e) None of these
(a) 55 (b) 60
55. The candidates who have no prior experience of working in
(c) 50.5 (d) 65.5
the banking industry are what per cent of the candidates
(e) None of these
who have worked in Public Sector Banks in both urban and
48. What was the ratio of the obese women in the year 2014 to rural areas together?
the obese men in the year 2014?
(a) 60.5 (b) 63.5
(a) 6 : 7 (b) 21 : 65
(c) 62 (d) 64
(c) 15 : 73 (d) 48 : 77
(e) None of these
(e) None of these
Directions (Qs. 56-60): Study the following pie-chart carefully
49. What is the difference between the number of obese women
and obese children together in the year 2012 and the number to answer these questions.
of obese men in the same year? Total number of passengers = 8500
(a) 5,475 (b) 5,745 Percentage of passengers
(c) 4,530 (d) 31,650
(e) None of these
50. What was the total number of children not suffering from Train-A
Train-Q
obesity in the year 2010 and 2011 together? 13%
19%
(a) 4,350 (b) 31,560 Train-L
Train-R
(c) 4,530 (d) 31,650 15%
9%
(e) None of these
Directions (Qs. 51-55): Study the following information carefully Train-M
Train-S
and answer the questions given below it. 20%
24%
Out of the 15,000 candidates eligible for an Officer’s post in
a Public Sector Bank, 450 candidates have prior experience of
working in Public Sector banks in rural area only. 25% of the total
number of candidates have prior experience of working in Public 56. What was the approximate average number of passenger
Sector Banks in urban areas only. 12% of the total number of in Train-S, Train-M and Train-L together?
candidates have prior experience of working in Private Sector (a) 1521 (b) 1641 (c) 1651 (d) 1671 (e) 1691
Banks in urban areas only. 2% of the total number of candidates 57. If in Train-R 34 per cent of the passengers are females and
have prior experience of working in Private Sector banks in rural 26 per cent are children, what is the number of males in that
areas only. 3,600 candidates have worked in both Public and
train?
Private Sector Banks in urban areas only. 600 candidates have
worked in both Public and Private Sector Banks in rural areas (a) 306 (b) 316
only. The remaining candidates have no prior experience of (c) 308 (d) 318
working in the Banking industry. (e) None of these
51. How many candidates have prior experience of working in 58. The number of passengers in Train-Q is approximately what
rural areas (both Public Sector and Private Sector Banks percentage of the total number of passengers to Train-A
together)? and Train-R?
(a) 4,350 (b) 4,950
(a) 90 (b) 70 (c) 75 (d) 80 (e) 86
(c) 4,800 (d) 4,900
(e) 4,850
www.newspaperkorner.wordpress.com
40 Practice Set - 3
www.newspaperkorner.wordpress.com
59. Which train has the second highest number of passengers? (a) 7 : 5 (b) 5 : 7
(a) A (b) Q (c) S (d) M (e) L (c) 16 : 15 (d) 14 : 16
60. How many more per cent (approximately) number of (e) None of these
passengers are there in Train-M as compared to the number 63. The Graduate female population of State C is what per cent
of passengers in Train-L? of the Std XII population of all the states together?
(a) 29 (b) 49 (c) 43 (d) 33 (e) 39 (a) 40 (b) 62.5
Directions (Qs. 61-65): Study the following pie-charts and table (c) 50 (d) 52.5
to answer these questions.
(e) None of these
State wise Details of the adult population of a country
64. The Std XII male population of State C is what per cent of
Graduate and above
the total Std XII population of all the states together?
Total no. = 24
(a) 8% (b) 12%
(c) 11% (d) 9%
F 14%
E 20% (e) None of these
65. What is the ratio of the Graduate male population of State E
A 16% Std XII female population of that State?
D 18% (a) 28 : 35 (b ) 35 : 28
B 18% (c) 32 : 45 (d) 45 : 32
C 18% (e) None of these
66. Total Graduate population of State F is what per cent of the
total Std XII population of State A?
Upto XII STD pass (a) 56 (b) 72
Total no. = 32 lakhs
(c) 68 (d) 76
(e) None of these
67. Std XII male population of state E is what per cent of the
E 19% F 20% Std XII male population of State F?
(a) 70 (b) 75
D 12% (c) 68 (d) 72
A 15%
(e) None of these
68. What is the ratio of the total Graduate and Std XII male
C 18% population of State A to the total Graduate and Std XII
B 16%
female population of that State?
Male Female (M : F) Ratio (a) 215 : 216 (b) 214 : 215
Graduate & Above Upto XII Std Pass (c) 217 : 215 (d) 215 : 217
State (e) None of these
M : F M : F
A 7 :5 7 : 9 69. What is the ratio of the total Graduate population of State D
B 5 :3 3 : 5 to the total Std XII population of that State?
C 5 :4 4 : 5 (a) 17 : 16 (b) 16 : 17
D 9 :8 7 : 7 (c) 64 : 51 (d) 51 : 64
E 9 :7 9 : 10 (e) None of these
F 4 :3 3 : 2
70. The Graduate female population of State B is what per cent
61. What is the difference between the Graduate male population of the Graduate female population of State E? (rounded off
and XII Std male population from State ‘A’?
to the nearest integer)
(a) 24,000 (b) 14,000
(a) 129 (b) 82
(c) 28,000 (d) 36,000
(c) 77 (d) 107
(e) None of these
62. What is the ratio of the Graduate female population of State (e) None of these
E to Std XII female population of State D?

www.newspaperkorner.wordpress.com
Practice Set - 3 41
www.newspaperkorner.wordpress.com
Directions (Qs. 71-75) : These questions are based on the graph given below.
Graph indicates the annual sales tax revenue collections (in Rupees in crores) of seven states from 1996 to 2001. The values
given at the top of each bar represents the total collections in that year.
49638
50000
WB West Bengal
45000 42348 UP Uttar Pradesh
40000
36068
35000 33168 TN Tamil Nadu
29870
30000
25000 MA Maharashtra
20000
15000 KA Karnataka
10000 GU Gujarat
5000
AP Andhra Pradesh
0
2010-11 2011-12 2012-13 2013-14 2014-15

71. If for each year, the states are ranked in terms of the (a) 2011 to 2012 (b) 2012 to 2013
descending order of sales tax collections, how many states (c) 2013 to 2014 (d) 2014 to 2015
don’t change the ranking more than once over the five years 75. Identify the state whose tax revenue increased exactly by
(a) 1 (b) 5 the same amount in two successive pair of years?
(c) 3 (d) 4 (a) Karnataka (b) West Bengal
(e) None of these (c) Uttar Pradesh (d) Tamil Nadu
72. Which of the following states has changed its relative (e) None of these
ranking most number of times when you rank the states in Directions (Qs. 76 to 80) : These questions are based on the
terms sof the descending volume of sales tax collections tables and information given below.
each year? Mulayam Software Co., before selling a package to its
(a) Andhra Pradesh (b) Uttar Pradesh clients, follows the given schedule :
(c) Karnataka (d) Tamil Nadu
(e) None of these Month Stage Cost (Rs.'000
73. The percentage share of sales tax revenue of which state per man-month)
has increased from 2011 to 2015?
1-2 Specification 40
(a) Tamil Nadu (b) Karnataka
3-4 Design 20
(c) Gujarat (d) Andhra Pradesh
5-8 Coding 10
(e) None of these
9 - 10 Testing 15
74. Which pair of successive years shows the maximum growth
rate of tax revenue in Maharashtra? 11 - 15 Maintenance 10
The number of people employed in each month is:
Month 1 2 3 4 5 6 7 8 9 10 11 12 13 14 15
No.of people employed 2 3 4 3 4 5 5 4 4 1 3 3 1 1 1

76. Due to overrun in “Design”, the Design stage took 3 months, 78. Under the new technique, which stage of Software
i.e. months 3, 4 and 5. The number of people working on development is most expensive for Mulayam Software
Design in the fifth month was 5. Calculate the percentage company?
change in the cost incurred in the fifth month.(Due to (a) Testing (b) Specification
improvement in “Coding” technique, this stage was (c) Coding (d) Design
completed in months 6 - 8 only ) (e) None of these
(a) 225% (b) 150% 79. Which five consecutive months have the lowest average
(c) 275% (d) 240% cost per man-month under the new technique?
(e) None of these (a) 1- 5 (b) 9 - 13
(c) 11 - 15 (d) 5 - 8
77. With reference to the above question, what is the cost
incurred in the new “coding” stage ? (Under the new (e) None of these
technique, 4 people work in the sixth month and 5 in the 80. What is the difference in the cost between the old and the
eighth) new techniques?
(a) ` 1,40,000 (b) `1,50,000 (a) `30,000 (b) `60,000
(c) ` 1,60,000 (d) ` 1, 80,000 (c) `70,000 (d) ` 40,000
(e) None of these (e) None of these
www.newspaperkorner.wordpress.com
42 Practice Set - 3
www.newspaperkorner.wordpress.com (a) Danger of famine
ENGLISH LANGUAGE
(b) Scarcity of water
Directions (Qs. 81-85) : Read the following passage carefully
(c) Scarcity of food grains
and answer the questions given below. Certain words are printed
in bold to help you locate them answering some of the questions. (d) Report of the international task force on conservation
of forests
A nine-membered international task force on conservation
(e) None of these
of forests has identified India as one of the 56 countries of the
world which are critically affected by widespread deforestation 84. The international task force has not
and steady destruction of natural watersheds. It has also warned (a) suggested a comprehensive scheme for dealing with
that unless urgent conservation measures are taken, the country the problem of deforestation
will face a serious food crisis in the next century. The report (b) emphasised the urgency of tackling the problem
correctly describes the Himalayan eco system as the most critical (c) underestimated the magnitude of problem of
watershed in the world, the degradation of which now threatens deforestation
the very processes of life in the Indo-Gangetic plain. The report (d) underestimated the intensity of the problem of the
documents some aspects of the process of environmental natural watersheds
destruction that is at work in this country and calls for an (e) None of these
expenditure of about `1,450 crores over the next five years to Directions (Q. 85) : Select the word which is SAME in meaning
reverse some of the damage that has taken place. While the task as the word given in bold as used in the passage.
force’s report is a welcome reminder of the urgency to the task of 85. Pace
halting and rolling back the most awesome form of destruction (a) Momentum (b) Rate
that this country has ever witnessed, it regrettably underestimates
(c) Progress (d) Gain
the magnitude of the effort involved and hence falls short of
suggesting a comprehensive solution to the problem. For instance, (e) Measure
the five-year plan. Its recommendation bears no relationship to Directions (Q. 86) : Select the word which is OPPOSITE in
the resources that will be actually needed to reach a net rate of meaning of the word given in bold as used in the passage.
zero deforestation. At present deforestation is proceeding at the 86. Critical
furious pace of 1.3 million hectares a year. (a) Substantial (b) Negligible
Past experience shows that more than ` 300 crores in (c) Secondary (d) Large
government expenditure alone is needed over four to five years (e) Unduly
at today’s prices merely to plant less than a million hectare. Directions (Qs. 87-90) : In each of the following sentences there
Realistically speaking, resources of the order of ` l,000 crores a are blank spaces. Below each sentence there are five pairs of
year will be necessary for the next five years to sustain a plan that words denoted by the numbers a, b, c, d and e. Find out which pair
actually provides a forest cover to badly denuded land and an of words can be filled up in the blanks to make the sentence
underestimation of the magnitude of such a plan is not the only meaningfully complete.
flaw in the report. An even more serious one, related to the first, 87. The fear ________ with feeling incompetent is the fear of
lies in its failure to establish a coherent set of priorities among the being humiliated, embarrassed and ________ .
different functions that an afforestation programme must perform. (a) endowed, criticised (b) afflicted, downtrodden
To take just one example, the vital task of planting fuel wood and (c) consistent, damaged (d) associated, vulnerable
fodder trees is put at par with the planting of trees to meet raw (e) imbued, exposed
materials requirement of industries. Owing to confusion, the task
88. If a professional ________ his professional development
force falls short of recommending the kind of radical measures.
and ________ education, he may quickly become obsolete.
Wood-based in dustries step against the spreadin g of
monocultures and entrusting afforestation programmes. (a) management, voluminous
(b) higher, vulnerable
81. The author’s chief concern appears to be –
(c) neglects, better
(a) conservation of forests
(d) refuses, higher
(b) destruction of natural watersheds
(e) denies, superior
(c) degradation of Himalayan eco systems
89. This book is readable, clear and ________ researched with
(d) to take radical measures for afforestation
________ detailed references.
(e) to discuss the report of international task force
(a) fabulously, intricate (b) meticulously, extensive
82. In order to reverse the damage caused by deforestation,
(c) leisurely, complete (d) hardly, national
India, in a period of five years, will be required to incur a
total expenditure of about 7. (e) closely, scattered
(a) 300 crores (b) 1000 crores 90. The Indian hospitality industry, which has been ________
a prolonged slump, is now entering a new ________ phase.
(c) 1450 crores (d) 2450 crores
(a) witnessing, ambitious (b) observing, listless
(e) 5000 crores
(c) recovering, debt (d) demonstrating, efficient
83. What seems to be the immediate cause for writing the
passage? (e) succumbing, lean
www.newspaperkorner.wordpress.com
Practice Set - 3 43
www.newspaperkorner.wordpress.com
Directions (Qs. 91-95) : The following questions consist of a (a) If one pray (b) One if prays
single sentence with one blank only. You are given six words (c) If one will praying (d) If one prayed
denoted by A, B, C, D, E & F as answer choices and from the six (e) No correction required
choices you have to pick two correct answers, either of which 97. Not only I but also my parents are looking forward to see
will make the sentence meaningfully complete. you soon.
91. ___________ before the clock struck 8 on Saturday night, (a) am looking forward to seeing
India Gate was swamped with people wearing black tee- (a) are looking to see forward
shirts and holding candles. (c) are looking forward seeing
(A) Minutes (B) Time (d) are looking forward to seeing
(C) Later (D) Quickly (e) no correction required
(E) Since (F) Seconds 98. They are waiting for her since morning.
(a) (B) and (E) (b) (A) and (C) (a) are waiting (b) are awaiting to
(c) (A) and(F) (d) (B) and (D) (c) have been a waiting (d) have been waiting for
(e) (C) and (E) (e) No correction required
92. The States should take steps to________ the process of 99. If he had asked me, I would have helped him.
teachers appointments as the Centre has already sanctioned (a) If he ask (b) had he asked
six lakh posts. (c) He had asked (d) If he asked
(A) fasten (B) move (e) No correction required
(C) hasten (D) speed 100. I am sure that scientists must discover a cure for AIDS by
(E) early (F) quicken the end of next decade.
(a) (D) and (F) (b) (A) and (C) (a) will have discovered (b) might discovered
(c) (C) and (F) (d) (D) and (E) (c) have discovered (d) must discover
(e) (B) and (D) (e) No correction required
93. A senior citizen’s son __________ threatened her every Directions (Qs. 101-105) : Rearrange the sentences given below
day and physically harmed her, forcing her to transfer her in a way which would make an appropriate a meaningful
property to him. paragraph. Mark the correct order of sentence A, B, C, D, E, F
(A) superficially (B) mistakenly and answer the questions given below.
(C) allegedly (D) miserably A. On my way back home from school, I saw a crowd of people
(E) doubtfully (F) purportedly in front of a house.
(a) (C) and (F) (b) (A) and (E) B. The women of the families were also taking part in the
(c) (C) and (E) (d) (D) and (F) quarrel.
(e) (A) and (C) C. However, I soon found that it was a quarrel between a
94. Medical teachers said that the management had continued landlord and his tenant.
to remain _________ to their cause leading to the stretching D. At first I could not make out as to what was happening.
of their strike. E. Even the children didn't kg behind and were throwing pots
(A) unmoved (B) lethargic and pans.
(C) unconcerned (D) apathetic F. Both were shouting at the top of their voices.
(E) indifferent (F) bored 101. After rearrangement which of the following sentences will
(a) (B) and (C) (b) (C) and (F) come ‘FIRST’?
(c) (A) and (E) (d) (A) and (D) (a) A (b) B (c) C (d) D (e) F
(e) (D) and (E) 102. After rearrangement which of the following sentences will
95. The parents had approached the high court to ______ the come ‘LAST’?
government _________order after their children, who (a) A (b) B (c) C (d) E (e) F
passed UKG, were denied admission by a school. 103. After rearrangement which of the following sentences will
(A) void (B) quash come ‘SECOND’?
(C) annual (D) stay (a) B (b) C (c) D (d) E (e) F
(E) lift (F) post 104. After rearrangement which of the following sentences will
(a) (A) and (D) (b) (B) and (C) come ‘THIRD’?
(c) (C) and (E) (d) (E) and (F) (a) A (b) B (c) C (d) D (e) E
(e) (C) and (D) 105. After rearrangement which of the following sentences will
Directions (Qs. 96-100) : Replace the words given in bold by one come FOURTH?
of the given options to make the sentence correct and meaningful. (a) A (b) C (c) D (d) E (e) F
In case of ‘no correction required’, (e) will be the answer. Directions (Qs. 106-115) : In the following passage some of the
96. If one prays honestly and sincerely, God will listen to one’s words have been left out, each of which is indicated by a number.
prayer. Find the suitable word from the options given against each number
www.newspaperkorner.wordpress.com
44 Practice Set - 3
www.newspaperkorner.wordpress.com
and fill up the blanks with appropriate words to make the 118. Under which Article of the Constitution, the two States of
paragraph meaningful. Telangana and Andhra Pradesh will have special status for
Man has been tampering (106) ecosphere for a very (107) equitable opportunities for education and public
time and is forced to (108) that the environmental resources are employment
(109). Environmental problems are (110) social problems. They (a) Article 2 of the Constitution of India
begin with people as (111) and end with people as victims. (b) Article 239 A of the Constitution of India
Unplanned uses of resource (112) in the depletion of fossil fuels
(c) Article 358 of the Constitution of India
and (113) of air and water, Deforestation has led to (114) imbalance
(d) Article 371-D of the Constitution of India
and draining away of national wealth (115) heavy expenditure on
oil and power generation. (e) None of these
106. (a) on (b) in (c) with (d) for (e) from 119. Which one of the following is not a "Navaratna" company?
(a) Bharat Electronics Limited
107. (a) short (b) long (c) small (d) tall (e) high
108. (a) see (b) look (b) Indian Oil Corporation
(c) do (d) recognise (c) Airports Authority of India
(e) realise (d) Rural Electrification and Corporation Limited
109. (a) plenty (b) scarce (e) None of these
(c) minute (d) enough 120. Which of the following has become the first airport in the
World to receive the prestigious “Sword of Honour” for
(e) minimum
safety management?
110. (a) really (b) coldly
(a) Kempegowda International Airport
(c) badly (d) happily
(b) Kochi International Airport
(e) seriously
(c) Indira Gandhi International Airport
111. (a) effect (b) result
(d) Rajiv Gandhi International Airport
(c) cause (d) wisdom
(e) None of these
(e) affect
121. POS data entry system is used most extensively by the
112. (a) cause (b) rest
(a) grocery industry
(c) consequence (d) result
(b) railroad industry
(e) get
(c) word-processing industry
113. (a) revolution (b) pollution
(d) banking industry
(c) resolution (d) evolution
(e) None of these
(e) solution
122. Which of the following statements is true?
114. (a) ecological (b) biological
(a) Marketing makes the company loose money due to
(c) logical (d) chronological
high cost.
(e) geographical
(b) Marketing is not important in profit- making companies
115. (a) by (b) in
(c) Marketing sharpens the mind set of the employees
(c) out (d) through
(d) Marketing is a time-bound seasonal function
(e) from
(e) Marketing is a waste
GENERAL/ECONOMY/BANKING AWARENESS 123. In how many languages does RBI prints the currency.
(a) 12 (b) 14
116. RBI increased the validity period of the in-principle approval (c) 15 (d) 16
of setting up of new banks from one year to (e) None of these
(a) 14 months (b) 16 months 124. CRR stands for
(c) 18 months (d) 20 months (a) Cash Rate Requirements
(e) None of these (b) Cash Reserve Ratio
117. The primary objective of the European Central Bank, as (c) Credit Rate Requirements
mandated in Article 2 of the Statute of the Bank, is (d) Credit Reserve Requirements
(a) to define and implement the monetary policy for the (e) None of these
Eurozone 125. Which of the prize is also known as the Alternative Nobel
(b) to conduct foreign exchange operations prize?
(c) to maintain price stability within the Eurozone. (a) Pulitzer prize
(d) to take care of the foreign reserves of the European (b) Magsaysay award
System of Central Banks and operation of the financial (c) Booker prize
markets. (d) Right livelihood award
(e) None of these (e) None of these
www.newspaperkorner.wordpress.com
Practice Set - 3 45
www.newspaperkorner.wordpress.com
126. Which country is not a member of SAARC (South Asian 135. Which of the following awards is given in the field of Science
Association for Regional Co-operation) ? & Technology?
(a) Myanmar (b) Sri Lanka (a) Kalinga Prize (b) Dhyanchand Award
(c) Bhutan (d) Maldives (c) Arjun Award (d) Moortidevi Award
(e) None of these (e) Shanti Swarup Bhatnagar Award
127. The book ‘Jhansi Ki Rani’ was written by 136. Which of the following book is written by British Nobel
(a) Devkinandan Khatri Prize-winning author Doris Lessing?
(b) Sharat Chand Chaudhary (a) Wolf hall (b) Sea of Poppies
(c) Vrindavanlal Verma (c) Silent Spring (d) The Grass is Singing
(d) Mahadevi Verma (e) None of these
(e) None of these 137. Major functions undertaken by merchant bankers are
128. ‘Big Bazaar’ chain of stores is run by which group? (a) Issue management
(a) Jindal Group (b) Reliance Group (b) Market maker in capital market
(c) Future Group (d) Aditya Birla Group (c) Capital structuring/restructuring
(e) None of these (d) All of the above
129. What does ‘Financial Inclusion’ mean? (e) None of these
(a) Allow the merger and acquisition of banks so that only 138. Which of the following is the first commercial bank in the
few big banks exist and continue to cater to the need country to launch a mutual fund ?
of corporate sector. (a) Bank of India (b) Canara Bank
(b) Providing insurance cover to each and every citizen (c) State Bank of India (d) Indian Bank
so that he/she can live a healthy and long life. (e) None of these
(c) Expanding the network of banks of such a way that 139. The term ‘mutual fund’ refers to
people from lower strata of society also get the benefit (a) contribution by the employees of a business enterprise
of services provided by banks in the provident fund scheme
(d) To manage banking operations smoothly and merge (b) fund created by commercial banks and other eligible
nationalised banks for further financial settlements. financial institutions for floating new shares in the
(e) None of these market to earn a higher profit
130. Green Banking means (c) the business of acquisition, holding managment,
(a) financing of irrigation projects by banks trading or disposal of securities participation
(b) development of forestry by banks certificates or any other instruments, income or growth
(c) financing of environment friendly projects by banks participation business and Unit Trust schemes
(d) Managing fishery by banks. (d) All of the above
(e) None of these (e) None of these
131. Who is the author of the book Patrons of the Poor : Caste 140. The money collected by a mutual fund is invested in
Politics and Policy Making in India? (a) real estate and plantation projects
(a) Narayan Lakshman (b) Greg Lindsay (b) capital market instruments such as shares, debentrures
(c) Khushwant Singh (d) Salman Rushdie and other securities
(e) None of these (c) gilt-edged securities only
132. What does SAPTA stand for: (d) government securities only
(a) South Asian Preferential Trade Agreement (e) None of these
(b) South Asian Post Trade Agreement 141. Functions of the mutual funds are
(c) SAARC Preferential Trade Agreement (a) collection of funds from the public
(d) SAARC Post Trade Agreement (b) investment of the resources raised in capital markets
(e) None of these (c) holding investment in trust and ensuring proper
133. With which one of the following activities are Golden Globe management of investment portfolio
Awards associated? (d) All of the above
(a) Journalism (b) Social work (e) None of these
(c) Peace initiatives (d) Films 142. The International Bank for Reconstruction and Development
(e) None of these (IBRD) is better known as
134. Which of the following schemes of the Govt. of India is (a) World Bank
associated with the health sector? (b) Asian Development Bank
(a) NRDP (b) IRDP (c) IMF
(c) ASHA (d) Bharat Nirman (d) It is known by its name
(e) All of these (e) None of these
www.newspaperkorner.wordpress.com
46 Practice Set - 3
www.newspaperkorner.wordpress.com
143. Which sister organization of the World Bank helps private 150. As per the Rail Budget 2016-2017, world's first bio-vacuum
activity in developing countries by financing projects with toilet developed by Indian Railways is being used in which
long-term capital in the form of equity and loans? Rajdhani train?
(a) Asian Development Bank (a) Howrah Rajdhani
(b) IMF (b) Swarna Jayanti Rajdhani
(c) International Developmental Association (c) August Kranti Rajdhani
(d) International Finance Corporation (d) Dibrugarh Rajdhani
(e) None of these (e) None of these
144. Which sister organization of the World Bank provides long
151. What is the theme of the Rail Budget 2016-2017?
term loans at zero interest to the poorest developing
(a) Reorganize, Restructure Rejuvenate Indian Railways:
countries?
'Chalo, Milkar Kuch Karen'
(a) Asian Development Bank
(b) IMF (b) Chalo, Milkar Kuch Naya Karen: 'Navinikaran,
(c) International Developmental Association Sashaktikaran, Shodh aur Vikas'
(d) International Finance Corporation (c) Nav Arjan, Nav Manak, Nav Sanranchna: 'Chalo, Milkar
(e) None of these Kuch Karen'
145. What does devaluation of a currency mean? (d) Reorganize, Restructure Rejuvenate Indian Railways:
(a) decrease in the internal value of money 'Nav Arjan, Nav Manak, Nav Sanranchna'
(b) decrease in the external value of money (e) None of these
(c) decrease both in the external and internal values of 152. The world's first mass dengue vaccination programme has
money been launched by which country?
(d) increase in the internal value of money (a) Israel (b) Philippines
(e) None of these (c) United States (d) Russia
146. Which of the following is known as plastic money? (e) None of these
(a) bearer cheques (b) credit cards 153. India's first Aadhaar enabled ATM has been launched by
(c) demand drafts (d) gift cheques which bank?
(e) None of these (a) State Bank of India (b) ICICI Bank
147. When did Regional Rural Banks start functioning in India? (c) DCB Bank (d) Axis Bank
(a) 1975 (b) 1947 (e) None of these
(c) 1956 (d) 1960
154. The National Maritime Day of India is celebrated on which
(e) None of these
day?
148. Which of the following correctly describes what sub-prime
(a) April 3 (b) April 4
lending is?
(1) Lending to the people with less than ideal credit status. (c) April 5 (d) April 6
(2) Lending to the people who are high value customers (e) None of these
of the banks. 155. What is/are the goal(s) of the Rail Budget 2016-2017?
(3) Lending to those who are not a regular customer (a) Reserved accommodation on trains available on
(a) Only 1 (b) Only 2 demand and time tabled freight trains.
(c) Only 3 (d) All (b) High end technology to improve safety record,
(e) None of these elimination of all unmanned level crossings and higher
149. Which of the following has the sole right of issuing paper average speed of freight trains.
notes in India? (c) Semi high speed trains running along the golden
(a) Union Government (b) Reserve Bank of India quadrilateral and zero direct discharge of human waste
(c) Ministry of Finance (d) Supreme Court (d) All of the above
(e) None of these (e) None of these

www.newspaperkorner.wordpress.com
Practice Set - 3 47
www.newspaperkorner.wordpress.com

Time : 30 min. Max. Marks : 50

1. Write a letter on any one of the following:-


(a) Write a letter to the Chairman of your Municipality about the bad condition of roads in your locality (20 marks)
(b) You are director of a small limited company engaged in construction business. Write a letter to your Bank requesting for
credit facilities for expanding business.
(c) Write a Letter to the Bank Complaining that their Automatic tellers are Frequently out of Service
2. Write a paragraph on any one of the following in not more than 150 words (10 marks)
(a) Rural Banking
(b) Cancer- a silent Killer
(c) If I were an astronaut
3. Write an essay on any one of the following in about 250 words. (20 marks)
(a) Economic slowdown in India.
(b) Black money- its causes and effects and eradication
(c) Unemployment compensation

www.newspaperkorner.wordpress.com
48 Practice Set - 3
www.newspaperkorner.wordpress.com
Answer Key
1 (e) 16 (d) 31 (e) 46 (c) 61 (b) 76 (b) 91 (a) 106 (c) 121 (a) 136 (d) 151 (a)
2 (d) 17 (a) 32 (b) 47 (b) 62 (e) 77 (a) 92 (b) 107 (b) 122 (c) 137 (d) 152 (b)
3 (a) 18 (e) 33 (c) 48 (d) 63 (c) 78 (d) 93 (c) 108 (e) 123 (c) 138 (c) 153 (c)
4 (b) 19 (d) 34 (e) 49 (a) 64 (a) 79 (c) 94 (b) 109 (b) 124 (b) 139 (c) 154 (c)
5 (a) 20 (b) 35 (d) 50 (d) 65 (e) 80 (b) 95 (c) 110 (a) 125 (d) 140 (b) 155 (d)
6 (d) 21 (b) 36 (e) 51 (b) 66 (e) 81 (e) 96 (e) 111 (c) 126 (a) 141 (d)
7 (d) 22 (e) 37 (c) 52 (d) 67 (b) 82 (c) 97 (d) 112 (d) 127 (c) 142 (a)
8 (c) 23 (d) 38 (a) 53 (b) 68 (c) 83 (d) 98 (d) 113 (b) 128 (c) 143 (d)
9 (e) 24 (b) 39 (d) 54 (c) 69 (a) 84 (a) 99 (e) 114 (a) 129 (c) 144 (c)
10 (c) 25 (b) 40 (a) 55 (e) 70 (c) 85 (b) 100 (a) 115 (d) 130 (c) 145 (b)
11 (d) 26 (a) 41 (b) 56 (d) 71 (b) 86 (c) 101 (a) 116 (c) 131 (b) 146 (b)
12 (c) 27 (b) 42 (c) 57 (a) 72 (b) 87 (d) 102 (d) 117 (c) 132 (a) 147 (a)
13 (e) 28 (a) 43 (c) 58 (e) 73 (d) 88 (c) 103 (c) 118 (d) 133 (d) 148 (a)
14 (d) 29 (d) 44 (b) 59 (d) 74 (c) 89 (b) 104 (c) 119 (b) 134 (c) 149 (b)
15 (e) 30 (b) 45 (e) 60 (d) 75 (a) 90 (a) 105 (e) 120 (d) 135 (e) 150 (d)

Answers & Explanations


1. (e) H ³ I = J = K £ L < M 6. (d) (i) 16 @ 12 Ó 8 = z
2. (d) From this expression we cannot deduce any relation 12
or, z = 16 +
between P and S. 8
3. (a) (i) 20 # 10 * 2 = m 3 35
or, z = 16 + =
or, m = (20 – 10) ¸ 2 2 2
(ii) z # 20 · 8 = ?
10
or, m = =5 7. (d) (i) 80 · 15 l 8 = t
2
(ii) 5 · 6 l 4 = ? or, t = (15 × 8) – 80
5–6×4 or, t = 120 – 80 = 40
Þ 6 × 4 – 5 = 19 (ii) t $ 5 % 8 = ?
4. (b) (i) 60 $ 20 % 4 = n or, ? = (40 + 5) × 8
or, n = (60 + 20) × 4 or, ? = 45 × 8 = 360
or, n = 80 × 4 = 320 Option (1)
(ii) 8 @ n Ó 10 = ? 40 # 40 é 10
or, (40 – 40) ¸ 10 = 0
320
or, ? +8 Option (2)
10
40 $ 40 % 10
= 32 + 8 = 40 or, (40 + 40) × 10 = 800
5. (a) (i) 15 · 12 l 5 = 45 Option (c)
or, p = 60 – 15 = 45 40@ 40 © 10
(ii) 4 $ 12 % p = ?
40
or, ? (4 + 12) × 45 or, + 40 = 4 + 40 = 44
10
or, ? = 16 × 45 = 720
www.newspaperkorner.wordpress.com
Practice Set - 3 49
www.newspaperkorner.wordpress.com
Option (d) property, within limits, can function for the general
40 · 40 l 10 advantage. Option (a) would weaken the argument as
or, 40 × 10 – 40 it talks about equitable growth and says that private
or, 400 – 40 = 360 property brings disparity.
(8-12): In the first step, the word that comes first in the 18-23:
alphabetical order shifts to the first in the alphabetical Z(+)
S(–) R(–)
order shifts to the left most position, while the
remaining line shifts rightward. In the next step, the
largest number shifts to the second position from left,
W(–) V(+)
pushing the remaining line rightward. This goes on
(wife of Z)
alternately till the words get arranged in an alphabetical
order and the numbers in a descending order at T(+) Q(–)
alternate positions. Y(–)
8. (c) Step III: bond 86 goal 12 33 like high 46 (wife of V)
24. (b) All institutes are academic. All academics are schools
Step IV: bond 86 goal 46 12 33 like high
= A + A = A = All institute are schools. Hence I does
Step V: bond 86 goal 46 high 12 33 like
not follow. Some institute are banks (I) ® conversion
Step VI: bond 86 goal 46 high 33 12 like ® Some banks are institute (I) + All institutes are
Step VII: bond 86 goal 46 high 33 like 12 academics = I + A =\ I = Some banks are academics.
9. (e) Input: mind new 27 35 19 59 own tower Hence the possibility of II exists.
Step I: mind 59 new 27 35 19 own tower 25. (b) Some banks are academics = All academics are schools
Step II: mind 59 new 35 27 19 own tower = I + A = I Some banks are schools. This does not lead
us to I. II follows because All institutes are school.
Step III: mind 59 new 35 own 27 19 tower
26. (a) No force is torque + All torques are powers = E + A =
Step IV: mind 59 new 35 own 27 tower 19
O* = Some powers are not forces. Hence the possibility
Hence step III will be that last but one.
of II is ruled out. All energies are forces + No force is
10. (c) Step IV: dear 63 few 51 16 29 yes now torque = A + E = E = No energy is torque + All torques
Step V: dear 63 few 51 now 16 29 yes are powers = E + A = O* = Some powers are not
Step VI: dear 63 few 51 now 29 16 yes energies. This does not rule out the possibility of I.
Step VII: dear 63 few 51 now 29 yes 16 27. (b) It follows from the solution to the above question. But
Hence 7 – 4 = 3 more steps will be required. I does not follow unless we know that All forces are
energies.
11. (d) We can’t proceed backward.
28. (a) The possibilities exit because we don’t have any
12. (c) Input: war 52 and peace 43 16 now 24
negative statements.
Step I: and war 52 peace 43 16 now 24
29. (d) No gadget is a machine + All machines are computer =
Step II: and 52 war peace 43 16 now 24
E + A = Q* = Some computers are not gadget. This
Step III: and 52 now war peace 43 16 24 rules out the possibility of II. As for I, it many be true
Step IV: and 52 now 43 war peace 16 24 but we can’t say so with certainty.
Step V: and 52 now 43 peace war 16 24 30. (b) There being no negative statements, the possibility
Step VI: and 52 now 43 peace 24 war 16 (conclusion II) exists but not certainty (conclusion I).
16. (d) The passage refers to the usefulness of the English 31. (e) From element I to II the design rotates through 180°
tongue and the intrinsic value of our literature. Since 32. (b) From element I to II the design is mirror image after
none of the choices infers anything from the passage, being rotated through 90° clockwise.
the correct option is (d). Although English is a colonial 33. (c) From element I to II the design is enclosed by another
language, the passage does not link English to the design.
colonial history. 34. (e) From element I to II the upper design encloses the
17. (a) The idea of the passage is that, the idea of private lower design.
www.newspaperkorner.wordpress.com
50 Practice Set - 3
www.newspaperkorner.wordpress.com
35. (d) From element I to II the design is divided into four Sector Banks in Urban Areas only
equal parts and the lower left part becomes shaded. = 1800 + 2250 = 4050
46. (c) Total number of obese men in 2013 55. (e) Number of candidate having no prior experience of
= 66000 × 35% = 23100 working in banking sector
Total number of obese women in 2013 = 15000 – (450 + 3750 + 300 + 1800 + 3600 + 600 + 2250)
= 54000 × 35% = 13500 = 15000 – 12750 = 2250
Total number of obese children in 2013
2250
= 16000 × 12.5% = 2000 Req. % = ´ 100 = 15%
15000
Required average = (32100 + 13500 + 2000) ¸ 3
56. (d) Required average number of passengers
= 38600 ¸ 3 = 12867
47. (b) Required percentage 1
= [(24 + 20 + 15) % of 8500]
3
78000 ´ 37.5%
´ 100 = 60%
78000 ´ 62.5% 1 8500 ´ 59
= ´ » 1671
48. (d) Required ratio 3 100

8500 ´ 9
60000 ´ 20% 57. (a) Number of passengers in Train R = = 765
= = 48 : 77 100
70000 ´ 27.5%
\ Number of males = (100 – 34 – 26)% of 765
49. (a) No, of obese women in 2012
765 ´ 40
= 20% of 60000 = 12000 = = 306
100
Number of obese children in 2012
= 25% of 12000 = 3000 19
Number of obese men in 2006 = 32.5% of 63000 = 20475 58. (e) Required per cent = (13 + 9 ) ´ 100 » 86

Required difference = 20475 – (12000 + 3000)


59. (d) It is clear from the pie - chart
= 20475 – 15000 = 5475
50. (d) Number of children not suffering from obesity in 2011 20 - 15
60. (d) Required percent = ´ 100 » 33
an d 2010 = 90% of 21000 + 85% of 15000 15
= 18900 + 12750 61. (b) Reqd difference
Total of these two equals of 31650.
7 7
(51-55): Distribution of officers in different categories is as = 24 × 105 × 16% × – 32 × 105 × 15% ×
12 16
follow: = 224000 – 210000 = 14000
62. (e) Reqd ratio
Pub. Pub. Pr i. Pr i. Pub.
Ru Ur Ru Ur ( R u + Ur ) 1 7
= 24 × ´ : 32 ´
12 7
´ = 25 : 64
450 3750 300 1800 3600 5 16 100 10
4
´ 15 ´ 24
9
Pub. + Pr i ( R u ) Pub. + Pr i ( U r ) 63. (c) Reqd % = 5
´100 = 50%
600 2250 ´18 ´ 32
9
52. (d) Total number of candidates. 4
´ 18% ´ 24
= 450 + 3750 + 3600 + 600 + 2250 = 10650 64. (a) Reqd % 9 ´100 = 8%
32
450 3 65. (e) Reqd ratio
53. (b) Reqd ratio = = 3: 2
300 2 9 1 10 19
= ´ ´ 24 : ´ ´ 32 = 27 : 32
Required ratio = 300 + 450 = 2 : 3 16 5 19 100

54. (c) Required number of candidates working in Private 14% of 24


66. (e) Reqd % 15% of 32 ´100 = 70%
www.newspaperkorner.wordpress.com
Practice Set - 3 51
www.newspaperkorner.wordpress.com
9 6300 - 4402
19% of 32 For Gujarat, required % = ´ 100 = 43%
19 ´100 = 75% 4402
67. (b) Reqd % = 3
20% of 32 ´ % share of A.P from 1997 - 2001 increased by
5
7202 - 4728
= ´ 100 = 52%
68. (c) Reqd ratio 4728

( graduate + XII ) male which is most among TN, KA, GU


A ® graduate + XII female
( ) 74. (c) Growth rate in 2013-2014

7 7 10,284 - 8067
24 ´16% ´ + 32 ´15% ´ = ´ 100 = 27.5% approx.
12 16 = 217 : 215 8067
5 9
24 ´16% ´ + 32 ´15% ´ which is maximum in comparision to other pair of years
12 16
69. (a) Reqd ratio = 24 × 17 : 32 × 12 75. (a) In KA revenue increased from 2012 to 2014
= 4839 – 4265 = 574
æ3 7 ö
70. (c) Reqd % = ç ´ 18 / ´ 20 ÷ ´100 & revenue increase from 2014 to 2015
è8 16 ø
= 5413 – 4839 = 574
= 77% 76. (b) As per the plan, number of men working in 5th month
71. (b) was 4 and these 4 men were supposed to do coding.
State Rank
Cost per man- month for coding = `10, 000.
MA 1 1 1 1 1
TN 2 2 2 2 2 Total cost in 5th month = 4 ×10,000 = `40,000/- Number
GU 3 4 4 4 5 of people actually working in 5th month is 5 & these 5
AP 4 3 3 3 3 men are doing the design part of the project. Cost per
KA 5 6 6 6 6 man - month for design = `20,000
UP 6 5 5 5 4
Total cost in 5th month = 5 ×20,000 = `1,00,000,
WB 7 7 7 7 7
1, 00, 000 - 40, 000
We see from table the required states are five MA, TN, % change = ´ 100 = 150%
AP, KA, WB 40,000

72. (b) 77. (a) Total man months required for coding
State Rank
MA 1 1 1 1 1 = (4 + 5 + 5) = 14
TN 2 2 2 2 2
Cost per man month coding = `10,000
GU 3 4 4 4 5
AP 4 3 3 3 3 Total cost incurred in new coding stage
KA 5 6 6 6 6 = 14 ×10,000 = `1,40,000/-
UP 6 5 5 5 4
78. (d) Total cost in a stage = ( No. of man months) × ( cost per
WB 7 7 7 7 7
man month in that stage )
From above table we see that GU & UP are the states Total cost in specification = (2 + 3)40,000 = ` 2,00,000
who change their relative rank two times but GU is not Total cost in design = (4 + 3 + 5)20, 000 = `2,40,000
given in the options hence U.P will be the required
state Total cost in coding = ` 1,40,000 (from previous Q.)
73. (d) For Tamil Nadu, Total cost in testing = = (4 + 1)15000 = `75000

8015 - 5604 Hence design is the most expensive stage.


required % = ´ 100 = 43%
5604
79. (c) Average cost/man month = total cost in that period
For Karnataka, No.of man months taken

5413 - 3829 Average cost per man month will be minimum for 11-15
required % = ´ 100 = 41% month = `10, 000.
3829
www.newspaperkorner.wordpress.com
52 Practice Set - 3
www.newspaperkorner.wordpress.com
80. (b) 125. (d) Right Livelihood prize is considered to be the
Month 3 4 5 6 7 8
alternative to Nobel prize. This is also an annual prize
Old 4 3 4 5 5 4
which is given to individuals who have done
New 4 3 5 4 5 5
something exemplary to solve some of the most
The difference is in the 5th, 6th and the 8th month pressing issues in the world.
Cost under old technique in these months 127. (c) Jhansi Ki Rani is written by Vrindavanlal Verma.
= (4 + 5 + 4) × 10,000 = 1,30,000/- Vrindavan Lal Verma is the acclaimed author of various
Cost under new tehnique = 5 × 20,000 + (4 + 5) × 10,000 books including a National Award winning book titled
= ` 1,90,000/- “Mrignayani”.
Hence the difference = 1,90,000 – 1,30,000 = `60,000/-

www.newspaperkorner.wordpress.com
www.newspaperkorner.wordpress.com

Practice Set - 4

INSTRUCTIONS
• This practice set consists two parts. One part is Objective test and other part is Descriptive test.
• Each question has five options, of which only one is correct. The candidates are advised to read all
the options thoroughly.
• There is negative marking equivalent to 1/4th of the mark allotted to the specific question for wrong
answer.

Time : 3 hrs. Max. Marks : 200

REASONING ABILITY 6. Which of the following is the tenth to the right of the
AND COMPUTER APTITUDE nineteenth from the right end of the above arrangement?
(a) M (b) T
Directions (Qs.1-3): In these questions, the relationship between (c) # (d) 2
different elements is shown in the statements. The statements
(e) None of these
are followed by two conclusions,
7. How many such symbols are there in the above arrangement,
Give answer (a) if only conclusion I is true each of which is immediately preceded by a number and
Give answer (b) if only conclusion II is true immediately followed by a letter?
Give answer (c) if either conclusion I or II is true (a) None (b) One
Give answer (d) if neither conclusion I nor II is true (c) Two (d) Three
Give answer (e) if both conclusions I and II are true (e) More than three
1. Statement: T < P £ U ; L > U £ K ; P ³ R Directions (Qs.8-10): In each of the questions below are given
three statements followed by four conclusions numbered I, II, III
Conclusions: I. K ³ R II. L > R and IV. You have to take the given statements to be true even if
2. Statement: H = I £ R ; M ³ R < S they seem to be at variance with commonly known facts. Read all
Conclusions: I. M = I II. M > I the conclusions and then decide which of the given conclusions
3. Statement: D > H ³ N ; S > I £ H logically follows from the given statements disregarding
commonly known facts.
Conclusions: I. N £ S II. I < D
8. Statements:
Directions (Qs.4-7): Study the following arrangement carefully Some buses are trains.
and answer the questions given below: No train is a dog.
F 4 © J 2 E % M P 5 W 9 @ I Q R 6 U H 3 Z 7« A T B 8 V # G $ Y D All dogs are parrots.
4. What should come in place of the question mark (?) in the Conclusions:
following series based on the above arrangement? I. No bus is a parrot.
JEM 59I RU3 ? II. Some parrots are trains.
(a) 7 A B (b) 7 A T III. Some parrots are buses.
(c) « 7 8 (d) A B V IV. No dog is a bus.
(e) None of these (a) Only either I or III follows
5. How many such consonants are there in the above (b) Only II follows (c) Only IV follows
arrangement, each of which is immediately preceded by a (d) Only I and III follow (e) None of these
number but not immediately followed by a number? 9. Statements:
(a) None (b) One Some cups are flowers.
(c) Two (d) Three Some flowers are boxes.
(e) More than three All boxes are tigers.
www.newspaperkorner.wordpress.com
54 Practice Set - 4
www.newspaperkorner.wordpress.com
Conclusions: Directions (Qs. 16 -17): Read the information given below and
I. Some tigers are cups. answer the questions that follow.
II. Some tigers are flowers. In a certain code
III. Some boxes are cups. (a) "rem tur dimi wee" means "my pencil was stolen",
IV. No tiger is a flower. (b) "soz rem legh ner" means "your pencil is here",
(a) None follows (c) "rem zet tur legh" means " blue pencil was here" and
(b) Only either II or IV follows (d) "dimi rem tur soz" means "your pencil was stolen".
16. Which of the following is the code for the word "here"?
(c) Only III follows
(d) Only either I or III follows (a) tur (b) ner (c) dimi (d) legh
(e) None of these (e) None of these
17. Which of the following indicates "Blue pencil is stolen"?
10. Statements:
(a) zet rem ner dimi (b) ner legh tur zut
Some cats are lions. (c) dimi rem soz ner (d) zet legh tur rem
All lions are hares.
(e) None of these
All hares are horses. Directions (Qs.18-20): Each of the questions below consists of a
Conclusions: question and two statements numbered I and II given below it.
I. Some cats are horses. You have to decide whether the data provided in the statements
II. Some horses are lions. are sufficient to answer the question. Read both the statements
III. Some hares are cats. and
IV. Some cats are hares. Give answer (a) if the data in statement I alone are sufficient to
(a) Only I and II follow (b) Only III and IV follow answer the question, while the data in statement II alone are not
sufficient to answer the question.
(c) Only I and III follow (d) All follow
Give answer (b) if the data in statement II alone are sufficient to
(e) None of these
answer the question, while the data in statement I alone are not
Directions (Qs.11-15): Study the following information carefully sufficient to answer the question.
and answer the given questions: Give answer (c) if the data either in statement I alone or in statement
Eight friends, P, Q, R, S, T, V, W and Y are sitting around a II alone are sufficient to answer the question.
square table in such a way that four of them sit at four corners of Give answer (d) if the data given in both the statements I and II
the square while four site in the middle of each of the four sides. together are not sufficient to answer the question.
The ones who sit at the four corners face the centre while those Give answer (e) if the data in both statements I and II together are
sit in the middle of the sides face outside. necessary to answer the question.
P, who faces the centre, sits third to the right of V. T, who 18. On which day of the week did Sourav visit Delhi?
faces the centre, is not an immediate neighbour of V. Only one I. Sourav visited Delhi after Monday but before Thursday
person sits between V and W. S sits second to right of Q. Q faces but not on an odd day of the week.
the centre. R is not an immediate neighbour of P.
II. Sourav visited Delhi before Friday but after Monday.
11. Who sits second to the left of Q? 19. What is R’s position from the left end in a row
(a) V (b) P I. M is tenth from the left end of the row.
(c) T (d) Y II. There are sixteen children between M and R.
(e) Cannot be determined 20. Town P is towards which direction of town T?
12. What is the position of T with respect of V? I. Town T is towards South of town K, which is towards
(a) Fourth of the left (b) Second to the left West of town P.
(c) Third to the left (d) Third to the right II. Town R is towards South of town V and towards East
(e) Second to the right of town T.
13. Four of the following five are alike in a certain way and so Direction (Qs. 21) : Read the following information carefully
form a group. Which is the one that does not belong to that and answer the questions which follow.
group? Except from a research report – ‘Average life expectancy in
(a) R (b) W (c) V (d) S (e) Y southern part of India is far more than that in Western India.
14. Which of the following will come in place of the question While the average life of a native of South India is 82 years, the
mark based upon the given seating arrangement? average life of a native of Western India is only 74 years.’
WP TR QW RS ? Based on the above fact, the proposal that the above study
(a) YT (b) VY` (c) VQ (d) PY (e) QV makes is that if an individual moves from Western India to South
15. Which of the following is true regarding R? India, his/her life expectancy would immediately increase by
(a) R is an immediate neighbour of V. eight years.
(b) R faces the centre. Which of the following statements would weaken the
(c) R sits exactly between T and S. above-mentioned study’s proposal that people belonging to
(d) Q sits third to left of R. Western parts of India should move to South India to increase
(e) None of these their life expectancy?
www.newspaperkorner.wordpress.com
Practice Set - 4 55
www.newspaperkorner.wordpress.com
(a) The average life expectancy of population living in (c) Unlike older times, industrial development has ceased
Eastern part of the country is also less than the to be a reason behind global warming these days
population living in South India. (d) if industrial development stops, global warming would
(b) Nearly 80% of the population in Southern India has a automatically come to an end.
minimum age of 83 years. (e) if sources other than industrial development are
(c) Higher life expectancy in Southern India can be identified and controlled, global waring will end
ascribed to the genetic makeup of the population completely.
belonging to that area. 25. Which of the statements numbered A, C, D and E mentioned
(d) The average life expectancy of South India is above represents a reason behind curtailing industrial
comparable to the best averages in the world. development by some of the countries?
(e) Higher life expectancy in Southern India can be (a) Either C or E (b) D
attributed to better environmental conditions and (c) C (d) Either A or C
better healthcare facilities. (e) A
Directions (Qs.22-25): Read the following information carefully Directions (Qs. 26-30): Study the following information to
and answer the questions which follow: answer the questions given below :
Fortunately, more and more countries are shifting their focus away A number arrangement machine when given an input of numbers,
from industrial development to control of climate change these days. rearranges them following a particular rule in each step. The
A. The countries which focus more on controlling climate following is an illustration of input and steps of rearrangement.
change than industrial development are only the richer ones Input : 48 245 182 26 99 542 378 297
which can afford to concentrate on areas other than Step I : 542 48 245 182 26 99 378 297
industrial development. Step II : 542 26 48 245 182 99 378 297
B. Many countries had once prioritised industrial development Step III : 542 26 378 48 245 182 99 297
which proved to be harmful to the environment in the long Step IV : 542 26 378 48 297 245 182 99
run. Step V : 542 26 378 48 297 99 245 182
C. Some experts are of the view that climate change is not as This is the final arrangement and step V is the last step for this
alarming an issue as it is made to be because it is a natural input.
phenomenon and has been occurring regularly throughout 26. What will the fourth step for an input whose second step is
the history of earth. given below?
D. If climate change continues at the present rate, it would Step: 765 42 183 289 542 65 110 350
bring in large-scale destruction to human habitation in a (a) 765 42 542 350 183 289 65 110
very short time. (b) 765 42 542 65 110 183 289 350
E. Industrial development is one of the biggest but definitely (c) 765 42 542 65 183 289 110 350
not the only reason behind global warming. (d) Cannot be determined
22. Which of the statements numbered A, B, C and D mentioned
(e) None of these
above would weaken the argument for the need of a shift
27. What should be the third step of the following input?
away from industrial development to that controlling climate
change? Input: 239 123 58 361 495 37
(a) A (b) B (a) 495 37 361 123 239 58
(c) C (d) D (b) 495 37 58 361 123 239
(e) Both B and D (c) 495 37 58 123 361 239
23. Which of the statements numbered (A), (B), (C), (D) and (E) (d) 495 37 361 239 123 58
can be assumed / inferred from the facts / information given (e) None of these
in the statement ? (As assumption is something supposed 28. How many steps will be required to get the final output
or taken for granted and an inference is something which is from the following input?
not directly stated but can be inferred from the given facts.) Input: 39 88 162 450 386 72 29
(a) E (b) C (a) Two (b) Three
(c) A (d) B (c) Four (d) Six
(e) Either D or A (e) None of these
24. Which of the following can be inferred from statement (E) if 29. What should be the last step of the following input?
it is considered to be true with regard to the given Input: 158 279 348 28 326 236
information?
(a) 348 28 326 158 279 236
(a) Nations also need to focus on sources other than those
(b) 348 28 326 236 158 279
generated due to industrial development.
(b) Other sources of pollution have more adverse effects (c) 348 28 236 158 279 326
as compared to those generated due to industrial (d) 348 28 158 326 236 279
development. (e) None of these
www.newspaperkorner.wordpress.com
56 Practice Set - 4
www.newspaperkorner.wordpress.com
30. If the first step of an input is 37. __________ permanently stores data and consists of non-
“785 198 32 426 373 96 49”, volatile chips.
then which of the following steps will be (a) RAM (b) CPU
“785 32 426 49 198 373 96”? (c) ROM (d) Matherboard
(a) Third (b) Fourth (e) None of these
(c) Fifth (d) Second 38. In Excel, the contents of the active cell can be changed
(e) None of these through
Directions (Qs. 31-35): Study the following information to (a) menu bar (b) formula bar
answer the given questions: (c) tool bar (d) footer bar
• Six plays are to be organised from Monday to Sunday - one (e) task bar
play each day with one day when there is no play. ‘No play’ 39. In a sequence of events that take place in an instruction
day is not Monday or Sunday. cycle. the second cycle is called.
• The plays are held in sets of 3 plays each in such a way that (a) fetch cycle (b) store cycle
3 plays are held without any break, ie 3 plays are held in (c) code cycle (d) decode cycle
such as way that there is no ‘No play’ day between them (e) execute cycle
but immediately before this set of immediately after this set 40. An e-mail address consists of
it is ‘No play’ day. (a) domain name followed by user’s name
• Play Z was held on 26th and play X was held on 31st of the (b) user’s name @ symbol followed by domain name
same month. (c) user’s name followed by domain name
• Play B was not held immediately after play A (but was held (d) User’s name space @ symbol followed by domain name
after A, not necessary immediately) and play M was held
(e) None of these
immediately before Q.
41. DSL stands for
• All the six plays were held in the same month.
(a) Data Subscriber Line (b) Direct Subscriber Line
31. Which plays was organised on Monday?
(c) Digital Subscriber Line (d) Dual Subscriber Line
(a) Z (b) M
(e) Divisible Subscriber Line
(c) Q (d) Cannot be determined
42. The .ppt extension is used for _____ files.
(e) None of these
32. Which day was play Z organised ? (a) word (b) access
(a) Tuesday (c) powerpoint (d) excel
(b) Monday (e) None of these
(c) Wednesday 43. You can group non-contiguous worksheets with
(d) Cannot be determined (a) ctrl key, mouse (b) alt + enter keys
(e) None of these (c) shift key, mouse (d) alt key, mouse
33. Which date was a ‘No play’ day? (e) None of these
(a) 26th (b) 28th 44. The advantage of using pre-written software packages is to
(c) 29th (d) Cannot be determined
(a) eliminate program writing
(e) None of these
(b) eliminate program testing
34. Which of the following is true?
(c) save time and cost
(a) Play B is held immediately before play M.
(b) Play Z is held after play B. (d) All the above
(c) There was a gap after 2 plays and then 4 plays were (e) None of these
organised. 45. Which of the following is an impact printer?
(d) First play was organised on the 25th. (a) Laser printer (b) LCD/LED printer
(e) Play B was held on Friday. (c) Ink-Jet printer (d) Thermal printer
35. Which day was play Q organised ? (e) Drum printer
(a) Friday (b) Wednesday
(c) Saturday (d) Cannot be determined DATA ANALYSIS & INTERPRETATION
(e) None of these Directions (Qs.46-50): In each of these questions a number
36. The term “Encoder-decoder” is used for a series is given. Below the series one number is given followed
(a) networker (b) joiner by (1), (2), (3), (4) and (5). You have to complete this series
(c) router (d) modem following the same logic as in the original series and answer the
(e) connection question that follow.
www.newspaperkorner.wordpress.com
Practice Set - 4 57
www.newspaperkorner.wordpress.com
46. 5 9 25 91 414 2282.5 53. Company D offers compound interest under Scheme II
3 (1) (2) (3) (4) (5) simple interest under Scheme IV. Abhijit invested ` 25,000
What will come in place of (3)? with this company under Scheme IV and after one year
(a) 63.25 (b) 63.75 switched to Scheme II along with the interest for one more
year. What is the total amount he will get at the end of two
(c) 64.25 (d) 64.75
years?
(e) None of these
(a) ` 28,939.25 (b) ` 29,838.75
47. 15 9 8 12 36 170
(c) ` 31,748.25 (d) ` 31,738.75
19 (1) (2) (3) (4) (5)
(e) None of these
What will come in place of (2)?
54. Abhishek invested an amount of ` 45,000 for two years
(a) 18 (b) 16 with Company B under Scheme III, which offers compound
(c) 22 (d) 24 interest, and Jeevan invested an equal amount for two years
(e) None of these with Company C under Scheme IV, which offers simple
48. 7 6 10 27 104 515 interest. Who earned more interest and how much?
9 (1) (2) (3) (4) (5) (a) Abhishek, ` 1,875 (b) Jeevan, ` 1,875
What will come in place of (4)? (c) Abhishek, ` 1,962 (d) Jeevan, ` 1,962
(a) 152 (b) 156 (e) None of these
(c) 108 (d) 112 55. Mr. Lal invested ` 30,000 in Company A under Scheme II,
(e) None of these which offers compound interest and ` 48,000 in Company
49. 6 16 57 244 1245 7506 D under Scheme II, which offers compound interest. What
4 (1) (2) (3) (4) (5) will be the total amount of interest earned by Mr. Lal in two
What will come in place of (4)? years?
(a) 985 (b) 980 (a) ` 14,728.80 (b) ` 17,428.50
(c) 1004 (d) 1015 (c) ` 14,827.70 (d) ` 16,728.20
(e) None of these (e) None of these
50. 8 9 20 63 256 1285 Directions (Qs. 56–60): Analyse the following chart showing
5 (1) (2) (3) (4) (5) the exports and imports of Sono Ltd. and answer the questions
What will come in place of (5)? based on this chart.
(a) 945 (b) 895
(c) 925 (d) 845
(e) None of these
Directions (Qs.51-55): Study the following table carefully to
answer the questions.
Rate of interest (P.C.P.A.) offered by five companies on deposits
under different schemes
Company ®
A B C D E
Scheme ¯
I 8.5 9.0 8.0 8.5 9.0
II 9.5 8.5 9.0 9.0 8.5
III 8.0 8.0 7.5 8.5 8.5
IV 10.0 9.5 10.5 9.5 10.0
51. Mr X. deposited an amount in Scheme II with Company for 56. Approximately by what percentage are the total Exports
two years. After that he withdrew the amount and reinvested greater/smaller than the total imports for the given period?
only the principal amount in Scheme IV of Company B for (a) Greater by 9 percent (b) Smaller by 10 percent
two years. Total amount of simple interest accrued from the (c) Smaller by 9 percent (d) Greater by 10 percent
two schemes is ` 14,800. What was the principal amount?
(e) None of these
(a) ` 48,000 (b) ` 42,000
57. If the absolute difference between imports and exports are
(c) ` 40,000 (d) Cannot be determined
ranked in ascending order, which year gets 4th rank?
(e) None of these
(a) 2010 (b) 2008
52. Company E offers compound interest under Scheme I and
Company A offers simple interest under Scheme IV. What (c) 2009 (d) 2011
will be the difference between the interest earned under (e) None of the above
Scheme I of Company E and Scheme IV of Company A 58. In which year was the fifth largest annual percentage increase
respectively in two years on an amount of ` 1.2 lakhs? in exports recorded?
(a) ` 1,428 (b) ` 1,328 (a) 2007 (b) 2005
(c) ` 1,528 (d) ` 1,548 (c) 2009 (d) 2008
(e) None of these (e) None of the above
www.newspaperkorner.wordpress.com
58 Practice Set - 4
www.newspaperkorner.wordpress.com
59. Which year saw the second largest annual percentage number of candidates appeared from Engineering
increase in imports? discipline?
(a) 2010 (b) 2005 (a) 50 (b) 150
(c) 2006 (d) 2004 (c) 200 (d) Cannot be determined
(d) None of these (e) None of these
60. What is the approximate percentage point difference in the 63. Approximately what was the difference between the number
maximum annual percentage increase in export and the of candidates selected from Agriculture discipline and
minimum annual percentage decrease in imports? number of candidates selected from Engineering discipline?
(a) 28 (b) 48 (a) 517 (b) 665 (c) 346 (d) 813 (e) 296
(c) 64 (d) 12 64. For which discipline was the difference in number of
(e) None of these candidates selected to number of candidates appeared in
Directions (Qs.61-65): Study the following pie-charts carefully interview the maximum?
and answer the questions given below: (a) Management (b) Engineering
Discipline-wise Breakup of the Number of candidates appeared (c) Science (d) Agriculture
in Interview and Discipline-wise Break up of the Number of (e) None of these
candidates selected by an organisation 65. Approximately what was the total number of candidates
Discipline-wise Breakup of Number of candidates appeared in selected from Commerce and Agricultural discipline
Interview together?
Total Number of Candidates Appeared (a) 1700 (b) 1800 (c) 2217 (d) 1996 (e) 1550
In the Interview = 25780 Directions (Qs.66-70): Study the information carefully to answer
Percentage Distribution the questions that follow:
There are two companies A and B. Both companies produce
Commerce
18 all the four different products, viz Computers, Phones, Pen drives
Others and Compact Discs (CDs). Company A produces a total of 800
Agriculture 12
14 products. The ratio of the total products produced by Company
A to that of Company B is 4 : 5 : 20% of the total products produced
by Company B are Pen Drives and 40% of them are CDs. Two-
Management Science fifths of the remaining products produced by Company B are
12 28 Phones. The total number of Computers produced by both the
companies together is 340.20% of the total products produced
by Company A are CDs. Company A produces equal number of
Pen Drives and Phones.
Engineering 66. What is the ratio of the number of Pen Drives produced by
16 Company A to the number of Computers produced by
Discipline-wise Breakup of Number of candidates selected after Company B?
Interview by the organization (a) 8 : 9 (b) 9 : 7
Total Number of Candidates selected (c) 7 : 9 (d) 4 : 9
After Interview = 7390 (e) None of these
Percentage Distribution 67. What is the total number of Phones produced by both the
companies together?
Commerce
(a) 430 (b) 420
Agriculture Others 16
7
(c) 390 (d) 530
14 (e) None of these
68. The number of Phones produced by Company A is what
Science percentage of the total number of products produced by
Management 32 Company B?
20 (a) 25 (b) 29
(c) 27 (d) 37
(e) None of these
Engineering 69. What is the average of the number of Pen Drives, CDs and
11 Computers produced by Company B?
61. What is the ratio of the number of candidates appeared in (a) 840 (b) 280
interview from other discipline and the number of candidates (c) 270 (d) 860
selected from Engineering discipline respectively (rounded (e) None of these
off to the nearest integer)? 70. What is the difference between the number of CDs produced
(a) 3609 : 813 (b) 3094 : 813 by Company B and the number of Computers produced by
(c) 3094 : 1035 (d) 4125 : 1035 Company A?
(e) 3981 : 767 (a) 200 (b) 250
62. The total number of candidates appeared in interview from (c) 300 (d) 350
Management and other discipline was what percentage of (e) None of these
www.newspaperkorner.wordpress.com
Practice Set - 4 59
www.newspaperkorner.wordpress.com
Directions (Qs.71-75): Study the following table carefully to 77. What is the average monthly expenditure on food by all the
answer the questions that follow: people together?
Numbers (N) of Six types of Electronic Products Sold by (a) ` 1,08,500 (b) `10,850
six different stores in a month and the price per product (c) ` 54,250 (d) ` 52,450
(P) (price in ` 000) charged by each Store. (e) None of these
Store A B C D E E
78. Whose monthly expenditure on all the heads together is
the lowest among them?
Product N P N P N P N P N P N P
L 54 135 48 112 60 104 61 124 40 136 48 126 (a) A (b) B
M 71 4.5 53 3.8 57 5.6 49 49 57 5.5 45 4.7 (c) C (d) D
N 48 12 47 18 52 15 54 12 62 11 56 11 (e) F
O 52 53 55 48 48 50 54 49 59 47 58 51
79. If the monthly expenditure of C on children’s education is
P 60 75 61 68 56 92 44 84 46 76 59 78 increased by 5%, then what will be his yearly expenditure
Q 43 16 44 15 45 15 48 16 55 18 55 15 on children’s education?
71. The number of L type products sold by Store F is what (a) ` 1,58,760 (b) ` 15,87,600
percent of the number of the same type of products sold by (c) ` 13,230 (d) ` 1,32,300
Store E? (e) None of these
(a) 76.33 (b) 124 80. What is the ratio of the monthly expenditure made by A on
(c) 83.33 (d) 115 travelling to the monthly expenditure made by D on clothes?
(e) None of these (a) 57 : 105 (b) 105 : 59
72. What is the ratio of the total number of N and L type products (c) 37 : 103 (d) 59 : 105
together sold by Store D and that of the same products (e) None of these
sold by Store A?
(a) 119 : 104 (b) 102 : 115 ENGLISH LANGUAGE
(c) 104 : 115 (d) 117 : 103 Directions (Qs. 81-90) : Read the following passage carefully
(e) None of these and answer the questions given below it. Certain words/phrases
73. What is the average price per product charged by all the are printed in bold to help you to locate them while answering
Stores together for Product Q? some of the questions.
(a) ` 14,700 (b) ` 15,700 The writer believes that the work experience should focus
more on the development in students of the ability to work with
(c) ` 15,200 (d) ` 14,800
the materials of their environment and solve problems as close to
(e) None of these reality as is practicable.
74. What is the difference in the amount earned by Store A Work experience is included as a separate subject in the school
through the sale of P type products and that earned by system, should not be treated in the same manner as other school
Store B through the sale of Q type products? subjects, i.e., accumulation of certain technical facts with a
(a) ` 38.4 lakhs (b) ` 0.384 lakh superficial blessing of practical aspect of this new subject in our
(c) ` 3.84 lakhs (d) ` 384 lakhs school curriculum.
(e) None of these Work experience when included as a separate subject in our school
75. What is the total amount earned by Store C through the curriculum will not doubt entail huge investment. But if the
sale of M and O type products together? concept of work experience is implemented with a sincere and
(a) ` 2719.2 lakhs (b) ` 271.92 lakhs competent guidance, this above investment will be a great asset
(c) ` 2.7192 lakhs (d) ` 27.192 lakhs to our nation.
(e) None of these Let us suppose, that work experience is designed in such a manner
to be productive and self sufficient, it may have some negative
Directions (Qs.76-90): Study the following table carefully to reaction from local small scale producers of those consumer goods.
answer the questions that follow: However, an assumption is made in the above conviction that the
Monthly Expenditure (in thousands) by five people on Rent, quality of consumer articles produced by a school would be good
Food, Children’s Education, Clothes and Travelling enough to compete the market.
Expenditure ® Children's Ignoring all the inherent dangers discussed earlier of
Rent Food Clothes Travelling incorporating work experience as a separate subject in our school
People ¯ Education
A 12.5 7.50 6.52 3.30 4.72 curriculum, it may still be worthwhile to include work experience
B 16.0 8.55 8.38 2.75 5.86 level in science teaching even with work experience as a separate
C 13.8 11.40 12.60 6.30 9.30 subject.
D 9.65 17.80 9.95 8.40 7.85 This concept of work experience in science teaching would help
E 14.5 9.00 10.25 3.90 5.42 to provide a proper training (or development of scientific attitude)
in work experience as a separate subject and in useful occupations
76. What is the total monthly expenditure made by D on rent, B
or vocations to our students. The above training may encourage
on clothes and E on travelling together?
students to set up small scale industries after their completion of
(a) ` 18,720 (b) ` 1,78,200 grade ten. This above approach may give some satisfaction to
(c) ` 17,800 (d) ` 1,84,720 our tax payers that the investment in the educational endeavour
(e) None of these is not a total wastage of their money.
www.newspaperkorner.wordpress.com
60 Practice Set - 4
www.newspaperkorner.wordpress.com
The writer’s experience (in various countries) suggests that to Directions (Qs. 87 & 88) : Choose the word/group of words
make work experience a worthwhile experience, it should also be which is most nearly the same in meaning as the word/group of
incorporated as an integral part into the high school subjects, words given in bold as used in the passage.
especially science subjects. To accomplish this, any school 87. Self sufficient
subject especially science must be taught on four levels; the (a) Adequate (b) Dependent
facts level, the concepts level, the values level, and the work (c) Overflowing (d) Self-advocating
experience level. (e) Self reliant
81. What is the subject which should not be treated in the 88. Superficial
same manner as other school subjects ? (a) Artificial (b) Shallow
(a) Technical subject (c) Complete (d) Profound
(b) Practical aspects of all the subjects (e) Imperfect
(c) Work experience Directions (Qs. 89 & 90) : Choose the word which is most opposite
(d) School system and procedures in meaning of the word given in bold as used in the passage.
(e) None of these 89. Ability
82. According to the writer, the work experience should focus (a) Competency (b) Aptitude
more ... (c) Inefficiency (d) Importance
A. on the development in students of extra curricular (e) Lethargy
activities. 90. Reality
B. on the development in students of the ability to work (a) Authentic (b) Superfluous
with the materials of their surroundings.
(c) Imagination (d) Lackadaisical
C. on the ability to solve problems as close to reality as is
(e) Feeble
practicable.
Directions (Qs. 91-92): In each of the following sentences there
(a) Only A (b) Only B are certain blankspaces. Below each sentence there are five
(c) Only C (d) Only B and C options denoted by the numbers (a), (b), (c), (d) and (e). Find out
(e) All A, B and C which option can be used to fill up the blank to make the sentence
83. The concept of work experience in science teaching would meaningfully complete.
help 91. The modernisation drive of the country has in fact not been
A. to provide proper training in work experience. ________________ by an improvement in the quality of
B. students to set up small scale industries after the many of its manufactured ________________.
completion of grade ten. (a) associated, sectors (b) accompanied, products
C. to give some satisfaction to our tax payers that they (c) prevailed, area (d) acquired, fields
could reap a little more profit from this endeavour. (e) conquered, items
(a) Only A 92. The Govt. _____________ that Indian IT industry will be
(b) Only B able to generate _____________of over $ 100 billion by
(c) Only C the later part of the coming decade.
(d) Only A and B (a) reveal, volume (b) states, service
(e) All, A, B, and C (c) predict, future (d) estimated, capital
84. What will happen if the work experience is designed to be (e) forecasts, revenue
productive and self-sufficient ? Directions (Qs. 93-97) : Rearrange the following five sentences
(a) It may lead to huge investment (A), (B), (C) , (D) and (E) in the proper sequence to form a
(b) It may help the nation in asset building meaningful paragraph; then answer the questions given below
(c) It may have some negative reaction from the local small them.
scale producers (A) A serious conceptual problem often raised about universal
(d) It may lead to accumulation of certain technical facts disarmament is that even if you succeed in eliminating all
(e) It may lead to better quality goods production nuclear weapons on earth, you cannot eliminate man's
knowledge of the science behind it.
85. Why the writer suggests that science must be taught on
four levels? (B) The goal of nuclear disarmament provides another
motivation for ridding the world of fissile materials.
(a) To make the students interested in the subjects
(b) To make them matured citizens of the country (C) What is to prevent some groups from starting to produce
these weapons all over again.
(c) To make work experience a worthwhile experience
(D) That genie is out of the bottle for good.
(d) To make the science subjects popular among the
students (E) Is nuclear weapon-free world a robust and stable concept?
(e) None of these 93. Which of the following should be the FIRST sentence after
rearrangement?
86. The idea that to make experience a worthwhile experience, it
should also be incorporated as an integral part into the (a) A (b) B (c) C (d) D (e) E
high school subjects comes from 94. Which of the following should be the SECOND sentence
(a) his interaction with other writers after rearrangement?
(b) his experience in science teaching (a) A (b) B (c) C (d) D (e) E
(c) his patriotism and love for higher education 95. Which of the following should be the THIRD sentence after
(d) his experience in several countries rearrangement?
(e) None of these (a) A (b) B (c) C (d) D (e) E
www.newspaperkorner.wordpress.com
Practice Set - 4 61
www.newspaperkorner.wordpress.com
96. Which of the following should be the FOURTH sentence 108. The Government has asked individuals /
after rearrangement?
(a)
(a) A (b) B (c) C (d) D (e) E
97. Which of the following should be the FIFTH sentence after with income of over 110 lakhs to
rearrangement? /
(b)
(a) A (b) B (c) C (d) D (e) E
electronic file tax returns for the year 2011 -12,
Directions (Qs. 98-107) : In the following passage there are /
blanks, each of which has been numbered. These numbers are (c)
printed below the passage and against each, five words are something which was optional till last year. No error
suggested one of which fits the blank appropriately. Find out
the appropriate words. (d) (e)
The exhibition will be remembered for accuracy and (98) 109. The power tariff had already / been increased twice in /
documentation as well as sensitive response. It is one thing to (a) (b)
record (99) what one sees and quite another, to record it with
(100) and imagination—pictorial (101) to which one can relate. the last 15 months and the Electricity Board had also
He, following in the (102) of some of the all-time greats of /
(c)
photography, is happy with the (103) statements he makes. What
(104) his work to us is its elegant (105), and smooth and levied additional monthly charges to consumers.
unassertive (106) free from any effort for effect. He is one of (d)
those convinced "moderns" to whom photography is more visual
poetry than technical (107). No error
98. (a) artistic (b) artesian (e)
(c) artistry (d) beautification 110. Despite of curfew / in some areas, minor /
(e) statistically
(a) (b)
99. (a) visually (b) perfection
(c) cryptically (d) awfully communal incidents were reported
/
(e) seemingly (c)
100. (a) foresight (b) insight from different areas of the walled city. No error
(c) sight (d) foregone /
(d) (e)
(e) candid
Directions (Qs. 111-115) : In each of the following questions
101. (a) idiom (b) caricature four words are given, of which two are most nearly the same or
(c) sketch (d) outline opposite in meaning. Find the two words which are most nearly
(e) portrait the same or opposite in meaning and indicate the number of the
102. (a) wake (b) footsteps correct letter combination.
(c) imprint (d) footprints 111. A. Disciple B. Student
C. Academy D. Martyr
(e) line
103. (a) audio (b) vascular (a) A-C (b) C-D (c) B-C (d) B-D (e) A-B
112. A. Magnetic B. Cherished
(c) visual (d) sensory
C. Valued D. Forlorn
(e) classic
(a) C-D (b) A-B (c) B-D (d) B-C (e) A-D
104. (a) brings (b) disarrays
113. (A) Ordinary (B) Vague
(c) lugs (d) endears
(C) Custom (D) Exceptional
(e) endeavour
(a) A-B (b) B-C (c) C-D (d) A-D (e) B-D
105. (a) grace (b) casualness
114. (A) Damaged (B) Hurried
(c) treatise (d) complexity (C) Condemned (D) Measured
(e) Utopia (a) A-B (b) C-D (c) B-D (d) B-C (e) A-D
106. (a) treaty (b) memoirs 115. A. Remote B. Troubled
(c) mementos (d) technique C. Secluded D. Apparent
(e) clique (a) B-C (b) A-D (c) B-D (d) C-D (e) A-C
107. (a) jargon (b) jugglery
GENERAL/ECONOMY/
(c) gibberish (d) wizard
BANKING AWARENESS
(e) complexion
Directions (Qs. 108-110) : Read each sentence to find out 116. The “Four Eyes” principle (mentioned by the Reserve Bank
whether there is any grammatical error or idiomatic error in it. of India) refers to:
The error,if any, will be in one part of the sentence. The letter of (a) Lenders (b) Borrowers
that part is the answer. If there is 'No error', the answer is ‘(e)’. (c) Wealth Managers (d) Micro-Fananciers
(ignore errors of punctuation, if any). (e) None of these
www.newspaperkorner.wordpress.com
62 Practice Set - 4
www.newspaperkorner.wordpress.com
117. The disinvestments proceeds are used for which purpose? 126. Choose the correct statements in the context of Cooperative
(a) To help co-operative sector Banks in India.
(b) To strengthen the private sector banks 1. Cooperative Banks operate on no profit no loss basis.
(c) To provide additional capital to public sector banks 2. Cooperative Banks are allowed to operate only in the
(d) For funding the capital expenditure under the social agriculture sector.
security schemes of the govt. 3. NABARD is a Cooperative Bank.
(e) None of these (a) 1 only (b) 1 and 2
118. ‘Take-Out’ Financing is an option primarily propounded for
what in India? (c) 1 and 3 (d) Either 1 or 2
(a) Small Scale Enterprises (b) Infrastructure funding (e) None of these
(c) Restaurants (d) Educational institutions 127. NRE deposit is
(e) None of these (a) Non Resident External deposit
119. Consider the following banks: (b) Non Resident Extra deposit
1. ABN Amro Bank 2. Barclays Bank (c) Non Resident Exchange deposit
3. Kookmin Bank (d) Non Refundable External deposit
Which of the following correctly represents their countries
of origin? (e) Non Resident Extended deposit
(a) Dutch, USA, Japan (b) Japan, USA, China 128. MINT is a acronym for group of countries which are
(c) Dutch, UK, South Korea (d) Dutch, USA, China promising investment destination. Which of the following
(e) None of these correctly represents MINT countries?
120. Who is the chairperson of State Bank Of India? (a) Myanmar, India, Nepal and Thailand
(a) Arundhati Bhattacharya (b) Naina Lal Kidwai (b) Mexico, Indonesia, Nigeria and Turkey
(c) Chanda Kochar (d) Shikha Sharma (c) Malaysia, Indonesia, Nigeria and Thailand
(e) None of these (d) Mexico, India, Nepal and Turkey
121. The Reserve Bank of India (RBI) extended the timeline for
full implementation of the Basel III capital regulations by a (e) None of these
year. Now the banks are required to comply with the Basel 129. National Girl Child Day was observed across India by the
III norms by 31 March 2019 instead of Union Government on
(a) 31 March 2016 (b) 31 March 2018 (a) 24 January (b) 25 January
(c) 31 March 2017 (d) 31 March 2015 (c) 27 January (d) 21 January
(e) None of these
(e) None of these
122. Factors responsible for the widespread popularity of
consumer finance in recent years: Providing access to 130. We know that the industrial sector plays an important role
purchasing power to the middle class consumer has been in the development of India. Which of the following sec-
the most significant achievement of this product class. tors does not come under industrial sector?
A complainant not satisfied with the decision of the Banking (a) Mining (b) Manufacturing
Ombudsman, has the right to appeal to the_______. (c) Banking (d) Utilities
(a) Governor SBP
(e) Construction
(b) AVP of the concerned bank
(c) High Court 131. Which of the following is NOT a banking-related term?
(d) Supreme Court (a) SME Finance (b) Overdraft
(e) None of these (c) Drawing power (d) Sanctioning Authority
123. Which of the following committee is associated with the (e) Equinox
review of Insider Trading Regulations? 132. Traditional marketing style involves
(a) Sodhi Committee (a) Telemarketing (b) Digital Marketing
(b) Sinha Committee (c) Indirect Marketing (d) Direct Marketing
(c) Tarapore Committee
(e) None of these
(d) Chandrashekhar Committee
133. Niche Market means a _____.
(e) None of these
124. Which of the following statements is/are correct in regard (a) free market (b) social market
to ‘micro-finance’? (c) equity market (d) capital market
(1) Micro-credit extended by banks to individual is (e) specified market for a target group
reckoned as a part of their priority sector lending. 134. Qualities essential in good marketing are ____.
(2) RBI has prescribed a particular model the banks to (a) aggressiveness (b) pushy
provide micro-finance.
Choose the correct answer using the codes given below: (c) perseverance (d) politeness
(a) 1 only (b) 2 only (e) Only (c) and (d)
(c) Both 1 and 2 (d) Neither 1 nor 2 135. The ‘Mahajan Commission’ was constituted to look into
(e) None of these the border dispute between which two states?
125. Who is the author of the book ‘Beyond the lines : An Auto- (a) Karnataka and Andhra Pradesh
biography’ ? (b) Andhra Pradesh and Maharashtra
(a) Sharad Pawar (b) Kushwant Singh (c) Maharashtra and Karnataka
(c) Kuldip Nayar (d) General J. J. Singh (d) Karnataka and Kerala
(e) None of these (e) None of these
www.newspaperkorner.wordpress.com
Practice Set - 4 63
www.newspaperkorner.wordpress.com
136. Which one of the following is not included in Article 19 of 145. As per the Union Budget 2016-17, how much amount was
the constitution of India, pertaining to the Right to allocated towards Agriculture and Farmers' welfare?
Freedom? (a) 34,980 crore rupees (b) 40,500 crore rupees
(a) Right to reside and settle in any part of the territory of (c) 35,984 crore rupees (d) 36,000 crore rupees
India (e) None of these
(b) Right to form associations or unions 146. In the Union Budget 2016-17, Arun Jaitley announced
(c) Right of minorities to establish and administer creation of a dedicated Long Term Irrigation Fund in
educational institutions NABARD. What will be the initial corpus of the fund?
(d) Right to assemble peaceably and without arms (a) About 15,000 crore rupees
(e) None of these (b) About 10,000 crore rupees
137. The share of deposits that banks must maintain in safe and (c) About 16,000 crore rupees
liquid assets such as government securities is known as (d) About 20,000 crore rupees
_______? (e) None of these
(a) Repo (b) Reverse Repo 147. The World Autism Awareness Day is observed on which
(c) SLR (d) CRR date?
(e) None of these (a) March 31 (b) April 1
138. The quantum of loan under Home loan is decided on the (c) April 2 (d) April 3
basis of (e) None of these
(a) income of the borrower 148. Which Indian personality was recently bestowed with the
(b) cost of the property/cost of construction Saudi Arabia's highest civilian honour "the King Abdulaziz
(c) repaying capacity of the person Sash"?
(d) All of the above (a) Pranab Mukherjee (b) Sushma Swaraj
(e) None of these (c) Dalbir Singh Suhag (d) Narendra Modi
139. Which of the following statements about mutual funds is (e) None of these
correct? 149. World's first 'White Tiger Safari' has opened in which state
(a) Distribution of any income in advance on a monthly of India?
basis or otherwise is not permitted (a) Rajasthan (b) Madhya Pradesh
(b) Mutual funds are listed in stock exchanges (c) Gujarat (d) West Bengal
(c) Lock in period for repurchase of units should not be (e) None of these
more than 36 months 150. Who has won the 2016 Miami Open Men's singles title?
(d) All of the above
(e) None of these (a) Rafael Nadal (b) Novak Djokovic
140. Stock exchange is a place where (c) David Ferrer (d) Kei Nishikori
(a) traders buy their stocks. (e) None of these
(b) stocks are exchanged by traders and businessmen 151. Who has won the 2016 Formula 1 Gulf Air Bahrain Grand
through barter system Prix?
(c) securities are traded (a) Kimi Raikkonen (b) Lewis Hamilton
(d) shares are traded (c) Nico Rosberg (d) Daniel Ricciardo
(e) None of these (e) None of these
141. Capital Market Regulator is - 152. In which of the following industry CASA ratio is used?
(a) RBI (b) IRDA (a) Merchandise Trade (b) Banking & Finance
(c) NSE (d) BSE (c) Software Industry (d) Travel & Tourism
(e) SEBI (e) None of these
142. FDI refers to - 153. Which among the following is a opposite activity of hedging
(a) Fixed Deposit Interest in share / currency /future markets?
(b) Fixed Deposit Investment (a) arbitrage (b) speculation
(c) Foreign Direct Investment (c) spread (d) short
(d) Future Derivative Investment (e) tall
(e) None of these 154. Which among the following correctly categorizes
143. Which among the following decides the oil Prices in India ? collateralized borrowing and lending obligation?
(a) Government of India (a) A market operation
(b) Government of Respective states (b) A money market instrument
(c) Ministry of Petroleum (c) A clearing system
(d) Oil Companies (d) A scheme of Reserve Bank of India
(e) None of these (e) A policy initiative of Government of India
144. In the Rail Budget 2016-2017, Suresh Prabhu announced a 155. Who is the writer of “From Khadi to Khaki – A police man’s
new overnight double-decker, air-conditioned train. What story”?
is the name of the proposed train?
(a) P. Chauhan (b) Amit Yadav
(a) UDAY (b) Sunrise Express
(c) B. Rathnakar Rai (d) Ramanand
(c) UDIT (d) All of the above
(e) None of these
(e) None of these
www.newspaperkorner.wordpress.com
64 Practice Set - 4
www.newspaperkorner.wordpress.com

Time : 30 min. Max. Marks : 50

1. Write a letter on one of the following: (20 marks)


(a) Write a Letter to the Bank expressing your displeasure at the continually long lines at its teller counter.
(b) Write a letter asking a Bank Authorities to transfer funds between Accounts.
(c) Write a Letter Asking a bank or other Creditor to give further Consideration to your requesting for Credit after credit has
been denied you.
2. Write a paragraph on any one of the following in not more than 150 words (10 marks)
(a) Banks are manufacturers of money
(b) A day when everything went wrong
(c) Features of economic recession
3. Write an essay on any one of the following in about 250 words. (20 marks)
(a) The risks of trading at the stock market
(b) The impact of interest rate changes on the economy
(c) Responsibility of media in molding public opinion

www.newspaperkorner.wordpress.com
Practice Set - 4 65
www.newspaperkorner.wordpress.com
Answer Key
1 (e) 16 (d) 31 (e) 46 (d) 61 (b) 76 (e) 91 (b) 106 (d) 121 (b) 136 (c) 151 (c)
2 (c) 17 (a ) 32 (a ) 47 (b) 62 (b) 77 (b) 92 (e) 107 (b) 122 (a ) 137 (c) 152 (b)
3 (b) 18 (a ) 33 (b) 48 (a ) 63 (e) 78 (a) 93 (b) 108 (e) 123 (a ) 138 (d) 153 (b)
4 (a ) 19 (e) 34 (d) 49 (e) 64 (c) 79 (a) 94 (e) 109 (d) 124 (a ) 139 (d) 154 (b)
5 (b) 20 (a ) 35 (c) 50 (c) 65 (a) 80 (d) 95 (a) 110 (a) 125 (c) 140 (c) 155 (c)
6 (b) 21 (c) 36 (d) 51 (c) 66 (e) 81 (c) 96 (d) 111 (e) 126 (a ) 141 (e)
7 (d) 22 (d) 37 (c) 52 (a ) 67 (a) 82 (d) 97 (c) 112 (d) 127 (a ) 142 (c)
8 (a ) 23 (d) 38 (b) 53 (e) 68 (c) 83 (d) 98 (a) 113 (d) 128 (b) 143 (d)
9 (e) 24 (a ) 39 (d) 54 (d) 69 (b) 84 (c) 99 (a ) 114 (c) 129 (a ) 144 (a )
10 (d) 25 (b) 40 (b) 55 (a ) 70 (c) 85 (c) 100 (b) 115 (e) 130 (c) 145 (c)
11 (b) 26 (c) 41 (c) 56 (c) 71 (e) 86 (d) 101 (c) 116 (c) 131 (e) 146 (d)
12 (c) 27 (d) 42 (c) 57 (a ) 72 (e) 87 (e) 102 (b) 117 (d) 132 (d) 147 (c)
13 (d) 28 (e) 43 (a ) 58 (a ) 73 (b) 88 (b) 103 (c) 118 (b) 133 (e) 148 (d)
14 (a ) 29 (a) 44 (d) 59 (a ) 74 (a) 89 (e) 104 (d) 119 (c) 134 (e) 149 (b)
15 (c) 30 (b) 45 (e) 60 (a ) 75 (d) 90 (c) 105 (a ) 120 (a) 135 (c) 150 (b)

Answers & Explanations


1. (e) R£P£U£K conclusion]. Hence, III does not follow. No relation is
U< L given between tigers and cups and hence, I does not
follow.
I. K ³ R is true.
10. (d) “Some cats are lions” (I) + “All lions are hares” (A)
II. L > R is true.
Þ “Some cats are hares” (I) [Q I + A = I]. Hence,
2. (c) H=I£R£M conclusion IV follows. Conversion of conclusion IV
Hence, M ³ I gives conclusion III. Now, conclusion IV + “All hares
I. M = I are horses" (A) gives conclusion I [ Q I + A = I]. “All
II. M > I lions are hares" (A) + "All hares are horses" (A) gives
Either I or II is true. conclusion "All lions are horses" (A) [ Q A + A = A].
3. (b) Now, "All lions are horses" implies "Some horses are
D>H³N
lions." Hence, conclusion II also follows.
H³I<S (11 -15) :
V
I. N £ S is not true.
S Q
II. I < D is true.
4. (a) The corresponding element of the succeeding term
moves six places forward.
5. (b) One R W
6. (b) 10th to the right of 19th from right
= (19 – 10) = 9th from the right = T
7. (d) © , @ and «. T P
8. (a) Conclusions I and III make a complementary pair. Y
Hence, either I or III follows. Conclusion II does not 13. (d) Others sit in the middle of the sides.
follow since “No train is a dog” (E) + “All dogs are 1 1
parrots” (A) gives the conclusion “Some parrots are 14. (a) Move 1 , 2, 2 , 3... sides clockwise on the square.
2 2
not trains” [Q E + A O «]. Conclusion IV does not 16. (d) (a) "rem tur dimi wee" Þ "my pencil was stolen".
follow because “Some buses are trains” + “No train is (b) "soz rem legh ner" Þ "your pencil is here".
a dog” gives the conclusion “Some buses are not
dogs” [ Q I + E = O]. (c) "rem zet tur legh" Þ "blue pencil was here".
9. (e) “Some flowers are boxes” (I) + “All boxes are tigers” (d) "dimi rem tur soz" Þ "your pencil was stolen".
(A) gives conclusion “Some flowers are tigers” (I) Thus from (b) and (c), we get "rem legh" Þ pencil here"
[Q I + A = I]. On conversion, we get “some tigers are (though not necessarily in the same order.) Further, from
flowers”. Hence, conclusion II follows but IV does not (a), (b), (c) & (d) together, we get "rem" Þ "pencil". Hence,
follow. “Some cups are flowers” (I) + “Some flowers we get, "legh" Þ "here".
are boxes” (I) gives no conclusion [ Q I + I= No
www.newspaperkorner.wordpress.com
66 Practice Set - 4
www.newspaperkorner.wordpress.com
17. (a) We have, "rem" Þ "pencil". Now, from (c) & (d), we Step III: 785 32 426 198 373 96 49
get "tur" Þ "was" and hence Step IV: 785 32 426 49 198 373 96
(since "legh" Þ "here"), we get, "zet" Þ "blue". Now, Solutions for questions 31 to 35:
from (a) and(d), we get, "rem tur dimi" Þ "pencil was Date Day Play
stolen". Hence, we get, "dimi" Þ "stolen". Now, from (d), 25 Monday A
we get, 26 Tuesday Z
"soz" Þ "your". Hence, from (b), we get, "ner" Þ "is". 27 Wednesday B
Thus, "zet rem ner dimi" Þ "blue pencil is stolen". 28 Thursday No play
18. (a) From I: Saurav visited Delhi on Wednesday. 29 Friday M
From II: He visited on Tue, Wed or Thu. 30 Saturday Q
31 Sunday X
19. (e) From I & II: R’s position = 10 + 16 + 1 = 27th from left.
46. (d) The given series is × 1.5 + 1.5, × 2.5 + 2.5, × 3.5 + 3.5,
20. (a) From I :
×4.5 + 4.5, × 5.5 + 5.5.
K P N 47. (b) The series is (15 – 6) × 1 = 9, (9 – 5) × 2 = 8, (8 – 4) × 3
–E = 12, (12 – 3) × 4 = 36, (36 – 2) × 5 = 170.
N
W E Similarly, for the question row,
T a = (19 – 6) × 1 = 13, b = (13 – 5) × 2 = 16
S 48. (a) The series is × 1 – 1, × 2 – 2, × 3 – 3, × 4 – 4, × 5 – 5.
Hence, P is north-east of T. 49. (e) The series is × 2 + 22, × 3 + 32, × 4 + 42, × 5 + 52, × 6 + 62.
From II : We can’t move to any conclusion because 50. (c) The series is × 1 + 1, × 2 + 2, × 3 + 3, × 4 + 4, × 5 + 5.
town has not even been mentioned. 14800
21. (c) Since life expectancy depends on genetic make-up 51. (c) Amount = = 40, 000
0.18 + 0.19
migration can’t enhance it.
52. (a) Required difference
22. (d) If climate change is a natural phenomenon, all this
maligning of industrial development amount to ìïæ 2 üï 1.2 ´10 ´ 2
9 ö
unnecessary fuss. = 1.2 í ç 1 + ÷ - 1ý -
23. (d) The realisation of wrong priorities is leading to the ïîè 100 ø ïþ 100
shift of focus. 53. (e) Required sum
24. (a) Since industrial development is “not the only reason” = (25000 + 2375 × 2) (1.09) (1.09)
behind the problem, we need to address this as well as = ` 35345.97
other reasons. 54. (d) Required difference = SI – CI
25. (b) Industrial development is being curtailed so that the = ` 9450 – ` 7488 = 1962
said destruction can be avoided. 55. (a) Required Interest =
(26-30): From the last step it is clear that there are two alternating
series of numbers: One in descending order and the other in
ascending order.
19
30 ´
2
´ 2´
103
10 2 { 2
+ 48 + 103 (1.09 ) - 1 }
When we go through input to step I, we find that the largest no. 56. (c) Total export = 110 + 150 + 150 + 200 + 175 + 200 + 275
becomes the first and remaining numbers shift rightward. In the + 200 + 260 + 250 = 1970
next step the smallest no. becomes the second and the rest shift Total import = 275 + 250 + 225 + 225 + 270 + 200 + 175 +
rightward. These two steps continue alternately untill the two 175 + 200 + 175 = 2,170
alternate series are formed.
(2170 - 1970)
26. (c) Step II: 765 42 183 289 542 65 110 350 % decrease in export = ´100
Step III: 765 42 542 183 289 65 110 350 2170
Step IV: 765 42 542 65 183 289 110 350 = 9.2% ; 9%
27. (d) Input: 239 123 58 361 495 37 Hence, option (c) is correct.
Step I: 495 239 123 58 361 37 57. (a)
Step II: 495 37 239 123 58 361 Year Absolute diff . Year Absolute diff.
Step III: 495 37 361 239 123 58
2002 165 2008 100
28. (e) Input: 39 88 162 450 386 72 29
Step I: 450 39 88 162 386 72 29 2003 100 2009 25
Step II: 450 29 39 88 162 386 72 2004 75 2010 60
Step III: 450 29 386 39 88 162 72 2005 25 2011 75
Step IV: 450 29 386 39 162 88 72
2006 95
Step V: 450 29 386 39 162 72 88
29. (a) Last step can be known directly. 2007 0
30. (b) Step I: 785 198 32 426 373 96 49 As observed from table 2010 gets 4th rank. Hence,
Step II: 785 32 198 426 373 96 49 option (a) is correct.
www.newspaperkorner.wordpress.com
Practice Set - 4 67
www.newspaperkorner.wordpress.com Similarly in Engineering Þ
(150 - 110)
58. (a) Increase in export in 2003 = ´100
110 16 11
25780 ´ - 7390 ´ » 4124 - 813 » 3311
100 100
40
= ´100 = 36.36% Commerce Þ
110
18 16
(150 - 150) 25780 ´ - 7390 ´ » 4640 - 1182 » 3458
Increase in export in 2004 = ´100 = 0% 100 100
150
Management Þ
(200 - 150)
Increase in export in 2005 = ´ 100 = 33.3% 12 20
150 25780 ´ - 7390 ´ » 3093 - 1478 » 1615
100 100
(200 - 175) 65. (a) Required number of candidates
Increase in export in 2006 = ´100 = 12.5%
200 7390
= 23% of 7390 = 23 × » 1700
(200 - 175) 100
Increase in export in 2007 = ´100 = 14.28% (66-70):
175
Company A produces total 800 items.
(275 - 200) 5
Increase in export in 2008 = ´100 = 37.5% Company B produces ´ 800 = 1000 items.
200 4
In 2009 there is decrease. Computers Phones Pen Drives Compact Discs
(260 - 200) A. 100 270 270 160
Increase in export in 2010 = ´100 = 30%
200
In 2011 there is decrease. Computers Phones Pen Drives Compact Discs
From above it is clear that the fifth largest annual B. 240 160 200 400
percentage in export was recorded in 2007 i.e., 14.28%.
Hence, option (a) is correct. 66. (e) Required ratio = 270 : 240 = 9 : 8
59. (a) In the year 2003, 2004, 2005, 2007, 2008, 2009, 2011 270
68. (c) Required % = ´ 100 = 27%
there is no increase in import. 1000
(270 - 225) 71. (e) Number of L type products sold by
Increase in import in 2006 = ´100 = 20% Store F = 48
225
Store E = 40
(200 - 175)
Increase in import in 2010 = ´ 100 = 14.28% 48
175 Required percentage = ´ 100 = 120
In 2010 there is second largest percentage increase in 40
import. Hence, option (a) is correct. 72. (e) Required ratio = (61 + 54) : (54 + 48) = 115 : 102
60. (a) The maximum percentage increase in exports is in 2008 73. (b) Average price
i.e., 37%.
(16 + 15 + 14.5 + 5.6 + 18.2 + 14.9 ) ´103
The minimum percentage decrease in imports is in 2003 =`
6
275 - 250
i.e. = ´ 100 = 9.09% 94.2
275 =` ´ 103 = ` 15700
6
Hence, required % = 37.5 – 9.09 = 28%
74. (a) Required difference
25780 ´ 12 11 = ` (60 × 75 – 44 × 15) × 103
61. (b) Required ratio = : 7390 × = ` (4500 – 660) × 103 = ` 38.4 lakhs
100 100
75. (d) Required total amount earned by selling both products
= 3094 : 813
M and O by store C
24 = ` (57 × 5.6 + 48 × 50) × 103
62. (b) Required percentage = ´ 10 = 150
16 = ` (319.2 + 2400) × 103
63. (e) Required difference = (11 – 7)% of 7390 = ` 27.192 lakhs
76. (e) D on rent = 9.65 × 103; B on clothes = 2.75 × 103; E on
4 ´ 7390 travelling = 5.42 × 103
= = 296
100 Total expenditure = 17.82 × 103 = ` 17820
64. (c) Such difference in Science 78. (a) Between two possible options A and B, B’s expenditure
is less than A’s is clothes only.
25 32
Þ 25780 × – 7390 × Expenditures on remaining heads are more than their
100 100 A’s counterparts.
» 7218 - 2365 » 4853 Hence the answer.
www.newspaperkorner.wordpress.com
68 Practice Set - 4
www.newspaperkorner.wordpress.com or enthusiasm for doing anything. Hence, ability and
79. (a) Increased expenditure of C on children’s education
lethargy are antonymous.
105
= 12.60 ×
100 90. (c) Reality and Imagination are antonymous, because
21 imagination means a thing experienced in the mind and
Yearly expenditure = 12.6 × × 12 × 1000
20 not in reality.
= ` 1,58,760 109. (d) Here, levied additional monthly charges on consumers
is used.
4.72 472
80. (d) Required ratio = = = 59 : 105. 110. (a) Here, Despite curfew is used.
8.40 840
124. (a) Micro-credit extended by banks to individuals is
87. (e) The word self-sufficient (Adjective) means able to reckoned as a part of their priority sector lending and
fulfil one’s ownneeds, without help from others. The no particular model has been prescribed for
word self-reliant (Adjective) bears the same meaning. microfinance and banks have been extended freedom
88. (b) In the passage, the word superficial (Adjective) means to formulate their own models.
not thorough, deep or complete. The word shallow 126. (a) Cooperative Banks operate on no profit no loss basis,
(Adjective) bears the same meaning. and they operate in all sectors including agriculture
sector. NABARD is not a Cooperative Bank National
89. (e) The word ability (Noun) means the mental or physical
Bank for Agriculture and Rural Development
capacity, power or skill required to do something. The
(NABARD) is an apex development bank in India.
word lethargy (Noun) means an extreme lack of energy

www.newspaperkorner.wordpress.com
www.newspaperkorner.wordpress.com

Practice Set - 5

INSTRUCTIONS
• This practice set consists two parts. One part is Objective test and other part is Descriptive test.
• Each question has five options, of which only one is correct. The candidates are advised to read all the
options thoroughly.
• There is negative marking equivalent to 1/4th of the mark allotted to the specific question for wrong
answer.

Time : 3 hrs. Max. Marks : 200


REASONING AND COMPUTER APTITUDE 3. Statements :
A. All stairs are lifts.
Directions (Qs. 1-3): In each of these questions a few statements B. No lift is an escalator.
are followed by four conclusions numbered I, II, III and IV.
C. Some escalators are helicopters.
Consider the given statements to be true even if they seem to be
at variance with commonly known facts. Read all the D. Some lifts are planes.
conclusion(s) and then decide which of the given conclusion(s) Conclusions :
logically follow(s) from the given statements. I. No stairs is an escalator.
1. Statements: II. Some helicopters are not escalators.
A. All mirrors are phones. III. Some stairs are planes.
B. Some phones are gadgets. IV. Some helicopters are escalators.
C. All gadgets are mirrors. (a) Only I and IV follow
Conclusions : (b) Only I and either II or IV follow
I. Some gadgets are phones. (c) Either II or IV follows
II. Some gadgets are mirrors.
(d) Only I, III or IV follow
III. Some gadgets are not mirrors.
IV Some mirrors are phones. (e) None follows
(a) Only I and II follow Directions (Qs.4-7): Read the following information and five
(b) Either I or II follows statements given below it carefully and answer the questions
(c) Only II and II follow which follows:
(d) Either II or IV follow People in the remote village of Kenal are now totally self-
(e) None follows sufficient and liberal and no longer wait till people from urban
2. Statements : areas dish out empowerment as a special privilege or favour.
A. All rackets are jackets. (A) Many of the residents formed a group and initiated a self-
B. No cow is cat. help micro-credit programme which provided help and
C. Only cats are dogs. support to the villagers to start a new business.
Conclusions :
I. Some rackets are not cats (B) There was a time when the villagers had to seek favours
II. Some cats are jackets. from their urban counterparts.
III. Some rackets are cats. (C) Kenal lacks the new advancements and technological
IV. No dog is a cow. developments owing to a lack of urban intervention.
(a) Only II and IV follow (D) Many other villages are now approaching the people from
(b) Only either I or III and IV follow Kenal to help them follow the same directions.
(c) Only III and IV follow (E) Women in Kenal too are realising the importance of
(d) Only I and IV follow empowerment and are taking every possible step to add to
(e) None follows their family’s kitty.
www.newspaperkorner.wordpress.com
70 Practice Set - 5
www.newspaperkorner.wordpress.com
4. Which of the statements numbered (A) , (B), (C), (D) and Directions (Qs.12-16): In the following questions, the symbols
(E) can be inferred from the given statement? (An inference d, ¬, %, # and @ are used with the following meaning as
is something which can be directly inferred from the given illustrated below.
facts.) ‘P % Q’ means ‘P is neither greater than nor equal to Q’.
(a) Only (A) (b) Only (B) ‘P d Q’ means ‘P is neither smaller than nor equal to Q’.
(c) Only (C) (d) Only (D) ‘P @ Q’ means ‘P is not greater than Q’.
(e) Both (A) and (E)
‘P ¬ Q’ means ‘P is not smaller than Q’.
5. Which of the statements numbered (A) , (B), (C), (D) and
(E) mentioned above would prove that intervention from ‘P # Q’ means ‘P is neither greater than nor smaller than Q’.
their urban counterparts can also be beneficial to a certain Now, in each of the follwoing questions assuming the given
extent? statements to be true, find which of the four conclusions I,
(a) (A) (b) (B) (c) (C) (d) (D) (e) (E) II, III and IV given below them is/are definitely true and
6. Which of the statements numbered (A) , (B), (C), (D) and give your answer.
(E) mentioned above represents a step which helped the 12. Statement: R ¬ T, T d M, M % K, K @ V
process of liberalization and empowerment of the villagers? Conclusions
(a) (A) (b) (B) (c) (C) (d) (D) (e) (E) I. V d M II. V d T
7. Which of the following represent a result / repercussion of III. M % R IV. K d R
the success experienced by the residents of the village (a) I and II are true (b) I and III are true
Kenal?
(c) II and IV are true (d) I, III and IV are true
(a) (B) (b) (C)
(c) Both (C) and (E) (d) (A) (e) None of these
(e) (D) 13. Statement: H d J, J # N, N @ R, R d W
Directions (Qs.8-11): Study the following information carefully Conclusions
and answer the given questions. I. W % N II. W % H
P, Q, R, S, T, V, W and X are captains of eight different III. R # J IV. R d J
cricket teams, namely Australia, New Zealand, India, Pakistan, Sri (a) Only I is true (b) Only II is true
Lanka, England, West Indies and South Africa, but not necessarily
(c) Only III is true (d) Only IV is true
in the same order. All of them are seated around a circular table
(e) Either III or IV is true
and are facing the centre.
P sits third to the left of the Sri Lankan captain. Only two 14. Statement: B @ D, D d F, F % M, M ¬ N.
people sit between T and W. Neither T nor W is an immediate Conclusions
neighbour of P. Neither T and W is the captain of Sri lanka. The I. B % F II. M d D
captain of South Africa sits second to the right of S. S is not an III. N % F IV. N % F
immediate neighbour of P.S. is not the Sri Lankan captain and P is (a) None is true (b) Only I is true
not the captain of South Africa. The Australian captain sits third (c) Only II is true (d) Only III is true
to the left of V. The Australian and Sri Lankan captains are not (e) Only IV is true
immediate neighbours. Only one person sits between S and the
15. Statement: F # Z, Z @ H, H % N, N d B
Indian captain. Captains of Pakistan and New Zealand are
Conclusions
immediate neighbours. S is not the captain of New Zealand’s
team. Only one person sits between Q and the captain of England. I. F @ H II. N % Z
The captain of England is an immediate neighbour of X. W and Q III. B % H IV. B % Z
are not immediate neighbours. (a) I and III are true (b) II, III and IV are true
8. How many people sit between T and the captain of England (c) I and II are true (d) I, II and III are true
when counted in clockwise direction from T? (e) None of the above
(a) None (b) One (c) Two (d) Four (e) Five 16. Statement : M % K, K ¬ W, W d V, V @ N
9. Who is the captain of the Australian team? Conclusions
(a) P (b) V (c) W (d) T (e) Q
I. N ¬ K II. M % W
10. Which of the following would come in place of question
III. K d V IV. V % M
mark based upon the given seating arrangement?
VS XR TV RP ? (a) None is true (b) Only I is true
(a) SW (b) WX (c) QW (d) QX (e) VR (c) Only II is true (d) Only III is true
11. Which of the following is true with respect to the given (e) Only IV is true
arrangement? Directions (Qs.17-19): Below in each question are given two
(a) R is the captain of South Africa statements I and II. These statements may be either independent
(b) W is an immediate neighbour of V. causes or may be effects of independent causes or a common
(c) The captain of Australia and England are immediate cause. One of these statements may be the effect of the other
neighbours. statement. Read both the statements and decide which of the
(d) Four people sit between W and Q. following answer choices correctly depicts the relationship
(e) X sits second to the left of S. between these two statements.
www.newspaperkorner.wordpress.com
Practice Set - 5 71
www.newspaperkorner.wordpress.com
Mark answer (a) if statement I is the cause and statement II is its 22. On which day of the week does Arti’s birthday fall?
effect. I. Sonu correctly remembers that Arti’s birthday falls after
Mark answer (b) if statement II is the cause and statement I is its Wednesday but before Sunday.
effect. II. Raj correctly remembers that Arti’s birthday falls before
Mark answer (c) if both the statements I and II are independent Friday but after Tuesday.
causes. Directions (Qs.23-25): Study the following information carefully
Mark answer (d) if both the statements I and II are effects of
and answer the given questions.
independent causes.
A word and number arrangement machine when given an
Mark answer (e) if both the statements I and II are effects of some
common cause. input line of words and numbers rearranges then following a
17. I. Computer education has been made compulsory for particular rule in each step. The following is an illustration input
all the classes by many schools. and rearrangement.
II. The current job market prefers computer-literate Input: base 35 or gone 62 49 87 ahead
workforce. Step I: 87 base 35 or gone 62 49 ahead
18. I. The standard of education in evening colleges of the Step II: 87 ahead base 35 or gone 62 49
State has been deteriorating. Step III: 87 ahead 62 base 35 or gone 49
II. The standard of school education has been fast Step IV: 87 ahead 62 base 49 35 or gone
deteriorating in the State. Step V: 87 ahead 62 base 49 gone 35 or
19. I. All domestic airlines increased the fares in all sectors and Step V is the last step of the rearrangement.
with immediate effect.
As per the rules followed in the above steps, find out in each of
II. Railways increased the fare of all its classes with
the following questions the appropriate step for the given input.
immediate effect.
Directions (Qs.20-22): Each of questions below consists of a 23. Input: power fail now 52 24 75 gate 34
question and two statements numbered I and II are given below it. Which of the following steps will be the last but one?
You have to decide whether the data provided in the statements (a) IV (b) V
are sufficient to answer the question. Read both the statements (c) VI (d) VII
and (e) None of these
Give answer (a) if the data in Statement I alone are sufficient to 24. Step II of an input is: 7 down 16 24 farm eager 62 sky.
answer the question, while the data in Statement II alone are not How many more steps will be required to complete the
sufficient to answer the question. rearrangement?
Give answer (b) if the data in Statement II alone are sufficient to
(a) Four (b) Five
answer the question, while the data in Statement I alone are not
(c) Six (d) Seven
sufficient to answer the question.
Mark answer (c) if the data in Statement I alone or in Statement II (e) None of these
alone are sufficient to answer the question. 25. Input: 14 35 when they came 61 48 home
Mark answer (d) If the data in both the Statements I and II even How many steps will be required to complete the
together are not sufficient to answer the question. rearrangement?
Mark answer (e) if the data in both Statement I and II together (a) Four (b) Five
are necessary to answer the question. (c) Six (d) Seven
20. How is A related to B? (e) None of these
I. A is the sister-in-law of C, who is the daughter-in-law Directions (Qs.26-30): In each of the following questions, a
of B, who is the wife of D. related pair of figures (unnumbered) is followed by five lettered
II. B is the mother of A’s son’s only uncle’ son. pairs of figures. Out of these five, four have relationship similar
21. Point X in which direction with respect to Y?
to that in the question figure. Only one pair of figures does not
I. Point Z is equal distance from both point X and point Y.
have similar relationship. Select that pair of figures which does
II. Walking 5 km to the East of point X and taking two
not have a relationship similar to that in the question figure.
consecutive right turns after walking 5 kms before each
The letter of that pair is your answer.
turn leads to point Y.

26.
I II I II I II I II I II I II
(a) (b) (c) (d) (e)

S
27.
S

I II I II I II I II I II I II
(a) (b) (c) (d) (e)
www.newspaperkorner.wordpress.com
72 Practice Set - 5
www.newspaperkorner.wordpress.com
= === SC * C C CC CC T P SC / CC

S S S
= SC SS SS * * ** * C T P SC / PP P
* C T / / / /
28. SC C

S
S
C T T T TT
CCC T / S
I II I II I II I II I II I II
(a) (b) (c) (d) (e)

29.
I II I II I II I II I II I II
(a) (b) (c) (d) (e)

30.
I II I II I II I II I II I II
(a) (b) (c) (d) (e)
Directions (Qs.31-35): Study the following information to answer 40. With reference to a computer, an assembler is a
the given questions: (a) symbol (b) language
In a certain code ‘a friend of mine’ is written as ‘4 9 1 6’ ‘mine lots (c) program (d) grammar
of metal’ is written as ‘3 1 0 9’ and ‘a piece of metal’ written as (e) person who assembles the parts
‘7 1 6 3’ 41. To request data from the DBMS ______ is used in
31. What is the code for ‘piece’?
application programs.
(a) 3 (b) 6
(c) 1 (d) 7 (a) query language (b) DML
(e) Cannot be determined (c) DDL (d) Any of the above
32. What does ‘9’ stand for? (e) None of these
(a) of (b) mine 42. IRC stands for
(c) friend (d) lots (a) Inter Relay chat (b) Internal Relay Chat
(e) metal (c) Internet Relay Chat (d) Internet Reliable Chat
33. Which of the following may represent ‘a pleasure of mine’? (e) Internal Reliable Chat
43. Which of the following types of virus is present in Microsoft
(a) 6 3 0 9 (b) 5 2 1 6
applications?
(c) 9 2 1 6 (d) 3 6 9 4
(a) File infector virus (b) Boot sector virus
(e) 5 0 4 1
(c) Macro virus (d) Micro virus
34. What does ‘0’ stand for?
(e) E-mail virus
(a) mine (b) metal
44. Which of the following softwares is needed to satisfy a
(c) of (d) lots
user’s specific processing requirements?
(e) a
(a) DTP software (b) Graphics software
35. ‘8 7 3’ would mean
(c) System software (d) Application software
(a) a metal piece (b) metal for friend (e) DSS software
(c) piece of advise (d) friend of mine 45. Compatibility in regard to computers refers to ............. .
(e) large metal piece (a) the software doing the right job for the user
36. EBCDIC is a coding system which provides (b) it being versatile enough to handle the job
(a) 64 different characters (b) 128 different characters (c) the software being able to run on the computer
(c) 256 different characters (d) 512 different characters (d) software running with other previously installed
(e) 1024 different characters software
37. OS and Interpreter are forms of (e) None of the above
(a) hardware (b) ROM
(c) RAM (d) memory DATA ANALYSIS & INTERPRETATION
(e) software Directions (Qs.46-50): Study the following information and
38. The data on hard disk recorded in cylinders are called
answer the questions that follow :
(a) ringers (b) tracks
The premises of a bank are to be renovated. The renovation
(c) sectors (d) rounders
is in terms of flooring. Certain areas are to be floored either with
(e) None of these
marble or wood. All rooms/halls and pantry are rectangular. The
39. _____ is used by low-level computer language. area to be renovated comprises of a hall for customer transaction
(a) Mathematical symbol (b) Mnemonic measuring 23 m by 29 m, branch manager's room measuring 13 m
(c) English word (d) Grammar by 17 m, a pantry measuring 14 m by 13 m, a record keeping cum
(e) Scientific symbol server room measuring 21m by 13 m and locker area measuring 29
www.newspaperkorner.wordpress.com
Practice Set - 5 73
www.newspaperkorner.wordpress.com
m by 21 m. The total area of the bank is 2000 square meters. The 54. Swapnil, Aakash and Vinay begin to jog around a circular
cost of wooden flooring is ` 170/- per square meter and the cost stadium. They complete their revolutions in 36 seconds, 48
of marble flooring is ` 190/- per square meter. The locker area, seconds and 42 seconds respectively. After how many
record keeping cum server room and pantry are to be floored with seconds will they be together at the starting point.
marble. The branch manager's room and the hall for customer (a) 504 seconds (b) 940 seconds
transaction are to be floored with wood. No other area is to be (c) 1008 seconds (d) 470 seconds
renovated in terms of flooring. (e) None of these
46. What is the respective ratio of the total cost of wooden 55. Excluding the stoppages, the speed of a bus is 64 km/hr and
flooring to the total cost of marble flooring ? including the stoppage the speed of he bus is 48 km/hr. For
(a) 1879 : 2527 (b) 1887 : 2386 how many minutes does the bus stop per hour?
(c) 1887 : 2527 (d) 1829 : 2527 (a) 12.5 minutes (b) 15 minutes
(e) 1887 : 2351 (c) 10 minutes (d) 18 minutes
47. If the four walls and ceiling of the branch managers room (e) None of these
(The height of the room is 12 meters) are to be painted at the Directions (Qs.56-60): Study the information in the following
cost of ` 190/- per square meter, how much will be the total pie-charts to answer these questions:
cost of renovation of the branch manager's room including Details of 1500 employees working in an
the cost of flooring ? Organisation in various scales
(a) ` 1,36,800/- (b) ` 2,16,660/-
Breakup of 1500 employees
(c) ` 1,78.790/- (d) ` 2,11,940/-
(e) None of these “across the scales
48. If the remaining area of the bank is to be carpeted at the rate
of ` 110/- per square meter, how much will be the increment VII
in the total cost of renovation of bank premises ? 8% II
VI
(a) ` 5,820/- (b) ` 4,848/- 22%
14%
(c) ` 3,689/- (d) ` 6,890/-
(e) None of these V II
49. What is the percentage area of the bank that is not to be 12% 15%
renovated ? IV III
(a) 2.2 (b) 2.4 10% 19%
(c) 4.2 (d) 4.4
(e) None of these Breakup of 800 male employees
50. What is the total cost of renovation of the hall for customer “across the scales
transaction and the locker area ?
(a) ` 2,29,100/- (b) ` 2,30,206/-
(c) ` 2,16,920/- (d) ` 2,42,440/- VII I
(e) None of these VI 10% 24%
51. Rachita enters a shop to buy ice-creams, cookies and 9%
pastries. She has to buy atleast 9 units of each. She buys V
more cookies than ice-creams and more pastries than cookies. 12% II
She picks up a total of 32 items. How many cookies does IV 18%
she buy ? 11% III
(a) Either 12 or 13 (b) Either 11 or 12 16%
(c) Either 10 or 11 (d) Either 9 or 11
(e) Either 9 or10 56. How many females are working in Scale II?
52. The simple interest obtained on an amount of ` 45,000 at (a) 144 (b) 81
the end of 4 years is ` 15,300. What would be the approximate (c) 96 (d) 138
compound interest obtained on the same amount at the (e) None of these
same rate of interest in the same period?
57. What is the ratio of male to female employees working in
(a) `18,244 (b) ` 19,500
Scale V?
(c) ` 16,285 (d) ` 18,566
(e) ` 17,364 (a) 7 : 8 (b) 2 : 1
2 (c) 8 : 7 (d) 1 : 2
53. Farah got married 8 years ago. Today her age is 1 times (e) None of these
7
her age at the time of her marriage. At present her daughter’s 58. What is the total number of female employees working in
age is one-sixth of her age. What was her daughter’s age 3 Scales III and IV together?
years ago?
(a) 219 (b) 157
(a) 6 years (b) 4 years
(c) 162 (d) 285
(c) 3 years (d) Cannot be determined
(e) None of these (e) None of these
www.newspaperkorner.wordpress.com
74 Practice Set - 5
www.newspaperkorner.wordpress.com
59. The number of male employees working in Scale VI is what Percentage of the total production produced by the seven
per cent of the total number of employees in the scale? companies
(rounded off to the nearest integer)
(a) 35 (b) 46 G, A,
(c) 44 (d) 34
F, 12% 15%
5%
(e) None of these
B, 11%
60. What is the ratio of the female employees in Scale VI to the
male employees in Scale I? E, 27%
(a) 32 : 23 (b) 22 : 33 C, 22%
(c) 12 : 23 (d) 23 : 12 D,
(e) None of these 8%
Directions (Qs.61-65): Study the table carefully to answer the
Cost of the total production (both items together) by seven
questions that follow:
companies. = ` 25 crores
Total number of employees in different departments of an
organization and (of these) percentage of females and males Ratio of production between items I and II and the per cent profit
earned for tshe two items.
Total Number Percentage Percentage
Department
of Employees of Females of Males Ratio of Per cent profit
IT 840 45 55 Production earned
Accounts 220 35 65
Company
Production 900 23 77
Item I Item II Item I Item II
HR 360 65 35
A 2 3 25 20
Marketing 450 44 56
B 3 2 32 35
Customer Service 540 40 60
C 4 1 20 22
61. What is respective ratio of the number of females in D 3 5 15 25
Production department to the number of females in the
E 5 3 28 30
Marketing department?
F 1 4 35 25
(a) 22 : 23 (b) 35 : 33
(c) 23 : 22 (d) 33 : 35 G 1 2 30 24
(e) None of these 66. What is the total cost of the production of item ‘I’ by
companies A and C together in ` crore?
62. What is the ratio of the number of females in the HR and
Accounts departments together to the number of males in (a) 9.25 (b) 5.9
the same departments together? (c) 4.1625 (d) 4.9
(a) 311 : 269 (b) 268 : 319 (e) None of these
(c) 269 : 311 (d) 319 : 268 67. What is the amount of profit eared by company ‘D’ on item
(e) None of these ‘II’?
63. What is the total number of employees in all the departments (a) ` 3.125 crores (b) ` 31.25 crores
together? (c) ` 3.125 lakhs (d) ` 31.25 lakhs
(a) 3260 (b) 3310 (e) None of these
(c) 3140 (d) 3020 68. Cost of production of item ‘I’ by company ‘F’ is what percent
(e) None of these of the cost of production of item ‘II’ by company ‘D’?
64. The total number of employees in the HR department forms (a) 16% (b) 33.33%
approximately what percent of the total number of (c) 66.67% (d) 12.5%
employees in the Accounts department? (e) None of these
(a) 149 (b) 178 (c) 157 (d) 164 (e) 137 69. What is the total profit earned by company ‘G’ for items ‘I’
65. What is the total number of males in the IT and Customer and ‘II’ together?
Service departments together?
(a) ` 78 lakhs (b) ` 1.62 crores
(a) 687 (b) 678
(c) ` 7.8 crores (d) ` 16.2 lakhs
(c) 768 (d) 876
(e) None of these
(e) None of these
70. What is the ratio of cost of production of item ‘I’ by company
Directions (Qs.66-72) : Seven companies A, B, C, D, E, F and G
‘A’ to the cost of production of item ‘I’ by company ‘D’?
are engaged in production of two items I and II. The comparative
data about production of these items by the seven companies is (a) 3 : 5 (b) 1 : 2
given in the following pie-chart and the table. Study them (c) 2 : 1 (d) 2 : 3
carefully and answer the questions given below. (e) None of these
www.newspaperkorner.wordpress.com
Practice Set - 5 75
www.newspaperkorner.wordpress.com
71. What is the total of the profit earned by company ‘B’ on DIRECTIONS (Qs. 76-80) : Study the following graph and table
production of item I and the profit earned by company ‘A’ carefully and answer the questions given below :
on production of item ‘II’? TIME TAKEN TO TRAVEL (IN HOURS) BY SIX
(a) ` 9.78 crores (b) ` 97.8 lakhs VEHICLES ON TWO DIFFERENT DAYS
(c) ` 52.8 lakhs (d) ` 5.28 crores
(e) None of these
20
72. The cost of production of both items together by company
‘E’ is equal to the total cost of production of both items
together by which of the two companies? 15

TIME (IN HOURS)


(a) C and D (b) B and G
(c) A and D (d) C and F
(e) A and B 10 Day 1
Directions (Qs.73-75): In each of these questions, a question is Day 2
followed by information given in three statements. You have to 5
study the question alongwith the information given in the
statements and decide the information in which of the statement(s)
0
is/are necessary and sufficient to answer the question. A B C D E F
73. In how many days can the work be completed by A, B and VEHICLES
C together?
(I) A and B together can complete the work in 6 days.
DISTANCE COVERED (IN KILOMETERS) BY
3 SIX VEHICLES ON EACH DAY
(II) B and C together can complete the work in 3 days.
4
1 Vehicle Day 1 Day 2
(III) A and C together can complete the work in 3 days.
3 A 832 864
(a) Only I B 516 774
(b) Only II C 693 810
(c) Only III D 552 765
(d) Any one of the three E 935 546
(e) All the three statements are necessary to answer the F 703 636
question. 76. Which of the following vehicles travelled at the same speed
74. What is the total compound interest earned at the end of on both the days ?
the three years? (a) Vehicle A (b) Vehicle C
(I) Simple interest earned on that amount at the same rate (c) Vehicle F (d) Vehicle B
and period is ` 4,500
(e) None of these
(II) The rate of interest is 10 per cent per annum
77. What was the difference between the speed of vehicle A on
(III) Compound interest for three years is more than the
day 1 and the speed of vehicle C on the same day ?
simple interest for the period by ` 465.
(a) 7 km/hr. (b) 12 km/hr.
(a) Only I and II
(c) 11 km/hr. (d) 8 km/hr.
(b) Only II and III
(e) None of these
(c) Only I and III
78. What was the speed of vehicle C on day 2 in terms of meters
(d) Any two of the three
per second ?
(e) Either II or III only
(a) 15.3 (b) 12.8
75. What is the percent profit earned by the shopkeeper on
selling the articles in his shop? (c) 11.5 (d) 13.8
(I) Labelled price of the articles sold was 130% of the cost (e) None of these
price. 79. The distance travelled by vehicle F on day 2 was
(II) Cost price of each article was ` 550. approximately what percent of the distance travelled by it
on day 1 ?
(III) A discount of 10% on labelled price was offered.
(a) Only I (b) Only II (a) 80 (b) 65 (c) 85 (d) 95 (e) 90
80. What is the respective ratio between the speeds of vehicle
(c) Only III
D and vehicle E on day 2 ?
(d) All the t hree statements are necessary to answer the
(a) 15:13 (b) 17:13
question
(e) Questions cannot be answered even with the (c) 13:11 (d) 17:14
information given in all the three statements. (e) None of these
www.newspaperkorner.wordpress.com
76 Practice Set - 5
www.newspaperkorner.wordpress.com seil their wares to the highest bidder. One Solution is to accept
ENGLISH LANGUAGE
and even encourage the commoditization of knowledge; if so,
Directions (Qs. 81-90) : Read the following passage carefully Indian universities and research centres should copy their western
and answer the questions given below it. Certain words/phrases counterparts by becoming more and more like corporations. These
have been printed in bold to help you locate them while answering centres of learning should convert themselves into engines of
some of the questions. growth. In this logic, if we increase academic salaries and research
A few weeks ago, a newspaper article quoted a well known grants to match IT paycheques we will attract good people into
scientist saying, "IT has destroyed Indian science". One caji academia, where, in any case, it is rumoured that a certain elusive
speculate about the various ways in which the growth of the IT feeling called ‘the quality of life’ is better.
sector and other similar knowledge industries such as 81. According to the passage what did the scientist actually
biotechnology has led to a decline in basic scientific research in mean when he said, “IT has destroyed Indian Science?”
India.
(a) The centresmeant for Scientific research are being
The most obvious reason is money; pay scales in IT and BT are utiliized by IT industries
much higher than one can aspire to in academia. The argument
(b) The IT industry does not employ people pursuing
goes: why should a bright B. Tech, or M.Sc. Student enroll in a
higher studies
Ph.D. programme when she can make a lot more money writing
(c) As information is readily available on the internet
code? Not only does a fresh IT employee make a lot more than a
because of IT, there is no need to seek further
fresh M.Tech. Student, his/her pay will rise much faster in IT than
information
in academia. A professor's pay at a government-run university,
even after the Sixth Pay Commission, tops out at far less than a (d) IT has distorted the truth as stated by indian science
senior executive's salary in a major industry. (e) The desire for money has overshadowed the search
Second, the social Status of IT and BT Jobs equal or even exceed for knowledge
the social status of corresponding academic positions, since they 82. Which of the following is possibly the most appropriate
are seen as knowledge industries, which plays to the best and title for the passage ?
worst instincts of the societal Order. As quintessential white (a) Is the Future of IT Bright ?
collar professions, neither do they compel a successful (b) The IT Industry and the World Economy
entrepreneur to resort to violence and corruption, nor do they (c) Research and Academics — Losing the Battie
demand any physical labour. Unlike real estate or road Against IT
construction, it is feit that IT workers can become rich while (d) Scientific Research and the Need for Well — Trained
staying honest and sweat-free. Faculty
Assuming that the labour pool for academia and IT is roughly the (e) Information Technology and its Advantages
same, the difference in our collective preferences biases the labour 83. Which of the following mentioned below is/are the author’s
market towards IT and away from academia. Further, when the suggestion/s to promote interest in Indian academic ?
imbalance between IT and academia continues for years and even
(A) Research centres should adopt the corporate culture
decades, a destructive loop, from academia’s point of view, is
created. When our best and brightest take IT jobs over academic as is done in the West.
ones for a decade or more, faculty positions in our universities (B) Lessening the number of research grants given.
and research centres are no longer filled by the best candidates. (C) Making academic salaries equivalent to those paid in
As faculty quality goes down, so does the capacity to train top- IT Industries.
class graduate students who, after all, are teachers in training. In (a) Only (C) (b) Only (A)
response to decreasing faculty quality, even those students who (c) Only (B) and (C) (d) Only (A) and (C)
otherwise choose an academic profession, decide to join industry (e) None of these
Or go abroad for their studies. These foreign trained graduates 84. Which of the following is NOT TRUE in the context of the
prefer to come back to corporate India — if at all they do come
passage?
back — and the downward cycle replicates itself in each
generation. In other words, academia is trapped within a perfect (A) It is believed that the quality of life is better when
storm created by a combination of social and economic factors. pursuing scientific research.
In this socio-economic calculus, the members of our societal (B) People currently seek knowledge only for the greater
classes should prefer an IT job to an academic one. Or, to put it good of the society
another way, the knowledge economy, i.e., the creation of (C) Money is not perceived to be as powerful as
knowledge for profit, trumps the knowledge society, i.e., the knowledge.
creation of knowledge for its own sake or the sake of the greater (a) Only (A) and (C) (b) Only (B)
good. As is said, “knowledge is power, but money is even more (c) Only (A) and (B) (d) Only (B) and (C)
power.” Perheps the scientist was alluding to this victory of (e) All (A), (B) and (C)
capitalism over the pursuit of pure knowledge when he accused
85. Which of the following according to the author, are factors
IT of having a negative influence on Indian science.
responsible for the declining interest in scientific research?
Surely, knowledge has become a commodity like any other and as
(A) Slower progress of work in research
a result, knowledge workers are like any other labourers, who will
www.newspaperkorner.wordpress.com
Practice Set - 5 77
www.newspaperkorner.wordpress.com
(B) Lesser monetary compensation in research related (F) For example, while polystyrene has some excellent uses
activities and is technically, recyclable, it is not a substance that
(C) Societal perception towards research biodegrades.
(a) Only(A) (b) Only (C) 91. Which of the following should be the LAST (SIXTH)
(c) Only (B) and (C) (d) Only (A) and (B) sentence after rearrangement ?
(e) All (A), (B) and (C) (a) A (b) B (c) C (d) D (e) E
92. Which of the following should be the FOURTH sentence
86. Which of the following is true about the perception towards
after rearrangement?
IT jobs as given in the passage ?
(A) They are physically tiring. (a) E (b) F (c) A (d) D (e) B
93. Which of the following should be the FIRST sentence after
(B) They are considered to be managerial level jobs.
rearrangement?
(C) They require usage of dishonest means.
(a) A (b) B (c) C (d) D (e) E
(a) Only (B) (b) Only (A) and (B)
94. Which of the following should be the SECOND sentence
(c) Only (C) (d) Only (B) and (C)
after rearrangement?
(e) All (A), (B) and (C) are true
(a) B (b) C (c) D (d) E (e) F
Directions (Qs. 87-88) : Choose the word/group of words which 95. Which of the following should be the FIFTH sentence after
is most similar in meaning to the word /group of words printed rearrangement?
in bold as used in the passage.
(a) A (b) B (c) C (d) D (e) E
87. CAPACITY
Directions (Qs. 96-105) : In the following passage there are
(a) qualification (b) capability blanks, each of which has been numbered. These numbers are
(c) voltage (d) quantity printed below the passage and against each, five words are
(e) volume suggested, one of which fits the blank appropriately. Find out the
88. ALLUDING appropriate word in each case.
(a) referring (b) breaking Hundreds of plants and animals are (96) every day due to
(c) escaping (d) imposing deforestation and urbanization what might happen if this
(e) clinging continues in the future ? The last mass extinction of plant and
animal species occurred 65 million years ago with the Dinosaurs.
Directions (Qs. 89-90) : Choose the word/group of words which
In all, five mass extinctions have occurred and scientists (97)
is most opposite in meaning to the word/ group of words printed earth is in sixth mass extinction. The world as it is now is threatened,
in bold as used in the passage. including people, who are responsible for earth’s (98). Pesticides
89. BRIGHT contaminating water; over harvesting of animals and plants; air
(a) soft (b) dark (c) dull (d) vivid (e) dim pollution; illegal fishing and the Clearing of land are direct results
90. ELUSIVE of urbanization and deforestation. People have (99) and damaged
(a) definite (b) happy almost half a earth's land, at a very unsustainable rate.
(c) mysterious (d) worthwhile Global warming is having a serious impact as well. A six-degree
(e) remarkable Celsius increase in global temperature killed 95% of All species
on Earth 251 million years ago. An increase of six-degree Celsius
Directions (91- 95) : Rearrange the following six sentences (A),
is forecast this Century if a change is not made to (100) the
(B), (C), (D), (E) and (F) in the proper sequence to form a
damage done to earth. Humans will be one of the 95% of species
meaningful Paragraph; then answer the questions given below
lost. Noticeable, changes of global warming include migration
them. (101) and the change in season Urnings. Migrating birds are
(A) It is therefore a contributing factor to the growth of landfills migrating earlier, which in turn is causing them to hatch eggs and
and waterway, pollution, both of which are costly and (102) young earlier than they did at the beginning of this Century.
energy intensive to solve. While this is just the tip of the iceberg many other (103) regarding
(B) Making an effort to use those resources and avoid the extinction of plant and animal species need addressing. It is
polystyrene ones can help to decrease your environmental more important now than ever before to pull our heads out of the
impact. sand arid make changes for the (104) of the earth. Future
(C) Non-biodegradable essentially means that any polystyrene generations are (105), as they are a species as well.
that makes its way into a landfill will stay there indefinitely, 96. (a) killing (b) alive
never breaking down and returning to the earth. (c) born (d) left
(D) Polystyrene, as a product, is very convenient to use, but it (e) lost
has some important effects we should consider when 97. (a) speak (b) told
making choices as consumers. (c) estimation (d) believe
(E) While recycling polystyrene material can cushion the (e) consider
environmental blow of its use, alternatives are available 98. (a) shape (b) development
that are created from renewable resources and biodegrade (c) deterioration (d) warmth
more readily.
(e) expansion
www.newspaperkorner.wordpress.com
78 Practice Set - 5
www.newspaperkorner.wordpress.com
99. (a) altered (b) created 111. (A) superficial (B) superfluous
(c) produced (d) made (C) enlightened (D) surplus
(e) brought (a) A-C (b) A-B (c) B-C (d) B-D (e) A-D
100. (a) void (b) dissipate 112. (A) appalling (B) sinister
(c) argument (d) reverse (C) perturbed (D) threatening
(e) increase (a) A-B (b) B-D (c) A-C (d) A-D (e) D-C
101. (a) delay (b) birds Directions (Q. 113-115) : Which of the phrases (a), (b), (c) and
(c) slowdown (d) hasten (d) given below each statement should be placed in the blank
(e) acceleration space provided so as to make a meaningful and grammatically
102. (a) spare (b) bear correct sentence? If none of the sentences is appropriate, mark
(c) destroy (d) amend (e) as the answer.
(e) generation 113. Although information technology has entered the homes
103. (a) animals (b) difficulty offices and hearts of many citizens of India, __________ .
(c) issues (d) humans (a) India provides the highest number of IT experts to the
(e) problem world every year
104. (a) extinction (b) better (b) many people in rural areas still remain ignorant of its
(c) wealth (d) sugma immense benefits
(e) demand (c) government has done its best by funding research in
105. (a) endangered (b) threaten this field appropriately
(c) evaluated (d) living (d) the face of communication in the years to come would
(e) compared change completely from the by gone years
Directions (Qs. 106-109) : Each question below has two blanks, (e) None of these
each blank indicating that something has been omitted. Choose 114. While the environment-friendly nuclear energy could make
the set of words for each blank that best fits the meaning of the a large addition to the energy resources, __________ .
sentence as a whole.
(a) experts have a lot of expectations from this cleaner
106. After having been friends for more than a decade, they had
method of producing energy
a __________ last year and have not ________ each other
ever since. (b) the government is determined to extract maximum out
of this technology in the near future
(a) fight, talked (b) argument, met
(c) international lobby has been pressurising the
(c) dispute, seen (d) quarrel, admired
developing nations to shift their energy production
(e) difference, introduced
from coal to nuclear power.
107. The workers, several of ________had complained about
(d) the problem of locating adequate numbers of uranium
their low wages earlier, have now ________ to move the
court for the labour rights. reserves to run the reactors is yet to be sorted out
(a) who, indicated (b) whom, decided (e) None of these
(c) which, threatened (d) them, resolved 115. As allegations of crores of rupees changing hands to permit
(e) number, warned illegal mining began to fly thick and fast, _____________.
108. The hutment dwellers were jubilant when the government (a) government ordered an enquiry which exposed a nexus
__________ an apartment to each of them at rates. between mine operators and bureaucrats
(a) demolished, fast (b) announced, less (b) it caused great damage to the surrounding ecosystem
(c) provided, high (d) acquired, low and the environment in general
(e) promised, subsidised (c) the officials have been irresponsible in failing to bring
109. The organisation was deeply ____________ by difficulties it to the notice of the court in time
a decade ago but the new CEO brought many __________ (d) the powerful mining lobby had bribed the officials to |
changes in it and took it to a new high. obtain permit for mining on ecologically sensitive land
(a) indebted, necessary (b) plagued, vital (e) None of these
(c) coping, more (d) hurt, critical
GENERAL/ECONOMY/BANKING AWARENESS
(e) shaken, inevitable
Directions (Q. 110-112) : In each of the following questions four 116. Which of the following is the apex institution in the matter
words are given, of which two words are most nearly the same or of housing finance?
opposite in meaning. Find the two words which are most nearly (a) HUDCO (b) IRBI
the same or opposite in meaning and indicate the number of the (c) IFCI (d) National Housing Bank
correct letter combination. (e) None of these
110. (A) instigate (B) enquire 117. Consider the following statement with respect to FCRA:
(C) construe (D) interpret (A) It stands for Forward Contracts Regulation Act.
(a) A-C (b) A-B (c) C-D (d) B-D (e) A-D (B) In commodity exchanges in India, Index Futures are
www.newspaperkorner.wordpress.com
Practice Set - 5 79
www.newspaperkorner.wordpress.com
not permitted, as some of the provisions of the FCRA (d) Urbanisation
do not allow the same. (e) Marketing in banks is not necessary as banking in
(C) It came into existence in 1952. India is more than 200 years old
Which of the above statements is/are correct about FCRA 126. Which of the following Indian scientists was known as
in India? ‘Missile Man’ of India?
(a) Only (A) (b) Only (B) (a) H J Bhabha (b) Vikram Sarabhai
(c) Only (C) (d) Only (A) and (B) (c) APJ Abdul Kalam (d) K Kastoori Rangan
(e) All (A), (B) and (C) (e) None of these
118. Which of the following initiatives is/are taken by the 127. National income of India is compiled by
Government of India to boost the development of (a) Finance Commission
agriculture?
(b) Indian statistical Institute
(A) Accelerated Irrigation Benefit Programme
(c) National Development Council
(B) Fertiliser Subsidy Scheme
(d) Central statistical organisation
(C) Public Distribution System (PDS)
(e) None of these
(a) Only (A) (b) Only (B)
128. The book ‘A History of Ancient and Early Medieval India
(c) Only (C) (d) Only (A) and (B)
– From the Stone Age to the 12 Century’ is written by
(e) All (A), (B) and (C)
(a) Manmohan Singh (b) Upinder Singh
119. Banks need liquidity to meet which of the following
objectives of banking? (c) Shiv Shankar Menon (d) Bipin Chandra
(A) To meet deposit withdrawal (e) None of these
(B) To maintain public confidence 129. Which was the first Indian Company to be listed on
(C) To fund loan demands. NASDAQ?
(a) Only (A) (b) Only (B) (a) Infosys (b) Satyam
(c) Only (C) (d) Only (A) and (B) (c) Reliance (d) TISCO
(e) Only (A) and (C) (e) None of these
120. Who amongst the following is the author of the book The 130. As we know, the Government is paying much attention to
Sense of Ending? “Micro Finance” these days. Which of the following is
(a) Julian Barnes (b) Aravind Adiga one of the examples of Micro Finance?
(c) John Banville (d) Yann Martel (a) Insurance for life
(e) Howard Jacobson (b) Investment in Mutual Funds
121. The rate at which domestic currency can be converted into (c) Self Help Groups
foreign currency and vice versa is known as
(d) Letter of Credit
(a) Exchange rate (b) MIBOR
(e) All of these
(c) Inter-bank call money rate
131. Which of the following organisations / agencies frame the
(d) Base rate (e) LIBOR
Monetary and Credit Policy. Which is followed by all banks
122. Domestic scheduled commercial banks (other than RRBs) do
in India?
not require RBI permission to open branches in ..... centres.
(a) Indian Bank’s Association
(a) Tier-I (b) Tier-II
(b) Reserve Bank of India
(c) Tier-III (d) All the above
(c) Securities and Exchange Board of India
(e) None of these
(d) Government of India
123. Sole aim of marketing is to increase ____.
(a) sales (b) number of employees (e) None of these
(c) profits (d) production 132. The Bank Case Information System (BCIS) is being
(e) Both (a) and (c) developed by which of the following bodies, in the wake
of increasing cases of bank frauds? It would contain names
124. Marketing is a _____.
of accused persons besides the details of borrowers and
(a) day-to-day function (b) one-off affair
the public servants concerned.
(c) one-man show (d) collective process
(a) Reserve Bank of India
(e) means to earn additional income
125. Marketing is a necessity today due to _____. (b) Indian Bank’s Association
(a) Liberalisation (c) Ministry of Finance
(b) Nationalisation (d) Central Statistical Organisation
(c) Fashion (e) Central Bureau of Investigation
www.newspaperkorner.wordpress.com
80 Practice Set - 5
www.newspaperkorner.wordpress.com
133. As we know, the Government is paying much attention to 140. Who can make investment in a commercial paper?
“Micro Finance” these days. Which of the following is one (a) Individuals
of the examples of Micro Finance? (b) Banking companies
(a) Insurance for life (c) Corporate bodies registered or incorporated in india
and unincorporated bodies
(b) Investment in Mutual Funds
(d) None Resident Indians (NRIs) and Foreign Institutional
(c) Self Help Groups Investor (Flls)
(d) Letter of Credit (e) All of the above
(e) All of these 141. A commercial paper is
134. Fiscal deficit in the Union Budget means (a) a paper issued by Reserve Bank of India an trade and
(a) The difference between crucial expenditure and current commerce
revenue. (b) an unsecured money market instrument issued in the
(b) The sum of budgetary deficit and net increase in from of a promissory note
internal and external borrowings (c) a document issued by IDBI for seeking refinance facility
(c) Net increase in union government borrowings from from Reserve Bank India
Reserve Bank of India
(d) The sum of monetized deficit and budgetary deficit. (d) All of the above
(e) None of these (e) None of these
135. DTAA stands for 142. The term ‘moral suasion’ refers to
(a) Direct Tariff Avoidance Agreement (a) The moral duty of a borrwoer to deal with only one
(b) Double Taxation Avoidance Agreement bank
(c) Direct Taxation Avoidance Agreement (b) The advice given by the Reserve Bank of India to
(d) Double Tariff Avoidance Agreement banks/financial institutions in the matter of heir lending
(e) None of these and other operations with the objective that they might
136. The National Rural Employment Guarantee Act provides implement or follow it
for at least how many days of wage employment to every (c) The banker’s duty of secrecy as regards the affairs
rural household in a year? and accounts of his customers
(a) 90 days (b) 180 days (d) All of the above
(c) 100 days (d) 300 days (e) None of these
(e) None of these 143. Land Development Banks provide long-term credit for
137. The term ‘bogey’ is associated with schemes of basic importance to agriculture as
(a) Cricket (b) Chess
(a) mechanization of agriculture i.e., loans for purposes of
(c) Golf (d) Baseball
tractors, power tillers, threshers etc
(e) None of these
(b) land reclamation, soil conservation, plantation of fruit
138. Net Asset Value (NAV) of a scheme is a number which
orchards, dairy development schemes etc, involving
basically represents the value of rupees per fund units as heavy expenditure
on a particular date of the assets fo the fundless liabilities (c) minor irrigation purposes like wells, dug-cum-bore
and outstanding expenses. It is represented as wells, tube wells pump sets and irrigation tanks
Assets of the fund – Liabilities
(d) Only (a) and (b)
+ Outstanding Expenses (e) All of the above
(a) Number of Fund Units Assets of the fund +Liabilities 144. Through which of the following the funds to the projects
+ Outstanding Expenses aided by the World Bank and the International Monetary
(b) Number of Fund Units Assets of the fund +Liabilities Funds (IMF) are distributed?
(a) Regional Branches of RBI
–Outstanding Expenses
(c) (b) NABARD
Number of Fund Units (c) Integrated Rural Development Programme
(d) Any one of the above (d) Agriculture Refinance and Development Corporation
(e) None of these (e) Ministry of Finance
139. What are the benefits of credit card for its holder? 145. Union Budget which is presented every year in the
(a) Card holder is relieved of the risk/botheration/tension parliament is also known as
of carrying cash while staying in hotels, dining in (a) Annual Budget (b) Common Budget
restaurants, shipping buying air (c) General Budget (d) Finance Report of the year
(b) Card holder can draw money from any branch of the (e) Statement of Allocation of funds
issuing bank 146. Which of the following terms is NOT used in Banking/
(c) Card holder can get a consolidated statement for all Finance related matters?
the transactions (a) Discount Rate (b) Letter of Credit
(d) All of the above (c) Cheque Clearing (d) General Ledger
(e) None of these (e) Law of Motion
www.newspaperkorner.wordpress.com
Practice Set - 5 81
www.newspaperkorner.wordpress.com
147. Which of the following organizations/agencies works solely 152. In the Rail Budget 2016-2017, Suresh Prabhu announced
to monitor and arrange flow of agriculture credit in India? three pillars of the strategy. Name the three pillars.
(a) NABARD (b) SIDBI (a) Reorganize, Restructure Rejuvenate Indian Railways:
(c) RBI (d) SEBI 'Chalo, Milkar Kuch Karen'
(e) None of these (b) Chalo, Milkar Kuch Naya Karen: 'Navinikaran,
148. Foreign currencies are represented in various abbreviations. Sashaktikaran, Shodh aur Vikas'
What is meant by USD? (c) Nav Arjan - New revenues, Nav Manak - New norms,
(a) UAE Dinar Nav Sanranchna - New structures
(b) US Dollar (d) Reorganize, Restructure Rejuvenate Indian Railways:
(c) Unit of Currency of Denmark 'Nav Arjan, Nav Manak, Nav Sanranchna'
(d) Ukrainian Soviet Dollar (e) None of these
(e) None of these 153. Name the proposed structure scheme announced in the
149. In which year, Indian Rupee was devalued for the first time? Union Budget 2016-17 to help Panchayat Raj Institutions
(a) 1965 (b) 1966 deliver Sustainable Development Goals.
(c) 1968 (d) 1971 (a) Rashtriya Gram Abhiyan
(e) None of these (b) Rashtriya Gram Swaraj Abhiyan
150. On which date, RBI started working? (c) National Village Self-governance Scheme
(a) 1 January 1934 (b) 1 January 1935 (d) Rashtriya Gram Swasashan Abhiyan
(c) 1 April 1935 (d) 31 March 1934 (e) None of these
(e) None of these 154. Dame Zaha Hadid, who died recently was a world-renowned
151. In the Rail Budget 2016-2017, a new service named Janani __:
Sewa will be introduced. What is the aim of this service? (a) Architect (b) Painter
(a) To introduce children's menu items on trains, baby (c) Playwright (d) Musician
foods, hot milk and hot water (e) None of these
(b) To introduce medical facilities for pregnant women on 155. Bedaquiline, a newly launched vaccine by Health Ministry,
trains is related with the treatment of which disease?
(c) To introduce women and child security on train (a) Zika Virus disease (b) Tuberculosis
(d) All of the above (c) Ebola virus disease (d) Swine flu
(e) None of these (e) None of these

www.newspaperkorner.wordpress.com
82 Practice Set - 5
www.newspaperkorner.wordpress.com

Time : 30 min. Max. Marks : 50

1. Write a letter on any one of the following topic: (20 marks)


(a) Write a letter to assistant manager of local power station telling him about the frequent power cuts in your area and
asking him to take action in order to solve the problem
(b) Write a letter to the editor of a newspaper, expressing your deep concern on the impact of adult graded cinemas on
young India.
(c) Write a letter to the high ranking official in a Bank complaining about the rude staff at the teller desks, the loan counters,
etc.
2. Write a paragraph on any one of the following in not more than 150 words : (10 marks)
(a) Why do we need to conserve resources?
(b) Unemployment Allowance.
(c) Students and Politics.
3. Write an essay on any one of the following in about 250 words : (20 marks)
(a) Government financial aid for students should be based solely on merit.
(b) People have become overly dependent on technology.
(c) Corruption is the grease that lubricates the squeaky gate?

www.newspaperkorner.wordpress.com
Practice Set - 5 83
www.newspaperkorner.wordpress.com
Answer Key
1 (e) 16 (d) 31 (d) 46 (c ) 61 (c) 76 (d) 91 (b) 10 6 (c) 1 21 (a ) 1 36 (c) 1 51 (a )
2 (b) 17 (b) 32 (b) 47 (c ) 62 (a ) 77 (c) 92 (c ) 10 7 (b) 1 22 (d) 1 37 (c) 1 52 (c )

3 (b) 18 (e) 33 (c) 48 (e) 63 (b) 78 (e) 93 (d) 10 8 (e) 1 23 (e) 1 38 (a ) 1 53 (b)
4 (b) 19 (e) 34 (d) 49 (b) 64 (d) 79 (e) 94 (e) 10 9 (b) 1 24 (d) 1 39 (d) 1 54 (a )

5 (c) 20 (e) 35 (e) 50 (a ) 65 (e) 80 (b) 95 (e) 11 0 (c) 1 25 (a ) 1 40 (e) 1 55 (b)


6 (a ) 21 (b) 36 (c) 51 (c ) 66 (b) 81 (e) 96 (e) 11 1 (d) 1 26 (b) 1 41 (b)

7 (e) 22 (e) 37 (e) 52 (e) 67 (d) 82 (c ) 97 (d) 11 2 (d) 1 27 (d) 1 42 (b)


8 (c) 23 (c) 38 (b) 53 (c ) 68 (e) 83 (d) 98 (c ) 11 3 (b) 1 28 (b ) 1 43 (e)

9 (a ) 24 (a ) 39 (b) 54 (c ) 69 (a ) 84 (e) 99 (a ) 11 4 (d) 1 29 (a ) 1 44 (d)


10 (b) 25 (c) 40 (c) 55 (b) 70 (c) 85 (c ) 10 0 (d) 11 5 (a ) 1 30 (c) 1 45 (c)

11 (c) 26 (a ) 41 (a ) 56 (b) 71 (b) 86 (a ) 10 1 (e) 11 6 (d) 1 31 (b) 1 46 (e)


12 (b) 27 (b) 42 (c) 57 (c ) 72 (d) 87 (b) 10 2 (b) 11 7 (e) 1 32 (e) 1 47 (a )

13 (e) 28 (e) 43 (c) 58 (a ) 73 (e) 88 (a ) 10 3 (c ) 11 8 (d) 1 33 (c) 1 48 (b)


14 (a ) 29 (d) 44 (d) 59 (d) 74 (e) 89 (c ) 10 4 (c ) 11 9 (e) 1 34 (b) 1 49 (b)

15 (c) 30 (c) 45 (c) 60 (e) 75 (b) 90 (a ) 10 5 (a ) 12 0 (a ) 1 35 (b) 1 50 (c)

Answers & Explanations


(1 -3 ):
According to statements: 1. (e) P
M
G
Gliders Airplane

Parachutes Helicopters
None of the conclusions follows.
or
jackets
cats
2. (b) rackets cows dogs

Gliders Airplane

Parachutes s
opte r L
Helic E
P
3. (b) S H

I, II or IV follow.

Airplane 4. (b) Sentence B is an obvious inference because of the use


Gliders
of words ‘are now’ in the first sentence of the given
Parachutes paragraph. That means, the village was earlier not self
sufficient.
www.newspaperkorner.wordpress.com
84 Practice Set - 5
www.newspaperkorner.wordpress.com
5. (c) Lack of new advancements and technologies 15. (c) F # Z Þ F = Z; Z @ H Þ Z £ H; H % N Þ
developments because of a lack of urban intervention H < N, N d B Þ N > B
proves that intervention from urban counterparts can So, F = Z £ H < N > B
lead to new advancements and technological Conclusions
developments and benefit the village. I. F @ H Þ F £ H (True)
6. (a) The formation of groups by residents to initiate self- II. N % Z Þ N > Z (True)
help micro-credit programme, as mentioned in sentence III. B % H Þ B < H (False)
A, represents the step which helped the process of IV. B % Z Þ B < Z (False)
liberalisation and empowerment of the villages.
Only I and II are true.
7. (e) Success experienced by the residents of the village 16. (d) M % K Þ M < K; K ¬ W Þ K ³ W; W d V Þ
Kenal has made them idols to be pursued for. This W > V, V @ N Þ V £ N
result is aptly depicted in sentence D. So, M < K ³ W > V £ N
(8 - 11):
Conclusions
S (Pakistan)
W (New Zealand) I. N ¬ K Þ N ³ K (False)
V (Sri Lanka)
II. M % W Þ M < W (False)
III. K d V Þ K > V (True)
X (South Africa) T (India)
IV. V % M Þ V < M (False)
Only III is true.
R (England) Q (West Indies)
17. (b) Job market is an important consideration in determining
P (Australia)
the curriculum of schools.
10. (b) There is pattern of going from second member of a pair
18. (e) There seems to be some common cause that is leading
to the first member of the next pair: +2, +3, +4 ... CW.
to deterioration in both kinds of education.
(12-16):
19. (e) Hike in fuel prices seems to be the common cause.
P%QÞP<Q
PdQÞP>Q 20. (e) From I: A is female
P@QÞP£Q Using this in II, we get:
P¬QÞP³Q B = mother of A’s son’s only uncle’s son
P#QÞP=Q = mother of A’s son’s cousion
12. (b) R ¬ T Þ R ³ T; T d M Þ T > M; M % K Þ M = mother of A’s brother-in-law’s wife.
<K
21. (b) Statement I is of no use because we don’t know the
K@VÞK£V
directions.
So, R ³ T > M < K ³ V
Conclusions Statement II is sufficient because distances and
I. V d M Þ V > M (True) directions have been given propertly.
II. V d T Þ V > T (False) 22. (e) From I: Arti’s birthday falls on Thu, Fri or Sat.
III. M % R Þ M < R (True) From II: It falls on Sat, Sun or Mon
IV. K d R Þ K > R (False) Combining I and II, it falls on Sat.
Only I and III are true. 23. (c) Input: power fail now 52 24 75 gate 34
13. (e) H d J Þ H > J; J # N Þ J = N; N @ R Þ N £ R
Step I: 75 power fail now 52 24 gate 34
RdWÞR>W
So, H > J = N £ R > W Step II: 75 fail power now 52 24 gate 34
Conclusions Step III: 75 fail 52 power now 24 gate 34
I. W % N Þ W < N (False) Step IV: 75 fail 52 gate power now 24 gate 34
II. W % H Þ W < H (False) Step V: 75 fail 52 gate 34 power now 24
III. R # J Þ R = J Step VI: 75 fail 52 gate 34 now power 24
IV. R d J Þ R > J
Step VII: 75 fail 52 gate 34 now 24 power
Only either III or IV is true.
14. (a) B @ D Þ B £ D; D d F Þ D > F; F % M Þ Hence step VI will be the last but one.
F < M, M ¬ N Þ M ³ N 24. (a) Input:
So, B £ D > F < M ³ N Step II: 75 down 16 24 farm eager 62 sky
Conclusions Step III: 75 down 62 16 24 farm eager sky
I. B % F Þ B < F (False)
Step IV: 75 down 62 eager 16 24 farm sky
II. M d D Þ M > D (False)
III. N % F Þ N < F (False) Step V: 75 down 62 eager 24 16 farm sky
IV. D d N Þ D > N (False) Step VI: 75 down 62 eager 24 farm 16 sky
So, none of the given conclusions is correct. Hence 6 – 2 = 4 more steps will be required.
www.newspaperkorner.wordpress.com
Practice Set - 5 85
www.newspaperkorner.wordpress.com
25. (c) Input: 14 35 when they came 61 48 home 49. (b) percentage area of bank not to be renovated
Step I: 61 14 35 when they came 48 home
Area bank not be renovated 48
Step II: 61 came 14 35 when they 48 home Þ Þ ´ 100 = 2.4%
Total area of bank 2000
Step III: 61 came 48 14 35 when they home
50. (a) Total cost of hall of customer transaction
Step IV: 61 came 48 home 14 35 when they
= `(170 × 667) = `113,390
Step V: 61 came 48 home 35 14 when they
Total cost of Locker area = `(190 × 609) = `115710
Step VI: 61 came 48 home 35 they 14 when
26. (a) The upper 1 left element rotates by 90° CW while the Total cost of customer transaction hall + locker area
other four rotate by 90° ACW. = `(113390 + 115710) = `229100
27. (b) The whole figure rotates by 90° ACW. The first and 51. (c) By options
the third, and the second and the fourth elements (a) Either 12 or 13
interchanged places. then ice-creams should not be given atleast 9. This
28. (e) The first column from left becomes the second row can be rejected.
from top. The second column from left becomes the (b) Either 11 or 12
lower row. The third column goes to the top while the
Ice-cream should be atleast 9. By this combination ice
fourth column becomes the third row. One element each
cream gets less than 9.
is added in the first and second row from top while one
element each is reduced from the third and fourth row (c) Either 10 or 11
from top. By giving cookies 10 or 11, we get all the possible
29. (d) The whole figure rotates by 45° ACW while the end condition fulfilled.
element gets inverted. (d) and (e), the ice-cream distribution can be more than
30. (c) The upper left rotates by 45° CW and the lower left cookies which violates our condition.
rotates by 45° ACW. The upper right rotates by 135° \ option (c) is the right answer.
CW and the lower right by 135° ACW. = 1040 – 920 = 120
(31-35) : 52. (e) SI = ` 15,300
a friend of mine = 4 9 1 6 ...(i) Time = 4 years
mine lots of metal = 3 1 0 9 ...(ii) Principal = ` 45,000
a piece of metal = 7 1 6 3 ...(iii)
From (i), (ii) and (iii), of = 1 ...(iv) 15300 ´100
Rate of interest = = 8.5%
From (i), (iii) and (iv), a = 6 ...(v) 45000 ´ 4
From (i), (ii), and (iv), mine = 9 ...(vi) ìï é 8.5 ù 4 üï
From (i), (iv), (v) and (vi), friend = 4 ...(vii) CI = 45000 í ê1 + ú - 1ý
From (ii), (iii) and (iv), metal = 3 ...(viii) ïî ë 100 û ïþ
From (ii), (iv), (vi) and (viii), lots = 0 ...(ix) = 45000 [1.385 – 1] = 45000 × 0.385
From (iii), (iv), (v) and (viii), piece = 7 ... (x) = `17325 » `17364
46. (c) Area of customer transaction room = 23m×29m = 667 sq.m
Area of branch manager room = 13m × 17 m = 221 sq. m 9
53. (c) Age of Farah = x = ( x - 8 ) ´
Area of Pantry room = 14m × 13m = 182 sq. m 7
Area of Server room = 21m × 13m = 273 sq. m 9 ( x - 8)
Þx=
Area of locker room = 29 m × 21 m = 609 sq. m 7
Total cost of wooden flooring = ` [(170 × (667 + 221)] Þ 7x = 9x – 72 Þ 2x = – 72
= ` (888 × 170) 72
Total cost of marble flooring Þx= = 36 years
2
= ` [(190 × (182 + 273 + 609)] = ` (190 × 1064)
36
Required Ratio = 888 × 170 : 1064 × 190 = 1887 : 2527 Present age of her daughter = = 6 years
6
47. (c) Area of 4 walls and ceiling of branch managers room \ Age of daughter 3 years ago
= 2 (lh + bh) + lb = 2 [17 × 12 + 13 × 12] + 13 × 17
= 6 – 3 = 3 years
= 941 sq. m
54. (c) LCM of 36 sec, 48 sec and 42 sec = 1008 sec
Total cost of renovatin = `190 × 941 = `178790
\ After 1008 seconds, they will be together at the
48. (e) Total area of bank is 2000 sq. m starting point.
Total area of bank to be renovated = 1952 sq. m 55. (b) Stoppage time per hour
Remaining Area = 2000 – 1952 = 48 sq. m
Total cost Remaining Area to be carpeted at the rate of 64 - 48 1 1
= = ´60 = hr = 15 minutes
`110/sq. meter = `(48 × 110) = `5280 64 4 4
www.newspaperkorner.wordpress.com
86 Practice Set - 5
www.newspaperkorner.wordpress.com
56. (b) Required number of females 67. (d) Cost of production of both items for company D
15 18 81
= 1500 ´ - 800 ´ = 225 - 144 = 81 = ´ 25 = ` 2 crores
100 100 100
57. (c) The ratio is Cost of production of item II for company D
12 æ 12 12 ö 5 5
800 ´ : ç1500 ´ - 800 ´ ÷ = ´ 2 = ` crores
100 è 100 100 ø 3+ 5 4
= 96:84 = 8 : 7 % profit earned by company D on item II = 25%
58. (a) The number is \ Amount of profit earned by company D on item II.

29 27 25 5 5
1500 ´ - 800 ´ = 435 - 216 = 219 = ´ =` crores
100 100 100 4 16

72 5
59. (d) Percentage = ´ 100 =` × 100 lakhs
15 ´14 16
= ` 31.25 lakhs
240
= » 34% 68. (e) From solution of Q. No. 82. We have the cost of
7
5
60. (e) Required ratio is production of item II by company D = ` crores.
4
(210 – 72) : 192 = 138 : 192 = 23 : 32
61. (c) Required ratio Cost of production of both items of company F.

23 44 5 5
= ´ 25 = ` crores
= 900 ´ : 450 ´ = 23 : 22 100 4
100 100
62. (a) Required ratio Cost of production of item I for company F.

æ 35 65 ö æ 65 35 ö 1 5 1
= ç 220 ´ + 360 ´ = ´ = ` crores
÷ : ç 220 ´ + 360 ´ ÷ 1+ 4 4 4
è 100 100 ø è 100 100 ø
= (22 × 35 + 36 × 65) : (22 × 65 + 36 × 35)
1
= (22 × 7 + 36 × 13) : (22 × 13 + 36 × 7) = 311 : 269 1
4
63. (b) Total no. of employees Required % = 5 ´100 = × 100 = 20%
5
= (840 + 220 + 900 + 360 + 450 + 540) = 3310 4
360 69. (a) Cost of production of both items for company G.
64. (d) Required % = ´ 100 » 164%
220
65. (e) Total no. of male employees in IT and Customer Service 12
= ´ 25 = ` 3 crores
100
55 60
= 840 ´ + 540 ´ = 462 + 324 = 786 Cost of production of item I for company G.
100 100
66. (b) Cost of production of both items for 1
= ´ 3 = ` 1 crore
1+ 2
15
Company A = ´ 25 = 3.7 crores Cost of production of item II for company G.
100
= ` (3 – 1) crores
22
Company C = ´ 25 = ` 5.5 crores = ` 2 crores
100
These costs will be divided in the ratio of production Profit of item I for company G = 30%
of items I and II. Profit of item II for company G = 24%
Cost of production of item I for \ Amount of profit earned on item
2 30
Company A = ´ 3.75 = ` 1.5 crores I= × 1 = ` 0.3 crores
2+3 100
4 Amount of profit earned on II
Company C = ´ 5.5 = ` 4.4 crores
4 +1 24
= ´ 2 = ` 0.48 crores
\ Total cost of production of item 1 by companies A 100
and C together
\ Total profit earned by company G for items I and II
= ` (1.5 + 4.4) crores = ` 5.9 crores together
www.newspaperkorner.wordpress.com
Practice Set - 5 87
www.newspaperkorner.wordpress.com
= ` (0.3 + 0.48) crores denoted by x, y and z respectively.
= ` 0.78 crores 1
= ` 78 lakhs. \ x+ y=
6
70. (c) From solution to Q. No. 81. we have the cost of 4
production of item I by y+ z =
15
Company A = ` 1.5 crores
3
Cost of production of both items by company D x+z=
10
8 Adding above equations we get,
= ´ 25 = ` 2 crores
100 1 4 3
2(x + y + z) = + +
Cost of production of item I by company D 6 15 10
5+8+9
3 3 2(x + y + z) =
= ´ 2 = ` crores = ` 0.75 crores 30
3+ 5 4
22
Required ratio = 1.5 : 0.75 = 2 : 1 2(x + y + z) =
30
71. (b) Cost of production of both items by company B
11
x + y+ z =
11 11 30
= ´ 25 = ` crores.
100 4 Therefore, (A + B + C) together can complete the work
30
Cost of production of item I by company B in
11
3 11 33 8
= ´ =` crores = 2 days.
3+ 2 4 20 11
Profit on item for company B = 32% 74. (e) Simple interest is the interest earned only on the
\ Amount of profit eared by company B on item I. Principal sum. But compound interest is equal to the
sum of the Simple interest and the interest earned on
32 33 66 this Simple interest during this period. So, we should
= ´ =` crores = 0.528 crores
100 20 125 know the amount of the Simple interest and rate of
Cost of production of both items by company A interest to calculate the additional interest or the
difference between them or, we should know the
15 15 difference in two interests and the Simple interest. Only
= ´ 25 = ` crores = ` 3.75 crores
100 4 then we can find the amount of Compound interest
during that period. Therefore, either statement II or III
Cost of production of item II by company A
can be dispensed with.
3 When statements I and II are used.
= ´ 3.75 = ` 2.25 crores
2+3
Profit on item II for company A = 20% é r 2 + 300r + 30000 ù
C.I. = S.I. ê ú
\ Amount of profit earned by company A on item II 30000
ëê ûú
20
= ´ 2.25 = 0.45 crores
100 é100 + 3000 + 30000 ù
\ Required total profit = 4500 ê
30000 ú = ` 4965
ë û
= ` (0.528 ¸ 0.45) crores. When statements I and II are used,
= ` 0.978 crores C.I. = S.I. + Difference in two interest.
= ` 97.8 lakhs = 4500 + 465 = ` 4965.
72. (d) Cost of production of both items by company E = 27%
75. (b) Statement II can be dispensed with as only per cent
of the total cost of 7 companies.
profit is to be found.
Cost of production of both items by company C = 22%
Let the cost price be x.
of the total cost of 7 companies.
\ Labelled price = 1.30 x.
Cost of production of both items by company F = 5%
of the total cost of 7 companies. Selling price after 10% discount on labelled price
73. (e) The question has 3 variables, i.e. the rate of work done = 0.9x × 1.30x = 1.17x
per day by each A, B and C. We need to have 3 1.17x - x
% profit = ´ 100 = 17%
equations to solve the problem. This is possible only x
by using all the 3 statements given. Thus, we see that the value of the cost price is not
Let the rate of work done per day by A, B and C be required.
www.newspaperkorner.wordpress.com
88 Practice Set - 5
www.newspaperkorner.wordpress.com
(76-80): 77. (c) Speed of vehicle A on day 1 = 52 km/hr
Day 1 Day 2 Speed of vehicle C on day 1 = 63 km/hr
Vehicle Time Distance Speed in Time Distance Speed in Difference = 63 – 52 = 11 km / hr
in hr in km km / hr in hr in km km/hr
78. (e) Speed of vehicle can day 2 = 45 km/hr
A 16 832 52 16 864 54
æ 5ö
Þ ç 45 ´ ÷ m / sec =12.5 m / sec
B 12 516 43 18 774 43 è 18 ø
79. (e) Percentage
C 11 693 63 18 810 45
Distance travelled by vehicle F on day 2
51 = ´ 100
D 12 552 46 15 765 Distance travelled by vehicle F on day 1

E 16 935 58.4 14 546 39 636 630


= ´ 100 » ´ 100 » 90%
703 700
F 19 703 37 12 636 53
80. (b) Speed of vehicle D on day 2 = 51
76. (d) Vehicle B. Speed of vehicle E on day 2 = 39
51 17
Required ratio = = or17:13
39 13

www.newspaperkorner.wordpress.com
www.newspaperkorner.wordpress.com

Practice Set - 6

INSTRUCTIONS
• This practice set consists two parts. One part is Objective test and other part is Descriptive test.

• Each question has five options, of which only one is correct. The candidates are advised to read all the
options thoroughly.

• There is negative marking equivalent to 1/4th of the mark allotted to the specific question for wrong
answer.

Time : 3 hrs. Max. Marks : 200

REASONING AND COMPUTER APTITUDE Volleyball, Archery, Rifle Shooting, Tennis, Boxing, Athletics and
Football. The order of persons, countries and games is not necessarily
Directions (Qs. 1-3): Each of the questions below consists of a the same.
questions and two statements numbered I and II given below it. K represents China for Archery. T represent USA but not
You have to decide whether the data provided in the statements for Volleyball or Rifle Shooting. The one who represents Japan
are sufficient to answer the question. competes for Boxing. F competes for Volleyball but not for Korea.
Read both the statements and — Give answer L represent Australia for Athletics. The one who represents Russia
(a) if the data in statement I alone are sufficient to answer the competes for Tennis. J does not represent Korea or Japan. R
question, while the data in statement II alone are not competes for Rifle Shooting.
sufficient to answer the question. 4. Which of the following combinations is correct?
(b) if the data in statement II alone are sufficient to answer the (a) J - Tennis - France (b) R - Tennis-Russia
question, while the data in statement I alone are not sufficient (c) R - Tennis - France (d) J - Tennis - Russia
to answer the question. (e) None of these
(c) if the data either in statement I alone or in statement II alone 5. Who represents Japan?
are sufficient to answer the question. (a) F (b) R
(d) if the data in both the statements I and II together are not (c) J (d) H
sufficient to answer the question. (e) None of these
(e) if the data in both the statements I and II together are 6. F represents which country?
necessary to answer the question. (a) France (b) Russia
1. Kiran is older than Manoj and Dilip is older than Neelam. (c) Japan (d) Korea
Who among them is the youngest? (e) None of these
I. Kiran is older than Neelam. Directions (Qs. 7-9) : Study the following information carefully
II. Manoj is younger than Dilip. and answer the given questions.
2. What is the relation between M and F?
I. M has two sons, one of whom is B. Any further increase in the population level in the city by
way of industrial effluents and automobile exhaustions would
II. The mother of F has two sons D and B.
pose a severe threat to the inhabitants.
3. H is in which direction with respect to V?
(A) All the factories in the city should immediately be closed
I. S is to the south of K, who is to the west of V.
down.
II. M is to the north of H, who is to the east of V.
(B) The automobiles should not be allowed to ply on the road
Directions (Qs.4-6): These questions are based on the following for more than four hours a day.
information. Study it carefully and answer the questions. (C) The government should restrict the issue of fresh licences
Seven members L, H, K, T, F, J and R represent different to factories and automobiles.
countries in Olympics viz, USA, China, Korea, France, Russia, (D) Cancer, heart attacks, brain strokes, tuberculosis are the major
Australia and Japan; each one competes for a different sport, viz. disease which are rapidly increasing in industrial cities.
www.newspaperkorner.wordpress.com
90 Practice Set - 6
www.newspaperkorner.wordpress.com
(E) All types of pollutants are very harmful for health. 12. Who is sitting opposite A?
(F) Excessive growth of industries has increased the pollution (a) G (b) D
level in the city. (c) E (d) A
7. Which of the following among (A), (B), (C) and (D) can be (e) None of these
an immediate course of action for the Government? 13. Who is next to E in clockwise direction?
(a) Only (A) (b) Only (B) (a) G (b) B
(c) Only (C) (d) Only (D) (c) F (d) A or F
(e) All of these (e) None of these
8. Which of the following among (A), (B), (C) and (D) can not 14. Which of the following pairs of persons has both the
be an immediate course of action for the government? persons sitting on the same side with first person sitting to
(a) Only (A) (b) Only (B) the right of second person?
(c) Both (A) and (B) (d) Only (D) (a) DF (b) CB
(e) None of these (c) FC (d) AG
9. Which of the following (A), (B), (D) and (E) may be the (e) None of these
effect of increment in the pollution level in the city? 15. Who is sitting opposite E?
(a) Only (A) (b) Only (B) (a) D (b) A
(c) Only (D) (d) Only (E) (c) F (d) A or D
(e) None of these (e) None of these
10. Cause: Govt has recently decided to hike the procurement 16. Which of the following statements is definitely true?
price of paddy for the rabi crops. (a) A is facing North (b) E is sitting opposite F
Which of the following will be a possible effect of the above (c) F is the left of G (d) C is to the left of A.
cause? (e) None of these
(a) The farmers may be encouraged to cultivate paddy for
Directions (Qs. 17-21): In the following questions, the symbols
the rabi season.
c , @ ,$, % and are used with the following meanings as
(b) The farmers may switch over to other cash crops in
illustrated below:
their paddy fields.
(c) There was a drop in production of paddy during Kharif ‘P @ Q’ means ‘P is not greater than Q’
season ‘P % Q’ means ‘P is not smaller than Q’
(d) Govt may not increase the procurement price of paddy ‘P Q’ means ‘P is neither smaller than nor equal to Q’.
during the next Kharif season. ‘P c Q’ means ‘P is neither greater than nor equal to Q’.
(e) Govt. will buy paddy from the open market during next ‘P $ Q’ means ‘P is neither greather than nor smaller than Q’.
few months. In each question four statements showing relationship have
11. Statement: Many patients suffering from malaria were been given, which are followed by four conclusions I, II, III & IV.
administered anti-malarial drug for a week. Some of them Assuming that the given statements are true, find out which
did not respond to the traditional drug and their condition conclusion(s) is/are definitely true.
deteriorated after four days. 17. Statement : M c D, D K, K @ R, R F
Which of the following would weaken the findings Conclusions : I. F c K II. D F
mentioned in the above statement? III. M c R IV. D R
(a) Those patients who responded to the traditional drugs (a) None is true (b) Only I is true
and recovered were needed to be given additional (c) Only II is true (d) Only III is true
doses as they reported relapse of symptoms. (e) Only IV is true
(b) The mosquitoes carrying malaria are found to be 18. Statement : B % K, K $ T, T F, H c F
resistant to traditional malarial drugs. Conclusions: I. B $ T II. T c B
(c) Majority of the patients suffering from malaria III. H c K IV. F c B
responded well to the traditional malarial drugs and (a) Only either I or II is true
recovered from the illness.
(b) Only III is true
(d) Many drug companies have stopped manufacturing
(c) Only IV is true
traditional malarial drugs.
(d) Only III and IV are true
(e) None of these
(e) Only either I or II and III and IV are true
Directions (Qs.12-16): Study the following information carefully 19. Statement : W B, B @F , F c R, R $ M
and answer the questions given below: Conclusions : I. W F II. M B
Eight persons, A, B, C, D, E, F, G and H, are sitting around a III. R B IV. M W
rectangular table in such a way that two persons sit on each of (a) Only I and IV are true
the four sides of the table facing the centre. Persons sitting on (b) Only II and III are true
opposite sides are exactly opposite to each other. (c) Only I and III are true
D faces North and sits exactly opposite H. E is on the (d) Only II and IV are true
immediate left of H. A and G sit on the same side. G is exactly (e) None of these
opposite B, who is on the immediate right of C. A is next to the left of D.
www.newspaperkorner.wordpress.com
Practice Set - 6 91
www.newspaperkorner.wordpress.com Directions (Qs.25): Study the following arrangement and answer
20. Statement : E @ K, K $ T, T c N, B % N
Conclusions : I. T % E II. K c N the questions given below:
III. B T IV. B E M 5 % PU2A$ 4 3 Z E K 1 9 Q RI @ D 7 F d8WN 6 #V c
(a) Only I, II and III are true J Y
(b) Only II, III and IV are true 25. Which of the following is the fourth to the left of the sixth
(c) Only I, III and IV are true to the left of W in the above arrangement?
(d) All are true (a) U (b) D
(e) None of these (c) E (d) c
21. Statements : Z $ B, B % M, M c F, F @ R (e) None of these
Conclusions : I. Z M II. F B
Directions (Qs. 26-30) : Study the following information carefully
III. R M IV. M @ Z
to answer the questions given below.
(a) Only I, II are true
(b) Only I, III and IV are true In a toy exhibition, a machine processes a given input by the
(c) Only III and IV are true following rule. Participants are shown one by one till it reaches its
(d) Only I or IV and II are true last step. Following is an illustration of the working of this
(e) None of these machine.
Directions (Qs.22-24): In each of the questions below is given Input : sui me ato fe zen u no
four statements followed by three conclusions numbered I, II Step I : fe sui me no ato zen u
and III. You have to take the given statements to be true even if Step II : no fe sui u me ato zen
they seem to be at variance with commonly known facts. Read all Step III : u no fe zen sui me ato
the conclusions and then decide which of the given conclusions Step IV : zen u no ato fe sui me
logically follows from the given statements disregarding
Step V : ato zen u me no fe sui and so on.
commonly known facts.
Now attempt the questions given below.
22. Statements : All stones are poles.
26. Which of the following steps would read as ‘not you only say
All poles are desks. wise yet are’ for the input ‘say not you are only wise yet’?
Some desks are nets.
(a) III (b) V
All nets are days.
(c) VI (d) VII
Conclusions : I. Some nets are stones.
(e) None of these
II. Some desks are stones.
27. If the Step V of an input is ‘so cd rom lay is nor it’, which
III. Some days are desks.
of the following would be its Step II?
(a) Only I and II follow
(b) Only I and III follow (a) is nor it rom lay so cd (b) nor it lay is so cd rom
(c) Only II and III follow (c) lay so cd it rom is nor (d) Data inadequate
(d) All I, II and III follow (e) None of these
(e) None of these 28. If the Step III of an input is ‘lo men chi from yet as know’,
23. Statements : Some months are weeks. which of the following would be its input?
Some weeks are years. (a) Data inadequate
All years are buses. (b) from lo men know chi yet as
All buses are trains. (c) men chi yet lo as know from
Conclusions : I. Some trains are weeks. (d) chi as know men know from lo
II. Some buses are weeks. (e) None of these
III. Some trains are months. 29. Which of the following correctly describes the ‘machine
(a) Only I and II follow logic’ in generating various steps based on the given input?
(b) Only I and III follow (a) Each step is generated on random basis.
(c) Only II and III follow (b) Words/letters are finally arranged in dictionary order.
(d) All I, II and III follow (c) The seventh letter interchanges with the fourth every time.
(e) None of these (d) Data inadequate
24. Statements : Some stations are rails (e) None of these
All rails are rivers. 30. What will be the step IV for the following input?
All rivers are papers. Input : may sen to cry if not hell
Some papers are cards. (a) cry may sen to if not hell
Conclusions : I. Some cards are stations. (b) if not hell to cry may sen
II. Some rivers are stations. (c) sen to if may not hell cry
III. Some cards are rivers. (d) not hell cry if may sen to
(a) None follows (b) Only I follows
(e) None of these
(c) Only II follows (d) Only III follows
(e) Only II and III follow
www.newspaperkorner.wordpress.com
92 Practice Set - 6
www.newspaperkorner.wordpress.com
Directions (Qs. 31-35) : In each of the questions given below which one of the five answer figures on the right should come after the
problem figures on the left, if the sequence were continued.
Problem Figures Answer Figures
31. Z D « O Z Z Z Z
D O D U OO U D D D D Z « D D D O D «O « D O U D
Z U « D « D« D Z O Z U Z U « O U DU « O D U D U « D
(a) (b) (c) (d) (e)
32. O Z D D D O = D D D V D D V D V V = V =
D D Z = O D Z = O V D = O V = 3 D D = O D = O D 3 D D 5 D
= «« V ­ ­ Z 3 5 5 O B C 5 3 5 O 3
(a) (b) (c) (d) (e)
® ­
33. ­ ® ® ® ® ® ® ®
D U O«D U D « O Z D U Z « U D O« O=U « O U =« S U =« S U = « S U =« S U =«

(a) (b) (c) (d) (e)

34.

(a) (b) (c) (d) (e)


= =
35.
= = « = = «= = « «= = =
« « «
(a) (b) (c) (d) (e)

36. A repair for a known software bug, usually available at no 42. A (n) ________ camera is a peripheral device used to capture
change on the Internet, is called a (n) still images in a digital format that can be easily transferred
(a) version (b) patch into a computer and manipulated using graphics software.
(c) tutorial (d) FAQ (a) digital (b) analog
(e) None of these
(c) classic (d) film
37. In programming, you use the following keys
(a) Arrow keys (e) None of these
(b) Function keys 43. What is the process of copying software programs from
(c) Alpha keys secondary storage media to the hard disk called?
(d) Page up and Page Down keys (a) configuration (b) download
(e) None of these (c) storage (d) upload
38. A Website address is a unique name that identifies a specific (e) installation
_______ on the web.
44. Which is the best definition of a software package?
(a) Web browser (b) Web site
(c) PDA (d) link (a) An add-on for your computer such as additional
(e) None of these memory
39. If you are going to a site you use often, instead of having to (b) A set of computer programs used for a certain function
type in the address every time, you should such as word processing
(a) save it as a file (b) make a copy of it (c) A protection you can buy for a computer
(c) bookmark it (d) delete is (d) The box, manual and licence agreement that accompany
(e) None of these commercial software
40. Programs designed specifically to address general purpose
(e) None of these
applications and special-purpose applications are called
(a) operating system 45. What characteristic of read-only memory (ROM) makes it
(b) system software useful?
(c) application software (a) ROM information can be easily updated.
(d) management information systems (b) ROM provides very large amounts of inexpensive data
(e) None of these storage.
41. An ad hoc query is a (c) Data in ROM is nonvolatile, that is , it remains there
(a) pre-planned question even without electrical power.
(b) pre-scheduled question
(d) ROM chips are easily swapped between different
(c) spur-of-the-moment question
brands of computer.
(d) question that will not return any results
(e) None of these (e) None of these
www.newspaperkorner.wordpress.com
Practice Set - 6 93
www.newspaperkorner.wordpress.com
DATA ANALYIS & INTERPRETATION

Directions (Qs. 46-50) : These questions are based on the table and information given below.
The amount of money invested (in rupees crore ) in the core infrastructure areas of two districts, Chittoor and Khammam, in Andhra
Pradesh, is as follows :
Chittoor District Khammam District
Core Area 2014 2015 Core Area 2014 2015
Electricity 815.2 1054.2 Electricity 2065.8 2365.1
Chemical 389.5 476.7 Chemical 745.3 986.4
Thermal 632.4 565.9 Thermal 1232.7 1026.3
Solar 468.1 589.6 Solar 1363.5 1792.1
Nuclear 617.9 803.1 Nuclear 1674.3 2182.1
Total 2923.1 3489.5 Total 7081.6 8352

46. By what percent was the total investment in the two districts 52. How many children play badminton as well as cricket?
more in 2015 as compared to that in 2014? (a) 9 (b) 10
(a) 14% (b) 21% (c) 4 (d) 6
(c) 24% (d) 18% (e) None of these
(e) None of these 53. Total how many children are there in the class?
47. Approximately how many times the total investment in (a) 33 (b) 31
Chittoor was the total investment in Khammam? (c) 36 (d) 35
(a) 2.8 (b) 2.0 (e) None of these
(c) 2.4 (d) 1.7 54. In all how many children play badminton?
(e) None of these (a) 14 (b) 17
48. The investment in Electricity and Thermal Energy in 2014 in (c) 12 (d) 13
these two districts formed what percent of the total (e) None of these
investment made in that year ? 55. How many children play badminton ?
(a) 41% (b) 47%
(a) 7 (b) 4
(c) 52% (d) 55%
(c) 6 (d) 15
(e) None of these
(e) None of these
49. In Khammam district, the investment in which area in 2014
56. 4 boys and 3 girls are to be seated in a row in such a way
showed the highest percent increase over the investment
that no two boys sit adjacent to each other. In how many
made in that area in 2014?
different ways can it be done?
(a) Electricity (b) Chemical
(a) 5040 (b) 30
(c) Solar (d) Nuclear
(c) 144 (d) 72
(e) None of these
(e) None of these
50. If the total investment in Khammam shows the same rate of
57. Mr. ‘X’ invested certain amounts in two different schemes
increase in 2016, as it had shown from 2014 to 2015, what
‘A’ & ‘B’. Scheme ‘A’ offers simple interest at 12 p.c.a. and
approximately would be the total investment in Khammam
Scheme ‘B’ offers compound interest at 10 p.c.p.a. Interest
in 2016 (in ` crore?)
accrued on the amount invested in Scheme A in 2 years was
(a) 9, 850 (b) 10, 020 ` 3600 and the total amount invested was ` 35.000. What
(c) 9, 170 (d) 8, 540 was interest accrued on the amount invested in Scheme ‘B’?
(e) None of these (a) ` 4,800 (b) ` 4,200
Directions (Qs. 51-55): Carefully and answer the questions that (c) ` 4,000 (d) Cannot be determined
follow. Read the following information: (e) None of these
Children in a class play only one or two or all the three games 58. 12 men take 36 days to do a work while 12 women complete
badminton, football and cricket. 5 children play only cricket, 8 3
th of the same work in 36 days. In how many days 10 men
children play only football and 7 children play only badminton. 3 4
children play only two games badminton and football, 4 children and 8 women together will complete the same work?
play only two games cricket and football and another 4 children (a) 6 (b) 27
play only two games badminton and cricket. 2 children play all (c) 12 (d) Data inadequate
the three games. (e) None of these
51. In all how many children play football? 59. A grocer purchased 2 kg.of rice at the rate of ` 15 per kg.
(a) 8 (b) 17 and 3 kg. of rice at the rate of ` 13 per kg. At what price per
(c) 15 (d) 14 1
kg. should he sell the mixture to earn 33 % profit on the
(e) None of these 3
cost price?
www.newspaperkorner.wordpress.com
94 Practice Set - 6
www.newspaperkorner.wordpress.com
(a) ` 28.00 (b) ` 20.00 (c) Only A or C and B together are necessary.
(c) ` 18.40 (d) ` 17.40 (d) Even A, B and C all together are not sufficient to answer
(e) None of these the question.
60. A boat takes 6 hours to travel from place M to N downstream (e) All three A, B and C together are necessary.
and back from N to M upstream. If the speed of the boat in 65. How much marks did Arun secure in English?
still water is 4 km./hr., what is the distance between the two A. The average marks obtained by Arun in four subjects
places? included English is 60.
(a) 8 kms. (b) 12 kms. B. The total marks obtained by him in English and
(c) 6 kms. (d) Data inadeqate Mathematics together is 170.
(e) None of these C. The total marks obtained by him in Mathematics and
Science together is 180.
Directions (Qs. 61-65): Each of the questions below consists of
a question and three statements denoted A, B and C are given (a) All three A, B and C together are necessary.
below it. You have to study the questions and all the three (b) Only A and B together are necessary
statements are decide whether the question can be answered (c) Only B and C together are necessary.
with any one or two of the statements or all the statements are (d) Only A and C together are necessary.
required to answer the question. (e) None of these
61. What is R’s share of profit in a joint venture? Directions (Qs. 66-70): Refer to the pie-chart given below and
A. Q started business investing ` 80,000/- answer the questions that follow :
B. R joined him after 3 months. Operating Profit 160 lakh.
C. P joined after 4 months with a capital of ` 1,20,000 and Tax
got ` 6,000 as his share of profit. Interest 12%
(a) Only A and C are required 40%
(b) Only B and C are required Retained
Profit
(c) All A, B and C together are required
20%
(d) Even with all A, B and C the answer cannot be arrived
(e) None of these
62. What is the area of a right angled triangle? Dividend
Depreciation 8%
A. The perimeter of the triangle is 30 cm.
20%
B. The ratio between the base and the height of the triangle
is 5 : 12. 2014-15
C. The area of the triangle is equal to the area of a rectangle Operating Profit 130 lakh.
of length 10 cms. Tax
(a) Only B and C together are required Interest 9%
(b) Only A and B together are required 30%
(c) Only either A or B and C together are required. Retained
Profit
(d) Only A and C together are required 25%
(e) None of these
63. What will be sum of two numbers?
A. Among the two numbers, the bigger number is greater Dividend
Depreciation 8%
than the smaller number by 6. 20%
B. 40% of the smaller number is equal to 30% of the bigger 2013-14
number. 66. The operating profit in 2014-15 increased over that in 2013-
C. The 1 ratio between half of the bigger number and 14 by
rd of the smaller number is 2 : 1.
3 (a) 23% (b) 22%
(a) Only B and C together are necessary (c) 25% (d) 24%
(b) Only A and B together are necessary (e) None of these
(c) Out of A, B and C any two together are necessary 67. The Interest burden in 2014-15 was higher than that in
(d) All three A, B and C together are necessary 2013-14 by
(e) None of these (a) 50% (b) 25 lakh
64. How much profit did Mahesh earn on the cost price of an (c) 90% (d) 41 lakh
article by selling it? (e) None of these
A. He got 15% discount on the marked price at the time of 68. If, on an average, 20% rate of interest was charged on
purchase.
borrowed funds, then the total borrowed funds used by
B. He sold it for ` 3060.
this company in the given two years amounted to ....
C. He earned 2% profit on the marked price. (a) 221 lakh (b) 195 lakh
(a) Only A and B both together are necessary. (c) 368 lakh (d) 515 lakh
(b) Only B and C both together are necessary. (e) None of these
www.newspaperkorner.wordpress.com
Practice Set - 6 95
www.newspaperkorner.wordpress.com
69. The retained profit in 2014-15, as compared to that in Directions (Qs. 76-80): Study the following information carefully
2013-14 was to answer these questions.
(a) higher by 2.5% (b) higher by 1.5% Percentage of employees in various departments of an
(c) lower by 2.5% (d) lower by 1.5% organization and these male-female ratio
(e) None of these Total No. of Employees = 2500
70. The equity base of these companies remained unchanged.

127%
8%
Then the total dividend earning (in lakh rupees) by the

%
11
nts
share holders in 2014-15 is

ic s
ou

t
gis
A cc
(a) 10.4 lakh (b) 9 lakh

Lo
HR
(c) 12.8 lakh (d) 15.6 lakh 16% Operations
18%
(e) None of these
Marketing IT
Directions (Qs. 71-75): Study the graph carefully to answer the 14%
15%
questions that follow:
Percent profit made by two companies over the years
Ratio– Male : Female
Income - Expenditure Department Male : Female
Percent Profit = ´ 100
Expenditure Administration 7:5
Company A Company B Accounts 2:3
HR 5:3
60
Marketing 7:8
50
IT 3:4
Per cent Profit

40
Operations 5:4
30 Logistics 6:5
20 Printing 2:1
10 76. What is the ratio of male employees in Administration to
0
those in Printing Department?
2010 2011 2012 2013 2014 2015 (a) 7 : 4 (b) 4 : 7
Years (c) 3 : 4 (d) 7 : 3
71. If in the year 2010 the expenditure incurred by Company A (e) None of these
and B were the same, what was the ratio of the income of 77. What is the difference between the total number of
Company A to that of Company B in that year? employees in IT and that in Operations Department?
(a) 27 : 28 (b) 14 : 23 (a) 75 (b) 150
(c) 13 : 19 (d) Cannot be determined (c) 100 (d) 50
(e) None of these (e) None of these
72. If the amount of profit earned by Company A in the year 78. What is the ratio of the total number of males in HR and
2013 was ` 1.5 lakhs, what was it expenditure it that year? Marketing to the total number of females in these two
(a) ` 1.96 lakhs (b) ` 2.64 lakhs departments?
(c) ` 1.27 lakhs (d) ` 3.75 lakhs (a) 13 : 15 (b) 15 : 13
(e) None of these (c) 13 : 17 (d) 17 : 14
73. What is the average percent profit earned by Company B (e) None of these
over all the years together? 79. How many female employees are there in the HR
(a) 19 (b) 24 Departments?
(c) 12 (d) 37 (a) 250 (b) 120
(e) None of these (c) 125 (d) 150
74. If in the year 2014, the incomes of both the companies A (e) None of these
and B were the same, what was the ratio of the expenditure 80. What is the difference between the numbers of male and
of Company A to the expenditure of Company B in that female employees in Logistics Department?
year? (a) 50 (b) 25
(a) 21 : 25 (b) 7 : 9 (c) 75 (d) 100
(c) 13 : 15 (d) Cannot be determined (e) None of these
(e) None of these
75. What is the ratio of the amount of profit earned by Company ENGLISH LANGUAGE
A to that by company B in the year 2015?
(a) 2 : 3 (b) 4 : 7 Directions (Q. 81-90) : In the following questions a paragraph
(c) 11 : 15 (d) Cannot be determined is given. Read the paragraph carefully and answer the questions
(e) None of these which follow each of these paragraph.
www.newspaperkorner.wordpress.com
96 Practice Set - 6
www.newspaperkorner.wordpress.com
The greatest consequences of the atmosphere crisis may 84. Which of the following can be inferred from the given
be global warming and ozone depletion. The Earth appears to be paragraph?
warming due to greenhouse effect. Scientists estimate that average (a) The author has made strong efforts to wipe out fashion
temperature could climb about 2 degrees Celsius in 20 years. from the minds of youth.
Global warming is the predicted result of the greenhouse effect, (b) Steps need to be taken in order to control the growing
created by the greenhouse gases, such as carbon dioxide and fad of fashion amongst the youth.
methane, in the atmosphere. This change in the global climate (c) The author is upset with the shift” of fashion from the
would have disastrous results, including drought, coastal flooding traditional ethnic wear to western outfits.
and increased species extinction. Also, the scientists have (d) Fashion world is responsible for lack of creativity
discovered a hole in the ozone layer. The ozone layer is the only among the youth.
protection for life on Earth against deadly ultraviolet radiation (e) None of these
from the Sun. Once the ozone layer is completely destroyed, all The government cannot afford to compartmentalise
life on Earth will cease to exist, killed by the deadly radiation. The education. It has to be emphasised that any country which does
planet will become a barren rock devoid of all life. And when this not have a good university education will never be listed as an
happens the process of industrialisation would have to take the independent country and will never be able to progress. Only
blame which has resulted in increased use of fossil fuels such as countries prepared to tolerate a second rate and subjugate status
coal, petroleum and diesel. in the world would neglect higher education. If India had any
81. Which of the following can be inferred from the given position in the comity of nations in the past, it was only because
passage? it had a better higher education than many of its Asian
(A) The use of all chemical products should be immediately counterparts. This clear emphasis on higher education, however,
does not imply that it should be supported and developed at the
stopped.
cost of primary and secondary education.
(B) Scientists will have a hard time to estimate the rate of 85. Which of the following best explains the main idea of the
change of temperature in the years to come due to paragraph?
enormous changes in the environment. (a) The government has made a wrong move in
(C) The environmental disaster in the future will happen compartmentalising education.
as a result of increased usage of fossil fuels. (b) To highlight the superiority of India in higher education
(a) Only A (b) Only C as compared to the rest of the world
(c) Both A and C (d) Both B and C (c) To emphasise more on primary education rather than
(e) None of these higher education
82. Which of the following best explains the usage of the phrase (d) Higher education is important for the growth of a country.
‘brewing a disastrous stewl’ (e) None of these
(a) The continuous rise in Earth’s temperature has proved 86. Which of the following is the most likely title of the bigger
to be very dangerous. passage (having the same main idea as that of the given
(b) Lack of efforts to curb the greenhouse effects would paragraph) in which this paragraph might have appeared?
degrade the environment in the near future. (a) Education—a fundamental right of every child
(c) The harmful ultraviolet radiations are slowly (b) Indian education—an unfortunate neglect of primary
destroying the life on Earth. education
(c) Lack of higher education opportunities in India
(d) The fraternity of scientists has been opposing the
process of industrialisation. (d) A comparison of higher education in India with that of
the west
(e) The increased discharge of chemicals in the air is
resulting in an atmosphere crisis. (e) Importance of higher education in the growth of a country
Although an eco-friendly process and one of the main ways
Fashion has become one of the largest fads among the
to save the environment, recycling is not gaining pace in the
youth. The amount of time wastage and expenditure on fashion is
very large. What bothers, however, is the fact that fashion is here industry. The process of recycling undergoes many processes.
to stay despite countless arguments against it. What is required, First is collecting and sorting of garbage. This requires a lot of
therefore, is that strong efforts should be made in order to displace manpower and tools which are expensive. Another disadvantage
the excessive craze of fashion from the minds of today’s youth. of recycling which makes it unviable is that for the manufacturers,
economically, the recycled material is not highly demanded since
83. Which of the following statements finds the least support
its quality is not as good as the original material. So if the marketing
by the argument made by the author in the given paragraph?
of recycled goods is not worthwhile, then the whole process of
(a) Youngsters should be motivated to do constructive recycling cannot be economically efficient. Recycled products
business rather than wasting time on fashion. face other disadvantages as well, such as short shelf life, difficulty
(b) The world of fashion being glamorous and glittery in deinking the dyed products and less durability.
attracts people towards itself. 87. The author has listed the disadvantages of recycling to
(c) Following the latest fashion increases the self-efficacy (a) explain the reason why recycling is not a popular
of people, thus increasing their overall mental abilities.
option besides having various positive effects on the
(d) Many universities have implemented a dress code to
environment
put a check on the increasing fad amongst the youth
which was affecting their grades. (b) give arguments to support the stopping of the process
(e) None of these of recycling completely
www.newspaperkorner.wordpress.com
Practice Set - 6 97
www.newspaperkorner.wordpress.com
(c) predict the effects of stoppage of recycling on the before money was (92) . People who had specific items or services
environment would (93) these with others for the things they needed. Good
(d) show that there are better options than recycling negotiation was the (94) to making good trades. While the barter
available for utilization system (95) based on basic needs, today the barter system
continues to thrive. The barter system transcends the monetary
(e) None of these
system. The barter system is making a (96) today. What makes
88. Why is the marketing of recycled products not worthwhile, the barter system even better today than ever before is that it can
according to the author ? now be done globally. In the past, bartering was simply done
(A) The original products are being sold at a much lower with those that were located nearby. Today, the barter system
price as compared to the recycled products. can be used in a much more (97) way than ever before yet it
(B) The overall effect of recycling on the environment is carries with it the same basic motivation - the need for something
detrimental rather than beneficial. that you don’t have and the excess of something that someone
else wants.
(C) It does not give appropriate economic returns since
The barter system is enjoying (98) interest today. Bartering allows
recycled products lack demand in the market.
you to get the things you need without having to (99) additional
(a) Only A (b) Both B and C money. Instead, you can use the things you no longer need or
(c) Only C (d) Both A and B want to get the things you do need. There are swap markets and
(e) None of these online auctions that (100) you to sell or trade your items or to
The human civilization stands on an edge of destiny. It purchase items that you want. Negotiation takes place just like it
lies today on the mercy of nuclear weapons, which have the did hundreds of years ago.
power to destroy us completely lest controlled wisely. In such a 91. (a) discovered (b) dates
crisis situation, it is important to develop sensitivity towards (c) began (d) started
the basic values of life. It includes an explicit realisation that (e) initiated
humans swim in an ocean where they must swim together and 92. (a) bought (b) imagined
sink together. At this juncture, a single rotten fish would damage (c) began (d) emerged
(e) invented
the ocean as a whole. Man cannot afford to tamper with his
93. (a) buy (b) sell
loyalty to peace and international understanding, which is the
(c) exchange (d) give
only way to deal with the situation. Anyone who does so, dooms
(e) return
the entire past, present and the future of the human civilisation.
94. (a) important (b) essential
89. What, according to the author, should be done to avoid the (c) result (d) key
destruction of life by nuclear weapons? (e) intention
(a) Ban the use of nuclear weapons throughout the world 95. (a) originated (b) stood
(b) Enforce stricter laws and boycotts against the nations (c) generated (d) created
using nuclear weapons (e) produced
(c) Generate more awareness regarding the harm of using 96. (a) issue (b) comeback
nuclear weapons (c) withdrawal (d) recall
(d) Work towards international harmony and (e) fading
understanding 97. (a) primitive (b) appreciated
(e) None of these (c) promoted (d) sophisticated
(e) better
90. Which of the following can be inferred from the given
98. (a) diminishing (b) revival
paragraph?
(c) perishing (d) declining
(A) Nuclear war, one day or the other, is poised to end the
(e) renewed
human civilisation.
99. (a) expend (b) exchanging
(B) It is not possible for the countries to exist in unison
regardless of what happens in other countries. (c) expand (d) consume
(C) Unless all the nuclear weapons on earth are destroyed, (e) cost
peace cannot be restored. 100. (a) insist (b) force
(a) Only A (b) Only B (c) allow (d) lure
(e) constraint
(c) Only B and C (d) Only A and C
(e) None of these Directions (Qs. 101-105) : Rearrange the following six sentences
Directions (Qs. 91-100) : In the foliowing passage there are A, B, C, D, E and F in the proper sequence to form a meaningful
blanks, each of which has been numbered. These numbers are paragraph; then answer the questions given below them.
printed below the passage and against each, five words/phrases A. A legal framework is thus now available for promoting
are suggested, one of which fits the blank appropriately. Find energy efficiency in all sectors of the economy.
out the appropriate word/phrase in each case. B. The increasing preference for commercial energy has led to
The barter system for getting goods and services (91) back a sharp increase in the demand for electricity and fossil
many centuries. In most cultures the barter system was used fuels.
www.newspaperkorner.wordpress.com
98 Practice Set - 6
www.newspaperkorner.wordpress.com
C. There is still a considerable potential for repairing such 112. Along with a sharp rise in, ________ a recession would
damage and reducing energy consumption by adopting eventually result in more men, women and children living in
energy-efficiency measures at various sectors of our ________.
country. (a) crime, apathy (b) fatalities, poor
D. This framework is nothing but the Energy Conservation (c) deaths, slums (d) unemployment, poverty
Act, 2001, the success of which greatly depends on the (e) migrations, streets
people who take the lead in supporting this programme. 113. The government has to provide financial aid to the ones
________ by severe floods in the city.
E. This use of fossil fuels has resulted in emission of a huge (a) desired, troubled (b) promised, havoc
quantity of carbon dioxide causing serious environmental (c) failed, affected (d) wanted, struck
damage. (e) decided, ill
F. These adopted measures will not only reduce the need to 114. An airplane with ________ passengers on board made an
create new capacity requiring high investment, but also unscheduled ________ as the airport to which it was
result in substantial environmental benefits. heading was covered with thick log.
101. Which of the following should be the FIRST sentence after (a) irritable, slip (b) faulty, stop
rearrangement? (c) variety, halt (d) tons, wait
(a) A (b) B (c) C (d) D (e) E (e) numerous, landing
102. Which of the following should be the FOURTH sentence 115. Deemed universities ________ huge fees, but have not
been successful in providing ________ education to our
after rearrangement?
students.
(a) E (b) F (c) A (d) D (e) B (a) collect, maintaining (b) pay, better
103. Which of the following should be the SECOND sentence (c) ask, good (d) charge, quality
after rearrangement? (e) demand, quantitative
(a) B (b) C (c) D (d) E (e) F
104. Which of the following should be the LAST (SIXTH) GENERAL/ECONOMY/BANKING AWARENESS
sentence after rearrangement? 116. Book entitled The Wrong Enemy: America in Afghanistan,
(a) A (b) B (c) C (d) D (e) E 2001-2014 has been written by
105. Which of the following should be the FIFTH sentence after (a) Carlotta Gall (b) Neville Maxwell
rearrangement? (c) Henderson Brooks (d) Lindsay Williams
(a) A (b) B (c) C (d) D (e) E (e) None of these
Directions (Qs. 106-110): In each of these sentences, parts of 117. Saraswati Samman is an annual award and is given in
the sentence are numbered (a), (b), (c), (d) and (e). One of these recognition of outstanding prose or poetry literary works
in Indian languages that are listed in Schedule VII of the
parts has some error in it. The error may be either of spelling or
constitution of India. The award is given by which
grammar or wrong word or unnecessary word etc. The letter of
institution?
the part that contains the error is the answer.
(a) Sahitya Akademi
106. My only concern (a)/is that (b) at this juncture (c)/ communal (b) National Integrity Cultural Academy
sentiments are rather (d)/heightened up-wardly. (e) (c) Kalakshetra
107. However, (a)/ this division (b)/ of power is not quiet (c)/ as (d) K. K. Birla Foundation
neat as it may (d)/ appear at first. (e) (e) None of these
108. He strongly felt that (a)/ that explanation (b)/ which was 118. When was Earth Hour Observed?
given (c)/ during the meeting (d)/ was not at all truth. (e) (a) March 22 (b) April 1
109. We decided to (a)/ dedicate this article on the women (b)/ (c) March 29 (d) March 31
who have been instrumental in (c)/ training generations of (e) None of these
(d)/ young girls to create a healthy (e)/ atmosphere. 119. Which of the following statements is correct?
110. When the opportunity came (a)/ it appeared (b)/ in a different (a) Interest payable on savings bank account is not
regulated by the RBI.
form (c)/ and from a different (d)/ direction then he had (e)/
(b) Interest payable on savings bank account is regulated
expected.
by the Ministry of Finance.
Directions (Qs.111-115) : Each question below has two blanks, (c) Interest payable on saving bank account is regulated
each blank indicating that something has been omitted. Choose by the State Government.
the set of words for each blank which best fits the meaning of the (d) Interest payable on savings bank account is regulated
sentence as a whole. by the Indian Banks’ Association.
111. Behaving in a ________ and serious way, even in a (e) None of the above statements is correct
________ situation, makes people respect you. 120. Which of the following best defines mortgage?
(a) calm, difficult (b) steady, angry (a) It is a movable or liquid asset that a borrower offers a
(c) flamboyant, tricky (d) cool, astounding lender to secure a loan.
(e) silly, sound (b) It is a security of immovable property for a deposit
www.newspaperkorner.wordpress.com
Practice Set - 6 99
www.newspaperkorner.wordpress.com
received by a bank. (d) Marketing for a social cause
(c) It is a debt instrument that is secured by the collateral (e) Society bye-laws
of specified movable property. 130. Sole aim of marketing is to ____.
(d) It is a debt instrument that is secured by the collateral (a) Increase sales
of specified immovable property. (b) Increase the number of workers
(e) None of these (c) Increase profits
121. Which of the following statements is/are NOT correct with (d) Increase production
respect to current accounts? (e) All of these
(a) Cheques are not issued against current accounts 131. Consumer Day is celebrated on ____.
(b) There is no restriction on the number of transactions (a) 15th March (b) 16th March
in the current accounts. (c) 20th April (d) 20th March
(c) Normally no interest is paid on current accounts. (e) None of these
(d) Both (a) and (b) 132 Financial inclusion means provision of ?
(e) Both (b) and (c) (a) Financial services namely payments, remittances,
122. Which organisation regulates Mutual Funds in India? savings, loans and insurance at affordable cost to
(a) SEBI persons not yet given the bank
(b) RBI (b) Ration at affordable cost to persons not yet given
(c) Ministry of Finance, Govt of India the same
(d) IRDA (c) House at affordable cost to persons not yet given
(e) None of these the same
123. In which of the following years did the government of India (d) Food at affordable cost to persons not yet given the same
devalue its currency twice? (e) Education at affordable cost to persons not yet
(a) 1949 (b) 1966 given the same Ans. A
(c) 1991 (d) 1949 and 1991 133. An ECS transaction gets bounced and you are unable to
(e) 1993 recover your money from your customer. Under which
124. What does the acronym LAF stand for? Act criminal action can be initiated?
(a) Liquidity Adjustment Fund (a) Indian Penal Code
(b) Liquidity Adjustment Facility (b) Negotiable Instrument Act
(c) Liquidity Adjustment Finance (c) Criminal Procedure Code
(d) Liquidity Adjustment Factor (d) Payment and Settlement Act
(e) None of these (e) Indian Contract Act
125. Which of the following market structures has a kinked 134 Which of the following terms is used in the field of
Demand Curve? finance and banking?
(a) Perfect competition (b) Monopoly (a) Metabolism (b) Genetic process
(c) Oligopoly (d) Monopolistic competition (c) Centrifugal force (d) Debt instrument
(e) None of these (e) Experimental error
126. Market information means 135. The term ‘moral suasion’ refers to
(a) knowledge of companies (a) The moral duty of a borrower to deal with only one bank
(b) cross- country information (b) The advice given by the Reserve Bank of India to
(c) knowledge of related markets banks/financial institutions in the matter of heir
(d) knowledge of current customers lending and other operations with the objective that
(e) None of these they might implement or follow it
127. Efficient marketing style requires (c) The banker's duty of secrecy as regards the affairs
(a) proper planning and accounts of his customers
(b) good communication skills (d) All of the above
(c) team work (e) None of these
(d) knowledge of the products 136. Which one of the following is not a money market
(e) All of these instrument?
128. Effective communication is not required in marketing if __. (a) Repo
(a) Demand exceeds supply (b) A negotiable Certificate of Deposit
(b) Supply exceeds demand (c) Commercial paper
(c) Buyer is illiterate (d) A Treasury Bill
(d) Seller is illiterate (e) Treasury Bond
(e) None of these 137. Land Development Banks provide long-term credit for
129. Social Marketing refers to _____. schemes of basic importance to agriculture as
(a) Share market prices (a) mechanization of agriculture i.e., loans for purposes
(b) Marketing to entire society of tractors, power tillers, threshers etc
(c) Internet Marketing (b) land reclamation, soil conservation, plantation of
www.newspaperkorner.wordpress.com
100 Practice Set - 6
www.newspaperkorner.wordpress.com
fruit orchards, dairy development schemes etc. (c) 2019 (d) 2020
involving heavy expenditure (e) None of these
(c) minor irrigation purposes like wells, dug-cum-bore 147. As per the Rail Budget 2016-2017, the capacity of e-
wells, tube wells, pump sets and irrigation tanks ticketing system will be enhanced from 2000 tickets/min
(d) None of these to
(e) All of the above (a) 7200 tickets/min (b) 5000 tickets/min
138. From which of the following financial organizations of the (c) 6400 tickets/min (d) 3200 tickets/min
world, India had taken soft loan for recapitalization of (e) None of these
banks? 148. How many stores will be opened under Prime Minister's
(a) Asian Development Bank Jan Aushadhi Yojana during 2016-17?
(b) International Monetary Fund (a) 1000 (b) 2000
(c) International Donors' Group (c) 3000 (d) 4000
(d) World Bank (e) None of these
(e) Bank of China 149. Name the proposed programme that will be started under
139. Through which of the following the funds to the projects National Health Mission through public-private partner-
aided by the World Bank and the International Monetary ship mode to provide dialysis services to renal patients in
Fund (IMF) are distributed? all district hospitals across the country.
(a) Regional Branches of RBI (a) National Services Programme for Renal Patients
(b) NABARD (b) National Dialysis Programme
(c) Integrated Rural Development Programme (c) National Dialysis Services Project
(d) Agriculture Refinance and Development Corporation (d) National Dialysis Services Programme
(e) Ministry of Finance (e) None of these
140. Which of the following is NOT a part of India's Money 150. Recently, India signed the Kamov helicopter deal, with
Market? which country?
(a) Bill Market (b) Call Money Market (a) Israel (b) Russia
(c) Ukraine (d) France
(c) Banks (d) Mutual Funds
(e) None of these
(e) Indian Gold Council
151. Who is the incumbent Managing Director of the Interna-
141. Union Budget which is presented every year in the
tional Monetary Fund (IMF)?
parliament is also known as
(a) Christine Lagarde
(a) Annual Budget (b) Common Budget
(b) Dominique Strauss-Kahn
(c) General Budget (d) Finance Report of the year
(c) Ban Ki-moon
(e) Statement of Allocation of funds
(d) Jim Yong Kim
142. Which of the following terms is NOT used in Banking/
(e) None of these
Finance related matters ? 152. 'Kakrapara Atomic Power Station' is located in which state
(a) Discount Rate (b) Letter of Credit of India?
(c) Cheque Clearing (d) General Ledger (a) Gujarat (b) Karnataka
(e) Law of Motion (c) Maharashtra (d) Rajasthan
143. Which of the following organizations/agencies works solely (e) None of these
to monitor and arrange flow of agriculture credit in India ?
153. Which one of the following tennis tournaments is the
(a) NABARD (b) SIDBI first to start the International Grand Slam in the tennis
(c) RBI (d) SEBI history?
(e) None of these (a) Australian open (b) French open
144. Foreign currencies are represented in various abbrevia- (c) Wimbledon open (d) US open
tions. What is meant by USD ? (e) None of these
(a) UAE Dinar 154. Which of the following is NOT decided by the Finance
(b) US Dollar Minister while presenting Union Budget every year?
(c) Unit of Currency of Denmark (a) Rate of Income Tax
(d) Ukrainian Soviet Dollar (b) Expenditure of Defence Services
(e) None of these (c) Relief packages to various sections of the society
145. What is the sum set aside for providing innovation (d) Repo & Reverse Repo rates
grants to employees, start-ups and small businesses to (e) Revenue Expected from various sources
support internal and external innovation, as per the Rail 155. The rate of MAT is changed almost in every budget.
Budget 2016-2017? What is the full form of MAT ?
(a) 20 crores (b) 60 crores (a) Maximum Alternate Tax
(c) 5.6 crores (d) 50 crores (b) Minimum Alternate Tax
(e) None of these (c) Minimum Affordable Tax
146. Suresh Prabhu in the Rail Budget 2016-2017 set a target to (d) Maximum Affordable Tax
eliminate all unmanned level crossings by which year? (e) None of the above
(a) 2017 (b) 2018
www.newspaperkorner.wordpress.com
Practice Set - 6 101
www.newspaperkorner.wordpress.com

Time : 30 min. Max. Marks : 50

1. Write a letter on one of the following topics (20 marks)


(a) You are a branch manger of a bank. Write a letter to head office recommending installation of ATMs in your office
premises and also in the extension counter in an Engineering college. Elaborate the progress of your branch will be
making on account of this move.
(b) You have noticed many stray animals on the road during the busy hours of the day. Write a letter to the editor of a
leading newspaper about the nuisance created by the stray animals.
(c) Write to plastic Leaders, manufacturers of plastic ware, Enterprise House, Nellore, canceling your order as the delivery
date has expired.
2. Write a paragraph on any one of the following in not more than 150 words (10 marks)
(a) Advertising is all glitter and little truth.
(b) If you were a Manager of a bank of a remote area of your country.
(c) Religion is not enemy of science rather its complement.
3. Write an essay on any one of the following in about 250 words. (20 marks)
(a) Secularism is driving force of Indian Nationalism
(b) The Civil Society - meaning and role.
(c) Who achieves at the creation of more states?

www.newspaperkorner.wordpress.com
102 Practice Set - 6
www.newspaperkorner.wordpress.com
Answer Key
1 (d) 16 (b) 31 (e) 46 (d) 61 (d) 76 (a) 91 (b) 106 (e) 121 (a ) 136 (e) 151 (a)
2 (d) 17 (a ) 32 (d) 47 (c) 62 (b) 77 (c) 92 (e) 107 (c) 122 (a ) 137 (e) 152 (a)
3 (b) 18 (e) 33 (d) 48 (b) 63 (b) 78 (d) 93 (c) 108 (e) 123 (c) 138 (d) 153 (c)
4 (d) 19 (b) 34 (b) 49 (b) 64 (e) 79 (d) 94 (d) 109 (b) 124 (b) 139 (d) 154 (c)
5 (d) 20 (d) 35 (a) 50 (a ) 65 (e) 80 (b) 95 (a) 110 (e) 125 (c) 140 (e) 155 (b)
6 (a ) 21 (c) 36 (b) 51 (b) 66 (a ) 81 (b) 96 (b) 111 (a ) 126 (c) 141 (c)
7 (c) 22 (c) 37 (b) 52 (d) 67 (b) 82 (e) 97 (d) 112 (d) 127 (e) 142 (e)
8 (a ) 23 (a ) 38 (d) 53 (a ) 68 (d) 83 (c) 98 (e) 113 (c) 128 (e) 143 (a)
9 (c) 24 (c) 39 (c) 54 (e) 69 (d) 84 (b) 99 (a) 114 (e) 129 (d) 144 (b)
10 (a ) 25 (e) 40 (c) 55 (c) 70 (c) 85 (d) 100 (c) 115 (d) 130 (a ) 145 (d)
11 (c) 26 (c) 41 (c) 56 (c) 71 (a ) 86 (e) 101 (b) 116 (a ) 131 (a ) 146 (d)
12 (e) 27 (a ) 42 (a) 57 (b) 72 (d) 87 (e) 102 (b) 117 (d) 132 (a ) 147 (a)
13 (b) 28 (e) 43 (c) 58 (b) 73 (e) 88 (c) 103 (d) 118 (c) 133 (d) 148 (c)
14 (d) 29 (e) 44 (b) 59 (c) 74 (c) 89 (d) 104 (d) 119 (a ) 134 (d) 149 (d)
15 (c) 30 (b) 45 (e) 60 (d) 75 (d) 90 (a) 105 (a) 120 (d) 135 (b) 150 (b)

Answers & Explanations


1. (d) We have been given [Information regarding H is absent. Hence, I alone is
Kiran > Manoj ...(i) not sufficient]
Dilip > Neelam ...(ii) From II:
Now, who is the youngest? We need information by M
which the above equations can be combined into a
single equation.
From I: If Kiran > Neelam then either V H
Manoj or Neelam will be the youngest. Hence, H is to the east of V
From II: If Dilip > Manoj then either Manoj or Neelam
be the youngest. Member Country Sport
Hence, neither I nor II is sufficient. K China Archery
2. (d) From I : M T USA Football
F France Volleyball
B(+) (+) L Australia Athletics
(4-6) :
From II: J Russia Tennis
R Korea Rifle Shooting
mother (–)
H Japan Boxing

10. (a) The very purpose of hiking the procurement price of a


F(–) D(+) B(+) crop is encouraging the farmers to cultivate it.
11. (c) Since majority of the patients have responded well to
Both the statements I and II are not independently
the drug one wonders how some of them have
sufficient because statement I does not say about F
witnessed deterioration in their condition.
and statement II does not say about M. E
H
From statement I and II together, F is the daughter of
M. But we don’t know the sex of M.
G B
3. (b) From I:
N A C
K V (12-16) :
W E
D F
S S www.newspaperkorner.wordpress.com
Practice Set - 6 103
www.newspaperkorner.wordpress.com
17. (a) M < D... (i), D > K ....(ii), ...(iii); R > F ....(iv) (26-30): Here the rule followed is: In each step the fourth word
K£R
These relationships can’t be of any help; none of the becomes first word and the last word becomes fourth word
quantities can be compared on their basis. and all other words shift one place rightwards except the
third, which shifts two place rightwards.
18. (e) B ³ K ...(i); K = T....(ii); T > F ....(iii); H < F... (iv)
In order to make things easier, let us represent the words
From (i) and (ii), B ³ K = T or B ³ T . Hence either I digitally from 1 to 7.
(B = T) or II (T < B) is true. Then we have:
From (ii), (iii) and (iv), K = T, T > F > H or H < K.
Hence III follows. From (i), (ii) and (iii), Input: 1 2 3 4 5 6 7
B ³ K =T > F or F < B. Hence IV follows. Step I: 4 1 2 7 3 5 6
19. (b) W > B ...(i); B £ F ...(ii); F < R ...(iii); R = M ...(iv) Step II: 7 4 1 6 2 3 5
From (ii), (iii) and (iv), B £ F < R = M ...(v) Step III: 6 7 4 5 1 2 3
Hence M > B and II follows. Also, R > B and III follows. Step IV: 5 6 7 3 4 1 2
From (i) and (v). We can’t be compared either with F or Step V: 3 5 6 2 7 4 1
with M. Hence I and IV do not follow.
Step VI: 2 3 5 1 6 7 4
20. (d) E £ K ...(i); K = T ...(ii); T < N ...(iii); B ³ N ...(iv)
Combining these, we get E £ K = T < N £ B . [Note: We have gone up to step VI because one of the
Hence T ³ E and I follows. questions (Q. 171) demands that.]
26. (c) Input: say not you are only wise yet
Again, K < N and II follows.
234567
B > T and hence III is true.
Arrangement: not you only say wise yet are
Again, B > E and hence IV follows.
Step VI : 2 3 5 1 6 7 4
21. (c) Z = B ...(i); B ³ M ...(ii); M < F...(iii); F £ R ......iv)
27. (a) Step V: s o cd rom lay is nor it
From (i) and (ii), Z = B ³ M or Z ³ M ...(v)
Hence I (Z > M) is not definitely true but IV ( M £ Z ) is: 3 5 6 2 7 4 1
From (ii) and (iii), B and F can’t be compared. Step II: 7 4 1 6 2 3 5
Hence II does not follow. is nor it rom lay s o cd
From (iii) and (iv), M < F £ R or R > M. Hence III 28. (e) Step III: lo men chi from yet as know
follows. 6 7 4 5 1 2 3
22. (c) All stones are poles + All poles are desks = A + A = A Input: 1 2 3 4 5 6 7
= All stones are desks ® conversion ® Some desks yet as know chi from lo men
are stones (I).
29. (e) The rule is given above.
Hence II follows. All stones are desks + Some desks
31. (e) In the first step the upper middle shifts to middle left
are nets = A + I = No conclusion. Hence I does not
® lower left ® upper middle while the remaining four
follow. Some desks are nets + All nets are days = I + A
shift one step CW in a cyclic order. In the next step the
= I = Some desks are days ® conversion ® Some
middle row elements shift from left to right in a cycle
days are desk (I).
order while the remaining five shift one step CW.
Hence III follows.
32. (d) In the first step the upper left and middle right
23. (a) Some weeks are years + All years are buses = I + A = I interchange places ; so do the upper right and middle
= Some weeks are buses ® conversion ® Some buses left. The remaining three elements shift one step CW.
are weeks (I). Hence II follows. Some weeks are buses In the next step the lower-left and lower-right elements
+ All buses are trains = I + A = I = Some weeks are are replaced by new ones while the remaining five shift
trains ® conversion ® Some trains are weeks (I). one step ACW in a cyclic order.
Hence, I follows. Some months are weeks + Some weeks 33. (d) In each step the upper middle rotates by 45° CW and
are trains = I + I = No conclusion. Hence III does not the lower middle rotates by 45° ACW. Among the
follow. middle-row elements two pairs interchange places
24. (c) Some stations are rails + All rails are rivers = I + A = I = while the fifth element is replaced by new one
Some stations are rivers ® conversion ® Some rivers successively from the right to left, beginning from the
are stations (I). right.
Hence II follows. All rivers are papers + Some papers 34. (b) From fig 1 to 2: The upper left and the middle left
are cards = A + I = No conclusion. interchange positions while the remaining five elements
Hence III does not follow. Neither does I. shift one step CW in a cyclic order.
25. (e) 4th to the left of 6th to the left of W = (4 + 6) = 10th to 35. (a) Follow the shifting of the elements in alternate steps.
the left of W = 9 46. (d) Total investment in 2014 = 2923.1 + 7081.6 = 10004.7
www.newspaperkorner.wordpress.com
104 Practice Set - 6
www.newspaperkorner.wordpress.com
Total investment in 2015 = 3489.5 + 8352.0 = 11,841.5 number of ways in which no two boys sit adjacent to
each other.
11841.5 - 10,004.7
\ % increase = ´ 100 = 18.36 % = 3! × 4! = 6 × 24 = 144
10,004.7
57. (b) Let the amount invested in scheme A be ` x.
47. (c) Total investment in Chittor = 6412.6
3600 ´ 100
Total investment in Khammam = 15433.6 \ x ´ 12 ´ 2 = 3600 Þ x = = ` 15000
100 24
Total investment in Khammam 15433.6 Total investment = ` 35,000
\ = = 2.40
Total investment in Chittor 6412.6 \ Amount invested in scheme B. = ` (35000 – 15000)
48. (b) Investment in electricity & thermal energy in 2014 in = ` 20000
two districts = 815.2 + 632.4 + 2065.8 + 1232.7
= 4746.1 éæ R ö
T ù é 10 2 ù
\ C.I = P êç1 + - 1ú = 2000 ê1 + - 1ú
% in terms of total investment è 100 ÷ø ëê 100 ûú
ëê úû
4746.1 = 20000 (1.21 – 1) = 2000 × 0.21 = ` 4200
= ´ 100 = 47.43%
10,004.7 58. (b) In 36 days 12 men can do 1 complete work.
49. (b) % increase in Khammam district in the area of 3
In 36 days 12 women can do th of the work.
Electricity = 2365.1 - 2065.8 ´ 100 = 14.5% 4
2065.8 Since time and the no. of persons is the same in both
cases.
986.4 - 745.3
Chemical = ´ 100 = 32.34% 3
745.3 1 woman’s daily work = th of 1 man’s daily work
4
1792.1 - 1363.5
Solar = ´ 100 = 31.43% 3
1363.5 8 women’s daily work = ´ 8 = 6 men’s daily work
4
2182.1 - 1674.3 (10 men + 8 womens daily work) = (10 men + 6 men)
Nuclear = ´ 100 = 30.32
1674.3 = 16 men’s daily work
Hence highest increase is in the area of chemical 12 men can do the work in 36 days
50. (a) % increase in investment from 2014 to 2015
12
\ 16 men can do the work is 36 ´ = 27 days.
8352 - 7081.6 16
= = 17.93%
7081.6 59. (c) Mixture : 2 kg of rice at `15/kg + 3 kg of rice at `.13/kg.
\ Total investment in 2016 = 1.1793 × 8352 Total weight = 2 + 3 = 5 kg
= ` 9850 crores Total cost price = (2 × 15) + (3 × 13) = 30 + 39 = ` 69
(51-55) : Venn-Diagram for questions :
69
Cost price per kg of the mixture = = ` 13.80
5
Badminton Football
8 1
7 Selling price to get 33 % profit
3 3
2
4 4 1
100 + 33 400
= 3 ´ ` 13.80 = × ` 13.80
5
100 3 ´ 100

Cricket 4
= ´ ` 13.80 = ` 18.40
3
51. (b) Number of football players = 3 + 2 + 4 + 8 = 17
52. (d) Number of badminton as well as cricket players 60. (d) Total Time = 6 hours
=4+ 2=6 Speed of the boat in still water = 4 km/hr.
53. (a) Number of children = 7 + 3 + 2 + 4 + 8 + 4 + 5 = 33 Let the distance between M and N be D.
54. (e) Number of badminton players = 7 + 3 + 2 + 4 = 16. and the speed of the stream be x.
55. (c) Number of football as well as cricket players = 2 + 4 = 6 é 4- x + 4- x ù
é 1 1 ù
56. (c) 3 girls can be seated in a row in 3! ways. Now, in the 4 Dê + ú = 6 or D ê ú =6
gaps 4 boys can be seated in 4! ways. Hence, the
ë4 + x 4 - x û ëê ( x + x )( 4 - x ) ûú
www.newspaperkorner.wordpress.com
Practice Set - 6 105
www.newspaperkorner.wordpress.com 65. (e) A. Total marks in 4 subjects including English
é 8 ù
Dê = 6 or 8D = 6 = 4 × 60 = 240
2 2ú
ë4 - x û 16 - x 2 B. Total marks in English and Maths = 170
C. Total marks in Maths and Science = 180
D=
6
8
( 3
) (
16 - x 2 = 16 - x 2
4
) The question can’t be answered because nothing has
been said about the marks in the fourth subject.
Since the speed of the stream (x) is not given, the Also, there are four unknowns but only three equations
distance D cannot be determined. can be formed with given data.
61. (d) The question cannot be answered because R’s share 66. (a) Increase in operating profit
in investment is not given.
160 - 130 300
25 + 144 = 169 = 13 = ´ 100 = = 23%
62. (b) Hypotenuse = 52 + 122 = 130 13
Base : Height : Hypotenuse = 5 : 12 : 13 67. (b) Interest burden increase
Base + Height + Hypotenuse = 30 cm
= 0.4 ´160 - 0.3 ´130 = 64 - 39 = 25 lakh
5
\ Base = ´ 30 = 5 cm 0.3 ´ 130
5 + 12 + 13 68. (d) Borrowed funds in 2013-14 = = 195 lakhs
0. 2
12
Height = ´ 30 = 12 cm 0.4 ´ 160
5 + 12 + 13 Borrowed funds in 2014-15 = = 320 lakhs
0.2
1 1
Area = ´ base ´ height = ´ 5 ´12 = 30 cm2 Total = 320 + 195 = 515.
2 2
69. (d) Retained Profit in 2013-14= 0.25 ´ 130 = 32.5
63. (b) A.x – y = 6
Retained Profit in 2014-15= .2 × 160 = 32
B. 0.4y = 0.3x
0. 5 100
x 4 x y
= ; C. . = 2 :1 Reduction = ´ 100 = = 1.5%
32.5 65
y 3 2 3
70. (c) Dividend earning in 2014-15= 0.08 × 160= 12.8
x 3 2 x 4 71. (a) Let the expenditure of companies A and B in 2010 be `
´ = Þ =
y 2 1 y 3 100 each (Since we have to find the ratio of income, we
can assume such value of expenditure.
B and C give the same expression / information and
hence are equivalent. 135
IA = 100 ´ = 135
04 100
4
x= y Þ x – y= 6
3 Similarly, IB04 = 140
4 y IA
y-y = 6 Þ = 6 04 135
3 3 = = 27 : 28
IB 140
04
4
y = 18 and x = ´ 18 = 24 æ 100 ö
3 E A 2013 = 1.5 ç = 3.75 lakh
72. (d) è 40 ÷ø
64. (e) Let the marked price be Rs. x
A. cost price = (1 – 0.15)x = ` 0.85x 73. (e) Required average per cent profit earned by Company
B. S.P. = `3060
C. Profit = 2% of x = 0.02x 40 + 45 + 40 + 35 + 30 + 45 235 1
B= = = 39 %
0.02x 6 6 6
Profit earned on the cost price = ´ 100 » 2.35%
0.85x 74. (c) Let the income of each company be ` x in the year
0.02x = 3060 – 0.85x 2014.
100 2
0.87x = 3060 or x = 3060 Then E A = x ´ = x
100 + 50 3
0.87

3060 100 10
EB = x ´ = x
Actual profit = 0.02x = 0.02 ´ = ` 70.34 100 + 30 13
0.87
www.newspaperkorner.wordpress.com
106 Practice Set - 6
www.newspaperkorner.wordpress.com 85. (d) All others are either distorted or partial reading of the
2 paragraph.
x
EA
= 3 = 13 :15 86. (e) This is what we have arrived at in the question above.
E B 10
x 87. (e) Don’t go for 1 because it could be correct only if we
13
substitute “in spite of” for “besides”.
75. (d) Since no amount (of income, exp or profit) is given in 88. (c) Read the fifth and sixth sentences of the paragraph.
the question, we can’t find the ratio of profits
89. (d) Read the third and fourth sentences together.
7 2 90. (a) (A) follows from the first two sentences of the
76. (a) Ratio = 12 ´ : 6´ = 7 : 4
12 3 paragraph. (B) is absured. (C) is not hinted at.
77. (c) 4% of 2500 = 100 (Qs. 101-105) : BECFAD
106. (e) Remove the word ‘upwardly’.
æ 5 7ö æ 3 8ö
78. (d) Ratio = ç 16 ´ + 15 ´ ÷ : ç16 ´ + 15 ´ ÷ = 17 : 14 107. (c) Replace ‘quiet’ with ‘quite’.
è 8 5 ø è 8 15 ø
108. (e) Replace ‘truth’ with ‘true’.
3 109. (b) Replace ‘on the women’ with ‘to the women’.
79. (d) ´ 16 ´ 25 = 150
8 110. (e) Replace ‘then’ with ‘than’.
11 é 6 5 ù 111 (a) calm, difficult
80. (b) 2500 ´ - = 25
100 êë11 11úû 112. (d) unemployment, poverty
113. (c) failed, affected
81. (b) (A) does not follow because it proposes an extreme
solution. (B) certainly is not one of the consequences 114. (e) numerous landing
of the atmosphere crisis. (C) can be inferred from the 115. (d) charge, quality
last sentence of the paragraph. 153. (c) Wimbledon Championship is the oldest and the most
82. (e) Read the second and third sentences of the paragraph prestigious tennis tournament in the world. The name
together.
of the tournament is derived from Wimbledon, a
83. (c) 3 goes in the opposite direction: it talks about the utility
suburb of London, where it has been held since 1877.
of fashion whereas the author does not talk of fashion
approvingly. 155. (b) MAT stands for Minimum Alternate Tax. It is
84. (b) Read the last sentence of the paragraph. charged on book profits of the company.

www.newspaperkorner.wordpress.com
www.newspaperkorner.wordpress.com

Practice Set - 7

INSTRUCTIONS
• This practice set consists two parts. One part is Objective test and other part is Descriptive test.
• Each question has five options, of which only one is correct. The candidates are advised to read all the
options thoroughly.
• There is negative marking equivalent to 1/4th of the mark allotted to the specific question for wrong
answer.

Time : 3 hrs. Max. Marks : 200

REASONING AND COMPUTER APTITUDE Directions (Qs. 6-10): Study the following information carefully
and answer the given questions based on it.
Directions (Qs. 1-5) : In the following questions, th symbols @,
©, %, $ and # are used with the followinbg meaning as illustrated Amit, Bunty, Reena, Veena and Dolly were selected for prizes
below : because of their excellent performance in annual examination
‘P % Q’ means ‘P is either smaller than or equal to Q’. conducted by school 'X'. On this occasion their parents were
‘P © Q’ means ‘P is greater than Q’. also invited by the school authority. Brajesh, Jayant, Manoj,
‘P # Q’ means ‘P is neither greater than nor smaller than Q’. Ritesh and Vinod were the fathers of the individuals but not
‘P $ Q’ means ‘P is smaller than Q’.
necessarily in the same order. Anju, Babita, Punam, Rita and Vinita
‘P @ Q’ means ‘P is either greater than or equal to Q’.
were the mothers of the individuals but not necessarily in the
Now in each of the following questions assuming the given
statements to be true, for conclusions I and II given below same order. Among the children only two are males. The first
them is/are definitely true? Give answer letter of the name of any parent was not the same as the first letter
(a) if only conclusion I is true. of the name of their child. The same was true of spouses also.
(b) if only conclusion II is true The first letter of the name of Dolly’s father was the same as that
(c) if either conclusion I or II is true of Reena’s mother. Similarly, Dolly’s mother’s name started with
(d) if neither conclusion I nor II is true
the first letter of Reena's father's name. Brajesh and Manoj were
(e) if both conclusion I and II are true
1. Statements : B # F, F $ H, H © K not the fathers of Veena and Amit respectively. The first letter of
Conclusions : I. H @ B Bunty’s mother was the same as the first letter of the name of
II. K $ B Veena’s father. Anju is not Dolly’s mother. Jayant is the husband
2. Statements : H @ T, T © N, N $ W of Punam. Reena’s performance was best among all; that is why
Conclusions : I. N $ H her mother Babita praised her.
II. W $ H
6. Who among the following is the father of Amit?
3. Statements : H $ F, F % M, M © J
Conclusions : I. J $ F (a) Can’t say (b) Jayant
II. M © H (c) Manoj (d) Vinod
4. Statements : M $ T, T % R, M © N (e) None of these
Conclusions : I. M $ R 7. Who among the following is the son of Rita?
II. N $ T (a) Can’t say (b) Amit
5. Statements : D $ T, T % B, B @ F (c) Bunty (d) Either Amit or Bunty
Conclusions : I. D # T
(e) None of these
II. D @ F
www.newspaperkorner.wordpress.com
108 Practice Set - 7
www.newspaperkorner.wordpress.com
8. Father’s name of Reena is (c) you then can men are when how
(a) Can't say (b) Vinod (d) how can then men are when you
(c) Brajesh (d) Either Vinod or Brajesh (e) None of these
(e) None of these 14. Input: stejpan mesic is the president of Croatia.
9. Which of the following couples is correctly matched? Which of the following will be step VIII for this input?
(a) Jayant, Rita (b) Manoj, Punam
(a) the mesic stejpan president is of Croatia
(c) Vinod, Babita (d) Ritesh Vinita
(b) the is of mesic Croatia stejpan president
(e) Brajesh, Anju
(c) stejpan mesic is president Croatia of the
10. Which of the following pairs is related in the same way as
(d) the stejpan mesic of is president Croatia
‘CAVE’ and ‘HILL’?
(a) OAR: BOAT (b) CAVITY: TOOTH (e) None of these
(c) ROOF: HOUSE (d) SHOE: SOCKS 15. If Step V of an input is
(e) None of these "will you hit centuries three again at",
what will be the middle three words of step VII?
Directions (Qs. 11-16): A word arrangement machine, when given
(a) will you hit (b) you hit centuries
a particular input, rearranges it following a particular rule.
(c) hit centuries three (d) centuries three again
The following is the illustration of the input and the steps of
(e) None of these
arrangement:
16. If step II of an input is
Input: and band land hand hind lack job
"has started new BSC batches for PO",
Step I: hind and band lack land hand job
what will be Step VI of the input?
Step II: hind band land job and lack hand
(a) new PO for started BSC batches has
Step III: hind and lack band hand land job
(b) PO new for started BSC batches has
Step IV: land band and job hand lack hind (c) PO new started for batches BSC has
Step V: hand land band lack and job hind (d) PO started batches has new for BSC
Step VI: hand band and hind land lack job and so on. (e) None of these
As per the rule followed in the above steps, find out the appropriate Directions (Qs. 17-18) : In each of these questions, a statement
step for the given input or vice versa in the following questions. is given, followed by two Conclusions. Mark your answer as
11. Input: do we he is it at all. (a) if only conclusion I follows;
Which of the following steps would be (b) if only conclusion II follows;
(c) if either conclusion I or II follows and
“all we he is do at it”?
(d) if both conclusions I & II follows.
(a) It is not possible to get the above step.
(d) if neither conclusion I nor II follows.
(b) Step VI
17. Statement :
(c) Step IX
Morning walks are good for health.
(d) Step X
Conclusions :
(e) None of these
I. All healthy people go for morning walks.
12. If Step IV of an input is "he is to do what her observe", II. Evening walks are harmful.
which of the following would definitely be the input? 18. Statement :
(a) to is he what observe her do The best way to escape from a problem is to solve it.
(b) he is to what observe her do Conclusions :
(c) is he to what observe her do I. Your life will be dull if you don’t face a problem.
(d) Can't say II. To escape from problems, you should always have
(e) None of these some solutions with you.
13. If Step III of an input is Directions (Qs. 19-21) : In each of the questions below, there are
“when then men can how are you” what would be step VII two or three statements followed by four conclusions numbered
of the input? I, II, III and IV. You have to take the given statements to be true
even if they seem to be at variance with commonly known facts
(a) then can are when you men how
and then decide which of the given conclusions logically follow
(b) how are men can you then when (s) from the given statements.
www.newspaperkorner.wordpress.com
Practice Set - 7 109
www.newspaperkorner.wordpress.com
19. Statement : 23. Which of the following among (A), (C), (D) and (E) may be
Some singers are rockers. the reason behind the better standard of education in private
All rockers are westerners. schools?
Conclusions : (a) Only (A) (b) Only (C)
(c) Only (D) (d) Only (E)
I. Some rockers are singers.
(e) Both (D) and (E)
II. Some westerners are rockers.
III. Some singers are westerners. Directions (Qs. 24-28): Each of the questions below consists of
a questions and two statements numbered I and II given below it.
IV. Some singers are not westerners.
You have to decide whether the data provided in the statements
(a) I, II, and III follow (b) I, II and IV follow are sufficient to answer the question.
(c) II, III and IV follow (d) I, III and IV follow Read both the statements and — Give answer
(e) None follows (a) if the data in statement I alone are sufficient to answer
20. Statements : the question, while the data in statement II alone are
No Indian is an Asian. not sufficient to answer the question.
Some Americans are Indians. (b) if the data in statement II alone are sufficient to answer
Conclusions : the question, while the data in statement I alone are
not sufficient to answer the question.
I. Some Indians are not Asians.
(c) if the data either in statement I alone or in statement II
II. Some Asians are not Americans.
alone are sufficient to answer the question.
III. Some Americans are not Asians. (d) if the data in both the statements I and II together are
IV. All Americans are Asians. not sufficient to answer the question.
(a) Either III or IV follows (b) Either I or II follows (e) if the data in both the statements I and II together are
(c) I and III follow (d) Either II or III follows necessary to answer the question.
(e) None follows 24. How many stations are there while going from station ‘X’ to
21. Statements : station ‘Y’?
Some charts are darts. I. Station ‘G’ precedes station ‘Y’ and station ‘K’ is next
All darts are carts. station after station ‘X’.
Some carts are smarts. II. Station ‘M’ is third from ‘K’ and there are 4 stations
Conclusions : between M and Y.
I. Some charts are carts. 25. How many books did Dinesh purchase in ‘X’ bookshop?
II. Some carts are darts. I. Dinesh wanted to purchase 65 books, but only 45
III. Some darts are smarts. books were available in shop ‘X’.
II. Dinesh selected 37 books but had money to purchase
IV. Some smarts are charts.
27 books and asked for some credit to which the shop-
(a) Only I and II follow (b) Only II and III follows
keeper of ‘X’ bookstall did not agree.
(c) I and II follow (d) I, III and IV follow 26. If the first day of a month is Thursday, how many days were
(e) None follows there in that month?
Directions (Qs. 22-23) : Study the following information carefully I. The fourth Sunday happened to be on 25th.
and answer the given questions. II. The last day of the month was the fifth Saturday of
The standard of education in private schools is much better that month.
than Municipal and Zila Parishad-run schools. 27. How many girls are taller than Samir in his class?
(A) The Municipal and Zila Parishad should make serious efforts I. When students of Samir’s class are ranked in
descending order of their height, Samir’s rank is 17th
to improve standard of their schools.
from the top among all the students and 12th among
(B) All Municipal and Zila Parishad schools should be closed boys.
immediately. II. Samir’s rank from the bottom on the basis of height
(C) Government should raise the standard of education in among boys is 18th and among all students is 29th.
Municipal and Zila Parishad run schools. 28. Among Nitin, Amit, Sudesh, Rekha and Sujata, who came
(D) Private schools charge high amount fees for imparting last for the programme?
education. I. Nitin came after Amit but not after Sujata.
(E) Private sector works better than government sector almost II. Rekha came after Sujata but not after Sudesh.
in all areas. 29. “We have formulated a 10-point programme to curb
22. Which of the following among (A), (B), (C) and (D) may be molestation cases against women and we are confident of
the conclusion which logically follows the given restricting the eve-teasing cases with the help of our sincere
information? cops.”—Director-General of Police of city 'X'.
(a) Only (A) (b) Only (B) Which of the following, if true, would weaken the statement?
(c) Only (C) (d) Only (D) (a) City 'X' is known among those cities where there are
(e) Both (A) and (B) least crime records.
www.newspaperkorner.wordpress.com
110 Practice Set - 7
www.newspaperkorner.wordpress.com 36. Devices that allow you to put information into the computer:
(b) City 'X' is the most populated city in the world.
(c) On several occasions women have complained against (a) input (b) output
(c) type (d) print
police cops and have charged them of indulging in
(e) None of these
eve-teasing. 37. The ___________ key and the _________ key can be
(d) Literacy rate of city 'X' is the highest amongst all cities. used in combination with other keys to perform shortcuts
(e) Many NGOs are co-operating with the police forces in and special tasks.
(a) Control, Alf
ensuring a peaceful life in city 'X'.
(b) Function, toggle
30. “Monetary loss can be made good but any loss to the (c) Delete, Insert
country's culture can never be filled.” — A minister of (d) Caps Lock, Num Lock
country X (e) None of these
If the above statement is true, then which of the following 38. Unsolicited commercial email is commonly known as
(a) spam (b) junk
has been assumed in the statement? (c) hoaxes (d) hypertext
(a) People hanker after money ignoring cultural values. (e) None of these
(b) One can’t bear monetary loss. 39. Text and graphics that have been copied are stored in an
(c) Traditional culture in country X is degrading day by area called the
(a) Pasteboard (b) copyboard
day.
(c) clipboard (d) cuttingboard
(d) It is human nature to hanker after money as well as (e) None of these
adopt foreign culture. 40. The process of writing out computer instructions is known
(e) Preserving culture is our utmost duty. as
(a) assembling (b) compiling
Directions (Q. 31-35): Read the following information to answer (c) executing (d) coding
the questions: (e) None of these
41. _______ is the data that has been organised or presented
Five persons P, Q, R, S and T are sitting around a circular table.
in a meaningful fashion.
The group comprises a Professor, a Doctor and a Politician. The (a) A process (b) Storage
Politician is sitting between the Doctor and his wife, S. P, the (c) Software (d) Information
Professor, is married to T, who is the sister of Q. The Doctor is (e) None of these
seated on the right of R. Both the females are unemployed. 42. A _________ is a computer attached to the Internet that
runs a special Web server software and can send Web pages
31. P is sitting on the right of which of the following? out to other computers over the Internet.
(a) Doctor (b) His wife (a) Web client
(c) Politician (d) S (b) Web system
(e) Cannot be determined (c) Web page
(d) Web server
32. How is P related to Q?
(e) None of these
(a) Brother (b) Brother-in-law 43. Bandwidth refers to
(c) Uncle (d) Husband (a) the cost of the cable required to implement a WAN
(e) Cannot be determined (b) the cost of the cable required to implement a LAN
(c) the amount of information a peer-to-peer network can
33. Who among the following is unmarried?
store
(a) Professor (b) Doctor (d) the amount of information a communications medium
(c) Politician (d) Cannot be determined can transfer in a given amount of time
(e) None of these (e) None of these
44. SMTP, FTP and DNS are applications of the ______ layer.
34. Who is the Doctor?
(a) data link (b) network
(a) S (b) Q (c) transport (d) application
(c) P (d) R (e) None of these
(e) Cannot be determined 45. The difference between people with access to computers
35. Who amonst them must be a graduate? and the Internet and those without this access is known as
the
(a) P (b) Q (a) Digital divide (b) Internet divide
(c) R (d) S (c) Web divide (d) Broadband divide
(e) More than One (e) None of these
www.newspaperkorner.wordpress.com
Practice Set - 7 111
www.newspaperkorner.wordpress.com
DATA ANALYSIS & INTERPRETATION 50. How many possible ways are there for sending petrol from
Directions (Qs. 46 - 50): These questions are based on the table any refinery to any district?
and information given below. (a) 63 (b) 42
There are 6 refineries, 7 depots and 9 districts. The refineries (c) 54 (d) 378
are BB, BC, BD, BE, BF and BG. The depots are AA, AB, AC, (e) None of these
AD, AE, AF and AG. The districts are AAA, AAB, AAC, Directions (Qs. 51-55): Study the following raph and table to
AAD, AAE, AAF, AAG, AAH ad AAI. Table A gives the answer the questions that follow.
cost of transporting one unit from refinery to depot. Table
Population (in lakhs) in three cities ‘A’, ‘B’ and C
B gives the cost of transporting one unit from depot to a
district. City A City B City C
Table A 70
BB BC BD BE BF BG 60
60
AA 928.2 537.2 567.8 589.9 589.9 800.1

Populations (In lakh)


AB 311.1 596.7 885.7 759.9 759.9 793.9 50 48 50
AC 451.1 0 320.1 780.1 720.7 1000 40
40 36 36
AD 371.1 150.1 350.1 750.1 650.4 980.1 32
30
AE 1137.3 314.5 0 1158 1158 1023 30
24 25 25
AF 617.1 516.8 756.5 1066 1066 406.3 20 20 20
20 16
AG 644.3 299.2 537.2 1093 1093 623.9 12 10
15
10
Table B
AA AB AC AD AE AF AG 0
AAA 562.7 843.2 314.5 889.1 0 754.8 537.2 1950 1960 1970 1980 1990 2000
Percentage of population who are below poverty line
AAB 532.7 803.2 284.5 790.5 95.2 659.6 442
AAC 500.7 780.2 0 457.3 205.7 549.1 331.5 City A % City B % City C %
AAD 232.9 362.1 286.2 275.4 523.6 525.3 673.2 Year below poverty below poverty below povery
line line line
AAE 345.1 268.6 316.2 163.2 555.9 413.1 227.8
1950 75% 60% 80%
AAF 450.1 644.3 346.2 372.3 933.3 402.9 379.1 1960 72% 55% 72%
AAG 654.5 0 596.7 222.7 885.7 387.6 348.5 1970 60% 48% 70%
AAH 804.1 149.6 627.2 360.4 1035.3 537.2 498.1 1980 60% 40% 66%
AAI 646 255 433.5 137.7 698.7 112.2 161.5 1990 64% 36% 60%
46. What is the least cost of sending one unit from any 2000 50% 25% 40%
refinery to any district? 51. What is the number of people of City C in the year 1990 who
(a) 95.2 (b) 0 are below poverty line?
(c) 205.7 (d) 284.5 (a) 18.6 lakh (b) 21.6 lakh
(e) None of these (c) 22.5 lakh (d) 24 lakh
(e) 25 lakh
47. What is the least cost of sending one unit from any refinery
52. What is the total population of city 'A', B and 'C' in year
to the district AAB? 2000 which is above poverty line?
(a) 0 (b) 284.5 (a) 72 lakh (b) 90 lakh
(c) 95.2 (d) 294.8 (c) 96 lakh (d) 116 1akh
(e) None of these (e) None of these
48. What is the least cost of sending one unit from refinery BB 53. The number of people of City C in year 1950 who are below
to any district? poverty line is what percentage of the number of people of
City A in year 1990 which are below poverty line?
(a) 284.5 (b) 311.1
(a) 72.5% (b) 52%
(c) 451.1 (d) 297.5 (c) 41.25% (d) 31.25%
(e) None of these (e) 27.75%
49. What is the least cost of sending petrol from refinery BB to 54. What is the average number of people of city A, B and C in
district AAA? year 1980 which are below poverty line?
(a) 765.6 (b) 1137.3 (a) 12.8 lakh (b) 13.6 lakh
(c) 1154.3 (d) 1174.8 (c) 14.5 lakh (d) 15.6 lakh
(e) None of these
(e) None of these
www.newspaperkorner.wordpress.com
112 Practice Set - 7
www.newspaperkorner.wordpress.com
55. The number of people of City A in year 1970 who are below 61. What is the total number of girls qualified from city A?
poverty line is how much % more than the number of people (a) 504 (b) 564
of City B in the year 1970 who are below poverty line?
(c) 594 (d) 614
(a) 12.5% (b) 15%
(c) 17.5% (d) 20% (e) 634
(e) 22.5% 62. What is the difference between the number of girls appeared
Directions (Qs. 56-60): Read the information carefully and from city D and the number of boys appeared from city F?
answer the questions below to it. (a) 613 (b) 687
Three dealers 'A', 'B' and 'C sold a total number of 1200 computer
(c) 746 (d) 797
systems (both laptop and desktop) in a month. The number of
desktops sold by Dealer 'A' is 10% of the total number of systems (e) None of these
sold by all three dealers and the total number of systems sold by 63. What is the ratio of the number of boys qualified from C to
Dealer 'B' is 415, which is 210 more than the number of laptops the number of girls qualified from 'E'?
sold by Dealer 'A'. The number of laptops sold by Dealer 'B' is
(a) 4:5 (b) 6:5
200% of the number of desktops sold by Dealer 'A' number of
laptops sold by Dealer-C is 125% of the number of laptops sold (c) 7:8 (d) 10:9
by Dealer-B. (e) 11:11
56. What is the total number of desktops sold by Dealer C? 64. What is the difference between the total number of boys
(a) 120 (b) 140 appeared and the total number of girls qualified from all six
(c) 160 (d) 175
cities?
(e) 180
57. What is the total number of laptops sold by all three dealers? (a) 39750 (b) 40800
(a) 645 (b) 725 (c) 41766 (d) 42843
(c) 730 (d) 745 (e) 43177
(e) 750
58. The number of desktops sold by Dealer A is what percentage 65. Total number of girls qualified from all six cities is what
of the number of laptops sold by Dealer C? percentage of the total number of qualified candidates
(a) 40% (b) 50% (Answer in approximate value)?
(c) 60% (d) 70% (a) 43% (b) 47%
(e) 80%
(c) 53% (d) 57%
59. What is the ratio of the number of desktops sold by Dealer
B to the number of laptops sold by Dealer C? (e) 45%
(a) 4:7 (b) 5:9 Directions (Qs. 66-70): Following table represents the number
(c) 5:11 (d) 7:12 of primary school teachers and the percentage of female teachers
(e) 8:15 among them in four different years.
60. What is the difference between the number of desktops
sold by Dealer 'B' and 'C' together and the number of total 1980 1990 2000 2010
computer systems (both desktop and laptop) sold by Dealer City
Total % F Total %F Total % F Total % F
A?
A 7200 42% 8400 49% 9750 40% 11200 48%
(a) 50 (b) 40
(c) 30 (d) 20 B 6600 47% 8200 52% 9200 37% 10500 50%
(e) 10 C 8500 56% 9600 41% 12800 42% 13200 45%
Directions (Qs. 61-65): In the given table the information D 7000 53% 7800 39% 8500 46% 9600 48%
about the number of candidates appeared in an examination, the E 10600 61% 11500 57% 12400 54% 13200 55%
number of candidates qualified in the examination, % of girls F 7500 48% 9000 51% 10600 55% 11900 60%
among total appeared candidates and % of boys among qualified G 7800 42% 8100 43% 8700 51% 9500 44%
candidates is given.
66. What is the total number of female teachers in the year
% Girls % Boys 2010?
Total Total
City among among (a) 32614 (b) 33553
Appeared Qualified
appeared qualified (c) 29986 (d) 39754
A 12300 1800 37% 67% (e) 27262
B 11500 1250 48% 60% 67. What is the difference between the total number of
C 16400 1680 43% 40% female teachers and the total number of male teachers in
D 15600 1500 55% 47% the year 1980?
E 17000 1120 51% 50% (a) 674 (b) 676
F 18100 2100 57% 53%
(c) 678 (d) 680
(e) 682
www.newspaperkorner.wordpress.com
Practice Set - 7 113
www.newspaperkorner.wordpress.com
68. Total number of female teachers of City A in 2000 is what Directions (Qs. 76-80): Study the given pie-chart and table and
percentage more than the total number of female teachers answer the following questions.
of City F in the year 1980? Percentage distribution of total TV viewers
(in prime time) among different TV channels.
(a) 8.33% (b) 9.35%
(c) 10.38% (d) 7.33%
Sahara TV
(e) 6.38% 8%
69. Total number of male teachers of City B in the year 1990 is
what percentage of the total number of female teachers of Colour TV
22%
City C in l990? Star TV
(a) 72% (b) 80% 24%
SAB TV
(c) 100% (d) 120% 14%
(e) 125% Zee TV
70. What is the ratio of the total number of female teachers of 20% Sony TV
City A in the year 2000 to the total number of female teacher 12%
of City B in the year 2010?
(a) 4:7 (b) 12:17 Percentage of urban TV viewers of these channels
(c) 13:18 (d) 21:34 (Total number of TV viewers is 12 lakh)
(e) 26:35 Channel % Urban Viewers
71. A certain amount of money deposited in a bank grows 70%
Star TV
up to `10580 in two years and up to `12167 in three years.
Sony TV 65%
If the interest is compounded annually, what is the rate of
Zee TV 68%
interest?
SAB TV 75%
(a) 8% pa (b) 12% pa
Colors TV 60%
(c) 15% pa (d) 18% pa
Sahara TV 72%
(e) 20% pa
72. The population of a city increases at the rate of 15% per 76. What is the total number of TV viewers from urban areas
annum. If the present population of the city is 32 lakh, what for all the channels?
will be its population after three years? (a) 721780 (b) 786486
(a) 46.4 lakh (b) 48.668 lakh (c) 811920 (d) 824864
(c) 49.24 lakh (d) 45.484 lakh (e) None of these
(e) None of these 77. What is the average number of TV viewers from rural areas
73. The present ages of A, B and C are in the ratio of 2:5 :10* for all the channels?
The present ages of B, C and D are the ratio of 4 :8 :13, what (a) 62178 (b) 64680
(c) 66370 (d) 68484
is the ratio of the present ages of A, B, C and D?
(e) None of these
(a) 2:8:10:13 (b) 12:30:60:91
78. Total number of rural viewers of Sony TV is what percentage
(c) 8:20:40:65 (d) 4:5:10:13
of the total number of urban viewers of the same channel?
(e) None of these
(a) 51.68% (b) 52278%
74. A square room is surrounded by a veranda of width 4 metres.
(c) 53.846% (d) 54.272%
If the area of the veranda is 320 sq m then what is the area of (e) None of these
the room? 79. Total number of Sahara TV viewers from urban areas is
(a) 196sqm (b) 256sqm what percentage more than the total number of rural viewers
(c) 324sqm (d) 400sqm of SAB TV?
(e) 484sqm (a) 61.24% (b) 62.83%
75. A man can swim 72 km upstream and 54 km downstream in (c) 63.58% (d) 64.57%
9 hours. Also, he can swim 84 km upstream and 90 km (e) None of these
downstream in 12 hours. What is the speed of the man in 80. What is the ratio of the total number of rural viewers of Zee
still water? TV to the total number of urban viewers of Star TV?
(a) 9 kmph (b) 12 kmph (a) 7:23 (b) 8:21
(c) 15 kmph (d) 18kmph (c) 9:25 (d) 11:32
(e) 21 kmph (e) None of these
www.newspaperkorner.wordpress.com
114 Practice Set - 7
www.newspaperkorner.wordpress.com
ENGLISH LANGUAGE To make this a reality we will need a standardized platform
from which to develop a student's unique education. This
Directions (Qs. 81-90) : Read the following passage carefully standardized platform will allow us to tailor a custom curriculum
and answer the questions given below it. Certain words have that will be matched to talents, interests and life goals. For the
been printed in bold to help you locate them while answering
educator, a standardized platform will create a way to assist the
some of the questions.
student in discovering a true purpose in life through a unique
Thinking of what education might look like in the next
educational experience. The basics of reading, writing and
decade, one quickly realizes that the trends in technology are
arithmetic will not be taught as much as they will be discovered
leaving a large number of our students behind. Today is an age of
and used. Learning will become a reciprocal experience between
exponential change. New and ever-improving technologies are
the teacher, the student and the machine.
popping up every day and in every corner of society.
Under a standardized platform, each of these three
Educating the best and the brightest in this brave new world will
participants will have a role to play. The teacher will be the
take a new and improved educational paradigm. Allowing our
facilitator, assisting the development of the curriculum and
educational tools to age in the corner of the classroom will be the
inspiring the direction the student takes. The student will be the
mistake that may cost us our future. Throwing away masses of
user, gathering resources, skills and knowledge in an efficient
children to inequitable access will ensure that we languish at the
and measured sequence. The machine will do the work of data
bottom of the global pool of employable workers for decades to
gathering and analysis, which will assist the teacher and student
come.
in refining the curriculum. This data gathering work of the machine
Technology will shape in a way we educate students in the will also free the teacher from the burden of record-keeping and
next decade. A user is not simply a person who uses. For the student, tedious tasks that currently distract from the real job of teaching
being a user should involve using the latest technology in a free and learning.
and autonomous manner. This new-found freedom will allow the Under a standardized system, grade level will be far less important.
student to become an active participant in his/her education instead Achievement and progression will be measured by
of a passive passenger. In our current technological society, being accomplishment and intelligence as a benchmark for success.
a user also means being tracked. Tracking a student means having The question of failure or success will be irrelevant and replaced
the ability to target education towards weaknesses and strengths. with a standard and consistent measurement of potential and
The ability to accurately customize curricula to the individual has overall intelligence. Information will no longer be missed but
been the holy grail of educational philosophy for many years. This continually rehearsed and monitored for retention by the machine.
golden age of technological development may soon enable this Tasks such as data tracking, reporting and record keeping are
dream to become a reality. currently accomplished manually. These tasks could easily be
Current educational curricula and individual assessment is delegated to an intuitive database. Developing a standard to
arbitrary at best. Accurately assessing a student can only be follow would eliminate these tasks and free the teacher to do their
achieved by using modern tracking and database technologies. main job of teaching students.
Imagine a world where every child has a tablet computer with The next decade may mark the moment in history when all are
ready access to the internet. Further, imagine that every student granted equal access to technology. Access to quality education
can access all the knowledge of humankind freely at any moment will only be gained through investment and universal
in time. Continue to imagine a world where a misspelled word standardization of this technology. If we continue to divert wealth
brings up a spelling challenge application instead of an auto towards fruitless goals and corporate greed, this opportunity will
correction. Try to contemplate what it would mean for a teacher be lost or hopelessly delayed.
to have a database of every misspelled word, every misunderstood 81. According to the author, which of the following will be the
concept or every missed, equation for each of their students. Try benefit(s) of introducing a standardized technological
to envision a teacher with the ability to customize the experience platform?
of the individual “user” with minimal effort. Imagine the curriculum (A) Potential of a child will take precedence over the grades
being automatically targeted to the user through an intuitive he/she scores.
educational platform that knows all strengths and each unique (B) Improving the educational syllabus would become
weakness. In the last few hundred years, most individuals would easier.
consider an education as something you receive. You often hear (C) Teachers would be able to devote more time to teaching.
the question asked, “Where did you receive your education?” (a) Only (C) (b) All (A), (B) and (C)
As we proceed through the next decade, education will slowly (c) Only (B) and (C) (d) Only (A) and (B)
move away from reception and toward being custom designed (e) Only (A)
for the individual user. New technology will not only allow us to 82. Which of the following is/are true in the context of the
receive an education, but also develop an education. The question passage?
we might ask in ten years, “How did you develop your education?” (A) In the presence of technology each student would
The question of where will still be important, but the how of the require constant monitoring by other individuals to
matter will be the focus that defines the individual. maximize learning.
www.newspaperkorner.wordpress.com
Practice Set - 7 115
www.newspaperkorner.wordpress.com
(B) Educational philosophy is based on the belief of 88. PARADIGM
tailoring educational syllabus to individual student (a) change (b) shift
capability. (c) future (d) model
(C) The author visualizes that each student 'will have (e) diagnosis
access to technology in the future. Directions (Qs. 89-90) : Choose the word which is most
(a) Only (A) and (B) (b) Only (B) OPPOSITE in meaning to the word printed in bold as used in the
(c) Only (C) (d) Only (B) and (C) passage.
(e) All (A), (B) and (C) 89. INEQUITABLE
83. Which of the following is possibly the most appropriate (a) abundant (b) complete
title for the passage? (c) unequal (d) fair
(a) Technology - A Welcome Relief to Teachers (e) sufficient
90. LANGUISH
(b) Revamping the Educational Sector - An Impossible
(a) ripen (b) hasten
Future (c) flourish (d) weaken
(c) Education - Arbitrary But a Reality (e) climb
(d) Technology and the Economy
Directions (Qs. 91-93) : Each question below has two blanks,
(e) Technology - Reshaping the Future of Education each blank indicating that something has been omitted. Choose
84. How, according to the author, will the perception towards the set of words for each blank which best fits the meaning of the
education change over the next decade? sentence as a whole.
(a) Where an individual gets his education will gain 91. What goes into making a marriage can only be ________
increasing importance by trial and error and couples are best left to ________ out
(b) Education will be viewed as a prerogative of the elite what works.
only (a) discovered, translate (b) regulated, find
(c) Creativity in an individual will be regarded as needless (c) learnt, figure (d) seen, thrash
(d) The importance of education will decline (e) experienced, judge
(e) None of these 92. The producer is known to ________ with new stars and
85. What does the author mean by the term “tracking a fresh talent, and though there have been a few hits and
misses, this filmmaker totally ________ for the new breed.
student”?
(a) try, demands (b) experiment, vouches
(a) Analyzing the strengths and weaknesses of a student (c) sign, goes (d) produce, promises
and designing an educational syllabus accordingly (e) work, supports
(b) Assessing whether a student is paying due attention 93. The Government stated that it had the ________ right to
to the existing curriculum offered by an institute of use as much force as was necessary to regain control of
learning areas by ________ terrorists.
(c) Analyzing the positives and negatives of an (a) free, marked (b) practical, left
educational institute and modifying it suitably to cater (c) fundamental, infest (d) basic, undertaken
to industrial requirements (e) legitimate, dominated
(d) Following a student to the educational institute that Directions (Qs.94-97) : Rearrange the following six sentences
he visits frequently in order to estimate the time he (A), (B), (C), (D), (E) and (F) in the proper sequence to form a
spends there meaningful paragraph; then answer the questions given below
(e) None of these them.
(A) However, many people may not be aware of the numerous
86. According to the author, why is the current education
other areas where it has been applied.
provided not satisfactory? (B) Today, even, those who have little knowledge about the
(a) The teachers providing education are not qualified production of virtual reality are now most likely aware of its
enough to emphasize on quality use in video games.
(b) Quality of education provided is not uniform across (C) Similarly, medical students have substituted a cadaver for a
the globe and individual assessment is subjective fiberglass mould of a body and a headset when training to
(c) The educational sector is fraught with corrupt practices perform surgery.
and thus does not produce the desired result (D) Virtual reality was an unfamiliar concept to many people till
(d) The education system is not guided by technology the early 90's.
and hence the pace of learning is slow (E) Introducing virtual reality to the real world, thus, has already
proven to be beneficial for every industry it encounters.
(e) There are too many schools in the country which
For example, astronaut trainees have recently used virtual
cannot be monitored effectively
reality to simulate a trip to space.
Directions (Qs.87-88) : Choose the word which is most SIMILAR 94. Which of the following should be the FOURTH sentence
in meaning to the word printed in bold as used in the passage. after rearrangement?
87. RECIPROCAL (a) E (b) F
(a) interactive (b) unresponsive (c) A (d) D
(c) reverse (d) regular
(e) B
(e) inverse
www.newspaperkorner.wordpress.com
116 Practice Set - 7
www.newspaperkorner.wordpress.com
95. Which of the following should be the FIRST sentence after able to (106) for healthcare, the government allowed the entry of
rearrangement? private sector to reduce the (107) between supply and demand
(a) A (b) B for healthcare. The establishment of the private sector has
(c) C (d) D resulted in the (108) of opportunities in terms of medical
(e) E equipment, information technology in health services. BPO,
96. Which of the following should be the LAST (SIXTH) telemedicine and medical tourism.
sentence after rearrangement? Large companies and (109) individuals have now started five
(a) A (b) B star hospitals which dominate the space for the high end market.
(c) C (d) D The private sector has made (110) progress, but on the flip side it
(e) E is also responsible for increasing (111) in the healthcare sector.
97. Which of the following should be the SECOND sentence The private sector should be more socially relevant and effort
after rearrangement? must be made to make private sector accessible to the weaker
(a) B (b) C sections of society.
(c) D (d) E 102. (a) shambles (b) failure
(e) F (c) demand (d) prosperity
98. Which of the following should be the FIFTH sentence after (e) ruined
rearrangement? 103. (a) bricks (b) emphasize
(a) A (b) B (c) request (d) stress
(c) C (d) D (e) important
(e) E 104. (a) through (b) across
Directions (Qs. 99-101): In each of these sentences, parts of the (c) sharing (d) with
sentence are numbered (a), (b), (c), (d) and (e). One of these parts (e) on
has some error in it. The error may be either of spelling or 105. (a) made (b) comprise
grammar or wrong word or unnecessary word etc. The letter of (c) consisted (d) is
the part that contains the error is the answer. (e) contained
99. The roll (a) / of the institute is to provide technical support 106. (a) cater (b) provide
to other institutions and to constantly monitor (b)/ their (c) manage (d) survive
facilities (c)/ and performance. (d)/ No error. (e) (e) give
100. The competitive (a)/ edge for survival (b)/ lays (c)/ in the 107. (a) gap (b) position
effective (d)/ use of information technology. No error. (e) (c) distance (d) length
101. The most popular (a)/ method adopted (b)/ by an (e) thought
organisation to communicate (c)/ job vacancies to the public 108. (a) reduction (b) sea
is through advertisement. (d)/ No error. (e) (c) cropping (d) disabling
Directions (Qs. 102-111) : In the following passage there are (e) emergence
blanks, each of which has been numbered. These numbers are 109. (a) needy (b) destitute
printed below the passage and against each, five words are (c) bigger (d) affluent
suggested, one of which fits the blanks appropriately. Find out (e) much
the appropriate word in each case. 110. (a) slowly (b) improve
Prior to independence the healthcare sector in India was in a (c) many (d) improvised
(102) with a large number of deaths and rampant spread of (e) tremendous
infectious diseases. After independence the Government of India 111. (a) speed (b) pace
laid (103) on primary healthcare and India has put in sustained (c) inequality (d) uniformity
efforts to better the healthcare system (104) the country, The (e) seriousness
government initiative was not enough to meet the demands of a Directions (Qs. 112-113) : Which of the phrases (a), (b), (c) and
growing population be it in primary, secondary or tertiary (d) given below each statement should be placed in the blank
healthcare. Alternate sources of finance were critical for the space provided so as to make a meaningful and grammatically
correct sentence? If none of the sentences is appropriate, mark
sustainability of the health sector.
(e), ie ‘None of these’, as the answer.
Till about 20 years ago, private sector ventures in the healthcare
112. Refuting the rationale behind frequent agitations for
seetor (105) of only solo practitioners, small hospitals and nursing
formation of separate States, a recent report __________ .
homes. The quality of service provided was excellent especially
(a) proved that such agitations result in loss of
in the hospitals run by charitable trusts and religious foundations.
governmental property
In 1980’s realizing that the government on its own would not be
www.newspaperkorner.wordpress.com
Practice Set - 7 117
www.newspaperkorner.wordpress.com
(b) indicated that the formation of small states does not 118. The tax levied on the manufacture or production of goods
necessarily improve the economy is called
(c) suggested that only large-scale agitations have been (a) Value Added Tax (b) Customs Duty
effective in bringing out desired change in the past (c) Sales Tax (d) Excise Duty
(d) recommended dividing large States into smaller ones (e) None of these
to improve governance 119. The source(s) of revenue of the state government is/are
(e) None of these (a) entertainment tax
113. Overlooking the fact that water scarcity intensifies during (b) land revenue
(c) excise duty on intoxicants
summer, __________.
(d) Only (a) and (b)
(a) the government issued guidelines to all builders to
(e) All (a), (b) and (c)
limit their consumption to acceptable limits 120. BAFTA awards are associated with which of the following
(b) provision for rainwater harvesting has been made to fields?
aid irrigation in drought-prone areas (a) Science and Technology
(c) the water table did not improve even after receiving (b) Film and Television
normal monsoon in the current year (c) Literature
(d) many residential areas continue to use swimming pools, (d) Bravery
wasting large quantities of water (e) None of these
(e) None of these 121. Which of the following statements is/are correct about
demat accounts in India?
Directions (Qs. 114 -115): In each of the following questions
(a) the term ‘demat’ refers to a dematerialised account for
four words are given of which two words are most nearly the individual Indian citizens to trade in listed stocks or
same or opposite in meaning. Find the two words which are most debentures in electronic form.
nearly the same or opposite in meaning. (b) A demat account is opened by an investor while
114. (A) Fallacy (B) Adage registering with an investment broker (or sub-broker).
(C) Dictum (D) Endorse (c) A demat account is a type of savings account offered
(a) B-D (b) C- D by all the scheduled banks.
(d) Only (a) and (b)
(c) B- C (d) A- D
(e) Only (a) and (c)
(e) A-B
122. Indian Journalists' Association (IJA) in London was set up
115. (A) Elevate (B) Frugal in?
(C) Exult (D) Lament (a) May 1947 (b) August 1947
(a) C- D (b) A-B (c) January 1950 (d) September 2000
(c) B- C (d) B- D (e) None of these
(e) A- D 123. When is World Water Day Celebrated annually?
(a) 22 March (b) 5 March
GENERAL/ECONOMY/BANKING AWARENESS (c) 13 March (d) 20 March
(e) None of these
116. Consider the following with respect to Debit Cards :
124. In a mature market, challenges are _____ .
(A) These cards can be used at point of sale centres. (a) Intense (b) Weak
(B) Banks earn huge amount as penalties for defaulting (c) Concentrated (d) Deep
payments through these cards. (e) None of these
(C) It can be used for normal withdrawals. 125. Delivery services offered by restaurants and food chains
Which of the above cannot be feature(s) of a Debit Card? on phone calls are _____.
(a) Only (A) (b) Only (B) (a) Sales promotion (b) Direct marketing
(c) Only (C) (d) Only (A) and (B) (c) Publicity (d) Personal selling
(e) Only (B) and (C) (e) None of these
117. Which of the following ministries gives Dada saheb Phalke 126. ______ is a strength associated with franchising?
(a) Reduces marketing promotional and administration
Award?
costs
(a) Ministry of Science of Technology
(b) Goal conflict does not arise
(b) Ministry of HR Development (c) Reduces levels of channel conflict within the channel
(c) Ministry of Inormation and Broadcasting (d) Combine the strengths of a large sophisticated
(d) Ministry of Finance marketing-oriented organization with the energy and
(e) None of these motivation of a locally owned outlet
(e) All of the above
www.newspaperkorner.wordpress.com
118 Practice Set - 7
www.newspaperkorner.wordpress.com
127. When firms create retail outlets and producers control (d) Non-Banking Finance Companies
purchasing, production and marketing, it creates _____ ? (e) State Finance Corporations
(a) Administered vertical marketing system 138. What is the full form of FEMA
(b) Central vertical marketing system (a) Foreign Exchange Money Agency
(c) Corporate vertical marketing system (b) Foreign Exchange Money Act
(d) Contractual vertical marketing system (c) Full Exchange money Agency
(e) Franchising (d) Foreign Exchange Management Act
128. Fiscal policy means: (e) None of these
(a) The credit policy 139. Which of the following is not a credit rating agency?
(b) The planning policy (a) WADA (b) Fitch
(c) The taxation policy (c) CRISIL (d) Moody
(d) The expenditure and public debt policy (e) S & P
(e) None of these 140. In which year, Indian Rupee was devalued for the first time?
129. Which organisation conducts the functions of a Central (a) 1965 (b) 1966 (c) 1968 (d) 1971
Bank in India? (e) None of these
(a) State Bank of India (b) Central Bank of India 141. SEBI was established in the year
(c) NABARD (d) RBI (a) 1994 (b) 1990 (c) 1992 (d) 1982
(e) All of them undertake parts of the functions (e) 1995
130. Which one of the following is not a quantitative control 142. Non-Banking Financial Companies (NBFCs) are financial
method of credit control? institution that
(a) Cash reserve Ratio (a) holds a banking license
(b) Statutory Liquidity Ratio (SLR) (b) does not hold a banking license
(c) are government undertaking institutions
(c) Bank – Rate
(d) are not government undertaking institutions
(d) Selective Credit Control
(e) None of these
(e) None of these
143. Where is the headquarters of "Society for worldwide
131. Which of the following awards is given by UNESCO to Interbank Financial Telecommunication - SWIFT" situated?
those who popularize use of science in life? (a) New York (b) Los Angels
(a) Booker Prize (b) Magsaysay Award (c) Brussels (d) Hague
(c) Kalinga Award (d) Kalidas Samman (e) None of these
(e) None of these 144. Where has National Financial Switch of IDRBT established?
132. When is International Right To Know day observed? (a) Mumbai (b) New Delhi
(a) 25th Jan. (b) 18th Dec. (c) Hyderabad (d) Bangalore
(c) 24th Oct. (d) 28th Sept. (e) None of these
133. State Bank of India was formed in 145. The following is the mechanism of injecting liquidity by
(a) 1955 (b) 1949 RBI to the financial system.
(c) 1959 (d) 1935 (a) Reverse Repo (b) Hike in CRR
(e) None of these (c) Hike in interest rate (d) Repo
134. The proposed ‘Micro-Finance Bill’ is concerned with: (e) None of these
(a) Regional Rural Banks (b) Co-operative Banks 146. Problem
(c) NABARD (d) All of the above
(e) None of these
135. ‘Out of My comfort Zone’ is the autobiography of a Cricketer
in which he has written the adverse effects of drinking fi gures
culture of his term. Name the Cricketer.
(a) Imran Khan (b) Sanath Jaisurya
(c) Allan Border (d) Steve Waugh Answer figures
(e) None of these
136. The word ‘NELP’ is associated with
(a) Discovery of Oil and Natural Gas
(b) National Highways Extension
(c) Communications
(d) Space Research 147. Chief of RBI is officially known as
(e) None of these (a) President of RBI
137. Companies which generally finance the early-stage, high- (b) Chief Executive of RBI
potential, high risk growth startup companies are _______? (c) Managing Director (MD) of RBI
(a) Micro-Finance companies
(d) Executive Director of RBI
(b) Venture Capital Funds
(e) Governor of RBI
(c) Corporate Financing Banks
www.newspaperkorner.wordpress.com
Practice Set - 7 119
www.newspaperkorner.wordpress.com
148. What does SAPTA stand for: 152. The Royal Commission on Indian Currency and Finance
(a) South Asian Preferential Trade Agreement was also known as __:
(b) South Asian Post Trade Agreement (a) Hilton-Young Commission
(c) SAARC Preferential Trade Agreement (b) Hartog Commission
(d) SAARC Post Trade Agreement (c) Muddiman Committee
(d) Buttler Commission
(e) None of these
(e) None of these
149. Which of the following is not correctly matched?
153. What is the Targeted Operating Ratio for the Indian Railways
(a) Mutual Fund-SEBI (b) ULIP-SEBI
for the year 2016-17 in the Railway Budget 2016-17?
(c) Pension-PFRDA (d) Insurance-IRDA
(a) 90% (b) 92%
(e) None of the above
150. Which among the following is a correct statement? (c) 91% (d) 89%
(a) Cooperative Banks are fully controlled by RBI (e) None of these
(b) Cooperative Banks are partially controlled by RBI 154. ICGS Arnvesh has been commissioned by the Indian Coast
(c) Cooperative Banks are not controlled by RBI Guard in which state?
(d) Cooperative Banks are fully controlled by Banking (a) Tamil Nadu (b) Kerala
Regulation Act, 1949 (c) West Bengal (d) Andhra Pradesh
(e) None of the above is correct (e) None of these
151. "Muhammad Yunus" is associated with Banking Industry 155. The World Tuberculosis Day is observed on which date?
and is acclaimed for his contribution in the field of ____? (a) February 10 (b) March 24
(a) Micro-finance (c) March 28 (d) April 5
(b) Technological advancement (e) None of these
(c) Consumer finance
(d) Banking Reforms in Bangladesh
(e) Low-cost housing

www.newspaperkorner.wordpress.com
120 Practice Set - 7
www.newspaperkorner.wordpress.com

Time : 30 min. Max. Marks : 50

1. Write a letter on any one of the following topics : (20 marks)


(a) You are chairman of local business establishment committee. Write a letter to the manager of a local bank branch
requesting that banks' working hours should be increased by one hour in the evening to facilitate depositing of cash by
your members into their account.
(b) Write a Letter Asking a bank or other Creditor to give further Consideration to your requesting for Credit after credit has
been denied you.
(c) Write a letter to Editor of local newspaper focusing on the increasing menace of thefts and robberies in your locality/
neighborhood. Also suggest useful measures to tackle the problem
2. Write a paragraph on any one of the following in not more than 150 words (10 marks)
(a) Is banning movie a solution to the problem of hurt feelings?
(b) Lobbying in business
(c) Internet Privacy
3. Write an essay on any one of the following in about 250 words. (20 marks)
(a) Online accounts management
(b) Micro finance and inclusive growth
(c) Tackling hunger globally

www.newspaperkorner.wordpress.com
Practice Set - 7 121
www.newspaperkorner.wordpress.com
Answer Key
1 (d) 16 (d) 31 (e) 46 (b) 61 (c) 76 (c) 91 (c) 106 (b) 121 (d) 136 (a) 151 (a )
2 (a ) 17 (e) 32 (b) 47 (c) 62 (d) 77 (b) 92 (b) 107 (a ) 122 (a) 137 (b) 152 (a )
3 (b) 18 (b) 33 (b) 48 (b) 63 (b) 78 (c) 93 (e) 108 (b) 123 (a) 138 (d) 153 (b)
4 (e) 19 (a ) 34 (b) 49 (a ) 64 (c) 79 (d) 94 (b) 109 (d) 124 (b) 139 (a) 154 (d)
5 (d) 20 (c) 35 (e) 50 (d) 65 (b) 80 (b) 95 (d) 110 (e) 125 (b) 140 (b) 155 (b)
6 (b) 21 (c) 36 (a ) 51 (b) 66 (d) 81 (b) 96 (e) 111 (c) 126 (e) 141 (c)
7 (c) 22 (a ) 37 (a ) 52 (c) 67 (b) 82 (d) 97 (a ) 112 (c) 127 (c) 142 (b)
8 (b) 23 (c ) 38 (a ) 53 (d) 68 (a ) 83 (e) 98 (c) 113 (d) 128 (d) 143 (c)
9 (c) 24 (e) 39 (c) 54 (e) 69 (c) 84 (e) 99 (a ) 114 (c) 129 (d) 144 (c)
10 (b) 25 (b) 40 (d) 55 (d) 70 (e) 85 (d) 100 (c) 115 (a ) 130 (d) 145 (d)
11 (d) 26 (b) 41 (d) 56 (c) 71 (c) 86 (d) 101 (c) 116 (b) 131 (c) 146 (d)
12 (a ) 27 (a ) 42 (d) 57 (d) 72 (b) 87 (a) 102 (a ) 117 (c) 132 (d) 147 (e)
13 (b) 28 (e) 43 (d) 58 (a ) 73 (c) 88 (d) 103 (d) 118 (d) 133 (a) 148 (a )
14 (c) 29 (c) 44 (d) 59 (d) 74 (b) 89 (d) 104 (b) 119 (e) 134 (d) 149 (b)
15 (e) 30 (e) 45 (a ) 60 (e) 75 (c) 90 (c) 105 (c) 120 (b) 135 (d) 150 (b)

Answers & Explanations


(1 - 5): Conclusions :
P % Q means P < Q I. M $ R Þ M < R (True)
P © Q means P > Q II. N $ T Þ N < T (True)
P # Q means P = Q 5. (d) D $ T Þ D < T
P $ Q means P < Q T%BÞT<B
P @ Q means P > Q B@ FÞB>F
1. (d) B # F Þ B = F Hence, D < T < B > F
F$HÞF<H Conclusions :
H© K Þ H> K I. D # T Þ D = T (Not true)
Hence, B = F < H > K II. D @ F Þ D > F (Not true)
Conclusions : (6 - 9 ):
I. H @ B Þ H > B (Not true)
II. K $ B Þ K < B (Not true) Name of Father's Name Mother's Name
2. (a) H @ T Þ H > T Children J M V R Bra Ri Pu Ba An Vi
T©NÞT>N Amit Ö × × × × × Ö × × ×
N$WÞN<W Bunty × Ö × × × Ö × × × ×
Hence, H > T > N < W
Reema × × Ö × × × × Ö × ×
Conclusions :
I. N $ H Þ N < H (True) Veena × × × Ö × × × × Ö ×
II. W $ H Þ W < H (Not true) Doly × × × × Ö × × × × Ö
3. (b) H $ F Þ H < F
F%MÞF<M Note:J = Jayant; M = Manoj; V = Vinod; R= Ritesh; Bra = Brajesh,
M©HÞM>J Ri = Rita; Pu = Punam; Ba = Babita; An = Anju; Vi = Vinita
Hence, H < F < M > J 10. (b) A cavity is a hole in a tooth.
Conclusions : (11-16): Here the rule followed is:
I. J $ F Þ J < F (Not true) P. If Input is 1 2 3 4 5 6 7, then Step I becomes 5 1 2 6 3 4 7.
II. M © H Þ M > H (True)
Q. If Step I is 1 2 3 4 5 6 7, then Step II becomes 1 3 5 7 2 4 6.
4. (e) M $ T Þ M < T
T%RÞT<R R. If Step II is 1 2 3 4 5 6 7, then Step III becomes 1 5 6 2 7 3 4.
M©NÞM>N S. If Step III is 1 2 3 4 5 6 7, then Step IV becomes 6 4 2 7 5 3 1.
Hence, N < M < T < R
www.newspaperkorner.wordpress.com
122 Practice Set - 7
www.newspaperkorner.wordpress.com 18. (b) The given statement does not tell anything about life.
Chart It only tells about problems and the way to escape
Input: A B C D E F G from them. Hence conclusion II follows.
Step I: E A B F C D G 19. (a) As positive premises give only positive conclusions.
Step II: E B C G A F D
Step III: E A F B D C G
Step IV: C B A G D F E S R
Step V: D C B F A G E So, W IV is ruled out.
Step VI: D B A E C F G
Step VII: D C F B G A E 20. (c) As no Indian is an Asian and some being part of all, so
Step VIII: A B C E G F D statement 1 Follows. Those some Americans who are
Step IX: G A B F C E D Indian are not Asians.
Step X: G B C D A F E

Again, rules P, Q, R and S are used to get steps V, VI, VII and Amer. I Asian
VIII respectively. The process continues for steps IX, X, ....
For the sake of convenience, we assign a letter for each
word of the Input: 21. (c) There is no correlation among darts and smarts or
and-A, band - B, land - C, hand - D, hind - E, lack - F, job - G smarts and charts.
11. (d) Ca
rts
Input: do we he is it at all
A B C D E F G
Given step: all we he is do at it
G B C D A F E 22. (e) XK...,....GY...... (I)
Now, see the chart. Letters assigned for step X match with K_ _ M; M _ _ _ _ Y ... (II)
the letters obtained for the given step. Combing I and II, we get XK _ _ M_ _ _GY.
12. (a) Thus, there are eight stations between X and Y.
Step IV: he is to do what her observe 24. (b) (I) gives no additional information; it may be
C B A G D F E calculated from the statement itself.
Input: A B C D E F G (II) leads us to an answer. Thursday is 1st day (given
to is he what observe her do in the statement). So first Saturday will be the 3rd
day. And 5th Saturday = 3 + 4 × 7 = 31st day.
13. (b)
Since, this is the last day, there were 31 days in
Step III: when then men can how are you the month.
E A F B D C G 25. (a) From I: (17–1 2) = 5 girl students are taller than Samir
Step VII: D C F B G A E From II: (29 – 18) = 11 girl students are shorter than
how are men can you then when Samir. But, from II alone it is not known how many
14. (c) girls are there in the class.
Input: 26. (e) From I: A > N > S From II: S > R > Sud
stejpan mesic is the president of Croatia From I and II, we get A > N > S > R > Sud
27. (c) How can the strategy to curb molestation of woman
A B C D E F G
be effective if the responsibility to carry out the
StepVIII: A B C E G F D
strategy is on the shoulders of those cops who have
Stejpan mesic is president Croatia of the been accused of heinous charges against women.
15. (e) (31- 35) : Two setting arrangements are possible
Step V: will you hit centuries three again at
D C B F A G E (Professor)
P T T (–) P (Professor)
Step VII: D C F B G A E
will you centuries hit again three at
16. (d) S Q (Doctor) S Q (Doctor)
Step II: has started new BSC batches for PO
R (Politician) R (Politician)
E B C G A F D
Step VI: D B A E C F G
46. (b) The least cost of sending one unit is 0 as it is obvious
PO started batches has new for BSC from table A & B that
17. (e) This statement does not mention anything about
healthy people. Neither does it mention about evening BC ¾¾¾¾
cost= 0
® AC ¾¾¾¾
cost = 0
® AAC
walks. Hence none of the conclusions follows.
www.newspaperkorner.wordpress.com
Practice Set - 7 123
www.newspaperkorner.wordpress.com
or BD ¾¾¾¾ ® AE ¾¾¾¾ ® AAA (100 - 67) 33 ´ 1800
cost = 0 cost = 0 = ´ 1800 = = 594
47. (c) From table A & table B 100 100

BC ® AC , Cost = 0 which is minimum & 55


62. (d) Girls-D = 15600 ´ = 8580
AC ® AAB , Cost = 284.5 100
BC ® AAB , Cost = 0 + 284.5 = 284.5 (100 - 57)
Boys-F = 18100 ´ = 7783
also we have 100
BD ® AE , Cost = 0 which is minimum \ Diff = 8580 – 7783 = 797
AE ® AAB , Cost = 95.2 which is least 40
63. (b) Þ Boys-C = 1680 × = 672
BD ® AAB , Cost = 0 + 95.2 = 95.2 100
Hence least cost from any refinery to AAB = 95.2 (100 - 50)
Girls-E = 1120 ´ = 560
48. (b) Cost from BB ® AB = 311.1 which is least 100
Cost from AB ® AAG = 0 which is also least \ Ratio = 6 : 5
64. (c) Total boys appeared
so least cost from BB ® AAG = 311.1 + 0 = 311.1
= 12300 × 0.63 + 11500 × 0.52 + 16400 × 0.57 + 15600
49. (a) Least cost from BB to AAA would be on the route BB × 0.45 +17000 × 0.49 + 18100 × 0.43
® AC ® AAA = 451.1 + 314.5 = 765.6 = 7749 + 5980 + 9348 + 7020 + 8330 + 7783 = 46210
50. (d) There are 6 refineries, 7 depot, 9 districts. So total ways Similarly, total girls qualified = 4444
from refinery to district = 6 × 7 × 9 = 378 Diff = 46210 – 4444 = 41766
60 65. (b) Total qualified candidates = 9450
51. (b) Reqd Ans = 36 ´ = 21.6 lakh
100 Total qualified girls = 4444
52. (c) Total = 60 × 0.5 + 48 × 0.75 + 50 × 0.6
4444
= 30 + 36 +30 = 96 lakh \ Reqd. Ans. ´ 100 = 47.026 ; 47%
53. (d) C – 1950 = 10 × 0.8 = 8 lakh, 9450
A – 1990 = 40 × 0.64 = 25.6 lakh 66. (d) Total No. = 11200 × 0.48 + 10500 × 0.5 + 13200 × 0.45 +
9600 × 0.48 + 13200 × 0.55 + 11900 × 0.6 + 9500 × 0.44
8
\ Reqd Ans = ´ 100 = 31.25% = 39754
25.6
67. (b) Total no. of females = 27938
32 ´ 0.6 + 30 ´ 0.4 + 25 ´ 0.66
54. (e) Avg = Total no. of males = 27262
3
\ Difference = 27938 – 27262 = 676
19.2 + 12 + 16.5 47.7
= = = 15.9 lakh 68. (a) FemaleF =1980
3 3
55. (d) City - A = 24 × .6 = 14.4 lakh, = 7500 × 0.48 = 3600
City -B = 25 × .48 = 12 lakh FemaleA = 2000 = 9750 × 0.4 = 3900
14.4 - 12 240 3900 - 3600
\ Reqd Ans = ´ 100 = = 20% \ Req. % = ´ 100 = 8.33%
12 12 3600
(56-60) : 69. (c) Male1990 = 8200 × 0.48 = 3936
Female1990 = 9600 × 0.41 = 3936
Dealer Desktop Laptop
A 120 205 3936
\ Req. % = ´ 100 = 100%
B 175 240 3936
C 160 300 9750 ´ 0.4 3900 26
70. (e) Ratio = = = = 26 : 35
10500 ´ 0.5 5250 35
57. (d) (205 + 240 + 300 = 745) 71. (c) Interest on `10580 in one year
120 = (12167 – 10580) = 1587
58. (a) Ans = ´ 100 = 40%
300 1587 ´ 100
\ Rate of interest = = 15%
175 7 10580
59. (d) Ratio = = = 7 :12
300 12 115 115 115
72. (b) Population = 32 ´ ´ ´ = 48.668
60. (e) Diff = (175 + 160) - (120 + 205) 100 100 100
= 335 – 325 = 10 73. (c) A : B : C = 2 : 5 : 10
61. (c) Number of girls Multiplying by 4, A : B : C = 8 : 20 : 40
www.newspaperkorner.wordpress.com
124 Practice Set - 7
www.newspaperkorner.wordpress.com
B : C : D = 4 : 8 : 13
20 32
Multiplying by 5 B : C : D = 20 : 40 : 65 1200000 ´ ´
100 100 = 20 ´ 32 = 8
\ A : B : C : D = 8 : 20 : 40 : 65 80. (b) Ratio = 24 70 24 ´ 70 21
74. (b) Let the side of the square be x metres. 1200000 ´ ´
100 100
\ Area = x2
83. (e) Technology - Reshaping the Future of Education
85. (a) Analyzing the strengths and weaknesses of a student
and designing an educational syllabus accordingly
x 4m 86. (d) The education system is not guided by technology
x and hence the pace of learning is slow
87. (a) The meaning of the word Reciprocal (Adjective) as
4m used in the passage is : mutual; involving two people
or groups who agree to help each other or behave in
Area of the veranda = (x + 8) (x + 8) – x2 = 320 the same way to each other.
x2 + 16x + 64 – x2 = 320 Hence, the words interactive and reciprocal are
16x = 256 synonymous.
\ Area = (16)2 = 256 sq m 88. (d) The meaning of the word Paradigm (Noun) as used in
75. (c) Let the speed of the man upstream be x kmph and that the passage is : a typical example or pattern of
downstream be y kmph. something.
Look at the sentence :
72 54 8 6 The war was a paradigm of the destructive side of
\ + =9 \ + =1
x y x y human nature.
\ 8u + 6v = 1 … (i) Hence, the words paradigm and model are synonymous.
89. (d) The meaning of the word Inequitable (Adjective) as
1 1 used in the passage is : not fair, not the same for
where u = and v =
x y everyone.
Hence, the words inequitable and fair are antonymous.
84 90 14 15
+ = 12 \ + =2 90. (c) The meaning of the word Languish (Verb) as used in
x y x y the passage is : to be forced to stay somewhere or
\ 14u + 15v = 2 … (ii) suffer something unpleasant for a long time.
From equations (i) and (ii), The word Flourish (Verb) means : to develop quickly:
to grow well; thrive.
1 1 Hence, the words languish and flourish are
u= and v = \ x = 12, y = 18
12 18 antonymous.
\ speed of the man in still water 99. (a) The word ‘roll’ makes no sense here. So replace it with
12 + 18 ‘role’, which means ‘function’.
= kmph = 15kmph 100. (c) Replace ‘lays’ with ‘lies’.
2
146. (d) From problem figure (1) to (2) the black square shifts
77. (b) Total no. of rural viewers = 388080
two lines and next to one black square is increased.
388080 From problem figure (2) to (3) black squares shift two
\ Avg = = 64680
6 lines and next to them one black square is increased.
From problem figure (3) to (4) one black square shifts
12 35 two line and two previous designs disappear. The same
78. (c) Rural viewerssony = 1200000 ´ ´ = 50400
100 100 order is repeated.
12 65
Urban viewersony = 1200000 ´ ´ = 93600
100 100
50400
Reqd% = ´ 100 = 53.846%
93600
79. (d) Urban sahara
8 72
= 1200000 ´ ´ = 69120
100 100
14 25
RuralSAB = 1200000 ´ ´ = 42000
100 100
69120 - 4200
\ Reqd% = ´ 100 = 64.57%
42000 www.newspaperkorner.wordpress.com
www.newspaperkorner.wordpress.com

Practice Set - 8

INSTRUCTIONS

• This practice set consists two parts. One part is Objective test and other part is Descriptive test.

• Each question has five options, of which only one is correct. The candidates are advised to read all the
options thoroughly.

• There is negative marking equivalent to 1/4th of the mark allotted to the specific question for wrong
answer.

Time : 3 hrs. Max. Marks : 200

REASONING AND COMPUTER APTITUDE neighbours of each other. Neither C nor E is an immediate
neighbour of either B or the representative from Canara Bank.
1. Which of the following symbols should replace the question Representative from Bank of Maharashtra sits second to right of
mark in the given expression in order to make the expressions D. D is neither the representative of Canara Bank nor Bank of
‘I > L’ as well as ‘M > K’ definitely true? India. G and the representative from UCO Bank are immediate
I>J> K?L<N=M neighbours of each other. B is not the representative of UCO
(a) > (b) < Bank. Only one person sits between C and the representative
(c) < (d) = from Oriental Bank of Commerce.
(e) Either < or < H sits third to left of the representative from Dena Bank.
Representative from Punjab National Bank sits second to left of
2. Which of the following symbols should be placed in the the representative from Syndicate Bank.
blank spaces respectively (in the same order from left to
4. Who amongst the following sit exactly between B and the
right) in order to complete the given expression in such a representative from Bank of India?
manner that ‘S > P’ definitely holds true but ‘S = P’ does not
(a) A and the representative from UCO Bank
hold true?
(b) F and G
P_Q _R_S
(c) H and the representative from Bank of Maharashtra
(a) >, >, > (b) <, =, < (d) H and G
(c) >, <. < (d) <, <, < (e) Representatives from Syndicate Bank and Oriental
(e) None of these Bank of Commerce
3. In which of the following expressions will the expression 5. Who amongst the following is the representative from
‘P < F’ be definitely false? Oriental Bank of Commerce?
(a) F = B > P < M (b) P > B > M = F (a) A (b) C (c) H (d) G (e) D
(c) P < B < F < M (d) B < P < M < F 6. Four of the following five are alike in a certain way based on
(e) None of these the given arrangement and thus form a group. Which is the
Directions (Qs. 4-8) : Study the following information carefully one that does not belong to that group?
and answer the given questions. (a) H - UCO Bank
Representatives from eight different Banks viz.. A, B, C, D, (b) A - Canara Bank
E, F, G and H are sitting around a circular table facing the centre (c) D - Bank of Maharashtra
but not necessarily in the same order. Each one of them is from a (d) E -Syndicate Bank
different Bank viz. UCO Bank, Oriental Bank of Commerce, Bank (e) F - Punjab National Bank
of Maharashtra, Canara Bank, Syndicate Bank, Punjab National 7. Who amongst the following sits second to left of B ?
Bank, Bank of India and Dena Bank. (a) C
F sits second to right of the representative from Canara (b) H
Bank. Representative from Bank of India is an immediate neighbour (c) The representative from Canara Bank
of the representative from Canara Bank. Two people sit between (d) The representative from Punjab National Bank
the representative of Bank of India and B, C and E are immediate
www.newspaperkorner.wordpress.com
126 Practice Set - 8
www.newspaperkorner.wordpress.com
8. Which of the following is true with respect to the given 13. If Raman gets the highest, Vicky should be ranked not lower
seating arrangement? than:
(a) B is the representative from Bank of Maharashtra. (a) Second (b) Third
(b) C sits second to right of H. (c) Forth (d) Fifth
(c) The representative from Dena Bank sits to the (e) None of these
immediate left of the representative from UCO Bank. 14. If Raman is ranked second and Ankit is ranked first, which
(d) A sits second to right of the representative from Bank of the following must be true?
of India. (a) Sunil is ranked third (b) Tony is ranked third
(e) The representatives from Bank of Maharashtra and (c) Priya is ranked sixth (d) All are true
Syndicate Bank are immediate neighbours of each other. (e) None of these
Directions (Qs. 9- 11) : A word arrangement machine, when 15. If sunil is ranked second, which of the following can be
given a particular input, rearranges it following a particular
true?
rule. The following is the illustration of the input and the steps
of arrangement : (a) Deepak gets more than Vicky
Input : Put pocket hand watch he for them . (b) Vicky gets more than Sunil
Step I : Put for he watch hand pocket them. (c) Priya gets more than Raman
Step II : Put he for watch pocket hand them. (d) Priya gets more than Vicky
Step III : Put hand pocket watch for he them. (e) None of these
Step IV : Put pocket hand watch he for them. Directions (Qs. 16-20) : In each group of questions below are
And so on goes the machine. Study the logic and answer two/three statements followed by two conclusions numbered I
the questions that follow : and II. You have to take the given statements to be true even if
they seem to be at variance from commonly known facts and then
9. If step III of a given input be ‘fly sky birds my su fur say’,
decide which of the given conclusions logically follows from the
what is the seventh step of the input ?
two/three statements disregarding commonly known facts.
(a) fly sky birds my su fur say
Give answer (a) if only conclusion I follows;
(b) fly birds sky my fur su say
Give answer (b) if only conclusion II follows;
(c) fly fur su my birds sky say Give answer (c) if either conclusion I or conclusion II follows;
(d) fly su fur my sky birds say Give answer (d) if neither conclusion I nor conclusion II follows;
(e) None of these Give answer (e) if both conclusion I and conclusion II follow.
10. If step VII of an input is ‘slow ran dhurwa pat hak dig vi’, 16. Statements : Some exams are tests. No exam is a
what is step V of that input ? question.
(a) slow dig hak pat dhurwa ran vi Conclusions : I. No question is a test.
(b) slow hak dig pat ran dhurwa vi II. Some tests are definitely not exams.
(c) slow dhurwa ran pat dig hak vi (17-18):
(d) slow ran dhurwa pat hak dig vi 17. Statements : All forces are energies. All energies are
(e) None of these powers. No power is heat.
11. Given the following : Conclusions : I. Some forces are definitely not powers.
Input : Ana dhir raj ran san rah aji II. No heat is force.
What step will be the following arrangement ? 18. Statements : All forces are energies. All energies are
Arrangement : Ana san rah ran dhir raj aji powers. No power is heat.
(a) IV (b) V Conclusions : I. No energy is heat.
(c) VI (d) VIII II. Some forces being heat is a possibility.
(e) None of these (19-20):
Directions (Qs. 12-15) : Study following statements and answer 19. Statements : No note is a coin. Some coins are metals.
the questions: All plastics are notes.
Seven students Priya, Ankit, Raman, Sunil, Tony, Deepak and Conclusions : I. No coin is plastic.
Vicky take a series of tests. No two students get similar marks. II. All plastics being metals is a possibility.
Vicky always scores more than Priya. Priya always scores more 20. Statements : No note is a coin. Some coins are metals.
than Ankit. Each time either Raman scores the highest and Tony All plastics are notes.
gets the least, or alternatively Sunil scores the highest and Deepak Conclusions : I. No metal is plastic.
or Ankit scores the least. II. All notes are plastics.
12. If Sunil is ranked sixth and Ankit is ranked fifth, which of Directions (Qs. 21-24) : Each of the questions below consists of
the following can be true? a question and three statements numbered I, II and III given
(a) Vicky is ranked first or fourth below it. You have to decide whether the data provided in the
(b) Raman is ranked second or third statements are sufficient to answer the question. Read all the
(c) Tony is ranked fourth or fifth three statements and —
(d) Deepak is ranked third or forth Give answer (a) if the data in Statement I and II are sufficient to
(e) None of these answer the question, while the data in Statement III are not required
to answerwww.newspaperkorner.wordpress.com
the question
Practice Set - 8 127
www.newspaperkorner.wordpress.com
Give answer (b) if the data in Statement I and III are sufficient to (d) People may blame the govt. and agitate for not getting
answer the question, while the data in Statement II are not required adequate water for cultivation.
to answer the question (e) None of these
Give answer (c) if the data in Statement II and III are sufficient to 26. The condition of the roads in the city has deteriorated
answer the question, while the data in Statement I are not required considerably during the first two months of monsoon and
to answer the question most of the roads have developed big pot holes.
Give answer (d) if the data in either Statement I alone or Statement Which of the following can be a possible effect of the above
II alone or Statement III alone are sufficient to answer the question. cause?
Give answer (e) if the data in all the Statements I, II and III together (a) The municipal corporation had repaired all the roads in the
are necessary to answer the question. city before onset of monsoon with good quality material.
21. Among six people P, Q, R, S, T and V each lives on a different (b) A large number of people have developed spine related
floor of a six storey building having six floors numbered injuries after regularly commuting long distances by
one to six (the ground floor is numbered 1, the floor above road within the city.
it, number 2 and so on and the topmost floor is numbered (c) The municipal corporation has been careful in choosing
6). Who lives on the topmost floor ? the contractors for repairing roads in the past.
(I) There is only one floor between the floors on which R (d) People always complain about potholed roads during
and Q live. P lives on an even numbered floor. the monsoon months.
(II) T does not live on an even numbered floor. Q lives on (e) None of these
an even numbered floor. Q does not live on the topmost
27. Ethologists, people who study animal behaviour, have
floor.
traditionally divided an organism's actions into two
(III) S lives on an odd numbered floor. There are two floors categories: learned behaviour (based on experience) and
between the floors on which S and P live. T lives on a instinctive behaviour (based on genotype). Some current
floor immediately above R's floor. scholars reject this distinction, claiming that all behaviour
22. There are six letters W, A, R, S, N and E. Is 'ANSWER' the is a predictable interaction of experience and genotype.
word formed after performing the following operations using Which of the following, if true, most strengthens the
these six letters only ? arguments made in the sentence above?
(I) E is placed fourth to the right of A. S is not placed (a) All organisms with identical genotypes and identical
immediately next to either A or E. experience sometimes respond differently in different
(II) R is placed immediately next (either left or right) to E. situations.
W is placed immediately next (either left or right) to S. (b) All organisms with different genotypes and identical
(III) Both N and W are placed immediately next to S. The experience always respond identically in identical
word does not begin with R. A is not placed situations.
immediately next to W. (c) All organisms with similar genotypes and similar
23. Point D is in which direction with respect to Point B ? experience always respond differently in identical
(I) Point A is to the west of Point B. Point C is to the north situations.
of Point B. Point D is to the south of Point C. (d) All organisms with identical genotypes and identical
(II) Point G is to the south of Point D. Point G is 4 m from experience always respond identically in identical
Point B. Point D is 9 m from Point B. situations.
(III) Point A is to the west of Point B. Point B is exactly (e) All organisms with identical genotypes and different
midway between Points A and E. Point F is to the south experience always respond identically in identical
of Point E. Point D is to the west of Point F. situations.
24. How is 'one' coded in the code language ? 28. When income tax rates are reduced, there is an increase
(I) 'one of its kind' is coded as 'zo pi ko fe' and 'in kind and in taxable income, and an increase in taxable income results
cash' is coded as 'ga, to ru ko' in increased total income tax revenues for the government.
(II) Its point for origin' is coded as 'ba le fe mi' and 'make a Which of the following is analogous to the argument
point clear' is coded as 'yu si mi de' above in terms of its logical features?
(III) 'make money and cash' is coded as 'to mi ru hy' and (a) If the city municipal corporation increases taxes on
'money of various kind' is coded as 'qu ko zo hy'. property by 1%, it will raise tax revenues, which can
25. It has been reported in many leading newspapers that the further be used for garbage disposal management.
current year's monsoon may be below the expected level as (b) The Leader of the opposition at the Parliament argued
many parts of the country are still not getting adequate that a restriction on the import of Chin ese
rainfall. manufacturing products will increase the sales of
Which of the following can be a possible fall out of the Indian manufacturing products.
above situation? (c) An advertisement using more than 25 words of text
(a) People from those affected areas with less rainfall may will not be read, and when an advertisement is not
migrate to urban areas. read, the product is not sold. Therefore, the fewer
(b) Govt. may announce ex-gratia payment to all the words an advertisement has, the more effective it will
farmers affected in these areas. be in promoting its product.
(c) Govt. may declare these areas as drought affected areas. (d) The balance of payment of a country will improve if
www.newspaperkorner.wordpress.com
128 Practice Set - 8
www.newspaperkorner.wordpress.com
the country reduces its imports and increases exports. 34. Problem Figures
(e) Citizens earning more than ` 15 lakhs per annum pay
at an increased tax rate of 40%. Citizens earning less
than ` 1 lakh per annum can be exempted from paying
income taxes without decreasing the revenues of the (a) (b) (c) (d) (e)
Income Tax Department. Answer Figures
Directions (Qs. 29-31) : Study the following information carefully
and answer the given questions.
India’s national bird does not have a safe sanctuary in the nation’s
capital itself. Last week about a dozen of them were brutally killed (a) (b) (c) (d) (e)
in some parts of the city and such killing have been going on for 35. Problem Figures
last couple of months.
(A) Killing of national bird should be declared as treason.
(B) Poachers should be banned from entering forests.
(C) Nation’s interest should be kept above all and no action (a) (b) (c) (d) (e)
should be taken towards such trivial issues. Answer Figures
(D) The government has not implemented strict rules and laws
for saving national heritage.
(E) India’s national bird will be included in the list of endangered
(a) (b) (c) (d) (e)
animals in future.
36. __________ allows users to upload files to an online site
29. Which of the following among (A), (B), (C) and (D) may be
so they can be viewed and edited from another location.
a negative course of action which is not admirable?
(a) Only (A) (b) Only (B) (a) General-purpose applications
(c) Only (C) (d) Only (D) (b) Microsoft Outlook
(e) Both (A) and (B) (c) Web-hosted technology
30. Which of the following among (A), (B), (C) and (D) may the (d) Office Live
possible cause of this brutal killing of India’s national bird? (e) None of these
(a) Only (A) (b) Only (B)
37. What feature adjusts the top and bottom margins so that
(c) Only (D) (d) Only (C) the text is centered vertically on the printed page ?
(e) None of these
(a) Vertical justifying (b) Vertical adjusting
31. Which of the following among (A), (B), (D) and (E) may be
the effect of this brutal killing? (c) Dual centering (d) Horizontal centering
(a) Only (A) (b) Only (B) (e) Vertical centering
(c) Only (D) (d) Only (E) 38. Which of these is not a means of personal communication
(e) Both (A) and (E) on the Internet ?
Directions (32-35) : In each of the questions given below which (a) chat (b) instant messaging
one of the five answer figures should come after the problem (c) instanotes (d) electronic mail
figures if the sequence were continued ?
(e) None of these
32. Problem Figures
39. What is the overall term for creating, editing, formatting,
storing, retrieving, and printing a text document ?
(a) Word processing (b) Spreadsheet design
(a) (b) (c) (d) (e) (c) Web design (d) Database management
Answer Figures (e) Presentation generation
40. Fourth-generation mobile technology provides enhanced
capabilities allowing the transfer of both __________ data,
(a) (b) (c) (d) (e) including full-motion video, high-speed Internet access, and
33. Problem Figures videoconferencing.
(a) video data and information
(b) voice and nonvoice
(a) (b) (c) (d) (e) (c) music and video
Answer Figures (d) video and audio
(e) None of these
41. __________ is a form of denial of service attack in which a
(a) (b) (c) (d) (e) hostile client repeatedly sends SYN packets to every port
on the server using fake IP addresses.
www.newspaperkorner.wordpress.com
Practice Set - 8 129
www.newspaperkorner.wordpress.com
(a) Cyborgaming crime (b) Memory shaving 46. What is the respective ratio between the number of mobile
(c) Syn flooding (d) Software piracy phones sold of company B during July and those sold during
December of the same company ?
(e) None of these
(a) 119:145 (b) 116:135
42. Which of these is a point-and-draw device ?
(c) 119 :135 (d) 119:130
(a) mouse (b) scanner (e) None of these
(c) printer (d) CD-ROM 47. If 35% of the mobile phones sold by company A during
(e) Keyboard November were sold at a discount, how many mobile phones
43. The letter and number of the intersecting column and row is of company A during that month were sold without a
the __________. discount?
(a) cell location (b) cell position (a) 882 (b) 1635
(c) 1638 (d) 885
(c) cell address (d) cell coordinates
(e) None of these
(e) cell contents
48. If the shopkeeper earned a profit of `433/- on each mobile
44. A set of rules for telling the computer what operations to phone sold of company B during October, what was his
perform is called a __________. total profit earned on the mobile phones of that company
(a) procedural language (b) structures during the same month ?
(c) natural language (d) command language (a) ` 6,49,900/- (b) ` 6,45,900/-
(e) programming language (c) ` 6,49,400/- (d) ` 6,49,500/-
45. A detailed written description of the programming cycle (e) None of these
and the program, along with the test results and a printout 49. The number of mobile phones sold of company A during
of the program is called __________. July is approximately what percent of the number of mobile
phones sold of company A during December ?
(a) documentation (b) output
(a) 110 (b) 140 (c) 150 (d) 105 (e) 130
(c) reporting (d) spec sheets 50. What is the total number of mobile phones sold of company
(e) Directory B during August and September together ?
(a) 10,000 (b) 15,000
DATA ANALYSIS & INTERPRETATION (c) 10,500 (d) 9,500
DIRECTIONS (Qs. 46-50) : Study the following pie-chart and (e) None of these
table carefully and answer the questions given below : Directions (Qs. 51-55) : Study the following information and
PERCENTAGEWISE DISTRIBUTION OF THE answer the questions that follow :
NUMBER OF MOBILE PHONES SOLD BY The graph given below represents the production (In tonnes)
A SHOPKEEPER DURING SIX MONTHS and sales (In tonnes)
of company a from 2010-2015
Total number of mobile phones sold = 45,000
PRODUCTION AND SALES (In tonnes)

900

800
December 700
16% Production
November 600
July Sales
12% 500
17%
October 400
8% 300

200
September August 2010 2011 2012 2013 2014 2015
25% 22% YEARS
The table given below represents the respective ratio of the
production (in tonnes) of Company A to the production (in
tonnes) of Company B, and the respective ratio of the sales
The respective ratio between the number of mobile phones
(in tonnes) of Company A to the sales (in tonnes) of
sold of company A and company B during six months
Company B.
Month Ratio Year Production Sales
July 8:7 2010 5:4 2:3
August 4:5 2011 8:7 11:12
September 3:2 2012 3:4 9:14
October 7:5 2013 11:12 4:5
November 7:8 2014 14:13 10:9
December 7:9 2015 13:14 1:1
www.newspaperkorner.wordpress.com
130 Practice Set - 8
www.newspaperkorner.wordpress.com
51. What is the approximate percentage increase in the Directions (Qs. 61-65) : Study the following graph and answer
production of Company A (in tonnes) from the year 2013to the questions given below :
the production of Company A (in tonnes) in the year 2014 ?
No. of students (in thousands) who opted for three
(a) 18 (b) 38 (c) 23 (d) 27 (e) 32
different specializations during the given
52. The sales of CompanyA in the year 2013 was approximately what
percent of the production of Company A in the same year ? five years in a university
40
(a) 65 (b) 73 (c) 79 (d) 83 (e) 69
35
53. What is the average production of Company B (in tonnes)
from the year 2010 to the year 2015 ? 30

Number of Students
Hindi

(in thousands)
(a) 574 (b) 649 (c) 675 (d) 593 (e) 618 25
54. What is the respective ratio of the total production (in 20 English
tonnes) of Company A to the total sales (in tonnes) of
15 Mathematics
Company A ?
10
(a) 81 : 64 (b) 64 : 55 (c) 71 : 81 (d) 71 : 55 (e) 81 : 55
55. What is the respective ratio of production of Company B 5
(in tonnes) in the year 2015 to production of Company B (in 0
tonnes) in the year 2012? 2011 2012 2013 2014 2015
Years
(a) 2 : 5 (b) 4 : 5 (c) 3 : 4 (d) 3 : 5 (e) 1 : 4 61. The total number of students who opted for Mathematics
Directions (Qs. 56-60) : Study the following pie-chart and answer in the years 2011 and 2014 together are approximately what
the questions given below: percent of the total number of students who opted for all
Preferences of students among six beverages in terms of three subjects in the same years ?
degree of angle in the pie-chart (a) 38 (b) 28
Total no. of students = 6800 (c) 42 (d) 32
(e) 48
62. Out of the total number of students who opted for the given
three subjects, in the year 2015, 38% were girls. How many
Beverage E, boys opted for Mathematics in the same year ?
64.8°
(a) 1322
(b) 1332
(c) 1312
(d) Cannot be determined
(e) None of these
63. What is the respective ratio between the number of students
56. The number of students who prefer Beverage C are who opted for English in the years 2012 and 2014 together
approximately what percent of the number of students who
and the number of students who opted for Hindi in the year
prefer Beverage D ?
2011 and 2015 together ?
(a) 7 (b) 12 (c) 18 (d) 22 (e) 29
(a) 11 : 5 (b) 12 : 7
57. How many students prefer Beverage B and Beverage E
together ? (c) 11 : 7 (d) 12 : 5
(a) 2312 (b) 2313 (c) 2315 (e) None of these
(d) 2318 (e) None of these 64. If the total number of students in the university in the year
58. What is the difference between the total number of students 2013 was 455030, then , the total number of students who
who prefer Beverage A and C together and the total number opted for the given three subjects were approximately what
of students who prefer Beverage D and F together? percent of the total students ?
(a) 959 (b) 955 (c) 952 (a) 19 (b) 9 (c) 12
(d) 954 (e) None of these (d) 5 (e) 23
59. The number of students who prefer Beverage E and F 65. What is the total number of students who opted for Hindi
together are what percent of the total number of students? and who opted for Mathematics in the years 2012, 2013 and
(a) 18 (b) 14 (c) 26 2014 together ?
(d) 24 (e) None of these (a) 97000
60. What is the respective ratio between the number of students (b) 93000
who prefer Beverage F and the number of students who
(c) 85000
prefer Beverage A ?
(d) 96000
(a) 3 : 11 (b) 3 : 13 (c) 6 : 11
(e) None of these
(d) 5 : 11 (e) None of these www.newspaperkorner.wordpress.com
Practice Set - 8 131
www.newspaperkorner.wordpress.com
Directions (Q. 66-70) : Study the following Pie-chart carefully
and answer the questions given below.
Guava
Survey conducted on 10500 people to find out various 14%
Professionals in the town and percentage of Female Mango
Professionals amongst them 28% Apple
12%
Various Professionals = 10500
Grapes
11%
Engineers Designers Banana
18% 16% 20% Custard
15%
Architects Doctors
11% 21%

Teachers
People staying in various villages
15% Lawyers
19%

A
D 22%
Percentage of Female Professionals
25%
Doctors 20%
Engineers 60%
B
Architects 40% C 21%
Teachers 80% 32%

Lawyers 40%
Designers 35%
71. Mango is the favourite fruit of 50% of the people from
village C. People having their favourite fruit as mango from
66. What is the ratio of the male Engineers and Designers to village C form approximately what per cent of the people
the same-occupation female professionals in the town? having their favorite fruit as mango from all the villages
(a) 41 : 44 (b) 55 : 53 (c) 31 : 35 (d) 44 : 35 together?
(e) None of these (a) 48 (b) 53
67. The total number of Lawyers in town is approximately (c) 61 (d) 57
what per cent of the total number of Doctors in the town? (e) 45
72. 20% of the people from village D have banana as their
(a) 95 (b) 98 (c) 90 (d) 85 (e) 81 favourite fruit and 12% of the people from the same village
68. What is the difference between the total number of male have guava as their favourte fruit. How many people from
and female professionals in the town ? that village like other fruits?
(a) 1284 (b) 1134 (c) 1054 (d) 1164 (a) 764 (b) 896
(e) None of these (c) 874 (d) 968
(e) None of these
69. Female Doctors are what per cent of the female Teachers
73. How many people in all have custard as their favourite fruit?
in the town? (a) 850 (b) 864
(a) 42 (b) 28 (c) 15 (d) 35 (c) 870 (d) 812
(e) None of these (e) None of these
70. What is the ratio of the number of male Architects to the 74. 50% of the people from village B have banana as their
favourite fruit. How many people from other villages have
number of male Teachers in the town ?
the same favourite fruit?
(a) 11 : 5 (b) 3 : 2 (c) 5 : 11 (d) 2 : 3 (a) 1160 (b) 551
(e) None of these (c) 1020 (d) 609
Directions (Q.71-75): Study the following Pie-chart carefully and (e) None of these
answer the questions given below: 75. What is the total number of people having their favourite
A survey conducted on 5800 villagers staying in various fruit as apple and grapes together?
villages and having various favourite fruits. (a) 1334 (b) 1286
Favourite Fruits (c) 1300 (d) 1420
(e) None of these
www.newspaperkorner.wordpress.com
132 Practice Set - 8
www.newspaperkorner.wordpress.com
Directions (Qs. 76-80) : Study the table carefully to answer the following questions.
Number of cars (in thousands) manufactured and sold by six companies over the years.

Company ® A B C D E F
Year ¯
M S M S M S M S M S M S
2010 2.58 1.96 1.98 1.62 1.97 1.53 2.46 2.11 2.35 2.16 1.88 1.50
2011 2.34 1.98 2.15 2.00 2.20 2.03 2.46 2.14 2.45 2.20 1.95 1.62 M - Manufacturer
2012 2.85 2.05 2.35 1.99 2.18 1.87 2.55 2.23 2.60 2.13 2.25 1.93 S - Sold
2013 2.87 2.11 2.62 2.01 2.25 1.95 2.62 2.30 2.79 2.31 2.39 2.08
2014 2.91 2.22 2.71 2.12 2.68 2.32 2.71 2.19 2.88 2.19 2.58 2.10
2015 2.94 2.25 2.84 2.15 2.86 2.36 2.76 2.28 2.90 2.32 2.67 2.30
76. What is the ratio of the total number of cars manufac- (e) (3) and (4)
tured by Companies A, B and C together in the year 2011 82. Drugs worth ` 3 lakhs were ......... from the apartment by the
to that of those manufactured by Companies D, E and F police.
together in the year 2013? (1) manufactured (2) ruptured
(a) 164 : 217 (b) 223 : 260 (3) seized (4) confiscated
(c) 260 : 223 (d) 217 : 164 (5) bought (6) compared
(e) None of these (a) (1) and (4) (b) (2) and (3)
77. What is the percentage of the number of cars sold by (c) (3) and (5) (d) (5) and (6)
Company D in the year 2012 with respect to that of those (e) (3) and (4)
manufactured by it in that year? (rounded off to two 83. An organization .......... to the mission of road safety has
digits after decimal) prepared an action plan for reducing accidents and related
(a) 87.45 (b) 77.28 injuries and fatalities.
(c) 92.54 (d) 79.65 (1) specified (2) inaugurated
(e) None of these (3) committed (4) kicked off
78. In which year was the maximum number of cars manufac- (5) succumbed (6) dedicated
tured by all companies together? (a) (3) and (6) (b) (1) and (5)
(a) 2011 (b) 2012 (c) (3) and (5) (d) (4) and (6)
(c) 2013 (d) 2014 (e) (1) and (3)
(e) None of these 84. A man reportedly .......... two passports with the same
79. What is the approximate per cent increase in the number of photograph, but under different names was arrested by the
cars sold by Company F in the year 2014 from the previous commissioner’s Task Force.
year? (1) possessing (2) examining
(a) 13 (b) 19 (c) 2 (d) 8 (e) 23 (3) surrendering (4) mastering
80. What is the total number of cars sold by Company C in all (5) holding (6) fixating
the years together? (a) (2) and (3) (b) (3) and (6)
(a) 120600 (b) 14205 (c) (1) and (5) (d) (1) and (4)
(c) 12060 (d) 14050 (e) (4) and (5)
(e) None of these 85. The Hollywood star and the Bollywood heroine are being
ENGLISH LANGUAGE ......... as the next big onscreen couple.
(1) labeled (2) explained
Directions (Qs. 81-85) : The following questions consist of a (3) worshiped (4) touted
single sentence with one blank only. You are given six words as (5) exclaimed (6) shouted
answer choices and from the six choices you have to pick up two (a) (2) and (4) (b) (1) and (3)
correct answers, either of which will make the sentence (c) (2) and (6) (d) (1) and (4)
meaningfully complete.
(e) (3) and (4)
81. The ability of a woman to do well does not .......... on whether
Directions (Qs. 86-90) : Rearrange the following seven sentences
it is a man’s world or not, because everyone has his/her
(1), (2), (3), (4), (5), (6) and (7) in the proper sequence to from a
own opportunities.
meaningful paragraph; then answer the questions given below them.
(1) trust (2) depend
(1) To elaborate briefly on these characteristics and dimensions
(3) reckon (4) live
that the author is talking about – NRMs are general tests
(5) rest (6) believe
intended to be used to classify students by percentile for
(a) (4) and (5) (b) (2) and (3)
measuring either aptitude or proficiency for admissions into
(c) (1) and (6) (d) (2) and (5) or placement within a program.
www.newspaperkorner.wordpress.com
Practice Set - 8 133
www.newspaperkorner.wordpress.com
(2) Contrastingly, the CRM, such as a locally produced (e) most
achievement test, measures absolute performance that is 92. (a) raise (b) brings
compared only with the learning objective, hence a perfect (c) refer (d) stop
score is theoretically obtainable by all students who have a
(e) cause
mastery of the pre-specified material, or conversely, all
students may fail the test. 93. (a) sensed (b) called
(3) In most of these books, the authors classify a measurement (c) nothing (d) but
strategy as either norm-referenced (NRM) or criterion- (e) term
referenced (CRM). 94. (a) due (b) results
(4) Another author points out how the type of interpretation (c) reason (d) those
that an NRM offers is the relative performance of the (e) because
students compared with that of all the others resulting in,
ideally, a bell curve distribution. 95. (a) done (b) known
(5) Numerous books on constructing and using language tests (c) ruled (d) bestowed
have been written by various authors. (e) said
(6) CRMs, on the other hand, are more specific, achievement 96. (a) mastering (b) sending
or diagnostic tests intended to be used for motivating (c) melting (d) calming
students by measuring to what percent they have achieved (e) increasing
mastery of the thought or learned material.
97. (a) make-shift (b) ceasing
(7) One of the authors clearly delineates the differences of these
two types by focusing on the categories of “test (c) troubled (d) perpetual
characteristics” and “logistical dimensions.” (e) absent
86. Which of the following should be the FIRST sentence after 98. (a) dwindling (b) manufactured
rearrangement? (c) descending (d) generating
(a) 7 (b) 2 (c) 3 (d) 4 (e) 5 (e) supplied
87. Which of the following should be the SEVENTH (LAST) 99. (a) progress (b) reduced
sentence after rearrangement? (c) existence (d) midst
(a) 1 (b) 2 (c) 3 (d) 4 (e) 5 (e) absence
88. Which of the following should be the FIFTH sentence after
100. (a) repel (b) waft
rearrangement?
(c) monitor (d) bring
(a) 1 (b) 2 (c) 3 (d) 6 (e) 5
89. Which of the following should be the SECOND sentence (e) access
after rearrangement? Directions (Qs. 101-107) : Read the following passage carefully
(a) 1 (b) 2 (c) 3 (d) 4 (e) 6 and answer the questions given below it. Certain words/phrases
90. Which of the following should be the THIRD sentence are printed in bold to help you to locate them while answering
after rearrangement? some of the questions.
(a) 1 (b) 2 (c) 7 (d) 4 (e) 5 In many countries, a combustible mixture of authoritarianism,
Directions (Qs. 91-100) : In the following passage there are unemployment and youth has given rise to disaffection with
blanks, each of which has been numbered. These numbers are strongmen rulers which has in turn spill over into uprising. Young
printed below the passage and against each, five words/phrases people in these countries are far better educated than their parents
are suggested, one of which fits the blank appropriately. Find were. In 1990 the average Egyptian had 4.4 years of schooling;
out the appropriate word/phrase in each case. by 2010 the figure had risen to 7.1 years. Could it be that education,
by making people less willing to put up with restrictions on
Greenhouse gases are only (91) of the story when it comes to freedom and more willing to question authority, promotes
global warming. Changes to one part of the climate system can democratization. Ideas about the links between education, Income
(92) additional changes to the way the planet absorbs or reflects and democracy are at the heart of what social scientists have
energy. These secondary changes are (93)climate feedback's, and long studied. Since then plenty of economists and political
they could more than double the amount of warming caused by scientists have looked for statistical evidence of a causal link
carbon dioxide alone. The primary feedback are (94)to snow and between education and democratization. Many have pointed to
ice, water vapour, clouds, and the carbon cycle.
the strong correlation that exists between levels of education
The most well (95) feedback comes from melting snow and ice in
and measures like the pluralism of party politics and the existence
the Northern Hemisphere. Warming temperatures are already (96)
a growing percentage of Arctic sea ice, exposing dark ocean water of civil liberties. The patterns are similar when income and
during the (97) sunlight of summer. Snow cover on land is also democracy are considered. There are outliers, of course – until
(98) in many areas. In the (99) of snow and ice, these areas go recently, many Arab countries managed to combine energy-based
from having bright, sunlight-reflecting surfaces that cool the wealth and decent education with undemocratic political systems.
planet to having dark, sunlight-absorbing surfaces that (100) But some deduce from the overall picture that as China and other
more energy into the Earth system and cause more warming. authoritarian states get more educated and richer, their people
91. (a) whole (b) part will agitate for greater political freedom, culminating in a shift to a
(c) material (d) issue more democratic form of government.
www.newspaperkorner.wordpress.com
134 Practice Set - 8
www.newspaperkorner.wordpress.com
This apparently reasonable intuition is shakier than it seems. (2) Fast paced economic growth and accountability of
Critics of the hypothesis point out that correlation is hardly those in power.
causation. The general trend over the past half century may have (3) Better standards of living and access to higher
been towards rising living standards, a wider spread of basic education.
education and more democracy, but it is entirely possible that (a) All (1), (2) and (3) (b) Only (2) and (3)
this is being by another variable. Even if the correlation were not
(c) Only (3) (d) Only (1) and (2)
spurious, it would be difficult to know which way causation ran.
Does more education lead to greater democracy? Or are more (e) None of these
democratic countries better at educating their citizens? A recent 105. What according to the author has led to uprisings in
NBER paper compared a group of Kenyan girls in 69 primary authoritarian countries?
school whose students were randomly selected to receive a (a) Lack of access to education.
scholarship with similar students in schools which received no (b) Vast numbers of uneducated and unemployable youth.
such financial aid. Previous studies has shown that the (c) Frustration with the existing system of governance.
scholarship programme led to higher test scores and increased
(d) Unavailability of natural energy resources like coal and oil.
the likelihood that girls enrolled in secondary school. Overall, it
significantly increased the amount of education obtained. For (e) Government’s overambitious plans for development.
the new study the authors tried to see how the extra schooling 106. Which of the following is/are true about China in the context
had affected the political and social attitudes of the women in of the passage?
question. Findings suggested that education may make people (1) China’s citizens are in favor of a more representative
more interested in improving their own lives but they may not form of government.
necessarily see democracy as the way to do it. Even in established (2) China has made huge strides in infrastructure
democracies, more education does not always mean either more developments.
active political participation or greater faith in democracy. Poorer (3) China is in the midst of a political revolution.
and less educated people often vote in larger numbers than their
more educated compatriots, who often express disdain for the (a) None (b) Only (1)
messiness of democracy, yearning for the kind of government (c) Only (1) and (3) (d) Only (2)
that would deal strongly with the corrupt and build highways, (e) All (1), (2) and (3)
railway lines and bridges at a dizzying pace of authoritarian China. 107. What does the phrase “messiness of democracy” convey
101. Which of the following most aptly describes the central in the context of the passage?
theme of the passage? (a) Democratic nations are chaotic on account of individual
(a) Democratic nations are richer and have a better track freedoms.
record of educating their citizens. (b) Most democratic countries frequently have violent
(b) Education does not necessarily lead to greater revolts among their citizens.
enthusiasm for a democratic form of government (c) The divide between the poor and educated is growing
(c) Educated societies with autocratic form of government wider in democracies.
enjoy a better quality of life than democracies. (d) High levels of pollution on account of frantic pace of
(d) Citizens can fulfill their personal aspirations only under infrastructure development.
a democratic form of government. (e) Resigned acceptance of intrinsic corruption in the
(e) Democracy makes citizens more intolerant as it does education system.
not restrict personal freedoms
Directions (Qs. 108-111) : Read each sentence to find out whether
102. Which of the following is most similar in meaning to the
there is any grammatical error or idiomatic error in it. The
word PROMOTES given in bold as used in the passage?
error,if any, will be in one part of the sentence. The letter of that
(a) Up grades (b) Prefers
part is the answer. If there is 'No error', the answer is ‘(e)’.
(c) Recommends (d) Advocates
(ignore errors of punctuation, if any).
(e) Publicizes
103. What conclusion can be drawn from the statistics cited 108. The poll contestants approached /
about Egypt’s education system? (a)
(a) Job prospects have been on the rise in Egypt in recent times.
(b) Authoritarian leaders have played a vital role in the commission complaining that the hoardings
reforming Egypt’s education system. /
(b)
(c) Egypt has one of the youngest and best educated
demographies in the world.
(d) Egypt is likely to be successful vibrant democracy. violated the code of conduct
/
(e) There has been a rise in education levels in Egypt in (c)
recent times.
104. In the context of the passage which of the following and influenced public perception. No error
characterize (s) democracies? /
(d) (e)
(1) Active participation of majority of educated citizens in
electoral process. www.newspaperkorner.wordpress.com
Practice Set - 8 135
www.newspaperkorner.wordpress.com 114. LABOUR
109. The country has / adequate laws but problems /
(1) Expedite (2) To move faster
(a) (b)
(3) Controlled (4) Toil
(a) Only (4) (b) Both (1) and (3)
arise when these are not
/ (c) Both (2), (3) and (4) (d) Only (1), (3) and (4)
(c)
(e) All (1), (2), (3) and (4)
implemented in letter and spirit. No error 115. MEAN
/ (1) Imply (2) Understand
(d) (e)
(3) Average (4) Characterized by
110. The Management feels that / malice
(a) (a) Only (3) (b) Both (1) and (4)
(c) Only (1), (3) and (4) (d) Only (1), (2) and (4)
the employees of the organisation are (e) All (1), (2), (3) and (4)
/
(b)
GENERAL/ECONOMY/BANKING AWARNESS
non-productive, and do not want to work hard. 116. A money deposited at a bank that cannot be withdrawn for
/ /
(c) (d) a preset fixed period of time is known as a __________ .
(a) Term deposit (b) Checking Account
No error (c) Savings Bank Deposit (d) No Frills Account
(e) (e) Current Deposit
117. A worldwide financial messaging network which exchanges
111. As far the issue of land encroachment / messages between banks and financial institutions is
(a) known as __________ .
(a) CHAPS (b) SWIFT
in villages is concerned, people will (c) NEFT (d) SFMS
/
(b) (e) CHIPS
118. The term 'Smart Money" refers to __________ .
have to make a start from their villages by (a) Foreign Currency (b) Internet Banking
/
(c) (c) US Dollars (d) Travelers' cheques
(e) Credit Cards
sensitising and educating the villagers about this issue. 119. Which one of the following is a retail banking product ?
/.
(d) (a) Home Loans (b) Working capital finance
(c) Corporate term loans (d) Infrastructure financing
No error (e) Export Credit
(e) 120. When there is a difference between all receipts and
expenditure of the Govt. of India, both capital and revenue,
Directions (Qs. 112-115) : Below is given a single word with
it is called __________ .
options to its meaning in different contexts. You have to select
(a) Revenue Deficit (b) Budgetary Deficit
all those options which are synonyms of the word when the
(c) Zero Budgeting (d) Trade Gap
context is changed. Select the correct alterative from (a), (b), (c),
(e) Balance of payment problem
(d) and (e) which represents all those synonyms.
121. Which of the following is NOT a function of the Reserve
112. MASK Bank of India ?
(1) Cover (2) Hide (a) Fiscal Policy Functions
(3) Conceal (4) Disguise (b) Exchange Control Functions
(a) Only (1) (b) Both (2) and (4) (c) Issuance, Exchange and destruction of currency notes
(c) Only (2), (3) and (4) (d) Only (1), (2) and (3) (d) Monetary Authority Functions
(e) All (1), (2), (3) and (4)
(e) Supervisory and Control Functions
113. REGULAR
122. Which of the following is NOT required for opening a bank
(1) Present (2) Common
account ?
(3) Indiscriminate (4) Uniform
(a) Identity Proof (b) Address Proof
(a) Only (4) (b) Both (2) and (4)
(c) Both (1) and (3) (d) Only (2), (3) and (4) (c) Recent Photographs (d) Domicile Certificate
(e) All (1), (2), (3) and (4) (e) None of these
www.newspaperkorner.wordpress.com
136 Practice Set - 8
www.newspaperkorner.wordpress.com
123. With reference to a cheque which of the following is the (a) Tade Ipadeola (b) Abdul Kalam
“drawee bank” ? (c) Rebecca Hunt (d) Victoria Grossack
(a) The bank that collects the cheque (e) None of these
(b) The payee's bank 131. Which Maharatna PSU introduced Voluntary retirement
(c) The endorsee's bank Scheme for female Workers?
(d) The endorser's bank (a) Coal India Limited
(e) The bank upon which the cheque is drawn (b) NTPC
124. Banking Ombudsman Scheme is applicable to the business (c) Bharat Heavy Electricals Limited
of __________ . (d) Steel Authority of India
(a) All scheduled commercial banks excluding RRBs (e) None of these
(b) All scheduled commercial banks including RRBs 132. In which country Smog insurance was introduced to tackle
(c) Only Public Sector Banks air pollution?
(d) All Banking Companies (a) USA (b) China
(e) All scheduled banks except private banks (c) India (d) Brazil
125. Base Rate is the rate below which no Bank can allow their (e) None of these
lending to anyone. Who sets up this ‘Base Rate’ for Banks? 133. Which of the following taxes is the major source of the
income of the Union Government?
(a) Individual Banks, Board
(a) Fringe Benefit Tax (b) VAT
(b) Ministry of Commerce (c) Cash Withdrawal Tax (d) Income Tax
(c) Ministry of Finance (e) Customs and Excise Duty
(d) RBI 134 ‘Sensex’ is the index of which of the following stock
(e) Interest Rate Commission of India exchanges?
126. What is a Debit Card ? (a) New York Stock Exchange
(b) Kolkata Stock Exchange
(a) It is a card issued by a Rating Agency
(c) National Stock Exchange (India)
(b) It is a card which can be used for withdrawing cash or (d) Hong Kong Stock Exchange
making payment even in the absence of any balance in (e) Bombay Stock Exchange
the account 135. Shovna Narayan is associated with which classical dance
(c) It is a card which can be used for withdrawing cash or form of India?
making payment if there is balance in the account
(d) It is a card which carries prepaid balance (a) Bharatnatyam (b) Kathakali
(e) It is a card which can be used for making STD calls (c) Kuchipudi (d) Kathak
127. Bad advances of a Bank are called __________ . (e) None of these
136. What does term ‘bancassurance’ mean?
(a) Bad debt (b) Book debt
(a) Insurance provided by banking organization
(c) Non Performing Asset (d) Out of order accounts (b) Insurance given to bank by central bank
(e) Overdrawn accounts (c) Insurance policy giving bank rate interests
128. By increasing repo rate, the economy may observe the (d) Insurance assured to public
following effects __________. (e) None of these
(a) Rate of interest on loans and advances will be costlier 137. When the price of a substitute of a commodity X falls, then
(b) Industrial output would be affected to an extent the demand for X
(c) Banks will increase rate of interest on deposits
(a) Rises (b) Falls
(d) Industry houses may borrow money from foreign (c) Remains unchanged (d) First rises and then falls
countries (e) None of these
(e) All of these 138. ‘Lenovo Group Ltd.’ which is a major manufacturer of
129. Which of the following schemes is launched to provide personal computers is basically a company based in
pension to people in unorganized sectors in India ? (a) South Korea (b) China
(a) Swabhiman (b) Jeevan Dhara (c) Singapore (d) India
(c) Jeevan Kalyan (d) ASHA (e) USA
(e) Swavalamban 139. Saraswati Samman is given for excellence in the field of
(a) Art and Culture (b) Music
130. Who is the Author of the Book ‘The Sahara Testament’?
(c) Classical Dance (d) Literature
(e) www.newspaperkorner.wordpress.com
None of these
Practice Set - 8 137
www.newspaperkorner.wordpress.com
140. Which of the following is the abbreviated name of the 149. Commercial papers are normally issued in the multiple of
organisation working for the welfare of the children? ____ crore.
(a) CII (b) CIC (a) 10 (b) 5
(c) CRY (d) PAC (c) 15 (d) 2
(e) None of these (e) None of these
141 Which of the following is known as plastic money? 150. Which of the following cannot be called as a debt
(A) Demand Draft instrument as referred in the financial transactions?
(B) Credit Card
(a) Certificate of deposit (b) Bonds
(C) Debit Card
(c) Stocks (d) Commercial papers
(a) Only (A) (b) Only (B)
(e) Debentures
(c) Only (C) (d) Only (B) and (C)
(e) All (A), (B) and (C) 151. Which of the following committee has given its recom-
mendations on "Financial inclusions"?
142. NREGA stands for
(a) National Rural Employment Guarantee Act (a) Rakesh Mohan committee
(b) Nuclear Rural Electrification Guarantee Act (b) Rangarajan committee
(c) Natural Resources Employment Guarantee Act (c) Sinha committee
(d) All of these (d) Gadgil committee
(e) None of these (e) None of these
143 Which of the following certainly is an effort in the 152. Which of the following policies of the financial sectors is
direction of Financial Inclusion? basically designed to transferring local financial assets
(a) No-frills Accounts into foreign assets freely and at market determined
(b) Foreign Currency Accounts exchange rates?
(c) Internet Banking (a) Capital account convertibility
(d) Anywhere Banking (b) Financial deficit management
(e) None of these (c) Minimum support price
144. Insurance cover for bank deposits in our country is (d) Restrictive trade practices
provided by
(e) None of these
(a) SBI (b) Government of India
153. What is the capital expenditure (CAPEX) pegged or the
(c) DICGC (d) GIC
Plan size proposed for the Indian Railways in the Railway
(e) None of these
Budget 2016-17?
145. PAN number is required for all transactions above:
(a) 2.2 lakh crore rupees
(a) ` 25,000 (b) ` 50,000
(b) 2.54 lakh crore rupees
(c) ` 1 lakh (d) ` 10 lakh
(e) No such restriction (c) 1.21 lakh crore rupees
146 In CAMELS, E stands for what? (d) 1.28 lakh crore rupees
(a) Equity (b) Earnings (e) None of these
(c) Equal (d) Eligibility 154. As per the Union Budget 2016-17, how much per cent of
(e) None of these FDI will be allowed through FIPB route in marketing of
147. Which of the following institution is meant for small and food products produced and manufactured in India?
medium enterprises? (a) 88% (b) 99%
(a) RBI (b) NABARD (c) 100% (d) 49%
(c) IFCI (d) SIDBI (e) None of these
(e) None of these 155. How much amount has been allocated towards
148. The commercial paper can be issued by _____? recapitalisation of Public Sector Banks?
(a) Corporates (a) 50,000 crore rupees
(b) Reserve Bank of India (b) 20,000 crore rupees
(c) IDBI (c) 35,000 crore rupees
(d) Every non-banking company
(d) 25,000 crore rupees
(e) None of the these
(e) None of these

www.newspaperkorner.wordpress.com
138 Practice Set - 8
www.newspaperkorner.wordpress.com

Time : 30 min. Max. Marks : 50

1. Write a letter on one of the following:- (20 marks)


(a) Your friend has been attending bank interviews for the last two years. He didn't get selected in his fourth interview for
which results were announced recently. He is feeling dejected. Write a letter to him suggesting that if he prepares well for
future he will succeed
(b) You and your friends visited Nanital for a long weekend. You stayed at hotel which promised a lot of facilities. But you
cam e back disappointed. Write a letter to manger pg the hotel complaining about some problems that you faced while
staying there.
(c) Write a letter to DCP of your locality complaining against the loud and blaring loudspeakers and offer some suggestions.
2. Write a paragraph on any one of the following in not more than 150 words (10 marks)
(a) Why Digitization hits metros in India
(b) Internet censorship and copyrights
(c) Is college admission too competitive?
3. Write an essay on any one of the following in about 250 words. (20 marks)
(a) Reservation in private sector.
(b) Election Reforms.
(c) Women’s Reservation.

www.newspaperkorner.wordpress.com
Practice Set - 8 139
www.newspaperkorner.wordpress.com
Answer Key
1 (d) 22 (b) 43 (c) 64 (b) 85 (d) 106 (b) 127 (c) 148 (a)
2 (d) 23 (b) 44 (e) 65 (e) 86 (e) 107 (c) 128 (c) 149 (b)
3 (b) 24 (e) 45 (a) 66 (e) 87 (b) 108 (c) 129 (e) 150 (c)
4 (c) 25 (e) 46 (c) 67 (c) 88 (d) 109 (c) 130 (a) 151 (c)
5 (e) 26 (b) 47 (c) 68 (b) 89 (c) 110 (e) 131 (a) 152 (a)
6 (b) 27 (d) 48 (d) 69 (d) 90 (c) 111 (b) 132 (b) 153 (c)
7 (d) 28 (c) 49 (e) 70 (a) 91 (b) 112 (e) 133 (e) 154 (c)
8 (e) 29 (c) 50 (a) 71 (d) 92 (e) 113 (d) 134 (e) 155 (d)
9 (a) 30 (c) 51 (d) 72 (e) 93 (b) 114 (a) 135 (d)
10 (b) 31 (d) 52 (b) 73 (c) 94 (a) 115 (c) 136 (a)
11 (c) 32 (a) 53 (c) 74 (b) 95 (b) 116 (a) 137 (b)
12 (d) 33 (d) 54 (e) 75 (a) 96 (c) 117 (b) 138 (b)
13 (c) 34 (c) 55 (c) 76 (b) 97 (d) 118 (e) 139 (d)
14 (e) 35 (d) 56 (b) 77 (a) 98 (a) 119 (b) 140 (c)
15 (a) 36 (b) 57 (a) 78 (e) 99 (e) 120 (a) 141 (d)
16 (d) 37 (a) 58 (c) 79 (d) 100 (d) 121 (a) 142 (a)
17 (b) 38 (c) 59 (d) 80 (c) 101 (d) 122 (d) 143 (a)
18 (a) 39 (a) 60 (a) 81 (d) 102 (d) 123 (e) 144 (c)
19 (e) 40 (b) 61 (d) 82 (e) 103 (e) 124 (b) 145 (b)
20 (d) 41 (a) 62 (e) 83 (a) 104 (a) 125 (d) 146 (b)
21 (e) 42 (a) 63 (a) 84 (c) 105 (c) 126 (c) 147 (d)

Answers & Explanations


1. (d) On putting sign (=) in place of question mark (?) 6. (b) From options
I > J > K = L < N = M Þ means I > L and M > K
H UCO Bank
2. (d) On putting sign (<, <, <) in place of blank space
P < Q £ R £ S Þ means S > P true
but S = P doesnot hold time. second to right
6. (c) 2C < A + E, A + E = C + D
A Canara Bank
Þ 2C < C + D Þ C < D ...(i)
A + D = B + C, C < D Þ A < B ...(ii)
2A > B + D, A < B Þ A > D ...(iii) first to right
A + E = C + D, A > D Þ E < C ...(iv)
From (i), (ii), (iii) and (iv), we get: B > A > D > C > E. D Bank of Maharashtra
(4 -8) : According to the question
A second to right
Bank of Maharashtra
H B
Canara Bank Syndicate Bank E Syndicate Bank

G D second to right
Bank of India Oriental
Bank of Commerce
F Punjab National Bank

F E
Punjab National Bank
second to right
UCO Bank C
Dena Bank Hence, 'A – Canara Bank' does not belong to that
group.

www.newspaperkorner.wordpress.com
140 Practice Set - 8
www.newspaperkorner.wordpress.com
(9-11) : Conclusion I : False
These questions follow the following steps. Conclusion II : False
Original 1 2 3 4 5 6 7 (17-18) :
Step I 1 6 5 4 3 2 7 Power
Step II 1 5 6 4 2 3 7
Step III 1 3 2 4 6 5 7 Energies Heat
Step IV 1 2 3 4 5 6 7
Step V 1 6 5 4 3 2 7 Force
Step VI 1 5 6 4 2 3 7
Step VII 1 3 2 4 6 5 7
Step VIII 1 2 3 4 5 6 7
9. (a) Step IV is similar to original statement. Considering
step III as original step VII will be the step IV in this 17. (b) Conclusion I : False
output scheme. Conclusion II : True
10. (b) Step VII 1 3 2 4 6 5 7 18. (a) Conclusion I : True
Slow ran dhurwa pat hak dig vi Conclusion II : False
\ Step V 1 6 5 4 3 2 7 (19-20):
Slow hak dig pat ran dhurwa vi
11. (c) Arrangement is 1, 5, 6, 4, 2, 3, 7. Hence step VI. Coin
Metals
12. (d) Raman scores the highest and Tony gets the least.
Note
Vickey always scores more than Priya which in turn
scores more than Ankit. If Sunil is ranked sixth and
Ankit is ranked fifth then considering above (a) and
(c) is ruled out. (b) is also not possible. only (d) is true. Plastics
Hence (d) is correct option.
13. (c) From the same conclusion as in the previous question. OR
If Raman gets the highest then Vickey should not be Note Metals
Coin
ranked lower than fourth.
Hence (c) is the correct option.
14. (e) There is a logical error in this question.
Hence (e) none of these is the correct option.
15. (a) If we consider Raman, he scores highest and Tony
lowest as given. If Sunil gets second then Vickey can’t Plastics
get more than Sunil. Priya also can’t get more than OR
Raman. Vickey gets more than Priya. Hence (b), (c), (d) Note Metals
Win
can be ruled out. only (a) is true.
Hence (a) is the correct option.

16. (d)
Test Exams
Plastics
19. (e) Conclusion I : True
Conclusion II: True
20. (d) Conclusion I : False
Question Conclusion II : False
OR 21. (e) From statements I, II and III
Test Exams Floor No. Person
6 P
5 T
4 R
3 S
2 Q
Question 1 V

www.newspaperkorner.wordpress.com
Practice Set - 8 141
www.newspaperkorner.wordpress.com
22 (b) From statements I and III not sold. Hence, the correct answer is Option (c). Note
1 2 3 4 5 6 the relationship is inverse in nature.
A S E 32. (a) The movement and other changes in designs can be
A N S W E R shown as :
23. (b) From statements I and III 1 to 2 2 to 3
North

C
West East N N
These two steps are repeated alternately.
A B E 33. (d) In the subsequent figures respectively one, two
South
zero............ curve(s) is/are added and curves move
along the line segment and get reversed in each
subsequent figure.
D F 34. (c) In the subsequent figures one design is left intact while
24. (e) From Statements I and II other three designs are inverted.
35. (d) In the subsequent figures the star moves three steps
one of its kind zo pi ko fe in clockwise direction inside the hexagon after every
in kind and cash ga to ru ko two figures. The equal sign moves respectively one
and two step(s) in clockwise direction along the sides
its point for origin ba je fe mi of the hexagon. The design C moves in and out the
hexagon in the subsequent figures and moves
make a point clear yu si mi de
respectively two and one step(s) in clockwise direction.
From statements I and III. In other words, this problem is based on the rule (1)
one of its kind zo pi ko fe = (5) and hence (2) = (6).
46-50:
in kind and cash ga to ru ko
Total number Total Number of Total Number of
make money and cash to mi ru hy of Mobiles Mobiles S old of Mobiles S old of
money of various kind qu ko zo hy S old Company A Company B
Ju ly 7650 4080 3570
From statements I, II and III A u g us t 9900 4400 5500
one of its kind zo pi ko fe Septemb er 11250 6750 4500
Octo b er 3600 2100 1500
in kind and cash ga to ru ko No v emb er 5400 2520 2880
Decemb er 7200 3150 4050
its point for origin ba le fe mi
46. (c) Number of mobiles sold of company B in July = 3570
make a point clear yu si mi de
Number of mobiles sold of company B in December
make money and cash to mi ru hy
= 4050
money of various kind qu ko zo hy Required Ratio = 3570 : 4050 = 119 : 135
25. (e) Possible fallout may be drought affected area which is 47. (c) Total mobiles sold by companyA during November = 2520
not given in any options. Total mobiles sold by this company at discount
26. (b) Option (a), (c), and (d) does not explain the effect of = 35% of 2520 = 882
given cause. Only option (b) can be effect of damaged Total mobiles sold by company A without discount
roads. = 2520 – 882 = 1638
27. (d) The argument stated in the information is that all animal 48. (d) Mobile phones sold of company B during October = 1500
actions are a result of an interaction between Total profit earned on the mobile phones
experiences and genotype. Therefore, if all experiences = `(433 × 1500) = ` 6,49,500
and genotype are identical, all actions will also be 49. (e) Number of mobile phones sold of company
identical. This is mentioned in option (d). A during July = 4080
28. (c) Option (d) is not analogous from any angle. Point of
Number of mobile phones sold by company A during
Chinese manufacturing is also not logical in option
December = 3150
(b). Option (c) exhibits the analogous relationship. If
number of words is high, the text will not be read and Required percentage = 4080 ´ 100 = 129.5 » 130%
when an advertisement text is not read, the product is 3150

www.newspaperkorner.wordpress.com
142 Practice Set - 8
www.newspaperkorner.wordpress.com
50. (a) Mobile phones sold of company B during August = 5500 60. (a) Required Ratio = 21.6º : 79.2º = 3 :11
Mobile phones sold of company B during September 61. (d) Required percentage
= 4500
Total number of mobile phones = 5500 + 4500 = 10,000 15000 + 30000
= ´ 100
5000 + 35000 + 15000 + 25000 + 30000 + 30000
2015 - 2014
51. (d) Percentage increase = ´ 100
2013 45000
= ´100 < 32.14 ; 32% ∋Approx (
140000
700 - 550
= ´ 100 = 27.2 » 27%
550 62. (e) In 2015,
Total number of girls = (20 + 20 + 15)
400
52. (b) Percent of production = ´ 100 = 72.72 » 73%
550 38 55 ´ 38 ´ 100
´ ´ 1000 = = 17100
100 100
53. (c) Year Production of B Total number of boys = 45000 – 17100 = 27900
2010 600 Total number of boys who opted for Mathematics
2011 700
2012 800 5
= 27900 ´ = 3100
2013 600 45
2014 650 63. (a) Required Ratio = ( 25 + 30) : (5 + 20) = 55 : 25 = 11 : 5
2015 700
(15 + 10 + 15) ´ 1000 ´ 100
64. (b) Required Percentage =
600 + 700 + 800 + 600 + 650 + 700 455030
= = 675
6
40´1000
54. (e) Total production of company A = 4050 = ´100 < 8.79 ; 9% ∋Approx (
455030
Total sales of company A = 2750
Required ratio Þ 4050 : 2750 = 81 : 55 65. (e) Required total number of students
55. (c) Required ratio = production of B in the year 2010 : = (5 + 35 + 15 + 15 + 20 + 5) ´ 1000
Production of B in the year 2012 = 95 ´ 1000 = 95000
Þ 600 : 800 Þ 3 : 4
66. (e) Male Engineers + Male Designers
56. (b) Required percentage
40% of (18% of 10500) + 65% of (16% of 10500)
14.4º
= ´100 < 11.765 ; 12% ∋ Approx ( Female Engineers + Female Designers
122.4º
= 60% of (18% of 10500) + 35% of (16% of 10500)
\ Required ratio = (40 × 18)
æ 57.6º + 64.8º ö
57. (a) Required Number = ç
è ÷ø ´ 6800 + (65 × 16) : (60 × 18 + 35 × 16)
360º
= (720 + 1040) : (1080 + 560)
122.4º = 1760 : 1640 = 44 : 41
= ´ 6800 = 2312
360º
19
58. (c) Required difference 67. (c) Required % = ´ 100 » 90%
21

( 79.2º +14.4º ) ~ (122.4º + 21.6º ) ´ 6800 68. (b) % of female professionals =


= [20% of 21% + 60% of 18% + 40% of 11% + 80% of
360º
15% + 40% of 19% + 35% of 16%]
93.6º ~ 144.0º 1
= ´ 6800 = 952 = [420 + 1080 + 440 +1200 + 760 + 560]%
360º 100

59. (d) Required percentage 4460


= % = 44.6%
100
64.8º +21.6º 86.4º
= ´ 100 = ´ 100 = 24% \ % of male professionals
360º 360º
= 100% – 44.6% = 55.4%
www.newspaperkorner.wordpress.com
Practice Set - 8 143
www.newspaperkorner.wordpress.com
\ Required diff
2.10 - 2.08
= (55.4 – 44.6)% of 10500 79. (d) The required per cent increase = ´ 100
2.08
= 10.8% of 10500 = 10.8 × 105 = 1134
69. (d) Required % 0.2
= ´ 100 = 9.6% » 8%
2.08
20% of 21 20 ´ 21
= ´ 100% » ´ 100%
89% of 15 80 ´ 15 80. (c) The required total
420 = (1.53 + 2.03 + 1.87 + 1.95 + 2.32 + 2.36) × 1000 = 12060
» 35%
12 81. (d) Option (d) is the answer and there are two reasons for
it. First one is that from the context of situation
60 ´ 11
70. (a) Required ratio = = 11: 5 independence of women from the man's world is
20 ´ 15
indicated so some word to show dependency should
71. (d) No of persons from village C
be used. Depend and Rest both show dependence so
= 32% of 5800 = 1856
these are the answers. Second one is that as blank
From village C 50% of 1856 = 928 persons favourite space is followed by ON a word that connects
fruit is mango.
grammatically so it should be used. Both 'depend on'
28% of 5800 = 1624 people’s favourite fruit is mango and 'rest on' are the correct usage, so it confirms the
928 selection on basis of the first reason. Other options
\ Required % = ´ 100 » 57%
1624 are not logical.
72. (e) People in village D = 25% of 5800 82. (e) Confiscated – means to officially take something away
= 1450 from somebody, especially as a punishment.
\ Required no. of people Seized definitely fits in the blank therefore option E is
= {100 – (20 + 12)}% of 1450 correct.
= 68% of 1450 = 986 83. (a) Organization and mission are two words which can be
73. (c) Required no. = 15% of 5800 = 870 connected with committed or dedicated both the verbs
74. (b) Required no. - organizations committed/ dedicated to …..Objectives.
= 20% of 5800 – (50% of 21% of 5800) Other words like kicked off/inaugurated/ succumbed
= 1160 – 609 = 551 do not match for objectives of organization.
75. (a) Required no. = (12 + 11)% of 5800 84. (c) Holding and Possessing both gives the same meaning
= 23% of 5800 = 1334 and sense to the sentence. Surrendering is
76. (b) The required ratio inappropriate. Mastering can not be linked with
passports. Fixating is also incorrect for the blank space
2.34 + 2.15 + 2.20
= as it gives no logical sense to the sentence. Examining
2.62 + 2.79 + 2.39
can grammatically fit into the blank but not in the
6.69 2.23 context of passage and would give a different direction
= = = 223 : 260
7.80 2.60 to what is being said in the sentence.
77. (a) The requried per cent 85. (d) 'Next big thing' is always a projection or a decision
based on current situation. It is neither worshipped
2.23
= ´ 100 » 87.45% nor shouted nor explained. Rather next big thing is
2.55
touted or labeled on basis of projection.
78. (e)
86-90. Use of words like 'contrastingly' and 'these' in most of
Year Total number of the sentences imply that some sentences precede these
cars manufactured sentences. Only Statement which introduces the topic
(in thousands)
of discussion is Option (E). So it is the first sentence
1. 2010 13.22 of sequence. Statement 3 extends the topic of books.
2. 2011 13.55 So follows naturally. 'these two types' of Statement 7
3. 2012 14.78 are two types discussed in Statement 3. Statement 6 is
4. 2013 15.54 about CRMs so the just before it there should be
5. 2014 16.47 discussion of NRMs. Last statement is 2 as it finishes
6. 2015 16.97 the passage on summarizing note. So the correct order
of sentences is 5374612.
www.newspaperkorner.wordpress.com
144 Practice Set - 8
www.newspaperkorner.wordpress.com
101. (d) Option (d) has plenty of illustrations, references and culminating in a shift to a more democratic form of
structure to support it in the passage. Other options government.
like (e) and (b) are true but not the central theme of the 107. (c) Option (a) is wrong as this is not intended from the
passage. Option (a) and (c) are not true. use of this phrase. Last part of passage has this phrase
103. (e) A rise from the 4% level to 7 % says that there is rise in and poor-rich divide has been discussed their in voting
education in Egypt. pattern or why rich people even educated do not vote.
104. (a) All of the options are mentioned in one or other part of Option (d) is out of the context. Option E can also be
the passage. rejected as corruption is not the issue here.
105. (c) First line of the passage is self explanatory. 109. (c) Here, arise when they are not is used.
106. (b) Only option which can be linked with the passage is 1. 110. (e) No error
Their people will agitate for greater political freedom,

www.newspaperkorner.wordpress.com
www.newspaperkorner.wordpress.com

Practice Set - 9

INSTRUCTIONS
• This practice set consists two parts. One part is Objective test and other part is Descriptive test.

• Each question has five options, of which only one is correct. The candidates are advised to read all
the options thoroughly.

• There is negative marking equivalent to 1/4th of the mark allotted to the specific question for wrong
answer.

Time : 3 hrs. Max. Marks : 200

REASONING AND COMPUTER APTITUDE Step VII: 34 back 19 packet 12 task 7 wind
and Step VII is the last step.
1. Beautiful beaches attract people, no doubt about that. Just
look at the city’s most beautiful beaches, which are amongst As per the rules followed in the above steps, find out in the given
the most overcrowded places in the state. questions the appropriate step for the given input.
Which of the following exhibits a pattern of reasoning similar 2. Input: 9 13 about tariff 24 call 29 even.
to the one exhibited in the argument above? Which of the following will be step IV?
(a) Moose and bear usually appear at the same drinking (a) 29 about 24 9 13 tariff call even
hole at the same time of day. Therefore, moose and (b) 29 about 24 call 9 13 tariff even
bear must be feeling thirsty at about the same time. (c) 29 about 24 call 13 9 tariff even
(b) Children who are scolded severely tend to misbehave (d) 29 about 24 call 13 even 9 tariff
more often than other children. Hence if a child is not (e) Cannot be determined
scolded severely, that child is less likely to misbehave 3. If Step II of an input is “37 desk 34 garden 5 father victory
(c) During warm weather my dog suffers more class than 17”, which of the following steps will be the last step?
during cool weather. Therefore, fleas must thrive in a (a) Step III (b) Step V
warm environment (c) Step IV (d) Step VI
(d) Tally accounting software helps increase the work (e) None of these
efficiency of its users. As a result, these users have
4. If Step I of an input is
more time for other activities.
59 bead tenure father 38 11 ultimate 24
(e) None of these
Which of the following will be Step III?
Directions (Qs. 2-6): Study the following information to answer
(a) 59 bead 38 tenure 11 father ultimate 24
the given questions:
(b) 59 bead 38 11 tenure father ultimate 24
A word and number arrangement machine when given an input
line of words and numbers, rearranges them following a particular (c) 59 bead 38 tenure father 11 ultimate 24
rule in each step. The following is an illustration of input and (d) 59 bead 38 father tenure 11 ultimate 24
steps of rearrangement: (e) None of these
Input: wind packet 19 7 back 12 task 34 5. If the last step of an input is 41 cost 32 over 28 project 17
Step I: 34 wind packet 19 7 back 12 task violet which of the following must be the input?
Step II: 34 back wind packet 19 7 12 task (a) project 32 cost over 17 41 violet 28
Step III: 34 back 19 wind packet 7 12 task (b) project 32 cost over 41 violet 17 28
Step IV: 34 back 19 packet wind 7 12 task (c) project cost 32 over 41 17 violet 28
Step V: 34 back 19 packet 12 wind 7 task (d) Cannot be determined
Step VI: 34 back 19 packet 12 task wind 7 (e) None of these
www.newspaperkorner.wordpress.com
146 Practice Set - 9
www.newspaperkorner.wordpress.com
6. Which of the following will be the Step III of the following Directions (Qs. 11-13): Each of the questions below consists of
input? a questions and two statements numbered I and II given below it.
Input: 24 12 entry sand butter 51 32 carry You have to decide whether the data provided in the statements
(a) 51 butter 32 24 12 entry sand carry are sufficient to answer the question.
(b) 51 butter 32 carry 24 12 entry sand Read both the statements and — Give answer
(c) 51 butter 32 carry 24 entry 12 sand (a) if the data in statement I alone are sufficient to answer
(d) 51 24 12 entry sand butter 32 carry the question, while the data in statement II alone are
(e) None of these not sufficient to answer the question.
7. In a famous experiment at the IISC campus, when a cat (b) if the data in statement II alone are sufficient to answer
smelled milk, it salivated. In the experiment, a bell was rung the question, while the data in statement I alone are
whenever food was placed near the cat. After a number of not sufficient to answer the question.
trials, only the bell was rung, whereupon the cat would (c) if the data either in statement I alone or in statement II
salivate even though no food was present. Such behaviour alone are sufficient to answer the question.
has been observed in other animals such as dogs, monkeys, (d) if the data in both the statements I and II together are
etc. and is a vital input for training domesticated animals. not sufficient to answer the question.
Which of the following conclusions may be drawn from the (e) if the data in both the statements I and II together are
above experiment? necessary to answer the question.
(a) The ringing of a bell was associated with food in the 11. In an examination ‘X’ four tests P, ‘Q, R and S are given.
mind of the cat. Which is the easiest one?
(b) Cats and other animals can be easily tricked. I. Most of the examinees attempted test,’Q’ first. While
(c) A conclusion cannot be reached on the basis of one ‘P’ was left incomplete by many.
experiment. II. Test ‘R’ is found easier than test ‘S’ by all the
(d) Two stimuli are stronger than one. examinees.
(e) None of these 12. Who among A, B, C, D and E teaches History?
Directions (Qs. 8 - 10) : Read the following statements to answer Statements :
the questions that follow: I. Each one of them teaches only one subject. B teaches
Group captain Malhotra is choosing the last part of his crew for Mathematics, while E teaches Science. A or C does not
the spaceship COSMOS, with which he plans to land on the teach Geography. A or D does not teach English.
moon. He needs 4 more crew members of whom at least two must II. C and E are teachers of English and Science
be pilots, the others being engineers. The candidates for Pilots respectively and A is the teacher of Mathematics.
are Dalbir, Eric and Farid. The candidates for Engineers are Lal, 13. How many books did Dinesh purchase in ‘X’ bookshop?
Monty, Naveen and Paul. I. Dinesh wanted to purchase 65 books, but only 45
Eric will not be a crew with Lal, Dalbir and Paul will not crew with books were available in shop ‘X’.
Naveen. II. Dinesh selected 37 books but had money to purchase
8. If Naveen is chosen, which of the following must be other 27 books and asked for some credit to which the shop-
members of the crew ? keeper of ‘X’ bookstall did not agree.
(a) Farid, Lal and Monty Directions (Qs.14-18): In each of the questions below are given
(b) Dalbir, Eric and Monty three statements followed by four conclusions numbered I, II, III
(c) Eric, Farid and Monty and IV. You have to take the given statements to be true even if
(d) Eric. Farid and Paul they seem to be at variance with commonly known facts. Read all
(e) None of these the conclusions and then decide which of the given conclusions
9. If Paul is chosen, which candidates will NOT be chosen to logically follows from the given statements disregarding
be on the crew ? commonly known facts.
(a) Dalbir, Eric and Monty 14. Statements:
(b) Dalbir, Eric and Farid Some boxes are trees.
(c) Dalbir, Farid and Lal Some trees are horses.
(d) Eric, Farid and Lal All horses are fruits.
Conclusions:
(e) None of these
I. Some fruits are boxes.
10. Given the above statements about the relationships among
the potential crew members, which of the following must be II. Some fruits are trees.
true? III. Some horses are boxes
A : If Dalbir is rejected, then Monty must be chosen. IV No fruits are boxes.
B : If Dalbir is rejected, then Farid must be chosen. (a) None follows
C : If Dalbir is chosen, then Paul must also be chosen. (b) Only either II or IV follows
(a) B only (b) C only (c) Only either I or IV and II follow
(c) A and B only (d) A and C only (d) Only either I or III and IV follow
(e) None of these (e) All follows
www.newspaperkorner.wordpress.com
Practice Set - 9 147
www.newspaperkorner.wordpress.com
15. Statements: grand daughter, does not travel with her grandfather and grand
All flowers are buses. mother. B travels with his father E in car Q. F travels with her
Some buses are cats. grand daughter D in car P. A travels with her daughter in car R.
All cats are tigers. 19. How many female members are there in the family?
Conclusions: (a) Three (b) Four
I. Some tigers are buses. (c) Five (d) Data inadequate
II. Some tigers are flowers. (e) None of these
III. Some cats are flowers. 20. Which of the following is one of the married couples?
IV. Some buses are tigers. (a) DB (b) BC
(a) None follows (b) Only I and II follow (c) EF (d) Data inadequate
(c) Only III and IV follow (d) Only I and IV follow (e) None of these
(e) All follows 21. In which car are three members travelling?
16. Statements: (a) P (b) Q
All fans are rooms. (c) Either P or Q (d) R
No room is green. (e) None of these
Some windows are green. 22. How is D related to E?
Conclusions:
(a) Daughter (b) Niece
I. Some windows are fans. (c) Grand daughter (d) Data inadequate
II. Some windows are rooms. (e) None of these
III. Some fans are green.
23. How is G related to A?
IV. No green is fan.
(a) Daughter (b) Sister
(a) Only I follows (b) Only III follows (c) Mother-in-law (d) Data inadequate
(c) Only IV follows (d) Only II and IV follow (e) None of these
(e) All follows 24. A supplement of Vitamin A and Zinc may boost children’s
17. Statements: resistance to Malaria (Observation from one experiment
Some tablets are rains. conducted last year in a village ‘X’). Which of the following,
All dogs are rains. if true, would weaken the statement?
All rains are chairs. (a) No adult in village ‘X’ has fallen sick because of
Conclusions: Malaria.
I. Some chairs are tablets. (b) Last three years there is hardly any case of child
II. All dogs are chairs. affected by Malaria from village ‘X’.
III. Some tablets are dogs. (c) The experiment with Vitamin A and Zinc is being
IV. Some tablets are chairs. duplicated in other nearby cities adjacent to village
‘X’.
(a) All follow
(d) Vitamin A and Zinc are readily available in village ‘X’.
(b) Only, I, II and III follow
(e) Villages adjacent to ‘X’ have reported substantial cases
(c) Only II, III and IV follow
of Malaria affecting mostly children.?
(d) None of these
25. “Forty per cent of our products are sold in rural area, fifty-
(e) Either I or III follow
three per cent are sold in semi-urban area, sixty per cent of
18. Statements: employees are from rural area.” Which of the following
No man is sky. statements is definitely true?
No sky is road. (a) The company’s products are purchased only by its
Some men are roads. employees and their family.
Conclusions: (b) The company does not desire to recruit urban
I. No road is man. employees.
II. No road is sky. (c) The company’s products are required in big urban
III. Some skies are men. cities and metro areas.
IV. All roads are men. (d) The company holds approximately 90% of the market
(a) None follows (b) Only I follows share in its product line.
(c) Only I and III follow (d) Only II follows (e) None of these
(e) All follows 26. ‘‘Cases of food poisoning have been reported from village
Directions (Qs. 19- 23): Study the following information carefully ‘X’. After a dinner party arranged for 100 people, 68 have
and answer the questions given below: been admitted to the hospital, 36 cases are reported to be
A, B, C, D, E, F and G are seven members of a family belonging to out of danger. The food, which was cooked and stored in
three generations. There are two married couples—one each of open space for almost 12 hours earlier was served after
first and second generations respectively. They travel in three reheating it. Investigation is going on.” A news report.
different cars P, Q and R so that no car has more than three Which of the following can be hypothesized for the above
members and there is at least one female in each car. C, who is a information?
www.newspaperkorner.wordpress.com
148 Practice Set - 9
www.newspaperkorner.wordpress.com
(a) Late night dinner parties for large number of people (a) Only (A) (b) Only (B)
result into food poisoning. (c) Only (E) (d) Only (F)
(b) State food is likely to be the cause of food poisoning. (e) Both (E) and (F)
(c) Cases of food poisoning need to be handled carefully. 29. Which of the following among (A), (B) (D) and (E) may be
(d) Cases of food poisoning are not reported in urban the cause of closing down of the mill?
dinner parties. (a) Only (A) (b) Only (B)
(e) Food poisoning is a matter of chance and no preventive (c) Only (D) (d) Only (E)
measure can be suggested. (e) Both (D) and (E)
Directions (Qs. 27-30) : Study the following information carefully 30. According to recent reports, CEOs of large organisations
and answer the given questions. are paid more than CEOs of small organisations. It does not
India’s national bird does not have a safe sanctuary in the nation’s seem fair that just because a CEO is heading a big
capital itselft. Last week about a dozen of them were brutally organisation she/he should be paid more. CEOs’ salary
killed in some parts of the city and such killing have been going should be related to performance, especially growth in terms
on for last couple of months. of sales and profits.
(A) Killing of national bird should be declared as treason. Of course, big organisations are more complex than ‘the
(B) Poachers should be banned from entering forests. small, but all CEOs require significant amount of energy
(C) Nation’s interest should be kept above all and no action and time in managing organisations. There is no proof that
should be taken towards such trivial issues. CEOs of big organisations are more stressed than CEOs of
(D) Now-a-days, workers and owners both are selfish, they care small organisations. All CEOs should be paid according to
only themselves. their performance.
(E) Thousands of workers become unemployed, if mill will close Which of the following, if true, would strengthen the
down. speaker’s argument?
(F) Unemployment gives hike to criminal activities. (a) CEOs of small organisations come from good
27. Which of the following among (A), (B), (C) and (D) may be educational background.
course of action for a profitable concern for long of the mill? (b) CEOs of big organisations are very difficult to hire.
(a) Only (A) (b) Only (B) (c) A few big family businesses have CEOs from within
(c) Only (C) (d) Only (D) the family.
(e) Both (B) and (C) (d) CEOs in big organisation take much longer to reach
28. Which of the following among (A), (B), (E) and (F) may be top, as compared to their counterparts in small
the effect of closing down of the mill? organisations. .
(e) None of these

Directions (Qs. 31-35) : In each of the questions given below which one of the five answer figures on the right should come after the
problem figures on the left, if the sequence were continued?
31. PROBLEM FIGURES ANSWER FIGURES

(a) (b) (c) (d) (e)


32. PROBLEM FIGURES ANSWER FIGURES

P O A S PB P CS B PB C O A B O C AP O A BO B P A O PA B
S O A P S
C S A O
B C A B O O C A PC S PB S C SS C C S
(a) (b) (c) (d) (e)
33. PROBLEM FIGURES ANSWER FIGURES

S C N B = = B B N N = O S B = B = S OS O B S O =
O C C C C
C S S C O S S O
= B N O C O N = S O B C = NN C N B = N B N
(a) (b) (c) (d) (e)

www.newspaperkorner.wordpress.com
Practice Set - 9 149
www.newspaperkorner.wordpress.com
34. PROBLEM FIGURES ANSWER FIGURES

(a) (b) (c) (d) (e)


35. PROBLEM FIGURES ANSWER FIGURES

(a) (b) (c) (d) (e)

36. A typical slide in a slide presentation would not include 41. Usually downloaded into folders that hold temporary
__________. Internet files, _____ are written to your computer's hard
(a) photo images charts, and graphs disk by some of the Web sites you visit.
(b) graphs and clip art (a) anonymous files (b) behaviour files
(3) clip art and audio clips (c) banner ads (d) large files
(d) full-motion video (e) cookies
(e) content templates 42. What is the easiest way to change the phrase, revenues,
37. The PC productivity tool that manipulates data organized profits, gross margin, to read revenues, profits, and gross
in rows and columns is called a __________. margin ?
(a) spreadsheet (a) Use the insert mode, position the cursor before the g
(b) word processing document in gross, then type the word and followed by a space
(c) presentation mechanism (b) Use the insert mode, position the cursor after the g in
(d) database record manager gross, then type the word and followed by a space
(e) EDI creator (c) Use the overtype mode, position the cursor before the
38. In the absence of parentheses, the order of operation is g in gross, then type the word and followed by a space
__________. (d) Use the overtype mode, position the cursor after the g
(a) Exponentiation, addition or subtraction, multiplication in gross, then type the word and followed by a space
or division (e) None of these
(b) Addition or subtraction, multiplication or division, 43. A program, either talk or music, that is made available in
exponentiation digital format tor automatic download over the Internet is
(c) Multiplication or division, exponentiation, addition or called a __________.
subtraction (a) wiki (b) broadcast
(d) Exponentiation, multiplication or division, addition or (c) vodcast (d) blog
subtraction (e) podcast
(e) Addition or subtraction, exponentiation, Multiplication 44. Which PowerPoint view displays each slide of the
or division presentation as a thumbnail and is useful for rearranging
39. To find the Paste Special option, you use the Clipboard slides ?
group on the _____ tab of PowerPoint. (a) Slide Sorter (b) Slide Show
(a) Design (b) Slide Show (c) Slide Master (d) Notes Page
(c) Page Layout (d) Insert (e) Slide Design
(e) Home 45. Different components on the motherboard of a PC unit are
40. An _____ program is one that is ready to run and does not linked together by sets of parallel electrical conducting lines.
need to be altered in any way. What are these lines called ?
(a) interpreter (b) high-level (a) Conductors (b) Buses
(c) compiler (d) COBOL (c) Connectors (d) Consecutives
(e) executable (e) None of these

www.newspaperkorner.wordpress.com
150 Practice Set - 9
www.newspaperkorner.wordpress.com
Directions (Q. 46-50) : Study the following Pie-chart carefully 51. What is the total expenditure during the period under review
to answer these questions. (7 months) in 2013 ?
Total players are 4200 out of which (a) ` 21, 07, 000 (b) ` 96, 07, 000
Female players are equal to 2000
(c) ` 21, 54, 000 (d) ` 21, 24, 000
Total players = 4200 Female players = 2000
(e) None of these
13% 52. What total expenditure has been made during the year 2013

Ru 0%
y
Ru 3%
17% Football
y

gb
and 2014 in the period covered in the graph ?

1
gb
1
Football
22% (a) ` 24, 87, 000 (b) ` 2, 70, 000
25% Lawn
35% Lawn Tennis 40% Tennis (c) ` 48, 27, 000 (d) ` 42, 78, 000
Cricket Cricket (e) None of these

15 ke y
Ho
1 0 k ey
Ho

53. What is the average monthly expenditure during the year

%
c
%
c

2015 covering the period shown in the graph ?


Percentage wise distribution Percentage of female (a) ` 2, 75, 000 (b) ` 2, 70, 000
of players who play five players who play
different sports (c) ` 3, 14, 000 (d) ` 2, 47, 000
different sports
46. What is the difference between the number of the female (e) None of these
players who play lawn tennis and the number of male 54. Which month has been the least expensive during 2015 ?
players who play rugby? (a) June (b) April (c) May (d) July
(a) 94 (b) 84 (c) 220 (d) 240 (e) None of these
(e) None of these 55. The expenditure in April 2015 was.....higher than that of
47. What is the average number of players (both male and corresponding period in 2014.
female) who play football and rugby together?
(a) 1.5% (b) 2% (c) 2.5% (d) 0.94%
(a) 620 (b) 357 (c) 230 (d) 630
(e) None of these (e) None of these
48. What is the respective ratio of the number of female players Directions (Qs. 56-60) : Study the graph to answer these
who play cricket and number of male players who play questions. The graph indicates the amount of money raised
hockey? through various financial instruments from capital market by
(a) 20 : 7 (b) 4 : 21 (c) 20 : 3 (d) 3 : 20 M/s PP Ltd. the amount is in ` Crore.
(e) None of these
public issue right issue private issue
49. What is the total number of male players who play football, 30000
cricket and lawn tennis together? 26430
25000
(a) 1724 (b) 1734 (c) 1824 (d) 1964
(e) None of these 20000 20170 18375
50. Number of male players who play rugby is approximately 14738
what percentage of the total number of players who play 15000 13658 13010
lawn tennis? 10000 11715 7598 7572
(a) 33 (b) 39 (c) 26 (d) 21 (e) 43 7568 6452
5000
Directions (Qs. 51-55) : Study the data presented in the following 6624
2575
2755
graph to answer the questions : 0
2244
2010-11 2011-12 2012-13 2013-14 2014-15
MONTHLY EXPENDITURE OF A FIRM
FROM JANUARY TO JULY DURING 56. The amount of private placement raised as a percentage of
THE YEARS 2013, 2014, 2015 total funds raised was the maximum in the year
(a) 2010 - 11 (b) 2013 - 14
330 (c) 2011 - 12 (d) 2014 - 15
Expenditure in ‘000 rupees

324 (e) None of these


2015 57. The total amount of funds raised was the least in which of
318 the following year
2014 (a) 2014 - 15 (b) 2011 - 12
312
2013 (c) 2010- 11 (d) 2013 - 14
306 (e) None of these
300

0
Jan.

Feb.

Mar.

Apr.

www.newspaperkorner.wordpress.com
June
May

July
Practice Set - 9 151
www.newspaperkorner.wordpress.com
58. The average amount raised by way of public issue for the
period 2010 - 11 to 2014 - 15 excluding the period 2011 - 12 is
(a) ` 16, 112 crore (b) ` 14, 159 crore
(c) ` 17, 628 crore (d) ` 13, 532 crore
(e) None of these
59. What is the percentage of amount raised by public issue to
the total amount raised?
(a) 52% (b) 49.9% (c) 50.19% (d) 46%
(e) None of these
60. What is the percentage decrease in amount raised by rights
issue in 2005 - 06 against that of 2012 -13 ?
(a) 44.4% (b) 42.2% (c) 54.4% (d) 45%
(e) None of these
Directions (Qs. 61-63) : Refer to the following bar diagram
showing the sales (in ` Crores) of top market brands among
pain killers in India and answer the questions.

25

20

15
2014
2005 64. What percentage of students in the Arts faculty are non-
2015
2006 US students?
10
(a) 14% (b) 9% (c) 30% (d) 11%
(e) None of these
5
65. How many students are there in the Engineering faculty?
(a) 420 (b) 410 (c) 390 (d) 440
0
(e) None of these
am
l
ran

lpo

d
i

v
mo

1 2 3 4 5 6 7 8 9
ex
se

lin

uli
oo

66. How many students are there at the University?


ifl
Ni

lon
ve

Ca

Vo
Su

m
M
mb
Vo

Ni
Do

(a) 4650 (b) 4560 (c) 4640 (d) 4450


Co

(e) None of these


61. What is the difference between the sales of Voveran in 2015
67. If six percent of Science students are Asian, how many
and those of Calpol in 2014 (in ` lacs )?
Asian students are studying Science?
(a) 1000 (b) 50
(a) 48 (b) 66 (c) 120 (d) 57
(c) 100 (d) 500
(e) None of these
(e) None of these
68. There are 34 European medical students. What percentage
62. Percentage of increase in sales from 2014 to 2015 is the of the faculty does this represent?
highest for which brand of a pain killer ?
(a) 14% (b) 18% (c) 16% (d) 15%
(a) Voveran (b) Volini
(e) None of these
(c) Dolonex (d) Sumo
Directions (Q. No. 69 & 70) : These questions are based on the
(e) None of these
graph which shows the demand and production statistics of 5
63. Percentage increase in sales from 2014 to 2015 is the lowest
T.V. companies.
for which brand of a pain killer ?
(a) Voveran (b) Volini
(c) Moov (d) Nise
(e) None of these
Directions (Q. No. 64-68) : Based on the information given below :
The pie charts below show the percentage of students in
each faculty at North West University and the number of non-US
students in the Arts faculty. These percentages have been
rounded to the nearest whole number. There are a total of 1049
students in Arts faculty. Use this information to answer the
following questions.
www.newspaperkorner.wordpress.com
152 Practice Set - 9
www.newspaperkorner.wordpress.com
69. What is the difference between the avergae demand and 78. Seema bought 20 pens, 8 packets of wax colours,
the average production of the five companies taken 6 calculators and 7 pencil boxes. The price of one pen is
together? ` 7, one packet of wax colour is ` 22, one calculator is
(a) 1400 (b) 400 ` 175 and one pencil box is ` 14 more than the combined
(c) 280 (d) 138 price of one pen and one packet of wax colours. How much
(e) None of these amount did Seema pay to the shopkeeper?
70. The production of the company D is how many times of the (a) ` 1,491 (b) ` 1,725
production of the company A? (c) ` 1,667 (d) ` 1,527
(a) 1.8 (b) 1.5 (e) None of these
(c) 2.5 (d) 1.11 79. A man walks at the speed of 5 km/hr and runs at the speed
(e) None of these of 10 km/hr. How much time will the man require to cover
71. The average marks in English subject of a class of 24 the distance of 28 km, if he covers half (first 14 km) his
students is 56. If the marks of three students were misread journey walking and half his journey running ?
as 44, 45 and 61 of the actual marks 48, 59 and 67 respectively, (a) 8.4 hrs (b) 6 hrs (c) 5 hrs (d) 4.2 hrs
then what would be the correct average? (e) None of these
(a) 56.5 (b) 59 (c) 57.5 (d) 58 80. a, b, c and d are four consecutive numbers. If the sum of a
(e) None of these and d is 103, what is the product of b and c ?
72. In a test, a candidate secured 468 marks out of maximum (a) 2652 (b) 2562 (c) 2970 (d) 2550
marks 'A'. If the maximum marks 'A' were converted to 700 (e) None of these
marks, he would have secured 336 marks. What were the
maximum marks of the test? ENGLISH LANGUAGE
(a) 775 (b) 875 (c) 975 (d) 1075 Directions (Q. 81-90) : Read the following passage carefully
(e) None of these and answer the questions given below it. Certain words are given
73. The simple interest accrued on an amount of ` 22,500 at the in bold to help you locate them while answering some of the
end of four years is ` 10,800. What would be the compound questions.
interest accrued on the same amount at the same rate at the The wakeup call that China represents to India is not limited
end of two years ? to its showpiece urban centres or that New Delhi hopes India will
(a) ` 16,908 (b) ` 5,724 experience the benefits that the Olympic games have brought to
(c) ` 28,224 (d) ` 8,586 Beijing. More pertinent is the comparison of the agricultural
(e) None of these sectors of the two countries. Why and how has China managed
74. Rubina could get equal number of ` 55, ` 85 and ` 105 to outstrip India in agriculture when 25 years ago the two countries
tickets for a movie. She spent ` 2940 for all the tickets. How were on par on most parameters? Both have traditionally been
many of each did she buy? agrarian economies and over half their populations continue to
(a) 12 (b) 14 depend on the land for their livelihood. With large populations
(c) 16 (d) Cannot be determined and histories of famine, India and China share concern on issues
(e) None of these such as food security. However, while India’s agricultural sector
75. Ramola's monthly income is three times Ravina's monthly is projected to grow by about 2.5 per cent this year – a slide from
income. Ravina's monthly income is fifteen percent more the previous year’s growth–, China’s has been steadily growing
than Ruchira's monthly income. Ruchira's monthly income at between 4 per cent and 5 per cent over the last fifteen years.
is ` 32,000. What is Ramola's annual income ? The widest divergence between India and China is in the
(a) ` 1,10,400 (b) ` 13,24,800 profitable horticultural sector with the production of fruits and
(c) ` 36,800 (d) ` 52,200 vegetables in China leaping from 60 million tonnes 1980 compared
(e) None of these to India’s 55 million tonnes at the same time, to 450 million tonnes
76. In an Entrance Examination Ritu scored 56 percent marks, in 2003 ahead of India’s corresponding 135 million tonnes.
Smita scored 92 percent marks and Rina scored 634 marks. China’s added advantage lies in the more diversified composition
The maximum marks of the examination are 875. What are of its agricultural sector with animal husbandry and fisheries
the average marks scored by all the three girls together? which account for close to 45 per cent of growth compared to 30
(a) 1929 (b) 815 (c) 690 (d) 643 per cent for India.
(e) None of these According to the latest report by the Economic Advisory
77. The respective ratio between the present age of Manisha Council, the traditional excuses for India’s substandard
and Deepali is 5 : X. Manisha is 9 years younger than performance in the farm sector are inadequate since India is placed
Parineeta. Parineeta's age after 9 years will be 33 years. The favourably when compared to China in terms of quantity of arable
difference between Deepali's and Manisha's age is same as land, average farm size, farm mechanisation etc. The reasons for
the present age of Parineeta. What will come in place of X? China having outperformed India are threefold: technological
(a) 23 (b) 39 improvements accruing from research and development (China
(c) 15 (d) Cannot be determined has over 1,000 R&D centres devoted to agriculture), investment in
(e) None of these rural infrastructure and an increasingly liberalised agricultural
www.newspaperkorner.wordpress.com
Practice Set - 9 153
www.newspaperkorner.wordpress.com
policy moving away from self-sufficiency to leveraging the (d) Research staff even those on contract are entitled to
competitive advantage with a focus on “efficiency as much as increments for good performance.
equity”. Investment in rural infrastructure, roads, storage facilities, (e) None of these
marketing facilities are also crucial but government support in 84. What argument has been posed against implementation of
India has mainly been through subsidies, not investment. There subsidies?
has been much debate about subsidies and their utility; the
(A) Subsidies sacrifice equity for efficiency.
opposing view being that subsidies are against the market reforms
and distort the market as well as reduce resource efficiency. In (B) Subsidies hamper efficient resource utilisation.
contrast to the 2,046 applications for the registration of new plant (C) Subsidies reduce private sector investment and
varieties in China over the past few years, data reveals that involvement in agriculture.
despite India having the largest number of agricultural scientists (a) Both (A) & (B) (b) Only (B)
in the world India’s current research track record is abysmal, (c) Both (B) & (C) (d) Only (A)
equivalent to what China achieved in the 1980s. Far from (e) None of these
developing new strains, the number of field crop varieties fell by 85. Which of the following cannot be said about Indian
50 per cent between 1997 and 2001 despite the fact that there was
agricultural universities?
sharp and
(A) Attendance is poor because of the dwindling funds to
sustained increase in funding. One reason is that majority of the
carry out research.
budget is eaten up by staff salaries with only 3 per cent being
allotted for research. In contrast, most agricultural research centres (B) Enrolments of students and qualified staff have fallen
in China must use Central government funding purely for research. because of the lack of funds for salaries.
Funds relating to salaries and other administrative incidentals (C) Allotment for research funding by the government is
must be generated by the centres themselves. The centres and non-existent.
scientists are thus encouraged to engage in joint ventures with (a) Only (B) (b) Both (A) & (B)
private sector companies to form commercial signoffs from their (c) Both (B) & (C) (d) All (A), (B) & (C)
research. In fact, research staff are now being hired on a contract (e) None of these
basis with pay based on performance and salaries raised
86. Which of the following is not responsible for China’s
proportionately for those who perform well. India needs to learn
successful transformation of its agricultural sector?
from China’s example and adopt a pragmatic approach if it has to
meet its targets of the Eleventh Five Year Plan. (A) Change in philosophy from self-sufficiency to
81. What has been the major area of difference in the competitiveness and efficiency
development of the agricultural sectors of India and China? (B) Greater allocation for subsidies
(a) Quantity of arable land in China is far greater than in (C) Increased investment in marketing and distribution
India. networks
(b) Food security is not a concern for China as the country (a) Only (B) (b) Both (B) & (C)
is basically self-sufficient. (c) Both (A) & (C) (d) All (A), (B) & (C)
(c) China has experienced substantial growth in (e) None of these
production in allied agricultural activities like Directions (Q. 87-88) : Choose the word which is most nearly
horticulture. the SAME in meaning as the word printed bold used in the
(d) India’s agricultural sector is too diversified so it is passage.
difficult to channel funds for development. 87. concern
(e) None of these
(a) importance (b) regret
82. Which of the following is/are area/s in which China has not
(c) business (d) anxiety
outdone India?
(e) emphasis
(A) Development of urban infrastructure
(B) Activities allied to agriculture like animal husbandry 88. pertinent
(C) Successful bids for international sporting events (a) intense (b) sizeable
(a) Either (A) or (B) (b) Only (B) (c) practical (d) visible
(c) Only (A) (d) Both (A) & (C) (e) significant
(e) None of these Directions (Q. 89-90): Choose the word which is most OPPOSITE
83. How are Chinese agricultural research facilities governed? in meaning of the word printed in bold as used in the passage.
(a) Salaries of staff are linked to performance and this 89. sustained
hampers productive research. (a) intermittent (b) discouraged
(c) habitual (d) defeated
(b) Their funding comes from the government alone to
(e) restrained
prevent private companies from manipulating the
90. pragmatic
direction of their research.
(a) faithful (b) unwilling
(c) A fixed proportion of government grants is allotted to
(c) idealistic (d) practical
be utilised for administrative incidentals which cannot
(e) www.newspaperkorner.wordpress.com
inexperienced
be exceeded.
154 Practice Set - 9
www.newspaperkorner.wordpress.com
Directions (Q. 91-95): Read each sentence to find out whether In our system, a vast gap (101) the life children lead at
there is any grammatical error in it. The error if any, will be in school and what they experience outside. The space where they
one part of the sentence. The letter of that part is the answer. If are (102) to learn about life is so far removed from (103) that we
there is no error, the answer is (e), ie ‘No error’ (Ignore the might as well ask (104) to buy space suits for their little ones,
errors of punctuation, if any). instead of school uniforms. (105) they read, listen to and copy
91. (a) During the flood / (b) Shilpa’s colleagues / (c) prayed for from the blackboard is so meticulously deodorised and (106)
/(d) her safety. (e) No error that it carries no resonance of experienced reality and (107) in
92. (a) Everyone admires Deepa /(b) because of her ability /(c) life.
to generate /(d) so good ideas. /(e) No error The school day becomes a (108) of didactic songs and
93. (a) The company has announced /(b) a bonus for all /(c) memorised information. Special (109) are marked by elaborate acts
employee who /(d) achieve their sales targets. /(e) No error of sycophancy and preaching. School authorities (110) stop talking
94. (a) The Cashier cannot/(b) give no explanation /(c) for the about values, but ignore the cynicism felt by the young over the
money /(d) that is missing. /(e) No error high levels of chicanery and verbosity they find in adult talk.
95. (a) The manager has /(b) confidence in the engineer /(c) 101. (a) occurs (b) separates
ability to /(d) design the new software. /(e) No error (c) bridges (d) escapes
Directions (Q. 96-110) : In each of the following sentences there (e) finds
are two blank spaces. Below each five words have been denoted 102. (a) about (b) worried
by letters (a), (b), (c), (d) and (e). Find out which pair of words can (c) compelled (d) supposed
be filled up in the blanks in the sentence in the same sequence to (e) deprived
make the sentence meaningfully complete. 103. (a) reality (b) school
96. Research shows that an organisation’s _____ to cater to (c) fantasy (d) imagination
the customers changing needs will _____ its success. (e) existence
(a) intent, realise (b) ability, determine 104. (a) teachers (b) principals
(c) capacity, lead (d) desire, insure (c) school (d) coaching classes
(e) flexibility, acquire (e) parents
97. In many rural areas hospitals are _____equipped because 105. (a) When (b) How
of the _____ of funds from the government. (c) What (d) Whether
(a) well, dearth (b) faulty, lacking (e) Whenever
(c) optimally, disparity (d) inadequately, scarcity 106. (a) sanctified (b) written
(e) ill surplus (c) emphasised (d) memorised
98. A company can _____ the potential of its employees _____ (e) imbibed
various training programmes. 107. (a) demand (b) culture
(a) emphasise, for (b) exploit, from (c) miseries (d) joy
(c) enhance, through (d) appraise, by (e) applicability
(e) identify, under 108. (a) programme (b) ritual
99. Adding to a growing body of research ______ cutting back (c) consult (d) tradition
on sweetened beverages it is now found that drinking
(e) store
______ sugary drinks may help lower blood pressure.
109. (a) persons (b) leaders
(a) for, all (b) sustaining, increased
(c) locations (d) occasions
(c) against, lesser (d) behind, more
(e) indications
(e) supporting, fewer
110. (a) always (b) deliberately
100. The blame game for the air tragedy is already in full ______
(c) seldom (d) relentlessly
with the authorities involved making attempts to ______
(e) invariably
for themselves.
Directions (Q. 111-115): Rearrange the following six sentences
(a) sway, defend (b) view, try
(A), (B), (C), (D), (E) and (F) in the proper sequence to form a
(c) fledged, protect (d) swing, cover
meaningful paragraph; then answer the questions given below
(e) roll, hide them.
Directions (Q. 101-110): In the following passage there are
(A) Basic human needs also include a sense of
blanks, each of which has been numbered. These numbers are belongingness, a feeling of control over one’s life.
printed below the passage and against each five words are
(B) Motivation and inspiration energise people into action.
suggested, one of which fits the blank appropriately. Find out the
(C) Ability to live up to one’s ideals besides all these is
appropriate words without changing the meaning of the passage.
also a fundamental need.
www.newspaperkorner.wordpress.com
Practice Set - 9 155
www.newspaperkorner.wordpress.com
(D) This is done not by pushing them in the right direction 119. Accounts are allowed to be operated by cheques in respect
as control mechanisms. of
(E) Such feelings touch us deeply and elicit powerful (a) Both Savings bank accounts and fixed deposit accounts
response. (b) Savings bank accounts and current accounts
(F) But it is done by satisfying basic human needs for (c) Both Savings bank accounts and loan accounts
achievement. (d) Both Savings bank accounts and cash accounts only
111. Which of the following should be the FIRST statement (e) Both Current accounts and fixed deposit accounts
after rearrangement? 120. Which of the following is correct statement?
(a) A (b) B (c) C (d) D (e) E (a) Normally no interest is paid on current deposit
112. Which of the following should be the SECOND statement accounts
after rearrangement? (b) Interest is paid on current accounts at the same rate as
term deposit accounts
(a) A (b) B (c) C (d) D (e) E
(c) The rate of interest on current account and savings
113. Which of the following should be the FOURTH statement
account are the same
after rearrangement?
(d) No interest is paid on any deposit by the bank
(a) A (b) B (c) C (d) D (e) E
(e) Savings deposits are the same as current deposits.
114. Which of the following should be the FIFTH statement
121. The usual deposit accounts of banks are
after rearrangement?
(a) Current accounts, electricity accounts and insurance
(a) A (b) B (c) C (d) D (e) E premium accounts
115. Which of the following should be the SIXTH (LAST) (b) Current accounts, Post office savings bank accounts
statement after rearrangement? and term deposit accounts
(a) A (b) B (c) C (d) D (e) E (c) Loan accounts, savings bank accounts and term
GENERAL/ECONOMY/BANKING deposit accounts
AWARENESS (d) Current accounts, savings bank accounts and term
deposit accounts
116. Which of the following statement is true? (e) Current bill accounts and term deposit accounts
(a) Banks cannot accept demand and time deposite from 122. Fixed deposits and recurring deposits are
public. (a) repayable after an agreed period.
(b) Banks can accept only demand deposits from public. (b) repayable on demand
(c) Banks can accept only time deposits from public. (c) not repayable
(d) Banks can accept both demand and time deposits from (d) repayable after death of depositors
public. (e) repayable on demand or after an agreed period as per
(e) Banks can accept demand and time deposits only from bank's choice.
government. 123. Which of the following is known as cross selling by Banks?
117. Interest payable on saving bank accounts is (A) Sale of a debit card to a credit card holder
(a) not regulated by RBI. (B) Sale of Insurance policy to a depositor
(b) regulated by State Governments (C) Issuance of Cash against Cheque presented by a third
party
(c) regulated by Central Government
(a) Only (A) (b) Only (B) (c) Only (C)
(d) regulated by RBI
(d) Both (A) and (C) (e) All (A), (B) and (C)
(e) regulated by Finance minister.
124. When a bank returns a cheque unpaid. It is called
118. Which of the following is the correct statement?
(a) payment of the cheque
(a) State bank of India is the sole authority to issue and
(b) drawing of the cheque
manage currency in India.
(c) cancelling of the cheque
(b) A nationalized bank is the sole authority to Issue and
(d) dishonour of the cheque
manage currency in India
(e) taking of the cheque.
(c) A cooperative bank is the sole authority to issue and
125. Distribution of Insurance products and Insurance policies
manage currency in India. by banks as corporate agents is known as
(d) RBI is the sole authority to issue and manage currency (a) General Insurance(b) Non-life Insurance
in India. (c) Bancassurance (d) Insurance banking
(e) None of these (e) Deposit Insurance

www.newspaperkorner.wordpress.com
156 Practice Set - 9
www.newspaperkorner.wordpress.com
126. What does the letter 'L' denote in term 'LAF' as referred 135. Which of the following organisations gives the Kalinga
every now and then in relation to monetary policy of the Prize?
RBI? (a) UNESCO
(a) Liquidity (b) Liability (c) Leveraged (b) CSIR
(d) Longitudinal (e) Linear (c) Ministry of Welfare
127. Interest on savings bank account is now calculated by banks (d) Department of Science and Technology
on (e) None of these
(a) minimum balance during the month 136. Which of the following taxes is collected by the Government
(b) minimum balance from 7th to last day of the month on sale of goods and services in the country?
(c) minimum balance from 10th to last day of the month (a) NAT (b) GET
(d) maximum balance during the month (c) SAT (d) VAT
(e) daily product basis (e) None of these
128. A centralized database with on line connectivity to branches, 137. Many a time we read a term “PPP” in financial newspapers.
internet as well as ATM network which has been adopted What is the full of the same as used in financial world?
by almost all major banks of our country is known as (a) Public Per Capita Power
(a) Investment banking (b) Per Capita Potential Purchases
(b) core banking (c) Purchasing Power Parity
(c) mobile banking (d) Present Purchasing Power
(d) national banking (e) None of these
(e) specialized banking 138. Fiscal Policy is related to
129. Malaysian Airlines flight that vanished from the radar (a) Issue of currency
screens over the South China Sea was coded as? (b) Credit creation
(a) MH 777-200ER (b) MH370 (c) Public revenue and expenditure
(c) MA377 (d) MA370 (d) All of these
(e) None of these (e) None of these
130. 1st Asian Grand Prix Archery Championships was held in? 139. The Doing Business Report, an international publicationi,
(a) Bangkok (b) China is published every year by which of the following
(c) Brazil (d) France organisation(s)?
(e) None of these (A) World Bank
131. The ‘World Heritage Day’ is observed on which of the (B) WTO
following dates? (C) Reserve Bank of India
(a) April 18 (b) May 18 (a) Only (A) (b) Only (B)
(c) June 18 (d) March 18 (c) Only (C) (d) Both (A) and (B)
(e) All (A), (B) and (C)
(e) None of these
140. Big Apple is the nickname for which city?
132. Which of the following in NOT a mode of foreign capital
(a) New York (b) Dallas
inflow to India?
(c) Portland (d) Las Angells
(a) FDI (b) FII
(e) None of these
(c) FRI Accounts (d) No Frills Accounts
141. Most of the public sector banks and housing finance
(e) All these are valid for foreign capital inflow. companies who are providing housing loans to individuals
133. The National Stock Exchange functions from get the refinance from....
(a) New Delhi (b) Kolkata (a) Reserve Bank of India
(c) Mumbai (d) Chennai (b) Ministry of Housing & Development
(e) None of these (c) Life Insurance Corportation of India
134. Many times we see banks advertise – “Anywhere Banking: (d) HUDCO
Anytime Banking”. Which of the following products/ (e) None of these
facilities launched by banks make it possible for the 142. The Foreign Exchange Reserves of India are kept in the
customers to avail banking services 24 hours all seven days?
custody of
(A) ATM
(a) Prime Minister Rahat Kosh
(B) Internet Banking
(b) International Monetary Fund
(C) Universal Cheque book facility
(c) World Bank
(a) Only (A) (b) Only (B)
(d) Reserve Bank of India
(c) Both (A) and (B) (d) Only (C)
(e) None of these
(e) All (A), (B) and (C) www.newspaperkorner.wordpress.com
Practice Set - 9 157
www.newspaperkorner.wordpress.com
143. The Reserve Bank of India keeps on changing various 149. Under which qualitative tool, RBI fixes maximum limit to
ratio rates frequently. Why is this done? loans and advances that can be made above which the
I. To keep inflation under control. commercial banks cannot exceed?
II. To ensure that Indian Rupees does not lose its market (a) Rationing of credit (b) Margin Requirement
value. (c) Loan to Value ratio (d) Moral suasion
III. To ensure that banks do not earn huge profits at the (e) None of these
cost of public money. 150. Which among the following is NOT a function of RBI?
(a) Only II (b) Only I (a) To work as monetary authority and implement its
Monetary Policy
(c) Only III (d) All I, II and III
(b) To serve as issuer of bank notes
(e) None of these
(c) Serve as banker to central and state governments
144. Inflation implies
(d) Serve as debt manager to other banks
(a) Rise in budget deficit (e) None of these
(b) Rise in general price index 151. Which of the following cannot be called as a Debt
(c) Rise in price of consumer goods Instrument as referred in financial transactions?
(d) Rise in money supply (a) Certificate of Deposits (b) Bonds
(e) None of these (c) Stocks (d) Commercial Papers
145. Which one of the following is not a constitutional body? (e) Loans
(a) Union Public Service Commission 152. What is Call Money ?
(b) Election Commission (a) Money borrowed or lent for a day or over night
(c) Planning Commission (b) Money borrowed for more than one day but upto 3
(d) Finance Commission days
(e) None of these (c) Money borrowed for more than one day but upto 7
146. If a country has a closed economy, it implies that this country days
(a) Does not allow private ownership of capital (d) Money borrowed for more than one day but upto 14
(b) Does not have flexible exchange rates days
(c) Does not have fixed exchange rates (e) None of these
153. As per the Union Budget 2016-17, target of amount
(d) Does not conduct trade with other countries
sanctioned under Pradhan Mantri Mudra Yojana has been
(e) None of these
increased to
147. If the Cash Reserve Ratio (CRR) is increased by the RBI, its
(a) 1,80,000 crore rupees (b) 1,90,000 crore rupees
impact on the expansion of credit creation will be to__:
(c) 1,70,000 crore rupees (d) 2,00,000 crore rupees
(a) decrease it (b) increase it (e) None of these
(c) no impact (d) Cant' say 154. India Aviation 2016 event has been hosted by which city of
(e) None of these India?
148. To combat raising inflation, RBI has to do which of the (a) Kolkata (b) Agra
following activities? (c) Hyderabad (d) Udaipur
(a) increase bank rate (e) None of these
(b) sell government securities 155. Who has won 2016 Abel Prize?
(c) increase reserve ratios (a) Endre Szemeredi (b) Andrew Wiles
(d) All of the above (c) John Rognes (d) Michael Atiyah
(e) None of these (e) None of these

www.newspaperkorner.wordpress.com
158 Practice Set - 9
www.newspaperkorner.wordpress.com

Time : 30 min. Max. Marks : 50

1. Write a letter on any one of the following topics: (20 marks)


(a) Write a letter to a Bank for stopping Payment on a Cheque.
(b) Write a letter to the Editor of news papers complaining about the irregular and inadequate bus services in your area.
(c) Write a letter requesting reimbursement for costs that you have already paid to repair or replace insured property
2. Write a paragraph about 150 words on one of the following topics: (10 marks)
(a) Relation between banking and inflation.
(b) Right to Education for the Right to Job.
(c) Mobile banking.
3. Write an essay in about 250 words on one of the following topics: (20 marks)
(a) Cooperative sector and its role in Indian Economy.
(b) Merits and demerits of commercial banking.
(c) Accountability in PSU.

www.newspaperkorner.wordpress.com
Practice Set - 9 159
www.newspaperkorner.wordpress.com
Answer Key
1 (c) 17 (d) 33 (d) 49 (b) 65 (b) 81 (c) 97 (d) 113 (a ) 129 (b) 145 (c)
2 (b) 18 (d) 34 (c) 50 (a ) 66 (b) 82 (a ) 98 (c) 114 (e) 130 (a ) 146 (d)
3 (c) 19 (c) 35 (b) 51 (d) 67 (d) 83 (e) 99 (e) 115 (c) 131 (a ) 147 (a )
4 (d) 20 (c) 36 (c) 52 (d) 68 (d) 84 (c) 100 (b) 116 (d) 132 (d) 148 (d)
5 (d) 21 (b) 37 (a ) 53 (c) 69 (c) 85 (d) 101 (b) 117 (d) 133 (c) 149 (a )
6 (a ) 22 (c) 38 (d) 54 (c) 70 (a ) 86 (a ) 102 (d) 118 (d) 134 (c) 150 (c)
7 (b) 23 (d) 39 (e) 55 (b) 71 (e) 87 (d) 103 (a ) 119 (b) 135 (a ) 151 (c)
8 (c) 24 (b) 40 (e) 56 (d) 72 (c) 88 (e) 104 (e) 120 (a ) 136 (d) 152 (a )
9 (d) 25 (e) 41 (e) 57 (a ) 73 (b) 89 (a ) 105 (c) 121 (d) 137 (c) 153 (a )
10 (c) 26 (b) 42 (a ) 58 (d) 74 (a ) 90 (c) 106 (a ) 122 (a ) 138 (c) 154 (c)
11 (c) 27 (e) 43 (d) 59 (b) 75 (b) 91 (e) 107 (e) 123 (e) 139 (a ) 155 (b)
12 (a ) 28 (e) 44 (a ) 60 (a ) 76 (d) 92 (d) 108 (a ) 124 (d) 140 (a )
13 (b) 29 (c) 45 (b) 61 (a ) 77 (e) 93 (c) 109 (d) 125 (c) 141 (c)
14 (c) 30 (a ) 46 (a ) 62 (c) 78 (c) 94 (b) 110 (c) 126 (e) 142 (d)
15 (d) 31 (d) 47 (d) 63 (d) 79 (d) 95 (b) 111 (b) 127 (c) 143 (d)
16 (c) 32 (a ) 48 (c) 64 (d) 80 (a ) 96 (b) 112 (d) 128 (b) 144 (d)

Answers & Explanations


1. (c) The question statement present a situation in which Step III: 29 about 24 9 13 tariff call even
number of species (people) are drawn to locale because Step IV: 29 about 24 call 9 13 tariff even
of its attribute (beauty) therefore the assumption in 3. (c) Step II: 37 desk 34 garden 5 father victory 17
the statement is that the crowds on the beach are there Step III: 37 desk 34 father garden 5 victory 17
to enjoy the beauty. Step IV: 37 desk 34 father 17 garden 5 victory Since all
Option (b) and (d) are easily eliminated. the elements of the input are fully arranged in Step IV,
Option (b) works at an entirely different logical this is the last step of the given input.
relationship – one between behavioural aspects – 4. (d) Step I: 59 bead tenure father 38 11 ultimate 24
where increase/decrease in scolding leads to increase/ Step II: 59 bead 38 tenure father 11 ultimate 24
decrease in misbehaviour. Step III:59 bead 38 father tenure I I ultimate 24
5. (d) Since it is a case of arrangement, we can’t obtain
Option (d) presents a totally different argument – where
previous steps with certainty.
increased efficiency leads to increase in free time.
6. (a) Input: 24 12 entry sand butter 51 32 carry
Option (a) is not similar- it speaks of two species and Step I: 51 24 12 entry sand butter 32 carry
the species share an attribute. The attribute of the locale Step II: 51 butter 24 12 entry sand 32 carry
is not the focus of the statement. Step III: 51 butter 32 24 12 entry sand carry
Option (c) is appropriate. Number of fleas- (the tiny 7. (b) This passage is a typical example of Response to
insects that live in the fur of animal's fur exposed to Stimuli. Clearly the cat starts salivating when the bell
warm weather). The assumption then is that the fleas rungs because she has related the ringing of bell with
are there (and proliferating) because of the warmth. food in the back of her mind.
This choice is the closest, in reasoning to the question (8-10):
statement. Group Captain – Malhotra
(2-6) : Here it is case of arrangement.The logic is: the words get Additional No. of crews = 4
arranged in alphabetical order. Whereas the numbers get arranged At least two are pilots.
in descending order. Numbers occupy odd places in the final
While others are engineers.
steps while words occupy even positions. When any element
Condidates for enginers : Lal, Monty, Naveen and
gets arranged the previous elements occupying that position
Paul. Candidates for pilots : Dalbir, Eric and Farid.
shifts one place towards right.
8. (c) As Naveen is chosen, Dalbir and paul cannot be
2. (b) Input: 9 13 about tariff 24 call 29 even
chosen. Therefore, the pilots chosen are Eric and
Step I: 29 9 13 about tariff 24 call even
Farid. Eric cannot be a crew with Lal. Hence, monty
Step II: 29 about 9 13 tariff 24 call even
will be chosen.
www.newspaperkorner.wordpress.com
160 Practice Set - 9
www.newspaperkorner.wordpress.com
9. (d) As it has been stated, Eric cannot be a crew with Lal. Besides this, we also came to know C is a female but
10. (c) Both (A) and (B) are true. still we do not know about the sex of G. But it is given
11. (c) From I, P is easier than Q. From II, R is easier than that there is at least one female in each car. The above
S. But what about relationships like that between P table shows that the car Q needs a female to fulfil the
and R? condition described above. Hence, the seventh person
12. (a) From I: A teaches History among A, B, C, D and E [The is a female, i.e., G is a female. But still we do not know
name of other four subjects is given in the statement among the cars Q and R, in which car C and G are. But
and A teaches none of them.] it is clear that both are not in the same car. Let us
From II: Either B or D teaches History. proceed to draw family tree.
13. (b) Desire to purchase a thing is not enough; it is money
that matters. E (+) Û F(-)
14. (c) Conclusion II follows from conversion of the
conclusion obtained from statement (b) and statement B(+) Û A (-)
(c) [ Q I + A = I]. Conclusion I, III and IV do not follow
because statement (a) + statement (b) gives no C(-) D ( -)
conclusion. But the conclusion I and IV make a
complementary pair IE-type. Hence, either of the two Since, C does not travel with her grandfather and
follows. grandmother, C is not in the car P and in Q. Hence, C is
15. (d) Conclusion IV follows from statement (b) and statement in the car R and G is in the car Q. Still we are not
(c). [Q I +A = 1]. Conclusion I follows from conversion aware of G's position in the family tree.
of conclusion IV. Statement (a) and statement (b) give 24. (b) Children are not being affected by malaria. This implies
no conclusion [ Q A + I = no conclusion ]. Therefore, they are already resistant to the disease. Why then
conclusion II and III do not follow. should we need the supplements?
16. (c) Statement (a) + statement (b) gives the conclusion “No 26. (b) As mentioned, ‘food was cooked and stored in the
fans are green” [say (d)] [Q A + E = E]. Now, conversion open space for almost 12 hours earlier’. Now, this may
of statement (d) gives conclusion IV. Now statement be the cause of food-poisoning.
(c) + conclusion IV gives the conclusion “Some 30. (a) If the given statement that CEO of small organization
windows are not fans”. Hence, I does not follow. come from good educational background is true it will
Conclusion III does not follow because conclusion IV strengthen the speaker’s argument, the reason being
follows. the speaker wants that CEOs of big and small
Again, statement (b) + conversion of statement (c) organizations should be paid similarly and if the CEO
gives the conclusion “Some windows are not rooms”. of small organization comes from good educational
Hence, conclusion II does not follow. background they should receive a similar amount which
17. (d) Only I, II and IV follow. Statement (a) + statement (c) a CEO of big organization receives.
gives conclusion IV [Q I + A = I]. Statement (b) + 31. (d) In the subsequent figures two and three designs are
statement (c) gives conclusion II [Q A + A = A]. Now,, inverted alternately. From problems Figure (1) to (2)
statement (b) + conversion of statement (a) gives no the first and the second designs from the top
conclusion. Hence, conclusion III does not follow. interchange positions and the lowermost design moves
Conversion of conclusion IV gives conclusion I. to the middle position. From problem Figure (2) and (3)
18. (d) Only II follows. Statement (a) + statement (b) gives no the lowermost design moves to the top and the second
conclusion [Q E + E = no conclusion]. Hence, I does design from the top moves to the lowermost position
not follow. Note that I does not follow from statement while the third and the fourth designs interchange
(c) either. Conclusion II follows from conversion of positions. From problem Figure (3) to (4) the top most
statement (b). Conclusion III does not follow from and the lowermost designs interchange positions and
statement (a). Conclusion IV does not follow from the fourth design moves to the second position. From
statement (c). Problem Figure (4) to (5) the topmost design moves to
(19-23): the fourth position, the lowermost design moves to
the topmost position while the second and the third
On the basis of the given clues we get the following
information: designs interchange positions. Therefore, from problem
Figure (5) to Answer Figure the third and the fourth
Car Persons travelling in car
designs would interchange positions and would be
P F (–), D (–)
inverted, the lowermost design would move to the top
Q B (+), E (+)
position and the second design from the top would
R A (–), ? (–)
move to the lowermost position.
www.newspaperkorner.wordpress.com
Practice Set - 9 161
www.newspaperkorner.wordpress.com
32. (a) The following changes occur in the subsequent 35. (b) From problem Figure (1) to (2) the plane of designs
figures: rotates through 90° clockwise and the second and the
(1) to (2) (2) to (3) fifth designs interchange positions. Similarly, the third
and the fourth designs interchange positions. Similar
changes occur from Problem Figure (3) to (4) and from
problem figure (5) to Answer Figure.
46. (a) Female players who plays lawn tennis

22
= 2000 × = 440
100

(3) to (4) (4) to (5) Male players who plays rugby

13 10
= 4200 × - 2000 ´
100 100

= 546 – 200 = 346

Difference = 440 – 346 = 94

47. (d) Average number of players who play football and rugby
(5) to (6) together

17 + 13
4200 ´
= 100 = 630
2
48. (c) Female players who plays cricket
40
= 2000 ´ = 800
100
33. (d) From problem Figure (1) to (2) and from problem Figure Male players who plays hockey
(5) to Answer Figure the smaller square with four
10 15
designs moves two steps in anticlockwise direction 4200 ´ - 2000 ´
100 100
and the designs in each row interchange positions.
The other three designs move two steps in clockwise = 420 – 300 = 120
direction. 800
34. (c) The following changes occur in the subsequent Ratio = = 20 : 3
120
figures: 49. (b) Number of male players who plays football, cricket
(1) to (2) (2) to (3) and lawn tennis
(5) to (6)
17 + 35 + 25 13 + 40 + 22
= 4200 × - 2000 ´
100 100
N = 3234 – 1500 = 1734
50. (a) Male players who plays rugby
13 10
= 4200 × - 2000 ´
N 100 100
= 546 – 200 = 346
(3) to (4) (4) to (5) Total number of players play lawn tennis
25
= 4200 ´ = 1050
100
N Required percentage
346
= ´ 100 » 33%
1050
N
www.newspaperkorner.wordpress.com
162 Practice Set - 9
www.newspaperkorner.wordpress.com
51. (d) Total expenditure 60. (a) Amount raised by rights issue in 2011 – 12 = ` 13658
= 306 + 300 + 300 + 306 + 300 + 306 + 306 Amount raised by right issue in 2012 – 13 = ` 7598
= Rs 2124 thousands
52. (d) Total expenditure in the year 2014 13658 - 7578
Percentage decrease = ´ 100 = 44.36%
= 300 + 306 + 306 + 312 + 318 + 300 + 312 13658
= Rs 2154 thousands 61. (a) Sales of Voveran in 2015 = 23 crore
Total expenditure in 2013 and 2014
Sales of Calpol in 2014 = 13 crore
= 2124 + 2154 = 4278 thousands
\ Required difference = 23 – 13 = 10 crore = 100 lakh
53. (c) Average monthly expenditure in year 2015
62. (c) Percentage increase in sales from 2014 to 2015 :
312 + 312 + 318 + 318 + 306 + 312 + 318
=
7 23 - 17
Voveran = ´ 100 = 35.29%
17
2196
= = 313.714 thousands » ` 3,14,000
7 9-7
Voline = ´ 100 = 22.22%
54. (c) In the year 2015, the least expenses of ` 306 thousands 9
is in the month of May.
10 - 7
55. (b) Expenditure in April 2014 = 312 thousands Dolonex = ´ 100 = 42.85% (highest)
Expenditure in April 2015 = 318 thousands 7

6 7-5
Required % = ´ 100 = 1.92 » 2% Sumo = ´ 100 = 40%
312 5
56. (d) The amount of private placement raised as a percentage 63. (d) Percentage increase in sales from 2014 to 2015 of :
of total fund in Voveran = 35.29%
7568 Volini = 22.22% (lowest)
2010 - 11 = ´ 100 = 17.8%
7568 + 14738 + 20170
5-3
Moov = ´ 100 = 66.66%
11715 3
2011 - 12 = ´ 100 = 22.6%
11715 + 13658 + 26430
19 - 15
Nise = ´ 100 = 26.66%
7572 15
2013 - 14 = ´100 = 32.4%
7572 + 2755 + 13010
64. (d) The number of non-US students in the Arts faculty
6452 = 79 + 21 + 6 + 2 + 4 = 112
2014 - 15 = ´100 = 57.2%
6452 + 2575 + 2244 \ The required percentage
Hence percentage raise was maximum in 2014-15
57. (a) Total amount of funds raised 112
= ´ 100 = 10.67% » 11%
2010 – 11 = 7568 + 14738 + 20170 = ` 42476 crore 1049
2011 – 12 = 11715 + 13658 + 26430 = ` 51803 crore 65. (b) Let the number of total students in the University be x.
2013 – 14 = 7572 + 2755 + 13010 = ` 23337 crore
Students in the Arts faculty, which represents 23% of
2014 – 15 = 6452 + 2244 + 2575 = ` 11271 crore
the total students
Hence total number of funds raised was the least in
year 2014 – 15. = 1049 = 0.23x
\ The total number of Engineering students
20170 + 18375 + 13010 + 2575
58. (d) Required average =
4 9 1049
= ´ » 410 students
= ` 13532.5 crore 100 0.23
59. (b) Amount raised by public issue 66. (b) Let the total number of students in the University be x
= 20170 + 26430 + 18375 + 13010 + 2575 = ` 80560 \ 1049 = 0.23x
Total amount raised
= 80560 + 7568 + 11715 + 6624 + 7572 + 6452 + 1473 + 1049 ´100
\ x= = 4560
13658 + 7598 + 2755 + 2244 = ` 161484 crore 23
80560 \ The total number of students is 4560
\ Required percentage = ´ 100 = 49.88%
161484
www.newspaperkorner.wordpress.com
Practice Set - 9 163
www.newspaperkorner.wordpress.com
67. (d) The number of Asian students studying Science
é 2 ù
= 22500 êæç1 +
12 ö
6 21 ÷ - 1ú
= ´ ´ 4560 » 57 êëè 100 ø úû
100 100
5
68. (d) The number of medical students = ´ 4560 = 228
100
Out of these, 34 are from Europe. éæ 3 ö2 ù
= 22500 ê ç1 + ÷ - 1ú
ê è 25 ø ú
34 ë û
This represents ´ 100 = 14.9% » 15% of the
228
faculty. éæ 28 ö 2 ù æ 784 ö
= 22500 êç ÷ - 1ú = 22500 ç -1
69. (c) Total demand è
ê 25 ø ú è 625 ÷ø
ë û
= 3000 + 600 + 2500 + 1200 + 3300 = 10600 units.
Average demand
159
= 22500 ´ = ` 5724
Total demand 10600 625
= = = 2120 units.
Number of T.V. companies 5
74. (a) Value of one ticket of each kind = 55 + 85 + 105 = ` 245
Total production
\ Required number of ticket of each kind
= 1500 + 1800 + 1000 + 2700 + 2200 = 9200 units.
Average production
2940
= = 12
Total production 9200 245
= = = 1840 units
Number of T.V. companies 5
75. (b) Ravina’s monthly income
Difference between the average demand and the
average production = 2120 – 1840 = 280 100 + 15 115
= 32000 ´ = 32000 ´ = ` 36800
Production of company D 2700
100 100
70. (a) = = 1.8
Production of company A 1500 = Ramola’s annual income = 36800 ´ 3 ´ 12
71. (e) Correct Average = ` 1324800

∋24´56( ∗ ∋48 ∗ 59 ∗ 67( ,∋44 ∗ 45 ∗ 61( 56


= 76. (d) Marks scored by Ritu = 875 ´ = 490
24 100

1344 ∗ 174 ,150 1368 92


< < < 57 Marks scored by Smita = 875 ´ = 805
24 24 100
72. (c) Suppose the maximum mark of the test be x. \ Average marks scored by all the three together

490 ∗ 805 ∗ 634 1929


Then, 468 ´100 < 336 ´100 = = = 643
x 700 3 3
77. (e) According to the question
468´100´ 700
\ x= < 975 Present age of Parineeta = 33 – 9 = 24 years
336 ´100
Present age of Manisha = 24 – 9 = 15 years
Interest ×100 Present age of Deepali = 24 + 15 = 39 years
73. (b) Rate of simple interest = Q 5 : X = 15 : 39
Principal × time
5´ 39
10800 ´100 \ X= < 13
= < 12% 15
22500 ´ 4
78. (c) Cost of one pencil box = 7 + 22 +14 = ` 43

é time ù \ Required amount = (20 ´ 7) + (8 ´ 22) + (6 ´ 175) + (7 ´ 43)


Compound interest = Principal êæç1+
rate ö
÷ – 1ú = 140 + 176 + 1050 + 301= ` 1667
êëè 100 ø úû
www.newspaperkorner.wordpress.com
164 Practice Set - 9
www.newspaperkorner.wordpress.com 83. (e) None of these is fully true.
14 14
79. (d) Total time required = + 84. (c) There has been much debate about subsidies and their
5 10
utility; the opposing view being that subsidies are
28 + 14 against the market reforms and distort the market as
= = 4.2 hrs
10 well as reduce resource efficiency.
80. (a) The numbers are 50, 51, 52 and 53 86. (a) China has concentrated on investment rather than on
\ b × c = 51 × 52 = 2652 subsidies.
81. (c) The widest divergence between India and China is the 92. (d) Substitute very for so.
profitable horticultural sector ..... China’s added 93. (c) Substitute employees.
advantage lies in the more diversified composition of 94. (b) Substitute any for no.
its agricultural sector. 95. (b) Substitute engineer’s.
82. (a) In all of these China has outdone India. (111-115) : BDFAEC

www.newspaperkorner.wordpress.com
www.newspaperkorner.wordpress.com

Practice Set - 10

INSTRUCTIONS
• This practice set consists two parts. One part is Objective test and other part is Descriptive test.

• Each question has five options, of which only one is correct. The candidates are advised to read all the
options thoroughly.

• There is negative marking equivalent to 1/4th of the mark allotted to the specific question for wrong
answer.

Time : 3 hrs. Max. Marks : 200

REASONING AND COMPUTER APTITUDE Direction (Qs. 6-8) : A word arrangement machine, when given a
particular input, rearranges it following a particular rule. The
Directions (Qs. 1-5) : In each question below are two/three following is the illustration of the input and the steps of arrangement.
statements followed by two conclusions numbered I and II. You Input: pull the cover and then push into
have to take the two/three given statements to be true even if Step I: pull the then and cover push into
they seem to be at variance from commonly known facts and then Step II: then the pull into push cover and
decide which of the given conclusions logically follows from the Step III: into pull the then and cover push
given statements disregarding commonly known facts. Step IV: into pull and then the cover push and so on.
Give answer (a) if only conclusion I follows 6. Input: Try your best until you get goal
Give answer (b) if only conclusion II follows. Which of the following steps would be ‘get goal try until
you your best’?
Give answer (c) if either conclusion I or conclusion II follows. (a) Step II (b) Step III
Give answer (d) if neither conclusion I nor conclusion II follows. (c) Step IV (d) Step V
Give answer (e) if both conclusion I and conclusion II follow. (e) None of these
(Qs. 1-5) : 7. If Step VI of an input is
Statements : All gliders are parachutes. ‘deep gutter ball into the has fallen’
which of the following would definitely be the input?
No parachute is an airplane. (a) has the ball fallen into deep gutter
All airplanes are helicopters. (b) ball has fallen into the deep gutter
1. Conclusions : I. No glider is an airplane. (c) deep gutter has fallen into the ball
II.All gliders being helicopters is a (d) gutter has deep ball fallen into the
possibility. (e) None of these
2. Conclusions : I. No helicopter is a glider. 8. If Step IV of an input is
II. All parachutes being helicopters is a ‘we can’t measure the depth without scale’,
possibility. what would be step VII?
3. Statements : Some mails are chats. (a) scale we the measure can’t depth without
All updates are chats. (b) the we scale without depth can’t measure
Conclusions : I. All mails being updates is a possibility. (c) without we scale the can’t measure depth
II. No update is a mail. (d) the we depth without scale can’t measure
(e) None of these
(Qs. 4-5) :
Directions (Q. 9) : In each question below is given a statement
Statement : No stone is metal. followed by three courses of action numbered (A), (B) and (C). A
Some metals are papers. course of action is a step or administrative decision to be taken
All papers are glass. for improvement, follow-up or further action in regard to the
4. Conclusions : I. All stones being glass is a possibility. problem, policy, etc. On the basis of the information given in the
II. No stone is a paper. statement, you have to assume everything in the statement to be
5. Conclusions : I. No glass is a metal. true, then decide which of the suggested courses of action
II. Atleast some glass is metal. logically follow(s) for pursuing.
www.newspaperkorner.wordpress.com
166 Practice Set - 10
www.newspaperkorner.wordpress.com
9. Statement: A heavy unseasonal downpour during the last 12. Which of the statements numbered (A), (B), (C), (D), (E)
two days has paralysed the normal life in the state in which and (F) can be assumed from the facts/ information given in
five persons were killed but this has provided a huge relief the statement ? (An assumption is something supposed or
to the problem of acute water crisis in the state. taken for granted)
Courses of action: (a) Only (A) (b) Only (B)
(A) The state government should set up a committee to (c) Both (B) and (C) (d) Both (D) and (E)
review the alarming situation. (e) Only (F)
(B) The state government should immediately remove all 13. Which of the statements numbered (A), (B), (C), (E) and (F)
the restrictions on use of potable water in all the major represents a disadvantage of the small grocery stores over
cities in the state. the Supermarkets from the perspective of a smaller brand ?
(C) The state government should send relief supplies to (a) Only (A) (b) Only (C)
all the affected areas in the state. (c) Only (E) (d) Only (F)
(a) None (b) Only (A) (e) Both (B) and (C)
(c) Only (B) and (C) (d) Only (C) 14. Which of the statements (A), (B), (C), (D) and (E) mentioned
(e) All (A), (B) and (C) above represents a reason for the shift from local grocery
10. Farmers found using chemical fertilizers in the organic- stores to supermarkets by the smaller brands ?
(a) Only(A) (b) Only (B)
farming area of their farms would be heavily fined.
(c) Only (D) (d) Both (A) and (D)
Which of the following statements is an assumption implicit (e) Both (C) and (E)
in the given statement ? (An assumption is something 15. Which of the statements numbered (A), (B), (C), (E) and (F)
supposed or taken for granted.) mentioned above would prove that the step taken by the
(a) Chemical fertilisers harm the crop. smaller brands (of moving to supermarkets) may not
(b) A farm's area for organic and chemical farming is necessarily be correct ?
different. (a) Only (A) (b) Only (C)
(c) Farmers who do not use chemical fertilizers in the (c) Only (E) (d) Only (F)
chemical farming area would be penalized as well. (e) Both (B) and (E)
(d) All farmers undertake both these kinds of farming 16. A very large number of technically qualified young Indians
(chemical as well as organic) in their farms. are coming out of colleges every year, though there are not
(e) Organic fertilizers are banned in the area for chemical enough opportunities for them to get gainful employment.
farming. Which of the following contradicts the views expressed in
11. Small brands are now looking beyond local grocery stores the above statement?
and are tying up with Supermarkets such as Big Bazaar to (a) Technically qualified persons are far superior to those
pull their business out of troubled waters. with standard degrees like B.A./B.Sc./B.Com. etc.
Which of the following can be inferred from the given (b) The Govt. has not done effective -perspective planning
information ? (An inference is something that is not directly for engaging technically qualified personnel while
stated but can be inferred from the given information) authorising the setting up of technical colleges.
(a) Merchandise of smaller brands would not be available (c) A huge gap exists between the level of competence of
at local grocery stores in the near future. technically qualified graduates and requirement of the
(b) Smaller brands cannot compete with bigger ones in a industry.
supermarket set-up. (d) Majority of the technically qualified persons are
(c) There is a perception among small brands that sale in a migrating from India to developed countries for better
supermarket is higher than that of small grocery stores. opportunities.
(d) Supermarkets generate more revenue by selling (e) None of these
products of bigger brands as compared to the smaller 17. The govt. has appealed to all citizens to use potable water
ones. judiciously as there is an acute shortage in supply. Excessive
(e) Smaller brands have always had more tie-ups with use may lead to huge scarcity in future months.
supermarkets as compared to small grocery stores. Which of the assumptions is implicit in the above statement?
Directions (Qs. 12-15) : These questions are based on the (An assumption is something supposed or taken for
information given above and the sentences labeled (A), (B), (C), granted)
(D), (E) and (F) as given below. (a) People may ignore the appeal and continue using water
as per their consideration.
(A) A smaller brand manufacturing a certain product of quality (b) Govt. may be able to tap those who do not respond to
comparable with that of a bigger brand. makes much more the appeal.
profit from the local grocery stores than from the supermarkets. (c) Govt. may be able to put in place alternate sources of
(B) As the supermarkets have been set up only in bigger cities at water in the event of a crisis situation.
present, this step would fail to deliver results in the smaller cities. (d) Large number of people may positively respond to the
(C) Supermarkets help the smaller brands to break into newer govt.'s appeal and help tide over the crisis.
markets without investing substantially in distribution. (e) Only poor are going to suffer from this shortage of
(D) Supermarkets charge the smaller brands 10% higher than water supply.
the amount charged to the bigger brands. 18. A few travellers were severely beaten up by villagers
(E) Being outnumbered by the bigger brands, visibility of the recently in a remote rural part of the state as the villagers
smaller brands at local grocery stores is much lower as found the movement of the travellers suspicious. The
compared to the supermarkets. district authority has sent a police team to nab the culprits.
(F) Smaller brands are currently making substantial losses in
Which of the following inferences can be drawn from the
their businesses.
above statement?
www.newspaperkorner.wordpress.com
Practice Set - 10 167
www.newspaperkorner.wordpress.com
(An inference is something which is not directly stated but 25. Statement : L = M > N; M > P; L < K
can be inferred from the given facts) Conclusions :
(a) The villagers dislike presence of strangers in their vicinity. I. K > P
(b) Villagers are generally suspicious in nature. II. N < K
(c) Travellers prefer to visit countryside. Directions (Qs. 26-30) : Study the following information
(d) The Govt. generally provides protection to travellers carefully to answer the given questions :
across the country.
Eight persons from different banks viz. UCO bank, Syndicate
(e) None of these bank, Canara bank, PNB, Dena Bank, Oriental Bank of Commerce,
19. There has been a considerable drop in sales of four wheelers Indian bank and Bank of Maharashtra are sitting in two parallel rows
during the past six months when compared to the number containing four people each, in such a way that there is an equal
of four wheelers sold during this period last year. distance between adjacent persons. In row-1 A, B, C and D are seated
Which of the following can the probable cause of the above and all of them are facing south. In row-2 P, Q, R and S are seated and
phenomenon? all of them are facing north. Therefore, in the given seating
(A) The govt. has imposed higher excise duty on four arrangement each member seated in a row faces another member of
wheelers at the beginning of this year. the other row. (All the information given above does not necessarily
(B) The petrol prices have risen considerably during the represent the order of seating as in the final arrangement)
past eight months. • C sits second to right of the person from Bank of
(C) The rate of interest on home and car loans have been Maharashtra. R is an immediate neighbour of the person
rising for the past seven months. who faces the person from Bank of Maharashtra.
(a) All (A), (B) and (C)(b) (A) and (C) Only • Only one person sits between R and the person for
(c) (B) and (C) Only (d) (B) Only PNB. Immediate neighbour of the person from PNB
(e) (A) Only faces the person from Canara Bank.
20. There is an alarming trend of skewed sex ratio against • The person from UCO bank faces the person from
women in India during the past decade and situation may Oriental Bank of Commerce. R is not from Oriental Bank
get out of hand if adequate steps are not taken to stop of Commerce. P is not from PNB. P does not face the
female foeticide. person from Bank of Maharashtra.
Which of the following can be an effective step to reverse • Q faces the person from Dena bank. The one who faces
the trend? S sits to the immediate left of A.
(A) The Govt. should immediately completely ban the use • B does not sit at any of the extreme ends of the line.
of scanners / sonography on expectant mothers at all The person from Bank of Maharashtra does not face
health centres. the person from Syndicate bank.
26. Which of the following is true regarding A ?
(B) The Govt. should announce a substantial incentive
(a) The person from UCO bank faces A
scheme for couples who have at least one girl child.
(b) The person from Bank of Maharashtra is an immediate
(C) The Govt. should launch a nationwide campaign to neighbour of A
create awareness against female foeticide. (c) A faces the person who sits second to right of R
(a) (A) only (b) (A) and (B) Only (d) A is from Oriental Bank of Commerce
(c) (B) and (C) Only (d) All (A), (B) and (C) (e) A sits at one of the extreme ends of the line
(e) None of these 27. Who is seated between R and the person from PNB ?
Directions (Qs. 21-25) : In these questions, relationship between (a) The person from Oriental Bank of Commerce
different elements is shown in the statements. The statements (b) P
are followed by conclusions. (c) Q
(d) The person from Syndicate bank
Give answer (a) if only Conclusion I is true
(e) S
Give answer (b) if only Conclusion II is true
Give answer (c) if either Conclusion I or II is true 28. Who amongst the following sit at extreme ends of the rows?
(a) D and the person from PNB.
Give answer (d) if neither Conclusion I nor II is true
Give answer (e) if both Conclusions I and II are true. (b) The person from Indian bank and UCO bank.
(c) The person from Dena bank and P.
21. Statement : F > G = H; G > J > K
(d) The persons from Syndicate bank and D.
Conclusions :
(e) C, Q
I. F > K
29. Who amongst the following faces the person from Bank of
II. K < H
Maharashtra ?
22. Statement : P < Q = R > S > T
(a) The person from Indian bank
Conclusions :
(b) P
I. T < Q
(c) R
II. R > P (d) The person from Syndicate bank
23. Statement : A < B < C; A > D; C < F (e) The person from Canara bank
Conclusions : 30. P is related to Dena bank in the same way as B is related to
I. D < C PNB based on the given arrangement. To who amongst the
II. F > D following is D related to, following the same pattern ?
24. Statement : U > A = I < O < E (a) Syndicate bank (b) Canara bank
Conclusions : (c) Bank of Maharashtra (d) Indian bank
I. I < E (e) Oriental Bank of Commerce
II. O > U
www.newspaperkorner.wordpress.com
168 Practice Set - 10
www.newspaperkorner.wordpress.com Answer Figures
Directions (Qs. 31-35) : In each of the questions given below
which one of the five answer figures should come after the
problem figures, if the sequence were continued ?

D
D

D
D
31. Problem Figures
(a) (b) (c) (d) (e)
36. When entering text within a document, the Enter key is
normally pressed at the end of every __________.
(a) Line (b) Sentence
(c) Paragraph (d) word
Answer Figures (e) file
37. When a real-time telephone call between people is made
over the Internet using computers, it is called __________.
(a) a chat session (b) an e-mail
(c) an instant message (d) Internet telephony
(e) None of these.
(a) (b) (c) (d) (e) 38. Which of the following is the first step in sizing a window ?
(a) Point to the title bar
32. Problem Figures (b) Pull down the View menu to display the toolbar
(c) Point lo any corner or border
(d) Pull down the View menu and change to large icons
(e) None of these
39. Which of the following software could assist someone who
cannot use their hands for computer input ?
Answer Figures (a) Video conferencing (b) Speech recognition
(c) Audio digitizer (d) Synthesizer
(e) None of these
40. ___________ a document means the file is transferred from
another computer to your computer.
(a) Uploading
(a) (b) (c) (d) (e) (b) Really Simple Syndication (RSS)
(c) Accessing
33. Problem Figures (d) Downloading
(e) Upgrading
41. Which computer memory is used for storing programs and
data currently being processed by the CPU ?
(a) Mass memory (b) Internal memory
(c) Non-volatile memory (d) PROM
Answer Figures (e) None of these
42. Computers that control processes accept data in a
continuous __________.
(a) data traffic pattern (b) data highway
(c) infinite loop (d) feedback loop
(e) slot
(a) (b) (c) (d) (e) 43. What refers to a set of characters of a particular design ?
34. Problem Figures (a) keyface (b) formation
(c) calligraph (d) stencil
(e) typeface
44. _________ is used by public and private enterprises to
publish and share financial information with each other and
industry analysts across all computer platforms and the
Internet.
Answer Figures
(a) Extensible Markup Language (EML)
(b) Extensible Business Reporting Language (XBRL)
(c) Enterprise Application Integration (EAI)
(d) Sales Force Automation (SFA) software
(e) None of these
45. The method of Internet access that requires a phone line,
(a) (b) (c) (d) (e) but offers faster access speeds than dial-up is the _____
35. Problem Figures connection.
(a) cable access (b) satellite access
D

(c) fiber-optic service


D

(d) Digital Subscriber Line (DSL)


D
D

(e) modem
www.newspaperkorner.wordpress.com
Practice Set - 10 169
www.newspaperkorner.wordpress.com
Directions (Qs. 46-50) : Study the pie-chart carefully to Appeared candidates = 45,000.
answer the following questions.
Percentage of Students Studying A
G
Different Specialisations in a Management Institute 15%
22%
Total number of students = 4500
B
11%
Finance
11% HR
22% F C
IT 18% 8%
21%
Marrketing
16% E D
Banking 9% 17%
12%
Qualified candidates = 9000.

Ratio of men to women in each specialisation G A


13% 18%
Specialisation Men Women
HR 2 3 F
Marketing 7 5 11%
International Business 5 1 B
Banking 17 10 16%
E
IT 1 2 14%
Finance 5 4 C
D 7%
46. What is the total number of men studying in the institute?
(a) 1665 (b) 2421 21%
(c) 2025 (d) 1954 51. What is the ratio of the number of appeared candidates
(e) None of these from States C and E together to that of the appeared
47. The number of women studying Marketing is candidates from States A and F together ?
approximately what per cent of the total number of students (a) 17 : 33 (b) 11 : 13
in the institute? (c) 13 : 27 (d) 17 : 27
(a) 5 (b) 12 (e) None of these
(c) 15 (d) 7 52. In which state, the percentage of qualified candidates with
(e) None of these respect to that of appeared candidates is minimum ?
48. What is the total number of men studying Banking and (a) C (b) F
International Business? (c) D (d) G
(a) 1125 (b) 297 (e) None of these
(c) 1015 (d) 594 53. What is the difference between the number of qualified
(e) None of these candidates of States D and those of G ?
(a) 690 (b) 670
49. The number of women studying Finance is what per cent
(c) 780 (d) 720
of the number of men studying the same?
(e) None of these
(a) 80 (b) 65
54. What is the percentage of qualified candidates with respect
(c) 95 (d) 70 to appeared candidates from States B and C taken
(e) None of these together? (rounded to two decimal places)
50. In which specialisation do the maximum number of women (a) 23.11 (b) 24.21
study? (c) 21.24 (d) 23
(a) HR (b) IT (e) None of these
(c) Marketing (d) Finance 55. What is the ratio between the number of candidates
(e) None of these qualified from Staes B and D together to the number of
Directions (Qs. 51-55) : These questions are based on the candidates appeared from State ‘C’ respectively ?
following pie charts. (a) 8 : 37 (b) 11 : 12
Classification of appeared candidates in a competitive test from (c) 37 : 40 (d) 7 : 37
different stats and qualified candidates from those states. (e) None of these
www.newspaperkorner.wordpress.com
170 Practice Set - 10
www.newspaperkorner.wordpress.com
Directions (Qs. 56-60): Study the following graph carefully and 61. What is the average number of children pursuing painting
answer the questions given below it. classes from all the States together ?
Marks obtained by two students in six subjects in an examination (a) 19735 (b) 17395
(c) 19725 (d) 18385
Maximum Marks
Physics Chemistry English History Geography Hindi (e) None of these
200 200 200 150 150 150 62. The children pursuing singing classes in State A are what
percent of the total number of children pursuing all the
200 three classes from the same State ?
(a) 30 (b) 35 (c) 40 (d) 45
150 (e) None of these
63. What is the difference between the total number of children
100 pursuing dancing and the total number of children pursuing
painting form all the States together?
50 (a) 20000 (b) 15000 (c) 25000 (d) 10000
(e) None of these
0 64. What is the respective ratio of the number of children
sh s y
g1li 2raphy 3ysic 4 tory 5 indi 6istr pursuing dancing from State B to the number of children
En eog Ph His H
C hem
G pursuing dancing from State D?
(a) 7 : 9 (b) 11 : 9 (c) 11 : 7 (d) 9 : 11
Student A Student B
(e) None of these
65. What is the average number of children pursuing singing,
56. Marks obtained by student B in Hindi was what percentage dancing and painting from State C ?
of the marks obtained by student B in physics ? (Rounded off to the nearest integer)
(a) 25 (b) 150 (c) 125 (d) 115.38 (a) 28859 (b) 29167
(e) None of these (c) 291216 (d) 28765
57. Approximately what was the average percentage of makrs (e) None of these
obtained by A in all the subjects ?
(a) 70 (b) 60 (c) 80 (d) 85 Directions (Qs. 66-70) : These questions are based on the
information and two graphs given below.
(e) None of these
58. Approximately what was the average marks obtained by B Figure 1 shows the amount of work distribution , in man -
in Geography, History and Hindi ? hours for a software company between offshore and onsite
(a) 120 (b) 80 (c) 140 (d) 110 activities. Figure 2 shows the estimated and actual work effort
(e) None of these involved in the different offshore activities in the same company
during the same period. [Note : onsite refers to work performed at
59. In how many subjects did student B obtain more then 70
the customer’s premise and offshore refers to work performed at
percentage of makrs ?
the developer’s premise.]
(a) 1 (b) 2 (c) 3 (d) 4
(e) None of these
Offshore
60. What was the difference in percentage of marks between A
500 Onsite
and B in History ?
(a) 30 (b) 25 (c) 40 (d) 20 400
(e) None of these 300
Directions (Qs. 61-65) : Study the following graph and answer 200
the questions given below it :
100
Number of Children pursuing various Hobby classes 0
in various States (Number in thousands) Design Coding Testing
Figure 1

Estimated
500 Actual
400
300
200
100
0
Design Coding Testing
Figure 2
www.newspaperkorner.wordpress.com
Practice Set - 10 171
www.newspaperkorner.wordpress.com
66. Which of the work requires as many man- hours as that 75. A 180 -metre long train crosses another 270-metre long train
spent in coding? running in the opposite direction in 10.8 seconds. If the
(a) Offshore, design and coding speed of the first train is 60 kmph, what is the speed of the
(b) Offshore coding. second train in kmph?
(c) Testing (a) 80 (b) 90
(d) Offshore, testing and coding. (c) 150 (d) Cannot be determined
(e) None of these (e) None of these
67. Roughly what percentage of the total work is carried out 76. Three digits numbers are formed using the digits 0, 2, 4, 6, 8.
onsite? A number is chosen at random out of these numbers. What
(a) 40 percent (b) 20 percent is the probability that this number has the same digits?
(c) 30 percent (d) 50 percent
(e) None of these 1 16
(a) (b)
68. The total effort in man- hours spent onsite is nearest to 16 25
which of the following? 1 1
(a) The sum of the estimated and actual effort for offshore (c) (d)
design 645 25
(b) The estimated man-hours of offshore coding (e) None of these
(c) The actual man-hours of offshore testing 77. Two person Ravi and Shyam can do a work in 60 days and
(d) Half of the man-hours of estimated offshore coding 40 days respectively. They began the work together but
(e) None of these Ravi left after some time and Shyam finished the remaining
69. If the total working hours were 100 which of the following work in 10 days. After how many days did Ravi leave?
tasks will account for approximately 50 hours? (a) 8 days (b) 12 days
(a) Coding (c) 15 days (d) 18 days
(b) Design (e) 20 days
(c) Offshore testing 78. The ratio of A’s age to B’s is 6 : 7. The product of their ages
(d) Offshore testing plus design is 672. What is the ratio of their ages after 6 years?
(e) None of these (a) 6 : 7 (b) 5 : 7
70. If 50 percent of the offshore work were to be carried out (c) 7 : 8 (d) 8 : 9
onsite, with the distribution of effort between the tasks (e) None of these
remaining the same, the proportion of testing carried out 79. An amount of money is to be divided among P, Q and R in
offshore would be the ratio of 3 : 5 : 7 respectively. If the amount received by R
(a) 40 Percent (b) 30 Percent is ` 4,000 more than the amount received by O, what will be
(c) 50 Percent (d) 70 Percent the total amount received by P and Q together?
(e) None of these (a) ` 8,000 (b) ` 12,000
71. Prakash, Sunil and Anil started a business jointly investing (c) ` 16,000 (d) Cannot be determined
`11 lakhs, ` 16.5 lakhs and ` 8.25 lakhs respectively. The (e) None of these
profit earned by them in the business at the end of three 80. Arun invested a sum of money at a certain rate of simple
years was ` 19.5 lakhs. What will be the 50% of Anil’s share interest for a period of four years. Had he invested the same
in the profit? sum for a period of six years, the total interest earned by him
(a) `4.5 lakhs (b) `2.25 lakhs would have been fifty per cent more than the earlier interest
(c) `2.5 lakhs (d) `3.75 lakhs amount. What was the rate of interest per cent per annum?
(a) 4 (b) 8
(e) None of these
(c) 5 (d) Cannot be determined
72. Naresh purchased a TV set for `11,250 after getting discount
(e) None of these
of 10% on the labelled price. He spent `150 on transport
and `800 on installation. At what price should it be sold so ENGLISH LANGUAGE
that the profit earned would be 15% if no discount was
offered? Direcctions. (Qs. 81-90) : Read the following passage carefully
(a) `12,937.50 (b) `14,030 and answer the questions given below it. Certain words in the
(c) ` 13,450 (d) `15,467.50 passage have been printed in bold to help you locate them when
(e) None of these answering some of the questions.
73. Mr Duggal invested `20,000 with rate of interest @ 20 pcpa. “Rivers should link, not divide us,” said the Indian Prime
The interest was compounded half-yearly for the first one Minister expressing concern over interstate disputes and urged
year and in the next year it was compounded yearly. What state governments to show “understanding and consideration,
will be the total interest earned at the end of two years? statesmanship and an appreciation of the other point of view.”
(a) ` 8,800 (b) ` 9,040 Water conflicts in India now reach every level; divide every
(c) ` 8,040 (d) ` 9,800 segment of our society, political parties, states, regions and sub-
(e) None of these regions within states, districts, castes and groups and individual
74. In how many different ways can the letters of the word farmers. Water conflicts within and between many developing
DESIGN be arranged so that the vowels are at the two ends? countries are also taking a serious turn. Fortunately, the “water
(a) 48 (b) 72 wars”, forecast by so many, have not yet materialized. War has
(c) 36 (d) 24 taken place, but over oil, not water. Water is radically altering
and affecting political boundaries all over the world, between as
(e) None of these
www.newspaperkorner.wordpress.com
172 Practice Set - 10
www.newspaperkorner.wordpress.com
well as within countries. In India, water conflicts are likely to (a) Political interventions
worsen before they begin to be resolved. Till then they pose a (b) Excessive analysis of the issue
significant threat to economic growth, security and health of the (c) Reformist movements by political parties
ecosystem and the victims are likely to be the poorest of the poor (d) Inadequate administrative and legislative frameworks
as well as the very sources of water - rivers, wetlands and aquifers. (e) None of these
Conflicts might sound bad or negative, but they are logical 85. Which of the following can be inferred about water conflicts?
developments in the absence of proper democratic, legal and (a) Water management techniques like dams, linking rivers,
administrative mechanisms to handle issues at the root of water etc have negative consequences.
conflicts. Part of the problem stems from the specific nature of (b) There is no real solution to water conflicts.
water, namely that water is divisible and amenable to sharing; (c) Despite receiving much attention water conflicts remain
one unit of water used by one is a unit denied to others; it has unresolved.
multiple uses and users and involves resultant trade-offs. (d) Water conflicts threaten the livelihood of those who
Excludability is an inherent problem and very often exclusion depend on water sources.
costs involved are very high: it involves the issue of graded (e) None of these
scales and boundaries and need for evolving a corresponding 86. According to the passage, which of the following is a
understanding around them. Finally, the way water is planned, limitation of water resulting in disputes?
used and managed causes externalities, both positive and (a) Water is not a divisible resource.
negative, and many of them are unidirectional and asymmetric. (b) Manipulation of water distribution is easy.
There is a relatively greater visibility as well as a greater (c) Water is an interconnected resource.
body of experience in evolving policies, frameworks, legal set- (d) Water is an immobile resource.
ups and administrative mechanisms dealing with immobile natural (e) None of these
resources, however contested the space may be. Reformists as Directions (Qs. 87 & 88) : Pick out the word which is the closest
well as revolutionary movements are rooted in issues related to in meaning to the word printed in bold as used in the passage.
land. Several political and legal interventions addressing the issue 87. radically
of equity and societal justice have been attempted. Most countries (a) suddenly (b) equally
have gone through land reforms of one type or another. Issues (c) completely (d) moderately
related to forests have also generated a body of comprehensive
(e) concurrently
literature on forest resources and rights. Though conflicts over
88. inherent
them have not necessarily been effectively or adequately resolved,
(a) functional (b) intense
they have received much more serious attention, have been
(c) persistent (d) characteristic
studied in their own right and practical as well as theoretical
(e) genetic
means of dealing with them have been sought. In contrast, water
Directions (Qs. 89 & 90) : Pick out the word which is opposite in
conflicts have not received the same kind of attention.
meaning to the word printed in bold as used in the passage.
81. According to the author which of the following is/are
89. asymmetric
consequences of water conflicts?
(a) unsteady (b) equilibrium
(A) Trans-border conflicts between developing countries.
(c) discouraging (d) superior
(B) Water bodies will remain unused and unaffected till
(e) contradictory
the conflict is resolved.
90. denied
(C) Water conflicts have altered the political boundaries
(a) considered (b) assigned
within countries.
(c) concerned (d) fined
(a) Only (A) (b) Only (B)
(e) acknowledged
(c) Only (C) (d) Both (A) & (C) Directions (Qs. 91-95) : Rearrange the following six sentences
(e) None of these (A), (B), (C), (D), (E) and (F) in the proper sequence to form a
82. The author’s main objective in writing the passage is to meaningful paragraph; then answer the questions given below
(a) showcase government commitment to solve the water them.
distribution problem (A) There are a number of items in the atomic energy
(b) make a strong case for war as the logical resolution for programme which are being made indigenously.
water conflicts (B) Given the overall energy situation in India, the use of
(c) point out the seriousness of the threat posed by nuclear power in some measure is inescapable even
unresolved water conflicts while thermal and hydro power continue to be the
(d) describe how the very nature of water contributes to dominant elements.
water struggles (C) However, commercial aspects of exploiting nuclear
(e) criticise governmental efforts for water conflict capabilities, especially for power-generation
resolution. programmes, have been recently given high priority.
83. What is the Prime Minister’s advice to resolve water (D) Atomic energy programmes have been subject to
disputes? severe restrictions for very obvious reasons as the
(a) Link all rivers to make national grid. Department of Atomic Energy is becoming self-reliant
(b) Politicians alone can solve the problem. in areas in which only a few countries have such
(c) Bridges and dams can resolve water issues. capability.
(d) Make consensual and conscious efforts. (E) Even to meet these nuclear power requirements, India
(e) Create public awareness. critically requires a commercial-level power-generation
84. According to the author, which of the following factors capability, with its commensurate safety and nuclear
aggravates water disputes? waste management arrangements.
www.newspaperkorner.wordpress.com
Practice Set - 10 173
www.newspaperkorner.wordpress.com
(F) Thus, in the Indian context energy security is also 101. (a) accepted (b) false
crucial, perhaps much more than it is for the USA, (c) real (d) greeted
because India imports a good part of its crude oil (e) expected
requirements, paying for it with precious foreign 102. (a) rarely (b) widely
exchange. (c) wrongly (d) leisurely
91. Which of the following will be the FIFTH sentence after (e) faithfully
rearrangement? 103. (a) principles (b) various
(a) A (b) B (c) measurement (d) idea
(c) C (d) D (e) speculation
(e) E 104. (a) meaning (b) worked
92. Which of the following will be the THIRD sentence after (c) affected (d) cause
rearrangement?
(e) witness
(a) A (b) B
105. (a) one (b) much
(c) C (d) D (c) everyone (d) more
(e) E (e) singular
93. Which of the following will be the SECOND sentence after
rearrangement? Directions (Qs. 106-110) : The following questions consist of a
(a) A (b) B single sentence with one blank only. You are given five as answer
(c) C (d) D choices you have to pick up two correct answers, either of which
(e) E will make the sentence meaningfully complete.
94. Which of the following will be the FIRST sentence after 106. Out of the 6.5 million abortions …………..in 2010, 68% were
rearrangement? performed by an unqualified person or in unsafe
(a) A (b) B environment.
(c) C (d) D (1) Recorded (2) Registered
(e) E (3) Counted (4) Performed
95. Which of the following will be the FOURTH sentence after (5) Conducted
rearrangement? (a) Only 1 (b) 1and 2
(a) A (b) B (c) 1and 5 (d) 4 and 5
(c) C (d) D (e) None of these
(e) E 107. Mr. John has been ……….for the top job at the ADB bank.
Directions (Qs. 96-105) : In the following passage there are (1) Shortlisted (2) Selected
blanks, each of which has been numbered. These numbers are (3) Appointed (4) Questioned
printed below the passage and against each, five words are (5) Approved
suggested, one of which fits the blank appropriately. Find out the (a) 3 and 5 (b) 1 and 2
appropriate word in each case. (c) 2 and 3 (d) 1 and 4
Since 96 times, people have 97 that human
activity could affect the environment. The discovery of past ice (e) None of these
ages shows that Earth’s climate is in constant 98 and that 108. The procedure adopted by the bank authorities was totally
99 history, scientists have 100 for the cause of these ……………..as they did not follow the principles of natural
changes. Though scientists discovered the greenhouse effect in justice.
the late 19th century, the theory of global warming wasn’t (1) Miscalculated (2) Flawed
101 as a scientifically proven fact until 1992 when the (3) Irrational (4) Accurate
United Nations held a Conference on Environment and (5) In place
Development. Today, global warming is a 102 accepted (a) 2 and 3 (b) 1 and 2
reality and 103 about its effects range from the hysteria to (c) 3 and 4 (d) 1and 5
the acceptance. Newspapers chronicle the slowly changing
climate and the actions that have 104 that change. From (e) None of these
developing nations to industrial countries, global climate affects 109. Delhi High Court directed RBI to respond to the plea of
105 . …………….commissioner of Income tax.
96. (a) long (b) great (1) Appointed (2) Sacked
(c) urgent (d) ancient (3) Removed (4) Selected
(e) stone-age (5) Responsible
97. (a) believed (b) succeeded (a) 1 and 2 (b) 2 and 3
(c) wished (d) lost (c) 1 and 5 (d) 5 and 4
(e) wanted (e) None of these
98. (a) hot (b) flux
(c) fight (d) changes 110. The High Court withdrew a …..…approval that allowed him
(e) reality to make numerous trips to hospitals in last 4 years.
99. (a) with (b) for (1) Ordered (2) Pending
(c) to (d) throughout (3) Blanket (4) Prior
(e) as (5) False
100. (a) contribute (b) talked (a) 5 and 1 (b) 1 and 2
(c) resigned (d) visited (c) 2 and 3 (d) 3 and 4
(e) searched (e) None of these
www.newspaperkorner.wordpress.com
174 Practice Set - 10
www.newspaperkorner.wordpress.com
Directions (Qs. 111-112): In each of the following questions four 122. World Kidney Day was observed on ________.
words are given of which two words are most nearly the same or (a) 14 March (b) 13 March
opposite in meaning. Find the two words which are most nearly (c) 16 March (d) 11 March
the same or opposite in meaning. (e) None of these
111. (A) Expanded (B) Proclaimed 123. Which private airline of India signed a deal with Boeing
(C) Shrunk (D) Facilitated worth 4.4 billion dollars to buy 42 Next Generation 737 MAX8
(a) A - D (b) B - D (c) C - D (d) B - C (e) A - C aircraft by 2018?
112. (A) Indelible (B) Erasable (a) IndiGo (b) Kingfisher Airlines
(C) Insignificant (D) Temporary (c) Spice Jet (d) Go Airways
(a) A - C (b) C - B (c) A - B (d) B - D (e) C - D (e) None of these
Directions (Qs. 113-115): Read each sentence to find out whether 124. The USP of a Credit Card is
there is any error in it. The error, if any, will be in one part of the (a) cashless operations (b) only for HNIs
sentence. The number of that part is the answer. If there is no (c) only for men (d) only for employed persons
error, the answer is (e). (Ignore errors of punctuation, if any). (e) transactions through cheque book
113. a) The driver of that car /b) is sounding horn for /c) the last 125. EMI can be a marketing tool if
ten minutes /d) but nobody tells him to stop. /e) No error (a) EMI is increasing (b) It is very high
114. a) If you go on letting /b) your dog chase cars /c) he will (c) It is very low
end by being /d) run down one day. /e) No error (d) EMI has no impact on marketing
115. a) He heard the guard /b) blowing the whistle and knew /c) (e) EMI is a flat rate
it is time for him /d) to enter the train. /e) No error 126. The USP of a Current Account is
(a) high profitability (b) liquidity
GENERAL/ ECONOMY/ BANKING AWARENESS (c) low rate of interest (d) costly transactions
116. Mortgage is a (e) friendly features
(a) security on movable property for a loan given by a bank. 127. Debit Card can be issued to
(b) security on immovable property for a loan given by a (a) Only Income Tax assesses
bank. (b) Only Professionals
(c) concession on immovable property for a loan given (c) Only women
by a bank. (d) All farmers
(d) facility on immovable property for a loan given by a (e) All Savings Account holders
bank. 128. Mutual Funds investments can be effectively canvassed
(e) security on immovable property for a deposit received among
by a bank. (a) only salaried persons
117. When the rate of inflation increases (b) students availing education loans
(a) purchasing power of money increases (c) HNI customers
(b) purchasing power of money decreases (d) sunrise industries
(c) value of money increases (e) poor farmers
(d) purchasing power of money remains unaffected 129. The target group for Personal Loans is
(e) amount of money in circulation decreases (a) All Private Limited Companies
118. Banks in our country normally publicise that additional (b) All Businessmen
interest rate is allowed on retail domestic term deposits of (c) All Salaried persons
(a) Minors (b) Married women (d) Minor children
(c) Senior citizens (d) Govt. employees (e) Newborn infants
(e) Rural residents 130. The full form of E.E.F.C. is–
119. Which of the following is NOT decided by the Finance (a) Export Earness Foreign Currency Account
Minister while presenting Union Budget every year? (b) Exchange Earness Foreign Currency Account
(a) Rate of Income Tax (c) Export Earness Foreign Credit Account
(b) Expenditure of Defence Services (d) Exchange Earness Foreign Credit Account
(c) Relief packages to various sections of the society (e) None of these
(d) Repo & Reverse Repo rates 131. Durand Cup is associated with
(e) Revenue Expected from various sources (a) Chess (b) Golf
120. GNP stands for: (c) Football (d) Cricket
(a) Group Net Product (e) None of these
(b) Gross National Product 132. The term ‘Fourth Estate’ refers to
(c) Grand Nuclear Process (a) Under-Developed State
(b) Parliament
(d) Group Networking Process
(c) Judiciary
(e) Grand National Product (d) Press
121. Term 'Balance of Payment' for_____. (e) None of these
(a) Diplomatic Relations 133. Which among the following in India is the second largest
(b) Exports & Imports econmic activity after agriculture?
(c) Annual production in a factory (a) Handloom
(d) Deficit in Union Budget (b) Poultry (c) Education and tuition
(e) Credit given to consumers (d) Carpentary (e) None of these
www.newspaperkorner.wordpress.com
Practice Set - 10 175
www.newspaperkorner.wordpress.com
134. Which of the following has been included in the list of 145. What is the current number of Local Area Banks in India?
Fundamental Rights to Indian citizen? (a) Three (b) Four
(a) Job to each and every single woman who has obtained a (c) Six (d) Eight
post graduation Degree and is not more than 29 years of age (e) None of these
(b) Equal opportunities to every woman wishing to join 146. SEBI has been imposed a restriction on money flow in equity
Indian Defence services at any level/position through ‘P-notes’. What is the full form of ‘P-notes ?
(c) Education to all children between 6-14 years of age
(a) permanent notes (b) purchase notes
(d) Social security to every person who has attained the
age of 70 years but does not have regular income to (c) participatory notes (d) private notes
support him/herself. (e) None of these
(e) None of these 147. DSCR indicates the ability of a company to-
135. Which of the following items does not fall in the category (a) Meet its current liabilities
of items being exported from India? (b) Service its share holders
(a) Tea (b) Footwear (c) Meet its long term debt obligations
(c) Milk (d) Steel (d) Raise further capital
(e) Garments (e) None of these
136. Which of the following organizations is known as Market 148. NBFCs are an important part of the Indian financial system.
Regulator in India? What is meant by this term ?
(a) SEBI (b) IBA (a) New Banking Financial Companies
(c) AMFI (d) NSDL (b) Non Banking Financial Companies
(e) None of these (c) Neo Banking Financial Confederation
137. Black money is (d) Non banking Fiscal Companies
(a) Counterleit currency (e) None of these
(b) Illegally earned money 149. Systematic investment Plans relates to :
(c) Income on which payment of tax is evaded (a) Mutual Funds
(d) Money earned through underhand deals (b) Life Insurance Companies
(e) None of these (c) Commercial Banks
138. Capital Adequacy Ratio is a thermometer of Bank's health. (d) Post office savings schemes
It is the ratio of a bank's __: (e) None of these
(a) capital to its risk (b) risk to capital 150. Euro money is the official currency of ?
(c) capital to assets (d) capital to liabilities (a) NATO (b) UN
(e) None of these (c) European (d) Germany and England
139. For the first time, in which year Basel Committee came up (e) None of these
with Capital Accords for banks? 151. Which of the following types of accounts are known as
(a) 1985 (b) 1988 "Demat Accounts"?
(c) 1990 (d) 1992 (a) Zero Balance Accounts
(b) Accounts which are opened to facilitate repayment of
(e) None of these
a loan taken from the bank. No other business can be
140. Which among the following is NOT a part of Tier-I capital? conducted from there
(a) Issued Capital (b) Fully Paid-up Capital (c) Accounts in which shares of various companies are
(c) Disclosed Reserves (d) Undisclosed Reserves traded in electronic form
(e) None of these (d) Accounts which are operated through internet banking
141. A consumer fails to make a payment due on a mortgage facility
loan. This is known as __: (e) None of the above
(a) Credit Risk (b) Refinancing Risk 152. Currency notes deposited in the currency chest are the
(c) Liquidity Risk (d) Settlement Risk property of ____?
(e) None of these (a) Respective bank (b) RBI
(c) SBI (d) Government of India
142. What is leverage ratio?
(e) Respective state Government
(a) capital to total assets (risk-weighted). 153. Ustad Ali Ahmad Hussain Khan, who passed away recently,
(b) capital to total assets (not risk-weighted). was exponent in which musical instrument?
(c) capital to total liabilities (a) Sarangi (b) Shehnai
(d) Tier-I capital to Tier-II capital (c) Sitar (d) Tabla
(e) None of these (e) None of these
143. Which of the following is India's first Credit Rating Agency? 154. What is India's rank in the United Nations' World Happiness
(a) ICRA (b) CRISIL Index 2016?
(c) ONICRA (d) CARE (a) 101 (b) 112
(c) 118 (d) 129
(e) None of these (e) None of these
144. RBI takes "Prompt Corrective Action" when a bank is faced 155. The Fulbright - Kalam Climate Fellowship has been launched
to __: by India and which country?
(a) Market Risk (b) Credit Risk (a) United States (b) Australia
(c) Liquidity Risk (d) Low Capital Adequacy (c) Russia (d) Germany
(e) None of these (e) None of these
www.newspaperkorner.wordpress.com
176 Practice Set - 10
www.newspaperkorner.wordpress.com

Time : 30 min. Max. Marks : 50

1. Write a letter on any one of the following:- (20 marks)


(a) Write a letter to the Chairman of your Municipality about the bad condition of roads in your locality.
(b) Write a letter to your younger brother who has asked your advice on the choice of a profession.
(c) Write a letter to the head of an NGO advocating a total ban on child labour.
2. Write a paragraph on any one of the following in not more than 150 words. (10 marks)
(a) Is modern civilization is failure?
(b) Corruption in India.
(c) Should use of computer be made compulsory in all government institutions?
3. Write an essay on any one of the following in about 250 words. (20 marks)
(a) Contemporary Banking Industry.
(b) India needs kraanti.
(c) Responsibilities of managers in banking sector.

www.newspaperkorner.wordpress.com
Practice Set - 10 177
www.newspaperkorner.wordpress.com
Answer Key
1 (a) 21 (b) 41 (e) 61 (e) 81 (a) 101 (a) 121 (b) 141 (a)
2 (b) 22 (e) 42 (c) 62 (c) 82 (c) 102 (b) 122 (b) 142 (b)
3 (d) 23 (a) 43 (c) 63 (a) 83 (d) 103 (e) 123 (c) 143 (b)
4 (a) 24 (d) 44 (b) 64 (d) 84 (e) 104 (c) 124 (a) 144 (d)
5 (b) 25 (b) 45 (e) 65 (b) 85 (a) 105 (c) 125 (c) 145 (b)
6 (e) 26 (b) 46 (b) 66 (a) 86 (e) 106 (d) 126 (e) 146 (c)
7 (b) 27 (e) 47 (d) 67 (c) 87 (c) 107 (c) 127 (e) 147 (c)
8 (a) 28 (d) 48 (c) 68 (c) 88 (d) 108 (b) 128 (a) 148 (b)
9 (d) 29 (a) 49 (a) 69 (a) 89 (b) 109 (b) 129 (c) 149 (a)
10 (b) 30 (d) 50 (b) 70 (b) 90 (b) 110 (d) 130 (c) 150 (c)
11 (c) 31 (d) 51 (a) 71 (b) 91 (e) 111 (e) 131 (c) 151 (c)
12 (e) 32 (b) 52 (d) 72 (d) 92 (a) 112 (c) 132 (d) 152 (b)
13 (b) 33 (c) 53 (d) 73 (b) 93 (d) 113 (b) 133 (b) 153 (b)
14 (c) 34 (a) 54 (b) 74 (a) 94 (b) 114 (c) 134 (c) 154 (c)
15 (a) 35 (e) 55 (c) 75 (b) 95 (c) 115 (c) 135 (c) 155 (a)
16 (d) 36 (b) 56 (d) 76 (d) 96 (d) 116 (b) 136 (a)
17 (d) 37 (d) 57 (a) 77 (d) 97 (a) 117 (b) 137 (c)
18 (d) 38 (a) 58 (a) 78 (e) 98 (d) 118 (c) 138 (a)
19 (a) 39 (b) 59 (c) 79 (c) 99 (d) 119 (d) 139 (b)
20 (c) 40 (d) 60 (d) 80 (d) 100 (e) 120 (b) 140 (d)

Answers & Explanations


1. (a) Hence, only conclusion I follows.
2. (b) Hence, conclusion II follows.
3. (d) According to statements.
Metals

Mails Updates

Chats
Metals
or
or

Mails 4. (a) Hence, conclusion I follows.


or 5. (b) Hence, only conclusion II follows.
Mails
(6-8) : It is a case of three-step type shifting. In a 3-step type
Chats Chats shifting, the change in going from Input to step I differs
from the change from step I to step II and step II to step III.
Hence, conclusion I follows. The change from Input to step I matches with change from
(4 -5) : step III to step IV; the change from step I to step II matches
According to statements with the change step IV to step V; and the change from step
II to step III matches with the change from step V to step VI.
Let us replace the word of the input by letters pull = A, the =
B, cover = C, and = D, then = E, push = F, into = G
Metals 1 2 3 4 5 6 7
Input : A B C D E F G
Step I : A B E D C F G
Step II: E B A G F C D
or Step·III: G A B E D C F
Step IV: G A D E B C F
www.newspaperkorner.wordpress.com
178 Practice Set - 10
www.newspaperkorner.wordpress.com
Step V: D A G F C B E II. O > U (False)
Step VI: F G A D E B C 25. (b) Statement : L = M > N; M > P; L < K
Step VII: F G E D A B C Conclusions :
Step VIII: E G F C B A D I. K > P (False)
6. (e) Step VI
Input: T r y your best until you get goal
II. N < K (True)
A B C D E F G

Oriental Bank
of Commerce
26- 30.

Maharashtra
get goal t r y until you your best R L

Canara Bank

Dena Bank
Bank of
F G A D E B C I E
Now, see the chart. You get FGADEBC in step VI. G F
7. (b) H
Step VI: deep gutter ball into t h e has fallen
T
F G A D E B C
T
Input: A B C D E F G
ball has fallen into t h e deep gutter
8. (a) C A B D
Step IV: we can’t measure t h e depth without scale
G A D E B C F
P Q S R
Step VII: F G E D A B C
scale we t h e measure can’t depth without

Punjab National
Syndicate Bank
9. (d) It is mentioned that unseasonal downpour paralysed R

Indian Bank
L

UCO Bank
the normal life in the state. Therefore, it is not prudent I
E

Bank
to set up a review committee. The Course of action (B) G
does not address the problem properly. Thus, only F H
Course of action (C) is suitable for pursuing. T T
10. (b) It is clear from the statement that a farm’s areas for organic
and chemical farming are different. 26. (b) The person from Punjab National Bank, Q faces A.
11. (c) There is a perception among small brands that sale in a B is from Bank of Maharashtra and he is a immediate
supermarket is higher than that of small grocery stores. neighbour of A.
12. (e) The statement clearly implies that smaller brands are A faces of person who sits second to the left of R.
currently making substantial losses in their businesses. A is from Dena Bank.
Consider the term ‘‘troubled waters’’. A sits third from the left and second from the right.
13. (b) Statement (C) represents a dlisadvantage of the small 27. (e) S is seated between R and Q, the person from Punjab
grocery stores over the Supermarkets from the National Bank.
perspective of a smaller brand. Small grocery stores do 28. (d) Persons at the extreme ends : C from Canara Bank; D
not help in distribution of any brand. from Oriental Bank of Commerce; P from Syndicate
14. (c) Statement (D) represents a reason for the shift from local Bank: R from UCO Bank.
grocery stores of supermarkets by the smaller brands. 29. (a) S from Indian Bank faces B from Bank of Maharashtra.
15. (a) Statement (A) will prove that step taken by the smaller 30. (d) P faces the immediate neighbour of A from Dena Bank.
brands may not necessarily be correct. B faces the immediate neighbour of Q from Punjab
16. (d) People are going abroad for better opportunities.
National Bank. Similarly, D faces immediate neighbour
17. (d) Assumption (d) is implicit in the given statement. If
of S from Indian Bank.
people don't show respect then there is no meaning of
this kind of appeal. 31. (d) Check the position of each element separately. The
18. (d) From given fact it can be inferred that the government elements are replaced after five steps.
generally provides protection to travellers accross the 32. (b) The lower-left element rotates by 45° and 90° ACW
country. alternately. For other elements, follow if 1 = 5 then 2 = 6 rule.
19. (a) All the three causes will effect the sales of four wheelers. 33. (c) In alternate steps ‘A’ rotates by 45° and 90° ACW and
20. (c) (A) would not be effective step as it will create a lots of shifts half-a-side and one side CW. ‘T’ rotates by
problems to others. Steps (B) and (C) will encourage in 45°ACW and shifts half-a-side CW in each step while
favour of the girl child. the middle element is replaced in each step.
21. (b) Statement : F > G = H; G > J > K 34. (a) In alternate staeps ‘K’ shift half-a-side and one side CW.
Conclusions : ‘A’ shifts from lower left to upper right and vice versa.
I. F > K (False) 35. (e) In each step the half-shaded oval rotates by 45° ACW
II. K < H (True) while the other elements shift half a side ACW.
22. (e) Statement : P < Q = R > S > T
(46-50):
Conclusions :
I. T < Q (True) Total number of
Specialisation M ale Female
II. R > P (True) students
23. (a) Statement : A < B < C; A > D; C < F HR 990 396 594
Conclusions : M arketing 720 420 300
I. D < C (True) International Business 810 675 135
II. F > D (False) Banking 540 340 200
24. (d) Statement : U > A = I < O < E IT 945 315 630
Conclusions : Finance 495 275 220
I. I < E (False) 4500 2421 2079
www.newspaperkorner.wordpress.com
Practice Set - 10 179
www.newspaperkorner.wordpress.com
46. (b) It is obvious from above.
æ 77500 ö
300 ´100 = çè ÷ = 19375
47. (d) The required per cent = = 6.6% » 7% 4 ø
4500 62. (c) Number of children in state A (singing)
48. (c) The required number of men = 675 + 340 = 1015
= 25000
220 ´ 100 Total number of children in state A (all classes)
49. (a) The required per cent = = 80%
275 = 25000 + 22500 + 15000
50. (b) IT is obvious from above table. = 62500
8+9 25000
51. (a) Required ratio = = 17 : 33
15 + 18 \ Required % = ´ 100 = 40
62500
52. (d) Here, do not find the ratio of the no. of qualified
63. (a) Total number of children (dancing)
candidates to that of the appeared. Simply check the
ratio of % qualified candidates with respect to the = (22500 + 22500 + 25000 + 27500)
appeared is the least for which state. = 97500
53. (d) Required difference = (21 – 13)% of 9000 = 720 Total number of children (painting)
(16 + 7)% of 9000 = (15000 + 12500 + 32500 + 17500)
54. (b) Reqd % = (11 + 8)% of 45000 ´ 100 = 24.21% = 77500
\ Difference = 97500 – 77500
(16 + 21)%of 9000 = 20000
55. (c) Reqd ratio = = 37 : 40 64. (d) Number of children from state B (dancing) = 22500
8% of 45000
Number of children from state D (dancing) = 27500
150
56. (d) Required percentage = ´ 100 = 115.38% \ Ratio = 22500 : 27500 = 9 : 11
130 65. (b) Average number of children from state C
70 30000 + 25000 + 32500
57. (a) % marks in Eng = ´ 100 = 35 =
200 3
110
% marks in Geo = ´ 100 = 73 87500
150 = = 29166.67 » 29167
3
120 66. (a) Man hours spent in coding = 430 + 100 = 530
% marks in Phy = ´ 100 = 60
200 Man hours spent in offshore design and coding
140 = 100 + 430 = 530.
% marks in His = ´ 100 = 93
150 Man hours spent in Testing = 290 + 180 = 570.
120 67. (c) Total work onsite » 80 + 100 + 170 = 350
% marks in Hin = ´ 100 = 80
150 Total work done = 350 +100 + 430 + 290 = 1170.
160
Hence approximately 30% of the work is carried out
% marks in Che = ´ 100 = 80 onsite.
200
350
421 \ % work = = 29.9% .
\ Average % marks = » 70 1170
6
58. (a) Marks obtained by B in Geography, History and Hindi 68. (c) Man hours spent onsite = 350.
= 100 + 110 +150 = 360 Sum of estimated and actual effort for offshore design
= 100 + 80 = 180.
360
\ Average = = 120 Estimated man-hours of offshore coding = 430
3
Actual man hours of offshore testing = 290.
59. (c) Percentage of marks obtained by student B
440
English Geography Physics History Hindi Chemistry Half of estimated offshore coding = = 220.
2
40 66.6 65 73.33 100 85 So, (c) is correct.
140 -110 69. (a) Total hours spent = 1170 (as calculated in Q. 27)
60. (d) Required difference = ´100
150 Hours in coding = 430 + 100 = 530 = 45.3%
30 House in Design = 190 = 16.2%
= ´100 = 20% House in Offshore testing = 290 = 24.8%
150
61. (e) Average number of children (painting) House in Offshore testing + Design = 290 + 190 = 480
= 41%.
æ 15000 + 12500 + 32500 + 17500 ö
=ç ÷ø Hence (a) is the closest option.
è 4
www.newspaperkorner.wordpress.com
180 Practice Set - 10
www.newspaperkorner.wordpress.com
70. (b) Total offshore work = 100 + 430 + 290 = 820 man hours. 3 3
50% of offshore work is carried out on-site = 410. (Ravi + Shyam) do of the work in 24 ´ = 18days
4 4
Distribution of effort is in ratio, 180 : 530 : 430 78. (e) Let the age of A = 6x years
= 18 : 53 : 43. Let the age of B = 7x years
Effort distributed to testing will be 6x × 7x = 672
43 672
´ 410 @ 154 man-hours. x2 = = 16 \ x = 4
18 + 53 + 43 42
Age of A after 6 years = 30 years
290 Age of B after 6 years = 34 years
Offshore testing work is = 145 .
2 Required ratio = 30 : 34 = 15 : 17
\ Proportion of testing carried out offshore is 79. (c) Different of amount received by R and Q is (7 – 5 =) 2,
Total amount received by P and Q = (3 + 5 =) 8:,
145
´ 100 = 33% @ 30%. . Then, 2 º ` 4000
140 + 140 + 154
71. (b) Profit will be shared in the ratio of 4000
8: º ´ 8 = ` 16000
11 × 3 : 16.5 × 3 : 8.25 × 3 2
= 11 : 16.5 : 8.25 = 44 : 66 : 33 81. (a) (A) follows because the passage talks of “water
conflicts” between developing countries. However, we
33 are told, such conflicts have not escalated into wars.
Anil’s share in the profit = ´ 19.5 = 14.5 lakh
143 As for (C), the alteration has been done by water, not
50% of Anil’s share = 2.25 lakh by water conflicts.
72. (d) Cost price of TV when discount is not offered 83. (d) Note that the Prime Minister urges the state
100 governments to show “an appreciation of the other
= 11250 ´ = `12500 point of view”.
90
85. (a) This can be inferred from the last sentence of the third para.
Total cost of TV after transport and installation 86. (e) Water is a divisible resource.
= 12500 + 800 + 150 = 13450 106. (d) Medical operations conducted by unqualified person
To earn 15% profit, he must sell at and that also in unsafe environment must have not
been recorded or registered. Counted is not right in
115
13450 ´ = `15467.50 the context of the situation. Only performed /
100 conducted can be the right choice.
73. (b) Interest earned in 1st half of a year 107. (c) Selected/Appointed is grammatically correct.
1 20 Approved is not right for a person. It might have been
= 20,000 ´ ´ = 2000 right if the sentence would have been like - Proposal
2 100
for Mr. John's appointment for the top job has been
Similarly, APPROVED by Board of Directors. Questioned is
During second half, interest earned = 2200 incorrect grammar usage.
During second year, interest earned = 4840 108. (b) Use of accurate or in place contradicts the statement.
(Note : Interest is calculated as compound) Procedure can be flawed or miscalculated so option
74. (a) Required no. of ways (b) is correct option. Irrational is not necessarily flawed.
= 2P2 × 4P4 = 48 109. (b) Sacked and removed both carry the same sense.
75. (b) Relative speed of two trains Appointed and Selected are also similar but not given
in options. So correct choice is (b)
180 + 270 m 4500 m 110. (d) Blanket and prior both completes the sentence
= =
10.8 s 108 s meaningfully. Here blanket means inherent in the order
4500 18 km but may not be directly or explicitly.
= ´ = 150 km/hr Statement 1 introduces the concept of Strategic change
108 5 h
through mobilizing the employees.
Speed of second train = 150 – 60 = 90 km/h.
76. (d) Since a 3-digit number cannot start with digit 0, the Statement 4 suggests how to mobilize the employees
through involvement.
hundredth place can have place can have any of the 4
digits. Now, the tens and units place can have all the 5 Statement 5 carries forward the point of Statement 4
digits. Therefore, the total possible 3-digit numbers by stating what can happen in absence of the employee
are 4 × 5 × 5, i.e., 100. involvement (isolation) from decision making.
77. (d) 18 days Statement 2 confirms what was stated in earlier
Shyam alone worked 10 days. So work done by him statements as managers also point to the same.
Statement 3 concludes the passage by stating what is
10 1 role of managers and how everything discussed earlier
= =
40 4 depends on it.
\ (Ravi + Shyam) have done 113. (b) It should be ‘has been sounding horn’.
114. (c) Replace ‘by’ with ‘up’.
1 3
1- = of the work. 115. (c) It should be ‘was’ in place of ‘is’.
4 4 www.newspaperkorner.wordpress.com
www.newspaperkorner.wordpress.com

Preliminary Exam
Practice Set - 1
INSTRUCTIONS
• This Preliminary Exam practice set consists of three sections. Quantitative Aptitude (Qs. 1-35); Reasoning
Ability (Qs. 36-70) and English Language (Qs. 71-100).
• All the questions are compulsory.
• Each question has five options, of which only one is correct. The candidates are advised to read all the
options thoroughly.
• There is negative marking equivalent to 1/4th of the mark allotted to the specific question for wrong answer.

Time : 1 hrs. Max. Marks : 100

QUANTITATIVE APTITUDE
(272 - 32)(124 + 176)
6. =?
DIRECTIONS (Qs. 1-10) : What will come in place of the question 17 ´15 –15
mark (?) in the following equations ? (a) 0 (b) 2.25 (c) 300 (d) 240
(e) None of these

117 ´ 117 ´ 117 - 98 ´ 98 ´ 98 50 ?


1. =? 7. =
117 ´ 117 + 117 ´ 98 + 98 ´ 98 ? 1
12
(a) 215 (b) 311 (c) 19 (d) 29 2
(e) None of these 25 4
(a) (b) (c) 4 (d) 25
a 4 3a + 2b 2 25
2. If = , then =?
b 3 3a - 2b (e) None of these
(a) 6 (b) 3 (c) 5 (d) –1
112 576 256
(e) None of these 8. ´ ´ =?
196 12 8
? 72 (a) 8 (b) 12 (c) 16 (d) 32
3. =
196 56 (e) None of these
(a) 18 (b) 14 (c) 324 (d) 212 ?
(e) None of these 9. = 550
2.25
17.28 ¸ ? (a) 825 (b) 82.5 (c) 3666.66 (d) 2
4. = 200
3.6 ´ 0.2 (e) None of these
(a) 120 (b) 1.20 (c) 12 (d) 0.12
5- 3
(e) None of these 10. =?
5+ 3
(3.537 - 0.948)2 + (3.537 + 0.948)2 (a) 4 + 15 (b) 4 - 15
5. =?
(3.537)2 + (0.948)2
1
(a) 4.485 (b) 2.589 (c) 4 (d) 2 (c) (d) 1
2
(e) None of these (e) www.newspaperkorner.wordpress.com
None of these
PS-2 Practice Set - 1
www.newspaperkorner.wordpress.com
DIRECTIONS (Qs. 11-15) : Study the following chart to answer (a) 207 : 76 (b) 76 : 207
the questions given below : (c) 152 : 207 (d) Data inadequate
(e) None of these
Villages % population below poverty line 16. A train is moving at a speed of 132 km/h. If the length of the
A 45 train is 110 metres, how long will it take to cross a railway
B 52
platform, 165 metres long ?
C 38
D 58
(a) 5s (b) 7.5 s (c) 10 s (d) 15 s
E 46 (e) None of these
F 49 17. If 15 women or 10 men can complete a project in 55 days, in
G 51 how many days will 5 women and 4 men working together
complete the same project ?
Proportion of population of seven villages in 1995 (a) 75 (b) 8 (c) 9 (d) 85
(e) None of these
G A 18. Ashu’s mother was three times as old as Ashu, 5 years ago.
15% 13% After 5 years, she will be twice as old as Ashu. How old is
Ashu at present?
B (a) 15 (b) 20 (c) 10 (d) 5
F
16% (e) None of these
13%
19. A conical flask has base radius ‘a’ cm and height ‘h’ cm. It
is completely filled with milk. The milk is poured into a
C cylindrical thermos flask whose base radius is ‘p’ cm. What
8% will be the height of the solution level in the flask ?
E
18% D a 2h 3hp2 p2 3a 2
(a) cm (b) cm (c) cm (d) cm
17% 3p 2
a2 3h 2 hp 2
(e) None of these
11. In 1996, the population of villages A as well as B is increased 20. A sum was put at simple interest at a certain rate for 2 years.
by 10% from the year 1995. If the population of village A in Had it been put at 3% higher rate, it would have fetched
1995 was 5000 and the percentage of population below ` 300 more. Find the sum.
poverty line in 1996 remains same as in 1995, find
(a) ` 6000 (b) ` 8230 (c) ` 5000 (d) ` 4600
approximately the population of village B below poverty
(e) None of these
line in 1996.
(a) 4000 (b) 45000 DIRECTIONS (Qs. 21-25) : Identify which number is wrong in
(c) 2500 (d) 3500 the given series.
(e) None of these 21. 2, 3, 4, 4, 6, 8, 9, 12, 16.
12. If in 1997 the population of village D is increased by 10% (a) 3 (b) 9
and the population of village G is reduced by 5% from 1995
(c) 6 (d) 12
and the population of village G in 1995 was 9000, what is the
total population of villages D and G in 1997? (e) None of these
(a) 19770 (b) 19200 22. 3, 4, 10, 32, 136, 685, 41
(c) 18770 (d) 19870 (a) 136 (b) 10
(e) None of these (c) 4116 (d) 32
13. If in 1995 the total population of the seven villages together (e) None of these
was 55,000 approximately, what will be population of village 23. 69, 55, 26, 13, 5
F in that year below poverty line ? (a) 26 (b) 13
(a) 3000 (b) 2500 (c) 5 (d) 55
(c) 4000 (d) 3500 (e) None of these
(e) None of these 24. 24576, 6144, 1536, 386, 96, 4
14. If the population of village C below poverty line in 1995 was
(a) 386 (b) 6144
1520, what was the population of village F in 1995 ?
(c) 96 (d) 1536
(a) 4000 (b) 6000
(e) None of these
(c) 6500 (d) 4800
(e) None of these 25. 11, 5, 20, 12, 40, 26, 74, 54
15. The population of village C is 2000 in 1995. What will be the (a) 5 (b) 20
ratio of population of village C below poverty line to that of (c) 40 (d) 26
the village E below poverty line in that year ? (e) None of these
www.newspaperkorner.wordpress.com
Practice Set - 1 PS-3
www.newspaperkorner.wordpress.com
DIRECTIONS (Qs. 26-30): Find out the approximate value which 2
should come in place of the question mark in the following 34. th of Anil's salary is equal to Bhuvan's salary and seven-
5
questions. (You are not expected to find the exact value. ) ninth of Bhuvan's salary is equal to Chandra's salary. If the
sum of the salary of all of them is ` 77,000, then, how much
26. 45689 = ? is Bhuvan's salary?
(a) 180 (b) 415 (a) ` 45,000 (b) ` 18,000
(c) 150 (d) 210 (c) ` 15,000 (d) ` 28,000
(e) 300 (e) None of these
35. A tap can fill an empty tank in 12 hours and a leakage can
(10008.99)2 empty the whole tank in 20 hours. If the tap and the leakage
27. ´ 3589 ´ 0.4987 = ?
10009.001 are working simultaneously, how long will it take to fill the
whole tank?
(a) 3000 (b) 300000
(a) 25 hours (b) 40 hours
(c) 3000000 (d) 5000
(c) 30 hours (d) 35 hours
(e) 9000000
(e) None of these
28. 399.9+ 206 × 11.009= ? REASONING ABILITY
(a) 2800 (b) 6666
(c) 4666 (d) 2400 36. Which is the third number to the left of the number which is
exactly in the middle of the following sequence of numbers?
(e) 2670
1234567892468975398764321
2 7 17 6 (a) 3 (b) 2
29. + ´ ¸ =? (c) 5 (d) 6
5 8 19 5
(e) None of these
1 37. In a certain code IDEAS is written as HEDBR and WOULD
(a) 1 (b) is written as VPTMC. How will RIGHT be written in the
2
same code ?
1 3 (a) QJHIS (b) QJFGS
(c) 2 (d) (c) SHHGU (d) QJFIU
2 4 (e) QJFIS
9 38. If the alphabet is written in the reverse order and every
(e) alternate letter starting with Y is dropped, which letter will
11 be exactly in the middle of the remaining letters of the
30. (299.99999)3 = ? alphabet.
(a) 27000000 (b) 9000000000 (a) M (b) N
(c) 180000 (d) 2.7 × 109 (c) O (d) M or O
(e) None of these
(e) 2700000 39. In a row of girls, Rita and Monika occupy the ninth place
31. A reduction of 20% in the price of sugar enables a purchaser from the right end and tenth place from the left end,
1 respectively. If they interchange their places, then Rita and
to obtain 2 kg more for ` 160. Find the original price per Monika occupy seventeenth place from the right and
2 eighteenth place form the left respectively. How many girls
kg of sugar. are there in the row?
(a) 25 (b) 26
(a) ` 12 (b) ` 20 (c) ` 16 (d) ` 18 (c) 27 (d) Data inadequate
(e) None of these (e) None of these
32. Mrs. X spends ` 535 in purchasing some shirts and ties for 40. In a certain code language 'Ka Bi Pu Ya' means 'You are
very intelligent'; 'Ya Lo Ka Wo' means 'They seem very
her husband. If shirts cost ` 43 each and the ties cost ` 21
intelligent'; 'La Pu Le' means 'You can see' and 'Sun Pun
each, then what is the ratio of the shirts to the ties, that are Yun Ya' means 'how intelligent she is', In that language,
purchased ? which of the following words means 'are'?
(a) 1 : 2 (b) 2 : 1 (c) 2 : 3 (d) 3 : 4 (a) Ka (b) Bi
(e) None of these (c) Ya (d) Pu
33. Anish spends 25% of his salary on house rent, 5% on food, (e) None of these
15% on travel, 10% on clothes and the remaining amount of 41. Ankit is related to Binny and Chinky, Daizy is Chinky’s
` 22,500 is saved. What is Anish's salary ? mother. Also Daizy is Binny’s sister and Aruna is Binny’s
sister. How is Chinky related to Aruna?
(a) ` 40,000 (b) ` 40,500
(a) Niece (b) Sister
(c) ` 45,500 (d) ` 50,000 (c) Cousin (d) Aunt
(e) None of these (e) None of these
www.newspaperkorner.wordpress.com
PS-4 Practice Set - 1
www.newspaperkorner.wordpress.com
42. Rama remembers that she met her brother on Saturday, which 48. Statements: All stones are rocks.
was after the 20th day of a particular month. If the 1st day of Some rocks are bricks.
that month was Tuesday, then on which date did Rama Some bricks are cement.
meet her brother ? Conclusions: I. Some cements are rocks.
(a) 24th (b) 23rd II. Some bricks are stone
(c) 25th (d) 26th III. Some stones are cement.
(e) None of these (a) Only I and either II or III follow
43. If it is possible to make only one such number with the first, (b) Only either II or III follows
the fourth and the sixth digits of the number 531697 which (c) Only I and II follow
is the perfect square of a two digit even number, which of (d) All follow
the following will be the second digit of the two digit even (e) None of these
number. If no such number can be made, give '@' as the 49. Statements: All flats are buildings.
answer and if more than one such number can be made, All buildings are bungalows.
give '©' as the answer. All bungalows are apartments.
(a) 4 (b) 2 Conclusions: I. Some apartments are flats.
(c) 6 (d) @ II. All flats are bungalows.
(e) © III. Some bungalows are flats.
44. In a certain code JOURNEY is written as TNISZFO. How is (a) None follows
MEDICAL written in that code? (b) Only I and II follow
(a) CDLJMBD (b) CDWDBM (c) Only II and III follow
(c) LDCJMBD (d) EFNJMBD (d) Only I and III follow
(e) None of these (e) All I, II and III follow
45. If 'K' denotes '×', 'B' denotes '÷', 'T' denotes '–' and 'M' 50. Statements: Some spectacles are lenses.
denotes '+', then – Some lenses are frames.
40 B 8 T 6 M 3 K 4 = ? All frames are metals.
(a) 19 (b) 11
Conclusions: I. Some lenses are metals
(c) –31 (d) 23
II. Some metals are spectacles.
(e) None of these
III. Some frames are spectacles.
DIRECTIONS (Qs. 46-50): In each question below are three (a) Only III follows
statements followed by three conclusions numbered I, II and III. (b) Only I follows
You have to take the three given statements to be true even if (c) Only I and either II or III follow
they seem to be at variance from commonly known facts and then (d) Only I and II follow
decide which of the answers (a), (b), (c), (d) and (e) is the correct (e) None of these
answer and indicate it on the answer sheet.
46. Statements: Some chairs are tables. DIRECTIONS (Qs. 51 - 55) : Read the following information
Some tables are drawers. carefully and answer the questions that follow:
all drawers are shelf.
Conclusions: I. Some shelves are tables. At a party, A, B, C, D and E are sitting in a circle. The group
II. Some drawers are chairs. comprises a professor, an industrialist and a businessman. The
III. Some shelves are drawers. businessman is sitting in between the industrialist and his wife
(a) Only I and III follow D. A, the professor is married to E, who is the sister of B. The
(b) Only I and either II or III follow industrialist is seated to the right of C. Both the ladies are
(c) Only II and either I or III follow unemployed.
(d) All I, II and III follow 51. What is A to B ?
(e) None of these (a) Brother (b) Uncle
47. Statements: All trees are flowers. (c) Brother-in-law (d) Can’t be determined
Some flowers are leaves. (e) None of these
No leaf is bud 52. A is sitting to the right of
Conclusions: I. No bud is a flower. (a) the industrialist (b) his wife
II. Some buds are flowers. (c) D (d) Can’t be determined
III. Some leaves are trees.
(e) None of these
(a) Only II and III follow
53. Who is the industrialist ?
(b) Only III follows
(a) D (b) A
(c) Only either I or II follows
(d) Either I or II and III follow (c) B (d) Can’t be determined
(e) None of these (e) None of these
www.newspaperkorner.wordpress.com
Practice Set - 1 PS-5
www.newspaperkorner.wordpress.com
54. Who in the group is unmarried? 63. If each of the letters in the above series of elements is given
(a) Professor (b) Industrialist a value equivalent to its serial number in the English
(c) Businessman (d) Can’t be determined alphabet, what will be the difference between the sum of
(e) None of these the consonants and the sum of the vowels used in the
55. Who among them must be graduate ? series ?
(a) B (b) A (a) 109 (b) 41
(c) C (d) E (c) 82 (d) 27
(e) None of these (e) None of these
DIRECTIONS (Qs. 56-60): In the questions given below, certain 64. Which of the following groups of elements will come in the
symbols are used with the following meanings: place of the question-mark in the series of elements given
A @ B means A is greater than B. below?
A * B means A is either greater than or equal to B. 682 $B5 E Q9 ? £D7
A # B means A is equal to B. (a) (13) 4 (b) 4 (13)
A $ B means A is either smaller than or equal to B. (c) U £ D (d) (13) £
A + B means A is smaller then B. (e) None of these
Now in each of the following questions, assuming the
65. BQ in the above series is related in ER in a similar way as AP
given statements to be true, find which of the two
is related to
conclusions I and II given below them is/are definitely true?
(a) If only conclusion I is true (a) RD (b) U
(b) If only conclusion II is true (c) KA (d) Q 3
(c) If either conclusion I or II is true (e) 6 B
(d) If neither conclusion I nor II is true DIRECTIONS (Qs. 66-70) : Study the following information
(e) If both conclusions I and II are true carefully to answer these questions.
56. Statements : B + D; E$T; T * P; P@B
Conclusions : I. P$D A group of people has six family members and an advocate.
II. P@D These are L, M, N, O, P, Q and R and having different professions.
57. Statements : E*F; G$H; H#E; G@K Each one of them is a journalist, businessman, architect, doctor
Conclusions : I. H @K and pilot but not necessarily in this order. There are three males
II. H*F and three females in the family out of which there are two married
58. Statements : P$Q; N#M; M@R; R*P couples. M is a businessman and is the father of P. N is a housewife
Conclusions : I. P + N and is daughter-in-law of O. L is neither a pilot nor a journalist. R
II. Q$M is an advocate. N is not the mother of P and O is not married to M.
59. Statements : D + T; E $V; F *T; E@D No lady is a journalist.
Conclusions : I. D $ V 66. Which of the following groups represents the three ladies
II. D + F in the group ?
60. Statements : T*U; U$W; V @L; W + V
(a) N, P, L (b) P, L, N
Conclusions : I. V @ T
II. L #W (c) L, N, O (d) O, P, L
DIRECTIONS (Qs. 61-65): Use the following series of elements (e) None of these
(alpha-number-symbol) to answer these questions. Every two- 67. Who is married to Q ?
digit number (given in brackets) is to be treated as single number. (a) N (b) O
2 8 5 6 B 9 $ Q 3 E 1 7 R D 4 £ (13) U K (18) A (14) P (c) L (d) Can’t be determined
61. Four of the following five groups of elements are alike in a (e) None of these
certain way and so form a group. Which is the one the does 68. Who among the following family members is an architect ?
not belong to that group ? (a) L (b) O
(a) 2 8 (b) 5 6 B
(c) P (d) Can’t be determined
(c) Q $ 9 (d) 1 3 E
(e) D R 7 (e) None of these
62. If each alphabet has a value of zero, each symbol (i.e., , $ 69. Which of the following is the profession of P ?
and ) has a value equivalent to the square of the numeral (a) Architect (b) Pilot
that immediately precedes the symbol or the value of 1 if it (c) Architect or pilot (d) Journalist
is not immediately preceded by a numeral, what will be the (e) None of these
sum of the values of the first 10 elements of the series starting
70. How is Q related to O ?
from the left end ?
(a) 118 (b) 46 (a) Father (b) Mother
(c) 79 (d) 107 (c) Mother-in- law (d) Son - in - law
(e) None of these (e) None of these
www.newspaperkorner.wordpress.com
PS-6 Practice Set - 1
www.newspaperkorner.wordpress.com
ENGLISH LANGUAGE with the ornaments. Seth at once rushed to the Palace of the King.
He said, “A man brought these ornaments to me and asked me to
DIRECTIONS (Qs. 71-80): Read the following passage carefully sell them. But they are the ornaments I made for the Prince who is
and answer the questions given below it. missing.” “Who is this man? Where is he?”, thundered the King.
This rogue must have murdered my little Prince and robbed his
Long ago there was a poor Brahmin named Krishnan. He
jewels!” “He is a Brahmin named Krishnan, your Majesty”, replied
could not find enough work to do. Sometimes, he and his family the goldsmith, and he is there, in my house. The king called for his
had to go without food. At last Krishnan decided to leave his most dreaded soldiers. “Arrest the Brahmin who is in the
village in search of work. Early next morning, he left the house. goldsmith’s house and throw him into the darkest dungeons of the
He walked the whole day until he came to a thick jungle. He was kingdom”, roared the King. The King’s guard stormed into the
tired, thirsty and hungry. While looking around for water to goldsmith’s house and seized Krishnan. Krishnan was thrown into
drink, he found a well. He went to the well and looked in. There he a dark dungeon to await his execution. He then remembered the
saw a jaguar, a monkey, a snake and a man. They had all fallen words of Naagesh, the snake. So he called out to him.
into the well. “O, noble Brahmin”, the jaguar called out to him, Suddenly, almost like magic, Naagesh slithered his way
“Please help me out, so that I can go back to my family.” down a narrow window into the dingy cell. “O, Lord!” hissed
“But you are a jaguar”, said Krishnan. “How do I know you Naagesh, “how did you manage to get yourself arrested?”
will not kill me?” “Don’t be afraid of me, I promise I will not do Krishnan cried and then told the snake what had happened. “I
you any harm”, replied the jaguar. Krishnan reached into the well have a plan”, hissed Naagesh. “I shall creep into the Queen’s
and pulled out the jaguar. The jaguar thanked him and said, “I’m room and bite her”, said Naagesh. “She will faint. No matter what
Shersingh. I live in a cave in the mountains. I shall be most they do, she will remain asleep. The poison will remain in her
delighted if I can repay my debt to you someday.” Krishnan then body until you place your hand on her forehead”, explained
heard the monkey calling out to him from the well. The Brahmin at Naagesh. He then left Krishnan and went to the palace. He crept
once pulled the monkey out. The monkey thanked the Brahmin. into the Queen’s room and bit her. The Queen fainted. The sad
“If you are ever in need of food, just drop in at my place below news that the Queen had been bitten by a snake spread all over
that big mountain. Bali is my name.” Now the snake called out to the Kingdom. Vaidyas came from far and near, but their medicines
him for help. “Help you!” exclaimed Krishnan. “You are a snake. had no effect. No one could revive the Queen. Finally, the King
What if you bite me?” “I shall never bite you”, said the snake. So declared that anyone who could cure the Queen would be
Krishnan pulled the snake out of the well. The snake said, handsomely rewarded. Many people went to the palace but all of
“Remember, if you are ever in any difficulty, just call out my them failed. “I can cure the Queen”, Krishnan told the guards. At
name-Naagesh, and wherever you are, I shall find you.” The jaguar, once they took him to the Queen. Krishnan sat beside the Queen
the monkey and the snake took leave of the Brahmin. But before and placed his hand on her forehead. Soon, she opened her eyes
they left, they spoke to him about the man in the well. “Please do and sat up. The King was overjoyed and shed tears of happiness.
not help him,” said Shersingh. “If you do”, said Naagesh, “you He embraced Krishnan and thanked him. “Your Majesty”, said
will be in trouble yourself.” As soon as they left, the man in the Krishnan. “I was sent to prison for a crime I did not commit.”
well began to call out for help. Krishnan felt sorry for the man and Krishnan told the King the whole story. The King was fuming
pulled him out of the well. “Thank you for your kindness”, said with rage when he heard what the goldsmith had done. He at
the man. “I am Seth Ghanshyamdas. I am a goldsmith. If you ever once had the goldsmith arrested. The King then presented
need my help, don’t hesitate to visit my humble house near the Krishnan with a large house and a thousand pieces of gold.
city.” The goldsmith then left for home. Krishnan sent for his family and they all lived happily ever after.
After some time, the Brahmin continued his journey. But he 71. Why did Krishnan decide to leave his village?
could not find any work. He then remembered Shersingh, Bali, (a) As he could not find much work in his own willage and
Naagesh and Seth Ghanshyamdas. He thought it was time to his family had to starve sometimes because of it.
seek their help. He first went to Bali. The monkey was overjoyed (b) As his family had requested him to do so.
to see him. He gave him a warm welcome and offered him some (c) As his village people had asked him to leave their village
really delicious fruits. The Brahmin told him how grateful he was. and look for work somewhere else.
Now Krishnan went to see Shersingh, the jaguar. As soon as
(d) As he wanted to search for food in a village different
Shersingh saw Krishnan coming, he ran out to welcome him. He
from his own.
gave Krishnan a beautiful gold necklace and other precious
jewellery. Krishnan thanked Shersingh for the jewellery and (e) None of the above
departed. His journey had at last brought him luck, he thought. He 72. Why did the jaguar, the monkey and the snake tell Krishnan
would be able to sell the ornaments for a good price. But who could not to save the man in the well?
help him to sell the ornaments? He then remembered Seth (a) As the man in the well was a goldsmith
Ghanshyamdas. He went to him. The goldsmith was glad to see (b) As the man in the well had cheated the snake, the
Krishnan. “I have come to ask for your help”, said Krishnan. “Here monkey and the jaguar
are some ornaments. Please give me a good price for them.” Seth (c) As the man in the well was a thief
Ghanshyamdas took the jewellery and examined it carefully. “I shall (d) As the snake, the monkey and the jaguar hated the man
certainly help you”, he said. “But let me show them to another as they had known him for a very long time
goldsmith. Please wait here, I will be right back.” He then went out (e) None of the above
www.newspaperkorner.wordpress.com
Practice Set - 1 PS-7
www.newspaperkorner.wordpress.com
73. Why was krishnan afraid to save Naagesh from the well? (d) He asked his guards to immediately look for someone
(a) As Naagesh had thr eatened h im with dire who could cure the Queen
consequences. (e) None of the above
(b) As he thought Naagesh would eat him. 80. What can possibly be the moral of the story?
(c) As he thought Naagesh would bite him once he was (a) Trust oneself before trusting overs
out of the well. (b) A good deed never goes in vain
(d) As he thought that Naagesh would capture him as soon (c) You cannot change people but you can change yourself
as he got out of the well. (d) Try and try until you succed
(e) None of the above. (e) One must be the change one wishes to see in this world
74. Why did Krishnan go to meet Seth Ghanshyamdas?
(a) As he thought that Seth Ghanshyamdas could help DIRECTIONS (Qs. 81-85): In each question below, a sentence
him in selling the ornaments gifted to him by Shersingh. with four words printed in bold type is given. These are numbered
(b) As he knew that Seth Ghanshyamdas had contact with as (a), (b), (c) and (d). One of these four words printed in bold may
the King which could prove to be beneficial. be either wrongly spelt or inappropriate in the context of the
(c) As Seth Ghanshyamdas had requested krishnan to sell sentence. Find out the word which is wrongly spelt or
ornaments only to him inappropriate, if any. The number of that word is your answer. If
(d) As Krishnan was extremely fond of Seth Ghanshyamdas all the words printed in bold are correctly spelt and also
(e) None of the above. appropriate in the context of the sentence, mark (e) ie. ‘All
75. What did Bali do after seeing Krishnan at his house? correct’ as your answer.
(1) He gave Krishnan directions to Shersingh’s house. 81. The whole (a)/ time she walked with her child in her arms,
(2) He welcomed Krishnan to his house. the only thing (b)/ that worried (c)/ her was her son’s
(3) He offered tasty fruits to Krishnan. feature. (d)/ All correct (e)
(a) Only 1 (b) Only 2 82. When the young artist returned (a)/ to his village, his
(c) Only 3 (d) Only 2 and 3 family held a festive (b)/ dinner on its lawn to celebrate his
(e) 1 and 3 triumpant (c)/ homecoming. (d)/ All correct (e)
76. What plan did Naagesh have to save Krishnan from the
83. Had she not suppressed (a)/ all the details of her Company’s
dungeon?
(a) That he would sneak Krishnan out of the dungeon project (b)/ her Company would have bagged (c)/ the
without anyone noticing contract. (d)/ All correct (e)
(b) That he would bite the King and make him unconscious 84. She trusted Mira with all her heart (a)/ and thus handled
(c) That he would bite Krishnan and make everyone believe (b)/ over her life’s (c)/ savings to her instantly. (d)/ All
that he was dead correct (e).
(d) That he would enter the Queen’s chamber and scare 85. It is difficullt (a)/ to see the picture (b)/ when you are inside
her (c)/ the frame. (d)/ All correct (e)
(e) None of the above
77. What did Seth Ghanshyamdas tell the King about Krishnan? DIRECTIONS (Qs. 86-95): In the following passage there are
(a) That Krishnan had brought fake ornaments for selling blanks, each of which has been numbered. these numbers are
(b) That krishnan was an honest Brahmin who had left his printed below the passage and against each, five words are
village suggested, one of which fits the blank appropriately. Find out
(c) That Krishnan had killed the Prince the appropriate word in each case.
(d) That Krishnan had brought those ornaments for selling
which had been made for the missing Prince One day a father of a very wealthy family (86) his son on a
(e) None of the above trip to the country with the purpose of (87) his son how the poor
78. What did the King do on learning the truth about Krishnan people live so he could be thankful for his wealth. They spent a
and Seth Ghanshyamdas? (88) of days and nights on the farm of what would be considered
(a) He put Krishnan back in the dungeon as he still held a (89) poor family. On their (90) from the trip, the father asked his
Krishnan responsible for the Prince’s death son, “How was the trip?" "It was great, Dad.” “Did you see how
(b) He called for Krishnan’s wife and family poor people can be?”, the father asked. “Oh yeah”, said the son.
(c) He presented gold to Krishnan and also a house to live in So what did you (91) from the trip?”, asked the father. The son
(d) He congratulated the snake on his efforts to save answered, “I saw that we have one dog and they had four. We
Krishnan have a pool that (92) in the middle of our garden and they have a
(e) None of the above creek that has no end.” “We have imported lanterns in our garden
79. What did the King do to save the Queen after even the and they have the stars at night.” “Our patio reaches to the front
Vaidyas failed to revive her? yard and they have the (93) horizon.” “we have a small piece of
(a) He punished the snake for having harmed the Queen land to live on and they have fields that go beyound our sight.”
(b) He announced a reward to anyone who could cure the “We have (94) who serve us, but they serve others.” “We buy
Queen our food, but they grow theirs.” “We have walls around our (95)
(c) He immediately called for Krishnan to cure the Queen to protect us; they have friends to protect them.”
www.newspaperkorner.wordpress.com
PS-8 Practice Set - 1
www.newspaperkorner.wordpress.com
With this the boy’s father was speechless. Then his son DIRECTIONS (Qs. 96 - 100): In each of the following sentences,
added, “Thanks dad for showing me how poor we are”. an idiomatic expression or a proverb is highlighted. Select the
86. (a) took (b) beat alternative which best describes its use in the sentence.
(c) drag (d) mould 96. He resigned the post of his own accord.
(e) showed (a) which he liked
87. (a) presenting (b) requesting (b) according to his convenience
(c) tell (d) trusting (c) voluntarily and willingly
(e) showing (d) according to his judgement
88. (a) two (b) couple (e) None of these
(c) much (d) few 97. As a politician he is used to being in the limelight all the time.
(e) many (a) giving speeches
89. (a) major (b) some (b) the object of admiration
(c) sorrow (d) very (c) the centre of attraction
(e) astutely (d) an object of public notice
90. (a) lane (b) journey (e) None of these
(c) leave (d) return 98. I ran out of money on my European tour.
(e) walking (a) exhausted my stock of (b) did not have enough
91. (a) reveal (b) think (c) lost (d) carried a lot
(c) saw (d) believe (e) None of these
(e) learn 99. Madhuri might scream blue murder, but I feel Deepali
92. (a) stands (b) reaches should get the promotion since she is better qualified for
(c) swims (d) leak the job.
(e) watery (a) someone has been murdered with some blue liquid
93. (a) more (b) scene (b) someone is being murdered and has become blue
(c) whole (d) last (c) suffer from persecution complex
(e) lucky (d) make a great deal of noise and object vehemently
94. (a) servants (b) mother (e) None of these
(c) computers (d) relatives 100. In modern democratic societies lynch law seems to have
(e) man become the spheres of life.
95. (a) minds (b) selves (a) law of the mob (b) law of the underworld
(c) property (d) pillars (c) law of the constitution(d) law of the parliament
(e) country (e) None of these

www.newspaperkorner.wordpress.com
Practice Set - 1 PS-9
www.newspaperkorner.wordpress.com
Answer Key
1 (c) 11 (d) 21 (b) 31 (c) 41 (a) 51 (c) 61 (d) 71 (a) 81 (d) 91 (e)
2 (b) 12 (a) 22 (d) 32 (b) 42 (d) 52 (d) 62 (a) 72 (e) 82 (c) 92 (a)
3 (c) 13 (d) 23 (c) 33 (d) 43 (a) 53 (c) 63 (b) 73 (c) 83 (a) 93 (c)
4 (d) 14 (c) 24 (a) 34 (b) 44 (a) 54 (d) 64 (e) 74 (a) 84 (b) 94 (a)
5 (d) 15 (b) 25 (c) 35 (c) 45 (b) 55 (b) 65 (c) 75 (d) 85 (a) 95 (c)
6 (c) 16 (b) 26 (d) 36 (b) 46 (a) 56 (c) 66 (c) 76 (e) 86 (a) 96 (c)
7 (d) 17 (a) 27 (b) 37 (e) 47 (c) 57 (e) 67 (b) 77 (d) 87 (e) 97 (c)
8 (d) 18 (a) 28 (e) 38 (b) 48 (e) 58 (a) 68 (a) 78 (c) 88 (b) 98 (a)
9 (a) 19 (a) 29 (a) 39 (b) 49 (e) 59 (b) 69 (d) 79 (b) 89 (d) 99 (d)
10 (b) 20 (c) 30 (a) 40 (b) 50 (b) 60 (d) 70 (e) 80 (b) 90 (d) 100 (a)

Answers & Explanations


( a3 - b3 ) x x
, 9. (a) Let = 550. Then, = 550
1. (c) Given Expression = 2.25 1.5
(a 2 + ab + b 2 )
where a = 117, b = 98 æ 550 ´ 15 ö
\ x = (550 ´ 1.5) = ç = 825
è 10 ÷ø
(a - b)(a 2 + ab + b 2 )
= = (a - b) = (117 – 98) = 19.
(a 2 + ab + b 2 ) ( 5 - 3) ( 5 - 3) ( 5 - 3) ( 5 - 3)2
10. (b) = ´ =
2. (b) Dividing numerator as well as denominator by b, we get: ( 5 + 3) ( 5 + 3) ( 5 - 3) (5 - 3)
a 4
3´ + 2 3´ + 2 5 + 3 - 2 15 2(4 - 15)
3a + 2b b 3 4+2 = = = (4 - 15)
= = = =3 2 2
3a - 2b 3 ´ a - 2 3 ´ 4 - 2 4 - 2
b 3 16
11. (d) Population of village B in 1995 = 5000 ´ » 6150
13
x 72 9
3. (c) Let = = .
196 56 7 110
Population of village B in 1996 = 6150 ´ = 6750
100
x 9 9 81 81 ´ 196
Then , = ´ = . So, x = = 324. Population below poverty line = 52% of 6750 » 3500
196 7 7 49 49
17
17.28 ¸ x 17.28 12. (a) Population of village D in 1995 = 9,000 × = 10, 200
4. (d) Let = 200 . Then, = 200 ´ 3.6 ´ 0.2 15
3.6 ´ 0.2 x
17.28 1728 110
\ x= = = 0.12 Population of village D in 1997 = 10, 200 ´
200 ´ 3.6 ´ 0.2 200 ´ 36 ´ 2 100
= 11, 220
(a - b)2 + (a + b)2 2(a2 + b2 )
5. (d) Given Expression = = =2 95
(a2 + b2 ) (a2 + b2 ) Population of village G in 1997 = 9,000 ´ = 8,550
100
240 ´ 300 \ Total population of village D and G in 1997
6. (c) Given Expression = = 300
240 = 11,220 + 8,550 = 19, 770
25 13. (d) Population of village F below poverty line
50 x 2
7. (d) Let = or x = 50 ´ = 625.
x æ 25 ö 2 13 49
çè ÷ø = 55000 ´ ´ » 3500
2 100 100
14. (c) Population of village F in 1995
\ x = 625 = 25.
æ 112 24 16 ö 100 13
´ ´ ÷ = 32 = 1520 ´ ´ = 6500
8. (d) Given Expression = çè 38 8
14 12 8 ø
www.newspaperkorner.wordpress.com
PS-10 Practice Set - 1
www.newspaperkorner.wordpress.com
15. (b) Population of village C below poverty line 3 4 10 32 136 685 4116
38 22. (d)
= 2000 ´ = 760 ×1+1 ×2+2 ×3+3 ×4+4 ×5+5 ×6×6
100
Thus, 32 is out of place and must be replaced by 33.
Population of village E below poverty line
5
2000 æ 46 ö
= ´ 18 ´ ç = 2070 23. (c)
8 è 100 ÷ø
6 × 9 + 1 5 × 5 + 1 2 × 6 +1 1 × 3 + 1
Thus, 5 does not fit in the series and should be
760
\ Required ratio = = 76 : 207 replaced by 4.
2070 24. (a) The succeeding numbers are obtained by dividing the
preceding numbers by 4. Therefore, the number 386
132 ´ 5
16. (b) Speed of the train = 132 km/h = m/s does not fit in the series and must be replaced by 384.
18 25. (c) There are two series in the given series :
Distance = (110 + 165) = 275 m I 5 12 26 54
Time required to cross the railway platform
275 ´ 18 ×2+2 ×2+2 ×2+2
= = 7.5 s
132 ´ 5 II 11 20 40 74
17. (a) 15 W = 10 M
×2–2 ×2–2 ×2–2
4 ´ 15 Hence the wrong term is 40.
Now, 5W + 4M = 5W + W = 5W + 6W
10 26. (d) ? = 45689 = 213.75 » 210
= 11 W
Now, 15 women can complete the project in 55 days, (10008.99)2
27. (b) ? = ´ 3589 ´ 0.4987
then 11 women can complete the same project in 10009.001
55 ´15 = (10009) 2 ´ 3600 = 0.50
= 75 days
11 = 10009 × 60 × 0.50 » 300000
18. (a) Let the present ages of Ashu’s mother and that of 28. (e) ? = 399.9 + 206 × 11.009
Ashu be x and y, respectively. = 400 + (200 + 6) × 11 = 400 + 2200 + 66 = 2670
Then, (x–5) = 3(y –5) or x – 5 = 3y – 15 2 7 17 6 2 7 17 5
or x – 3y = – 10 ...(i) 29. (a) ?= + ´ ¸ = + ´ ´
5 8 19 5 5 8 19 6
and (x + 5) = 2 (y + 5)
And x + 5 = 2y + 10 or x – 2y = 5 ...(ii) 2 595
= + = 0.40 + 0.65 » 1.05 » 1
From (i) and (ii), we have x = 35 and y = 15 5 912
Hence, the present age of Ashu = 15 years 30. (a) ? = (299.99999)3 » (300)3 = 27000000
19. (a) Volume of the conical flask = Volume of the cylindrical 31. (c) Total amount used for purchasing = ` 160. A reduction
flask upto the required height (x) cm of 20% in the price means, now a person gets 5/2 kg
for ` 32 and this is the present price of the sugar.
1 2 2
pa h = pp 2 ´ x Þ x = ha cm 32
3 3p 2 \ Present price per kg = ´ 2 = ` 12.8
5
20. (c) Let the sum = Rs. x and original rate = y % per annum Let the original price be ` x. Then new price is arrived
then, New rate = (y + 3)% per annum after reduction of 20% on it.
x ´ ( y + 3) ´ 2 x ´ y ´ 2 Þ x × 0.8 = 12.8 or x = ` 16.
\ - = 300 32. (b) Mrs. X spends = ` 535
100 100
\ Total cost = 43 shirt + 21 ties = 535
xy + 3x - xy = 15000 By hit and trial, S = 10, T = 5
Þ Total cost = 43 × 10 + 21 × 5 = 535
\ x = 5000 Thus, the sum = ` 5000 Hence, Ratio of shirts to ties = 10 : 5 = 2 : 1
33. (d) Total expense percentage = (25 + 5 + 15 + 10)% = 55%
×2 ×2
Savings % = 100 – 55 = 45%
21. (b) 2 3 4 4 6 8 8 12 16 Q 45 º 22500
×2 ×2 22500
×2 ×2 \ 100% º ´ 100 = ` 50000
45
www.newspaperkorner.wordpress.com
Practice Set - 1 PS-11
www.newspaperkorner.wordpress.com
34. (b) Let Anil's salary be ` x. 44. (a) J O U R NE Y
2x –1 +1 +1
\ Bhuvan's salary = `=
5 T N I S Z F O
2 x 7 14 x Similarly,
Chandra's salary = `= ´ =
5 9 45 ME D I C A L
2 x 14 x –1 +1 +1
\ Anil : Bhuvan : Chandra = x : : = 45 :18 :14
5 45 C D L J MB D
\ Bhuvan's salary 45. (b) 40 B 8 T 6 M 3 K 4 = ?
é 18 ù Þ ? = 40 + 8 – 6 + 3 × 4
= `= ê 45 + 18 + 14 ´ 77000ú = ` 18000
ë( ) û Þ ? = 5 + 6 – 12 = 11
35. (c) Part of the tank filled in an hour 46. (a)
Tables
1 1 5–3 1 Chairs ers
= – = = aw
Dr
12 20 60 30
Hence, the tank will be filled in 30 hours
Hence conclusions I. II. III.
36. (b) There are 25 numbers in the given sequence.
So, middle number = 13th number = 8.
Trees
Clearly, the third number to the left of this 8 is 2. Leaves
47. (c) rs Buds
37. (e) Coding for: I D E A S we
F lo
–1¯ +1¯ –1¯ +1¯ –1¯
Hence conclusions I. II. III.
H E D B R
But I and II are complementary pairs.
Coding for: W O U L D
–1¯ +1¯ –1¯ +1¯ –1¯
Stone
V P T M C
48. (e) Cement
s Bricks
Similarly, R I G H T ck
Ro
–1¯ +1¯ –1¯ +1¯ –1¯ Hence conclusions I. II. III.
Q J F I S
38. (b) Cancelling every second letter after reversing the 49. (e)
alphabet the series becomes.
ZXVT RPNLJHFDB
The middle letter is N.
39. (b) Total no. of girls = 17 + 10 – 1 or 18 + 9 – 1 = 26. Flats
40. (b) From first 2 sentences ‘Ka Ya’ means ‘very intelligent’.
From 1st and 3rd sentences ‘Pu’means ‘you’ Buildings
\ In first sentence ‘are’ means ‘Bi’
Bunglows

41. (a) Daizy Binny Aruna Apartments


(Binny’s sister (Binny’s sister)
and Chinky’s Mother) Hence conclusions I. II. III.

50. (b)

Chinky
Spectacles
42. (d) 1st of month was Tuesday, hence the date on first Frames
Saturday was 5th.
Hence the other Saturdays of the month are 12, 19,
26. Rama met her brother on 26th. es
Lenc
als
Met
43. (a) 5 3 1 6 9 7
576 = 24 ×24 Hence conclusions I. II. III.
www.newspaperkorner.wordpress.com
PS-12 Practice Set - 1
www.newspaperkorner.wordpress.com
For (Qs. 51 to 55) Middle element and last element also follow the same
trend. Hence (?) should be replaced by R 13.
C 65. (c) Number of elements between B and Q is the same as
(B Man) that between E and R. Similarly, number of elements
between A and P is the same as that between K and A.
Wif For (Qs. 66 - 70) : The given information can be tabulated as
B e D
(Ind) follows
Sis
ter
Wife Pers on S ex Relations hip Profes sion
(Pro) A E L Female W ife of M A rchitect

A, the professor is married to E and E is the sister of B. Father of P. Business man


The wife of the industrialist is D. The industrialist is seated M M ale Husband of L.
to the right of C. Thus, A and C cannot be industrialists.
Son of Q and O.
Therefore, B is the industrialist and C is the business man.
Now, we come to the following deductions : Female Daughter-in-law of
N Hous ewife
A – Professor B – Industrialist O and Q.
C – Businessman D – Female, hence unemployed O Female W ife of Q Pilot
E – Female, hence unemployed P M ale Son of M and L. Journalis t
51. (c) A is the husband of E and E is the sister of B. Hence,
A is the brother-in-law of B. Grandfather of P. Doctor
52. (d) It cannot be determined, as no information has been Q M ale Husband of Q.
provided in the paragraph about the sitting position
Father of M .
of A.
53. (c) As deduced earlier, B is the industrialist. R – – A dvocate
54. (d) It cannot be determined whether the businessman is
married or unmarried. Two married couples : LM and QO.
55. (b) As A is a professor, he must be a graduate. 71. (a) Refer to the sentence ‘He could not find.........in search
56. (c) B < D ...(i), E £ T ...(ii), T ³ P ....(iii), P > B...(iv) From of work’.
(i) and (iv), we get, P > B < D Þ no conclusion. But 72. (e) Refer to the sentence “Please do not help him”..........of
the second para of the passage.
the exhaustive possibilities are P > D , P = D and 73. (c) Refer to the sentence “You are...........bite me?”........of
P < D. Hence either I or II is true. the second para of the passage.
57. (e) E ³ F ...(i), G £ H ...(ii), H = E ...(iii), G > K ...(iv) 74. (a) Refer to the sentence “He would
From (ii) and (iv), we get, H ³ G > K Þ H > K . be.......Ghanshyamdas” .......of the third para of the
passage.
Hence I is true.
75. (d) Refer to the sentence, “He gave him........delicious
From (i) and (iii), we get, H = E ³ F Þ H ³ F . Hence fruits”........of the third para of the passage.
II is true. 76. (e) Refer to the sentence “I shall creep........bite her........
58. (a) P £ Q ...(i) , N = M ....(ii), M > R ...(iii) , R ³ P ...(iv) hand on her forehead” of the fourth para of the passage.
77. (d) Refer to the sentence, “A man brought .......prince who
From (ii), (iii) and (iv), we get, N = M > R ³ P
is missing”........of the third para of the passage.
Þ N > P or P < N . Hence I is true. 78. (c) Refer to the sentence “He at once........pieces of gold”
From (ii), (iv) and (i), we get, M > R ³ P £ Q Þ No of the second last sentence of fourth para of the
passage.
conclusion about the relationship between M and Q
79. (b) Refer to the sentence “Finally, the king
can be established.
declared........handsomely rewarded” of the fourth para
61. (d) Other groups consist of consecutive elements.
of the passage.
62. (a) 2 + 22 + 8 + 5 + 6 + 0 + 9 + 92 + 0 + 3 = 118 80. (b) The moral of the story can possibly be “A good deed
Where value of alphabet = 0; symbol = square of the never goes in vain”.
previous number; number = the value itself. 81. (d) It should be ‘here was her son’s future’.
63. (b) The letters used in the series are B Q E R D U K A P. 82. (c) The correct spelling should be ‘triumphant’.
Sum of the positions of consonants according to 83. (a) The word suppressed should be replaced by revealed
alphabet = 2 + 17 +18 + 4 + 11 + 16 = 68. Similarly, sum or leaked in the sentence.
of the vowels’ positions = 5 + 21 + 1 = 27. Hence required 84. (b) It should be ‘handed over’ which means the act of
difference = 68 – 27 = 41. moving power or responsibility from one person to
64. (e) First element of each group follows the order + 3, + 3, another.
+3... and so on. 85. (a) The correct spelling is difficult.
www.newspaperkorner.wordpress.com
www.newspaperkorner.wordpress.com

Preliminary Exam
Practice Set - 2
INSTRUCTIONS
• This Preliminary Exam practice set consists of three sections. Quantitative Aptitude (Qs. 1-35); Reasoning
Ability (Qs. 36-70) and English Language (Qs. 71-100).
• All the questions are compulsory.
• Each question has five options, of which only one is correct. The candidates are advised to read all the
options thoroughly.
• There is negative marking equivalent to 1/4th of the mark allotted to the specific question for wrong answer.

Time : 1 hrs. Max. Marks : 100

QUANTITATIVE APTITUDE
(7 + 7 + 7) ¸ 7
6. =?
DIRECTIONS (Qs. 1-10): What will come in place of question 5+5+ 5¸5
mark (?) in the following questions?
1
137 ´ 137 + 137 ´ 133 ´ 133 ´ 133 (a) 1 (b)
1. =? 5
137 ´ 137 ´ 137 - 133 ´ 133 ´ 133
(a) 4 (b) 270 15 3
(c) (d)
1 1 11 11
(c) (d)
4 270 (e) None of these
(e) None of these
7. 33124 ´ 2601 – (83)2 = (?)2 + (37)2
2. If n
3 = 81 , then n = ? (a) 37 (b) 33
(a) 2 (b) 4
(c) 6 (d) 8 (c) 34 (d) 28
(e) None of these (e) None of these

3. ë( ) ( û )
é 5 7 + 7 + 4 7 + 8 7 ù - (19) 2 = ?
8. 5
17 51 1 3
´ 4 ´11 + 2 = ?
37 52 7 4
(a) 143 (b) 72 7
(a) 303.75 (b) 305.75
(c) 134 (d) 70 7
3 1
(e) None of these (c) 303 (d) 305
4. (4444 ÷ 40) + (645 ÷ 25) + (3991÷ 26) = ? 4 4
(a) 280.4 (b) 290.4 (e) None of these
(c) 295.4 (d) 285.4
(e) None of these 32 + 48
9. =?
.23 - .023 8 + 12
5. =?
.0023 ¸ 23 (a) 2 (b) 2
(a) 0.207 (b) 207 (c) 4 (d) 8
(c) 2070 (d) 0.0207
(e) None of these
(e) None of these
www.newspaperkorner.wordpress.com
PS-14 Practice Set - 2
www.newspaperkorner.wordpress.com
18. The sum of digits of a two digit number is 15. If 9 be added
1
10. =? to the number, then the digits are reversed. The number is
9- 8
(a) 96. (b) 87
1 1 (c) 78 (d) 69
(a) (3 - 2 2) (b)
2 3+ 2 2 (e) None of these
(c) 3 - 2 2 (d) 3 + 2 2 19. Three cubes of a metal are of edges 3 cm, 4 cm and 5 cm.
These are melted together and from the melted material,
(e) None of these
another cube is formed. The edge of this cube is :
11. If the compound interest on a certain sum of money for
3 years at 10% p.a. be ` 993, what would be the simple (a) 8 cm (b) 10 cm
interest ? (c) 9 cm (d) 6 cm
(a) ` 800 (b) ` 950 (e) None of these
(c) ` 900 (d) ` 1000 2
20. Yashika got married 8 years ago. Today her age is 1 times
(e) None of these 7
12. In a class, 20 opted for Physics, 17 for Maths, 5 for both and her age at the time of her marriage. At present her daughters
10 for other subjects. The class contains how many age is one-sixth of her age. What was her daughter’s age 3
students? years ago ?
(a) 35 (b) 42 (a) 2 years (b) 3 years
(c) 52 (d) 60 (c) 4 years (d) 5 years
(e) None of these (e) None of these
13. How much water must be added to 100 cc of 80% solution
DIRECTIONS (Qs. 21-25) : In each of the following questions a
of boric acid to reduce it to a 50% solution ?
number series is given. A number in the series is expressed by
(a) 20 cc (b) 40 cc
letter ‘N’. You have to find out the number in the place of ‘N’ and
(c) 80 cc (d) 60 cc use the number to find out the value in the place of the question
(e) None of these mark in the equation following the series.
14. Successive discounts of 20% and 15% are equivalent to a
single discount of 21. 68 68.5 69.5 71 N 75.5 78.5
(a) 35% (b) 32% N × 121 + ? = 10000
(c) 17.5% (d) 22.5% (a) 1160 (b) 1200
(e) None of these (c) 1150 (d) 1180
15. The dimensions of a rectangular room when increased by (e) None of these
4 metres are in the ratio of 4 : 3 and when decreased by 22. 19 20 24 33 49 74 N 159
4 metres, are in the ratio of 2 : 1. The dimensions of the room are N2 ¸ 10000 = ?
(a) 6 m and 4 m (a) 121.0 (b) 12.1
(b) 12 m and 8 m (c) 1.21 (d) 0.121
(c) 16 m and 12 m (e) None of these
(d) 24 m and 16 m 23. 51 43 N 30 25 21 18
(e) None of these N2 – 2N = ?
16. Two cars start together in the same direction from the same (a) 1155 (b) 1224
place. The first goes with a uniform speed of 10 km/h. The (c) 1295 (d) 1368
second goes at a speed of 8 km/h in the first hour and (e) None of these
1 24. 2 5 14 41 122 365 N
increases its speed by km with each succeeding hour..
2 2
After how many hours will the second car overtake the first N - 16 % of 5670 – (?)2 = 102
3
one, if both go non-stop?
(a) 9 hours (b) 5 hours (a) 7 (b) 149
(c) 7 hours (d) 8 hours
(c) 49 (d) 7
(e) None of these
17. 24 men working 8 hours a day can finish a work in 10 days. (e) None of these
Working at the rate of 10 hours a day, the number of men 25. 510 254 N 62 30 14 6
required to finish the same work in 6 days is 40% N + ? = 92
(a) 30 (b) 32 (a) 31.4 (b) 29.8
(c) 34 (d) 36 (c) 50.4 (d) 30.6
(e) None of these (e) None of these
www.newspaperkorner.wordpress.com
Practice Set - 2 PS-15
www.newspaperkorner.wordpress.com
DIRECTIONS (Qs. 26-30): Find out the approximate value which 35. The total cost of 12 apples and 8 guavas is ` 76 and the
should replace the question mark (?) in the following questions. total cost of 8 apples and 12 guavas is `64. What is the total
(You are not expected to find out the exact value). cost of one apple and one guava?
(a) ` 5 (b) ` 7
26. 196.1 × 196.1 × 196.1 × 4.01 × 4.01 × 4.001 × 4.999 × 4.999
(c) ` 8 (d) ` 10
= 196.13 × 4 × ?
(e) None of these
(a) 100 (b) 16
REASONING ABILITY
(c) 10 (d) 64
(e) 32 36. If the following series is written in the reverse order, which
2 1 3 6 number will be fourth to the right of the seventh number
27. ´ + ¸ =? from the left ?
7 8 7 14
7, 3, 9, 7, 0, 3, 8, 4, 6, 2, 1, 0, 5, 11, 13
2 3 (a) 0 (b) 5
(a) (b) (c) 9 (d) 11
56 56
(c) 1 (d) 2.5 (e) None of these
37. In a certain code language ‘ne ri so’ means ‘good rainy
50 day’, ‘si ne po’ means ‘day is wonderful’ and ‘ri jo’ means
(e)
60 ‘good boy’. Which of the following means ‘rainy’ in the
28. 10.12.01 + 2.93.001 = ? code?
(a) 130 (b) 160 (a) ne
(c) 115 (d) 147 (b) si
(e) None of these (c) ri
29. 1999.9997 = 4.76 ´ ? (d) s o
(a) 11 (b) 45 (e) None of these
(c) 49 (d) 6 38. If SMOOTH is coded as 135579, ROUGH as 97531 and
(e) 9 HARD as 9498, then SOFT will be coded as
(a) 1527 (b) 1347
1 (c) 4998 (d) 8949
30. 23% of 4011 + of 5555 = ?
7 (e) 8994
(a) 7000 (b) 1900 39. Saroj is mother-in-law of Vani who is sister-in-law of Deepak.
(c) 9022 (d) 1700 Rajesh is father of Ramesh, the only brother of Deepak.
(e) 1450 How is Saroj related to Deepak?
31. Average age of 36 children of the class is 15 years. 12 more (a) Mother-in-law (b) Wife
children joined whose average age is 16 years. What is the (c) Aunt (d) Mother
average age of all the 48 children together ? (e) None of these
40. A directional post is erected on a crossing. In an accident,
(a) 15.25 years (b) 15.5 years
it was turned in such a way that the arrow which was first
(c) 15.3 years (d) 15.4 years
showing east is now showing south. A passerby went in a
(e) None of these wrong direction thinking it is west. In which direction is he
32. Profit earned by selling an article of ` 1,450 is same as the actually travelling now ?
loss incurred by selling the article for ` 1,280. What is the (a) North
cost price of the article? (b) South
(a) ` 1,385 (b) ` 1,405 (c) East
(c) ` 1,355 (d) ` 1,365 (d) West
(e) None of these (e) None of these
33. Two-fifth of two-third of a number is 48. What is 30% of the DIRECTIONS (Qs. 41 - 45): In each question below, there are
number? three statements followed by two conclusions numbered I and II.
(a) 60 (b) 56 You have to take the three given statements to be true even if
(c) 180 (d) 210 they seem to be at variance from commonly known facts and then
decide which of the given conclusions logically follows from the
(e) None of these
three statements disregarding commonly known facts.
34. In a hall 192 children are made to sit in rows and columns
Give answer (a) if only conclusion I follows.
and no. of rows is more than the no. of columns by 4. What
is the no. of children in each column ? Give answer (b) if only conclusion II follows.
(a) 16 (b) 12 Give answer (c) if either I or II follows.
(c) 14 (d) 18 Give answer (d) if neither I nor II follows.
(e) None of these Give answer (e) if both I and II follow.
www.newspaperkorner.wordpress.com
PS-16 Practice Set - 2
www.newspaperkorner.wordpress.com
41. Statements: All shoes are pens. 50. Which of the following is a group of brothers ?
Some pens are razors. (a) ABF (b) ABD
Some razors are desks. (c) BFC (d) BDF
Conclusions : (e) None of these
I. Some desks are shoes. DIRECTIONS (Qs. 51-55) : Study the following paragraph and
II. Some razors are shoes. then answer the questions that follow.
42. Statements:
Five golfers C, D, E, F and G play a series of matches in which the
Some benches are windows.
following are always true of the results. Either C is the last and G
Some windows are walls. is the 1st or C is the 1st and G is the last. D finishes ahead of E.
Some walls are trains. Every golfer plays in and finishes every match. There are no ties
Conclusions: in any match, i.e. no two players ever finish in the same position
I. "Some trains are benches." in a match.
II. No train is bench. 51. Which of the following cannot be true ?
43. Statements : (a) E finishes second.
All brushes are chocolates. (b) F finishes second.
All chocolates are mirrors. (c) E finishes ahead of F.
All mirrors are tables. (d) F finishes ahead of D.
Conclusions: (e) None of these
I. Some tables are brushes 52. If D finishes third, then which of the following must be
II. Some mirrors are chocolates. true?
(a) G finishes first.
44. Statements :
(b) E finishes ahead of F.
Some pencils are knives.
(c) F finishes ahead of E.
All knives are papers.
(d) F finishes behind D.
Some papers are books. (e) None of these
Conclusions: 53. If C finishes first, then in how many different orders is it
I. Some books are pencils. possible for the other golfers to finish?
II. Some papers are pencils. (a) 1
45. Statements: (b) 2
Some roofs are figures. (c) 3
All figures are lions. (d) 4
All lions are goats. (e) None of these
Conclusions: 54. Which of the following additional conditions make it certain
I. Some goats are roofs. that F finishes second ?
II. All goats are figures (a) C finishes ahead of D.
(b) D finishes ahead of F.
DIRECTIONS (Qs. 46-50) : Study the following information
(c) F finishes ahead of D.
carefully to answer the questions that follow.
(d) D finishes behind G.
There are six persons A, B, C, D, E and F. C is the sister of F. B is
(e) None of these
the brother of E’s husband. D is the father of A and grandfather of
55. If exactly one golfer finishes between C and D, then which
F. There are two fathers, three brothers and a mother in the group.
46. Who is the mother ? of the following must be true?
(a) A (b) B (a) C finishes first.
(c) D (d) E (b) G finishes first.
(e) None of these (c) F finishes third.
47. Who is E’s husband ? (d) E finished fourth.
(a) B (b) C (e) None of these
(c) A (d) F
(e) None of these DIRECTIONS (Qs. 56-60): In the following questions the symbols
48. How many male members are there in the group? @—
@ , = © and —
Ó are used with the following meaning:
(a) One (b) Two
(c) Three (d) Four P © Q means P is less than Q.
(e) None of these P @ Q means P is greater than Q.
49. How is F related to E ? P —
@ Q means P is greater than or equal to Q.
(a) Uncle (b) Husband
P = Q means P is equal to Q.
(c) Son (d) Daughter
(e) None of these Ó Q means P is either smaller than or equal to Q.
P —
www.newspaperkorner.wordpress.com
Practice Set - 2 PS-17
www.newspaperkorner.wordpress.com
Now in each of the following questions, assuming the given 65. How many such vowels are there in the above sequence,
statements to be true, find which of the two conclusions I and II each of which is immediately preceded by a consonant and
given below them is/are definitely true? Give answer. immediately followed by a vowel?
(a) if only conclusion I is true. (a) None (b) One
(b) if only conclusion II is true. (c) Two (d) Three
(c) if either I or II is true. (e) More than three
(d) if neither I nor II is true, and DIRECTIONS (Qs. 66 - 70) : Study the wing information carefully
(e) if both I and II are true. and answer the questions given below:
56. Ó B
Statements: B @ V, K © C, C — A shopkeeper placed eight varieties of cloth pieces of
Conclusions : I. V @ C different colours viz. brown, pink, green, white, blue, violet, black
II. B @ K and yellow, in a row such that blue cloth is placed fifth from the
left end. Green is placed to the extreme right. White cloth is to the
57. Ó R
Statements : K @ T, S = K, T — second to the left of blue. Violet cloth is exactly in between blue
Conclusions : I. S @ R and white. Black is sixth to the left of green and fourth to the left
II. T = R of brown. Pink is not the third to the right of violet.
66. Which colour cloth is placed fifth to the left of yellow?
58. Statements : U = M, P —
@ U, M —
@B
(a) Violet (b) Brown
Conclusions : I. P = B (c) Green (d) White
II. P @ B (e) None of these
59. Statements: L —
@ N, J —
Ó P, P —
@L 67. Which of the following is definitely true?
(a) Green cloth is second to the right of yellow.
Conclusions : I. J = L (b) White cloth is placed fifth to the left of brown.
II. P = N (c) White cloth is placed exactly in between violet and
60. Statements: H —
@ G, D @ E, H = E black.
(d) Pink cloth occupies second position from the left end.
Conclusions : I. D @ H
(e) All are true.
II. G ©D
68. Which colour cloth is third to the left of Brown ?
DIRECTIONS (Qs. 61-65): Study the following sequence (a) Yellow (b) Blue
carefully and answer the questions given below: (c) Green (d) White
(e) Pink
M E 5 P B 2 A 7 K N 9 T R U 4 6 I J D F 1Q 3 W 8 V I S Z 69. Which of the following is the correct position' of Yellow
61. How many such numbers are there in the above with respect to Pink?
sequence, each of which is both immediately preceded by (a) Sixth to the right (b) Sixth to the left
and immediately followed by a consonant ? (c) Fifth to the right (d) Fourth to the left
(a) None (b) One (e) Fifth to the left
(c) Two (d) Three 70. Which colour cloth is to the immediate right of Pink?
(e) More than three (a) White (b) Blue
62. If the order of the first twenty letters/numbrs in the above (c) Black (d) Brown
sequence is reversed and the remaining letters/numbers (e) Green
are kept unchanged, which of the following will be the ENGLISH LANGUAGE
fourteenth letter/number from the right end after the
rearrangement? DIRECTIONS (Qs. 71-85) : Read the following passage carefully
and answer the questions given below it. Certain words/phrases
(a) B (b) 6
have been printed in bold to help you locate them while answering
(c) 2 (d) 1
some of the questions.
(e) None of these
63. Which of the follwing letter/number is the eighth to the left There was a country long time ago where the people would
change a king every year. The person who would become the
of the nineteenth letter/number from the left end?
king had to agree to a contract that he would be sent to an island
(a) N (b) T
after one year of his being a king.
(c) 1 (d) D
One king had finished his term and it was time for him to go
(e) None of these to the island and live there. The people dressed him up in
64. Four of the following five are alike in a certain way with expensive clothes and put him on an elephant and took him around
regard to their position in the above sequence and so form the cities to say goodbye to all the people. This was a moment of
a group. Which is the one that does not belong to that sadness for all kings who ruled for one year. After bidding farewell,
group? the people took the king to a remote island in a boat and left him
(a) WIQ (b) PAE there. On their way back, they discovered a ship that had sunk
(c) NR7 (d) 4JR just recently. They saw a young man who had survived by holding
(e) D16 on to a floating piece of wood. As they needed a new king, they
www.newspaperkorner.wordpress.com
PS-18 Practice Set - 2
www.newspaperkorner.wordpress.com
picked up the young man and took him to their country. They (b) He would be gifted with expensive clothes and
requested him to be king for a year. First he refused but later he jewellery
agreed to be the king. People told him about all the rules and (c) He would be sent on an island which required a lot of
regulations and that how he would be sent to an island after one work to be done
year. (d) He would be asked to buy an elephant and go to the
After three days of being a king, he asked the ministers if remote island himself
they could show him the island where all the other kings were (e) None of the above
sent. They agreed and took him to the island. The island was 73. What did the young man notice on his visit to the remote
covered with a thick Jungle and sounds of vicious animals were island after three days of being king?
heard coming out of it. The king went a little bit further to check. (a) That the animals on the island were too many to be
Soon he discovered dead bodies of all the past kings. He killed
understood that as soon as they were left on the island, the wild (b) That the island was very big
animals had come and killed them. (c) That the jungle on the island was full of animals that
The king went back to the country and collected 100 strong had killed all the previous kings placed on the island
workers. He took them to the island and instructed them to clean (d) That the island was very beautiful and clean
the jungle, remove all the deadly animals and cut down all excess (e) Not mentioned in the passage
trees. He would visit the island every month to see how the work 74. What happened to the island in the first month of the young
was progressing. In the first month, all the animals were removed king’s tenure?
and many trees were cut down. In the second month, the whole (A) The wild animals were removed from the island.
island was cleaned out. The king then told the workers to plant (B) The whole island was cleaned.
gardens in various parts of the island. He also took with himself (C) Many unnecessary trees were cut down.
useful animals like chickens, ducks, birds, goats, cows etc. In the (a) Only A (b) Only B
third month, he ordered the workers to build big houses and (c) Only C (d) B and C
docking stations for ships. Over the months, the island turned (e) A and C
into a beautiful place. The young king would wear simple clothes 75. What could be said about the island after the young king
and spend very little from his earnings as a king. He sent all the had made his servants work on it ?
earnings to the island for storage. When nine months passed like (a) The dangerous island had been turned to a beautiful
this, the king called the ministers and told them: “I know that I place
have to go the island after one year, but I would like to go there (b) The island still remained the same despite all the young
right now.” But the ministers didn’t agree to this and said that he king’s efforts
had to wait for another three months to complete the year. (c) The island was barely tolerable now
Three months passed and now it was a full year. The people (d) The island was safe from all types of attacks
dressed up the young king and put him on an elephant to take (e) None of the above
him around the country to say goodbye to others. However, this 76. Which of the following describes the young king correctly?
king was unusually happy to leave the kingdom. People asked (A) He was intelligent
him, “All the other kings would cry at this moment. Why is it that (B) He had foresight
you are laughing?” He replied, “Don’t you know what the wise (C) He was cunning
people say? They say that when you come to this world as a (a) Only A (b) A and B
baby, you are crying and everyone else is smiling. Live such a life (c) Only C (d) B and C
that when you die, you will be smiling and everyone around you (e) All A, B and C
will be crying. I have lived that life. While all the other kings were 77. What was the king’s request to the ministers after the
lost into the luxuries of the kingdom, I always thought about the completion of nine months?
future and planned for it. I turned the deadly island into a beautiful (a) That he should not be sent to the island after
abode for me where I can stay peacefully.” completion of one year
71. Why did the people of the kingdom change the king every (b) That he would like to go to the island immediately
year? (c) That he should be paid more in order to improve work
(a) As their first king had invented this system and had on the island
recorded it in the form of a contract (d) That he should be sent to the island after a year
(b) As they believed that the new king would bring better (e) None of the above
ideas to the kingdom. 78. How did the young king arrange for money on the island?
(c) As they wanted their king to relax on an island after (a) By selling a large amount of his property
one year of hard work (b) By spending all the money derived from his income as
(d) Not mentioned in the passage a king
(e) None of the above (c) By borrowing money from the ministers and sending it
72. What would happen to the king once his term of one year to the island
was over ? (d) By spending very less of his income as a king and
(a) He would be paraded in the cities as a farewell and sending it to the island for storage
then be taken to a remote island (e) None of the above
www.newspaperkorner.wordpress.com
Practice Set - 2 PS-19
www.newspaperkorner.wordpress.com
79. Why were the people of the kingdom puzzled when the (C) The ducklings hurried towards the lake and the mother duck
young king was taken around the country to say goodbye began to walk back and forth dragging one wing on the
to everyone? ground.
(a) As they could not believe that one year had elapsed (D) A mother duck and her little ducklings were on their way to
so soon the lake one day.
(b) As they were nor aware that the young king was (E) The fox stared in disbelief at the mother duck and her
actually a wise sage ducklings as he could not reach the ducklings because they
(c) As the young king was happy to go to the island unlike were in the middle of the lake by now.
the previous kings (F) When the fox saw her he became happy as he thought that
(d) Not mentioned in the passage the mother duck was hurt and couldn’t fly and that he could
(e) None of the above easily catch and eat her!
80. What can possibly be the moral of the story? 86. Which of the following should be the FIRST sentence after
rearrangement?
(a) Always put others before yourself
(a) A (b) B
(b) Give respect to others
(c) C (d) D
(c) Live in the present and forget about the future
(e) E
(d) Do not put things off until tomorrow
87. Which of the following should be the SECOND sentence
(e) Always think and plan ahead
after rearrangement ?
DIRECTIONS (Qs. 81-83): Choose the word/group of words which (a) A (b) B
is most similar in meaning to the word/group of words printed in (c) C (d) D
bold as used in the passage. (e) F
81. CONTRACT 88. Which of the following should be the THIRD sentence
(a) work (b) signature after rearrangement?
(c) deal (d) temporary (a) A (b) B
(e) get (c) C (d) D
82. BIDDING (e) E
(a) wishing (b) auctioning 89. Which of the following should be the FOURTH sentence
(c) wasting (d) playing after rearrangement?
(e) talking (a) B (b) C
83. ABODE (c) D (d) E
(a) stop (b) mountain (e) F
(c) plenty (d) house 90. Which of the following should be the LAST (SIXTH)
(e) dwelling sentence after rearrangement?
(a) A (b) B
DIRECTIONS (Qs. 84 & 85) : Choose the word / group of words (c) D (d) E
which is most opposite in meaning to the word / group of words
(e) F
printed in bold as used in the passage.
DIRECTIONS (Qs. 91-95) : Which of the phrases (a), (b), (c) and
84. SURVIVED
(d) given below each sentence should replace the phrase printed
(a) scratched (b) died
in bold in the sentence to make it grammatically correct? If the
(c) lived (d) fell sentence is correct as it is given and ‘No correction is required’,
(e) suffered mark (e) as the answer.
85. VICIOUS 91. As it was a dark and stormy night, Lata was too scared to go
(a) simple (b) small home alone.
(c) tough (d) harmless (a) very scary to (b) much scared to
(e) ferocious (c) as scared to (d) to scared too
DIRECTIONS (Qs. 86-90) : Rearrange the following six (e) No correction required
sentences (A), (B), (C), (D), (E) and (F) in the proper sequence to 92. Since it was her engagement party, Riya was dress to kill.
form a meaningful paragraph; then answer the questions given (a) dresses to kill (b) dressed to kill
below them. (c) dressed to killings (d) dressing to killed
(A) All of a sudden the mother duck saw a fox in the distance, (e) No correction required
was frightened and shouted, “Children, hurry to the lake, 93. Ramesh worries endlessly about his son’s future as he was
there’s a fox !” so poor in studies.
(B) The mother duck ran, leading the fox away from the lake (a) worry endless (b) worried endless
and as soon as the fox came very close, the mother duck (c) worried endlessly (d) worries endless
quickly spread her wings and rose up in the air. (e) No correction required
www.newspaperkorner.wordpress.com
PS-20 Practice Set - 2
www.newspaperkorner.wordpress.com 97. Why do you wish to tread on the toes?
94. Now that the actual criminal had been caught, Kunal was
happy that he was finally let of the hook. (a) To give offence to them
(a) off the hook (b) of the hookings (b) To follow them grudgingly
(c) off the hooks (d) of the hooks (c) To treat them indifferently
(e) No correction required (d) To be kicked by them
(e) None of these
95. The little boy appeared all of a sudden out of nowhere and
98. He intends setting up as a lawyer in the adjoining district.
take everyone by surprise.
(a) To establish himself (b) To migrate
(a) took everyone as surprised
(c) To join (d) To settle
(b) take everyone with surprised (e) None of these
(c) took everyone by surprises 99. The autographed bat from the famous cricketer Sunil
(d) took everyone by surprise Gavaskar is worth a jew's eye.
(e) No correction required (a) Not a worthy possession
DIRECTION (Qs. 96-100): In each of the following sentences, (b) unnecessary
an idiomatic expression or a proverb is highlighted. Select the (c) A costly items
alternative which best describes its use in the sentence. (d) A possession of high value
(e) None of these
96. He went to his friend's house in the evening as was his
100. The speaker gave a bird's eye view of the political conditions
wont. in the country.
(a) as usual (b) as he wanted (a) a personal view (b) a general view
(c) as his want was (d) as he wanted that day (c) a biased view (d) a detailed presentation
(e) none of these (e) None of these

www.newspaperkorner.wordpress.com
Practice Set - 2 PS-21
www.newspaperkorner.wordpress.com
Answer Key
1 (c) 11 (c) 21 (e) 31 (a) 41 (d) 51 (a) 61 (e) 71 (d) 81 (c) 91 (e)
2 (d) 12 (b) 22 (c) 32 (d) 42 (c) 52 (c) 62 (a) 72 (a) 82 (a) 92 (b)
3 (a) 13 (d) 23 (b) 33 (e) 43 (e) 53 (c) 63 (e) 73 (c) 83 (e) 93 (c)
4 (b) 14 (b) 24 (a) 34 (a) 44 (b) 54 (c) 64 (e) 74 (e) 84 (b) 94 (a)
5 (c) 15 (b) 25 (d) 35 (b) 45 (a) 55 (d) 65 (a) 75 (a) 85 (d) 95 (d)
6 (d) 16 (a) 26 (a) 36 (a) 46 (d) 56 (b) 66 (e) 76 (e) 86 (d) 96 (a)
7 (e) 17 (b) 27 (c) 37 (d) 47 (c) 57 (d) 67 (c) 77 (b) 87 (a) 97 (a)
8 (b) 18 (c) 28 (a) 38 (a) 48 (d) 58 (c) 68 (d) 78 (d) 88 (c) 98 (a)
9 (b) 19 (d) 29 (e) 39 (d) 49 (c) 59 (d) 69 (a) 79 (c) 89 (e) 99 (d)
10 (d) 20 (b) 30 (d) 40 (b) 50 (a) 60 (e) 70 (c) 80 (e) 90 (d) 100 (b)

Answers & Explanations


1223
a 2 + ab + b2 \?= = 305.75
1. (c) Given Expression = , where a = 137, b = 133 4
a3 - b3
32 + 48 16 ´ 2 + 16 ´ 3 4 2 +4 3
( a 2 + ab + b 2 ) 1 1 1 9. (b) = =
= = = = 8 + 12 4´ 2 + 4´3 2 2 +2 3
( a - b)( a + ab + b ) ( a - b) 137 - 133 4
2 2

n 4( 2 + 3)
2. (d) 3n = 81 Þ 3n / 2 = 34 Þ = 4Þ n=8 = =2
2 2( 2 + 3)
3. (a) ë ( ) (
é 5 7 + 7 ´ 4 7 + 8 7 ù - (19)2 = ?
û ) 1 1 9+ 8 3+ 2 2
10. (d) = ´ = = (3 + 2 2).
9- 8 9- 8 9+ 8 9-8
Þ (6 7 ´12 7) - (361) = ?
11. (c) Let Principal = ` P
Þ 72 ×7 × 7 – 361 = ?
3
æ 10 ö
- P = 993 Þ æç ´ ´ - 1ö÷ P = 993
\ ? = 504 – 361 = 143 11 11 11
P ç1 +
4. (b) (4444 ¸ 40) + (645 ¸ 25) + (3991 ¸ 26) = ? è 100 ÷ø è 10 10 10 ø
Þ ? = (111.1) + (25.8) + (153.5) Þ ? = 290.4
æ 1331 - 1000 ö
0.207 0.207 0.2070 Þ çè ÷ P = 993 or ,
5. (c) Given Expression = = = = 2070. 1000 ø
0.0023 0.0001 0.0001
23 993 ´ 1000
P= = 3000
21 ¸ 7 3 331
6. (d) Given Expression = =
5 11 æ 3000 ´ 3 ´ 10 ö
5+5+
5 \ Simple interest = ` çè 100
÷ø = ` 900
7. (e) 33124 ´ 2601 – (83) 2 = (?) 2 (37) 2
12. (b)
Þ (?)2 = 2 2
33124 ´ 2601 – (83) - (37) 15 5 12
Þ (?)2 = 182 × 51 – 6889 – 1369
Þ (?)2 = 9282 – 6889 – 1369
Þ (?)2 = 1024
Opting only Opting Opting only
\ ? = 1024 = 32 Physics both Maths
17 51 1 3
8. (b) 5 ´ 4 ´11 + 2 = ?
37 52 7 4
Opting other
æ 202 259 78 ö æ 11 ö subjects 10
Þç ´ ´ ÷ + ç ÷ =?
è 37 52 7 ø è 4 ø
11 Total no. of students = 15 + 5 + 12 + 10 = 42
Þ 303 + =?
4 www.newspaperkorner.wordpress.com
PS-22 Practice Set - 2
www.newspaperkorner.wordpress.com
13. (d) Concentration of boric acid = 80% = 80 cc 21. (e) The series is + 0.5, + 1, +1.5, +2,....
Quantity of water = 20 cc 22. (c) The series is + 12, + 22, +32, +42,....
Let x cc of water be added to get the concentration of 23. (b) The series is –8, –7, –6, –5, ....
50%. 24. (a) The series is × 3 – 1 in each term.
80 50 80 1 25. (d) The series is ¸ 2 –1 in each term.
Þ = or = or x = 60 cc
100 + x 100 100 + x 2 26. (a) 196.1 × 196.1 × 196.1 × 4.01 × 4.01 ×4.001 × 4.999 × 4.999
14. (b) Successive discounts of 20% and 15% on ` 100 = (196.1)3 × 4× ?
yields to or 4 × ? = 4.01 ×4.001 × 4.999 × 4.999 or ? = 4 × 5 × 5 = 100
100 × 0.8 × 0.85 = ` 68
\ Single discount = (100 - 68) = 32% 2 1 3 14 1 1
27. (c) ?= ´ + ´ = +1 =1 =1
15. (b) Let the length and breadth of the rectangular room be 7 8 7 6 28 28
l and b.
28. (a) ? = (10.1)2.01 + (2.9)3.001 = (10)2 + (3)3 = 100 + 27 = 130
l+4 4
We have, = Þ 3 l + 12 = 4b + 16 29. (e)
b+4 3 4.76 ´ ? = 1999.9997
Þ 3 l – 4b = 4 .................. (i) 4.76 × ? = 44.72 or ? = 9
l-4 2 1
Again, we have = Þ l - 4 = 2b - 8 30. (d) ? = 23% of 4011 + of 5555 = 922.53 ¸ 79357 = 1700
b-4 1 7
Þ l - 2b = -4 .................. (ii) 31. (a) Required average age

Solving (i) and (ii), we get l = 12 and b = 8. æ 15 ´ 36 + 12 ´ 16 ö æ 540 + 192 ö


16. (a) Let the second car overtakes the first car after t hours. =ç
36 + 12 ÷ years = ç 48 ÷ years
è ø è ø
Distance covered by the first car = Distance covered
= 15.25 years.
by the second car.
32. (d) Let the CP of the article be ` x.
t -1ö
Þ 10 t = 8 + æç 8 + ö÷ + æç 8 + ö÷ + ...... + æç 8 +
1 2 According to the question,
÷ 1450 – x = x – 1280
è 2ø è 2ø è 2 ø
Þ 2x = 1450 + 1280 = 2730
1
or 10t = 8t + [1 + 2 + .... + (t - 1)] 2730
2 Þx= = ` 1365
2
1 t(t - 1) 1
or 10t = 8t + or 2t = (t 2 - t) 33. (e) Let the number be x.
2 2 4 According to the question,
Þ t = 9 hrs. [ t ¹ 0 ] 2 2
17. (b) m1 ´ d1 ´ t1 ´ w 2 = m 2 ´ d 2 ´ t 2 ´ w1 = x ´ ´ = 48
3 5
24 × 10 × 8 × 1 = m2 × 6 × 10 × 1
48 ´ 15
24 ´ 10 ´ 8 Þx= = 180
Þ m2 = = 32 men 4
6 ´ 10
18. (c) Let the two digit number be 10x + y 180 ´ 30
\ 30% of 180 = = 54
We have x + y = 15 ............(i) 100
and (10x + y) + 9 = (10y + x) or 9x – 9y = – 9 34. (a) 12 × 16 = 192
or x – y = – 1 ......... (ii) In other words, number of rows = 16
From (i) and (ii) x = 7 and y = 8 and number of children in each row = 12 and number of
The number is 10 × 7 + 8 = 78 children in each column = 16
19. (d) Let edge of the new cube = x cm. 35. (b) Let the CP of an apple be ` x and that of a guava
Volume of the newly formed figure (cube) = ` y.
= sum of volume of smaller cubes. \ 12x + 8y = 76 and
i.e. (x)3 = (3)3 + (4)3 + (5)3 = 27 + 64 + 125 = 216 Þ x = 6 cm 8x + 12y = 64
20. (b) Let present age of yashika = x years
On adding,
\ Age of yashika at the time of marriage
20x + 20y = 140
9 Þ 20 (x + y) = 140
= (x – 8) years Q x= ( x - 8)
7 140
72 Þ x +y= =7=`7
20
Þ 7 x = 9 x - 72 \ x= = 36 years
2 36. (a) The given series when written in the reverse order
becomes.
1
\ Present age of the daughter = 36 ´ = 6 years 13, 11, 5, 0, 1, 2, 6, 4, 8, 3, 0, 7, 9, 3, 7
6 The 7th number from the left is 6. The 4th number to
\Her daughter’s age 3 years ago = (6 – 3) = 3 years the right of 6 is 0.
www.newspaperkorner.wordpress.com
Practice Set - 2 PS-23
www.newspaperkorner.wordpress.com
37. (d) "Some pencils are papers"
ne ri so good rainy day Conclusion II is converse of this Conclusion.
45. (a) Some roofs are figure. (I-type)
si ne po day is wonderful

ri jo good boy
All figures are lions. (A-type)
38. (a) SMOOTH º 135579 I + A Þ I-type Conclusion
ROUGH º 97531 "Some roofs are lions."
HARD º 9498 Þ O = 5 Some roofs are lion. (I-type)
Clearly, only the first option i.e. 1527 contains 5. Hence,
the correct code for SOFT in the code would be 1527.
39. (d) Saroj Rajesh
mother All lions are goats. (A-type)
father
I + A Þ I-(A-type) Conclusion
"Some roofs are goats"
Vani Deepak Ramesh Conclusion I is converse of this Conclusion..
Sister-in-law only brother
46. (d) A’s wife E is the mother.
47. (c) A is the husband of E.
wife
N (E) 48. (d) Clearly there are four male members A, B, D and F.
49. (c) Clearly, F is the son of E
50. (a) A, B are brothers of each other while F is the brother of C.
40. (b) (N) W E(S)
51. (a) Either C or G has to be first and D has to come before
E. Hence, E cannot, finish second.
S (W) 52. (c) F finishes second when D finishes third. Thus F finishes
When the arrow turns, East becomes South, North ahead of E.
becomes East, West becomes North and South 53. (c) In the event of C finishing first, G finishes last and we
becomes West. will have the following three possible ordering of
So the traveller must be actually travelling in the
finishes.
South thinking it is West.
41. (d) All shoes are pens. (A-type) CFDEG, CDEFG and CDFEG.
54. (c) When F finishes ahead of D, than F will definitely
finish at the second place.
Some pens are razors. (I-type) 55. (d) When there is exactly one golfer between C and D,
A + I Þ No Conclusion then E finishes at the fourth place.
42. (c) All the three Premises are Particular Affirmative (I-type).
56. (b) B > V ....(i) K < C ...(ii); C £ B ...(iii)
No Conclusion follows from Particular Premises.
Conclusion I and II from Complementary Pair. No relationship can be find out between V and C.
Therefore, either I or II follows Hence I does not follow.
43 (e) All brushes are chocolates. (A-type) From (ii) and (iii), B > K. Hence II follows.
57. (d) K > T ...(i) ; S = K ...(ii); T £ R ...(iii)
All chocolates are mirrors. (A-type) Neither relationship can be established .
A + A Þ A-type Conclusion 58. (c) U = M ...(i) P ³ U ...(iii); M ³ B ...(iii)
"All brushes are mirrors"
Combining, we get P ³ U = M ³ B Þ P ³ B
All biushes are mirrors. (A-type)
Þ P = B or P > B
59. (d) L ³ N ...(i) ; J £ P ...(ii); P ³ L ...(iii)
All mirrors are tables. (A-type) Neither relationship can be established.
"All brushes are tables"
60. (e) H ³ G ....(i); D > E ...(ii); H = E ...(iii)
Conclusion I is converse of this Conclusion.
Conclusion II is converse of the second Premise. Combining, we get D > E = H ³ G
44. (b) Some pencils are knives. (I-type) Þ D > H and G < D
61. (e) Four

All knives are papers. (A-type M E 5 PB2A7 K N 9T RU4 6 I J D F 1 Q 3W8V I S


I + A Þ I-type Conclusion Z
www.newspaperkorner.wordpress.com
PS-24 Practice Set - 2
www.newspaperkorner.wordpress.com
62. (a) F D J I 6 4 U R T 9 N K 7 A 2 B P 5 E M 1 Q 3 W 8 V I S Z 77. (b) Refer to the sentence that ‘I know.......complete the
63. (e) Eighth to the left of the nineteenth letter/number from year’ of the statement made by king in the fourth para
the left Þ (19 – 8=) 11th letter/number from left. Hence, of the passage.
required element is 9. 78. (d) Refer to the second last sentence of the fourth para
64. (e) Except it second element in each group is third to the that he sent all the.......for storage.......of the passage.
right of first element while third element of each group is 79. (c) Refer to the third sentence that however, the
second to the left of first element of the respective group. king.......leave the kingdom.......of the last para of the
65. (a) There are no such vowels. passage.
80. (e) The moral of the story is ‘Always think and plan
(For Answers 66-70)
ahead’.
L R 81. (c) The word contract means agreement. Compact, bond,
I

Yellow
Brown
White

Green
Violet

deal, bargain etc. are the most similar word in meaning.


Black

E
Blue
Pink

G
F H 82. (a) The word bidding means order, command, wish,
T T
desire, request, direction etc. Hence wishing is most
66. (e) Black colour cloth is placed fifth to the left of Yellow similar in meaning of the given word.
colour cloth. 83. (e) The word abode means house, dwelling, residence
67. (c) White cloth is placed exactly in betwen Violet and Black. and habitation etc.
68. (d) White cloth is third to the left of Brown. 84. (b) The word survived means remained alive, lived,
69. (a) yellow is sixth to the right of Pink. endured etc. Hence died is the most opposite word in
70. (c) Black is to the immediate right of Pink. meaning.
71. (d) The reason behind the question is not mentioned in 85. (d) The word vicious means brutal, savage, dangerous,
the passage. cruel etc. Its opposite word will be gentle, kindly,
72. (a) Refer to the second sentence of the first para of the harmless etc.
passage that the king had to agree to a contract....... (For Answers 86-90)
being a king. Sentences are re-arranged in order as DACFBE. A mother
73. (c) Refer to the sentence that the island was covered....... duck is the clue and qualifier of a sentence which is followed
discovered dead bodies.......past kings of the second by A, C, F and finally B and E.
last sentence of the third para of the passage.
91. (e) Here, too is used as emphatic word. Lata was so scared
74. (e) Refer to the fourth sentence that in the first
that she could not go home alone. Hence, no
month.......trees were cut down.......of the fourth para
correction is required.
of the passage.
92. (b) The structure of sentence is subject + was/were +
75. (a) Refer to the last sentence “I turned the deadly
island..............a beautiful abode.......peacefully” of the third form of verb + object. Thus, Riya was dressed to
sixth para of the passage. kill.
76. (e) From reading the passage thoroughly we come to the 93. (c) The given sentence is the statement of simple past
conclusion that the king was intelligent, foresight and tense. Hence it should be ‘worried’ instead of worries.
cunning as he made deadly island a beautiful place to 94. (a) The phrase ‘let off’ means to give them only a light
live in with all luxuries of the kingdom. punishment.
95. (d) It should be ‘took’ instead of ‘take’.

www.newspaperkorner.wordpress.com
www.newspaperkorner.wordpress.com

Preliminary Exam
Practice Set - 3
INSTRUCTIONS
• This Preliminary Exam practice set consists of three sections. Quantitative Aptitude (Qs. 1-35); Reasoning
Ability (Qs. 36-70) and English Language (Qs. 71-100).
• All the questions are compulsory.
• Each question has five options, of which only one is correct. The candidates are advised to read all the
options thoroughly.
• There is negative marking equivalent to 1/4th of the mark allotted to the specific question for wrong answer.

Time : 1 hrs. Max. Marks : 100

QUANTITATIVE APTITUDE
4096 ´ 56
6. =?
DIRECTIONS (Qs. 1-10) : What should come in place of the 764 - 652
question mark (?) in the following questions ? (a) 36 (b) 48
1. (786 × 64) ÷ 48 = ? (c) 32 (d) 44
(a) 1050 (b) 1024 (e) None of these
(c) 1048 (d) 1036 7. (98360 + 25845 – 36540) ÷ 2500 = ?
(e) None of these
(a) 36.585 (b) 30.082
2. 3
13824 ´ ? = 864 (c) 32.085 (d) 35.066
(a) 1296 (b) 1156 (e) None of these
(c) 1600 (d) 1024 8. 7414 + 3698 + 1257 + 1869 = ?
(e) None of these (a) 14328 (b) 14438
3 (c) 13428 (d) 13248
3. 60% of 20% of th of ? = 450
5 (e) None of these
(a) 6200 (b) 6,240
9. (91)2 + (41)2 - ? = 9858
(c) 6150 (d) 6275
(e) None of these (a) 11236 (b) 10816

4. 196 × 948 ÷ 158 = ? (c) 10404 (d) 9604


(e) None of these
(a) 1156 (b) 1200
10. (2640 ÷ 48) × (2240 ÷ 35) = ?
(c) 1188 (d) 1176
(a) 3520
(e) None of these
(b) 3515
5. 3.5 + 11.25 × 4.5 – 32.5 = ?
(a) 18.275 (b) 21.625 (c) 3495
(c) 32.375 (d) 25.45 (d) 3490
(e) None of these (e) None of these
www.newspaperkorner.wordpress.com
PS-26 Practice Set - 3
www.newspaperkorner.wordpress.com (a) ` 2 (b) ` 3.5
DIRECTIONS (Qs. 11-15): What should come in place of question
mark (? ) in the following number series ? (c) ` 2.5 (d) ` 1.5
(e) None of these
11. 121 117 108 92 67 ? 22. When 3,626 is divided by the square of a number and the
(a) 31 (b) 29 answer so obtained is multiplied by 32, the final answer
(c) 41 (d) 37 obtained is 2,368. What is the number ?
(e) None of these (a) 7 (b) 36
12. 50 26 14 ? 5 3.5 (c) 49 (d) 6
(a) 6 (b) 8 (e) None of these
(c) 10 (d) 12 23. The sum of the digits of a two digit number is 14. The
(e) None of these difference between the first digit and the second digit of
13. 3 23 43 ? 83 103 the two digit number is 2. What is the product of the two
(a) 33 (b) 53 digits of the two digit number ?
(c) 63 (d) 73 (a) 56 (b) 48
(e) None of these (c) 45 (d) Cannot be determined
14. 748 737 715 682 638 ? (e) None of these
(a) 594 (b) 572 24. A car runs at the speed of 50 kmph when not serviced and
(c) 581 (d) 563 runs at 60 kmph, when serviced. After servicing the car
(e) None of these covers a certain distance in 6 hours. How much time will
15. 1 9 25 49 81 ? 169 the car take to cover the same distance when not serviced ?
(a) 100 (b) 64 (a) 8.2 hours (b) 6.5 hours
(c) 81 (d) 121 (c) 8 hours (d) 7.2 hours
(e) None of these (e) None of these
16. The ratio of ducks and frogs in a pond is 37 : 39 respectively. 25. Venkat has some ducks and some sheep. If the total number
The average number of ducks and frogs in the pond is 152. of animal heads is 81 and the total number of animal feet are
What is the number of frogs in the pond ? 268, how many sheep does Venkat have?
(a) 148 (b) 152 (a) 28 (b) 53
(c) 156 (d) 144 (c) 44 (d) Cannot be determined
(e) None of these (e) None of these
17. The number of employees in Companies A, B and C are in a 26. The sum of the two digits of a two digit number is 13. The
ratio of 4 : 5 : 6 respectively. If the number of employees in difference between the two digits of the number is 3. What
the Companies is increased by 25%, 30% and 50% is the two digit number?
respectively, what will be the new ratio of employees (a) 85 (b) 49
working in Companies A, B and C respectively ? (c) 57 (d) Cannot be determined
(a) 13 : 10 : 18 (b) 10 : 13 : 17 (e) None of these
(c) 13 : 15 : 18 (d) Cannot be determined 27. 25 shirt pieces of 125 cms. each can be cut from a reel of
(e) None of these cloth. After cutting these pieces 90 cms. of cloth remains.
18. The average of five positive numbers is 213. The average of What is the length of the reel of cloth in metres?
the first two numbers is 233.5 and the average of last two (a) 3215 metres (b) 35.15 metres
numbers is 271. What is the third number ? (c) 32.15 metres (d) 3515 metres
(a) 64 (b) 56 (e) None of these
(c) 106 (d) Cannot be determined 28. The sum of the squares of two consecutive positive odd
(e) None of these numbers is 650. Which is the larger number?
19. Sonali invests 15% of her monthly salary in insurance (a) 17 (b) 21
policies. She spends 55% of her monthly salary in shopping (c) 23 (d) 15
and on household expenes. She saves the remaining (e) None of these
amount of Rs. 12,750. What is Sonali’s monthly income ? 29. The profit earned after selling a pair of shoes for ` 2,033 is
(a) `. 42,500 (b) ` 38,800 the same as loss incurred after selling the same pair of shoes
(c) ` 40,000 (d) ` 35,500 for ` 1,063. What is the cost of the shoes?
(e) None of these (a) ` 1,650 (b) ` 1,548
20. What approximate amount of compound interest can be (c) ` 1,532 (d) Cannot be determined
obtained on an amount of ` 9, 650 at the rate of 6% p.a. at
(e) None of these
the end of 3 years ?
(a) ` 1,737 (b) ` 1,920 30. When an amount of ` 1,58,965 is divided equally amongst
(c) ` 1,720 (d) ` 1, 860 120 people, how much approximate amount would each
(e) ` 1,843 person get?
21. A milkman sells 120 litres of milk for ` 3,360 and he sells 240 (a) ` 1,330 (b) ` 1,315
litres of milk for Rs. 6,120. How much concession does the (c) ` 1,335 (d) ` 1,320
trader give per litre of milk, when he sells 240 litres of milk ? (e) ` 1,325
www.newspaperkorner.wordpress.com
Practice Set - 3 PS-27
www.newspaperkorner.wordpress.com
DIRECTIONS (Qs. 31-35) : Study the following graph carefully and answer the questions that follow :
The graph given below represents the number of users of two broadband services A and B across
5 cities P, Q, R, S and T.

800

700

A
600 B

500

400
P Q R S T
CITY
31. What is the total number of users of brand B across all five Conclusions : I. Some notes are books.
cities together ? II. Some pencils are books.
(a) 2700 (b) 3000 III. Some books are papers.
(c) 3100 (d) 2900
IV. No book is a paper.
(e) 3200
(a) Only I and either III or IV follow
32. The number of users of brand A in city T is what percent of
the number of users of brand B in City Q ? (b) Either III or IV follows
(a) 150 (b) 110 (c) Only I and III follow
(c) 140 (d) 160 (d) Neither II nor III follows
(e) 120 (e) None of these
33. What is the average number of users of brand A across all 37. Statements : Some tables are chairs.
five cities together ?
No cupboard is table.
(a) 560 (b) 570
Some chairs are cupboards.
(c) 580 (d) 590
(e) 550 Conclusions : I. Some chairs are not tables.
34. What is the difference between the total number of users of II. All chairs are either tables or cupboards.
Brand A and B together in city R and the total number of III. Some chairs are tables.
users of brand A and B together in city P ? IV. All chairs are tables.
(a) 170 (b) 140 (a) Only I and IV follow
(c) 130 (d) 150 (b) Only either II or III follows
(e) 160
(c) Only I and III follows
35. What is the respective ratio of the number of users of brand
(d) Either II or III and I follow
A in city P to the number of users of brand B in city S ?
(a) 5 : 7 (b) 4 : 7 (e) None of these
(c) 2 : 5 (d) 3 : 4 38. Statements : No table is fruit.
(e) 5 : 6 No fruit is window.
REASONING ABILITY All windows are chairs.
Conclusions : I. No window is table.
DIRECTIONS (Qs. 36-40) : In each of the questions below are
II. No chair is fruit.
given three statements followed by four conclusions numbered
I, II, III and IV. You have to take the given statements to be true III. No chair is table.
even if they seem to be at variance with commonly known facts. IV. All chairs are windows.
Read all the conclusions and then decide which of the given (a) Either I or III follows (b) All follow
conclusions logically follows from the given statements (c) Only I and II follow (d) Only III and IV follow
disregarding commonly known facts. (e) None of these
36. Statements : All books are notes. 39. Statements : No man is sky.
Some notes are pencils. No sky is road.
No pencil is paper. Some men are roads.
www.newspaperkorner.wordpress.com
PS-28 Practice Set - 3
www.newspaperkorner.wordpress.com
Conclusions : I. No road is man. (a) None follows
II. No road is sky. (b) Only either I or II follows
III. Some skies are men. (c) Only either I or II and III follows
IV. All roads are men. (d) Only III follows
(a) Either I or IV follows (b) Only I follows (e) All follow
(c) Only I and III follow (d) Only II follows 45. Statements : J#N, K@N, T$K
(e) None of these Conclusions : I . J%T
40. Statements : All papers are books. II. T$N
All bags are books.
III .N@J
Some purses are bags.
(a) None follows
Conclusions : I. Some papers are bags.
II. Some books are papers. (b) Only I or II follow
III. Some books are purses. (c) Only I and III follow
(a) Only I follows (d) Only II and III follow
(b) Only II and III follow (e) All follow
(c) Only I and III follow DIRECTIONS (Qs. 46-50) : Study the information given below
(d) Only I and II follow
to answer the questions that follow :
(e) None of these
(i) There is a family of 5 persons A, B, C, D and E.
DIRECTIONS (Qs. 41-45) : In the following questions, the (ii) They are working as a doctor, a teacher, a
symbols %, *, @, $ and # are used with the following meaning trader, a lawyer and a farmer.
as illustrated below : (iii) B, an unmarried teacher, is the daughter of A.
‘P @ Q’ means ‘P is not smaller than Q’. (iv) E, a lawyer, is the brother of C.
‘P # Q’ means ‘P is not greater than Q’. (v) C is the husband of the only married couple in the family.
‘P % Q’ means ‘P is neither greater than nor equal to Q’. (vi) Daughter-in-law of A is a doctor.
‘P * Q’ means ‘P is neither smaller than nor greater than Q’. 46. Which of the following is a group of female members in the
‘P $ Q’ means ‘P is neither smaller than nor equal to Q’. family ?
41. Statements : T$K, K#R, R*M (a) A and D (b) D and E
Conclusions : I. M*K (c) A, C and E (d) B and D
II . M % T (e) None of these
III. M$K 47. Which of the following is the married couple ?
(a) All follows (a) A and B (b) C and D
(b) Only either I or III follows (c) A and D (d) B and C
(c) Only either I or II follows (e) None of these
(d) Only either II or III follows 48. Which of the following is a group of male members in the
(e) None of these family ?
42. Statements : M%R, R#T, T*N (a) A, B and C (b) B and D
Conclusions : I. N*R (c) C and E (d) A, C and D
II . N$R (e) None of these
III .N$M 49. Who is the doctor in the family ?
(a) All follows (a) A (b) B
(b) Either I or II follows (c) C (d) D
(c) Either I or II and III follows (e) None of these
(d) Either I or III and II follows 50. Who is the trader in the family ?
(a) A (b) B
(e) None of these
(c) C (d) D
43. Statements : V@M, A$M, R#V
(e) None of these
Conclusions : I. R#A
II . V@A DIRECTIONS (Qs. 51-55): Answer these questions referring
III. R$M to the symbol-letter-number sequence given below:
(a) Only I follows (b) Only II follows EG4BH 75@K8DN £Q Z$W3C19*lB 2S6
(c) Only III follows (d) None follows 51. How many such consonants are threre in the above
(e) All follow sequence which are immediately preceded by a symbol and
44. Statements : B*D, D@H, H%F immediately followed by a digit ?
Conclusions : I. B*F (a) One (b) Two
II . B$F (c) None (d) Three
III. D$F (e) More than three
www.newspaperkorner.wordpress.com
Practice Set - 3 PS-29
www.newspaperkorner.wordpress.com
52. What should come in place of the question mark (?) in the 61. DH is related to FG in the same way as LV is related to ......
following sequence ? (a) NU (b) UN
4H@, KDQ, ?, ILS (c) VL (d) NO
(a) ZW1 (b) NQ$ (e) None of these
(c) @8N (d) $W9 62. ‘Offhand’ is related to ‘premeditation’ in the same way as
(e) None of these ‘above board’ is related to :
53. Which of the following is exactly in the midway between (a) Integrity (b) Honesty
the ninth from left end and the seventh from right end ? (c) Guide (d) Competition
(a) Q (b) Z (e) None of these
(c) $ (d) W
63. Rearrange the first four letters, in any way, of the word
(e) None of these
DECISION. Find how many words can be formed by using
54. If the first fifteen elements are written in the reverse order all the four words.
then which of the following will be seventh to the left of
(a) One (b) Two
twelfth element from right end ?
(a) 7 (b) @ (c) Three (d) More than three
(c) 5 (d) K (e) None of these
(e) None of these 64. If the last four letters of the word ‘CONCENTRATION’ are
55. How many such digits are there in the above sequence written in reverse order followed by next two in the reverse
which are immediately preceded as well as followed by dig- order and next three in the reverse order and then followed
its ? by the first four in the reverse order, counting from the end
which letter would be eighth in the new arrangement ?
(a) None (b) One
(a) N (b) T
(c) Two (d) Three
(c) E (d) R
(e) None of these
56. In the following sequence or instructions, 1 stands for Run, (e) None of these
2 stands for Stop, 3 stands for Go, 4 stands for Sit and 5 65. Five boys took part in a race. Raj finished before Mohit but
stands for Wait. If the sequence is continued, which behind Gaurav. Ashish finished before Sanchit but behind
instruction will come next ? Mohit. Who won the race?
44 5453 4531 453 1245 4534 53 (a) Raj (b) Gaurav
(a) Wait (b) Sit (c) Mohit (d) Ashish
(c) Stop (d) Run (e) None of these
(e) None of these
57. In a certain code, a number 13479 is written as AQFJL and DIRECTIONS (Qs. 66-70) : In each of the questions given below
2568 is written as DMPN. How is 396824 written in that which one of the five answers figures should come after the
code? problem figures, if the sequence were continued?
(a) QLPNMJ (b) QLPNMF 66. Problem figures
(c) QLPMNF (d) QLPNDF
(e) None of these
58. In a certain code OVER is written as $#%*. and VIST is
written as #+×–. How is SORE written in that code?
(a) ×$*% (b) %×$* Answer figures
(c) ×*$% (d) × %*
(e) None of these
59. A boy goes to see a film and finds a man who is his relative.
The man is the husband of the sister of his mother. How is (a) (b) (c) (d) (e)
the man related to the boy?
67. Problem figures
(a) Brother (b) Nephew
(c) Uncle (d) Father
(e) None of these
60. Laxman went 15 km to the west from my house, then turned
left and walked 20 km. He then turned East and walked 25 Answer figures
km and finally turning left covered 20 km. How far was he
from my house?
(a) 5 km (b) 10 km
(c) 40 km (d) 80 km
(e) None of these (a) (b) (c) (d) (e)
www.newspaperkorner.wordpress.com
PS-30 Practice Set - 3
www.newspaperkorner.wordpress.com
68. Problem figures wife rushed to embrace him. She feared a damaging incident had
occurred.
The king spoke seldom that day and awoke the next day to
make a proclamation to his servants and subjects. The whole
kingdom feared what was in store for them from their angry king.
Answer figures But to their surprise he said to all gathered, "From now on I will
be a different king. A softer and a patient king."
True to his words from that day on, the king had truly turned
on a new leaf; he cleaned out the corruption and injustice in a
tender manner with punishments aimed to renew the person from
(a) (b) (c) (d) (e) within.
69. Problem figures One fine day his evil advisor gathered courage to ask the
reason for his paradigm shift. And the king answered. When I
went on horseback that morning a month ago, I noticed a dog
brutally chasing a cat. The cat managed to sneak into a hole only
after the dog bit her leg, maiming her for life. soon afterwards, the
dog barked at a farmer who picked up a sharp stone and hit it
Answer figures
straight in the dog's eye. Bleeding profusely, the dog yelped in
pain. As the farmer walked on, he slipped on the edge of the road
and broke his head.
All this happened in a matter of minutes before me and then
I realized that evil begets evil. I thought about it deeply and was
(a) (b) (c) (d) (e) ready to give up my worldly life for the betterment of my subjects.
70. Problem figures I wanted to give up the evil in me as I did not want evil to encounter
me.
Sniggering away the immoral advisor thought what a perfect
time it was to dethrone the king, because the Raja had grown kind
hearted and patient and would not endeavour a combat. Thinking
how he would plan his attack, he stumbled over a step that took
Answer figures him hurling down the remaining steps, bringing him to a stop
with a crash. He howled in pain only to discover he had broken
the bones in both his legs.
71. How can Raja Shankara be described before his
transformation?
(1) He was unjust
(2) He was preoccupied with himself
ENGLISH LANGUAGE (3) He was cruel
(a) Only (1) (b) Only (2)
DIRECTIONS (Qs. 71-80) : Read the following passage carefully
(c) Only (3) (d) Only (1) adn (2)
and answer the questions given below it.
(e) All the three (1), (2) and (3)
Once upon a time there lived a vicious king, Raja Shankara– 72. Why was the king not happy with the old Valuer?
short-tempered and temperamental. "God I am" he said to his (a) As the Valuer was not good at his work
image as he stared into the mirror everyday, many times a day, He (b) As he had dishonoured the king
was obsessed with himself. He loved no one but himself. He was (c) As the Valuer had been dishonest with the king about
blinded towards the injustice in his kingdom because he had little the prices that he set for goods
time for his subjects. He wasted most of his time in pouring milk
(d) As the king beloeved that he was not earning much
and honey over himself.
because of the Valuer's honesty
Interruption in his possessed life was dealt with stern (e) None of these
reprimanding and sometimes on petty issues he would behead 73. What proclamation did the Raja make to his subjects?
his servants. Provoked by his evil advisor Twishar, he went on
(a) That he was giving up his throne for the betterment of
with his self indulged life, unaware of the plot his very devoted the kingdom
advisor was planning. A plot to dethrone the king, rule the kingdom
(b) That his advisor would be the king from then on
with his wicked ways only to harness wealth and the reputation
(c) That he would be a better king to them than he had
of a king.
been all this while
One morning the king went on his usual moring horseback (d) That his subjects were not good enough to deserve
rounds but returned with a very sad look on his face. He locked better treatment than what was already being meted
himself inside his platial room only to unlock it at sundown. Just out to them
as the doors creaked open and Raja Shankara emerged from it, his
(e) None of these
www.newspaperkorner.wordpress.com
Practice Set - 3 PS-31
www.newspaperkorner.wordpress.com
74. What did Raja Shankara's wife think about the Raja's peculiar DIRECTIONS (Qs. 81-85) : Which of the phrases (a), (b), (c) and
behaviour that particular day? (d) given below each sentence should replace the phrase printed
(a) She was afraid that something really bad had happened in bold in the sentence to make it grammatically correct? If the
(b) She was afraid that the Raja would beat her up because sentece is correct as it is given and no correction is required,
of his unusual mood mark (e) as the answer.
(c) She thought that he was in his usual sour mood
81. Her entry to the office party was restrict as an official
(d) She thought that the Raja had received threats to his
enquiry had ben constituted against her.
life from his servants
(a) was restricting
(e) None of these
(b) is restricted
75. What was the reason for Raja Shankara's change in
(c) was restricted
behaviour?
(d) is restricting
(a) His advisor's words had made him realize his mistake (e) No correction required
(b) He had felt bad for a poor family on his tour around his 82. Rima was at her wit's end trying to figure out what to buy
kingdom for her frind's birthday.
(c) His wife had betrayed him and hence he was upset (a) at her witting end
(d) He had realized that doing good to people would bring (b) at her wit ends
good to him (c) to her wit's end
(e) He had realized that evil begets evil. (d) so wit's end
76. What can possibly be the moral of the story? (e) No correction required
(a) Believe in yourself 83. Pritesh while away his time in playing games on the computer
(b) Money is not everything in life instead of studying.
(c) Don't trust people (a) whiled away his time
(d) Better late than never (b) whiled against his time
(e) As you sow so shall you reap (c) whiling away his time
77. How did Raja Shankara treat all his servants if they (d) while awayed his time
interrupted him, before he changed into a good person? (e) No correction required
(a) He treated all his servants with respect 84. Mohan had make up his mind about going on the world
(b) He would scold them and sometimes cut their heads tour all alone.
off over trivial issues (a) made minds
(c) He would dismiss them from their duties never to be (b) make his mind
reinstated (c) make up his minding
(d) He would rob them of all their possessions (d) made up his mind
(e) None of these (e) No correction required
78. What plan did Twishar have with regard to the Raja? 85. She rushed to the station but could find any trace of her
(a) He planned to take over the Raja's throne through daughter there.
devious means (a) not found trace (b) find no trace
(b) He was a loyal servant to the Raja and always had his (c) found not trace (d) finding no trace
best interests in mind (e) No correction required
(c) He had plans to provoking the Raja into ruling the DIRECTIONS (Qs. 86-90) : In each question below, a sentence
kingdom in more wicked ways with four words printed in bold type is given. These are numbered
(d) He planned to kill the raja with the help of the Raja's as (a), (b), (c) and (d). One of these four words printed in bold may
servants be either wrongly spelt or inappropriate in the context of the
(e) None of these sentence. Find out the word which is wrongly spelt or
79. Why did Twishar think that this was the best time to inappropriate, if any. The number of that word is your answer. If
dethrone the Raja? all the words printed in bold are correctly spelt and also
(a) As the Raja was sick, aliling and was on his deathbad appropriate in the context of the sentence, mark (e) i.e., 'All
(b) As he knew that the Raja had finally lost his mind correct' as your answer.
(c) As he knew that the Raja had become a soft natured
86. Discussion (a)/ is an exchange of knowledge (b)/ whereas
person and would not attempt to fight.
arguement (c)/ is a depiction (d)/ of ignorance. All correct
(d) As he was sure that he would be able to convince that (e).
people of the kingdom that he Raja had become weak
87. He was arrested (a)/ for the crime (b)/ and was charged
(e) None of these
(c)/ with attempt (d)/ to murder. All correct (e)
80. What did the dog do to the cat?
88. commit (a)/ yourself to lifelong learning (b)/ as the most
(a) The dog chased the cat and sent it in the man'sdirection
valuable (c)/ aset (d)/ you will have is your mind.
(b) The dog bit the cat's leg and crippled her for life
All correct (e)
(c) The dog killed the cat
89. Belive (a)/ that life is worth (b)/ living and your belief will
(d) Not mentioned in the passage
create (c)/ the fact. (d)/ All correct (e)
(e) None of these www.newspaperkorner.wordpress.com
PS-32 Practice Set - 3
www.newspaperkorner.wordpress.com
90. The best educated (a)/ human bing (b)/ is the one who 92. (a) journey (b) sand
understands (c)/ most about the life in which (d)/ he is (c) running (d) border
placed. All correct (e). (e) hunt
DIRECTIONS (Qs. 91-100) : In the following passage there are 93. (a) dead (b) captured
blanks, each of which has been numbered. These numbers are (c) presentable (d) missing
printed below the passage and against each, five words are (e) hurt
suggested, one of of which fits the blank appropriately. Find out 94. (a) as (b) until
the appropriate word in each case. (c) from (d) with
(e) through
Once upon a time, two friends were (91) through the desert. 95. (a) decided (b) fell
During some point of the (92) they had an argument, and one (c) made (d) want
friend slapped the other one in the face. The one who got slapped (e) left
was (93), but without saying anything, he wrote in the sand, 96. (a) home (b) stuck
"Today my best friend slapped me in the face." They kept on (c) blended (d) mixed
walking (94) they found an oasis, where they (95) to take a both. (e) sitting
The one, who had been slapped, got (96) in the quicksand and 97. (a) separated (b) leaked
started drowing, but the friend saved him. After the friend (97) (c) died (d) recovered
from the near drowning, he wrote on a stone, "The friend who (e) saved
had slapped and saved his best friend asked him, "After I hurt 98. (a) so (b) how
you, you wrote in the sand and (98) you write on a stone, why?" (c) when (d) tomorrow
The other friend (99), "When someone hurts us, we should write (e) now
it down in sand where winds of forgiveness can erase it away. 99. (a) called (b) tell
But, when someone does something good for us, we must (100) (c) replied (d) questioned
it in stone where no wind can ever erase it." (e) asked
91. (a) crawling (b) speaking 100. (a) talk (b) push
(c) swimming (d) walking (c) engrave (d) add
(e) dancing (e) bury

Answer Key
1 (c) 11 (a) 21 (c) 31 (b) 41 (b) 51 (b) 61 (a) 71 (e) 81 (c) 91 (d)
2 (a) 12 (b) 22 (a) 32 (c) 42 (c) 52 (a) 62 (d) 72 (d) 82 (e) 92 (a)
3 (e) 13 (c) 23 (b) 33 (c) 43 (d) 53 (b) 63 (a) 73 (c) 83 (a) 93 (e)
4 (d) 14 (e) 24 (d) 34 (d) 44 (b) 54 (c) 64 (d) 74 (a) 84 (d) 94 (b)
5 (b) 15 (d) 25 (b) 35 (a) 45 (d) 55 (a) 65 (b) 75 (e) 85 (b) 95 (a)
6 (c) 16 (c) 26 (a) 36 (a) 46 (d) 56 (d) 66 (b) 76 (e) 86 (c) 96 (b)
7 (d) 17 (e) 27 (c) 37 (c) 47 (b) 57 (d) 67 (e) 77 (b) 87 (e) 97 (d)
8 (e) 18 (b) 28 (e) 38 (e) 48 (c) 58 (a) 68 (e) 78 (a) 88 (d) 98 (e)
9 (b) 19 (a) 29 (b) 39 (d) 49 (d) 59 (c) 69 (c) 79 (c) 89 (a) 99 (c)
10 (a) 20 (e) 30 (e) 40 (b) 50 (c) 60 (b) 70 (b) 80 (b) 90 (b) 100 (c)

Answers & Explanations


786 ´ 64 60 20 3
1. (c) ?= = 1048 3. (e) ´ ´ ´ ? = 450
48 100 100 5
9
2. (a) 3
13824 ´ ? = 864 Þ ´ ? = 450
125
3 450 ´ 125
24 ´ 24 ´ 24 ´ ? = 864 Þ?= = 6250
9
Þ 24 ´ ? = 864 196 ´ 948
4. (d) ? = 196 × 948 ÷ 158 = = 1176
158
864
Þ ?= 5. (b) ? = 3.5 + 11.25 × 4.5 – 32.5
24
= 3.5 + 50.625 – 32.5 = 54.125 – 32.5 = 21.625
\ ? = 36 × 36 = 1296
www.newspaperkorner.wordpress.com
Practice Set - 3 PS-33
www.newspaperkorner.wordpress.com 18. (b) According to the questions, third number will be
4096 ´ 56 64 ´ 56 = 5 × 213 – 2 × 233.5 – 2 × 271
6. (c) ?= = = 32
764 – 652 112 = 1065 – 467 – 542 = 56
19. (a) Let Sonali’s monthly income = ` x
7. (d) ? = (98360 + 25845 – 36540) ÷ 2500
Sonali’s percentage monthly spendings
= 87665 ÷ 2500 = 35.066 = (55 + 15)% = 70%
8. (e) ? = 7414 + 3698 + 1257+ 1869 = 14238 Percentage savings = 100 – 70 = 30%
ATQ,
9. (b) (91)2 + (41)2 – ? = 9858 \ 30% of x = 12750
Þ 8281 + 1681– ? = 9858 12750 ´ 100
Þ x= = ` 42500
30
Þ ? = 9962 – 9858 = 104
éæ r ö
t ù éæ 6 ö
3 ù
\ ? = 104 ×104 = 10816 20. (e) C .I . = P êç1 + ÷ - 1ú = 9650 ê ç 1 + ÷ - 1ú
êëè 100 ø úû êëè 100 ø úû
10. (a) ? = (2640 ÷ 48) × (2240 ÷ 35) = 9650 (1.191016 – 1)
= 55 × 64 = 3520 = 9650 × 0.191016 » ` 1843
21. (c) The rate of milk when milkman sells 120 litres of milk
121 117 108 92 67 31 for ` 3360
11. (a)
–2
2
–3
2
–4
2
–5
2
–6
2 æ 3360 ö
\ SP = ç = 28
è 120 ÷ø `
50 26 14 8 5 3.5 The rate of milk when milkman sells 240 litres of milk
12. (b) for ` 6120.
÷2+1 ÷2+1 ÷2+1 ÷2+1 ÷2+1
æ 6120 ö
\ SP = çè ÷ = 25.5
3 23 43 63 83 103 240 ø `
13. (c)
\ Required discount = (28 – 25.5) = ` 2.5
+20 +20 +20 +20 +20
22. (a) Let the number be x.
748 737 715 682 638 583 3626
ATQ, ´ 32 = 2368
14. (e) x2
–11 –22 –33 –44 –55
2 3626 ´ 32
Þ x = = 49
15. (d) 1 9 25 49 81 121 169 2368
\ x = 49 = 7
23. (b) Let the two digits number be 10n + m and n > m.
12 32 52 72 92 112 132 As given,
16. (c) Let the number of ducks and frogs in the pond be 37x n + m = 14
n–m=2
and 39x respectively.
On solving the equation,
ATQ, n = 8, m = 6
37 x + 39 x \ Product of digits = 8 × 6 = 48
= 152
2 24. (d) After servicing, speed of car = 60 km/h
\ Distance covered in 6 hours
152 = (60 × 6)km = 360 km
Þ 38x = 152 Þ x= =4
38 Before servicing, time taken to cover 360 km
\ Number of frogs = 39x
\ Time taken = 360 km = 7.2 hours
= 39 × 4 = 156 50 km/h
17. (e) The number of employees in companies A, B and C be
25. (b) Let Venkat has x ducks and y sheep.
4x, 5x and 6x respectively \ x + y = 81
After increase in the number of employees, required \ x = 81 – y ...(i)
ratio will be and 2x + 4y = 268
Þ 162 – 2y + 4y = 268
125 130 150 Þ 2y =268 – 162 = 106
= 4x ´ : 5x ´ : 6x ´
100 100 100 106
= 4 × 25 : 5 × 26 : 6 × 30 Þ y= = 53
2
= 10 : 13 : 18 \ Number of sheep = 53
www.newspaperkorner.wordpress.com
PS-34 Practice Set - 3
www.newspaperkorner.wordpress.com 37. (c) Statements : Some tables are chairs.
26. (a) x + y = 13 ...(i)
x–y=3 ...(ii) Conclusions : Some chairs are tables.
On adding, (conversion)
2x = 16 Hence, III follows.
Þ x=8 Statements : No cupboard is table.
Conclusions : Some cupboards are not table.
\ y= 5
(Implication)
\ Numbers are 85 and 58.
No table is cupboard.
27. (c) Length of the reel (conversion)
= (25 × 125 + 90) cm Statements : Some chairs are cupboards.
= 3215 cm = 32.15 m Conclusions : Some cupboards are chairs.
28. (e) 17 × 17 = 289 (conversion)
19 × 19 = 361 Since, No table is cupboard.
29. (b) Le the CP of the shoes be ` x. Some cupboards are chairs.
\ 2033 – x = x – 1063 Conclusions : Some chairs are not table.
Þ 2x = 2033 + 1063 = 3096 (E + I = O* type)
Hence, I follows.
3096 38. (e) Statements : No table is fruit.
Þx= = ` 1548
2 Conclusions : Some tables are not fruit.
30. (e) Amount received by each person (Implication)
No fruit is table. (conversion)
æ 158965 ö Statements : No fruit is window.
=` ç ÷ = ` 1325 Conclusions : Some fruits are not window.
è 120 ø
(Implication)
31. (b) Total number of users of brand B across all Five cities No window is fruit.
= 600 + 500 + 650 + 700 + 550 = 3000 (Conversion)
32. (c) 700 = x % of 500 Statements : No fruit is window
All windows are chairs.
x ´ 500 700
700 = Þx= = 140 Conclusions : Some chairs are not fruit.
100 5 (E + A = O* type)
500 + 550 + 600 + 550 + 700 Statements : All windows are chairs.
33. (c) Required average = Conclusions : Some windows are chairs.
5
(Implication)
= 580 Some chairs are windows.
34. (d) Required difference = 1250 – 1100 = 150 (Conversion)
500 Hence none follows.
35. (a) Required Ratio = = 5:7 39. (d) Statements : No man is sky.
700
Conclusions : Some men are not sky.
36. (a) Statements : All books are notes.
(Implication)
Conclusions : Some books are notes. No sky is man. (conversion)
(Implication) Statements : No sky is road.
Some notes are books. Conclusions : Some skies are not road.
(conversion) (Implication)
Hence I follows. No road is sky. (conversion)
Statements : Some notes are pencils. Hence II follows.
No pencil is paper. Statements : Some men are roads.
Conclusions : Some note are not paper. Conclusions : Some roads are men.
(I + E = O type) (conversion)
Statements : Some notes are pencils. No sky is man.
Conclusions : Some pencils are notes.
(conversion) Some men are roads.
Statements : No pencil is paper. Some roads are not sky.
Conclusions : Some pencils are not papers. (E + I = O* type)
(Implication)
No sky is road.
No paper is pencil.
(Conversion)
Since III and IV form a complementary I-E pair, either
Some roads are men.
of the two must follow. www.newspaperkorner.wordpress.com
Practice Set - 3 PS-35
www.newspaperkorner.wordpress.com
Some men are not sky. From (i), (ii) and (iii), we get
(E + I = O* type)
T = N ³ R > M ...(iv)
Statements : Some men are roads.
Hence, from (iv) we get N = R (conclusion I) or N > R
(conclusion II). Hence, either conclusion I or
conclusion II follows. Also, from (iv) we get N > M
No road is sky. (conclusion III). Hence, conclusion III follows.
Conclusions : Some men are not sky. 43. (d) Here, V ³ M ...(i)
(I + E = O type)
A> M ...(ii)
Statements : Some roads are man.
R£V ...(iii)
From (i) and (ii), we can’t get any specific relationship
between ‘V’ and ‘A’. Hence, conclusion II does not
No man is sky. follow. Again from (i) and (iii), we can’t get any specific
Conclusions : Some roads are not sky. relationship between ‘M’ and ‘R’. Hence, conclusion
(I + E = O type) III does not follow. On the similar basis we can’t get
Hence, only II follows any specific relationship between ‘A’ and ‘R’ using all
40. (b) Statements : All papers are books. (i), (ii) and (iii). Hence, conclusion I does not follow.
Conclusions : Some papers are books.
44. (b) Here, B = D ...(i)
(Implication)
Some books are papers. D³H ...(ii)
(conversion) H<F ...(iii)
Hence II follows. From (i) and (ii), we get
Statements : All bags are books. B = D ³ H ...(iv)
Conclusions : Some bags are books. Now, from (iii) and (iv), we can’t get any specific
(Implication)
relationship between B and F (or D and F). Hence, no
Some books are bags.
(conversion) conclusion follows. But, conclusion I and II make a
Statements : Some purses are bags. complementary pair. Hence, either conclusion I or
Conclusions : Some bags are purses. conclusion II follows.
(conversion) 45. (d) Here, J £ N ...(i)
Statements : Some purses are bags. ...(ii)
K³N
T>K ...(iii)
From (i), (ii) and (iii), we get
T>K³N³J ...(iv)
All bags are books. From (iv), we get J < T (conclusion I) and T > N
Conclusions : Some purses are books. (conclusion II). Hence, conclusion I and conclusion II
(I + A = I-type) follow. Conclusion III can be obtained from conversion
Some books are purses.
of (i). Hence, conclusion III follows.
(conversion)
For (Qs. 46-50)
Hence, III follows.
41. (b) Here, T > K ...(i)
K£R ...(ii) A
(Farmer)
R= M ...(iii)
From (ii) and (iii), we get
R =M³K ...(iv)
Daughter Sons
Now, from (iv) we get M > K (conclusion III) for M = K
(conclusion I). Hence, either conclusion I or conclusion
B (Unmarried)
III follows. Again, from (i) and (iv) we can’t get any (Teacher)
specific relationship between ‘M’ and ‘T’. Hence, Husband E (Unmarried)
D C (lawyer)
conclusion II does not follow. (Doctor) (Trader)
42. (c) Here, M < R ...(i)
R£T ...(ii) 46. (d) Clearly, B and D are the females members in the family.
T=N ...(iii) 47. (b) From the above table C and D the married couple.
www.newspaperkorner.wordpress.com
PS-36 Practice Set - 3
www.newspaperkorner.wordpress.com
48. (c) C and E are the male members in the family. 62. (d) ‘Offhand’ means fairness in ‘premeditation.’ ‘above
49. (d) D, the wife of the trader C is the doctor in the family. board’ has the same sense with the term competition.
50. (c) C is the trader in the family. 63. (a) The first four letters are D, E, C, I and only word DICE
51. (b) EG4 BH75 @K 8D N£Q Z$W3C1 9* 1B2S6 can be formed so the answer is (a).
52. (a) The first, second and third element of each group is 64. (d) The new letter sequence is
sixth element to the right of the respective element of NOITARTNECNOC
previous group as given in all in the sequence.
The eighth letter from the end is R.
53. (b) There are 27 elements in all in the sequence.
65. (b) The order in which the five boys reach the finishing
So, (27 – 9 – 7 = ) 11 elements are between the 9th from
line is Gaurav, Raj, Mohit, Ashish, Sanchit.
left and 7th from right.
Hence, ( 9 + 6 =) 15th element from the left and will be Hence Gaurav won the race.
the required answer. 66. (b) In each successive problem figure design ‘ ’ rotates
54. (c) 7th to the left of 12th from right anticlockwise through 45°, 90°, 45°, 90° respectively
= (12 + 7 =) 19th from right and a new design of its left and then right is added
= (27 – 19 + 1 = ) 9th from left respectively. Thus the answer figure (e) is obtained.
But the first 15 elements are reversed. 67. (e) From problem figures 1 to 4 two designs of right change
= (15 – 9 + 1 =) 7th from left in the original sequence = 5. their places with each other and from problem figure 2
55. (a) For the condition to be fulfilled, three digits should be to 5, two designs of left change their places with each
together but it is not so in the given sequence. other. In the same way from problem figure 3 to 6 two
56. (d) 4, 45, 453, 4531, 45312, 45, 453, 4531 designs of right will change their places with each other.
The next coded digit will be 1. Hence, the instruction Thus the answer figure (e) is obtained.
Run will come next. 68. (e) From problem figure 1 to 2 and 3 to 4 the design turning
over horizontally situates toward right side and the
57. (d) 1 3 4 7 9 2 5 6 8 design rotating through 90° anticlockwise situates in
A Q F J L D M P N the middle. As per this rule the answer figure (e) is
obtained.
69. (c) In each subsequent figure design change their places
3 9 6 8 2 4 as below:
Thus, Q L P N D F
1
2
O V E R V I S T 3
58. (a) $ # % * # + ´ – 4
5
From above table, SORE is coded as :
From problem 1 to 2 designs 1 and 2
S O R E From problem 2 to 3 designs 3 and 4
´ $ * % From problem 3 to 4 designs 5 sets up at top
59. (c) The sister of one’s mother is one’s maternal aunt. From problem 4 to 5 designs 1 and 2
Hence the man is the husband of the boy’s maternal and 5 to 6 design 3 and 4 change their places with each
aunt. other thus the answer figure (c) is obtained.
70. (b) In the consecutive figure, the design '· ' comes at
60. (b) B 20km A middle from lower right, at upper left from middle and
W than at middle. The design ‘ ’ comes at left middle
15 km from upper middle, at right from left middle and at upper
S N
middle from right middle. A new design comes lower
My House E left from middle.
25km

C 20km D

From the above diagram required distance


= 25 – 15 = 10 km.

www.newspaperkorner.wordpress.com
www.newspaperkorner.wordpress.com

Preliminary Exam
Practice Set - 4
INSTRUCTIONS
• This Preliminary Exam practice set consists of three sections. Quantitative Aptitude (Qs. 1-35); Reasoning
Ability (Qs. 36-70) and English Language (Qs. 71-100).
• All the questions are compulsory.
• Each question has five options, of which only one is correct. The candidates are advised to read all the
options thoroughly.
• There is negative marking equivalent to 1/4th of the mark allotted to the specific question for wrong answer.

Time : 1 hrs. Max. Marks : 100

QUANTITATIVE APTITUDE 7. Sophia invests 25% of her monthly salary in insurance


policies. She spends 15% of her monthly salary in shopping
DIRECTIONS (Qs. 1-5) : What should come in place of question and 35% of her salary on household expenses. She saves
mark (?) in the following number series? the remaining amount of ` 9,050. What is Sophia’s annual
1. 36 20 ? 8 6 5 income?
(a) 10 (b) 12 (a) ` 84,500 (b) ` 5, 30, 000
(c) ` 3, 25, 200 (d) ` 4, 34, 400
(c) 14 (d) 16
(e) None of these
(e) None of these
8. The number of employees in companies A, B and C are in a
2. 668 656 632 584 ? 296 ratio of 3 : 2 : 4 respectively. If the number of employees in
(a) 392 (b) 438 the three companies is increased by 20%, 30% and 15%
(c) 488 (d) 536 respectively, what will be the new ratio of employees
(e) None of these working in companies A, B and C respectively ?
3. 1 121 441 961 1681 ? (a) 18 : 13 : 24 (b) 13 : 18 : 23
(a) 2701 (b) 2511 (c) 17 : 3 : 23 (d) 18 : 11 : 23
(c) 2611 (d) 2801 (e) None of these
9. The ages of Vaibhav and Jagat are in the ratio of 12 : 7
(e) None of these
respectively, After 6 years the ratio of their ages will be 3 :
4. 9 49 201 1009 ? 20209 80841 2. What is the difference in their ages?
(a) 4054 (b) 4049 (a) 8 years (b) 12 years
(c) 4050 (d) 4041 (c) 9 years (d) 10 years
(e) None of these (e) None of these
5. 31 35 44 60 85 ? 10. What is the least number to be added to 8008 to make it a
(a) 121 (b) 111 perfect square?
(c) 109 (d) 97 (a) 273 (b) 87
(e) None of these (c) 264 (d) 92
6. The average of five positive numbers is 308. The average of (e) None of these
first two numbers is 482.5 and the average of last two 11. The product of two consecutive odd numbers is 6723, What
numbers is 258.5. What is the third number? is the square root of the smaller number?
(a) 224 (b) 58 (a) 9 (b) 729.
(c) 121 (d) Cannot be determined (c) 6561 (d) 81
(e) None of these (e) None of these
www.newspaperkorner.wordpress.com
PS-38 Practice Set - 4
www.newspaperkorner.wordpress.com
12. 60 per cent of first number is 40 per cent of the second 21. I. 12x2 + 11x + 12 = 10x2 + 22x
number. What is the respective ratio of the first number to II. 13y2 – 18y + 3 = 9y2 – 10y
the second number? 18 6 12 8
(a) 2 : 3 (b) 21 : 31 22. I. 2
+
- 2 = 2
x x x x
(c) 7 : 10 (d) Cannot be determined
II. y3 + 9.68 + 5.64 = 16.95
(e) None of these
13. The owner of a book shop charges his customer 28% more 23. I. 1225x + 4900 = 0
than the cost price. If a customer paid ` 1,408 for some II. (81)1/4 y + (343)1/3 = 0
books, then what was the cost price of the books ?
(a) ` 1,100 (b) ` 1,111 (2)5 + (11)3
24. I. I. = x3
(c) ` 1,110 (d) ` 1,000 6
(e) None of these II. 4y3 = – (589 ¸ 4) + 5y3
14. The difference between 56% of a number and 39% of the 25. I. (x7/5 ¸ 9) = 169 ¸ x3/5
same number is 425. What is 63% of that number?
II. y1/4 × y1/4 × 7 = 273 ¸ y1/2
(a) 1525 (b) 1650
(c) 1700 (d) 1575 DIRECTIONS (Qs. 26-30) : What should come in place of the
(e) None of these question mark (?) in the following questions?
15. Find the average of the following set of scores:
456, 328, 489, 453, 511, 328, 222, 205 26. (84)2 - (67)2 + ? = 2588
(a) 374 (b) 388 (a) 361 (b) 529
(c) 362 (d) 391 (c) 441 (d) 625
(e) None of these (e) None of these
27. 668 ÷ 167 × 284 = ?
DIRECTIONS (Qs. 16-20) : What approximate value should come (a) 1156 (b) 1136
in place of the question mark (?) in the following questions? (c) 1096 (d) 1116
(You are not expected to calculate the exact value.) (e) None of these
16. [(1.3)2 ´ (4.2)2 ] ¸ 2.7 = ? 28. 3
10648 ´ 3 5832 = ?
(a) 7 (b) 21 (a) 396 (b) 216
(c) 18 (d) 11 (c) 432 (d) 576
(e) 16 (e) None of these
17. 746 ¸ 32 ´15 = ? 5
29. 60% of 25% of th of ? = 630
(a) 350 (b) 345 6
(c) 355 (d) 340 (a) 5060 (b) 5200
(e) 335 (c) 4880 (d) 4500
(e) None of these
18. 834 ´ 349 = ? 30. (85410 + 36885 + 24705) ÷ 1600 = ?
(a) 525 (b) 556 (a) 90.25 (b) 94.386
(c) 534 (d) 550 (c) 95.50 (d) 91.875
(e) 540 (e) None of these
19. (3986 + 2416 + 3897) ¸ 754 = ? 31. What amount of compound interest can be obtained on an
amount of ` 8, 840 at the rate of 5% p.a at the end of 3 years?
(a) 18 (b) 14
(a) ` 1,393.405 (b) ` 1,326
(c) 11 (d) 9
(c) ` 1,384.50 (d) ` 1340
(e) 21
(e) None of these
20. 41.25 + 11.085 × 2.75 = ? 32. A trader sells 150 metres of cloth for ` 6, 600 and he sells
(a) 63 (b) 67 300 metres of cloth for ` 12, 750. How much concession
(c) 76 (d) 72 does the trader give per metre of cloth, when he sells 300
(e) 80 metres of cloth?
DIRECTIONS (Qs. 21-25) : In the following questions, two (a) ` 3 (b) ` 2.5
equations numbered I and II are given. You have to solve both the (c) ` 1.5 (d) ` 2
equations and give answers. (e) None of these
33. When 3888 is divided by the square of a number and the
(a) if x > y answer so obtained is multiplied by 21, the final answer so
(b) if x ³ y obtained is 252. What is the number?
(c) if x < y (a) 324 (b) 16
(d) if x £ y (c) 256 (d) 144
(e) if x = y or the relationship cannot be established (e) None of these
www.newspaperkorner.wordpress.com
Practice Set - 4 PS-39
www.newspaperkorner.wordpress.com
34. The sum of the digits of a two digit number is 14. The 42. ENGLAND is written as 1234526 and FRANCE as 785291.
difference between the first digit and the second digit of How will GREECE be written in this coding scheme ?
the two digit number is 4. What is the two digit number ? (a) 381191 (b) 381911
(a) 86 (b) 95 (c) 394132 (d) 562134
(c) 68 (d) 77 (e) None of these
(e) None of these 43. If table is called chair, chair is called cot, cot is called pot
35. A car runs at the speed of 40 when not serviced and runs at and pot is called filter, where does a person sit?
65 kmph. when serviced. After servicing, the car covers a (a) pot (b) cot
certain distance in 5 hours. How much approximate time (c) chair (d) filter
will the car take to cover the same distance when not
(e) None of these
serviced?
44. Pointing to a photograph Arun said, ‘She is the mother of
(a) 10 (b) 7
my brother’s son’s wife’s daughter.’ How is Arun related
(c) 12 (d) 8
to the lady's husband?
(e) 6
REASONING ABILITY (a) Uncle (b) Daughter-in-law
(c) Cousin (d) Brother
36. ‘Talk’ is related to ‘Speak’ in a certain way. Similarly, ‘Honest’ (e) None of these
is related to ‘Truthful’. Following the same logic, ‘Listen’ is 45. Five boys are standing in a row facing East. Deepak is to
related to ‘.............’. the left of Sameer, Tushar and Shailendra. Sameer, Tushar
(a) Music (b) Ears and Shailendra are to the left of Sushil. Shailendra is between
(c) Hear (d) Ignore Sameer and Tushar. If Tushar is fourth from the left, then
(e) Sound how far is Sameer from the right?
37. Three of the following are alike in a certain way and form a (a) First (b) Second
group. Find the odd one out. (c) Third (d) Fourth
(a) Bird (b) Insect (e) None of these
(c) Aeroplane (d) Kite
DIRECTIONS (Qs. 46-50) : In each of the questions below are
(e) None of these
given three statements followed by the conclusions numbered I,
38. Arrange the given words in alphabetical order and tick the
II, III & IV. You have to take the given statements to be true even
one that comes in the middle.
if they seem to be at variance with commonly known facts and
(a) Restrict (b) Rocket then decide which of the given conclusions logically follow from
(c) Robber (d) Random the given statements. Give answer
(e) Restaurant
46. Statements : Some pots are buckets.
39. Select the combination of numbers so that letters arranged
Some buckets are bags.
accordingly will form a meaningful word.
Some bags are purses.
R A C E T
Conclusions : I. Some purses are buckets.
1 2 3 4 5
II. Some bags are pots.
(a) 1, 2, 3, 4, 5 (b) 3, 2, 1, 4, 5
III. Some purses are pots.
(c) 5, 2, 3, 4, 1 (d) 5, 1, 2, 3, 4
IV. Some pots are bags.
(e) None of these
(a) All follow (b) None follows
40. Veena walked 5m towards north, took a left turn and walked
(c) Only I and III follow (d) Only II and IV follow
7 m. She took a left turn again and walked 8m before taking
a left turn and walking 7 m. She then took a final left turn (e) None of these
and walked 1 m before stopping. How far is Veena from the 47. Statements : All glasses are roads.
starting point ? No road is stick.
(a) 3 m (b) 6 m Some sticks are pens.
(c) 4 m (d) 2 m Conclusions : I. Some glasses are sticks.
(e) 7 m II. Some pens are sticks.
41. In a school, the following codes were used during physical III. Some roads are sticks.
exercise. 1 means start walking, 2 means keep standing, 3 IV. No glass is a stick.
means start running at the same spot, 4 means sit down. (a) None follows
How many times a student, who performs the following (b) Only I or IV and II follow
sequence without error from the begining to the end, has to (c) Only either I or II and IV follows
sit down ? (d) Either I or II follows
1 2 3 4 2 3 1 4 4 3 2 2 1 2 4 3 1 4 4 12 (e) None of these
(a) 2 (b) 3 48. Statements : Some ice is ring.
(c) 6 (d) 5 No ring is paint.
(e) None of these Some rings are gold.
www.newspaperkorner.wordpress.com
PS-40 Practice Set - 4
www.newspaperkorner.wordpress.com
Conclusions : I. No gold is paint. DIRECTIONS (Qs. 56-61) : In each of the questions given below
II. No ice is gold. a group of digits is given followed by four combinations of
III. Some rings are paints. letters/symbols. You have to find out which of the four
IV. All golds are ring. combinations correctly represents the group of digits based on
(a) None follows (b) Only I and III follow the letter/symbol codes and the conditions given below. If none of
(c) Only I and II follow (d) Only III and IV follow the four combinations represents the group of digits correctly,
(e) None of these give (e) i.e. “None of these” as the answer.
49. Statements : No candle is bell. Digit: 3 9 6 2 8 7 5 4 1
Some shoes are bells. Symbol : K T $ F H # % D M
All tables are shoes.
Conditions for the coding the group of digits:
Conclusions : I. Some tables are bells.
1. If the first digit is odd and last digit is even, the codes for
II. No table is bell.
the first and the last digits are to be interchanged.
III. Some shoes are candles.
2. If the first as well as the last digit is even, both are to be
IV. No flower is fruit. coded by the code for last digit.
(a) Only I and IV follow (b) Only I and II follow 3. If the first as well as the last digit is odd, both are to be
(c) Only III and IV follow (d) Either II or III follows coded as ‘X’.
(e) None of these 56. 564923
50. Statements : Some cats are rats. (a) %$DTFK (b) K$DTFK
Some rats are ants. (c) X$DTFX (d) K$DTF%
Some ants are flies. (e) None of these
Conclusions : I. Some flies are ants. 57. 658247
II. Some ants are not rats. (a) $%HFD# (b) #%HFD$
III. No rat is fly. (c) %$HFD# (d) %#HFD$
IV. No cat is fly. (e) None of these
(a) Only I and IV follow (b) Only II follows 58. 436958
(c) Only I and II follow (d) Only IV follows (a) DK$T%D (b) DK$T%H
(e) None of these (c) HK$T%H (d) #%$HK#
(e) None of these
DIRECTIONS (Qs. 51-55) : On the basis of the following
59. 756834
information, answer the questions that follow.
(a) #%$HKD (b) D%$HK#
Six people are sitting on the ground in a hexagonal shape. (c) D%$HKD (d) #%$HK#
The hexagon’s vertices are marked as A, B, C, D, E and F but not (e) None of these
in any order. However, all the sides of the hexagon are of same 60. 291378
length. A is not adjacent to B or C; D is not adjacent to C or E; B (a) FTMK#H (b) XTMK#X
and C are adjacent; F is in the middle of D and C.
(c) HTMK#F (d) FTMK#F
51. If one neighour of A is D, then who is the other one ?
(e) None of these
(a) B (b) C
61. 128547
(c) E (d) F
(a) XFH%DX (b) XFH#DX
(e) None of these
(c) MFH%DX (d) XFH%D#
52. Who is placed opposite to E ? (e) None of these
(a) F (b) D
(c) C (d) B DIRECTIONS (Qs. 62-65) : In the following questions the symbols
$, @, *, # and ? are used with the following meanings.
(e) None of these
53. Who is at the same distance from D as E is from D ? A $ B means A is greater than B.
(a) B (b) C A @ B means A is either greater than or equal to B.
(c) D (d) F A * B means A is equal to B.
A # B means A is smaller than B.
(e) None of these
A ? B means A is either smaller than or equal to B.
54. Which of the following is not a correct neighbouring pair ?
Now in each of the following questions assuming the given
(a) B & F (b) C & F
statements to be true, find which of the two conclusions I and II
(c) D & F (d) A & E
given below them is/are definitely true ? Given answer
(e) None of these
(a) If only conclusion I is true.
55. Which of the following is not a correct neighbouring triplet?
(b) If only conclusion II is true.
(a) B, C, F (b) A, F, B (c) If neither I nor II is true.
(c) D, A, B (d) F, A, E (d) If both I and II are true.
(e) None of these (e) None of these
www.newspaperkorner.wordpress.com
Practice Set - 4 PS-41
www.newspaperkorner.wordpress.com
62. Statements : M # N, T $ U, N # U One day, a widely respected sage who had been to the
Conclusions : I. M ? T Himalayas came to their town. Govind asked him about the potion.
II. T $ N To his surprise the sage answered, “I have learnt how to brew
63. Statements : P $ T, G ? N, T @ N such a potion. But it is a difficult process.” “Tell me!” insisted
Conclusions : I. P $ N Govind, hardly able to believe his luck. “You have to collect the
II. G ? T dew which settles on the leaves of a banana tree every morning
64. Statements : P ? Q, R $ S, Q @ S during winter. There is a condition, though. The tree should be
Conclusions : I. P $ S planted and watered regularly with your own hands. Store the
II. R # Q collected dew in an earthen vessel and when you have five litres,
65. Statements : D @ F, G $ H, F ? H bring it to me. I will recite a sacred mantra to transform the dew
Conclusions : I. G $ F into the potion. A drop of the potion will be sufficient to change
II. D @ H any object into gold.”
DIRECTIONS (Qs. 66-70): Study the following arrangement Govind was worried “Winter is only for a few months in the
carefully and answer the questions given below : year. It will take me years to collect the dew.” “You can plant as
many trees as you want,” replied the sage. Govind went home
P1%TR A 5#DM7K EG28$H314VU6F 9Z and after talking to his wife, began clearing the large fields which
66. How many such symbols are there in the above arrangment, has been lying vacant for years. He planted rows of banana
each of which is immediately preceded by a consonant and saplings. He tended them with great care. His wife helped him
also immediately followed by a consonant ? too. She would take the banana crop to market and get a good
(a) None (b) One price. Over the years the plantation grew and finally after six
(c) Two (d) Three years Govind had five litres of dew. He went to the sage who
(e) More than three smiled, uttered a mantra and sprinkled a few drops of dew on a
67. Four of the following five are alike in a certain way based on copper vessel. To Govind’s dismay, nothing happened. “you have
their position in the above arrangement. Which is the one cheated me!” he shouted at the sage.
that does not belong to that group ? The sage however smiled. Govind’s wife then came forward
(a) V1F (b) EK8 with a box. The sage opened it and revealed stacks of gold coins
(c) R % # (d) 6V9 inside. Turning to Govind he said, “you worked hard on your
(e) $G3 land and created a plantation. Your wife sold the produce in the
68. How many such vowels are there in the above arrangement, market. It was your hard work which created this wealth, not
each of which is immediately preceded by a digit and imme- magic. If I had told you this earlier, you would not have listened.”
diately followed by a consonant ? Govind understood the wisdom behind the sage’s words and
(a) None (b) One worked even harder from that day on.
(c) Tow (d) Three 71. Why did Govind’s father give him large tracts of land?
(e) More than three (a) It was his way of instilling a sense of responsibility in
69. Which of the following is exactly in the middle between the his son
fifth element from the left end and the seventh element from (b) Govind was his only son and sole heir
the right end? (c) To provide Govind with sufficient funds to pursue his
(a) G (b) 2 interest of discovering a magic potion
(c) E (d) (d) He wanted Govind to continue to look after the tenants
(e) None of these (e) None of these
70. If the positions of last twelve elements in the above ar- 72. Which of the following can be said about the sage?
rangement are reversed, which of the following will be the (a) He was cunning and plotted with Govind’s wife to
eigth element to the right of the eleventh element from the cheat him.
left ? (b) He had no magical powers as such and used to swindle
(a) H (b) I people
(c) (d) 9 (c) He was a good judge of people
(e) None of these (d) He did not deserve his good reputation
ENGLISH LANGUAGE (e) He was dishonest because he had cheated Govind out
of his gold
DIRECTIONS (Qs. 71-75) : Read the following passage carefully 73. Why was Govind’s wife worried ?
and answer the questions given below it. (a) Govind had no knowledge of farming and could not
cultivate the land he had inherited from his father
Govind’s father was a rich landlord, who was loved and
(b) Govind had not friends because he was obsessed with
respected by all his tenants. When he died, he left large tracts of
finding a potion which would turn any thing into gold
land to Govind. But Govind did not spend a single day looking
(c) Govind was only interested in studying under different
after his land. He had a funny idea, that there existed a magic
sages and neglected his family duties
potion which, if it was poured on any object, would turn it into
(d) Since Govind had devoted all his time and wealth to
gold. He spent all his time trying to learn more about this potion.
finding a magic potion, they would soon be poor
People took advantage of him and cheated him. His wife grew
(e) Govind’s experiments to find a magic potion were
anxious. Given the amount of money Govind was spending, she
dangerous
was sure that they would soon be paupers. www.newspaperkorner.wordpress.com
PS-42 Practice Set - 4
www.newspaperkorner.wordpress.com
74. Why did Govind’s wife help him in the fields? 84. (a) severe (b) no
A. To support her husband in his endeavour to find a (c) additionally (d) variety
magic potion. (e) plenty
B. The sage had advised her to help her husband succeed.
85. (a) time (b) process
C. He needed someone to help him collect the dew.
(a) Only (B) (b) Only (A) (c) return (d) event
(c) Both (A) and (B) (d) All (A), (B) and (C) (e) action
(e) None of these 86. (a) neglected (b) abandoned
75. Why did Govind decide to cultivate a banana crop? (c) defaulted (d) depended
(a) The soil of his land was suitable only for cultivating bananas
(e) disappointed
(b) It was the most highly priced commodity in the region
(c) It could be grown at any time of the year including winter 87. (a) benefit (b) easier
(d) His wife pressurised him to do so (c) reckless (d) disorganised
(e) The ingredient for the magic potion could only be (e) secure
obtained from a banana tree 88. (a) sense (b) confidence
DIRECTIONS (Qs. 76-80) : Read each sentence to find out (c) challenge (d) doubt
whether there is any grammatical error or idiomatic error in it. (e) believe
The error, if any, will be in one part of the sentence. The number
of that part is the answer. If there is no error, the answer is (e). DIRECTIONS (Qs. 89-93): Read this sentence to find out whether
(Ignore errors of punctuation, if any.) there is any grammatical mistake/error in it. The error, if any,
76. His proposal had (a) / to be send to (b) / the President of the will be in one part of the sentence. Mark that part with the error
company (c) / for her approval (d). No error (e). as your answer. If the sentence is correct as it is, mark 'No
77. Each tuesday evening we visited (a) / the farmers in the error' as your answer. (Ignore the errors of punctuation if any.)
area (b) / and held a meeting (c) / to discuss the problems 89. A red and sore tongue/is an indicator from/lack of iron
they faced (c). No error (e).
Vitamin-B 12/in the body.
78. Though our training facilities (a) / are limited only a (b) / few
(a) a red and sore tongue
employees have been (c) / selected for training (d). No error (e).
(b) is an indicator from
79. During the interview (a) / the panel asked me (b) / several technical
(c) lack of iron and Vitamin-B12
questions (c) / and I answered all of it (d). No error (e).
80. He decided to work for (a) / an NGO, but most of his (b) / (d) in the body
classmates opted for high paid (c) / jobs in multinational (e) No error
companies (d). No error (e). 90. In the high-strung life/of over- crowded metros/there a
constantly tug of war/over space and resources.
DIRECTIONS (Qs. 81-88) : In the following passage there are (a) in the high-strung life
blanks each of which has been numbered. These numbers are (b) of over-crowded metros
printed below the passage and against each, five words are (c) there a constantly tug of war
suggested, one of which fits the blank appropriately. Find out the (d) over space and resources
approptiate words in each case. (e) No error
When we 81 started thirty years ago in 1977, we did not 91. The foremost criterion of selection we adopted/were the
know anything about how to run a bank for the poor. We therefore number of years of training/a singer had received/under a
looked at how others ran their operations and 82 from their particular guru.
mistakes. In Bangladesh, conventional banks and credit co- (a) The foremost criterion of selection we adopted
operatives always 83 lump sum repayments. This created 84 (b) were the number of years of training
problems because repaying in a lump sum was a mental hurdle for (c) a singer had received
borrowers. They tended to delay repayment and get further into (d) under a particular guru
debt in the 85. In the end they usually 86 totally on the loan, (e) No error
which was a loss to the bank. In structuring our own loans, I 92. Excess weight is the result of/unhealthy eating habits/which
decided to ask for a daily payment, Monitoring repayment was
are inherent risk factors/responsible for many diseases.
87 and it filled people with 88 that they could repay their loans.
(a) excess weight is the result of
81. (a) firstly (b) freshly
(b) unhealthy eating habits
(c) foremost (d) initially
(c) which are inherent risk factors
(e) recently
(d) responsible for many diseases
82. (a) copied (b) observed
(e) No error
(c) learned (d) understood 93. The therapeutic benefits/at helping others/have long been/
(e) improving recognised by people.
83. (a) asked (b) insisted (a) the therapeutic benefits (b) at helping others
(c) demanded (d) settled (c) have long been (d) recognised by people
(e) lend (e) No error
www.newspaperkorner.wordpress.com
Practice Set - 4 PS-43
www.newspaperkorner.wordpress.com 97. Which of the following should be the THIRD sentence
DIRECTIONS (Qs. 94-98): Rearrange the given five sentences
(A, B, C, D) and (E) in a proper sequence so as to form a after rearrangement?
meaningful paragraph and then answer the given questions. (a) D (b) B
(c) C (d) E
A. With so many products and opportunities available in the (e) A
market, it is very easy to get this planning wrong. 98. Which of the following should be the FOURTH sentence
B. Planning, therefore, is imperative and should begin as early after rearrangement?
as possible. (a) A (b) B
C. What amount will we need and when will we need it? (c) E (d) D
D. Most of us would put our children's education above any (e) C
other priority in life including our own retirement.
DIRECTIONS (Qs. 99 & 100): Which phrase should replace
E. So, let's try to find the best solution by asking two important
the phrase given in bold in the sentence to make it grammatically
question.
correct? If the sentence is correct as it is given, then mark ‘No
94. Which of the following should be the SECOND sentence
correction required’ as your answer.
after rearrangement?
(a) D (b) B 99. In some cases, factors like low salary, lack of growth
(c) C (d) E prospects and lack of motivation compel all employee to
(e) A look for a change.
95. Which of the following should be the FIFTH sentence after (a) compel those employees
rearrangement? (b) compelling all employees
(a) A (b) B (c) compelling the employee
(c) C (d) E (d) compel employees
(e) D (e) No correction required
96. Which of the following should be the FIRST sentence after 100. The easiest way for prevent stress caused by work or home
rearrangement? pressures is to indulge in high levels of physical activity.
(a) A (b) B (a) easily way to (b) easier ways for
(c) E (d) C (c) easiest way to (d) easier way from
(e) D (e) No correction required

Answer Key
1 (b) 11 (a) 21 (b) 31 (a) 41 (c) 51 (c) 61 (a) 71 (b) 81 (d) 91 (b)
2 (c) 12 (a) 22 (c) 32 (c) 42 (a) 52 (a) 62 (b) 72 (c) 82 (c) 92 (e)
3 (e) 13 (a) 23 (a) 33 (e) 43 (b) 53 (b) 63 (d) 73 (d) 83 (a) 93 (b)
4 (d) 14 (d) 24 (a) 34 (b) 44 (a) 54 (a) 64 (c) 74 (e) 84 (a) 94 (b)
5 (a) 15 (a) 25 (d) 35 (d) 45 (d) 55 (a) 65 (a) 75 (e) 85 (b) 95 (d)
6 (b) 16 (d) 26 (c) 36 (c) 46 (b) 56 (c) 66 (a) 76 (b) 86 (c) 96 (e)
7 (d) 17 (a) 27 (b) 37 (b) 47 (e) 57 (a) 67 (e) 77 (e) 87 (b) 97 (e)
8 (e) 18 (e) 28 (a) 38 (a) 48 (a) 58 (c) 68 (a) 78 (a) 88 (b) 98 (c)
9 (d) 19 (b) 29 (e) 39 (d) 49 (e) 59 (b) 69 (c) 79 (d) 89 (b) 99 (d)
10 (d) 20 (d) 30 (d) 40 (d) 50 (c) 60 (e) 70 (d) 80 (c) 90 (c) 100 (c)

Answers & Explanations


1. (b) 36 20 12 8 6 5
E55555F E55555
F E555F E5555F E55555
F 4. (d) 9 49 201 1009 4041 20209 80841
¸ 2+ 2 ¸2 + 2 ¸2 + 2 ¸2 + 2 ¸2 + 2 E55F E555F E555F E555F E55555F E555F
´5 + 4 ´4 +5 ´5 + 4 ´4 + 5 ´5+ 4 ´4 + 5
2. (c) 668 656 632 584 488 296
E55555F E5555F E5555F E5555F E5555F 5. (a) 31 35 44 60 85 121
-12 -24 -48 -96 -192 E55555F E55555
F E5555F E5555F E55555F
E55555F E5555F E5555F E5555F + 22 +32 +42 +52 +62
´2 ´2 ´2 ´2
6. (b) Third number
3. (e) 1 121 441 961 1681 2601 = 5 × 308 – 2 × 482.5 – 2 × 258.5
­ ­ ­ ­ ­ ­
12 112 212
= 1540 – 965 – 517 = 58
312 412 512
www.newspaperkorner.wordpress.com
PS-44 Practice Set - 4
www.newspaperkorner.wordpress.com
7. (d) Let Sophia’s monthly salary = `. x. \ 63 % of x
ATQ, 42500 63
Sophia’s % monthly expenditure = ´ = 1575
= (25 + 15 + 35)% = 75% 17 100
Saving % = 100 – 75 = 25% 15. (a) Required average
\ 25% of x = 9050 456 + 328 + 489 + 453 + 511 + 328 + 222 + 205
Þ x = 9050 × 4 = ` 36200 =
8
\ Sophia’s annual income
= ` (12 × 36200) = ` 434400 2992
= = 374
8. (e) Let the number of employees in the companies A, B 8
and C be 3x, 2x and 4x respectively.
Required ratio 1.69 ´ 17.64
16. (d) ? = = 11.04 » 11
3x ´ 120 2 x ´ 130 4 x ´ 115 2.7
= : : 17. (a) ? = 23.31 × 15 = 350
100 100 100
= 18 : 13 : 23 18. (e) ? = 28.88 × 18.68 = 539.52 » 540
9. (d) Let the present ages of Vaibhav and Jagat be 12x and 10299 10300
7x years respectively. 19. (b) ? = » » 14
754 750
According to the question, 20. (d) ? = 41.25 + 30.48
12x + 6 3 = 71.5 » 72
=
7x + 6 2 21. (b) I. 12x2 + 11x + 12 = 10x2 + 22x
Þ 24x + 12 = 21x + 18 2x2 – 11x + 12 = 0
Þ 24x – 21x 2x2 – 8x – 3x + 12 = 0
= 18 – 12 (x – 4) (2x – 3) = 0
Þ 3x = 6 x = 4, x = 3/2
II. 13y2 – 18y + 3 = 9y2 – 10y
6 4y2 – 8y + 3 = 0
Þx= =2
3 4y2 – 6y – 2y + 3 = 0
\ Required difference = 12x – 7x = 5x = 5 × 2 (2y – 3) (2y – 1) = 0
= 10 years 3 1
y= ,
10. (d) 8008 º 89.5 2 2
892 = 7921; 902 = 8100 \x³y
\ Required number
18 6 12 8
= 8100 – 8008 = 92 22. (c) + - =
2 x x2 x2
11. (a) 81 × 83 = 6723 x
\ Smaller number = 81 18 + 6x - 12 8
Now, 81 = 9 Þ = Þ 6x + 6 = 8
2
x x2
12. (a) Let the first number be x and the second number be y.
According to the question, 2
\ x== 0.33
60 40 6
x´ = y´ II. y3 + 9.68 + 5.64 = 16.95
100 100
Þ y3 = 16.95 – 15.32
3x 2y
Þ = Þ y3 = 1.63 = y = 3 1.63
5 5
x 2 23. (a) I. 35x + 70 = 0
Þ =
y 3 -70
\x= = -2
13. (a) CP of the books 35
II. (81)1/4 y + (343)1/3 = 0
æ 100 ö Þ 3y + 7 = 0 Þ 3y = – 7
=` ç ´ 1408 ÷
è 128 ø
7
= ` 1100 \ y= – = – 233 \ x > y
14. (d) Let the number be x. 3
According to the question,
( 2 )5 + (11)3
(56 – 39)% of x = 425 24. (a) I. = x3
6
x ´17
Þ = 425 32 + 1331 1363
100 Þ = x3 Þ = x3
6 6
425 ´100 42500
Þx= = \ x3 = 227.167
17 17
www.newspaperkorner.wordpress.com
Practice Set - 4 PS-45
www.newspaperkorner.wordpress.com x–y=4 ..... (ii)
-589 589
II. 4y3 = + 5y3 Þ = y3 From equation (i) & (ii), we get
4 4 y=5
\ y3 = 147.25 \ x > y Now, x + y = 14
25. (d) I. x7/5 ¸ 9 = 169 ¹ x3/5 \ x = 14 – 5 = 9
x 7 / 5 169 Thus, required two-digit number
= = 10 x + y = 10 × 9 + 5
9 x3 / 5 = 90 + 5 = 95
Þ x 10/5 = 9 × 169 Þ x2 = 9 × 169 35. (d) After servicing, the distance covered in 5 hours
x = ± ( 3 ´13) = ±39 = 65 × 5 = 325 km.
Without servicing, speed = 40 km/h
273
II. y1/4 × y1/4 × 7 = Distance 325
y1/ 2 \ Time = = = 8 hours
Speed 40
273 36. (c) As talk is related to speak and honest to truthful
y= = 39 similarly listen is to hear.
7
x£y 37. (b) All except the insect fly in the sky.
38. (a) Arranging the words in alphabetical order, we have
26. (c) Þ (84 + 67) (84 – 67) + ? = 2588 Random, Restaurant, Restrict, Robber, Rocket.
Þ 151 × 17 + ? = 2588 So the work in the middle is Restrict and the correct
answer is (a).
Þ ? = 2588 – 2567 = 21 39. (d) Clearly, the given letters, when arranged in the order 5,
\ ? = 21 × 21 = 441 1, 2, 3, 4 from the word ‘TRACE’.
27. (b) ? = 4 × 284 = 1136
N 7m
28. (a) ? = 3 10648 ´ 3 5832 = 22 × 18 = 396
8m 5m
60 25 5
29. (e) ´ ´ ´ ? = 630 40. (d) W E
100 100 6 2m
\ ? = 8 × 630 = 5040
1m
30. (d) ? = 147000 ¸ 1600 = 91.875 S 7m
éæ r ö
t ù 41. (c) Code for sit down = 4.
31. (a) C.I. = P êç 1 + ÷ - 1ú In the given sequence; 4 is repeated six times. Hence,
êëè 100 ø úû the student has to sit down six times.
42. (a) Going through information provided, we get codes for
éæ 5 ö
3 ù éæ 21 ö 3 ù G ® 3, R ® 8, E ® 1, C ® 9.
= 8840 êç1 + - 1ú = 8840 êç ÷ - 1ú
è 100 ÷ø êëè 20 ø Therefore, Greece will be coded as 381191.
ëê ûú ûú
43. (b) A person sits on a chair. Since ‘chair’ is called ‘cot’,
é 9261 ù 8840 ´ 1261 our answer is ‘cot’.
= 8840 ê -1 = = ` 1393.405 44. (a) One’s brother’s son’s wife’s daughter implies paternal
ë 8000 úû 8000
grand-daughter of one’s brother. Now, the mother of
32. (c) SP of 150 metres of clothes = ` 6600 paternal grand-daughter of one’s brother implies wife
é 6600 ù of one’s nephew.
\ SP of 1 m cloth = ` ê = ` 44
ë 150 úû
Thus, we can conclude that Arun is the paternal uncle
of the female’s husband.
SP of 300 metres of cloth = ` 12750 45. (d) The boys are standing as follows from L ® R
é12750 ù E
\ SP of 1 m cloth = ` ê = ` 42.5
ë 300 úû Deepak Sameer Shailendra Tushar Sushil N
\ Concession = ` (44 – 42.5) = ` 1.5
33. (e) Let thenumber = x. 46. (b) Conclusions : Some buckets are pots.
ATQ, (conversion)
3888 Some bags are buckets.
´ 21 = 252 (conversion)
x2 Some purses are bags. (conversion)
3888 No mediate inference follows.
Þ x2 = ´ 21 = 324 \ No given Conclusions follows.
252
47. (e) Statements : All glasses are roads.
\ x = 324 = 18
34. (b) Let two digit number = 10 x + y
ATQ,
x + y = 14 ..... (i) No road is stick.
www.newspaperkorner.wordpress.com
PS-46 Practice Set - 4
www.newspaperkorner.wordpress.com
Conclusions : No glass is stick. (A + E = E-type) 51. (c) The other neighbour of A is E.
52. (a) F is placed opposite to E.
Statements : Some sticks is pens. 53. (b) Clearly, C is the required person.
Conclusions : Pens are not glass. (E + I = O* type) 54. (a) B and F are not neighbours.
Statements : No road is stick. For (56-61): Simply follow the rules of the codes and do these
sums.
56. (c) X$DTFX (Because 1st and last digits are odd.)
Some sticks are pens. 57. (a) $%HFD# (No any condition.)
Conclusions : Some pens are not road. 58. (c) HK$T%H (Because 1st and the last digits are even.)
(E + I = O* type)
59. (b) D%$HK# (Because 1st digit is odd and the last digit is
Hence only IV follows.
48. (a) Statements : Some ice is ring. even.)
Conclusions : Some rings are ice. (conversion) 60. (e) HTMK#H (Because Ist and the last digits are even.)
Statements : Some rings are gold. 61. (a) XFH% D X (Because Ist and last digits are odd)
Conclusions : Some gold are ring. (conversion) 62. (b) M < N ......(i), T > U .....(ii); N < U ......(iii)
Combining these, we get M < N < U < T
Hence M < T and T > N.
63. (d) P > T ....(i); G £ N .....(ii), T ³ N .....(iii)
Statements : No ring is paint. From (i) and (iii), P > N
Conclusions : Some gold are not paint.
(I + E = O-type) From (ii) and (iii), G £ T..
Statements : Some ice is ring. 64. (c) P £ Q ....(i); R > S ..... (ii); Q ³ S .....(iii)
Thus the relationships can’t be established.
65. (a) D ³ F....(i); G >H ...(ii); F £ H ...(iii)
No ring is paint. From (ii) and (iii), G > F. Hence I follows.
Conclusions : Some ice is not paint. But II can’t be established.
(I + E = O-type )
66. (a) P1 %TR A 5 # D M 7 K E G 2 8 $ H3 1 4V U6 F
Statements : No ring is paint.
Conclusions : No paint is ring. (Implication) 9Z
Hence none follows. In the above series there is no consonant symbol-
49. (e) Statements : Some shoes are bells. consonant sequence.
Conclusions : Some bells are shoes. (conversion) 67. (e) Except it in each choice second and third elements are
Statements : No candle is bell. second to the left of first elements and third to the
right of first element respectively.
Some bells are shoes. 68. (a) We have to look for digit- vowel-consonant sequence
Conclusions : Some shoes are not candle. in the following series.
(E + I = O*) P1 % T R A 5 # D M 7 K E G 2 8 $ H 3 1 4 V U 6 F 9
Statements : All tables are shoes. Z
Conclusions : Some tables are shoes. (Implication) There is no such sequence.
Some shoes are tables. (conversion) 70. (d) After changing the series becomes as follows:
Statements : No candle is bell. P1 %T R A 5 # D M 7 K E G 2 8 Z 9 F6UV41
Conclusions : No bell is candle. (conversion) 3H$
Hence none follow. Now, eigth element to the right of eleventh from the
50. (c) Statements : Some cats are rats.
left, i.e., 9.
Conclusions : Some rats are cats. (conversion)
Statements : Some rats are ants. 71. (b) Refer to the Ist sentence of para 1 ‘Govind’s father
Conclusions : Some ants are rats (conversion) ........................ left large tract of land to Govind’.
Statements : Some ants are flies. 72. (c) Refer to the 3rd sentence of para 2 “To his
Conclusions : Some flies are ants. (conversion) surprise.................into gold”.
No mediate inference follows. 73. (d) Refer to the last two sentences of para 1 of the passage.
Hence, only I and II follow. 75. (e) Refer to the sixth sentence of para 2 of the passage
For Qs. (51-55) ‘You have..................during winter’.
The following hexagonal arrangement is possible. 76. (b) ‘Sent’ is the third form of verb ‘send’ in passive voice.
B 78. (a) Replace ‘though’ by ‘As’ to express cause and effect
in the sentence.
E C 79. (d) It should be ‘and I answered all of them’ to properly
express cause and effect in the sentence.
A F 80. (c) Apply the adverb ‘highly’ before the adjective ‘paid’.

D www.newspaperkorner.wordpress.com
www.newspaperkorner.wordpress.com

Preliminary Exam
Practice Set - 5
INSTRUCTIONS
• This Preliminary Exam practice set consists of three sections. Quantitative Aptitude (Qs. 1-35); Reasoning
Ability (Qs. 36-70) and English Language (Qs. 71-100).
• All the questions are compulsory.
• Each question has five options, of which only one is correct. The candidates are advised to read all the
options thoroughly.
• There is negative marking equivalent to 1/4th of the mark allotted to the specific question for wrong answer.

Time : 1 hrs. Max. Marks : 100

QUANTITATIVE APTITUDE
4 2 1
5. 13 + 5 ´ 2 = ?
7 7 2
DIRECTIONS (Qs. 1-10) : What will come in place of question
mark (?) in the following questions ? 11 3
(a) 25 (b) 25
14 7
1. 48% of 525 + ? % of 350 = 399
(a) 42 (b) 46 3 5
(c) 28 (d) 26 (c) 26 (d) 26
7 14
(e) None of these (e) None of these
5 4 1 6. 784 ÷ 16 ÷ 7 = ?
2. 2 × 3 + ? = 12
9 5 5 (a) 49 (b) 14
(c) 21 (d) 7
13 4
(a) 2 (b) 2 (e) None of these
45 5
3 5
22 5 7. of 455 + of 456 = ?
(c) 3 (d) 3 2 8
45 9
(a) 448 (b) 476
(e) None of these (c) 480 (d) 464
3. ? + 172 = 335 (e) None of these
(a) 46 (b) 42 8. 6425 ÷ 125 × 8 = ?
(c) 1764 (d) 2116 (a) 411.2 (b) 41.12
(e) None of these (c) 64.25 (d) 421.25
28 ´ 5 - 15 ´ 6 (e) None of these
4. =? 9. 1.05% of 2500 + 2.5% of 440 = ?
7 2 + 256 + (13)2
(a) 37.50 (b) 37.25
27 22 (c) 370.25 (d) 372.50
(a) (b) (e) None of these
115 117
10. 4900 ÷ 28 × 444 ÷ 12 = ?
25 22 (a) 6575 (b) 6475
(c) (d)
117 115 (c) 6455 (d) 6745
(e) None of these (e) None of these
www.newspaperkorner.wordpress.com
PS-48 Practice Set - 5
www.newspaperkorner.wordpress.com
DIRECTIONS (11-15): Study the following graph carefully and (a) 9 : 4 (b) 8 : 3
answer the questions that follow: (c) 7 : 2 (d) 8 : 5
Percentage of employees in different departments of a (e) None of these
company Total No. of employees = 4500 15. The total number of females are what per cent of the total
number of males in the organisation ?
(a) 90 (b) 70
(c) 80 (d) 60
(e) None of these

Accounts
12%
Design DIRECTIONS (Qs. 16-20) : What will come in place of the
32% 8% question mark (?) in the following number series?
HR
16. 7 9 12 16 ?
e
Marketing (a) 2 2 (b) 19
ativ
tr 18%
minis (c) 20 (d) 21
A d 8% (e) None of these
Customer
Relation 17. 384 192 96 48 ?
22% (a) 36 (b) 28
(c) 24 (d) 32
(e) None of these
18. 5 6 14 45 ?
Percentage of females in each department in the same
company Total No. of females in the organisation = 2000 (a) 183 (b) 185
(c) 138 (d) 139
(e) None of these
19. 8 9 13 22 ?
Accounts

(a) 30 (b) 31
12%

(c) 34 (d) 36
Design (e) None of these
28% 20. 6 11 21 41 ?
HR 16%
(a) 81 (b) 61
e
istrativ (c) 71 (d) 91
Admin % Marketing
10 (e) None of these
Customer 14%
21. Number of students studying in colleges A and B are in the
Relation ratio of 3 : 4 respectively. If 50 more students join college A
20%
and there is no change in the number of students in college
B, the respective ratio becomes 5 : 6. What is the number of
students in college B ?
11. What is the total number of males from Design, Customer
(a) 450 (b) 500
Relation and HR departments together ?
(c) 400 (d) 600
(a) 1550 (b) 1510
(e) None of these
(c) 1540 (d) 1580 22. Cost of 12 belts and 30 wallets is ` 8940. What is the cost of
(e) None of these 4 belts and 10 wallets?
12. What is the ratio of number of males in HR department to the (a) ` 2890 (b) ` 2980
number of males in Accounts department respectively ? (c) ` 2780 (d) ` 2870
(a) 3 :17 (b) 4 : 15 (e) None of these
(c) 2 : 15 (d) 2 : 13 23. 80% of a number is equal to three-fifth of another number.
(e) None of these What is the ratio between the first and the second number
13. The number of females in the Marketing department are respectively?
approximately what per cent of the total employees in (a) 3 : 4 (b) 4 : 3
Marketing and Customer Relation Departments together? (c) 4 : 5 (d) 5 : 4
(a) 26 (b) 36 (e) None of these
(c) 6 (d) 46 24. Ghanshyam purchased an article for `1850. At what price
(e) 16 should he sell it so that 30% profit is earned?
14. What is the respective ratio of number of employees in (a) ` 2450 (b) ` 2245
Administrative department to the number of males in the (c) ` 2405 (d) ` 2425
same department ? (e) None of the above
www.newspaperkorner.wordpress.com
Practice Set - 5 PS-49
www.newspaperkorner.wordpress.com
25. What is the compound interest accrued on an amount of 35. The average age of 60 boys in a class was calculated as 12
` 8500 in two years @ interest 10% per annum? years. It was later realised that the actual age of one of the
(a) ` 1875 (b) ` 1885 boys in the class was 12.5 years but it was calculated as 14
(c) ` 1775 (d) ` 1765 years. What is the actual average age of the boys in the
(e) None of these class?
26. A train running at the speed of 60 kmph crosses a 200 m (a) 11 years (b) 11.275 years
long platform in 27 s. What is the length of the train ? (c) 11.50 years (d) 11.975 years
(a) 250 m (b) 200 m (e) None of these
(c) 240 m (d) 450 m REASONING ABILITY
(e) None of these
36. Bihar is related to India in the same as Florida is related to
27. Which of the following has the fractions in ascending order?
(a) Canada (b) Mexico
5 3 4 2 5 4 3 2 (c) North America (d) USA
(a) , , , (b) , , ,
11 8 9 7 11 9 8 7 (e) None of these
2 3 4 5 2 4 3 5 37. Unscramble the letters in the given words and find the odd
(c) , , , (d) , , ,
7 8 9 11 7 9 8 11 one out.
(e) None of these (a) UMRSME (b) EIWNTR
28. Sum of the digits of a two digit number is 8 and the digit in (c) PIGRSN (d) LCUOD
the ten’s place is three times the digit in the unit’s place. (e) None of these
What is the number? 38. If the first and second letters in the word DEPRESSION
(a) 26 (b) 36 were interchanged, also the third and the fourth letters, the
(c) 71 (d) 62 fifth and the sixth letters and so on, which of the following
(e) None of these would be the seventh letter from the right ?
29. 10 men can complete a piece of work in 8 days. In how many
(a) R (b) O
days can 16 men complete that work?
(a) 4 days (b) 5 days (c) S (d) P
(c) 6 days (d) 3 days (e) None of these
(e) None of these 39. If the positions of the third and tenth letters of the word
30. 71% of a number is more than its 46% by 120. What is 30% DOCUMENTATION are interchanged, and likewise the
of that number? positions of the fourth and seventh letters, the second and
(a) 160 (b) 150 sixth letters is interchanged, which of the following will be
(c) 140 (d) 148 eleventh from the right end ?
(e) None of these
(a) C (b) I
31. Average of five consecutive odd numbers is 95. What is
the fourth number in descending order? (c) T (d) U
(a) 91 (b) 95 (e) None of these
(c) 99 (d) 97 40. If the numbers from 1 to 45 which are exactly divisible by 3
(e) None of these are arranged in ascending order, minimum number being on
32. Latika spends 45% of her monthly income on food and 30% the top, which would come at the ninth place from the top?
of the monthly income on transport. Remaining amount of (a) 18 (b) 24
`4500 she saves. What is her monthly income? (c) 21 (d) 27
(a) `16000 (b) `18000
(c) `16500 (d) `18500 (e) None of these
(e) None of these
DIRECTION (Qs. 41 - 45) : In each of the questions below are
33. Amount of simple interest accrued on an amount of Rs
given two or three statements followed by the conclusions
28500 in seven years is Rs 23940 what is the rate of interest
% per annum? numbered I and II. You have to take the given statements to be
(a) 10.5 (b) 12.5 true even if they seem to be at variance with commonly known
(c) 11 (d) 12 facts and then decide which of the given conclusions logically
(e) None of these follows from the given statements. Give answer
34. A and B started a business investing amounts of `150000 (a) if only conclusion I follows.
and `250000 respectively. What will be B’s share in the
(b) if only conclusion II follows.
profit of `160000 ?
(a) `100000 (b) `60000 (c) if neither I nor II follows.
(c) `80000 (d) `110000 (d) if both I and II follow.
(e) None of these (e) None of these
www.newspaperkorner.wordpress.com
PS-50 Practice Set - 5
www.newspaperkorner.wordpress.com
41. Statements : All toys are dolls . (a) A (b) B
All dolls are jokers. (c) E (d) D
Some toys are cars. (e) None of these
Conclusions : I. Some cars are jockers. 50. Which of the following groups of teachers has History as
II. Some dolls are cars. the compulsory subject?
42. Statements : All pens are boxes. (a) A, C and D (b) B, C and D
Some boxes are blades. (c) C and D (d) A, B and C
Some blades are files. (e) None of these
Conclusions : I. Some blades are pens.
II. Some pens are files. DIRECTIONS (Qs. 51-55) : Study the following information
carefully and answer the questions given below.
43. Statements : All books are ledgers.
All pens are keys. In a certain code, the symbol for 0 (zero) is D and for 1 is $.
Some pens are books. There is no other symbol for all other numbers greater than 1. The
Conclusions : I Some ledgers are keys. numbers greater than 1 are to be written using only the two
II. Some keys are books. symbols given above. The value of symbol for 1 doubles itself
44. Statements : Some roses are thorns. everytime it shifts one place to the left. Study the following
All thorns are flowers. example.
No flower is a petal. ‘0’ is written as D ‘1’ is written as $
Conclusions : I. No petal is a rose. ‘2’ is written as $D ‘3’ is written as $$
II. Some flowers are roses. ‘4’ is written as $DD
45. Statements: All leaders are good team workers. and so on.
All good team workers are good orators. 51. Which of the following will represent 7 × ( 1 + 4 ¸ 2) ?
Conclusions: I. Some good team workers are leaders. (a) $D$D$ (b) $$D$D
II. All good orators are leaders. (c) $DD$$ (d) $DD$$
(e) None of these
DIRECTIONS (Qs. 46- 50) : Read the following information 52. Which of the following symbol arrangement will represent
carefully to answer the questions that follow. ‘9’ ?
There are six teachers A, B, C, D, E and F in a school. Each (a) D$$$ (b) $D$D
of the teachers teaches two subjects, one compulsory subject (c) $DD$ (d) $$DD
and the other optional subject. D’s optional subject is History (e) None of these
while three others have it as compulsory subject. E and F have 53. The symbol arrangement $DD$$ represents which of the
Physics as one of their subjects. F’s compulsory subject is following numbers ?
Mathematics which is an optional subject of both C and E. History
(a) 18 (b) 25
and English are A’s subjects but in terms of compulsory and
optional subjects, they are reverse of those of D’s. Chemistry is (c) 17 (d) 16
an optional subject of any one of them. There is only one female (e) None of these
teacher in the school who has English as her compulsory subject. 54. The symbol arrangement $$$DD$D represents which of the
46. What is C’s compulsory subject ? following numbers ?
(a) History (b) Physics (a) 28 (b) 48
(c) Chemistry (d) English (c) 26 (d) 50
(e) None of these (e) None of these
47. Who is a female member in the group ? 55. Which of the following represents the number 24 ?
(a) A (b) B (a) $DD$D (b) $$DDD
(c) C (d) D (c) $$$DD (d) $DD$$
(e) None of these (e) None of these
48. Who among the following has same optional subjects as
DIRECTIONS (Qs. 56- 60) : In these questions symbols #, @, $,
that of the compulsory subject of F?
*, % are to be used with different meanings as follows:
(a) D (b) B
(c) A (d) C ‘A # B’ means ‘A is neither smaller than nor equal to B’.
(e) None of these ‘A @ B’ means ‘A is smaller than B’.
49. Disregarding which is compulsory and which is the optional ‘A $ B’ means ‘A is not greater than B’
subject, who has the same two subjects combination as F ? ‘A * B’ means ‘ A is not smaller than B’.
www.newspaperkorner.wordpress.com
Practice Set - 5 PS-51
www.newspaperkorner.wordpress.com
‘A % B’ means ‘A is neither smaller than nor greater than B’. (a) 15 (b) L
In each question, three statements showing relationships have (c) K (d) I
been given, which are followed by two conclusions I & II. (e) None of these
Assuming that the given statements are true, find out which 65. If it is possible to make a meaningful word from the eighth,
conclusion(s) is/are definitely true. Mark answer sixteenth, seventeenth and twenty-second leters from your
(a) if only conclusion I is true; left in the given series, which will be th first letter of that
(b) if only conclusion II is true; word? If no such word can be formed, your answer would be
(c) if either conclusion I or II is true; X, and if more than one such word can be formed, answer is
P.
(d) if neither I nor II is true and
(a) M (b) T
(e) if both conculsions I and II are true.
(c) X (d) E
56. Statements: T @ J, J * M, M $ B (e) P
Conclusions: I. T # M
DIRECTIONS (Qs. 66-70) : In each of the questions given below
II. J $ B which one of the five answer figures on the right should come
57. Statements: R# F, F @ K, K $ V after the problem figures on the left, if the sequence were
Conclusions :I. R # V continued ?
II. V # F 66. Problem Figures
58. Statements: E @ A, A % F, F $ Q
Conclusions :I. E @ Q c
c c

c
II. Q *A
59. Statements: L # M, M % D, D * Q
Answer Figures
Conclusions: I. M # Q
II. Q @ L
60. Statements: W $ F, F @ H, H # R

c
c

c
Conclusions: I. W # R
(a) (b) (c) (d) (e)
II. W $ R
67. Problem Figures
DIRECTIONS (Qs. 61-65): Answer these questions referring
to the letter sequence given below: « S oS o =« r r « = S
N O PQ YB ZAR S H I J K I LM T UV G E FW X D C B «r S o « = r
61. If letters of the above given series are written in reverse r = S
order then which letter will be third to the left of eighteenth Answer Figures
letter from your right?
(a) Z (b) G = S S = B = S B = B = S B
D

(c) I (d) L « « « « S «
(e) None of these r r r r r
62. What will come in place of question mark (?) in the following (a) (b) (c) (d) (e)
series ?
NDP, QWB, ZER, ? 68. Problem Figures
(a) SVJ (b) AFS
(c) IVS (d) SFA
(e) None of these MOUNT OSNPT TOSNP ODNCP PODNC
63. Which of the following is the fifth to the right of thirteenth
letter from you left ?
Answer Figures
(a) T (b) J
(c) S (d) Z
(e) None of these
CPODN OPNDC OLZNC OLNZC OLNDC
64. If every altenate letter starting from O is replaced with odd
numbers starting from 1, which letter or number will be third
to the left of tenth letter from your right ? (a) (b) (c) (d) (e)
www.newspaperkorner.wordpress.com
PS-52 Practice Set - 5
www.newspaperkorner.wordpress.com
69. Problem Figures the Valuer which will make him do and say what you want him to
do and say," said the man. "Go to him and give him a fine present,
= = S == S V = then say to him: "You said the horses are worth a measure of rice
« V S Z Z V rr S but now tell what a measure of rice is worth! Can you value that
S Z « V V rZ «
r o o r« oo « Z o standing in your place by the king?" If the says he can, go wth
him to the king, and I will be there, too."
Answer Figures The horse-dealer thought this was a good idea. So he gave
a fine present to the Valuer, and said what the other man had told
= V r V = V V = = V him to say., The stupid Valuer took the present, and said: "Yes, I
« S = r « « « can go before the king with you and tell what a measure of rice is
r S « o S r S r S
o Z ro Z o Z Z o worth. I can value that now." Well, let us go at once," said the
(a) (b) (c) (d) (e) horse-dealer. So they went before the king and his ministers in
the palace.
70. Problem Figures The horse-dealer bowed down before the king, and said:
"O King, I have learned that a measure of rice is the value of my
« o «D oD « o five hundred horses. But will the king be pleased to ask the Valuer
Z r D D Z r c A« « c A r S A what is the valuer of the measure of rice". The king, not knowing
c c o r r o D what had happened, asked, "How now, Valuer, what are five
hundred horses worth?" "A measure of rice, O King!" said he.
Answer Figures "Very good, then! If five hundred horses are worth a measure of
rice, what is the measure of rice worth?" "The measure of rice is
o D D « « «
worth your whole city," replied the foolish fellow.
A r S « B o A r S S A rA r S The ministers clapped their hands, laughing, and saying,
« r A D oD oo D "What a foolish Valuer! How can such a man hold that office?
(a) (b) (c) (d) (e) We used to think this great city was beyond price, but this man
says it is worth only a measure of rice, "Then the king was
ashamed, and drove out the foolish fellow. "I tried to please the
ENGLISH LANGUAGE
king by setting a low price on the horses, and now see what has
DIRECTIONS (Qs. 71-80) : Read the following passage carefully happened to me!" said the Valuer, as he ran away from the
and answer the questions given below it. laughing crowd.
71. Who did the king appoint as the new Valuer?
Once upon a time a dishonest king had a man called the
(a) A minsister (b) A horse merchant
Valuer in his court. The Valuer set the price which ought to be
(c) Himself (d) A stingy peasant
paid for horses and elephants and the other animals. He also set
(e) None of these
the price on jewellery and gold, and things of that kind. This man
72. Why was the king not happy with the old Valuer?
was honest and just, and set the proper price to be paid to the
(a) As the Valuer was not good at his work
owners of the goods. The king, however, was not pleased with
(b) As he had dishonoured the king
this Valuer, because he was honest. "If I had another sort of a
man as Valuer, I might gain more riches," he thought. (c) As the Valuer had been dishonest with the king about
the prices that he set for goods
One day the king saw a stupid, miserly peasant come into
(d) As the king believed that he was not earning much
the palace yard. The king sent for the fellow and asked him if he
because of the Valuer's honesty
would like to be the Valuer. The peasant said he would like the
(e) None of these
position. So the king had him made Valuer. He sent the honest
73. Which of the following words can be used to describe the
Valuer away from the palace.
king?
Then the peasant began to set the prices on horses and (1) Smart (2) Dishonest
elephants, upon gold and jewels. He did not know their value, so (3) Cheat
he would say anything he chose. As the king had made him (a) Only (1) (b) Only (2)
Valuer, the people had to sell their goods for the price he set. By (c) Only (2) and (3) (d) Only (1) and (3)
and by a horse-dealer brought five hundred horses to the court (e) All the three (1), (2) and (3)
of this king. The Valuer came and said they were worth a mere 74. What can possibly be the moral of the story?
measure of rice. So the king ordered the horse-dealer to be given (a) Slow and steady wins the race
the measure of rice, and the horses to be put in the palace stables. (b) Change is the only permanent thing in life
The horse-dealer then went to see the honest man who had (c) An honest answer is the sign of true friendship
been the Valuer, and told him what had happened. "What shall I (d) Haste makes waste
do?" asked the horse-dealer. "I think you can give a present to (e) No legacy is so rich as honesty
www.newspaperkorner.wordpress.com
Practice Set - 5 PS-53
www.newspaperkorner.wordpress.com
75. Why did the Ministers laugh at the new Valuer? 81. As it was already afternoon, Rohan decided to check out of
(a) As he had sold the king's city at a very low price the hotel and go home.
(b) As he had displayed his stupidity by quoting an (a) for checking out (b) to checking out
abysmally low price on the king's city (c) to check outing (d) to checked out
(c) As he had cheated the horse dealer (e) No correction required
(d) As he had not claculated the price of the five hundred 82. Five people which ignored an evacuation order were trapped
horses correctly in a mountain region encircled by a wildfire.
(e) None of these (a) who ignored an (b) those ignoring an
76. What did the new Valuer do when he got the present from (c) who ignores a (d) that ignored a
the horse dealer? (e) No correction required
(a) He accepted the present and resigned from his post as 83. Since she was the most popular model on the ramp, she
was requested by the horse-dealer
thought no end to herself.
(b) He accepted the present and agreed to state the worth
(a) no ending to herself (b) no ends of herself
of a measure of rice in the presence of the King
(c) no end of herself (d) no end with herself
(c) He accepted the present and immediately returned the
(e) No correction required
horse-dealer's horses
(d) He refused to accept the present from the horse-dealer 84. The actress says that she is not comfortable walk into the
and asked him to leave the premises ramp in designer-wear.
(e) None of these (a) to walks on (b) walking down
77. Which of the following can be said about the old Valuer? (c) walking with (d) walks to
(1) He was honest. (2) He was intelligent. (e) No correction required
(3) He was revengeful. 85. Not many bank on the power of realism and comedy to
(a) Only (1) (b) Only (3) make an impression and so, when one does, it makes head
(c) Only (1) and (2) (d) Only (1) and (3) truns.
(e) All the three (1), (2) and (3) (a) making heads turn
78. What was the worth of a measure of rice according to the (b) make head turnings
new Valuer? (c) making heads turining
(a) The king's entire city (d) makes heads turn
(b) The king's life (e) No correction required
(c) Two horses DIRECTIONS (Qs. 86-90) : In each question below, a sentence
(d) Not mentioned in the passage with four words printed in bold type is given. These are numbered
(e) None of these as (a), (b), (c) and (d). One of these four words printed in bold may
79. Why did the horse-dealer go to meet the old Valuer? be either wrongly spelt or inappropriate in the context of the
(a) As the new Valuer had set a very inappropriate price sentence. Find out the word which is wrongly spelt or
for his five hundred horses
inappropriate, if any. The number of that word is your answer. If
(b) As his five hundred horses were stolen from him by
all the words printed in bold are correctly spelt and also
the king
appropriate in the context of the sentence, mark (e) i.e., All
(c) As he was a very good friend of the old Valuer
correct' as your answer.
(d) As the king head requested him to do so
(e) None of these 86. The city's fashion-conscious ladies (a)/ came together at a
80. What advice did the old Valuer give to the horse-dealer? city hotel to check out an exibition (b)/ by various (c)/
(a) He asked the horse-dealer to inquire with the king about designers (d)/ and labels. All correct (e)
the worth of a measure of rice 87. The ministry's proposal (a)/ for an autonomous (b)/
(b) He asked the horse-dealer to bribe the new Valuer and overarching authority (c)/ for higher education and research
get his horses back was finally approval. (d)/ All correct (e)
(c) He asked the horse-dealer to forget about his horses
88. Silense (a)/ is to retreat (b)/ in wordless prayer, gazing (c)/
and go on with his life
out the window of your heart, and going for slow
(d) He asked the horse-dealer to publicize his plight and
thus get his horses back meandering (d)/ walks in a garden. All correct (e)
(e) None of these 89. A majority of Army tanks continue to grope (a)/ in the dark,
DIRECTIONS (Qs. 81-85) : Which of the phrases (a), (b), (c) and stricken (b)/ as they are with an ecute (c) case of night
(d) given below each sentence should replace the phrase printed blindness. (d)/ All correct (e)
in bold in the sentence to make it grammatically correct? If the 90. Back home, the ever affable (a)/ Bollywood singer shares
sentence is correct as it is given and no correction is required, the excitement (b) of having (c)/ performed (d)/ at the Royal
mark (e) as the answer. Hall in London. All correct (e)
www.newspaperkorner.wordpress.com
PS-54 Practice Set - 5
www.newspaperkorner.wordpress.com
DIRECTIONS (Qs. 91-100) : In the following passage there are 92. (a) try (b) told
blanks, each of which has been numbered. These numbers are (c) were (d) bent
printed below the passage and against each, five words are (e) learnt
suggested, one of which fits the blank appropriately. Find out 93. (a) main (b) exactly
the appropriate word in each case. (c) many (d) because
There was once a gardener who (91) care of the king's (e) too
garden. In the garden, lived a family of monkeys. Seeing the 94. (a) call (b) make
gardener at work, the monkeys soon (92) to imitate him. As the (c) stall (d) go
gardener tended the plants and weeded and watered the garden, (e) visit
he was amused to see that the monkeys also did (93) what he did. 95. (a) forcefully (b) hardly
The gardener was happy that he had so much unpaid help. (c) usually (d) costly
One day the gardener wanted to (94) a fair in the city. He (e) truly
had an idea. He called the chief of the monkeys and said to him, "I 96. (a) ask (b) bring
have to go out for the day. Can you and your family water my (c) got (d) throw
plants like you (95) do? I promise you that if you help me I will (e) create
(96) sweets for you from the fair.: The monkeys agreed. But after 97. (a) party (b) time
the gardener had left, they had a (97). How much water were they (c) answer (d) doubt
to pour for each plant? then one of them said. "For plants with (e) water
big roots, we must pour (98) of water and for the ones with small 98. (a) body (b) many
roots, we pour only a litle water." So, the monkeys (99) out each (c) lots (d) weight
plant and then pushed it back again after looking at the root. As (e) quantity
a result, many plants (100) and died. On his return, the gardener 99. (a) thrashed (b) saw
realized that he had been very foolish to trust a bunch of mere (c) stick (d) pulled
monkeys to do his job. (e) splashed
91. (a) took (b) was 100. (a) withered (b) crushed
(c) great (d) handle (c) killed (d) grew
(e) mended (e) smiled

www.newspaperkorner.wordpress.com
Practice Set - 5 PS-55
www.newspaperkorner.wordpress.com
Answer Key
1 (a) 11 (b) 21 (d) 31 (e) 41 (d) 51 (d) 61 (b) 71 (d) 81 (e) 91 (a)
2 (e) 12 (c) 22 (b) 32 (b) 42 (c) 52 (c) 62 (e) 72 (d) 82 (a) 92 (e)
3 (d) 13 (e) 23 (a) 33 (d) 43 (d) 53 (e) 63 (a) 73 (c) 83 (c) 93 (b)
4 (c) 14 (a) 24 (c) 34 (a) 44 (b) 54 (d) 64 (d) 74 (e) 84 (b) 94 (e)
5 (e) 15 (c) 25 (e) 35 (d) 45 (a) 55 (b) 65 (e) 75 (b) 85 (d) 95 (c)
6 (d) 16 (d) 26 (a) 36 (d) 46 (a) 56 (d) 66 (d) 76 (b) 86 (b) 96 (b)
7 (c) 17 (c) 27 (c) 37 (d) 47 (d) 57 (b) 67 (e) 77 (c) 87 (d) 97 (d)
8 (a) 18 (e) 28 (d) 38 (d) 48 (d) 58 (a) 68 (d) 78 (a) 88 (a) 98 (c)
9 (b) 19 (e) 29 (b) 39 (c) 49 (c) 59 (b) 69 (a) 79 (a) 89 (c) 99 (d)
10 (b) 20 (a) 30 (e) 40 (d) 50 (d) 60 (c) 70 (c) 80 (e) 90 (e) 100 (a)

Answers & Explanations


1. (a) 48 % of 525 + ? % of 350 = 399 375 11
Þ ?= = 26
48 ? 14 14
Þ ´ 525 + ´ 350 = 399
100 100 6. (d) ? = 784 ¸ 16 ¸ 7
Þ 25200 + ? × 350 = 399 × 100 784
Þ ?= ¸ 7 Þ ? = 49 ¸ 7 = 7
Þ ? × 350 = 39900 – 25200 = 14700 16
14700 3 5
Þ ?= = 42 7. (c) ?= of 455 + of 456
350 7 8
5 4 1
2. (e) 2 ´ 3 + ? = 12 3 5
9 5 5 Þ ? = × 455 + × 456
7 8
23 19 61
Þ ´ +? = Þ ? = 195 + 285
9 5 5 Þ ? = 480
?= -
61 437 8. (a) ? = 6425 ¸ 125 × 8
Þ
5 45 Þ ? = 51.4 × 8
549 - 437 Þ ? = 411.2
Þ ?= 9. (b) ? = 1.05% of 2500 + 2.5% of 440
45
112 22 1.05 2.5
Þ ?= =2 Þ ?= ´ 2500 + ´ 440
45 45 100 100
2
3. (d) ? + 17 = 335 2625 1100
Þ Þ ?= +
? + 289 = 335 100 100
Þ ? = 335 - 289 = 46 3725
Þ ? = 46 × 46 = 2116 Þ ?= = 37.25
100
28 ´ 5 - 15 ´ 6 10. (b) ? = 4900 ¸ 28 × 444 ¸ 12
4. (c) ?=
7 2 + 256 + (13) 2 Þ ? = 175 × 37
140 - 90 Þ ? = 6475
Þ ?= 11. (b) Number of employees in design, customer relation and
49 + 16 + 169
HR departments together
50 25 4500 × (32 + 22 + 8)%
Þ ?= =
234 117
4500 ´ 62
4 2 1 = = 2790
5. (e) ? = 13 + 5 ´ 2 100
7 7 2
95 37 5 Number of women employees in these departments
Þ ?= + ´ = 2000 × (28 + 20 + 16)%
7 7 2
95 185 2000 ´ 64
Þ ?= + = = 1280
7 14 100
190 + 185 \ Required number of males
Þ ?=
14 = 2790 – 1280 = 1510
www.newspaperkorner.wordpress.com
PS-56 Practice Set - 5
www.newspaperkorner.wordpress.com 19. (e) Pattern of the series would be as follows
12. (c) Number of employees in HR department
4500 ´ 8 8 9 13 22 38
= = 360
100 +(1)2 +(2)2 +(3)2 +(4)2
\ Number of males 20. (a) Pattern of the series would be as follows
2000 ´16 6 11 21 41 81
= 360 -
100
+5 +10 +20 +40
= 360 – 320 = 40 21. (d) Let total number of students in college A = 3x
Number of employeess in Accounts department and total number of students in college B = 4x
4500 ´12 After 50 more students join college A
= = 540
100 3x + 50 5
New Ratio = =
\ Number of males 4x 6
Þ 18 x + 300 = 20 x
2000 ´ 12
= 540 - Þ 2x = 300
100
300
= 540 – 240 = 300 Þ x= = 150
2
\ Required ratio = 40 : 300 = 2 : 15
Total number of students in college
13. (e) Number of employees in marketing and customer
B = 4x = 4 × 150 = 600
relation departments 22. (b) Q Cost price of (12 belts + 30 wallers) = ` 8940
4500 ´ 40 Q Cost price of 3 × (4 belts + 10 wallets) = ` 8940
= = 1800
100 8940
Q Cost price of 4 belts + 10 wallets = = ` 2980
Number of females in the marketing department 3
23. (a) Let the first number be x
2000 ´ 14
= = 280 and the second number be y
100 3
Then, 80% of x = of y
280 5
\ Required percentage = ´ 100 » 16
1800 80 3
Þ ´ x = ×y
14. (a) Total number of employees in administrative 100 5
department 4 3
Þ ´x = ×y Þ 4x = 3y
4500 ´ 8 5 5
= = 360
100 x 3
Þ = =3:4
Number of males in the same department y 4
= 360 – 200 = 160 24. (c) Q Cost price of an article = ` 1850
\ Required ratio For 30% profit, selling price of this article
= 360 : 160 = 9 : 4 130
15. (c) Required percentage = 1850 ´ = ` 2405
100
2000 25. (e) Compound Interest after two years
= ´ 100 = 80
2500 2
æ 10 ö
16. (d) Pattern of the series would be as follows = 8500 ç1 + - 8500
è 100 ÷ø
7 9 12 16 21
11 11
+2 +3 +4 +5 = 8500 ´ ´ - 8500
10 10
17. (c) Pattern of the series would be as follows = 10285 – 8500 = ` 1785
384 192 96 48 24 26. (a) Let length of the train be x m
÷2 ÷2 ÷2 ÷2 5 50
Speed of the train be 60 km/h = 60 ´ = m/s
18. (e) Pattern of the series would be as follows 18 3
5× 1+1=6
x + 200
6 × 2 + 2 = 14 Then, = 27
14 × 3 + 3 = 45 50
\ 45 × 4 + 4 = 184 3

www.newspaperkorner.wordpress.com
Practice Set - 5 PS-57
www.newspaperkorner.wordpress.com 41. (d) Statements : All toys are dolls.
3( x + 200)
Þ = 27
50
Þ 3x + 600 = 1350 All dolls are jokers
Þ 3x = 1350 – 600 Conclusions : All toys are jokers.
Þ 3x = 750 [A + A = A-type conclusion]
Statements : Some toys are cars.
750
Þ x= = 250 m
3 Conclusions : Some cars are toys. (Conversion)
27. (c) Each fractions, decimal value are given below :
5 3 4 2
= 0.454, = 0.375, = 0.444, = 0.286 Statements : All toys are jokers.
11 8 9 7
Conclusions : Some cars are jokers.
2 3 4 5 (I + A = I-type conclusion)
So, ascending order of the fractions is , , , .
7 8 9 11 \ I follows.
28. (d) Let ten’s digit be x and unit’s digit be 8 – x Some cars are toys
Then, x = 3 (8 – x)
Þ x = 24 – 3x, Statements : All toys are dolls.
Þ 4x = 24 Conclusions : Some cars are dolls.
24 Some dolls are cars. (Conversion)
Þ x= =6 \ II follows.
4
42. (c) Conclusions : Some pens are boxes.
\ unit’s digit = 8 – x = 8 – 6 = 2 (Implication of first statement)
So, required number = 62 Some boxes are pens.
29. (b) Suppose 16 men can complete the same work in x days (Conversion of first statement)
Then, Men days Some blades are boxes.
10 8 (Conversion of second statement)
16 x Some files are blades.
(Conversion of third statement)
16 : 10 : : 8 : x
No mediate inference follows. Hence, no given
Þ 16 × x = 10 × 8
Conclusions follows.
10 ´ 8 43. (d) Statements : Some pens are books.
Þ x= = 5 days
16 Conclusion : Some books are pens. (conversion)
36. (d) Bihar is a state in India. Similarly, Florida is a state in
USA.
37. (d) (a) Summer (b) Winter (c) Spring (d) Cloud Statements : All pens are keys.
All others are name of seasons. Some books are keys.
38. (d) The new letter sequence is EDRPSEISNO. (I + A = I-type conclusion)
The seventh letter from the right is P. Some keys are books. (Conversion)
\ II follows.
Statements : Some pens are books.
D E P R E S S I O N

1 2 3 4 5 6 7 8 9 10
All books are ledgers.
Conclusions : Some pens are ledgers.
(I + A = I-type)
39. (c)
Some ledgers are pens. (conversion)
D O C U M E N T A T I O N

1 2 3 4 5 6 7 8 9 10 11 12 13 Statements : All pens are keys.


Conclusions : Some ledgers are keys.
(I + A = I-type)
40. (d) The series will be as follows: \ I follows.
44. (b) Statements : Some roses are thorns.
3 6 9 12 15 18 21 24 27 30 33 36 39 42
45 All thorns are flowers.
www.newspaperkorner.wordpress.com
PS-58 Practice Set - 5
www.newspaperkorner.wordpress.com
Conclusions : Some roses are flowers. 56. (d) T < J, J ³ M, M £ B
(I + A = I-type)
No relation between T and M, and J and B.
Some flowers are roses. (conversion)
So neither I nor II is true.
Hence, II follows.
Statements : No flower is petal. 57. (b) R > F, F < K, K £ V
Conclusions : No petal is flower. (conversion) No relation between R and V. So conclusion I is not
true.
But V ³ K > F or V > F
Some flowers are roses.
Conclusions : Some roses are not petals. So, conclusion II is true.
(E + I = O*-type) 58. (a) E < A, A = F, F £ Q
Hence, I does not follow. Combining all, Q ³ F = A > E or E < Q and Q ³ A
45. (a) Statements : All leaders are good team
So, only conclusion I is true.
workers.
Conclusions : Some good team worker s are 59. (b) L > M, M = D, D ³ Q
leaders. (Conversion) Combining all, L > M = D ³ Q or M ³ Q and Q < L.
Hence, I follows. So, only conclusion II is true.
Statements : All good team workers are
good orators.
60. (c) W £ F, F < H, H > R
Conclusions : Some good orators are good Although no direct relation between W and R but I
team workers. (conversion) and II together show all three probable relations. Hence
Hence, II does not follow. either I or II is true.
For (Qs. 46 to 50) 61. (b) 18 + 3 = 21st letter from the right in the reverse series
The given information is summarised in a table as follows : or, 21st letter from the left in the original series.
62. (e) N + 3 = Q, Q + 3 = Z, Z + 3 = S
Subjects D – 2 = W, W – 2 = E, E – 2 = V
Teachers Compulsory Optional P + 3 = B, B + 3 = R, R + 3 = I Hence, ? = SVI
A History English 63. (a) 13 + 5 = 18th from you left
B History Chemistry 64. (d) 10 + 3 = 13th from the right
C History Mathematics 65. (e) A, L, M, E
D (Female) English History MALE, LAME, MEAL
E Physics Mathematics 66. (d) Symbols in the square are rotating 90° angle clockwise.
F Mathematics Physics 67. (e)
46. (a) History is the compulsory subject of C.
47. (d) D is a female member in the group.
48. (d) The compulsory subject of F (mathematics) is the
optional subject of C.
49. (c) E has physics and mathematics as his two subjects.
50. (d) A, B and C all have history as the compulsory 69. (a)
subjects.
For (Qs. 51-55):
These questions are based on the binary system which is made
of digits 0 and 1, here represented by D and $ repectively.
51. (d) 7 × (1 + 4 ¸ 2) = 7 × (1 + 2) = 7 × 3 = 21
Now, 21 = 16 + 0 + 4 + 0 + 1
= 1 × 24 + 0 × 23 + 1 × 22 + 0 × 21 + 1 × 20
= (10101)2
52. (c) 9 = 8 + 0 + 0 + 1
= 1× 23 + 0 × 22 + 0 × 21 + 1 × 20 = (10001)2 70. (c)
53. (e) $ DD$$ = (10011)2
= 1 × 24 + 0 × 2 3 + 0 × 22 + 1 × 2 1 + 1 × 20
= 16 + 0 + 0 + 2 + 1 = 19
54. (d) $$DD$$D = (1100010)2
= 1 × 25 + 1 × 2 4 + 0 × 2 3 + 0 × 22 + 1 × 21 + 0 × 20
= 32 + 16 + 0 + 0 + 2 + 0 = 50
55. (b) 24 = 16 + 8 + 0 + 0 + 0
= 1 × 24 + 1 × 23 + 0 × 22 + 0 × 21 + 0 × 20 = (11000)2
www.newspaperkorner.wordpress.com

You might also like